Sunteți pe pagina 1din 574

8.

5 x 11 inch

Differential Calculus Booster JEE main & Adv


11 March 2017 11:05:32 AM
Differential Calculus
Booster
with Problems & Solutions

JEE
Main and Advanced
About the Author
REJAUL MAKSHUD (RM)
Post Graduated from Calcutta University in PURE MATHEMATICS.
Presently, he trains IIT Aspirants at RACE IIT Academy, Jamshedpur.
Differential Calculus
Booster
with Problems & Solutions

JEE
Main and Advanced

Rejaul Makshud
M. Sc. (Calcutta University, Kolkata)

McGraw Hill Education (India) Private Limited


chennai

McGraw Hill Education Offices


Chennai New York  St Louis  San Francisco Auckland  Bogotá Caracas
Kuala Lumpur  Lisbon  London  Madrid  Mexico City  Milan  Montreal
San Juan  Santiago  Singapore  Sydney  Tokyo  Toronto
McGraw Hill Education (India) Private Limited
Published by McGraw Hill Education (India) Private Limited,
444/1, Sri Ekambara Naicker Industrial Estate, Alapakkam, Porur, Chennai - 600 116, Tamil Nadu, India

Differential Calculus Booster

Copyright © 2017, McGraw Hill Education (India) Private Limited.

No part of this publication may be reproduced or distributed in any form or by any means, electronic, mechanical, photocopying, recording, or otherwise
or stored in a database or retrieval system without the prior written permission of the publishers. The program listings (if any) may be entered, stored
and executed in a computer system, but they may not be reproduced for publication.

This edition can be exported from India only by the publishers,


McGraw Hill Education (India) Private Limited

ISBN (13): 978-93-5260-575-0


ISBN (10): 93-5260-575-6

Information contained in this work has been obtained by McGraw Hill Education (India), from sources believed to be reliable. However, neither
McGraw Hill Education (India) nor its authors guarantee the accuracy or completeness of any information published herein, and neither McGraw
Hill Education (India) nor its authors shall be responsible for any errors, omissions, or damages arising out of use of this information. This work
is published with the understanding that McGraw Hill Education (India) and its authors are supplying information but are not attempting to render
engineering or other professional services. If such services are required, the assistance of an appropriate professional should be sought.

Typeset at Bharati Composers, D-6/159, Sector-VI, Rohini, Delhi 110 085, and text and cover printed at

Cover Designer: Creative Designer

Visit us at: www.mheducation.co.in


Dedicated to
My Beloved Mom and Dad
Preface

DIFFERENTIAL CALCULUS BOOSTER with Problems & Solutions for JEE Main and Advanced is meant for aspirants
preparing for the entrance examinations of different technical institutions, especially NIT/IIT/BITSAT/IISc. In writing this
book, I have drawn heavily from my long teaching experience at National Level Institutes. After many years of teaching I
have realised the need of designing a book that will help the readers to build their base, improve their level of mathemati-
cal concepts and enjoy the subject.
This book is designed keeping in view the new pattern of questions asked in JEE Main and Advanced Exams. It has
nine chapters. Each chapter has the concept booster followed by a large number of exercises with the exact solutions to
the problems as given below:

Level - I : Problems based on Fundamentals


Level - II : Mixed Problems (Objective Type Questions)
Level - III : Problems for JEE Advanced
Level - IV : Tougher problems for JEE Advanced
(0.......9) : Integer type Questions
Passages : Comprehensive link passages
Matching : Matrix Match
Reasoning : Assertion and Reason
Previous years papers : Questions asked in Previous Years’ IIT-JEE Exams

Remember friends, no problem in mathematics is difficult. Once you understand the concept, they will become easy.
So please don’t jump to exercise problems before you go through the Concept Booster and the objectives. Once you are
confident in the theory part, attempt the exercises. The exercise problems are arranged in a manner that they gradually
require advanced thinking.
I hope this book will help you to build your base, enjoy the subject and improve your confidence to tackle any type of
problem easily and skillfully.
My special thanks goes to Mr. M.P. Singh (IISc. Bangalore), Mr. Yogesh Sindhwani (Head of School, Lancers International
School, Gurugram), Mr. Manoj Kumar (IIT, Delhi), Mr. Nazre Hussain (B.Tech.), Dr. Syed Kashan Ali (MBBS) and Mr.
Shahid Iqbal, who have helped, inspired and motivated me to accomplish this task. As a matter of fact, teaching being the
best learning process, I must thank all my students who inspired me most for writing this book.
I would like to convey my affectionate thanks to my wife, who helped me immensely and my children who bore with
patience my neglect during the period I remained devoted to this book.
I also convey my sincere thanks to Mr Biswajit Das of McGraw Hill Education for publishing this book in such a
beautiful format.
viii  Preface

I owe a special debt of gratitude to my father and elder brother, who taught me the first lesson of Mathematics and to
all my learned teachers—Mr. Swapan Halder, Mr. Jadunandan Mishra, Mr. Mahadev Roy and Mr. Dilip Bhattacharya, who
instilled the value of quality teaching in me.
I have tried my best to keep this book error-free. I shall be grateful to the readers for their constructive suggestions
toward the improvement of the book.

Rejaul Makshud
M. Sc. (Calcutta University, Kolkata)
Contents

Preface vii

1. Real Function 1.1–1.95


Basic concepts of Real Functions  1.1
Algebraic operation on domain of a function  1.2
Range of a function  1.2
Types of functions  1.2
Classification of functions with respect to its images  1.12
Inverse Function  1.14
Compostion of Functions  1.14
Even and Odd Functions  1.15
Algebra of Even and Odd Functions  1.15
Even and Odd extensions of a function  1.16
Periodic Function  1.16
Functional Equation  1.18
Exercises  1.18
Answers  1.40
Hints and Solutions  1.40

2. Inverse Trigonometric Functions 2.1–2.84


Introduction to Inverse Function  2.1
Inverse Trigonometric Functions  2.1
Graphs of Inverse Trigonometric Functions  2.2
Constant Property  2.4
Conversion of Inverse Trigonometric Functions  2.4
Composition of Trigonometric Functions and its Inverse  2.5
Composition of Inverse Trigonometric Functions and Trigonometric Functions  2.5
Sum of Angles  2.6
Multiple Angles  2.7
More Multiple Angles  2.7
Exercises  2.8
Answers  2.24
Hints and Solutions  2.25
x  Contents

3. The Limit 3.1–3.67


Meaning of x a  3.1
Neighbourhood of a Point  3.1
Limit of a function  3.1
Formal definition of a limit  3.1
Concept of infinity  3.1
Concept of limit  3.1
Algebra of limits  3.2
Evaluation of limit  3.2
Exercises  3.5
Answers  3.22
Hints and Solutions  3.22

4. The Continuity and Differentiability 4.1–4.71


Introduction  4.1
Continuity  4.1
Reasons of Discontinuity  4.1
Types of discontinuities  4.1
Continuity of an Interval  4.3
Single Point Continuity  4.4
Properties of Continuous Functions  4.4
Differentiability  4.4
Exercises  4.9
Answers  4.28
Hints and Solutions  4.28

5. Differentiation 5.1–5.66
Introduction  5.1
Definitions  5.1
Derivative of some standard functions  5.2
Differentiation of Composite function  5.3
Differentiation by Inverse Trigonometric Function  5.3
Formula for differentiation of inverse function  5.3
Differentiation of Implicit Functions  5.4
Logarithmic Differentiation  5.4
Differentiation of Infinite Series  5.4
Differentiation of Parametric Functions  5.4
Differentiation of a function w.r.t another function  5.4
Higher Order Derivatives  5.4
Differentiation of a Determinant  5.4
Leibnitz Rules for Differentiation  5.5
Exercises  5.5
Answers  5.21
Hints and Solutions  5.21
Contents  xi

6. Rolle’s theorem and Lagrange’s mean value theorem 6.1–6.24


Rolle’s Theorem  6.1
Lagranges Mean Value Theorem  6.2
Constant Function Theorem  6.2
Constant Difference Theorem  6.2
Applications of L.M.V. Theorem  6.2
Cauchy’s Mean Value Theorem  6.3
Generalised Mean Value Theorem  6.3
Nature of the cubic polynomial  6.3
Exercises  6.4
Answers  6.10
Hints and Solutions  6.11

7. Monotonocity 7.1–7.36
Definitions  7.1
Test for Finding the Monotonicity at an End Point  7.2
Necessary Condition for Monotonicity  7.2
Sufficient Condition for Monotonicity  7.2
Critical Point  7.2
Application of Monotonocity in Isolations of Roots  7.2
Algebra of Monotonous Functions  7.2
General Approach to Proving Inequalities  7.3
Concave up and Concave Down  7.3
Inflection Point  7.3
Exercises  7.4
Answers  7.14
Hints and Solutions  7.14

8. The Tangent and Normal 8.1–8.43


Introduction  8.1
Secant  8.1
Tangent  8.1
Normal  8.1
Relation between tangent and normal  8.1
Equations of tangent and Normal  8.2
Conditions of different types of tangents  8.2
Equation of Tangent and Normal to a second degree curve  8.2
Length of Intercepts of the tangents by the axes  8.3
Tangents at the Origin or Through Origin  8.3
Angle between two curves  8.3
Shortest distance  8.3
Common tangent  8.3
Tangent to the curve at point P intersects the curve again at Q  8.3
Lengths of tangent, sub-tangent, normal and sub-normal to the curve at a point  8.4
xii  Contents

Exercises  8.4
Answers  8.14
Hints and Solutions  8.14

9. The Maxima and Minima 9.1–9.38


Definitions  9.1
Concept of Local Maxima and Local Minima  9.1
The First Order derivative Test  9.2
Test for Local maxima / Local minima, when f(x) is not differentiable at x = a  9.3
Extremum at end-points  9.3
More idea on Local Maximum and Local Minimum  9.3
Global Maximum / Global Minimum  9.3
Algebra of Global Extrema  9.4
The Second Order derivative Test  9.4
Exercises  9.6
Answers  9.21
Hints and Solutions  9.22
Chapter

1 Real Function

1.2  Domain
Concept Booster
The first set is called the domain of a function. It is denoted
1.  Basic concepts of Real Functions as Df . In f (x) = x + 4 above, Df = A

1.1  Definition 1.3 Co-domain


Let A and B be two non-empty sets. A function ‘f ’ is a rule The second set is called the co-domain of a function. It is
between two sets A and B in such a way that for every ele- denoted as Cf . In f (x) = x + 4 above, Cf = B
ment in A there exists a unique element in B. It is denoted Example-1.  Let A = {1, 2, 3, 4} and B = {4, 5, 6, 8, 10}
as f: A Æ B and it is read as f is a function from A to B and f is a relation from A to B such that f = {(1, 4), (2, 5),
or f maps from A to B. (3, 6), (4, 8)} is f a function?
Let A = {1, 2, 3, 4}, B = {5, 6, 7, 8, 9} If so, then find its domain.
and f (x) = x + 4. Sol.
A B
A B
f 4
1 5
5 2 6
1
6 8
2 3
7 10
3 4
8
4 9

Yes, it is a function. Domain = {1, 2, 3, 4}

Note: 1.4  Image


1. Every point in A is related to some point in B If f(1) = 5, then 5 is called in image of 1 under the function f
2. A point in A cannot be related to two or more points and 1 is called an inverse element of 5 or pre-image of 5.
in B.
3. Two or more points in A can be related to a single 1.5  Range
point in B. The set of all images is called the range of a function. It
4. There some point in B which are not related to any is denoted as Rf Here, Rf = {5, 6, 7, 8} Õ B. In another
point in A. way, we can define, the range of a function is a subset of
5. Every function is a relation but every relation is not co-domain.
a function.
6. If the sets A and B consists of m and n elements 1.6  Real function
respectively, then the total number of function If the domain and co-domain are the subsets of a real num-
between A to B is nm and the total number of rela- ber, then it is called a real valued function or simply a real
tions between A to B is 2nm . function. It is generally denoted as f : R Æ R.
1.2  Differential Calculus Booster
__________
1.7 Number of real functions between two sets A 5. y = ​÷x  2 – 3x   
+ 2  ​
and B
Ans.
A f B
1. Df = R, Rf = [2, •)
a1 b1 2. Df = [2, •), Rf = [0, •)
a2 b2
a3 b3 3. Df = [– 3, 3], Rf = [0, 3]
a4 b4 4. Df = (– •, 2] » [2, • ) and Rf = [0, •)
am bn 5. Df = (– •, 1] » [2, •) and Rf = [0, •)
m n
4. Types of functions
If the number of elements of a set A contains are m and the
set B are n, then the number of real functions between two Basic functions can be divide into two categories
sets A and B are nm. (i) Algebraic Function
Let A = {1, 2, 3} and B = {2, 6}. Then the number of (ii) Transcendental Function
functions between two sets A and B are 23 = 8
4.1  Algebraic Functions
2. Algebraic operation on domain of a function
(A) Polynomial functions
(i) Dom ( f ± g) = Dom f « Dom g (i) Constant Function
(ii) Dom ( f ± g ± h) = Dom f « Dom g Dom h (ii) Identity Function
(iii) Dom ( f ◊ g) = Dom f « Dom g (iii) Parabolic Function
(iv) Dom ( f ◊ g ◊ h) = Dom f « Dom g « Dom h (iv) Even Power Function

(  )
f (v) Cubical Function
(v) Dom ​ __
​ g  ​  ​ = Dom f « Dom g – {x : g (x) = 0} (vi) Odd Power Function
____ (B) Rational functions
    ) = {x : f(x) ≥ 0}
(vi) Dom (​÷f(x) ​
(i) Reciprocal Function
(vii) Dom (​  loga ( f (x)) )​ = {x : f (x) > 0}
(ii) Even power reciprocal Function
(viii) Dom (​  a f(x) )​ = Dom f provided a > 0 (iii) Odd power reciprocal Function
(C)   Irrational functions
3.  Range of a function
(i) Square root Function
There is no specific method to find out the range of a func- (ii) Even root Function
tion. But the following points should be kept in mind in
(iii) Odd root Function
finding the range of a function.
(D) Piece-wise defined functions
(i) First we find the domain of a function f(x).
(i) Modulus Function
(ii) If Df lie finite number of points, then the Rf is the
set of corresponding values of {f(x)} (ii) Signum Function/Sign Function
(iii) If Df = R, R – {some finite points}, then we express (iii) Greatest Integer Function
x in terms of y and define x. (iv) Least Integer Function
(iv) If D f is a finite interval, say, [a, b], then Rf is (v) Fractional part Function
the greatest and the least values of y = f (x) i.e.
Rf = [Least Value, Greatest Value] 4.2 Transcendental Functions
Note: We should note that determining range of a function
(Non-algebraic functions)
is comparatively more difficult proposition than determin- (i) Trigonometric Functions
ing domain of a function. (ii) Inverse Trigonometric Functions
Q.  Find the domains and ranges of each of the following (iii) Exponential Functions
functions (iv) Logarithmic Functions
1. y = x2_____
+2 (A)  Polynomial Function
2. y = ÷​ x  – 2 ​
  A function f  : R Æ R is defined as
_____
​ 9  – x2 
3. y = ÷  ​ f (x) = a0 + a1 x + a2 x2 + ... + an x n where n Œ W
______
4. y = ÷​ x  2 – 4  
 ​
Real Function  1.3

Degree of a polynomial
The highest index power of x having non-zero co-efficient
is called the degree of the polynomial. The degree of the
above polynomial is n, when an π 0.
For examples,
The degree of the polynomials
1. f(x) = 3 is 0
2. f(x) = 2x + 4 is 1 Df = R and Rf = [0, •)
3. f(x) = x2 + 3x + 2 is 2
4. f(x) = x3 + 3x2 + 5x + 6 is 3 (iv) Even power Function
5. f(x) = x2014 + 10x2000 + 5x199 + 6 is 2014 A function f :R Æ R is defined as
6. f(x) = 0 is undefined. f (x) = x2n, n ΠN
Note: i.e. f (x) = x2, x4, x6, x8, x10,......
1. The domains and ranges of a polynomial depends
on the degree of a polynomial.
2. If the degree of a polynomial is odd, then its domains
and ranges are R.
3. If the degree of a polynomial is even, then its will
not be all real number.
(i) Constant Function
A function f  : R Æ R is defined as f (x) = constant
=c
Y
Domains and ranges will remain same
(O, C) y = c
i.e Df = R and Rf = [0, •)
(v) Odd Power Function
X¢ X
O A function f : R Æ R is defined as
f (x) = x2n + 1, n Œ N
i.e. f(x) = x3, x5, x7, x9, x11,......

D f = R, Rf = {c}
(ii) Identity Function
A function f  : R Æ R is defined as f (x) = x
Y

x
y=

X¢ X Df = R and Rf = R
O
(B)  Rational Functions
A function f : R Æ R is defined as
g(x)

f(x) = ____
​    ​, h(x) π 0, g(x) and h(x) are two polynomials
h(x)
functions.
Df = R, R f = R
(i) Reciprocal Function
(iii) Parabolic Function
A function f :R Æ R is defined as f (x) = x2 A function f :R Æ R is defined as
1.4  Differential Calculus Booster

1 ​  lim ​ (y – mx) =   
and    ​  lim ​ (f (x) – mx) = c.
f(x) = __
​ x ​ x Æ ± • x Æ ± •
1
Suppose f(x) = x + __ ​ x ​

Now  ​  
y
__ (  )
lim ​ ​ ​   ​  ​ =  
x Æ • x
f (x)
____
​ lim ​ ​ ​  x   
x Æ •
(  )
​  ​ =  
x Æ •
x + __
______
​ lim ​ ​ ​  x   
1
​ x ​ 
(  )
​  ​ = 1

lim ​ (y – x) =  
and ​  
x Æ •
​ lim ​ (y – x) =  
x Æ •
​ lim ​ ​ x + __
x Æ •
1
(  )
​ x ​ – x  ​

=0
fi  y = x is an oblique asymptote
1
Note:  1. Let f (x) = _____
​       ​
x–2
Df = R – {0} = Rf Vertical Asymptote: Dr = 0
Asymptote fi x – 2 = 0
It is a straight line which touches the curve at infinity. fi x = 2
There are three types of asymptotes Horizontal Asymptote: y =   ​ lim ​ f(x)
x Æ •

(  )
(i) Vertical Asymptote
1
(ii) Horizontal Asymptote fi ​ lim ​ ​ _____
y=  ​     ​  ​ = 0
x Æ • x – 2
(iii) Oblique Asymptote.
Then  Df = R – V.A = R – {2}
(i) Vertical Asymptotes  A line x = a is said to be a
and  Rf = R – H.A = R – {0}.
vertical asymptotes of the graph y = f(x) if
​ lim    ​ f (x) = ± • (ii) Even power reciprocal function
x Æ a
1 A function f  : R Æ R is defined as
Suppose f(x) = _____
​     
 ​
x–2 1
f(x) = __
​  2  ​ 
Then the vertical asymptote is x – 2 = 0 x
Y
fi x=2
1
Also, if f(x) = ______
​  2    
 ​, then its vertical asymptotes
x – 9
are x2 – 9 = 0 fi x = ± 3 X¢ X
(ii) Horizontal Asymptotes  A line y = b is said to be
a horizontal asymptote of the graph y = f(x) if
​ f(x) = b =   
​ lim 
    ​  lim ​ f(x)
x Æ • x Æ – •

1 Df = R – {0} and Rf = R+
Suppose f(x) = _____
​      ​
x–1
1
Then the horizontal asymptote is Note:  The graph of f (x) = ___
​  2n  ​,  n Œ N – {1}

(  )
x
1
y=  ​ lim ​ ​ _____
​ lim ​ f(x) =   ​     ​  ​ = 0
x Æ • x Æ • 1 – x 1 1 __ 1 1
i.e. f(x) = __
​  4  ​ , __
​    ​,  ​    ​,  ___
​    ​,  .....
x–2 x x6 x8 x10
Also, let f(x) = ​ _____ 
 ​ 
x+2
Then its horizontal asymptote is

y= 
​ lim  
xÆ • xÆ • ( 
x+2
x–2 )
​ lim ​ ​ ​ ______ ​   
​ f(x) =    ​= 1

(iii) Oblique Asymptotes


A line y = mx + c is said to be an oblique asymptote
of the graph y = f(x) if

​   
y
lim ​ ​ __
​ x ​  ​ =   
x Æ ± •
(  ) f(x)
​  lim ​ ​ ____
​  x    (  )
​   ​ = m
x Æ ± •
Real Function  1.5

(iii) Odd power reciprocal functions


A function f  : R Æ R is defined as

1
f(x) = _____    ​, n Œ N
​  2n + 1
x
1 1 __ 1 1
i.e. f(x) = __
​  3  ​ , __
​    ​,  ​    ​,  __
​    ​,  ....
x x5 x7 x9

Df = R – {0} = Rf
i.e. f(x) = x1/2, x1/4, x1/6, x1/8,....
Df = [0, •) and Rf = [0, •)

(iii) Cube root Function


A function f : R Æ R is defined as
__
f(x) = 3÷
​ x 
  ​ 

(C)  Irrational Functions


The algebraic function containing one or more terms
having non-integral rational powers of x are called irrational
functions.
A function f : R Æ R is defined as
2p + 1

_____​ 
f(x) = (g(x)​)​ ​ m    ​, p, m, Œ N
D f = R = Rf
It is undefined for g(x) < 0
Note:
More over these functions are also not defined when __
1. The graph of y = f (x) = 3​÷x 
   ​is the image of the graph
denominator is zero.
of y = f(x) = x3 with respect to the line y = x.
(i) Square root Function
A function f : R Æ R is defined as
__
f(x) = ÷
​ x 
  ​ 

Df = [0, •) and Rf = Rf = [0, •) 2n+1 __


Note:  The graph of y = f(x) = ​÷x 
   ​, n Œ N
(ii) Even root Function
i.e y = x1/3, x1/5, x1/7,....
A function f : R Æ R is defined as
2n __
f(x) = ​÷x 
  ​,  n ŒN
1.6  Differential Calculus Booster

(iii) Greatest Integer Function (G.I.F)


A function f : R Æ R is defined as
f (x) = [x] £ x
Greatest integer of x means, we shall consider of all
those integers which are less than or equal to x.
i.e. any Real Number = Integral + Fractional Part
fi  Greatest integers of x = Integral part of x.
Thus, [2.1] = 2, [3.5] = 3, [0.8] = 0, [1.9] = 1, [– 0.7]
= – 1, [– 1.7] = – 2, [– 2.8] = – 3 etc.

Therefore, [x] = n, if n £ x < n + 1.
Df = R = Rf

(D)  Piece-wise Defined Functions Ï- 2 : - 2 £ x < -1


Ô- 1 : -1£ x < 0
(i) Modulus Function ÔÔ
A function f  : R Æ R is defined as Thus y = f (x) = [x] = Ì0 : 0 £ x <1
Ô1 : 1£ x < 2
Ï x : x≥0 Ô
f(x) = |x| = Ì ÔÓ2 : 2£ x<3
Ó- x : x < 0

Df = R and Rf = [0, •)
(ii) Signum Function (Sign function/sgn function)
A function f : R Æ R is defined as f (x) = sgn (x)

Ï | x| Ïx
Ô : xπ0 Ô : xπ0
= Ìx Ì | x| Df = R and Rf = I
ÔÓ0 : x = 0 Ô0 : x = 0
Ó
Properties of greatest integer function
Ï1 : x > 0 (i) [x] = x, if x Œ I
Ô
= Ì0 : x = 0 . (ii) [– x] = – 1 – [x], if x œ I
Ô- 1 : x < 0
Ó Ï 0 : x ŒI
(iii) [x] + [–x] = Ì
Ó- 1 : x œ I
(iv) [x + m] = [x] + m, m Œ I
Ïx+y : x, y Œ I
ÔÔ
(v) [x + y] = Ì [ x ] + [ y] : 0 £ fx + fy < 1
Ô
ÔÓ[ x ] + [ y] + 1 : 1 £ f x + f y < 2
(vi) If [x] = n if n £ x < + 1
(vii) [x] ≥ n fi x ≥ n, n Œ I
(viii) [x] > n fi x ≥ n + 1, n Œ I
(ix) [x] £ n fi x < n + 1, n Œ I
Df = R and Rf = {– 1, 0, 1}
Real Function  1.7

(x) [x] < n fi x < n, n Œ I For examples,


(i) If x = 3.45, [x] = 3
[ 1
(xi) [x] + ​ x + __
Œ N
] [  2
​ n ​  ​ + ​ x + __ ] [  n – 1
​ n ​  ​ + ...+ ​ x + ____
​  n    ]
 ​  ​ = [nx], n Thus, {x} = x – [x] = 3.45 – 3 = 0.45
(ii) If x = 5, then [x] = 5
Thus, {x} = x – [x] = 5 – 5 = 0

[  ] [  ] [  ] [  ]
n+1
(xii) ​ ​ _____
2
 ​  
n+2
 ​ + ​ ​ _____
4
 ​  
n+4
 ​ + ​ ​ _____
8
 ​  
n+8
 ​ + ​ ​ _____
16
 ​  
 ​ + ... = n, n (iii) If x = – 2.75, [x] = – 3
Thus, {x} = x – [x] = – 2.75 + 3 = 0.25
Œ N As we know that, x – 1 < [x] £ x
(iv)  Least Integer Function fi – x £ – [x] < 1 – x
fi – x + x £ x – [x] < 1 – x + x
A function f : R Æ R is defined as f(x) = (x) ≥ x
fi 0 £ x – [x] < 1.
The least integer of x means, we shal considere of all
fi 0 £ {x} < 1.
those integers, which are more than and equal to x.
Though ‘0’ is not a function, but fractional part func-
For examples, (2.5) = 3, (3.1) = 4, (1.7) = 2, (0.5) = 1
tion evaluates to zero for integral values. We should keep
(– 1.4) = – 1 etc.
this exception in mind, while working with fractional part
Ï- 2 : - 3 < x £ - 2
Ô- 1 : - 2 < x £ - 1 function.
Ô Ïx + 3 : - 3 £ x < - 2
Ô0 : -1 < x £ 0 Ôx + 2 : - 2 £ x < - 1
y = f(x) = Ì Ô
Ô1 : 0 < x £1 Ô x + 1 : -1 £ x < 0
Ô2 : 1< x £ 2 y = f(x) = Ì
Ô Ôx : 0 £ x <1
Ó3 : 2< x£3 Ôx - 1 : 1 £ x < 2
Ô
Thus, y = (x) = n, n < x £ n + 1 Óx - 2 : 2 £ x < 3
y Thus, y = {x} = x – n, n £ x < n + 1

y = [x]

x¢ x

Df = R and Rf = [0, 1).

Properties of Fractional part Function


(i) 0 £ {x} < 1

(ii) {– x} = 1 – {x}, x œ I
Df = R and Rf = I
(iii) {– x} = 0, x Œ I
Properties of Least Integer Function (iv) {x + m} = {x}, m Œ I
(v) If {x} = f, 0 < f < 1
(i) (x) = x, x Œ I
then x = n + f, when n Œ I
(ii) (x) = [x], x Œ I 1
(iii) (x) = [x] + 1, x œ I For examples, if {x} = __ ​   ​ 
2
In fact, (x) = – [– x] 1
then x = n + __ ​   ​ , n Œ I
(iv) (x + n) = (x) + n, n Œ I 2
(v) (– x) = – x, x Œ I (vi) If {x} < f, 0 < f < 1
(vi) (– x) = 1 – (x), x œ I then n £ x < n + f, n Œ I
1
(v)  Fractional part Function For examples, if {x} < __ ​   ​ 
3
A function f : R Æ R is defined as 1
__
then n £ x < n + ​   ​ , n Œ I
f(x) = {x} = x – [x] 3
1.8  Differential Calculus Booster

(vii) If {x} > f, 0 < f < 1 Characteristics of cosine function


then n + f < x < n + 1, n Œ I 1. Df = R
1 2. Rf = [– 1, 1]
For examples, if {x} > __ ​    ​ 3. It is an even function.
4
1 4. It is a periodic function.
then n + __
​   ​  < x < n + 1, n Œ I 5. It is non-monotonic function.
4
6. If cos x = 1 fi x = 2 np, n Œ I
4.2  Transcendental Functions 7. cos x = – 1 fi x = (2n + 1) p, n Œ I
p
All functions which are not algebraic are called transcen- 8. cos x = 0 fi x = (2n + 1) __ ​   ​ , n Œ I
dental functions. 2
9. If cos x > 0
(i) Trigonometric Functions fi (  p p
)
x Œ ​ (2n – 1)​ __ ​ , (2n + 1)​ __ ​   ​, n Œ I
2 2
(a)  Sine function:  A function f  : R Æ R is defined as 10. If cos x < 0
f(x) = sin x
Graph of f(x) = sin x
fi (  p
2
p
2 )
x Œ ​ (2n + 1) ​ __ ​ , (2n + 3) ​ __ ​   ​, n Œ I
11. If x > y fi cos x < cos y, " x, y Œ (0, p)
Y (c)  Tangent function:  A function f  : R Æ R is defined as
f(x) = tan x.
y=1
Graph of f(x) = tan x
x p x Y
o
y = –1

Y¢ X¢ X
O
Characteristics of sine function
1. Df = R
2. Rf = [– 1,1] Y¢
3. It is an odd function. Characteristics of tangent function
4. It is a periodic function.
p
5. It is non-monotonic function. 1. Df = R – (2n + 1) ​ __ ​ , n Œ I
p 2
6. If sin x = 1 fi x = (4n + 1) __
​   ​ , n Œ I 2. Rf = R
4
3. It is an odd function.
p
__
7. sin x = – 1 fi x = (4n – 1) ​   ​ , n Œ I 4. It is a periodic function.
4
8. sin x = 0 fi x = np, n Œ I 5. It is monotonic function.
9. If sin x > 0 fi x Œ (2np, (2n + 1)p), n Œ I p
6. If tan x = 1 fi x = (4n + 1) ​ __ ​ , n Œ I
10. If sin x < 0 fi x Œ ((2n – 1)p, 2np), n Œ I. 4

( 
p
p 7. If tan x = – 1 fi x = (4n – 1) ​ __ ​ , n Œ I
 p
11. If x > y fi sin x > sin y, " x, y Œ ​ – ​ __ ​ ,
2
__
)
​   ​   ​
2

4
8. If tan x = 0 fi x = np, n Œ I
(b)  Cosine function:  A function f  : R Æ R is defined as
9. If tan x > 0
f(x) = cos x
Graph of f(x) = cos x
fi (  p
2 )
x Œ ​ np, (2n + 1) ​ __ ​   ​, n Œ I
Y
10. If tan x < 0

( 
(0, 1)
p

y=1

X

2 )
x Œ ​ (2n – 1) ​ __ ​ , np  ​, n Œ I
O
y = –1
11. If x > y fi tan x > tan y " x, y Œ R – np, n Œ I
(d)  Co-tangent function:  A function f : R Æ R is defined
Y¢ as f(x) = cot x
Real Function  1.9
p
Graph of f(x) = cot x 7. If cosec x = – 1 fi x = (4n – 1) ​ __ ​ , n Œ I
4
8. cosec x can never be zero.
9. If cosec x > 0 fi x Œ (2np, (2n +1)p), n Œ I
10. If cosec x < 0 fi x Œ ((2n – 1)p, 2np), n Œ I.
(f)  Secant function:  A function f : R Æ R is defined as
f(x) = sec x
Graph of f(x) = sec x

Characteristics of co-tangent function


1. Df = R – np, n Œ I
2. Rf = R
3. It is an odd function. X¢ X
O
4. It is a periodic function.
5. It is monotonic function.
p
6. If cot x = 1 fi x = (4n + 1) ​ __ ​ , n Œ I Y¢
4
p Characteristics of secant function
7. If cot x = – 1 fi x = (4n – 1) ​ __ ​ , n Œ I p
4 1. Df = R – (2n + 1) __​   ​ , n Œ I
2
p
8. If cot x = 0 fi x = (2n + 1) __ ​   ​ , n Œ I 2. Rf = (– •, – 1] » [1, •)
2
9. If cot x > 0 3. It is an even function.
fi (  )p
x Œ ​ np, (2n + 1) ​ __ ​   ​, n Œ I
2 4. It is a periodic function.
10. If cot x < 0 5. It is non-monotonic function.
fi (  p
2 )
x Œ ​ (2n – 1) ​ __ ​ , np  ​, n Œ I

6.
7.
If sec x = 1 fi = 2np, n Œ I
If sec x = – 1 fi x = (2n + 1) p, n Œ I
(e)  Co-secant function:  A function f : R Æ R is defined as
f(x) = cosec x 8. sec x can never be zero.
Graph of f (x) = cosec x 9. If sec x > 0

Y
fi (  p p
)
x Œ ​ (4n – 1) ​ __ ​ , (4n + 1) ​ __ ​   ​, n Œ I
2 2
10. If sec x < 0
fi (  p p
)
x Œ ​ (4n + 1) ​ __ ​ , (4n + 3) ​ __ ​   ​, n Œ I
2 2
(ii)  Inverse Trigonometric Functions

O
X
[ 
p p
(a) sin– 1x : A function f : [– 1, 1] Æ ​ – ​ __ ​ , __
– 1
f(x) = sin x = arc sin x
2 2 ]
​   ​   ​ is defined as

Graph of f (x) = sin– 1 x.

Characteristics of co-secant function


1. Df = R – np, n Œ I
2. Rf = (– •, – 1] » [1, •)
3. It is an odd function.
4. It is a periodic function.
5. It is non-monotonic function.
p
6. If cosec x = 1 fi x = (4n + 1) ​ __ ​ , n Œ I
4
1.10  Differential Calculus Booster

haracteristics of arc sine function


C Characteristics of arc tangent function
1. Df = [– 1, 1] 1. Df = R

[ 
p p
]
2. Rf = ​ – ​ __ ​ , __
2 2
​   ​   ​ ( 
p p
2. Rf = ​ – __
​   ​ , __
2 2 )
​   ​   ​

3. It is not a periodic function.


3. It is not a periodic function.
4. It is an odd function since, tan– 1(– x) = – tan– 1x
4. It is an odd function.
5. It is strictly increasing function.
since, sin– 1 (– x) = – sin– 1x
6. It is one-one function
5. It is strictly increasing function.
p
6. It is one-one function. 7. For 0 < x < __ ​   ​ , tan– 1x < x < tan x.
2
p (d) cot– 1 x : A function f : R Æ (0, p) is defined as
7. For 0 < x < ​ __ ​ , sin x < x < sin– 1.
2 f (x) = cot– 1x.
(b) cos– 1x : A function f : [– 1, 1] Æ [0, p] is defined as
f (x) = cos– 1x = arc cos x Graph of f (x) = cot– 1x
Graph of f (x) = cos– 1x

Characteristics of arc co-tangent function


1. Df = R
haracteristics of arc cosine function
C 2. Rf = (0, p)
1. Df = [– 1, 1] 3. It is not a periodic function.
2. [0, p] 4. It is neither even nor odd function
3. It is not a periodic function. since, cot– 1 (– x) = p – cot– 1 x
4. It is neither even nor odd function 5. It is strictly decreasing function.
since, cos– 1(–  x) = p – cos– 1(x) 6. It is one-one function
5. It is strictly decreasing function. p
7. For 0 < x < __ ​   ​ , cot x < x < cot– 1x
2
[ 
6. It is one-one function.
7. For 0 < x < __
p
​   ​ , cos– 1x < x < cos x
2
p p
2 2 ]
(e) cosec– 1: A function f : (– •, –1] » [1, •) Æ ​ – ​ __ ​ , ​ __ ​   ​

( p p
2 2 )
(c) tan– 1x : A function f : R Æ ​ – ​ __  ​, ​ __  ​  ​ is defined as
– {0} is defined as f (x) = cosec– 1x.
Graph of f (x) = cosec– 1x.
f (x) = tan– 1 x.
Graph of f (x) = tan– 1x Y

y = p/2

X¢ X
O

y = – p/2

X = –1 Y ¢ X=1
Real Function  1.11

Characteristics of arc co-secant function Now, f(x) = a cos x + b sin x + c


= (sin q cos x + cos q sin x)
1. Df = (– •, –1] » [1, •)
= r = sin (q + x)

[ 
p p
2. R f = ​ – ​ __ ​ , __
2 2 ]
​   ​   ​ – {0}
As we know that, – 1 £ sin (q + x) £ 1
fi – r + c £ r sin (q + x) + c £ r + c
3. It is an odd function, since fi – r + c £ f(x) £ r + c
_______ ______
cosec– 1(– x) = – cosec– 1(x) – ​÷a  2 + b2 
 ​ + c £ f(x) £ ​÷a  2+ b2 ​ 
+c
_______
4. It is non-periodic function.
​ a  2 + b2 
Thus, the maximum value of f (x) is ÷  ​ + c
5. It is one-one function. _______
​ a  2 + b2 
and the minimum values of f (x) is – ÷  ​ + c
6. It is strictly decreasing function with respect to its
domain.
p
__ (iii)  Exponential Function
​   ​ 
7. For 0 < x <
2 A function f  : R Æ R is defined as
cosec– 1x < cosec x
f(x) = ax, a π 1, a > 0, x Œ R
(v) sec– 1x : A function f : (– •, –1] » [1, •) Æ [0, p]
{  }
p Case I:   When a > 1
– ​ __
​   ​   ​ is defined as f(x) = sec– 1 x.
2
Graph of f (x) = sec– 1 x.


Df = R and Rf = R+
Case II: When 0 < a < 1

Characteristics of arc secant function


1. Df = (– •, –1] » [1, •)


p
2. Rf = [0, p] –​ __ {  }
​   ​   ​
2
3. It is neither an even function nor an odd function,

since sec– 1 (– x) = p – sec– 1(x)
Df = R and Rf = R+
4. It is non-periodic function.
5. It is one-one function. (iv) Logarithmic Function
6. It is strictly decreasing function with respect to its A function f  : R Æ R is defined as f(x) = loga x, a π 1,
domain. a > 0, x > 0
p
7. For 0 < x < __ ​   ​  Case I: When a > 1
2
sec– 1x < x < sec x.

(ii)  The maximum and minimum values of


f (x) = a cos x + b sin x + c
We have f(x) = a cos x + b sin x + c
Let a = r sin q and b = r cos q
_______
a
Then  ​ and tan (q) = __
r = ​÷a  2 + b2  ​   ​
b
1.12  Differential Calculus Booster

Df = R+ and Rf = R

Case II: when 0 < a < 1

Let A = {1, 2, 3, 4 }, B = {5, 6, 7, 8, 9}


and f (x) = x + 4.
Df = R+ and Rf = R Thus, f (1) = 5, f (2) = 7, ....., f (4) = 8. Clearly f is a
one-one function.
(v) Equal Functions (b) Checking algebraically:
(i) If x π y fi f (x) π f (y), then it is one-one
function.
(ii) If x π y fi f(x) = f(y), then it is not a one-one
function.
(c) Checking geometrically: Draw a system of lines
parallel to x-axis on the given curve. If those lines
intersects the curve in only one point, then it is one-
one function otherwise many-one function.
(d) Checking by Calculus: If a function is either strictly
increasing or strictly decreasing function, then it is
one-one function.
i.e. either f ¢(x) > 0 or f ¢(x) < 0, then it is one-one
function.
Example-1.  Let y = f(x) = 3x + 4
Then f  ¢(x) = 3 > 0
fi  f(x) is strictly increasing function.
fi  f(x) is one-one function.

Two functions f and g are said to be equal if Example-2.  Let f(x) = tan–1(3 log (x) + 5)
1 1
(i) Df = Dg Then f  ¢(x) = _______________
​       ​ × __
​   ​ > 0 for all positive x
(ii) Rf = Rg 1  +  (3 log x + 5)2 x
(iii) f(x) = g (x), " x Œ D fi  f(x) is strictly increasing function.

5. Classification of functions with respect to its images fi  f(x) is one-one function.

There are 5 types of functions w.r.t its images such as Example-3.  Let f(x) = 2 tan x + 3 sin x + 4 cos x + 10
(i) One-One Function Then f ¢(x) = 2 sec 2x + 3 cos x – 4 sin x.
(ii) Many-One Function fi f ¢(x) > 0 for all x in R.
(iii) Onto Function fi f(x) is strictly increasing function.
(iv) Into Function fi f(x) is one-one function.
x
(v) One-one-onto Function Example-4.  Let f(x) = _____
​       ​
1 + |x|
5.1 One-One function Case I: when x ≥ 0
x
(a) Definition: If different elements of the first set pro- Then f(x) = _____
​      

1+x
vide us different images in the second set, then it is
known as one-one function or injective function. (1 + x) – 1 1
fi f(x) = ​ _________
  
 ​ = 1 – _____
​      ​
1+x 1+x
Real Function  1.13

1 (b) Number of onto function between two sets A and


fi f ¢(x) = _______
​       ​ > 0
(1 + x)2 B: If two set A and B having m and n elements
respectively, where m ≥ n, then the number of onto
Case II: when x < 0 function between two sets A and B = Number of
x distribution of m balls into n boxes where no box is
Then f (x) = _____
​       ​
1–x remain empty.
x–1+1 1
f(x) = – ​ ________  ​ 
 = – 1 – _____
​     
 ​
x–1 x–1 5.4  Into Function
1
f ¢(x) = _______
​       ​> 0 for all x < 0 If a function is not onto function, then it is into function.
(x – 1)2 In another way we can say, if the range of a function is a
Thus f is strictly increasing function. Hence, f is one-one proper subset of a co-domain then it is into function.
function.
(e) Number of one-one function between two sets A
and B.


The number of into functions
= The total number of functions

  – The number of onto functions.
If A and B having m and n elements respectively,
then the total number of one-one functions between
5.5 One-one onto Function
ÏÔ0 : n<m
two sets A and B is = Ì n (a) Definition: If a function is one-one as well as onto
ÔÓ Pm : n≥m function, then it is one-one-onto function or bijective
function.
5.2  Many-One Function
Note: One-one-onto function exists only when the
If a function is not one-one function, then it is many-one number of elements of both the sets are same.
function.
Number of many one function
Number of many one function = Number of total func-
tion – Number of one-one function.

5.3 Onto Function
(a) Definition: If the range of a function is equal to its
co-domain, then it is called Onto function or surjec-
tive function
i.e. Rf = Co-domain. (b) Number of one-one onto functons between two
sets A and B.


1.14  Differential Calculus Booster

Let two sets A and B having m and n elements (viii) If f : A Æ B be a one one function, then f – 1of =
respectiovely. IA and fof – 1 = IB , where IA and IB are the identity
Then the number of one-one onto functions between functions of the sets A and B respectively.
ÏÔ0 : mπn (ix) Let f : A Æ B, g : B Æ C be two invertible functions,
two sets A and B = Ì n then gof is also invertible with (gof) – 1 = (f –1og –1).
ÔÓ Pm : m=n
Rule to find out the Inverse of a Function
6. Inverse Function (i) First, we check the given function is bijective or
not.
(ii) If the function is bijective, then inverse exists,
otherwise not.
(iii) Find x in terms of y and replace y by x, then we get
inverse of f. i.e. f – 1(x).

7. Compostion of Functions
Let f and g be two real functions such that f : A Æ B and
g : B Æ C.
Here the set B is common to the two functions. Thus we
can define a function h : A Æ C for which h(x) = g(f (x)),
Let f : X Æ Y be a bijective function. If we can make a where for all x Œ A, f (x) Œ B, g( f (x)) Œ C. Then h = go f
function g from Y to X, then we shall say that g is the inverse is the composition of f and g. We read as h as g of f or
of f. “g composed with f ”
1
i.e. g = f – 1 π __ ​   ​ . A B C
f
Thus, f – 1(  f (x)) = x f g
X f(X) g{f(x)}
Note:
(i) The inverse of a function exists only when the
function f is bijective.
gof
(ii) If the inverse of a function is exists, then it is called
Similarly we can define a function u : A Æ D such that
an invertible function.
u(x) = (ho (go  f  ))(x) = h (g(  f (x)))
(iii) The inverse of a bijective function is unique.
f g h
(iv) Geometrically f – 1(x) is the image of f (x) with respect
to the line y = x.
A B C D

x f(x) g(fx) h(g(f(x)))

gof ho(gof)

Properties of Composition of Functions


(i) Let f : A Æ B and g : B Æ C be two real functions.
Then go f is defined only when if Rf Õ Dg.
(ii) Composition of functions is not commutative i.e. if
f and g are two real functions such that fog & go f,
(v) In another can say that f – 1(x) is the symmetrical
then fog π go  f.
with respect to the line y = x.
(iii) Composition of functions is associative. i.e. if f, g, h
(vi) A function f(x) is said to be involution if for all
are three real functions such that (fo (go) oh)) are exist,
x for which f(x) and f(f(x)) are defined such that
then (fo (goh)) = ((fog) oh)
f(f(x)) = x.
(iv) The composition of two bijections is bijection i.e f
(vii) If f is an invertible function, then (f  – 1) = f.
and g are two bijections, then go  f is also bijection
Real Function  1.15

(v) The composition of any function with the identity 4. Product


function is the function itself. (i) E × E = E
i.e. foI = Io f = f (ii) O × O = E
8. Even and Odd Functions (iii) E × O = O
(iv) O × E = O
8.1  Even Function 5. Reciprocality
1
A function f (x) is said to be an even function if f (– x) (i) ​ __ ​  = E
E
= f (x) for all x in Df . Geometrically, an even function is
1
symmetrical about y-axis. (ii) ​ __  ​  = O
O
6. Quotient
E 1
(i) ​ __ ​ = E × __
​   ​  = E × E = E
E E
O 1
(ii) ​ __ ​  = O × __
​     ​ = O × O = E
O O
O 1
(iii) ​ __ ​  = O × __
​   ​  = O × E = O
E E
E 1
(iv) ​ __ ​  = E × __
​    ​  = E × O = O
O O
7. Composition
8.2 Odd Function (i) E(E) = E
(ii) E(O) = E
A function f (x) is said to be an odd function if f(– x) = – f(x),
" x ΠDf . Geometrically, an odd function is symmetrical (iii) O(E) = E
about the origin. (iv) O(O) = O
Note: Composition of functions will be odd only when
all are odd.

Properties of even and odd functions


1. A function which is both even as well as odd is a
zero function.
i.e. f (x) = 0, " x Œ Df
2. A non-zero constant function is an even function.
i.e. f (x) = c, c π 0 is an even function.
3. Odd function always pass through the origin.
9. Algebra of Even and Odd Functions
4. Even function is many-one function.
Let the even function, odd function and neither even nor odd 5. The derivative of an odd function is even and an even
functions are denoted as E, O and Æ EO function is odd.
1. Addition 6. The integral of an odd function is even and even
(i) E + E = E function is odd.
(ii) O + O = O 7. Inverse of an even function is not defined.
(iii) E + O = EO 8. Every function can be expressed as a sum of an even
(iv) O + E = EO and an odd function.
2. Negativity 1
Let f (x) = __​   ​  × (2f (x))
(i) – E = E 2
1
(ii) – O = O = __
​   ​  × ((f (x) + f (– x)) + (f (x) – f (– x)))
2
3. Difference
1 1
(i) E – E = E = __
​   ​  (f (x) + f (x)) + __
​   ​  (f (x) – f (– x))
2 2
(ii) O – O = O
= g(x) + h(x), (say)
(iii) E – O = EO
(iv) O – E = EO =E+O
1.16  Differential Calculus Booster

To prove g(– x) = g(x) & h(– x)


1
Now, g(x) = __
​    ​ (  f (x) + f (– x))
2
1
fi g(– x) = __ ​   ​  ( f (– x) + f(x))
2
1
fi g(– x) = __ ​   ​  ( f (x) + f(– x)) = g(x)
2
fi g(x) is an even function.
1
Also, h(x) = __ ​   ​  ( f (x) – f (– x))
2
Let f (x) be defined on [a, b]
1
fi h(– x) = __ ​   ​  ( f (– x) – f (x)) Then its odd extension can be written as
2
1
fi h(– x) = – ​ __ ​  ( f (x) – f (– x)) = – h(x) Ï f ( x) : a£x£b
2 g(x) = Ì
fi h(x) is an odd function. Ó- f (– x ) : - b £ x < - a
Hence, the result. 11. Periodic Function
10. Even and Odd extensions of a function A function f : R Æ R is said to a periodic func-
tion if there exists a positive real number T such that
10.1  Even Extension f (x + T) = f (x), " x Œ R
Let f (x) be defined on [0, a] The least positive value of T is called the fundamental
Then the even extension of f is defined as period or simply the period of a function.
Geometrically, if the graph of y = f (x) repeat itself after
Ï f ( x) : 0 £ x £ a a fixed interval, then the width of the interval is called the
g(x) = Ì
Ó f (– x ) : - a £ x < 0 period of the function y = f (x).

Geometrically, an even extension of a function is the Example-1.  Let y = sin x


mirror image of the graph of f (x) with respect to y-axis.

Since the graph of y = sin x repeats after a fixed interval


2p
Thus, the period of sin x is 2p
Let f (x) be defined on [a, b]. Then its even extension can
be written as
Example-2. Let y = tan x

Ï f ( x) : a £ x £ b
g(x) = Ì
Ó f (– x ) : - b £ x < - a

10.2 Odd Extension
Let f (x) be defined on [0, a]
Then its odd extension is defined as

Ï f ( x) : 0£ x£a
g(x) = Ì
Ó - f (– x ) : -a £ x < -0 Since the graph of y = tan x repeats after a fixed
interval p
Geometrically, odd extension of a function is symmetrical
about the origin. Thus, the period of y = tan x is p
Real Function  1.17

Example-3.  Let y = {x} = x – [x] f (x)


(iv) ​ ____  , g(x) π 0 is the L.C.M of {T1, T2}
 ​ 
g(x)

Example-6.  Find the period of f (x) = sinx + sin 2x


Sol.  Here, the period of sin x = 2p and the period of
2p
sin 2x = ___
​   ​ = p
2
Thus, the period of f (x) = L.C.M of {2p, p} = 2p

Example-7.  Find the period of f(x) = sin 2x + cos 3x


Since the graph of y = x – [x] repeats after a fixed
interval 1. 2p
Sol.  Here, the period of sin 2x = ___
​   ​ = p and
So the period of y = x – [x] is 1. 2
2p
the period of cos 3x = ___
​   ​ 
Basic concepts of L.C.M 3
{ 2p
Thus, the period of f (x) = L.C.M of ​ p , ___ }
​   ​   ​
{ 
L.C.M. of {a, c, e, ...} 3

a
1. L.C.M of ​ __
​   ​,
b
c
__
​   ​,
d
e
__
}
​   ​ , ...  ​ =
f
​ ___________________
   
     ​
H.C.F. of {b, d, f, ...} 
L.C.M. of {p, 2p} 
= ________________
​    
    ​
2. L.C.M of rational with rational is possible. H.C.F. of {1,3}
3. L.C.M of irrational with irrational is also possible. 2p
4. But L.C.M of rational with irrational is not = ___
​   ​ = 2p
1
possible. 3. If f (x) and g(x) both are periodic with periods T1 and T2
5. Two important irrational number are p, e. respectively, and both are even & co-functions and pairwise
Example-4. The L.C.M  of comparable funtions, then the period of
(i) f (x) + g(x)
1 1 __
​ __
​   ​ , __
2 3 4 { 
1
}
L.C.M. of {1, 1, 1}
​   ​ , ​   ​   ​ =​ __________________
  
   ​
H.C.F. of {2, 3, 4}
(ii) f (x) – g(x)
(iii) f (x). g(x)
1 f (x) 1
= __
​   ​  = 1 (iv) ​ ____  , g(x) π 0 is the L.C. M of __
 ​  ​    ​ × {T1, T2}
1 g(x) 2
Example-8.  Find the period of f (x) = sin4x + cos4x
Properties of periodic functions Sol.  Here, the period of sin4x = p and the period of
1. If f (x) is a periodic function with period T then the cos4x = p
period of Since both the functions are periodic, even, co-function
(i) f (a x) as well as both are pairwise comparable functions, so the
(ii) f (ax + b) period of f (x) is
1
(iii) f (ax + b) + c = ​ __ ​  × L.C.M. of {p, p}
2
T
(iv) df (ax + b) + c is __
​    ​  p
|a| = __
​   ​ 
2
Example-5.  The period of Example-9.  Find the period of f (x) = cos(cos x) + cos
(i) sin 2x (sin x)
(ii) sin (2x + 3) Sol.  Here, the period of cos(cos x) is p and the period of
(iii) sin (2x + 3) + 5 cos(sin x) is also p
2p
(iv) 7 sin (2x + 3) + 5 is ​ ___ ​ = p Since both the functions are periodic, even, co-function
2 as well as both are pairwise comparable functions, so the
2. If f (x) and g (x) are two periodic functions with periods period of f (x) is
T1 and T2 respectively, then the period of 1
= __
​   ​  × L.C.M. of {p, p}
(i) f (x) + g(x) 2
(ii) f (x) – g(x) p
= __
​   ​ 
(iii) f (x). g(x) 2
1.18  Differential Calculus Booster

4. If f (x) is a periodic functions with period T, then the Thus, the period of cos x is 2p, whereas the period of
period of sin x is also 2p
1
(i) ​ ____    ​  7. Constant function is periodic having no fundamental
f (x) period.
(ii) k f(x), k Œ R + – {1} 8. Algebraic function is non periodic.
(iii) (f (x))1/n is also T.
____ Note:
Example-10.  Find the period of f (x) = ÷​ sin
  x  
​ 1. The functions sinn x, cosn x, cosecn x, secn x are
Sol.  Since the period of sin x is 2p, so the period of f (x) periodic with period 2p when n is odd and p when
is also 2p n is even.
2015 _____ 2. The functions tann x, cotn x are periodic with periods
Example-11.  Find the period of f (x) = ​÷cos
  ​
x   p, whatever n may be.
Sol.  Since the period of cos x is 2p, so the period of f (x) 3. The functions |sin x|, |cos x|, |tan x|, |cot x| |cosec x|,
is also 2p |sec x| are periodic with period p.
1
Example-12.  Find the period of f (x) = ____
​     
 ​ 4. The functions |sin x| + |cos x|, |tan x| + |cot x|,
sin x p
| cosec x | + |sec x| are periodic with period ​ __ ​ 
Sol.  Since the period of sin x is 2p, so the period of f (x) 2
is also 2p 5. The functions |sin x| – |cos x|, |tan x| – |cot x|,
|cosec x| – |sec x| are periodic with period p.
5. If f (x) is a periodic function with period T and g(x) be ____ _____ _______ _____
a non-periodic function, then g(f (x)) is periodic with period 6. The functions ​÷sin  x   ​, ÷
​ cos
  ​, ÷
x   ​ cosec
  ​ and ÷
x   ​ sec
  ​
x  
T and f (g(x)) is non periodic. are periodic with period 2p.
7. The functions sin–1(sin x), cos–1(cos x) sec–1 (sec x),
Example-13.  Find the period of f (x) = sin–1 (sin x) cosec–1 (cosec x) are periodic with period 2p.
Sol.  Let h(x) = sin x and g(x) = sin–1x
8. The functions tan–1 (tan x), cot–1 (cot x) are periodic
Thus, the period of h(x) is 2p and g(x) is non-periodic. with period p.
Therefore, the period of f (x) is = 2p
9. The functions sin–1x, cos–1x, tan–1x, cot–1x are non-
Note:  1. The function f (x) = sin (sin–1x) is non-periodic, periodic.
whereas sin x is periodic but sin–1x is non-periodic. 10. The function x –[x] is periodic with period 1.

6. If f (x) is a periodic function with period T then f  ¢(x) is 12. Functional Equation


also periodic with period T
It is an equation where the unknown is a function. A func-
Example-14.  Find the period of f (x) = sin x tional equation asks for a formula satisfying certain condi-
Sol.  We have f  ¢(x) = cos x tions. On solving such type of equation we can get one or
more functions as solutions.

Exercises
2x
5. If f (x) = ​ ______
    ​, show that, f (tan q) = sin 2q
1 + x2 
(Problems Based on Fundamentals)
Basics Concepts of Real Functions
x
(  )
6. If f (x) = log ​ ____
​     
x –1
 ​  ​, show that,

1. If f (x) = 3x4 – 5x2 + 9, find f (x – 1). f (x + 1) + f (x) = log ​ ​ _____  (  )
x+1
x–1
 ​  ​


x
2. If f (x) = _____
​     
x+1
1
 ​, prove that (f (x))3 = f (x3) + 3f  ​ __ (  )
​ x ​  ​ (  )
1+x
7. If f (x) = log ​ ​ _____ 
1–x (  )
2x
 ​  ​, show that, f  ​ ​ _____
  
1 + x2
 ​  ​ = 2f (x)

(  )
1–x
3. If f (x) = log ​ ​ _____ 
1+x
 ​  ​, show that, 8. If f (x) = cos(log e x), find the value of

( 
1
x+y
f (x) + f (y) = f  ​ ​ _____ 
1 – xy

​   ​ ) f (x) f (y) – __
​   ​  ( f (x) + f (y))
2
ax – b
9. If y = f (x) = ​ ______ 
4. If f(x) = x3 – __
1 1
(  )
​  3  ​ , show that, f (x) + f  ​ __
x
​ x ​  ​ = 0. bx – a
 ​, show that x = f (y)
Real Function  1.19

1
10. If for non zero x, af (x) + bf  ​ __ (  )
​ x ​  ​ =
1
__
​ x ​ – 5,
3x2 + 9x + 17
(x) f (x) = ​ ___________
  
   ​
3x2 + 9x + 7
where a π b, find f (x). 15. Find the domain of each of the following functions
9x _____
11. If f (x) = _____
​  x    ​  , then prove that f (x) + f (1 – x) = 1. ​ x  – 2 ​ 
(i) f (x) = ÷
9 +3 _____

(  2 tan x
12. If f ​ ​ ________
1 + tan2 x
   
) 1
 ​  ​ = __
​   ​  (1 + cos 2x) (sec2 x + 2 tan x)
2


​ x  + 5 ​ 
(ii) f (x) = ÷
_____
​ 4  – x  
(iii) f (x) = ÷ ​
_____ _____
then find f (x). ​ x  – 2 ​ + ÷
(iv) f (x) = ÷ ​ 4  – x  

_____ _____
ABC of a Real Functions ​ x  – 3 ​ – ​÷7  – x  
(v) f (x) = ÷ ​

÷ 
_____
13. Which one of the followings are functions? If so then x–2
find its Domains and Ranges. If not, then find when (vi) f (x) = ​ ​ _____ 

 ​ ​
5–x

it can be a function and also find their Domains and _____
x


Ranges..
(i) y = m x + c

÷_____
(vii) f (x) = ​ _____
​     
x–1
 ​ ​

​ 1  – __
(viii) f (x) = ÷
1
(ii) y = 0 ​ x  
​ ​
_____
​  x ​ – 1 ​ 
(iii) x = 0
(ix) f (x) = ÷
1
(iv) y = 5 ​ __
_____
​ ​  _____
(v) x = 3
(x) f (x) = ÷
x–2
x    ​ ​ 
(vi) x2 + y2 = 9
(vii) y2 = 4x 16 Find the domain of each of the following functions
_____
(viii) x2 = 4y (i) f (x) = ÷   – 2 ​ 
​ |x|
_____
x2 y2 ​ 4  – |x| ​ 
(ii) f (x) = ÷
(ix) ​ __ ​  + ​ __ ​  = 1 _____ _____
9 4 (iii) f (x) = ÷   – 1 ​ + ​÷4
​ |x|   – |x| ​ 

÷ 
2 ______
x 2 y
(x) ​ ___  ​ – ​ __ ​  = 1
|x| – 1
(iv) f (x) = ​ ​ _____    ​ ​
16 9 3 – |x|
_____
(xi) x2 – y2 = 4 (v) f (x) = ÷   – x ​ 
​ |x|
(xii) y = (x – 1) (x – 2) (x – 3) 1
(vi) f (x) = ______
​  _____
     ​
Domain of Real Functions ÷   – x  
​ |x| ​
1
14 Find the domain of each of the following func- (vii) f (x) = _______
​  _____
     ​
tions. ​ x  – |x|  
÷ ​
______
x–2   – x  
(i) f (x) = ​ _____   ​ (viii) f (x) = ÷
​ [x] ​
x+3
1
x+4 (ix) f (x) = _______
​  ______
    ​
(ii) f (x) = ​ _____   ​   – x  
​÷[x] ​
x–3
1

1
(iii) f (x) = __
​ x ​ – 1 (x) f (x) = _______
​  ______
     ​
​÷x  – [x] ​ 
1
(iv) f (x) = __
​ x ​ + 1 Algebraical functions
1

x–1
(v) f (x) = ​ _________     ​ 17. Find the domain of f (x) = ______
​       ​
x2 – 3x + 2 [x – 2]
x+3
x2 – 5x + 4 18. Find the domain of f (x) = ​ ______   ​
(vi) f (x) =​ __________     ​ [x + 1]
x2 – 3x + 2
x2 + x + 1
x2 + 4 19. Find the domain of f (x) = ​ _________  ​


(vii) f (x) = _____
​  2  
 ​ [x] – 4
x +2
1
x2 + 9 20. Find the domain of f (x) = _______
​  ______
    ​
(viii) f (x) = ​ _____   ​ ​ x  – [x] ​ 
÷
x2 – 4
1
x2 + x + 1 21. Find the domain of f (x) = _______
​  ______
    ​
(ix) f (x) = ​ ________  
 ​ ÷   – x  
​ [x] ​
x2 – x + 1
1.20  Differential Calculus Booster

22. Let f (x) = cos[p 2]x + cos[– p 2]x


[  ]
1
39. Prove that [2x] = [x] + ​ x + __
​   ​   ​
2
p
Then find f ​ __
2 (  )
​   ​   ​, f (p)
​   ​  ]​ + [​  x + __
40. Prove that [3x] = [x] + [​  x + __
1
​    ​ ]​
2
3 3
23. Find the exact value of
[loge 1] + [loge 2] + [loge 3] + ... + [loge 66]
24. 2
Solve for [x] – 3[x] + 2 = 0

2x2
41. Find the number of integral values of ​ ​ _____  
x2 + 1
 ​  ​ [  ]
25. Solve for [x] 2 – 2[x] – 8 = 0 42. Solve for x : {x} + {sin x} = 2
26. Find the number of solution of x2 – 4 – [x] = 0 43. If {x} and [x] represent fractional and integral part
27. Find the value of of x respectively, then find the value of

[  ] [​  __​ 14  ​ + ____ ​ + ​  __ ​     ​  ​ + ... + [​  __


100 ] [ 4 100 ] 4 100 ] ( 
1 1 1 2 1 99
​ __
​   ​   ​ +
4
​     ​   ​   ​  + ____ ​   ​  + ​ ____  ​  ​ 2014 {x + r}
​  ​ ​ ​ ​ ______
[x] + S
r = 1 2014
 ​  

​. )
where [,] = G.I.F.
44. Solve for x : 4{x} = x + [x], where {x} and [x] are the
28. Find the value of

[  ] [  ] [ 
fractional and integral part of x respectively.
3
4
1
[1007] + ​ 503 ​ __  ​  ​ + ​ 252 __
5
​   ​   ​ + ​ 126 ​ ___  ​  ​ + ...
8 16 ] 45. Solve for x : 2x + 3{x} = 4[x] – 2
29. Solve the equation [sin x] = 0 46. Solve for x : x2 – 4x + [x] + 3 = 0
30. If y = 2[x] + 3 = 3 [x – 2] + 5, then find the value 47. Find the number of values of x satisfying
of [x + y] {x2} + [x4] = 1
______
31. Find the doamin of f (x) = ÷   – 2 ​ 
​ [x] 48. Find the number of solutions of
______ {x} + {tan p x} = 0
​ 4  – [x] ​ 
32. Find the domain of f (x) = ÷ . 49. Find the number of integral values of

{  }
______ ______
33. Find the domain of f (x) = ÷   – 1 ​ + ÷
​ [x] ​ 3  – [x] ​  x2 + 1
y = ​ ​ _____   ​, 0 £ x £ 2
 ​  

÷ 
______ 2
[x] – 2
34. Find the domain of f (x) = ​ ​ ______   
 ​ ​ 50. Find the values of x satisfying the equation
5 –  [x]
35. Find the value of 2{ 
cos2 x  – 2
​ ​ ________
 ​  
1
 ​ = __
​    ​
4 }
[  ]
sin2 x
(i) ​ ​ _____
2
 ​  
 ​
51. Find the domain of the function
f (x) = ÷
​ 2{x}
______________
  2 – 3{x}  
+ 1 ​, x > 0

[  (  ) ]
p
(ii) ​ sin2 ​ __
​   ​   ​ x  ​
2
Trigonometric functions
52. Find the max and min values of

(iii) ​[ cos ​( __


f (x) = 3 sin x + 4 cos x + 10
​   ​  )​ x ]​
p
2
53. Find the range of f (x) = sin x + cos x + 3
2
54. Let A = sin4 q + cos4 q, then find A
​ [  ]
cos2 x
(iv) ​ _____
2014
 ​  
​. 55. Find the max and min values of
f (q) = sin6q + cos6q

36. Find the set of values of x satisfying 56. If A = cos2q + sin4q and B = cos4q + sin2q such that
[sin x] + [cos x] = 1, " x Π[0, 2p] m1 = Max of A and m2 = Min of B then find the
37. If x satisfies the equation value of m12 + m22 + m1m2
[x + 0.19] + [x + 0.20] + [x + 0.21] + ... + [x + 0.22] 57. Find the max and min values of
+ ... + [x + 0.91] = 542
f (q) = sin2 (sin q) + cos2 (cos q)
Then find the value of [100x]
38. Find the value of 58. Find the minimum value of
f (q) = (3 sin(q) – 4 cos(q) – 10)
2015
​ _____
2 [  ] [  ] [  ] [  ]
2016
 ​ + ​ _____
​   ​   ​   ​  
4
2018
 ​ + ​ _____
​   ​  
8
2022
 ​ + ​ ​ _____
16
 ​  
 ​ + ... (3 sin(q) + 4 cos(q) – 10)
Real Function  1.21

59. Find the range of A = sin2010q + cos2014q 79. Find the domain of the function
x2 sin2 x + 4
60. Find the minimum value of f (x) = ​ _________
  
   ​, where
2–x
x–4 ( 
f(x) = log1/2 ​ ​ _____ 
 ​  
​ )
(  )
p x sin x
__
x Œ ​ 0, ​   ​   ​
2 80. Find the domain of the function

Exponential functions f (x) = log10 (1 – log10 (x2 – 5x + 16))


x
 ​ __  ​ – 1
61. Find the domain of f (x) = e​ ​2 ​ 81. Find the domain of the function
62. Find the domain of f (x) = | ex – 2 | f (x) = log10 ((log10 x2) – 5log10x + 6)
63. Find the domain of f (x) = | e|x| – 2 | 82. Find the domain of the function

|  1
64. Find the domain of f (x) = ​  e– |x| – __
​   ​   ​
2 | ( 
1
f (x) = log ​ ______
​     
x – [x]
 ​  ​. )
÷ 
_______
1
65. Find the domain of f (x) = ​ e–|x|  – __
​       ​ ​ 83. Find the number of real solutions of
  2 
2 – x – log e x = 0
66. Find the domain of f (x) = (e2x + ex + 1)
Equal Functions
67. Find the domain of f (x) = (ex + 1)2 + 3
84. Let f : {1, 2} Æ {1, 4} and g : {1, 2} Æ {1, 4} such
68. Find the domain of f (x) satisfying ex + e f (x) = e that f (x) = x2 & g(x) = 3x – 2 Is f = g?
69. Find the number of solutions of x2
85. Let f (x) = x and g(x) = ​ __
x ​  Is f = g?
2x + 3x + 4x – 5x = 0.
__
70. Find the number of solutions of 1 + 3x/2 = 2x ​ x  2 ​  Is f = g?
86. Let f (x) = x & g(x) = ÷
Logarithmic functions 87. Let f (x) = 2logx & g(x) = log(x2) Is f = g?
71. Find the domain of the function

f (x) = log2 (x2 – 4x + 3)


x–1
x–2 ( 
88. Let f (x) = log ​​ _____ 
 ​  
​ and )
72. Find the domain of the function g(x) = log(x – 1) – log (x – 2). Is f = g?

÷ 
x
__
_________
f (x) = e​ ​ ​2  ​– 1​ + log(l – x) + x1001 1 – cos  2x
89. Let f (x) = sin x & g(x) = ​ ​ ________
 ​ ​  
Is f = g?

2
73. Find the domain of the function

( 
Classification of Functions
f (x) = log ​ _____
x
​     
x–2
 ​  ​ ) 90. Let A = {1, 2, 3} and B = {4, 5}
74. Find the domain of the function Then find the total number of one-one function
1 between two sets A and B.
f (x) = ________
​       ​
log(x – 2) 91. Let A = {1, 2, 3} and B = {p, q, r}
75. Find the domain of the functions Then find the total number of function between
A and B.
log(l – x) 92. Let A = {1, 2, 3, 4} and B {a, b, c, d, e}
f (x) = ​ __________
      ​
x2 – 3x + 2  Then find the number of one-one function between
A and B.
76. Find the domain of the function
93. Let a function f : A Æ B, where A = {1, 2, 3, 4} and
log(3 – x) B = {3, 4, 5, 6} such that f (1) = 3. Then find the
f (x) = ​ _________    ​ number of one-one function between A to B.
x2 – 5x + 4
94. Let a function f : A Æ B, where A = {1, 2, 3} and
77. Find the domain of the function B = {4, 5, 6} such that f (1) π 4. Then find the number
1 of one-one function between A to B.
f (x) = __________
​       ​
log (9 – x2)  95. Let a function f : A Æ B, where
A = {1, 2, 3} and B = {6, 7, 8}
78. Find the domain of the function such that a < b fi f (a) < f (b),
_____ _____
+ ​÷3  – x ​) 
f (x) = log1/2 (​÷x  – 1 ​  where a Œ A & b Œ B.
1.22  Differential Calculus Booster

Then find the number of one-one function between 113. Find the number of bijective functions between two
A to B. sets A and B, where A = {a, b, c, d, e} and B = {p,
q, r, s, t}
96. Find the number of many-one function between two
sets A = {3, 4, 5} and B = {2, 3, 4, 5}. 114. Find the number of bijective functions between two
sets A = {1, 2, 3, 4} and B = {5, 6, 7, 8} such that
97. Find the number of many-one function between two f (1) = 5.
sets A = {1, 2, 3, 4} and B = {3, 4, 5}.
Inverse of a Function
98. Find the number of many-one function between
two sets A = {1, 2, 3} and B = {4, 5, 6} such that 115. A function f : R Æ R is defined as f(x) = 3x + 5. Find
f (1) π 4 f –1(x).

99. A function f : R – {– 1} Æ R – {1} is defined as 116. A function f : (0, •) Æ (2, •) is defined as f (x) = x2
x + 2. Find f –1(x).
f (x) = _____
​      ​. prove that f is onto function.
x+1 x2
1 117. A function f : R + Æ [0, 1] is defined as f (x) = ​ _____  
 ​.
100. Let a function f : R Æ A is defined as f (x) = ​ _____
2
  
 ​. 2
x +1
–1
x +1 Find f    (x).
If f is onto function, then find the set of values of
A. 118. A function f : [1, •) Æ [1, •) is defined as
x2
101. Let a function f  :  A Æ B is defined as f (x) = ​ _____
2
  
 ​. f(x) = 2x(x – 1). Find f –1 (x).
x +1
If f is onto function, then find the set of values of 10x – 10 –x
119. If a function f is bijective such that f (x) = ​ _________   
 ​.
A « B. 10x + 10 – x
–1
Find f  (x)
102. Find the number of onto function between two sets
A = {1, 2, 3, 4} and B = {4, 5} 120. A function f : R Æ R is defined as f (x) = x + sinx.
Find f  –1(x)
103. Find the number of onto function between two sets
A = {1, 2, 3, 4} and B = {4, 5, 6} 121. A function f : [2, •) Æ [5, •) is defined as
f (x) = x2 – 4x + 9. Find its inverse.
104. Find the number of onto function between two sets A
122. Find all the real solutions to the equation
and B where A = {1, 2, 3, 4, 5} and B = {6, 7, 8}.

÷ 
_____
1 1
105. Find the number of into function between two sets x2 – ​ __ ​  = ​ x + __
​   
 ​ ​ .
A and B, where A = {1, 2, 3} and B = {4, 5}. 4 4

106. Find the number of into function between two sets 123. For what values of m is f (x) = (m + 2)x3 – 3mx2 + 9m
A and B where A = {1, 2, 3, 4} and B = {5, 6, 7}. x – 1 is invertible?
_________________________
107. Find the number of into functions between two sets A   –  x3  +  3bx2   
124. Let f (x) = 3​÷a    –  3bx + b3  +  b ​. Find b,
and B where A = {1, 2, 3, 4, 5} and B = {6, 7, 8}. if f (x) is inverse of itself.
108. A function f : R Æ R is defined as f (x) = 3x + 5.
Prove that f is a bijective function. Inverse Trigonometric Functions
109. A function f : R + Æ (1, •) is defined as f (x) = x2 + 1. 125. Find the domain of f (x) = sin– 1 (3x + 4)
Prove that the function is bijective. 126. Find the domain of f (x) = cos– 1 (4x + 5).
1 127. Find the domains of
110. A function f : R + Æ (0, 1) is defined as f (x) = ​ _____
   ​.
x2 + 1
Prove that f is a bijective function. x
f (x) = cos–1​ __ (  ) 1
​    ​ – 1  ​ + ex + ______
2
​       ​
|x – 1| 
1 1
( 
111. A function f : R Æ ​ – ​ __ ​ , __
2 2 )
​   ​   ​ is defined as 128. Find the domains of

x
f (x) = ​ _____
   
x2 + 1
 ​, " x Œ (–1, 1). Prove that f (x) is a
|x| – 3
f (x) = cos–1 ​ ​ _____
5 ( 
 ​  
1
 ​ + ​ _____
ex + 1 )
   ​

bijective function. 129. Find the domains of


112. If A = {1, 2, 3, 4} and B = {2, 3, 4} then find
the number of one-one onto functions between A
and B.
1– |x|
2 (  ) ( 
f (x) = sin–1​ ​ _____
 ​  
ex – 1
 ​ + ​ ​ ______
ex + 1
  )
 ​  
​ + 2015
Real Function  1.23

131. Find the domain of


​ 
3
130. Find the domain of f (x) = cos–1 ​ __________( 
    ​  ​
4 + 2 sin x ) 146. If the roots of
(c – 1)(x2 + x + 1)2 – (c + 1)(x4 + x2 + 1) = 0 are
1–x
real and distinct and f(x) = ​ _____ 
(  )
 ​, then find the value
1 + x2
(  (  ) )
f (x) = sin–1 ​ ​ _____
   

​  ​ + cos(sin x). 1+x
1
2x of f (f (x)) + f ​ f ​ __ ​   ​   ​  ​.
–1
2
132. Find the domains of f (x) = sin (log2 x).
133. Find the domains of f (x) = sin–1(log4 x2). ( 
p
) p
( 
147. If f (x) = sin x + sin2 ​ x + ​ __ ​   ​ + cos x cos ​ x + ​ __ ​   ​
2
3 3 )
134. Find the domains of
(  )
and g​ __
5
​   ​   ​ = 1, then find (go f)(x)
4

( 
1
f (x) = sin–1 ​ ______
​  2    
|x – 1|
 ​  ​
) 148. Let f (x) = 1 + x2. Find a function g(x) such that
f (g(x)) = 1 + x2 – 2x3 + x4
Composition of Functions 149. Let g(x) = 1 + x – [x] and
135. Let two real functions f and g are defined as Ï- 1 : x < 0
f : R Æ R, f (x) = x2 + 1 and g : R Æ R, f (x) = x – 1. Ô
f (x) = Ì 0 : x = 0 then for all x, find f (g(x))
Determine fog & go f. Ô 1 : x>0
Ó
136. Find fog and go f for the functions f (x) = sin x and a x
_____ 150 Let f (x) = _____
​      ​, x π –1 then for what value of a is
x+1
g(x) = ​÷x  – 2 ​ 
f (f (x)) = x?
137. Find the domain of the function fog, where
_____
​ x  – 3 ​ and g(x) = x2 + 1.
f (x) = ÷ Even and Odd Functions
__ Determine the nature of each of the following functions.
2

( 
138. Find fog, where f (x) = ÷
​ x 
  ​  and g(x) = x – 1.

139. A function f (x) is defined as


3x – 1
151. f (x) = x​ ​ ______
3x + 1

 ​  
​ )
2
f (x) = (a – xn)1/n, x > 0, n Œ I + 152. f (x) = x sin (x + 1)
1
Then find f(f (x)) + f  ​  f ​ __(  (  ) )
​ x ​  ​  ​. 153. f (x) = tan–1 ​( sin(cos–1(x)) )​
154. f (x) = sin x + cos x

( 
140. Find the number of distinct real solutions of the
equation f ​( f ​( f (x) )​ )​ = 0, where f (x) = x2 – 1 155.  (x) = ​ _____
x
e –1 2
x
 ​ + __
​  x     ​    ​ + 1  ​ )
ÔÏ x 2 : x ≥ 0
141. If f (x) = Ì and g(x) = – |x|, x Œ R ( 
3–x
156. f (x) = log​​ _____ 
3+x
  
​  ​ )
ÔÓ x : x < 0 _____
157. f (x) = log​( x + ​÷x  2 + 1 ​ 
 ​ )
Then find fog & go f
_________ _________
​ x  2 – x + 1 ​
158. f (x) = ÷   ​ x  2 + x + 1 ​
–÷  
142. A function f is defined as
159. Determine the nature of the function

( 
Ï1 + x : x ≥ 0
f(x) = Ì
Óx - x : x < 0
. Then find fo f 2–x
f (x) = x sin2 x + tan(x5) + log​ ​ _____ 
2+x
 ​  ​. )
160. Determine the nature of the function
143. Let f : R + Æ R be defined as f (x) = x2 – x + 2 and 2 7

g : [1, 2] Æ [1, 2] be defined as g(x) = {x} + 1, where f (x) = (tan(x5))​ex​ ​sgn(​x​ ​)


{,} = Fractional part function. If the domain and 161. Determine the nature of the function
range of f (g(x)) are [a, b] and [c, d), then find the _____
x2 (log(x + ​÷x  2+ 1 ​ 
))
b __
value of ​ __
d f (x) =​  ___________________
       7  ​
a ​ + ​ c ​. 2
(x cos x + tan x + x )
144. If g is the inverse of f and f  ¢(x) = sin x, find g¢(x). 162 Determine the nature of the function
_____
1
145. If g(x) = ____
​     ​ 
1 – x
and g2 (x) = g(g(x)) and g3 (x) = g(g(g(x))), (  ( 
sin​ tan​ log​( x + ​÷x  2 + 1 ​ 
f (x) =​  ____________________________
_____     
   
  )​  ​  ​
 ​
))
then find the range of g2016 (x). ÷​ x  2 + 1 ​ + sin(cos x) + cos(sin x)  
1.24  Differential Calculus Booster

163. Express the function f (x) = 4sin x as a sum of an even 183. Let f : R – {0} Æ R be any real function such that
and an odd function.
2015
1
f (x) + 3f ​ __ (  )
​ x ​  ​ = 5x. Find f (x).
164. Express the function f (x) = (1 + x) as a sum of
an even and an odd function. 184. Let 2f (sin x) + 3f (cos x) = 5, " x Œ R. Find f (x).
185. Let a function f saisfy f (x + 1) = f (x) + x, " x Œ N
ÔÏ x + x 2 : 0 £ x < 3
165. Let f (x) = Ì where f (1) = 0. Find f (x).
ÔÓ x + x : 3 < x £ 5
186. Suppose f is real function satisfying f (x + f (x)) = 4f (x)
Then find its even and odd extension. and f(1) = 4
166. Let f (x) = x + e x + sin x be defined on [0, 2]. Then Then find f(5).
find its even and odd extension. 187. Find all polynomial P(x) which satisfy the relation
167. Let f (x) = x2 + x + sin x – cos x + loge (1 + x) be P(x + 1) = P(x) + 2x + 1, where P(0) = 0
defined on [0, 1]. Find its even and odd extension in 188. Find the natural number ‘a’ for which
the interval [– 1, 1]
n
Periodicity of functions ​S  ​ ​ ​f (a + k) = 16 (2 n – 1), where f satisfies the
k =1
168. Find the period of f (x) = 3 sin 4x + 4 cos 3x
relation f (x + y)
169. Find the period of f (x) = 3 sin 4x + 4 |sin 4x|
= f (x). f (y) for all x, y are in N and f (1) = 2.
170. Find the period of f (x) = sin x. cosec x
189. Let a polynomail function f satisfies the relation
171. Find the period of f (x) = tan x. cot x.
172. Find the period of
f (x) + f ​ __
1
(  ) 1
(  )
​ x ​  ​ = f (x). f ​ __
​ x ​  ​ for all x Œ R – {0}
__ __
  x  ) + 7 cos (3​÷  2 ​ x).
f (x) = 5 sin (2​÷2 ​ Then find f(x).

173. Find the period of 190. If f (x) is a polynomial function of x, satisfying


__ __
f (x). f (y) = f (x) + f (y) + f (xy) – 2 such that f (1) = 2
  x  ) + 2 cos (5​÷3 ​
f (x) = 3 sin (2​÷3 ​   x  ).
and f (2) = 5, then find the value of f (6) + 2009.
174. Find the period of 191. If P(x) be a polynomial satisfying the identity

(  ( 
p p P(x2) + 2x2 + 10x = 2x P(x + 1) + 3, find P(x).
3 )
f (x) = sin2 x + sin2 ​ x + ​ __ ​   ​ – cos x cos ​ x + __
​   ​   ​
3 ) 192. If f(x) is a polynomial of degree 2 such that
|sin x + cos x| f (0) = 1 and f (x + 2) = f (x) + 4x + 2, find the poly-
175. Find the period of f (x) =​  ______________
  
    ​ nomial f (x).
|sin x| + |cos x|  

|sin x + cos x|
176. Find the period of f (x) =​  _____________
  
    ​ (Mixed Problems)
|sin x| – |cos x| 
Choose the most appropriate one

(  )
1 |sin x| _____ sin x _____
177. Find the period of f (x) = __
​   ​ ​  ​ _____ ​ + ​    ​  ​ 1. If f (x) = ÷   – 1| ​
​ |x  and g(x) = sin x, then (fog)(x) =
2 cos x |cos x|
______
(a) sin ÷   – 1|  ​
​ |x   (b) |sin x/2 – cos x/2|
178. Find the period of f (x) = 3 sin {2x} + 2 cos {3x}
(c) |sin x – cos x| (d) None of these

( 
179. Let f (x) + f (x + 3) = 5 for all x in R. Prove that f (x)
is periodic with period 6.
180. Find the period of the function ‘f ’ which satisfy the

f (x) =
x1 + x 2
2. If f (x1) + f (x2) = f ​ ________
 ​   
1 + x1 x 2
  )
 ​  ​, x1x2 Œ (–1, 1), then

(  (  )
equation f (x + 4) + f (x – 4) = f (x)
181. If f is a function satisfying the equation

1–x
(a) log​ ​ _____ 
1+x )
 ​  ​
1–x
(b) tan–1 ​ ​ _____ 
1+x
 ​  ​
__

(  )  ​( ​ _____ 
1 – x)
f (x – 1) + f(x + 1) = ​÷2 ​     f (x), then prove that f (x) is 2x 1+x
periodic with period 8. (c) log ​ ​ _____
    ​  ​ (d) tan–1  ​  ​
1 + x2
182. Let f (x + 1) + f (x + 5) = f (x + 3) + f (x + 7) for
2x + 2– x
all x in R. Prove that f (x) is periodic Functional 3. If f (x) =​  ________
 ​  , then f (x + y) f (x – y) =
Equation. 2
Real Function  1.25

1 (a) One-one (b) Onto


(a) ​ __ ​  [ f (x + y) + f (x – y)]
2 (c) One-one onto (d) Many-one into
1
(b) ​ __ ​  [ f (2x) + f (2y)] 12. If f (x) and g(x) are two functions of x such that
2 f (x) + g(x) = ex and f (x) – g(x) = e–x then
1
(c) ​ __ ​  [ f (x + y). f (x – y)] (a) f (x) is an odd function
2
(d) None of these (b) g(x) is an odd function

÷  ( 
__________

1
4. The domain of the function f (x) = ​ log​ _____
​     
|sin x|

 ​  ​ ​ is )

(c) f (x) is an even function
(d) g(x) is an even function


(a) R – {–  p , p}
(c) R – {2np /n Œ Z}
(b) R – {np /n Œ Z}
(d) (–  •, •)
(  )
px px
(  )
13. The period of the function sin​ ​ ___ ​   ​ + cos​ ​ ___ ​   ​ is
2 2
10x – 10 – x (a) 4 (b) 6
5. The inverse of the function f (x) = ​ __________
     ​ is
10x + 10 – x (c) 12 (d) 24

(  )
1 1+x 14. The domain of definition of the function y(x) is given
(a) log10 (2 – x) (b) ​ __ ​  log10 ​ _____
​    ​  ​ by the equation 2x + 2y = 2 is
2 1–x
(a) 0 < x £ 1 (b) 0 £ x £ 1
(d) ​ __ ​  log  ​( ​ _____
2 – x)
1 1 2x
(c) ​ __  ​ log10 (2x – 1) 10    
​  ​ (c) – • < x £ 0 (d) – • < x < 1
2 4 2
15. Suppose f(x) = (x + 1) for x ≥ – 1. If g(x) is the func-
6. Domain of the function tion whose graph is reflection of the graph of f (x)
_________________________ with respect to the line y = x, then g(x) equals :
f (x) = ÷   0.5(3x – 8)    
​ log – log0.5 (x2 + 4) ​ is

__
   ​ – 1, x ≥ 0
(a) – ​÷x 
1
(b) ​ _______
     ​, x > – 1

(  ) ( 
(x + 1)2

8
(a) ​ __
​   ​ , •  ​
3
8
(b) ​ – •, __
​   ​   ​
3 )
__
(c) ​÷x    ​ + 1, x ≥ – 1
__
(d) ​÷ x ​  –1, x ≥ 0
x
(c) (– •, •) (d) (0, •) 16. If f : [0, •) Æ [0, •) and f(x) = ____​     ​, then f is
1 + x
7. The domain of the function (a) One-one and onto
f (x) = 16 – xC2x – 1 + 20 – 3xP4x – 5 (b) One-one but not onto
(a) {2, 3} (b) {2, 3, 4} (c) Onto but not one-one
(d) Neither one-one nor onto
(c) {1, 2, 3, 4} (d) (1, 2, 3, 4, 5}
17. Let f (x) = |x – 1|. Then
 x2 – x + 1
8. Range of the function f (x) =​  __________
   ​  (a) f (x2) = (f (x))2 (b) f(x + y) = f (x) + f (y)
x2 + x + 1
(c) f (|x|) = |f (x)| (d) None of these
(a) R (b) [3, •)
18. Let function f : R Æ R be defined by

1
(c) ​ __[  ]
​   ​ , 3  ​
3
(d) None of these f (x) = 2x + sin x for x Œ R. Then f is:
(a) One-one and onto
9. Function f : R Æ R, f (x) = x3 + 7 is (b) One-one but not onto
(a) One-one onto (b) One-one into (c) Onto but not one-one
(c) Many-one onto (d) Many one into (d) Neither one-one nor onto
10. Let f : (– 1, 1) Æ B be a function defined by 19. Let f (q) = sin q (sin q + sin3q). Then f (q)

(  )
2x (a) ≥ 0 only when q ≥ 0 (b) £ 0 for all real q
f (x) = tan–1 ​ ​ _____
   ​  ​ then f is both one-one and onto (c) ≥ 0 for all real q (d) £ 0 only when q £ 0
1 – x2
when B is in the interval 1
20. If  : [1, •) Æ [2, •) is given by f (x) = x + __
​ x ​ then
[  ]
p p
(a) ​ – ​ __ ​ , ​ __ ​   ​
2 2 ( 
p p
(b) ​ – ​ __ ​ , __
2 2 )
​   ​   ​ f –1(x) =
______
(  )
(c) ​ 0, __
p
​   ​   ​
2
p
​2 )
(d) 0,  ​ ​__ ​   ​
x + ​÷x  2 – 4  ​
(a) ​ _________
2

 ​    
x
(b) ​ _____
   
1 + x2
 ​
_____
11. If A = {x| –1 £ x £ 1} = B and f : A Æ B _____
x – ​÷x  2 – 4 ​ 
f (x) = sin(p x), then f is (c) ​ ________  ​     (d) 1 + ​÷x  2 – 4 ​

2
1.26  Differential Calculus Booster

1 ___________
21. If f (x) = cos(log x), then (c) ​ __ ​  (1 + ​÷1  + 4 log  2 x) ​ (d) Not defined
2
1 x
f (x) f (y) – __
2 y [  (  )
​   ​​  f ​ __ ]
​   ​  ​ + f(xy)  ​ has the value
29. Let f (x) = ​ _____
ax
x+1
   ​, x π – 1. Then, for what value of a
1 is f (f (x))?
(a) – 1 (b) ​ __ ​  __ __
2
  
(a) ​÷2 ​   
(b) – ​÷2 ​
(c) – 2 (d) None of these
(c) 1 (d) – 1
log2 (x + 3) 30. Which of the following function is periodic
22. The domain of definition of f(x) = __________
​  2     ​ is
x + 3x + 2 (a) f (x) = x – [x] where [x] denotes the greatest
(a) R/{– 1, – 2} (b) (– 2, •) integer less than or equal to the real number x

(c) R/{– 1, – 2, – 3} (d) (– 3, •)/{– 1, 2}


1
(b) f (x) = sin ​ __ (  )
​ x ​  ​ for x π 0, f (0) = 0

23. Let f(x) = (1 + b2)x2 + 2bx + 1 and let m(b) be the (c) f (x) = x cos x
minimum value of f (x). As b varies, the range of (d) None of these
x
m(b) is
31. Let f : (0, •) Æ R and F (x) = ​Ú   ​ ​   ​f  (t) dt.
(a) [0, 1] [  ] 1
(b) ​ 0, ​ __ ​   ​
2
0

[  ]
2 2
1 If F(x ) = x (1 + x), then f (4) equals:
__
(c) ​ ​   ​ , 1  ​ (d) (0, 1]
2 (a) 5/4 (b) 7
24. The domain of definition of the function (c) 4 (d) 2
1 _____ __ 2
y = __________
​  ​ x  + 2 ​
     ​ + ÷  is: 32. If g(f (x)) = |sin x| and f(g(x) = (sin ​÷x 
  ​)  , then:
log10 (1 – x) 2 __
(a) f (x) = sin  x, g(x) = ÷
​ x 
   ​
(a) (– 3, – 2) excluding – 2.5
(b) f (x) = sin x, g(x) = |x|
(b) [0, 1] excluding 0.5 __
(c) f(x) = x2, g(x) = sin ÷ ​ x 
  ​ 
(c) (– 2, 1) excluding 0
(d) f and g can not be determined
(d) None of these.
33. Let E = {1, 2, 3, 4} and F = {1, 2}. Then the number
25. Let g(x) = 1 + x – [x] and of onto functions from E to F is :
Ï- 1, x < 0 (a) 14 (b) 16
Ô
f (x) = Ì 0, x = 0 , then for all x, f (g(x)) = (c) 12 (d) 8
Ô 1, x > 0
Ó 34. If f (x) = cos [p2] x + cos [– p2] x, where [x] stands
(a) x (b) 1 for the greatest integer function, then:
(c) (x) (d) g(x)
–1
26. If f (x) = 3x – 5, then f  (x)
p
(  )
(a) f ​ __
​   ​   ​ = –1
2
(b) f (p) = 1

(  )
1
(a) Is given by ______
​       ​ (c) f (– p) = 0
p
(d) f ​ __
​   ​   ​ = 1
3x – 5 4
x  +  5 2x – 1
(b) Is given by _____
​   ​    35. If S is the set of all real x such that​  ____________   
   ​
3 2x3 + 3x2 + x
is positive, then S contains:
(c) Does not exist because f is not one-one

(d) Does not exist because f is not onto (  ) 3


(a) ​ – •, – ​ __ ​   ​
2 (  )
3 1
(b) ​ – ​ __  ​, – ​ __ ​   ​
2 4
x2 + x + 2
(c) ​( – ​ __ ​ , __ (d) ​( __
27. Range of the function f (x) = ​ _________
4 2)
​    ​, 3 )​
; ​ x Œ R is
     1 1 1
x2 + x + 1 ​   ​   ​
2
(a) (1, •) (b) (1, 11/7) 36. Domain of definition of the function
(c) (1, 7/3] (d) (1, 7/5)

÷ 
_____________
p
28. If the function f : [1, •) Æ [1, •) is defined by + __
f (x) = ​ sin–1 (2x)   ​   ​ ​  for real valued x, is
6
f (x) = 2x(x – 1), then f – 1(x) is:

(  )
1 x(x – 1)
(a) ​​ __
​   ​   ​​
2

1 ___________
(b) ​ __ ​  (1 + ​÷1  + 4 log  
2 2 x)  ​
[  ]
1 1
(a) ​ – ​ __ ​ , __
4 2
​   ​   ​ [  ]
1 1
(b) ​ – ​ __ ​ , __
2 2
​   ​   ​
Real Function  1.27

( 
1
(c) ​ – ​ __ ​ ,
1
__
)
​    ​  ​ [  __​ 14  ​ ]​
1
(d) ​ – ​ __ ​ ,
2x(sin x + tan x)
44. Let f(x) = _______________
​    
   ​ . Then the function f(x)

2 9
37. For a positive integer n, let
4
is [ 
x + 21p
2​ ​ _______
p    

​  ]
​ – 41

( 
(a) odd (b) even
q
2 )
f n (q) = ​ t an ​ __  ​   ​ (1 + sec q )(1 + sec 2q ) (c) periodic (d) None.
(1 + sec 4q) ... (1 + sec 2nq) then:
45. Let f: [– 2, 2] Æ R, where
(  ) p
(a) f 2 ​ ___
​    ​  ​ = 1 (  ) p
(b) f 3​ ___
​    ​  ​ = 1
16 32 x2 + 1
f(x) = x3 + sin x + ​ ​ _____[  ]
a   
 ​  ​ be an odd function. Then
(c) f ​( ___ (d) f ​( ____
p p
​    ​  )​ = 1
128 )
4  5 ​    ​  ​= 1 the set of values of ‘a’ is
64
(a) R (b) (– •, 5)
38. If f (x) = sin x + cos x, g(x) = x2 – 1, then g(f (x)) is (c) (5, •) (d) [– 5, 5]
invertible in the domain:
46. Let A = {a1,........, a5} and B = {y1,........, y5}. Then
[  ]p
(a) ​ 0, __
​   ​   ​
2 [ 
p
(b) ​ – ​ __ ​ ,
4
p
__
]
​   ​   ​
4
the number of one-one functions of
f : A Æ B such that f(xi) π yi, i = 1, 2, 3, 4, 5 is
(c) ​[ – ​ __ ​ , ​ __ ​  ]​
p p
(d) [0, p] (a) 40 (b) 44
2 2
(c) 24 (d) 60
39. The range of the function
47. The number of bijective functions of f : A Æ A, where
f : [0, 1] Æ R, f (x) = x3 – x2 + 4x + 2 sin–1 x is A = {1, 2, 3} such that f(1) π 3, f(2) π 1, f(3) π 2
is
(a) [– p – 2, 0] (b) [2, 3]
(a) 1 (b) 2
(c) [0, 4 + p] (d) (0, 2 + p] (c) 9 (d) None
40. The range of the function 48. The number of onto functions from A to B where
________ ________ A = {1, 2, 3, 4, 5} and B = {3, 4, 5} is
f (x) = ÷  
​ sin(cos x) ​
  +÷  
​ cos(sin x) ​
  is
______ (a) 100 (b) 120

(a) Î1, 1 + ​÷cos(1) ​ ˚ 
______ ______ (c) 140 (d) 150

(b) Î​÷cos(1) ​, 1 + ÷   (1) ​˚ 
​ cos  1
______ ______ 49. The domain of the function f(x) = __________
​  _________
     ​ is
______
  (1) ​ 
(c) Î​÷cos ,1+÷   (1) ​˚ 
​ sin  ​÷  ÷
x–1
  – 3 ​ ​
​ 81   
______
  (1) ​˚ 
(d) Î1, 1 + ​÷sin  (a) {2, 3} (b) {3, 4}
(c) (2, 3) (d) (3, 4)
sin x + sin 3x + sin 5x + sin 7x
41. If f (x) =​ ____________________________
      ​, then the
    
cos x + cos 3x + cos 5x + cos 7x 50. The domain of the function
period of f (x) is
f(x) = logs (4 – x) (x – 1) – sin–1 [2x – 3] is
p p (a) (1, 2) (b) (1, 2.5)
(a) ​ __ ​   (b) ​ __ ​ 
4 3 (c) (1, 1.5) (d) (3, 4).
p
__
(c) ​   ​   (d) p
2
42. If the period of f (x) = cos (x + 4x + 9x + ... + n2 x) (Problems for JEE-Advanced)
p
is __
​   ​  then the value of n, n Œ N, is
7 (  )
1
1. If 2f(x) + 3f ​ __
​ x ​  ​ = x – 3, x π 0, find f(x)
(a) 2 (b) 3
2. If f(x) + 2f(1 Рx) = x2 + 2, " x ΠR, find f(x).
(c) 4 (d) 5
43. Let f (x) = [sin 3x] + |cos 6x|, where [,] = G.I.F. Then 3. If f(2x + 3y, 2x – 7y) = 20x, find f(x, y).
the period of f (x) is 4. If f(x) is a polynomial function satisfying


(a) p /3
(c) p /6
(b) 2p /3
(c) p /12 (  )
1
f(x) + f ​ __ (  ) 1
​ x ​  ​ = f(x). f ​ __
​ x ​  ​ and f(3) = 28, find f(4).
1.28  Differential Calculus Booster

÷ 
_______
5. If f(x + y, x – y) = xy, find the arithmetic mean of [x] – 1
f(x) = ​ ​ ______   ​ ​ 
, where [,] = G.I.F
f(x, y) and f(y, x). 4 – [x]

2F(n) + 1 22. Find the domain of f(x) = sin–1(log2 x)


6. If F(n + 1) = ​ ________ n Œ N and F(1) = 2, find
 ​ , 

(  )
2
1 + x2
the value of F(2015) 23. Find the domain of f(x) = sin–1​ ​ _____
   

​  ​
2x

(  (  ) )
7. If f(x) = cos(log e x), find the value of
x2
24. Find the domain of f(x) = cos–1​ log2​ ​ __ ​   ​  ​
1
f(x). f(y) – __
x
​   ​  ​ ​ __
2 {  (  ) }
​ y ​  ​ + f(x y)  ​ 2

8. If f(x + 1) + f(x – 1) = 2f(x) and f(0) = 0 then find 25. Find the domain of
f(n), n ΠN.

(  )
4x 1 f(x) = log4 (log5(log3 (18x – x2 – 77)))
9. Let f(x) = ​ _____      ​
, find the value of f ​ _____
​     ​ 
  ​ + f ​
4x + 2 2015 __________________
26. ​ 1  – logx log
Find the domain of f(x) = ÷    
x
2 (4 – 12) ​

(  ) (  )
2
_____ 3
_____
(  )
2014
_____
​     ​  ​ + f ​ ​     ​  ​ + .... + f ​ ​ 
2015 2015 2015
 ​  ​.
27. Find the domain of the function y = f(x) where
 4 2
3y + 2​x​ ​ = 24x​​ ​ – 3
(  1
10. Let f  ​ x + __ )
​ x ​  ​ = x2 + __
1
​  2  ​ , x π 0, find f(x)
x 28. Find the domian of definitions of

(  ( 
______

))
n – 1 m
​  ​ ​ ​ f(r), find ​S  ​ ​ ​f (n)
11. Let f(1) = 1 and f(n) = 2 S ÷​ 4  – x2 ​ 
r = 1 r = 1 f(x) = sin​ loge ​ ​ ______    
​  ​  ​
1–x
12. Consider the function f(x) and g(x) are defined as
f(x) = (x + 1) (x2 + 1) (x4 + 1)... (​x2​
2007
​ + 1) and 29. Find the domain of the function
2008
g(x) (​x2​ ​ – 1) = (f(x)–1), find g(2). f(x) = (x12 – x9+ x4 – x + 1) –1/2
13. If f(x – 1) = x2 – 1, find f(x). 30. Find the range of the function f(x) = x2 + x + 1
8
14. Let f be a function such that f  ​ ​ _______
__    
( 
__  ​  ​ = x,
​÷ 1 ​  + ​÷ x  ​ )
x2
31. Find the range of the function f(x) = ​ _____
x2 + 1
  
 ​

for all x ≥ 0, find the value of f(4). x2


32. Find the range of the function f(x) = ​ _____
4
  
 ​
15. Find the natural domain of the function x +1
_____ 1 33. Find the range of the function
f(x) = ÷ + ______
​ 1  – x ​  ​  _____
    ​
​ 1  + x ​ 
÷ x2 + x + _1
16. Find the domain of the function f(x) = ​  _________
     ​
x4 + x2 + 1 
_____ _____
f(x) = ÷ + ​÷5  – x  ​ 
​ x  – 2 ​  x2 – x + 1
34. Find the range of the function f(x) = ​ _________
     ​
x2 + x + 1
17. Find the domain of the function
_____ ______ 35. Find the range of the function
f(x) = log2 ​( ​÷x  – 2 ​ + ​÷4  – x  ​ ) 
​ _____ _____
f(x) = log2 (​÷x  – 2   ​ 4  – x   
 ​+ ÷ ​)
18. Find the domain of the function
1
log(3 – x) 36. Find the range of the function f(x) = x2 + _____
​  2    
 ​
f(x) = ​ ___________
2
  
   ​. x +1
x   – 3x + 2 
_____ _____ 37. Find the range of the function
19. Find the domian of f(x) = ÷   – 1 ​
​ |x| ​ 3  – |x| ​
 + ÷  
1
_____ f(x) = x4 + 4x2 + ___________
​  4      ​ + 10
÷   – 1 ​
​ |x|   x + 4x2 + 9
20. Find the domain of f(x) = ​ _______
_____      ​
​ 4  – |x| ​
÷  
38. Find the range of the function
21. Find the domain of
Real Function  1.29

f(x) = log2 ​( log1/2 (x2 + 4x + 4) )​


39. Find the range of the function
( 
2x(sin17 x + tan99 x)
f(x) = ​ ​  ________________
[ 
x + 21p
2​ ______
​  p    ]
  
  
​  ​ – 41 )
 ​  ​, [x] £ x

cos2 x + 1
f(x) = sin ​ ​ ________
2
–1
 ​  
 ​(  ) 55. Find the inverse of the functions
_______
__
40 Find the range of the function ​ x  + ​÷x 
f (x) = ÷ where f : [0, •) Æ [0, •)
  ​ ​ , 
f(x) = 2x + 3x + 4x + 2– x + 3 – x + 4 – x + 10
56. Find the inverse of the functions
41. Find the range of the function

42.
f(x) = 3 tan2 x + 12 cot2 x + 5
Find the range of the function
1
f(x) = 4x3 – 3x, where f : ​ __ [  ]
​   ​ , 1  ​ Æ [– 1, 1]
2
__ ________________________
f(x) = ÷     sin x + cos x + 4
​ 3 ​ ​ a  – x3 + 3x2    
57. Let f(x) = 3÷ – 3bx + b3 + b  ​.
43. Find the range of the function
1 Find b if f(x) is the inverse of itself.
f(x) = _______________________
​          ​ ax + 1
2 cos2 x + 4 sin x cos x + 4   58. Find the value of a so that f(x) = ​ ______ ​ is identical

(  )
x+3
x2 with f  –1(x)
44. Find the range of the function f(x) = cos–1​ ​ _____  
 ​  ​
1 + x2
59. Determine the nature of the function f (x), where f (x)
45. Find the range of the function satisfying the relation f(x + y) + f (x – y) = 2f (x).

( 
_______

)
f(y)
  2 + 1  ​
÷​ 2x
f(x) = cos  ​ ​  ________
–1  
 ​  
 ​ 60. If f is a polynomial function satisfying 2 + f(x). f(y)
x2 + 1 = f(x) + f(y) + f(xy) for all x, y in R and if f(2) = 5,
then find f(f (2))
46. Find the range of the function 61. If f (x) is symmetrical about x = a and x = b, then

[  ] [  ]
1 1 find the period of f (x).
f(x) = sin–1 ​ x2 + ​ __ ​   ​ + cos–1 ​ x2 – ​ __ ​   ​ where [,] =
2 2 62. Find the number of integral values of a for which the
G.I.F
function f (x) = log(log1/3 (log7(sin x + a))) be defined
47. Find the range of the function for every real value of x.
x2 + 2x + 3
f(x) = ​  _________
x     , 
​x>0 63. Find the domain of the function

48. Find the range of the function


2
(  (  1
f(x) = log2​ – log1/2​ 1 + ​ ___
​x1/4 ) )
   ​  ​ – 1  ​
​ ​
x +x+2 64. Find the domain of the function
f(x) = ​ _________  ​ where x Œ R

x2 + x + 1 f(x) = log2 (log3 (log4 (tan–1x)2))
x
e
49. Find the range of the function f(x) = ​ ______
    ​ 65. Find the domain of the function
1 + [x]
{x}
50. Find the range of the function f(x) = _______
​ 
1 + {x}
  
 ​, where
1
( 
f(x) = sin–1​ ______
|x – 1| ) 1
 ​  ​ + _________________
​  2     ​  ________________
     ​
  x + sin x  
÷​ sin 2
+ 1   ​
{,} = F.PF 66. Find the domain of the function
51. Find the range of the function 1
f(x) = __________________
​  ________________
      ​
p
f(x) = 2 cos x + sec2 x, x Œ ​ 0, ​ __ ​   ​
2 (  )   – |x – 10x  
÷​ 4x 2
+ 9| ​

52. Find the range of the function 67. Find the domain of the function f(x)
p
f(x) = 3 tan x + cot3 x, x ​ 0, __
​   ​   ​
2 (  ) _______________________
  –1 |sin x| –   
= ​÷sin cos–1 |cos x|  ​ for all x [0, 2p]
53. Determine the nature of the function
2
68. Find the domain of the function
sin4 xe ​x​ ​

÷  ( 
____________
f(x) = ​ __________
)
  
   ​, where [,] = G.I.F
[ 
x + 3p
_____
​ ​  p   
5
__
​  ​ – ​    ​
2 ] 3x – x2
f(x) = ​ log0.5​ ​ ______
   
x – 1 
 ​  ​ ​

54. Determine the nature of the function 69. Find the domain of the function
1.30  Differential Calculus Booster

(  )
f(x) = log7​ log5​( log3​( log2 (2x3 + 5x2 – 14x) )​ )​  ​ 86. If f(x) = ax + b and f ​( f​( f (x) )​ )​ = 27x + 26 where a,
b ΠR, find the value of a2 + b2 + 2
70. Find the domain of the function
x x
____________________ 87. If f(x) = _______
​  ______
   ​  , prove that f ​( f (f(x)) )​ = ________
​  ______
     ​
f(x) = ÷​ 3  x–1 + 5x–1    
+ 7x–1 – 83   ​ ÷​ 1  + x  
2
 ​ ​÷1  + 3x2 
 ​

71. Find the domain of the function 88. Find the domain of the function
___________ __________________________________
f(x) = ÷​ 3  – 2x –   
21–x    ​ f(x) = ÷   10 (log10x) –    
​ log log10 (4 – log103)  –  lo​g10
​ ​3 ​
72. Find the domain of the function
89. Find the domain of the function
log2015​( 1 – log7(x2 – 5x + 13) )​ ___________________
f(x) = ÷   1/2log2 [x   
​ log 2
+ 4x + 5], ​ [x] £ x  
73. Find the domain of the function
________ ______ 90. Let f(x) = 1 + x2. Find a function g(x) such that
f(x) ÷   – [x
​ 24 2

] ​    – 4  ​
​ |x|   f(g(x)) = 1 + x2 – 2x3 + x4.

(  )
1 + x2
74. Find the range of the function f(x) = sin–1 ​ ​ _____
2x
  
 ​  ​ p
(  ) ( p
91. Let f(x) = sin2 x + sin2 ​ x + ​ __ ​   ​ + cos x. cos​ x + __
3 )
​   ​   ​
3

(  )
x
75. Find the range of the function f(x) = cos–1​ ​ _____
2
   ​  ​
1 + x2
Ï2 x
Ô
and g(x) = Ì 1
: 0 £ x <1
then find g​( f (x) )​
ÔÓ x + 4 : 1 £ x < 2
(  )
x2 + 1
76. Find the range of the function f(x) = sin–1​ ​ _____
x2 + 2

 ​  ​
ax2 + 2x + 1
92. Let f(x) = ​ ___________
  
   ​. Find the values of a if

( ÷  )
______ 2x2 – 2x + 1
p2
77. Find the range of the function f(x) = tan​ ​ ​ __ ​  – x ​   

9 f  : R Æ [– 1, 2] is onto.

93. Find x, if ​| ​| |x2 – x + 4  –2 |​ – 3 |​ = x2 + x – 12.


( ÷  )
______
p 2
78. Find the range of the function f(x) = sin​ ​ ___
​   ​ – x  
​  ​ 94. Find the set of values of a for which the function
16
f : R Æ R is given by f(x) = x3 + (a + 2)x2 + 3ax
79. Find the range of the function f(x) = log2 (4x – x2) + 5 is one-one.

(  )
x2 + e
80. Find the range of the function f(x) = log​ ​ _____ 
x2 + 1
  ​  ​
95. Let g(x) = f(x) – 1. If f(x) + f(1 – x) = 2 for all x in R,
then find the line about which g(x) is symmetrical.
81. Find the range of the function 96. If f(2 + x) = f(2 – x) and f(7 – x) = f(7 + x) and

(  ( ÷  ) )
_____ f(0) = 0, find the minimum number of roots of
4 – x2 f(x) = 0, where 20 £ x £ 20.
f(x) = sin ​ log​ ​ ​ _____ 
  
​ ​  ​  ​
1–x
97. If a, b ΠR + and for all x in R, f(a + x) = b + [b3
82. Find the range of the function + 1 – 3b2 f(x) + 3b{f(x)}2 – {f(x)}3]1/3. Prove that
_________ f(x) is periodic.
f(x) = log​( sin–1(​÷x  2 + x + 1 ​ 
) )​
98. If 4x – 2x + 2 + 5 + ​| |b – 1|–3 |​ = |sin y|, " x, y Œ R,
find the possible value of b.
83. Find the range of the function

f(x) = sin​( log(5x2 – 8x + 4) )​ (Tougher Problems for JEE-Advanced)


84. Find the range of the function 1. Find the domain of the function
f(x) = cot ​( log0.5(x – 2x + 3) )​
–1 4 2
(  ( 
f(x) = log2​ log3​ log4/p (tan–1 x) –1  ​  ​ ))
2. Find the domain of the function
85. Find the range of the function
______________________________
______________________
​ 1  – cos x​÷1      
f(x) = ÷
_____________
– cos x​÷1     
____
– cos x​÷  
....• 
  ​  ​  ​  ​
1
f(x) = sin–1​ ______
( 
|x – 1| ) 1
 ​  ​ + ________________
​  2     ​  ______________
     ​
   x + sin x  
÷​ sin 2
+ 1  ​
Real Function  1.31

3. Find the domain of the function 18 Find the range of the function
1
f(x) = __________________ x2 – 3x + 2
​  ________________
       ​. f(x) = ​  ___________
  
    ​
  – |x – 10x  
÷​ 4x 2
+ 9|  ​ x2 + x – 6   
19. Find the range of the function
4. Find the domain of the function
2
_______________________ f(x) = (tan–1 x)2 + _______
​  ______
   
 ​
f(x) = ÷   –1 |sin x| –   
​ sin cos–1 (cos x) ​ in [0, 2p] ÷​ 1  + x2 
 ​
20. Find the range of the function
5. Find the domain of the function _________________ _______________
_______________ ​ a  2cos2 x + b2   
f(x) = ÷ ​ b  2 cos2 x + a2  
sin2 x ​ + ÷  sin2 x ​,
​ 
f(x) = ÷ 2{x}2 – 3 {x}  
+ 1 ​, x Œ [– 1, 1], b>a
where {,} = F.P.F 21. Find the range of the function
6. Find the domain of the function p
f(x) = log​( (cos x) cos x + 1 )​, x Œ ​ 0, __
​   ​   ​
2 (  )
÷  ( 
_____________
3x – x2
f(x) = ​ log0.3​ ​ ______ ) 22. Find the range of the function

( 

 ​  ​  ​. _______

)
  
x – 1    2 + 1  ​
÷​ 2x  
f(x) = cos ​ ​  ________
–1
2
 ​  
 ​.
7. Find the domain of the function x +1
f(x) = log10​( 1–log10 (x2 – 5x + 16) )​ 23. Find the range of the function

8. Find the domain of the function (  ​(16 sin2 x + 1)  ​


f(x) = log2​ 2 – lo​g​÷​ __2 ​
   )
___________
​÷   0.3(x   
– log – 1) ​ 24. Find the range of the function
f(x) = _____________
​  ___________
  
    ​ f(x) = tan–1​( log5/4 (5x2 – 8x + 4) )​
  2 + 2x   
​÷– x + 8 ​

÷ 
_______
x–1 25. Find the range of the function
9. Find the domain of f(x) = ​ _______
​     ​ ​ 

x – 2{x} f(x) = 6x + 3 – x + 6 – x + 3x + 2.

10. Find the domain of f(x) = log[x2](4 – |x|) 26. Find the Range of

(  )
_____
1 + x2
11. Find the domain of f(x) = cos–1​ ​ _____   ​ 2  – x2 
​  ​ + ÷
     ​
2x
27. Find the range of
[x2 – 3]
f(x) = cos–1​ ​ ______
5
 ​  (  )
 ​ + log2 (|x| – 1)
___________
1
f(x) = __
x+1
​ p ​ (tan–1 x + sin–1 x) + ​ ___________

    ​
x2 + 2x + 5  
​ 
12. Find the domain of f(x) = ÷ x2  
[x] – 1 +   28. Find the range of

[  ] [  ]
_____________
1 1
 ​ 13. Find the domain of f(x) =   x – (cos
÷​ sin   x) ​, f(x) = sin–1​ x 2 + ​ __  ​  ​ + cos–1 ​ x 2 + ​ __  ​  ​, where [,]
2 2
where (x) ≥ x. = G.I.F

{  ÷ 
______
14. Find the domain of definition of the function
1
f(x) = ___
1
​    ​ + log(2{x}–5) (x2 – 3x + 10) + ​ _______
   
______  ​
p2
29. Find the range of f(x) = log​ cos ​ ​ __ ​  – x2 
9
 ​  ​ }
[x] ​ 1  – |x|  ​
÷   30. Find the range of f(x) = [1 + sin x] + [cos x – 1]
15. Find the domain of definition of the function + [tan–1 x] where [,] = G.I.F.
________
f(x) = ÷  
​ sin(cos + log(– 2 cos2 x + 3 cos x – 1)
x) ​  31. Find the range of the function

{  }
_____
( 
3p
+ ​e​
2 sin x + 1
cos –1​ ​ ________
  )
____  ​  
​ ​
2 ​÷sin x 

​ f(x) = log10​ sin–1(​÷x  – 5  ​) + ___
​   ​   ​
2
16 Find the domain of the definition of the function 32. Find the range of the function
_________________
f(x) =  
÷​ 1 – log
x
x log2 (4  – 12) ​
   f(x) = loge (2 sin x + tan x – 3x + 1),

17. Find the domain of the definition of the function p p


where ​ __ ​  £ x £ __
​   ​ 
___________________ 6 3
​ 
f(x) = ÷ 2
log1/2log2 [x   
+ 4x + 5]  ​ 33. Find the range of the function
1.32  Differential Calculus Booster
__
​ ​{ ​÷2 ​
f(x) = lo​g​​ __5 ​
  
  (  sin x – cos x) + 3 }​ 48. Find the solution of the equation
÷

÷(   )
__________ ______
34. Solve for x : ​÷3  – 4cos  
2
x ​ > 2 sin x + 1 2 1
x – x + 1 = __
3
​   ​  + ​ ​ x – __
​   ​    
​ ​
2 4
1
35. Let m = ​ __ [  ] ​[ __​ 14 ​  + ____
​   ​   ​ +
4
​     ​  ]​ + ​[ __
1
200 4 200 ]
1 2
​   ​  + ____
​     ​  ​ + .... + 49. Let f(x) = sin2016 x – x – cos2016 x and, g(x) = sin x + cos x,

[  (  ) ]
p
then find the general solution of f(x) = ​ g​ ___
​    ​  ​  ​where
1 199
​ __ [ 
​   ​  + ____
4 200
​    ]
 ​  .​ Find the value of m + 50. [,] = G.I.F.
10

50. Find the maximum and minimum values of


36. Solve for x : {x} + sin{x} = 2, where {,} = F.I.F
f(x, y) = 7x2 + 4xy + 3y2, where subject to the condition
37. Solve for x : x2 – 4x + [x] + 3 =0 x2 + y2 = 1.

38. Let F(x) be a function defined by F(x) = x – [x], x Integer Type Questions
ΠR Р{0}, [,] = G.I.F, find the number of solutions 1. If the fundamental period of the function
1
of F(x) + F​ __ (  )
​ x ​  ​ = 1
x–p
(  )
f( ) = 4 cos4 ​ ​ _____
4p 2 (  )
x–p
 ​  ​ – 2 cos ​ _____
​  2 ​  ​
2p 
39. Find the number of values of x satisfying n
is m(p) , where m and n are positive integers, then
{x2} + [x4] = 1 (m + n) is
2. If the domain of the function
40. Find the least value of the function _____ _____
f(x) = |x – a| + |x – b| + |x – c| + |x – d| where a < b < f (x) = log2 ​( ​÷x  – 2 ​ + ÷ ​ )​ is [a, b]
​ 4  – x  
c < d and x takes any arbitrary real number. then find the value of (2b – 3a) is

( 
3. If the natural domain of the function
)
89
1

÷  (  )
41. Find the value of ​S   ​ ​   ​​ __________
​       ​  ​ __________
x–2
n = 1 1 + (tan no)2 f (x) = ​ log2 ​ ​ _____
    ​  ​ ​ is [a, b), then find the value of
3–x
42. Find the set of values of x satisfying the equality (2a – b) is

[  ] [  ]
3
​ ​ __
4
__
x ​  ​ + ​ ​ x ​  ​ = 5, where [,] = G..I.F belongs to the
4. If the equation |x – 2| – |x + 1| = p has exactly one
solution, then find the number of integral values
(  ]
interval ​ a, ​ ​ ​__
b
__
b
c ​  ​ ​,​ where a, b, c are natural number
and ​ c ​ is in its lowest forms. Find the value of a +
of p.
5. If the range of the function
b + c + abc + 30. x2 + x + a
f (x) =  ​_________
   
5 3
 ​, x Œ R is ​ __ [  ]
​   ​, __
​   ​ ​
(  ) (  )
1 x 1 x – 1
2
x + 2x + a 6 2
43. If the equation ​​ __
​   ​   ​​ ​ + ​​ __
(  )
​   ​   ​​ ​ + b = 0 has a positive
4 2 a2 + a
then find the value of ​ ​ ______
   
​​
  
solution, find the value of b. 5
6. The number of integral values of x satisfying the

( [  ] )
44. If f(x) = px + q and f​( f ( f(x)) )​ = 8x + 21 where p
15
and q are real numbers, find the value of p2 + q2 equation sgn ​ ​ _____ ​  ​ = ​[ 1 + {2x} ]​ is
​  2  ​  
+ p + q. x +1
7. Find the number of integral values_____of x for which
45. Find the set of values of p for which the function 3
_____
f(x) = x3 – 2x2 – px + 1 is one-one. ​ = ​÷x  – 3 ​
satisfying the equation ​÷x  + 1    
8. Find the number of solutions of
ax2 + 6a – 8 .
46. A function f : R Æ R is given by f(x) = ​ ___________
  
    ​ sgn(x + 1) = 2x2 – x
a + 6x – 8x2
9. Find the number of integers in the do main of the
Find the values of ‘a’ for which the function ‘f ’ is 1
onto. function f (x) = ___________
​  _______   
___
÷​    ​÷81 ​ ​
x–1
     –3
2(ex – e–x)(sin x + tan x)
47. Prove that f(x) = ​  ____________________
       ​ is an odd  ​ 10. If the range of the function
x + 2p
2​ ______
​  p    [ 
​  ​ – 3 ] x2
f (x) =  ​_____
4 [  ]
 ​ is ​ a, __
  
b
​ c ​ ​, where a, b, c Œ I
x +1
function, where [,] = G.I.F
Real Function  1.33

( 
x 2008
then find the value of (a + b + c + 2)
11. If the range of the function

4
(i) If f(x) = ​ _____
4x + 2 r = 1
r
​  ​ ​ ​f ​ _____
   ​  where x Œ Q, then S ​     
2009
 ​  ​ is )
equal to
ex – 1
f (x) = ​ _____  
 ​ is (a, b), then find the value of
ex + 1 (a) 1004 (b) 2006
(a + b + 2) 2 2 (c) 2007 (d) None
3x–3
12. If f (x) = ax + b and f ( f (x)) = 8x + 21 where a, b Œ R, (ii) If f(x) = _______  ​, x Œ Q, then the value of
​  1–x  x 
3 +3
then find the value of (a + b + 3).
(  )
54
r
​S   ​ ​ ​ f ​ ​ ___  ​  ​ is
Comprehensive Link Passages r  = 1 55

Passage I (a) 54 (b) 27


If the function f : R Æ R and g : R Æ R be defined as (c) 1 (d) 55
f(x) = ex and g(x) = 3x – 2. On the basis of the above ax
(iii) If f(x) = ​ _______
x  __   ​ (a > 0), then the value of
information answers the following questions. a +÷ ​ a 
   ​

(  )
(i) The function fog is 2n–1
r
​S ​ ​ ​ 2f ​ __
​  n   ​  ​
(a) e3x–2 (b) e3x+2 r = 1 2
(c) 3ex – 2 (d) 3ex + 2
(ii) The function goo is (a) 1 (b) 2n
a
(a) e3x – 2 (b) e3x+2 (c) 2n–1 (d) (2n – 1) ​ __ ​ .
x
2
(c) 3e – 2 (d) 3ex + 2
Passage III

[  ]
(iii) Domain of (fog) –1 (x) is
1
(a) (– •, •) (b) (– •, 0) Let [,] = G.I.F, then ​ x + __
​   ​   ​ = [2x] – [x] for every x Œ R.
2
(c) (0, •) (d) [1, 3] 1
Any real x can be taken as k + f or k + ​ __ ​  + f , where k
(iv) Domain of (goo) –1 (x) is 2
1
is an integer and. 0 £ f < __​   ​ 
(a) (– •, 2] (b) [– 2, 2] 2
(c) (– •, •) (d) (– 2, •) If x = k + f, then [x] = k, ​ x + ​ __   ​  ​ = k [ 
1
2 ]
Passage II and [2x] = [2k + 2f] = 2k.

[  ]
Let f(x) be a function satisfying the functional equation
1
f(x) + f(1 – x) = k, for every x Œ Q, where k is a constant Thus ​ x + __
​   ​   ​ = [2x] – [x].
2
quantity.
Let m be a positive integer.
r
Again, if x = k + __
1
2 [  ]
1
​   ​  + f, then [x] = k, ​ x + __
​   ​   ​ = k + 1
2
Put x = _____
​      ​ in the given equation we get
m+1 and [2x] = [2k + 2 f + 1] = 2k + 1.
r
f ​ _____
​     
m+1 (  m+1–r
​ 
m+1) ( 
 ​  ​ + f ​ _________
 ​ 
  ​= k ) [  ]
1
Thus, ​ x + __
​   ​   ​ = [2x] – [x].
2

( 
(i) If x < 2k, x and k being positive integers then the
(  ) [  ]
m m

r =1 m
r
fi ​S  ​ ​ ​ f ​ _____
​     
+ 1 )r =1
 m + 1 – r
 ​  ​ + ​S  ​ ​ ​f ​ __________
​ 
m+1
 ​  
   ​ = mk n + 2k
value of ​ ​ ______  ​  
 ​ is
2k + 1
(a) n (b) n + 1
(  ( 
m m
r
fi ​S  ​ ​ ​ f ​ _____
​     
m+1 )
r=1 m + 1
t
 ​  ​ + ​S  ​ ​ ​f ​ _____)
​      ​  ​ = mk (c) 0 (d) None

[  ] [  ]
r=1
1 1
(ii) For every real x, ​ x + __
​   ​   ​ + ​ x + __
​   ​   ​ + [x] is equal
2 2
where t = m + 1 – r. to
(a) [3x] (b) [2x] + 1
(  )
m
r
fi 2​S  ​ ​ ​ f ​ _____
​      ​  ​ = mk (c) [2x] + 3 (d) None
r = 1 m +1
(iii) For every positive integers n, the sum of
​S ​ ​ ​ f ​( _____
m

m + 1)
r mk

[  ]
 ​  ​ = ​ ___ ​ 
[  ] [  ] [ 
fi   ​     
r = 1 2 n+1
​ ​ _____
2
n+2
 ​ + ​ ​ _____
 ​  
22
 ​ 

  ​ + ​ ​ 
+ 22
n______
23
 ​  
n + 2k
 ​ + ... + ​ ​ ______
2k + 1
 ​  
 ​ is
]
1.34  Differential Calculus Booster

(a) 0 (b) n Matrix Match


(c) 2k n (d) infinite. (For JEE-Advanced Examination only)
Passage IV 1. Match the statement in Column-I with the state-
Let two sets A and B consists of m and n elements respec- ment in Column-II
tively. Then the number of onto functions between two sets
Column I Column II
A and B is the number of distributions of ‘m’ balls into ‘n’

(  ]
boxes, where no box is remain empty. (A) Range of p
(P) ​ 0,  ​ ​__ ​   ​ ​
(i) The number of onto functions between two sets f(x) = cos–1(|1 – x2|) ​4 ​
A = {1, 2, 3, 4} and B = {5, 6} is (B) Range of


(a) 16
(c) 20
(b) 14
(d) 18
f(x) = cot–1
(x2– 4x + 5)
(Q) [  ] p
​ 0, __
​   ​   ​
2

(ii) The number of onto functions between two sets


A = {a, b, c, d} and {1, 2, 3, 4, 5} is
(a) 0 (b) 10
(C) Range of
f(x) = log |log x| (  ) (  )
1
(R) ​ 0, __
2
1
​   ​   ​ » ​ __
​   ​ , 1  ​
2

(c) 20 (d) 625 (D) Range of f(x)


​  log  3  
  ​
(iii) The number of onto functions between two sets (S) (0, 1) » (1, •)
= ___________
2x
A = {1, 2, 3, 4, 5} and B = {5, 10, 15} is cos–1 (2x – 1)
(a) 100 (b) 150
(c) 200 (d) 125 2. Match the statement in Column-I with the state-
Passage V ment in Column-II
Let two sets A and B consists of m and n elements respec- Column I Column II
tively. Then the number of one-one function between two
(A) Period of (P) 1
ÔÏ n P : n > m 1
sets is = Ì m f(x) = __
​   ​  {sin (3x) +
ÔÓ 0 : n < m 3
|sin 3x| + [sin 3x]}
(i) The number of one-one functions between two sets
A = {1, 2, 3} and B = {3, 4} is (B) Period of f(x) = cos(tan x + (Q) 2p
cot x) + cos(tan x – cot x) ​ ___ ​ 
(a) 0 (b) 3 3
cos 4 (px) + x – [x]
(c) 2 (d) 1 (C) Period of f(x) = e (R) 10p
(ii) The number of one-one functions between two sets + cos2 (px)
A = {2, 4, 6} and B = {4, 6, 8} is
(D) Period of (S)

|  | | 
(a) 4 (b) 6 p
(c) 8 (d) 10 2
x
5
x
f(x) = ​ sin ​    ​  ​ + ​ cos __
3 __ 5
|
​    ​  ​
__
​   ​ 
2
(iii) The number of one-one function between two sets
A = {1, 2, 3} and B = {2, 3, 4, 5} is
(a) 4 (b) 5 3. Match the statement in Column-I with the state-
ment in Column-II
(c) 6 (d) 8
(iv) The number of one-one functions between tswo sets Column I Column II
A = {1, 2, 3} and B = {4, 5, 6} such that f(1) = 4 (Functions) (Ranges)

[  )
is 1
(A) f (x) = sin(px) (P) ​ 0,  ​ ​__ ​   ​ ​
(a) 3 (b) 2 ​2 ​
(c) 1 (d) 6. x – [x]
(v) The number of one-one functions between two sets (B) f(x) = ________
​    ​  (Q) [1, •)
1 – [x] + x
A = {1, 2, 3} and B = {4, 5, 6} such that f(1) π 4

[  )
is ex
______ p
(C) f(x) = ​ 1 + [x]
   ​  (R) ​ ​ __
​   ​  ​, p  ​
(a) 3 (b) 2 ​ 4​
(c) 4 (d) 6 (D) f(x) =cot–1(2x – x2) (S) [–1, 1]
Real Function  1.35
p
4. Match the statement in Column-I with the state- f(x) = sin2n+2 x + cos2n+2 x is __ ​   ​ , n Œ N
2
ment in Column-II
Reason (R):  If f(x) and g(x) be two periodic func-
Column I Column II tions with periods T1, T2 respectively, as well as both
are even, co-functions and comparable functions, then
(A) f(x) = 2x + (1 – x) sgn(x) (P) one-one into
1
the period of f(x) + g(x) is __
​   ​  × L.C.M of {T1, T2}.
(B) f(x) = 2x + (x – 1) sgn(x) (Q) Many-one onto 2
7. Assetion (A):  The range of the function

[  [ 
3
(C) f(x) = x  – x + 1 (R) Onto but not
one-one.
1
f(x) = sin–1​ x2 + __ ] 1
​    ​  ​ + cos–1 ​ x2 – __
2 ]
​   ​   ​ is p
2
Reason (R):  The range of the above function is a
2x + 1 ​ 
(D) f(x) = ​ _____  (S) neither one-one singletone set.
3x + 4 nor onto.
8. Assetion (A): A function f: R Æ R is defined as
f(x + y) = f(x) + f(y), then it is an odd function.
Assertion (A) and Reason (R) Reason (R): A function f(x) = 0 is the only one
function, which is even as well as odd function.
(A) Both A and R are true and R is the correct explana-
tion of A.
(B) Both A and R are true but R is not the correct

1
9. Assetion (A): Let f(x) = ​ ​ __ | |  | |
​ x ​  ​ – 2  ​ Then the number
of solutions of f(x) = 1 is 4.
explanation of A.
Reason (R): Inverse of an even function is
(C) A is true and R is false.
defined.
(D) A is false and R is true.
10. Assetion (A): Every periodic function is an even
1. Assertion (A):  The number of one-one function function.
between two sets A = {a, b, c} and B = {1, 2, 3}
is 6.
Reason (R): The domain of f(x) = sin–1​ log3​ __
{1, 9]
x
[  (  ) ]
​    ​  ​  ​ is
3
Reason (R):  The number of onto function between
two sets A = {1, 2, 3, 4} and B = {5, 6} is 14.
Questions asked in Previous Years’
2. Aseertion (A):  The range of the function JEE Main Exams (2002 to 2014)

f(x) = 2x + 3x + 4x + 2–x + 3 –x + 4 –x + 10 is 16 1. Which one is not periodic?


__
(a) |sin 3x| + sin2 x (b) cos ÷ ​ x  2
  ​  + cos  x
Reason (R):  For any two positive numbers a, b, 2
(c) cos 4x + tan x (d) cos 2x + sin x
a+b ___
​ _____  ≥ ​÷ab 
 ​     ​ [JEE-Main, 2002]

|  |
2

x 2
x
3. Assertion (A):  The solution set of ​ ​ ____
  
x – 1
 ​  ​ + |x| (  (  ) )
x
2. The domain of sin–1​ log3​ __
​    ​  ​  ​ is
3
= ​ _____  ​ is (1, •) » {0}
  
|x – 1| (a) [1, 9] (b) [– 1, 9]
Reason (R):  If |f(x) + g(x)| = |f(x)| + |g(x)|, then (c) [– 9, 1] (d) [– 9, – 1]
f(x) g(x) ≥ 0 [JEE-Main, 2002]
4. Assertion (A):  The number of solutions of 3. Domain of definition of the function
5
|x| + |x2–1| = __
​   ​  is 6 3
6 f(x) = _____  ​ + log10 (x3 – x) is
​    2 
4–x
Reason (R):  The number of solutions of
(a) (– 1, 0) » (1, 2)
sin x = x2 + x + 1 is 0
(b) (1, 2) » (2, •)
5. Assetion (A):  The domain of the function
_____ _____ (c) (– 1, 0) » (1, 2) » (2, •)
​ x  – 2  ​ + ​÷4  – x  ​
f(x) = ÷  is [2, 4]
(d) (1, 2) [JEE-Main, 2003]
Reason__ (R):  The range of the function f(x) is 4. If R Æ R satisfies f(x + y) = f(x) + f(y) for all x, y
Î​÷2 ​
   , 2˚
n
6. Assetion (A):  The period of the function ​   ​ ​ ​ f(r) is
in R such that f(1) = 7, then S
r =1  
1.36  Differential Calculus Booster

7(n + 1) 12. The largest interval lying in


(a) ​ _______
 ​  
  (b) 7n(n + 1)
2


7n(n + 1)
(c) ​ ________
 ​   
7n
(d) ​ ___ ​  is
2
x
f(x) = 4 –x + cos–1 ​ __
2
(  )
​    ​ – 1  ​ + log(cos x) is defined
2 2
[JEE-Main, 2003]

5. The range of the function f(x) = 7–x


Px–3 is [ 
p p
4 ​2 ​ )
(a) ​ – ​ __ ​ , ​ ​ __ ​   ​ ​ [  )p
(b) ​ 0,​  __
​   ​   ​ ​
​2 ​
(a) {1, 2, 3, 4} (b) {1, 2, 3, 4, 5, 6} (c) [0, p] ( 
p p
(d) ​ – ​ __ ​ , __
2 2 )
​   ​   ​
(c) {1, 2, 3} (d) {1, 2, 3, 4,5}
[JEE-Main, 2007]
[JEE-Main, 2004]
13. Let f : N Æ Y be a function defined as f(x) = 4x + 3,
sin–1(x – 3) where Y = {y Œ N: y = 4x + 3 for some x Œ N} and
6. The domain of the function f(x) = ​ _________
_____  ​  

is f is invertible, then f  –1(x) is
÷​ 9  – x2 
 ​
x– 3  3x + 4
(a) ​ _____
 ​  
  (b) ​ ______
 ​ 

(a) [1, 2] (b) [2, 3) 4 3
(c) [2, 3] (d) [1, 2)  x + 3  x + 3
(c) 4 + ​ _____
 ​  
  (d) ​ _____
 ​ 

[JEE-Main, 2004] 4 4
__ [JEE-Main, 2008]
7. If R Æ S defined by f(x) = sin x – ÷    cos x + 1 is
​ 3 ​ 
onto, then S is 14. Let f(x) = (x + 1)2 – 1, x ≥ – 1
(a) [0, 1] (b) [– 1, 1] Statement-I: The set {x: f(x) = f  –1 (x)} = {0, 1}
(c) [0, 3] (d) [– 1, 3] Statement-II: f is a bijection
[JEE-Main, 2009]
[JEE-Main, 2004]
15. No question asked in 2010.
8. The graph of the function is symmetrical about the 1
line x = 2, then 16. The domain of the definition f(x) = ______
​  _____
    ​ is
÷   – x  
​ |x| ​
(a) f(x) = f(–x) (b) f(2 + x) = f(2 – x)
(a) (– •, 0) (b) (– •, •) – {0}
(c) f(x + 2) = f(x – 2) (d) f(x) = – f(– x)
[JEE-Main, 2004] (c) (– •, •) (d) (0, •)
9. Let f : (– 1, 1) Æ B be a function is defined by [JEE-Main, 2011]
17. No question asked in 2012.
f(x) = tan ​ ​ _____
–1
(  ) 2x
  ​   
1 – x2
​, then f is one-one and onto, when 18. Let A and B be two sets containing 2 elements and
4 elements respectively. The number of subsets of
B is the interval A × B having 3 or more elements is

[  ) p
(a) ​ 0,  ​ ​__ ​   ​ ​
​ ​2 (  )
(b) ​ 0, __
p
​   ​   ​
2


(1) 220
(3) 211
(2) 219
(4) 256
( 
p p
)
(c) ​ – ​ __ ​ , ​ __ ​   ​
2 2 [  ] p
(d) ​ – ​ __ ​ , __
2 2
p
​   ​   ​ [JEE-main, 2013]
19. If X = {4n Р3n Р1: n ΠN} and Y = {9(n Р1):
[JEE-Main, 2005]
n ΠN} where N is the set of natural numbers, then
10. A real valued function f(x) satisfies the functional X » Y is equal to
equation f(x – y) = f(x) f(y) – f(a – x) f(a + y) where
(1) N (2) Y – X
a is a given constant and f(0) = 1, then f(2a – x) is
(3) X (4) Y
(a) f(x) (b) – f(x)
[JEE-Main, 2014]
(c) f(– x) (d) f(a) + f (a – x)
[JEE-Main, 2005] 20. Let A and B be two sets containing four and two
elements respectively. Then the number of subsets
3x2 + 9x + 17 of the set A × B, each having at least three elements
11. If x is real, the max value of ​ ____________
  
    ​ is is
3x2 + 9x + 7 
(a) 1/4 (b) 41 (1) 256 (2) 275
(c) 1 (d) 17/7 (3) 510 (4) 219
[JEE-Main, 2006] [JEE-Main, 2015]
Real Function  1.37

(x – a)(x – b)
Question asked in Previous Years’ 11. For real x, the function f(x) =​  ____________
    ​ 
will
(x – c)
JEE-Advanced Examinations assume all real value provided

{ p p (a) a > b > c (b) a < b < c


1. Given A = ​ x: __
find f(A)
​   ​  £ x £ __
6 3}
​   ​   ​and f(x) = cos x – x(x + 1),
(c) a > c < b (d) a £ c £ b
[IIT-JEE, 1980] [IIT-JEE, 1984]

( 
______

)
2. Let f be a function with domain {x, y, z} and range
{1, 2, 3}. It is given that exactly one of the following ÷​ 4  – x2 ​ 
12. If f(x) = sin ​ ln ​ ______    ​  ​, then the domain of f(x) is
statement is true and the remaining two are false: 1–x
f (x) = 1, f(y) π 1, f(z) π 2 Determine f  –1(1) .......... [IIT-JEE, 1985]

[IIT-J22, 1982] 13. No questions asked in 1986.

( ÷ 
_______ 14. Let f(x) be a function satisfying the condition

p 2
3. The values of f(x) = 3 sin ​ ​ ​ ___ ​  – x2 
16 )
 ​  ​ lies in the f(– x) = f(x) for all x. If f(0) exists, then find its
value
interval .......... [IIT-JEE, 1983] [IIT-JEE, 1987]

4. If f(x) = (a – xn)1/n, where a > 0 and n is a positive 15. If f1(x) and f 2 (x) are defined on domains D1 and
integer, then f(f(x)) = x Is it true? [IIT-JEE, 1983] D 2 respectively, then ( f1(x) + f 2 (x)) is defined on
D1 » D2. Is it true? [IIT-JEE, 1988]
x2 + 4x + 30
5. The fucntion f(x) = ​ ___________
  
    ​ is not one to one. 16. There are exactly two distinct linear functions .........
x2 – 8x + 18 and .......... which map [–1, 1] onto [0, 2].
Is it true? [IIT-JEE, 1983]
[IIT-JEE, 1989

(  )
6. If f(x) = |x – 1|. Then 1 px
17. If the function f(x) = __
​   ​  – tan​ ​ ___ ​   ​, – 1 < x < 1 and
(a) f(x2) = (f(x))2 (b) f(x + y) = f(x) + f(y) 2 2
___________
(c) f(|x|) = |f(x)| (d) None of these. g(x) = ÷​ 3  + 4x –  
4x2 ​, then the domain of (go f ) is
[IIT-JEE, 1983]
1
__
[  (  )
x
__
]
7. If f(x) = cos (log(x)), then f(x) f(y) – ​   ​  ​ f ​ ​ y ​  ​ + f(xy)  ​
(a) (– 1, 1) [  ] 1 1
(b) ​ – ​ __ ​ , __
2 2
​   ​   ​

[  ] [  ]
2 1 1
has the value (c) ​ – 1, ​ __ ​   ​ (d) ​ __
​   ​ , 1  ​
(a) – 1 (b) 1/2 2 2
(c) – 2 (d) None. [IIT-JEE, 1990]
[IIT-JEE, 1983] 18. No questions asked in 1991.
8. The domain of the function
_____ 19. The values of b and c for which the identity f(x + 1)
1
f(x) = __________
​  ​ x  + 2 ​
    ​ + ÷  is – f(x) = 8x + 3 is satisfied, where f(x) = bx2 + cx + d,
log10 (1 – x)
are
(a) (– 3, – 2) – {– 2.5} (b) (0, 1) – {0. 5} (a) b = 2, c = 1 (b) b = 4, c = – 1
(c) (– 2, 1) – {0} (d) None of these. (c) b = – 1, c = 4 (d) None
[IIT-JEE, 1983] [IIT-JEE, 1992]
9. Which of the following functions are periodic? 20. The value of the parameter a, for which the function
(a) f(x) = x – [x], [.] = G.I.F f(x) = 1 + a x, a π 0 is the inverse of itself, if
(a) – 2 (b) – 1
Ï Ê 1ˆ
sin : x π 0 (c) 1 (d) 2
(b) f(x) = ÔÌ ÁË x ˜¯
Ô0 [IIT-JEE, 1992]
Ó : x=0
21. No questions asked in 1993.
(c) f(x) = x cos x 22. Let f(x) = sin x and g(x) = n |x|. If the ranges of the
(d) None of these. [IIT-JEE, 1983] composite functions (fog) and (gof) are R1 and R2

(  (  ) )
10. The domain of the function f(x) = sin–1​ log2​ ​ __ ​   ​  ​
x
2
2


respectively then
(a) R1 = {u: – 1 < u < 1}, R2 = {v:– • < v < 0}
is ..... [IIT-JEE, 1984] (b) R1 = {u: – • < u £ 1}, R2 = {v: – 1 £ v £ 0}
1.38  Differential Calculus Booster
1
(c) R1 = {u: – 1 < u < 1}, R2 = {v: – • < v < 0} (c) ​ __ ​  (1 + 4 log2 x) (d) Not defined
2
(d) R1 = {u: – 1 £ u £ 1}, R2 = {v: – • < v £ 0}
[IIT-JEE, 1999]
[IIT-JEE, 1994]
2 31. The domain of definition of the function y(x) is given
23. Let f(x) = (x + 1) – 1, x ≥ – 1. Then the set
by the equation 2x + 2y = 2 is
{x : f(x) = f  –1(x)} is (a) 0 < x £ 1 (b) 0 £ x £ 1

{ 
__ __

2
    – 3
– 3 + i​÷3 ​
(a) ​ 0, – 1, ​ ________
 ​  , ​ _______

2
  
– i​÷3 ​
 ​    ​ } (c) – • < x £ 0 (d) – • < x < 1
[IIT-JEE, 2000]
(b) {–1, 0, 1} 32. The domain of definition of the function
(c) {–1, 0} log2 (x + 3)
f(x) = ​ ___________
  
    ​ is
(d) Empty [IIT-JEE, 1995] x2 + 3x + 2  
24. If f is an even function defined on the interval (– 5, 5), (a) R – {– 1, – 2} (b) (– 2, •)
then the real values of x satisfying the equation (c) R – {– 1, – 2, – 3} (d) (– 3, •) – {–1, – 2}

(  )
x + 1 [IIT-JEE, 2001]
f(x) = f ​ ____ ​   ​  ​ are ..... [IIT-JEE, 1996]
x + 2 Ï- 1, x < 0
(  p
) p
25. If f(x) = sin2 x + sin2​ x + ​ __ ​   ​ + cos x cos ​ x + __
3 ( 
​   ​   ​ and
3 ) Ô
33. Let g(x) = 1 – x + [x] and f(x) = Ì 0, x = 0 ,

(  )
5 Ô 1, x > 0
g​ ​ __ ​   ​, = 1, then (go f)(x) is then for all x is Ó
4
(a) – 2 (b) – 1 (a) x (b) 1
(c) 2 (d) 1 (c) f(x) (d) g(x)
[IIT-JEE, 1996] [IIT-JEE, 2001]
1
26. A function f : R Æ R, where R is the set of real- 34. If f : [1, •) Æ [2, •) is given by f(x) = x + ​ __
x ​, then
ax2 + 5x – 8
numbers, is defined by f(x) = ​  ____________
  
   ​. Find –1
f   (x) equals
a + 5x – 8x2 _____ _____
the interval of values of a for which is onto. Is the x + ​÷x  2– 4 ​  x + ​÷x  2 – 4 ​

(a) ​ ________ ​    (b) ​ _________ ​   
function one to one for a = 3? Justify your answer. 2_____ 4
_____
[IIT-JEE, 1996] x–÷​ x  2 – 4 ​ 
(c) ​ _________
 ​
     (d) 1 + ​÷x  2 – 4 ​ 
27. No questions asked in 1997. 2
__ [IIT-JEE, 2001]
2
28. If g(f(x)) = |sin(x)| and f(g(x)) = (     ​) )​​ ​ then
​​ sin(​÷x  2 2
35. Let f(x) = (1 + b )x + 2bx + 1 and let m(b) be the
__ minimum value of f(x). As b varies, the range of
(a) f(x) = sin2 x, g(x) = ÷
​ x 
  ​ 
m(b) is

[  ]
(b) f(x) = sin x, g(x) = |x|
1
(c) f(x) = x2, g(x) = sin (​÷x 
__
  ​)  (a) [0, 1] (b) ​ 0, ​ __  ​  ​
2
(d) f and g can not be determined.

29. If f(x) = 3x – 5, then f   (x) is –1


[IIT-JEE, 1998] [  ]
1
(c) ​ __
​   ​ , 1  ​
2
(d) (0, 1]
[IIT-JEE, 2001]
1
(a) ​ ______
   ​  36. Let e {1, 2, 3, 4} and F = {1, 2}. Then the number
3x – 5 of onto functions from E to F is
(  )5
(b) ​ x + __
​   ​   ​
3


(a) 14
(c) 12
(b) 16
(d) 8
(c) does not exist because f is not one-one
[IIT-JEE, 2001]
(d) does not exist because f is not onto. ax
37. Let f(x) = ​ _____
   ​, x π – 1. Then for what value of a
[IIT-JEE, 1999] x+1
30. If the function f : [1, •) Æ [1, •) is defined by is f(x(x))
__
= x? __
f(x) = 2x(x–1), then f  –1(x) is   
(a) ​÷2 ​   
(b) – ​÷2 ​

(  )
__________ (c) 1 (d) – 1
1 x(x–2) 1
(a) ​​ __
​   ​   ​​ ​ (b) ​ __ ​  (1 + ​÷1  + 4 log  
2 x  ​) [IIT-JEE, 2001]
2 2
Real Function  1.39

38. Suppose f(x) = (x + 1)2, x ≥ – 1. If g(x) is the inverse Then f – g is


whose graph is reflection of the graph of f(x) with (a) one-one and into
respect to the line y = x, then g(x) equals (b) neither one-one nor onto.
__ 1
  ​  – 1, x ≥ 0
(a) – ​÷x  (b) ​ _______     ​, x > – 1 (c) many one and onto
(x + 1)2 (d) one-one and onto.
_____ __
, x ≥ –1
(c) ​÷x  + 1 ​
  (d) ​÷ x  ​ – 1, x ≥ 0 [IIT-JEE, 2005]
[IIT-JEE, 2002] 45. X and Y are two sets and
f : X Æ Y If { f(c) = ; c à X, y à Y} and
39. Let function f : R Æ R be defined by f(x) = 2x + sin x
for all x in R. Then ‘f ’ is {f  –1(d) = x; d à Y, x à X} then the true statement
(a) one-to-one and onto is
(b) one-to-one but not onto. (a) f(f  –1(b)) = b (b) f  –1(f(a)) = a
(c) onto but not one-to-one. (c) f(f  –1(b)) = b, b à Y (d) f  –1(f(a)) = a, a à X
(d) neither one-to-one nor onto. [IIT-JEE, 2005]
[IIT-JEE, 2002]
x
_____ 46. No question asked in between 2006 to 2010.
40. If f : [0, •) Æ [0, •) and f(x) = ​      ​, then f is
x+1 47. Find the domain of the function
(a) one-to-one and onto


(b) one-to-one but not onto.
(c) onto but not one-to-one.
​ 
8.3x–1
f(x) = sin–1​ ________   
1 – 32(x–1)
 ​  ​ (  ) [IIT-JEE, 2011]

(d) neither one-to-one nor onto.


48. The function f : [0, 3] Æ [1, 29] is defined by
[IIT-JEE, 2003]
f(x) = 2x3 – 15x2 + 36x +1, is
x2 + x + 2
41. Range of the function f(x) = ​ __________    
 ​ for all x in (a) one-one and onto
R, is x2 + x + 1
(b) onto but not one-one
(a) (1, •) (  )
11
(b) ​ 1, ___
​   ​   ​
7
(c) one-one but not onto

(  ] (  )
(d) neither one-one not onto [IIT-JEE, 2012]
7 7
(c) ​ 1, ​ ​ __ ​   ​ ​ (d) ​ 1, ​ __ ​   ​
​3 ​ 5 2
49. Let f : (–1, 1) Æ R be such that for f(cos 4q) = ________ ​     ​ 
[IIT-JEE, 2003] 2 – sec2 q
42. The domain of definition of the function for q Œ ​ 0, __
p
(  ) (  )
p p
​   ​   ​ » ​ __
​   ​ , __
​   ​   ​. Then find the value of f ​

÷ 
____________ 4 4 2
p
f(x) = ​ sin–1(2x)    + __
​   ​   ​for real valued x is
6
( __​ 13 ​  )​ is / are
[  ]
1 1
(a) ​ – ​ __ ​ , __ [  __​ 12 ​  ]​
1
(b) ​ – ​ __ ​ ,
(a) ​( 1 – ​ __
÷​  2 ​ ​  )​ (  ÷  )
​   ​   ​ __ __
4 2 2 3 3
(b) ​ 1 + ​ __
​   ​ ​    ​
[  __​ 14 ​  ]​

(  )
– 1 1
(c) ​ ___
​   ​ , __ ​   ​   ​
1
(d) ​ – ​ __ ​ ,
2

(c) ​( 1 – ​ __ ( 
÷​  3 ​ ​  )​ ÷​  3 ​ ​  )​
2 9 4 __ __
2 2
[IIT-JEE, 2003]   (d) ​ 1 + ​ __  

43. If f(x) = sin x + cos x, g(x) = x2 – 1, then g(f(x)) is


[IIT-JEE, 2012]
invertible in the domain

[  ] [ 
50. No questions asked in 2013.

p
(a) ​ 0, ​ __ ​   ​
p p
(b) ​ – ​ __ ​ , __
​   ​   ​ ]
2 4 4

p p
( 
51. Let f :​ – ​ __ ​ , __ )
​   ​   ​ Æ R be given by
[  ]
p p
(c) ​ – ​ __ ​ , ​ __ ​   ​
2 2
(d) [0, p]
2 2
f(x) = (log(sec x + tan x))3 Then
[IIT-JEE, 2004] (a) f(x) is an odd function
(b) f (x) is a one-one function
Ï x , if x is rational
44. Let f(x) = Ì (c) f(x) is an onto function
Ó0 , if x is irrational
(d) f(x) is an even function.
Ï0 , if x is rational [IIT-JEE, 2014]
and g(x) = Ì
Ó x , if x is irrational 52. No questions asked in 2015, 2016.
1.40  Differential Calculus Booster

Answers

LEVEL II Matrix Match


1. (b) 2. (a) 3. (b) 4. (b) 5. (b) 1. (A) Æ (Q), (B) Æ (P), (C) Æ (S), (D) Æ (R)
6. (a) 7. (a) 8. (c) 9. (a) 10. (b) 2. (A) Æ (Q), (B) Æ (S), (C) Æ (P), (D) Æ (R)
11. (b) 12. (b,c) 13. (a) 14. (d) 15. (d) 3. (A) Æ (S), (B) Æ (P), (C) Æ (Q), (D) Æ (R)
16. (b) 17. (d) 18. (a) 19. (c) 20. (a) 4. (A) Æ (S), (B) Æ (R), (C) Æ (Q), (D) Æ (P).
21. (d) 22. (d) 23. (d) 24. (c) 25. (b)
Comprehensive Link Passages
26. (b) 27. (c) 28. (b) 29. (d) 30. (a)
31. (c) 32. (a) 33. (a) 34. (a,c) 35. (a,d) P-I : (i) (a), (ii) (c), (iii) (c), (iv) (d)
36. (a) 37. (a,b,c,d) 38. (b). 39. (c) P-II : (i) (a), (ii) (c), (iii) (c)
40. (c) 41. (c) 42. (b) 43. (b) 44. (a) P-III : (i) (c), (ii) (a), (iii) (b)
45. (c) 46. (b) 47. (b) 48. (c) 49. (b) P-IV : (i) (b), (ii) (a), (iii) (b)
50. (b) P-V : (i) (a), (ii) (b), (iii) (a) (iv) (b), (v) (c)

Integer type questions Assertion and Reason


1. (5) 2. (2) 3. (2) 4. (5) 5. (4) 1. (B) 2. (A) 3. (A) 4. (B) 5. (B)
6. (7) 7. (1) 8. (2) 9. (2) 10. (5) 6. (A) 7. (B) 8. (B)
11. (4) 12. (8)

Hints and solutions

( 
1 + xy – x – y
= log ​ ​ _____________
     ​ ​
   )
(  )
1 + xy + x + y
1. Given f (x) = 3x4 – 5x2 + 9
x+y
Thus, f (x – 1) 1 – ​ ______ 
  ​
1 + xy
= 3(x – 1) 4 – 5 (x – 1)2 + 9 _________
= log ​ ​    ​  ​
x+y
= 3(x4 – 4x3 + 6x2 – 4x + 1) – 5(x2 – 2x + 1) + 9 1 + ______
​      

1 + xy
= 3x4 – 12x3 + 13x2 – 2x + 7
2. If f (x) = x + __
1 1
​ x ​, prove that ( f (x))3 = f (x3) + 3f  ​ __
1
​ x ​  ​ (  ) (  x+y
= f ​ ​ ______ 
1 + xy
  ​  ​ )
2. Given f (x) = x + __ ​ x ​ Hence, the result.
1
4. Given f (x) = x3 – __
We have f (x3) + 3f  ​ __ (  ) 1
​ x ​  ​
​  3  ​ 
x
= x3 + __
1
(  1
​  3  ​  + 3 ​ x + __
x
)
​ x ​  ​
1
Now, f (x) + f  ​ __
​ x ​  ​ (  )
( 
= ​​ x + __
13
)
​ x ​  ​​ ​ = x3 – __
x
1
​  3  ​  + __
1
​  3  ​  – x3
x
= ( f (x))3 =0
Hence, the result. Hence, the result.
2x
3. We have f (x) + f (y) 5. Given f (x) = ​ _____
    ​

(  )
1 + x2
(  )
1–x
= log ​ ​ _____ 
1+x
1–y
 ​  ​ + log ​ ​ _____  
1 +y
​  ​ Now, f (tan q)

( 
1–x
= log ​ ​ _____ 
1+x
1–y
 ​ × ​ _____ 
1 +y )
 ​  ​
2 tan q
= ​ ________ 
1 + tan2 q
  ​

( 
1 – x – y + xy
= log ​ ​ _____________
    ​  ​
  
1 + x + y + xy )
Hence, the result.
= sin (2q)
Real Function  1.41


x
6. Given f (x) = log ​ _____
​     
x–1
 ​  ​ (  ) fi

(bx – a) y = ax – b
b x y – ay = ax – b
We have f (x + 1) + f (x) fi x (by – a) = ay – b

(  ( 
ay – b

x+1
= log ​ ​ _____
x    )
​  ​ + log ​ _____
​     
x–1
x
 ​  ​ ) fi x = ​ ______ 
by – a
 ​

( x + 1 _____
= log ​ ​ _____
x   
x
​ × ​     
x–1
 ​  ​ ) fi x = f (y)
Hence, the result.
= log ​( ​ _____ 
x – 1)
x+1 10. We have
 ​  ​

1
a f (x) + bf  ​ __ (  )
1
​ x ​  ​ = __
​ x ​ – 5 ...(i)
Hence, the result.
1

1+x
7. Given f (x) = log ​ ​ _____  
1–x (  )
​  ​
Replace x by ​ __
x ​, we get

(  )
1

( 
a f  ​ __
​ x ​  ​ + bf (x) = x – 5 ...(ii)
Now,
2x
f ​ ​ _____
    ​  ​
)

(  )
1 + x2 Multiplying (i) by a (ii) by b and subtracting, we
2x get
1 + ​ _____

  
1 + x3
= log ​ ​ _________ 
 ​
 ​  ​

1
(a2 – b2) f (x) =  ​ __ ( 
​ x ​ – 5  ​ – b (x – 5))
2x
1 – ​ _____
1+x
 2  ​
( 
1
a ​ __ )
​ x ​ – 5  ​ – b (x – 5)
_________________

( 
f (x) = ​    
 ​
  

x2 + 2x + 1
= log ​ ​ _________
    ​  ​
x2 – 2x + 1 )
9x
11. Given f (x) = _____
​  x    
(a2 – b2)

{ (  ) }
x+1 2 9 +3
= log ​ ​​ ​ _____ 
 ​  ​​ ​  ​
x–1  ​ We have f (x) + f (1 – x)

= 2 log ​( ​ _____ 
x – 1)
x+1 9x 91 – x
 ​  ​ = _____  ​ + ________
​  x     ​ 1 – x   ​ 
9 +3 9 +3
= 2f (x)
9x 9
Hence, the result. = _____  ​ + _______
​  x     ​      ​
9 + 3 9 + 3 ◊ 9x
9. If f (x) = cos (loge x), then find the value of
9x 3
= _____  ​ + _____
1
f (x) f (y) – __ (  x
​   ​ ​  f (xy) + f ​ __
2
​ y ​  ​  ​ (  ) )
​  x    
9 + 3 3 + 9x
​     
 ​

9x 3
9. Given f (x) = cos (loge x) = _____​  x      ​ + _____
​     
 ​
9 + 3 9x + 3
1
We have f (x) f (y) – __
2 ( x
​    ​​ f (xy) + f ​ __
​ y ​  ​  ​ (  ) )
9x + 3
= ​ _____
9x + 3
  ​

​   ​​ { 2f (x) f (y) – ​( f (xy) + f ​( __


​ y ​ )​ )​ }​
1 x
= __ = 1.
2
Hence, the result.
{ 
1 2 cos (log x) cos (log y) –
= __
​   ​ ​   ​         ​  ​
2 ​(cos (log x) + cos (log y)) ​ } (  2 tan x
12. We have f  ​ ​ ________
1 + tan2 x
   
)
 ​  ​

= __
2 { 
1 cos (log x + log y) +  cos (log x – log y)
​   ​ ​   ​   
(  x   
(  ) )

– ​ cos (log xy) + cos ​ log  ​ __y ​  ​  ​
​  ​
} = __
1
​   ​  (1 + cos 2x) (sec2 x + tan x)
2
1

{ 
​ ​ __
= ​   ​  × 2 cos2 x × (1 + tan2 x + 2 tan x)

}
2
cos (log (xy)) + cos ​( log ​( __
​ y ​ )​ )​
x
1 = cos2 x × (1 + tan x)2
__    
2 – ​ cos (log xy) + cos ​  log​ __
= ​    ​ ​  ​     ​  ​
= {cos x × (1 + tan x)​}2​ ​
​ ( ( ( ​ y ​  )​ ​ )​
  x
​​

= (cos x + sin x)2
=0 = 1 + sin (2x)
ax – b Thus, f (sin 2x) = 1 + sin (2x)
9. We have y = f (x) = ​ ______  ​
bx – a fi f (x) = 1 + x
1.42  Differential Calculus Booster

13. Do Yourself (vi) Df = (0, •)


14. (i) Df = R – {– 3} (vii) Df = f
(ii) Df = R – {3} (viii) Df = f
(iii) Df = R – {0} (ix) Df = f
(iv) Df = R – {0} (x) Df = R – I
(v) Df = R – {1, 2} 17. Dr = 0 gives [x – 2] = 0
(vi) Df = R – {1, 2} fi 0 £ (x – 2) < 1
(vii) Df = R fi 2£x<3
(viii) Df = R – {– 2, 2} Thus, D f = R – [2, 3)
(ix) Df = R 18. Dr = 0 gives [x + 1] = 0
(x) Df = R
fi 0£x+1<1
15. (i) Df = [2, •)
fi – 1 £ x < 0
(ii) Df = [– 5, •)
Thus, D f = R  –  [– 1, 0)
(iii) Df = (– •, 4]
(iv) Df = [2, 4] 19. Dr = 0 gives [x] – 4 = 0
(v) Df = [3, 7]
fi [x] = 4
(vi) Df = [2, 5]
fi 4£x<5
(vii) Df = [– •, 0] » [1, •)
Thus, D f = R – [4, 5)
(viii) Df = (– •, 0) » [1, •)
(ix) Df = [0, 1] 20. Dr = 0 gives x – [x] = 0
(x) Df = (– •, 1] » [2, •) fi [x] = x
_____
16. (i) Given f (x) = ÷   – 2 ​ 
​ |x| fi xŒI
Thus, |x| – 2 ≥ 0 Also, x – [x] > 0
fi |x| ≥ 2 fi x > [x]
fi x ≥ 2, x £ – 2 fi x Œ f, 0 < f < 1
Therefore, Df = (– •, – 2] » [2, •) Thus, D f = R – I
_____
(ii) Given f (x) = ​ 4  – |x| ​ 
÷ 21. Dr = 0 gives [x] – x = 0
Thus, 4 – |x| ≥ 2
fi |x| £ 4 fi [x] = x
fi – 4 £ x £ 4 fi xŒI
Therefore, Df = [– 4, 4] Also, [x] – x > 0
_____ _____ fi [x] > x
(iii) Given f (x) = ÷   – 1 ​ + ÷
​ |x| ​ 4  – |x| ​ 
fi x=f
We have |x| – 1 ≥ 0 and 4 – |x| ≥ 0
Thus, D f = f
fi |x| ≥ 1 and |x| £ 4
fi x ≥ 1, x £ – 1 and – 4 £ x £ 4 22. We have f (x) = cos[p 2]x + cos[– p 2]x
fi x Œ [– 4, – 1] » [1, 4] fi f (x) = cos9x + cos (–10)x
Therefore, Df = [– 4, – 1] » [1, 4] fi f (x) = cos9x + cos (10)x

÷ 
______

(  )
|x| – 1 p
(iv) Given f (x) = ​ ​ _____     ​ ​ Now, f ​ ​ __ ​   ​
3 – |x| 2

(  )
|x| – 1
We have ​ _____   ​ ≥ 0 9p
3 – |x| = cos ​ ​ ___ ​   ​ + cos(5p)
2
|x| – 1 =0–1
fi ​ _____   ​ £ 0
|x| – 3 = – 1
fi 1 £ |x| £ 3 Also, f(p)
fi x Œ [– 3, – 1] » [1, 3] = cos(9p) + cos(10p)
Therefore, Df = [– 3, – 1] » [1, 3] = – 1 + 1
(v) Df = [0, •) =0
Real Function  1.43

23. We have fi 0 £ x + 4 – x2 < 1


[log2 1] + [log2 2] + [log2 3] + ... + [log2 64] Now, x + 4 – x2 ≥ 0
= [log2 1] + [log2 2] + ... + [log2 4] fi x2 – x – 4 £ 0
  + [log2 5] + [log2 6] + ... + [log2 8]
[ 
___ ___

  + [log2 9] + [log2 10] + ... + [log2 16] fi


2
    1_______
1 – ​÷17 ​
x Œ ​ ​ _______
 ​ 
, ​ 

2
   
​ 17 ​
 ​   ​ ]
  + [log2 17] + [log2 18] + ... + [log2 32]
Also, x + 4 – x2 < 1
  + [log2 33] + [log2 34] + ... [log2 64]
fi x + 3 – x2 < 0
  + [log2 65] + [log2 + 66]
fi x2 – x – 3 > 0

(  ) ( 
___ ___

)
= 2.3 + 3.4 + 4.16 + 5.32 + 6.2
   
1 – ​÷13 ​    
1 + ​÷13 ​
fi x Œ​ – • ​ _______  ​ » ​ •,​ _______
 ​    ​  
 ​
= 6 + 12 + 64 + 160 + 12 2 2
Thus, the number of solution is infinite.
= 254.
27. We have
24. We have [x] 2 – 3[x] + 2 = 0


a2 – 3a + 2 = 0, a = [x]
(a – 1)(a – 2) = 0
[  ] [  ] [ 
1
​ __
1 1
​   ​   ​ + ​ ​ __  ​ + ____
4 4 100
1
​     ​  ​ + ​ __
2
​    ​ + ____
4 100 ] 1
​     ​  ​ + ... + ​ __[  99
]
​    ​ + ​ ____  ​  ​
4 100

= [​  __
​   ​  × 100 ]​ = 25
fi a = 1, 2 1
when a = 1 fi [x] = 1 4
fi1£x<2 28. We have

[  ] [  ] [  ]
fi x Π[1, 2)
3 1 5
When a = 2 fi [x] = 2 [1007] + ​ 503 ​ __ ​   ​ + ​ 252 ​ __ ​   ​ + ​ 126 ​ ___  ​  ​ + ...
4 8 16
fi2£x<3
fi x Π[2, 3)
Hence, the solution set is x Π[1, 3)
= [​  ​ ________
2013 + 1
2
 ​  
 ]​ + ​[ ​ 20134
+2
________
]​ + [​  ​ 2013
 ​  

+4
________
8 ]​
 ​  

25. We have [x] 2 – 2[x] – 8 = 0




b2 – 2b – 8 = 0, b = [x]
(b – 4)(b + 2) = 0
[ 
2013 + 8
+ ​ ​ ________
16
 ​  
 ​ + ...]
fi b = – 2, 4 = 2013
when b = – 2 fi [x] = – 2 29. We have [sin x] = 0
fi – 2 £ x < 1 fi 0 £ sin x < 1
fi x Œ [– 2, – 1) Ïsin x ≥ 0
when b = 4 fi [x] = 4 fi Ì
Ó sin x < 1
fi4£x<5
fi x Œ [4, 5) Ï 2 np £ x £ (2 n + 1)p
Ô
Hence, the solution set is fi Ì p p : n ŒI
ÔÓ2 np + 2 £ x < 2 np - 2
x Œ [– 2, – 1) » [4, 5).
26. We have x2 – 4 [x] = 0 30. We have 2[x] + 3 = 3[x – 2] + 5
Case-I: When x Œ I fi 2[x] + 3 = 3[x] – 6 + 5
fi x2 – 4 – x = 0 fi [x] = 4
2
fi x –x–4=0 fi 4£x<5
______ ___
± ​÷1  + 16 ​ 
1__________ ± ​÷17 ​
   
1_______ Also, y = 2[x] + 3 = 2.4 + 3 = 11
fi x = ​   ​
     = ​   ​ 

2 2
Now, [x + y] = [x + 11] = [x] + 11 = 4 + 11 = 15
Case-II: When x œ I
31. We have f (x) = [x] – 2
fi x2 – 4 – (x – {x}) = 0
fi [x] – 2 ≥ 0
fi x2 – 4 – x + {x} = 0
fi [x] ≥ 2
fi {x} = x + 4 – x2
1.44  Differential Calculus Booster



x≥2
x Œ [2, •)
(  [  ] [  ] )
1
fi [100 x] – ​ [x] + ​ x + ____
18
​     ​  ​ + ... + ​ x + ____
100
​    ​  ​  ​
100

– ​( ​[ x + ​ ____  ​  ​ + ... + [​  x + ​ ____  ​  


100 ] 100 ] )
Thus, D f = [2, •) 92 99
​  ​ = 542
32. We have 4 – [x] ≥ 0
fi [x] £ 4
fi [100x] – 7 × 19 – 8 × 8 = 542
fi x<4+1=5
[ 7.42 £ x < 8]
fi x Œ (– •, 5)
Thus, D f = (– •, 5) fi [100x] = 542 + 133 + 64 = 739
33. Let D1 : [x] –1 ≥ 0
Henec, the result.
fi [x] ≥ 1 38. We have
fi x≥1
Also, let D2 : 3 – [x] ≥ 0
fi [x] £ 3
[  ] [  ] [  ] [  ]
2015
​ _____
​   ​  
2
2016
 ​ + ​ _____
​   ​  
4
2018
 ​ + ​ _____
​   ​  
8
2022
 ​ + ​ ​ _____
16
 ​  
 ​ + ...


Thus, Df = [1, 4)
x<3+1=4  ​ 2014 +
= ​[________
2
 1
 ​     [​  ​ 20144 ​ 
]​ + ________ ]​ + ​[ ​ 20148 ​ +4 
+2 ________

  ]​

+ ​[ ​ ________
16 ]
[x] – 2 2014 + 8
 ​    ​ + ...
34. We have ​ ______   ​ ≥ 0, [x] π 5
5 – [x]
[x] – 2 = 2014.
fi ​ ______   ​ £ 0, [x] π [5, 6)
5 – [x] 39. Case-I: When x is integer
fi 2 £ [x] £ 5, x π [5, 6) i.e x = n
L.H.S = [2 n] = 2 n
fi 2£x£4
Df = [2, 4]
35. As we know that, 0 £ sin2 x, cos2 x £ 1
R.H.S = [n] + ​ n + __
1
2 [  ]
​   ​   ​ = n + n = 2n
Case-II: when x is not an integer.
Thus, the values of i.e. x = n + f, where 0 £ f < 1
sin x
​ ​ _____
2 [  ] 2
[  (  ) ] [  (  ) ]
p
 ​ = 0 = ​ sin2 ​ __
 ​  
p
​    ​  ​ x  ​ = ​ cos2 ​ __
2 
​    ​  ​ x  ​
2 
L.H.S = [2(n + f)] = 2n + [2 f]

Ï2 n : 0 £ f < 0.5
[  ]
cos2 x
= ​ ​ _____ ​  ​
2014
= Ì
Ó 2 n + 1 : 0.5 £ f < 1

36. When [sin x] = 0, then [cos x] = 1


1
R.H.S = [n + f] + ​ n + f + __ [ 
​   ​   ​
2 ]
= n + [f] + n + [​  f + __
fi x = 0, p, 2p and x = 0, 2 p
​   ​  ]​
1
fi x = 0, 2 p
2

= 2n + [f] + [​  f + __
Also, when [cos x] = 0, then = [sin x] = 1
​    ​ ]​
1
p p
fi x = __
​   ​  and x = __
​   ​  2
2 2
p Ï2 n : 0 £ f < 0.5
fi __
x = ​   ​  = Ì
2 Ó2 n + 1 : 0.5 £ f < 1
Thus, the solution set is ​ 0, __ { 
p
​   ​ , 2 p  ​
2 } Hence, the result.
37. We have 40. Case-I: when x is an integer.
[x + 0.19] + [x + 0.20] + [x + 0.21] + ... i.e x = n
L.H.S = [3n] = 3n
+ ... + [x + 0.22] + ... + [x + 0.91] = 542
1
R.H.S = [n] + ​ n + __ [  ] [  ] 2
​   ​   ​ + ​ n + __
​   ​   ​
[ 
19
fi ​ x + ____
100 ] [ 
100
20
] 91
​    ​  ​ + ​ x + ​ ____  ​  ​ + ... + ​ x + ____[ 
​    ​  ​ = 542
100 ]
3
= n + n + n = 3n
3
Real Function  1.45

Case-II: when x is not an integer = [x] + {x}


i.e. x = n + f = x
L.H.S = [3[n + f] = [3 + 3f] = 3n + [3f]

Ï 1
2014

r = 1
( 
{x + r}
​  ​ ​   ​​ ​ ______
Thus, [x] + S
2014
 ​  
 )
​= x

Ô3n : 0 £ f < 3 44. We have 4{x} = x + [x]


Ô
Ô 1 2 fi 4 {x} = [x] + {x} + [x]
= Ì3n + 1: £ f <
Ô 3 3 fi 3 {x} = 2 [x]
Ô 2
2
Ô3n + 2 : 3 £ f < 1 fi {x} = __ ​   ​  [x]
Ó 3

[  1
R.H.S = [n + f] + ​ n + f + __ ] [  2
​   ​   ​ + ​ n + f + __
3
​   ​   ​
3 ] As we know that, 0 £ {x} < 1


2
0 £ __
​    ​ [x] < 1
= 3n + [f] + [​  f + __
​   ​  ]​ + ​[ f + __
3
​   ​  ]​
1 2
fi 0 £ [x] < 2
3 3
fi [x] = 0, 1
Ï 1
Ô3n : 0 £ f < 3 when [x] = 0 fi {x} = 0
Ô
Ô 2

1
= Ì3n + 1: £ f <
2 when [x] = 1 fi {x} = __
​    ​
3 3 3
Ô 2 5
Ô 2 Thus x = [x] + {x} = 0, 1 + __
​   ​  = 0, __
​   ​ 
Ô3n + 2 : 3 £ f < 1 3 3
Ó
Hence, the result.
5
Hence, the solutions are ​ 0, __ {  }
​   ​   ​
3
[  ]
2x2
41. We have ​ _____
​  2   
x +1
 ​  ​
45. We have 2x + 3{x} = 4[x] – 2

fi 2([x] + {x}) + 3{x} = 4[x] – 2

[  2
2
= ​ 2  – ​ _____
x +1
  
 ​  ​
] fi 5{x} = 2[x] – 2

[ 
2
= 2 + ​ – _____
​  2    
x +1
 ​  ​
] fi ( 
2[x] – 2
{x} = ​ ​ _______
5
 ​  
 ​ )
= 2 + (– 2), 2 + (–1) } ​
(  (   2
– 2 £ ​ – ​ _____
x2 + 1
  
) )
 ​  ​ < 0  ​ As we know that, 0 £ {x} < 1


= 0, 1
Hence, the number of integral values is 2.
fi { 
2[x] – 2
0 £ ​ ​ _______
5
 ​   }
 ​ < 1

42. We have {x} + {sin x} = 2 fi 0 £ 2[x] – 2 < 5


since {x} < 1 and {sin x} < 1 7
fi 2 £ [x] < __
​   ​ 
so {x} + {sin x} < 2 2
Thus, the given equation has no solution. fi [x] = 2, 3
Therefore, x = j 2
when [x] = 2 fi {x} = __
​    ​
43. As we know that {x + r} = {x} as x Œ I 5

( 
2 12
2014

r = 1
{x + r}
​  ​ ​   ​​ ​ ______
we have [x] + S
2014
 ​  

​ ) Thus. x = [x] + {x} = 2 + __
4
__
​   ​  =
5
___
​   ​ 
5
Also, [x] = 3 fi {x} = ​    ​
2014

r = 1
( 
{x + r}
​  ​ ​   ​​ ​ ______
= [x] + S
2014
 ​  

​ ) 5
Thus, x = [x] + {x} = 3 + __
4
​   ​  =
19
___
​   ​ 
5 5
{x}
= [x] + ​ ​ _____
​   
2014 ( (  ) (  ) {x}
 ​  ​ + ​ _____
​   
2014
 ​  ​ + ... 2014 times  ​ ) 12 19
Hence, the solutions are ​ ___ { 
​   ​ , ___
5 5
​   ​   ​.}
(  (  ) )
{x}
= [x] + ​ 2014 × ​ _____
​   
2014
 ​  ​  ​ 46. We have x2 – 4x + [x] + 3 = 0
1.46  Differential Calculus Booster



x2 – 4x + x – {x} + 3 = 0
x2 – 3x + 3 = {x}
fi ( 
cos2 x – 2
– 1 £ ​ ​ ________
2
 ​  
1
)
 ​ £ – ​ __ ​ 
2

As we know that 0 £ {x} < 1


fi 0 £ x2 – 3x + 3 < 1
cos2 x – 2
From (i), we get, ​ ​ ________
2
 ​   (  1
 ​ – (– 1) = ​ __ ​ 
4 )


when

x2 – 3x + 3 < 1
x2 – 3x + 2 < 0
cos2 x – 2
fi ​ ​ ________
2
 ​   (  3
 ​ = – ​ __ ​ 
4 )
3
fi (x – 1)(x – 2) < 0 fi (cos2 x – 2) = – ​ __  ​
2
fi 1<x<2
2 3 __
__ 1
when x2 – 3x + 3 ≥ 0 fi cos  x = 2 – ​   ​  = ​   ​ 
2 2


fi x Œ R
Thus the solution set is 1 < x < 2 which does not
fi (  )
1 2
cos2 x = ​​ ___
p
(  )
​  __  ​  ​​ ​ = cos2 ​ __
÷   
​ 2 ​
​   ​   ​
4
satisfy the given equation.
p
Hence, the equation does not have any solution. fi x = 2 n p ± ​ __ ​ , n Œ I
4
47. It has two solutions only when at x = 1 and 51. F(x) is defined only when 2 {x}2 – 3 { } + 1 ≥ 0
x = – 1.
48. We have 0 £ {x} < 1 and 0 £ {tan(p x)} < 1 fi 2 {x}2 – 2 {x} – {x} + 1 ≥ 0
Now, {x} + {tan (p x)} ≥ 0 fi (2 {x} – 1)({x} – 1) ≥ 0
Thus, the equation holds only when 1
fi {x} £ __
​   ​ , {x} ≥ 1
2
{x} = 0 fi x Œ 1
But {x} ≥ 1 is not possible.
and {tan(p x)} = 0 fi x Œ I 1
Thus, {x} £ __
​   ​ 
Thus, the equation have infinitely many solution. 2

{ 
x2 + 1
49. Given, y = ​ ​ _____
 ​  
 ​ }
fi [  1
2 ] [  ]
1
Df = ​ – 1, – ​ __ ​   ​ » ​ 0, ​ __ ​   ​ » {1}
2
2
52. Here, a = 3, b = 4 and c = 10
fi ( 
x2 + 1
y = ​ ​ _____
2
 ​   ) [ 
x2 + 1
 ​ – ​ ​ _____
2 ]
 ​  
 ​ Thus, the minimum values of f (x)
______
when x Œ [0, 1) = – ​÷a  2 + b2 
 ​ + c = – 5 + 10 = 5
and the maximum values of f (x)
fi y Œ (0, 1) ______
when x Œ [1, 2) ​ a  2 + b2 
=÷  ​ + c = 5 + 10 = 15.
__ __
fi y Œ [0, 1) 53. Rf = [min f (x), max f (x)] = Ζ ​÷2 ​
    + 3, ÷     + 3˚
​ 2 ​
when x = 2
fi y Œ (0, 1) 54. We have, A = sin4 q + cos4 q
Thus the number of integral values of y satisfy the = (sin2 q)2 + (cos2 q)2
given expression is 1. = 1 – 2 sin2 q. cos2 q
i.e. y = 0 only.
(sin2 q)2
= 1 – ​ _______
) [ 
 ​   
( 
cos2 x – 2
50. We have ​ ​ ________
2
 ​  
cos2 x – 2
 ​ – ​ ​ ________
2
 ​  
1
 ​ = __ ]
​   ​  
4
...(i)

2
{– sin2 (2 q)}
= 1 + ​ ___________
 ​    
2
Also, 0 £ cos2 x £ 1 As we know that, – 1 £ {– sin2 (2 q)} £ 0
fi 0 – 2 £ cos2 x – 2 £ 1 – 2 2
1 {– sin (2 q)}
fi – ​ __ ​  £ ​ ___________
 ​
    £0+1
fi – 2 £ (cos2 x – 2) £ – 1 2 2
{– sin2 (2 q)}
(  )
1
– 2 cos2 x – 2 – 1 fi – ​ __ ​  + 1 £ ​ ___________
 ​
     £0+1
fi ​ ___ ​ £ ​ ​ ________
 ​   ​ £ ___
​   ​  2 2
2 2 2
1
fi ​ __ ​  £ A £ 1
2
Real Function  1.47

55. We have, f (q) = sin6 q + cos6 q 3 1


B = m2 = ​ __ ​  + __
3
​    ​. 0 = __
​    ​
2 3 2 3 4 4 4
= (sin  q) + (cos q)
Now, the value of m12 + m22 + m1m2
= (sin2 q + cos2 q)2
9 3 37
– 3 sin2 q cos2 q (sin2 q + cos2 q) = 1 + ___
​    ​ + __
​    ​ = ___
​   ​ 
16 4 16
= 1 – 3 sin2 q cos2 q
3 57. We have, sin2 (sinq) + cos2 (cosq)
= 1 – __​   ​  (4 sin2 q cos2 q)
4 = sin2 (cosq) + cos2 (cosq) + sin2 (sinq) – sin2 (cosq)
3
= 1 – ​ __ ​ (sin2 2q) = (sin2 (cosq) + cos2 (cosq)) + sin2 (sinq) – sin2 (cosq)
4
3 = 1 + (sin2 (sinq) – sin2 (cosq))
= 1 + __ ​   ​  (– sin2 2q)
4 Max value of f (q)
As we know that, – 1 £ (– sin2 2q) £ 0

3 3(– sin  2q)
2
(  (  (  ) )
p
= 1 + ​ sin2 ​ sin ​ __
2 (  (  ) ) )
p
​   ​   ​  ​ – sin2 ​ cos ​ __
​   ​   ​  ​  ​
2
fi – ​ __ ​  £ ​ __________
 ​ £0
     = 1 + sin2 (1)
4 4
3 3(– sin2 2q) Min value of f (q)
fi 1 – ​ __ ​  £ 1 + ​ __________
 ​
    £1
4 4 = 1 + (sin2 (sin(0)) – sin2 (cos(0)))
1 = 1 – sin2 (1)
fi ​ __ ​  £ f (q) £ 1
4 58. We have, f (q) = (3sin(q) – 4 cos(q) – 10)
Hence, the maximum value = 1 and the minimum (3sin(q) + 4cos(q) – 10)
value = 1/4 = (9sin2 (q) – 16 cos2 (q))
56. We have      – 10(3sinq + 4cosq) – 10(3sinq – 4 cosq)

A = cos2 q + sin4 q = (9sin2 (q) – 16cos2 (q)) – 10 (3sinq + 4cosq


  + 3sinq  – 4cosq)
1 1
= __
​   ​  (2cos2 q) + __ ​   ​  (2sin2 q)2 = (9sin2 (q) – 16 cos2 (q)) – 60 sin (q)
2 4
1 1 = 25 sin2q – 60 sin (q) – 16
= ​ __ ​  (1 + cos (2q)) + __ ​   ​  (1 – 2cos(2q))2
2 4 = (5 sinq – 6)2 – 36 – 16
1
= __
​   ​  (1 + cos (2q)) + __
1
​   ​  (1 – 2cos(2q) + cos2 (2q)) = (5 sinq – 6)2 – 52
2 4
Hence, the minimum value of f (q) = 121 – 52 = 69
1 __ 1 1 __ 1 1
= ​   ​  + ​   ​  cos (2q) + ​   ​  – ​   ​  cos (2q) + __
__ __ ​   ​ (cos2 (2q)) 59. Now, 0 < sin2010 q £ sin2 q ....(i)
2 2 4 2 4
3 __
__ 1 2 and 0 < cos2014 q £ cos2 q ....(ii)
= ​   ​  + ​   ​  (cos  (2q))
4 4
Adding (i) and (ii), we get,
3 1
Max value of A = m1 = __ ​    ​ + __
​   ​ . 1 = 1
4 4 0 < sin2012 q + cos2014 q £ sin2 q + cos2 q
Also,
fi 0 < A £ 1
B = sin2 q + cos4 q
Thus, the range of A = (0, 1]
1 1
= __
​   ​  (2sin2 q) + __ ​   ​  (4cos4 q)
2 4 x2 sin2 x + 4
60. We have, f (x) = ​  __________
  
    ​
1 1 x sin x
= __
​   ​  (2sin2 q) + __ ​   ​  (2cos2 q)2
2 4 4
= x sin x + ​ _____ ​£ 4
    
1 1 x sin x
= ​   ​  (1 – cos (2q)) + __
__ ​   ​  (1 + cos (2q))2
2 4 Hence, the minimum values of f (x) is 4
1 1
= ​ __ ​  (1 – cos (2q)) + __ ​   ​  (1 + 2cos (2q) + cos2 (2q)) 61. Here, the function f is defined for all x in R
2 4
Thus, Df = R
1 1 1
= ​ __ ​  + __
​   ​  + __ 2
​   ​  cos (2q) 62. Here, the function f is defined for all x in R.
2 4 4
3 1 2 Thus, Df = R
= __
​   ​  + __
​   ​  cos  (2q)
4 4 63. Here, the function f is defined for all x in R.
Thus, the minimum value of Thus, Df = R
1.48  Differential Calculus Booster

64. Here, the function f is defined for all x in R. Thus, the number of solution is 1.
Thus, Df = R 71. f is defined if (x2 – 4x + 3)
65. Here, the function f is defined for fi (x – 1)(x – 3) > 0
1
e–|x| – __
​   ​  ≥ 0 fi x < 1, x > 3
2
1 Thus, Df = (– •, 1) » (3, •)
fi e ≥ __
–|x|
​   ​  x
​ 2  ​–1
2 __
72. Here, e​ ​ ​is defined for all x in R log(1 – x) is defined
fi –|x| ≥ loge ​ __
1
​   ​   ​
2 (  ) fi
for 1 – x > 0
x<1
fi –|x| ≥ loge 1 – loge 2 = –loge 2 fi x Œ (– •, 1)
fi |x| £ loge 2 Also, x1001 is defined for all x in R.
fi –loge 2 £ x £ loge 2 Hence, the domain of the given function
= R « (– •, 0) « R
(  )

1
fi loge ​ __
​   ​   ​ £ x £ loge 2 = (– •, 1)
2

[ 
x
1
fi ​ x Œ loge ​ __ (  ) ]
​   ​   ​, loge 2  ​
2
73. Here, the function f is defined for ​ _____
fi x < 0 and x > 2
x–2
    ​ > 0

D = [​  log ​( __
​   ​  )​, log  2 ]​
1 fi x Œ (– •, 0) » (2, •)
Thus, f e e
2 Thus Df = (– •, 0) » (2, •)
66. Here, the function f is defined for all x in R
74. Here, the function f is defined for (x – 2) > 0
Thus, Df = R
fi x>2
67. Here, the function f is defined for all x in R
fi x Œ (2, •)
Thus, Df = R
Thus, Df = (2, •)
68. We have ex + e f (x) = e
75. Here, log(1 – x) is defined for 1 – x > 0
fi e f(x) = e – ex fi x<1
fi f (x) = loge (e – ex)
fi x Œ (– •, 1)
It is defined for e – ex > 0 2
Also, x – 3x + 2 = 0 gives x = 1, 2
fi e > ex
Thus, Df = (– •, 1)
fi ex < e
fi x<1 76. Here, the function log(3 – x) is defined for
fi x Œ (– •, 1) 3 – x > 0 fi x < 3 x Œ (– •, 3)
Thus, Df = (– •, 1) Also, x2 – 5x + 4 = 0 gives x = 1, 4
69. We have 2x + 3x + 4x – 5x = 0 Thus, Df = (– •, 3) – {1} = (– •, 1) » (1, 3)
fi 2x + 3x + 4x = 5x 77. Here, the function f is defined for

(  ) (  ) (  )
x x x
2
fi ​​ __
3
​   ​   ​​ ​ + ​​ __
4
​   ​   ​​ ​ + ​​ __
​   ​   ​​ ​ = 1 fi (9 – x2) > 0, (9 – x2) π 1, (9 – x2) π 0
5 5 5
fi (x2 – 9) < 0, x2 π 8, x2 π 9
__
fi (x + 3)(x – 3) < 0, x π ± 2​÷2 ​
  ,  x π ± 3
__
fi – 3 < x < 3, x π ± 2​÷2 ​
  ,  x π ± 3
__ __
Thus,   )  » (2​÷2 ​
Df = (– 3, – 2​÷2 ​   ,  3)
78. Here, the function f is defined for
(x – 1) ≥ 0 & (3 – x) £ 0
fi x≥1&x£3
fi x Œ [1, 3]
Df = [1, 3]

(  )
Clearly it has only 1 solution.
2–x
70. We have 1 + 3x/2 = 2x 79. Here, the function f is defined for ​ _____
​   ​  ​ > 0
x –4
The given equation holds good only for x = 2
Real Function  1.49

2–x
fi ​ ​ _____ 
x–4 ( 
 ​  ​ < 0 ) Hence, f = g
85. Here Df = R & Dg = R – {0}
fi 2<x<4
Since Df π Dg, so f π g
__
Thus, Df = (2, 4)
​ x  2 ​  = |x|
86. Here, f (x) = x & g(x) = ÷
80. Here, the function f is defined for Thus, R f = R & Rg = [0, •)
(1 – log10 (x2 – 5x + 16)) > 0 Since R f π Rg, so f π g
fi 2
log10 (x – 5x + 16) < 1 87. Here, D f = R + & Dg = R – {0}

fi (x2 – 5x + 16) < 10 Since, D f π Dg, so f π g

fi (x2 – 5x + 6) < 0
(  )
x–1
88. Here, f is defined for ​ ​ _____ ​  ​ > 0
x–2
fi (x – 2)(x – 3) < 0
fi x < 1 and x > 2
fi 2<x<3
fi x Œ (– •, 1) » (2, •)
Thus, Df = (2, 3)
Thus, Df = (– •, 1) » (2, •)
81. Here the function f is defined for Also, the function g is defined for
(log10 x)2 – 5 (log10 x) + 6 > 0 x > 1 and x > 2
fi a2 – 5a + 6 > 0 a = log10 x fi x>2
fi (a – 2)(a – 3) > 0, a = log10 x Thus, Dg = (2, •)
fi a < 2 & a > 3, a = log10 x Since D f π Dg, so f π g
fi log10 x < 2 & log10 x > 3 89. Given f (x) = sin x and
x < 102 & x > 103 and x > 0
÷ 
fi ________ _____
1 – cos2x
fi 2 3
x Œ (0, 10 ) » (10 , •) g(x) = ​ ​ ________ 
    2 x  
 = ​÷sin
 ​ ​ ​ = |sin x|
2
Thus, Df = (0, 102) » (103, •) Clearly, Rf = [– 1, 1] & Rg = [0, 1]
82. Here, the function f is defined for Since Rf π Rg, so f π g
x – [x] > 0, x œ I 90. 0
fi x > [x], x œ I 91. 3P3 = 3! = 6
Thus, Df = R + – I 92. 5P4 = 5! = 120.
83. Given equation can be written as 2 – x = loge x 93. 3P3 = 3! = 6
From the graph it is clear that it has only one 94. 4
solution. 95. is 3 since
Y (i) 1 < 2 fi f (1) = 6 < f (2) = 7
(ii) 1 < 3 fi f (1) = 6 < f (3) = 8
y = log x (iii) 2 < 3 fi f (2) = 7 < f (3) = 8.
96. Number of many one function = Number of total
X¢ X function – Number of one-one function.
O
y=x–2
= 43 – 4P3 = 64 – 24 = 40.
97. Number of many one function = Number of total
function – Number of one-one function.

= 34 – 0 = 81
84. Here, Df = {1, 2} = Dg and Rf = {1, 4} = Rg 98. Number of many one function = Number of total
Also f (1) = 1, f (2) = 4 and g(1) = 1, g(2) = 4 function – Number of one-one function.
Thus all the conditions of equal functions are = 33 – 2 = 27 – 2 = 25
satisfied.
1.50  Differential Calculus Booster
x
99. Now, let y = _____
​     
x+1
 ​ ( 4!
= 34 – ​ ​ __________

2! × 1! × 1! 2!
3!
    ​ × __
​    ​  ​ )
fi y x + y = x = 81 – 36
fi x(y – 1) = – y = 45.
– y 107. The number of into functions = The total number of
fi x = ______
​     ​ functions – the number of onto functions.
(y – 1)
Then the range of a function is R – {1} ​ 
5!
  ( 
= 35 – ​ __________
3!
    ​ × __
5!
​    ​ + __________
​ 
3! × 1! × 1! 2! 2! × 2! × 1! 2!

3!
    ​ × __
​    ​  ​ )
Thus, Rf = R – {1} = Co-domain
= 243 – 150
Hence, f is onto function.
= 93.
100. Clearly, the range of a function is = (0, 1]
i.e. R f = (0, 1]. Since f is onto function, so 108. Given f(x) = 3x + 5.
Rf = Co-domain fi f ¢(x) = 3 > 0
fi A = Rf = (0, 1] fi f is strictly increasing function.
Hence, the set A is (0, 1]. fi f is one-one function.
101. Clearly, domain of a function is R. Also, the range of a function is R
i.e. Df = A = R. fi f is onto function.
Now, range of the function is [0, 1) Hence, f is a bijective function.
Thus, Rf = [0, 1). 109. Given f (x) = x2 + 1
Since f is onto function, so Rf = B. fi f ¢(x) = 2x > 0 " x Œ R +
Thus, B = [0, 1) fi f is strictly increasing function.

Hence, A « B = [0, 1). fi f is one-one function.


102. Number of onto function = Number of distribution Also, the range of a function is (1, •)
of 4 balls into 2 boxes, where no box is remain fi Rf = (1, •) = Co-domain
empty. fi f is onto function.
4! 4! 2!
= ​ ______
    ​ × 2! + ​ ______
    ​ × ​ __  ​ Hence, the function f is bijective.
3! × 1! 2! × 2! 2!
1
= 8 + 6 = 14. 110. Given f (x) = _____
​  2     ​
x +1
103. Number of onto function = Number of distribution 2x
of 4 balls into 3 boxes, where no box is remain fi f ¢(x) = ​ _______
    ​ > 0, " x Œ R +
(x2 + 1​)2​ ​
empty.
4! fi
= ​ ______
    ​ × 2! f is strictly increasing function.
3! × 1! fi f is one-one function.
=8 Also, the range of a function is (0, 1)
104. Number of onto function = Number of distribution fi Rf = (0, 1) = co-domain
of 5 balls into 3 boxes, where no box is remain fi f is onto function
empty.
5! 3! 5! 3! Thus, f is a bijective function.
= __________
​        ​ × __
​    ​ + __________
​       ​ × __
​    ​ x
3! × 1! × 1! 2! 2! × 2! × 1! 2! 111. Given f (x) = _____
​  2      ​, " x Œ (– 1, 1)
= 60 + 90 x +1
1 – x2
= 150. f ¢ (x) = ​ _______     ​ > 0 " x Œ (– 1, 1)
105. The number of into function = Total number of (1 + x2​)2​ ​
functions – the number of onto functions fi f is strictly increasing function
=2 –63 fi f is one-one function.


=8–6
= 2.
1
Also, the range of a function is ​ – ​ __ ​ ,
2 (  1
__
)
​   ​   ​
2

106. The number of into functions = Total number of


functions – the number of onto functions
fi ( 
1
Rf = ​ – ​ __ ​ ,
2
1
__
)
​   ​   ​ = Co-domain
2
Real Function  1.51

fi f is onto function.
Thus f is a bijective function.
112. Since the number of elements of A and B are not
same, so the number of one-one onto function is 0.
113. Since the number of elements of both the sets
are same, so the number of bijective function is
5
P5 = 5! = 120.
114. Hence, the number of bijective functions
= 3P3 = 3! = 6.
115.
Given f (x) = 3x + 5 1
f (x) = 1 – _____
​  2      ​
fi f ¢ (x) = 3 > 0 x +1
fi f is strictly increasing function. 2x
f ¢(x) = ​ _______    ​ > 0, " x Œ R +
fi f is one-one function (x + 1)2
2

Also, Rf = R = Co-domain fi f is strictly increasing function.


fi f is onto function. fi f is one-one function.
Thus, f is a bijective function.
x2
Hence, f –1 is exists. Also, let y = ​ _____  
 ​
x2 + 1
Let y = 3x + 5
fi y.x2 + y = x2
y–5
fi x = ​ _____ ​ 
  fi x2 (y – 1) = – y
3
– y y
x –5
Thus, f –1(x) = ​ _____
 ​ 

. fi x2 = ______
​     ​ = ______
​      ​
3 (y – 1) (1 – y)

÷ 
______
116. Given f (x) = x2 + 2 y
fi x = ​ ______
​        ​ ​
(1 – y)
fi Rf = (0, 1) = Co-domain
fi  f is onto function.
Thus, f is a bijective function.
fi f –1 (x) is exists.

÷ 
_____
x
Hence,  f   (x) = ​ _____
–1
​      ​ ​.

1–x

118. Given f (x) = 2x(x – 1).


fi f ¢(x) = 2x(x – 1) × (2x – 1) × loge 2 > 0
fi f  ¢(x) = 2x > 0 for every x > 0 for all x in [1, •)
fi f is strictly increasing function. fi f is strictly increasing function.
fi f is one-one function. fi f is one-one function.
Also, Rf = (2, •) = Co-domain
fi f is onto function.
Thus f is a bijective function.
Therefore, the inverse of the given function is
exists.
Let y = x2 + 2
fi x2 = y_____ –2
fi x=÷ ​ y  – 2 ​ 
_____
Hence, f –1 (x) = ÷
​ x  – 2 ​

Also, Rf = [1, •)
x2
117. Given f (x) = ​ _____  
 ​
x2 + 1 fi Rf = [1, •) = Co-domain
1.52  Differential Calculus Booster

fi f is onto function. fi f ¢(x) = 2x – 4 ≥ 0 for all x in Df


Thus, f is a bijective function.
fi f is strictly increasing function.
So its inverse is exists. fi f is one-one function.
Let y = 2x(x – 1) Also, Rf = [5, •) = Co-domain
y = 2​x​ ​– x
2
fi fi f is onto function.
fi x2 – x = log2 (y) Therefore, f is a bijective function.
fi x2 – x – log2 (y) = 0 Hence, its inverse is exists.
___________ Let y = x2 – 4x + 5
  + 4 log  
1  ± ​÷1  2 (y) ​
fi x = ​  _______________
 ​      fi x2 – 4x + (5 – y) = 0
2 ___________
___________
  + 4 log  
1  + ​÷1    – 4(5   
4   ± ​÷16 – y) ​
2 (y) ​ fi x = ​  _______________
 ​    
fi x = ​ _______________
 ​      2
2 ___________
___________
  + 4 log  
1  + ​÷1    +  16  –  
4   ± ​÷4y 20 ​
2 (y) ​ fi x = ​  _______________
 ​     
Thus, f –1 (x) = ​  _______________
 ​      2
2 _______
119. Since f is a bijective function, so its inverse is   – 1) ​ 
4 ± ​÷4(y ______
exists. fi x = ​  ___________
 ​
       – 1) ​ 
= 2 ± ​÷(y
2
10x – 10 –x  102x – 1 ______
Let y = ​ _________
x  ​ = ​ _______
–x    ​. fi   – 1) ​ 
x = 2 + ​÷(y , since x ≥ 2
10 + 10 102x + 1 ______
fi y.102x + y = 102x – 1 fi f –1(x) = 2 + ÷   – 1) ​ 
​ (x
fi 102x (y – 1) = – y – 1
1
1
4 ( 
122. Consider the function f : [0, •) Æ ​ – ​ __ ​ , •  ​, where. )

y+1
102x = –  ​ _____ 
 y + 1
 ​ = ​ _____  ​ f (x) = x2 – __ ​   ​ 
y–1 1–y 4

(  )
 y + 1 Clearly, f is one-one and onto function. So its inverse
fi 2x = log10 ​ ​ _____   ​  ​ is exists.

[ 
1–y
1
)
(  )
 1 + y ​ __ ​ , •  ​ ​ Æ [0, •).
Let its inverse is f  –1 (x) : ​  – ​ 

1
x = __
​   ​  log10 ​ ​ _____ 
 ​  ​ ​ 4 ​

÷ 
2 1–y ______
1
fi f –1 (x) = ​ x + __
( 
​   ​  ​ . 
Thus, f   (x) = __
–1 
2
l  1 + x
1–x )
​    ​  log10 ​ ​ _____ 
 ​  ​. 4
Consequently, we can say that, the two sides of the
120. Given f (x) = x + sin x given equation are inverse to each other.
fi f ¢(x) = 1 + cos x ≥ 0 for all x in R. Thus, the intersection point is the solution of the
fi f is strictly increasing function given equation. f (x) = x.
fi f is one-one function. 1
fi x2 – __
​   ​  = x
Also, the range of a function is R. 4
fi 1
f is a onto function. fi x2 – x = __ ​    ​
Thus, f is a bijective function. 4


Hence, f  –1 is exists.
Therefore, f –1(x) = x – sin x
(  ) 1 2 1
fi ​​ x – __
​   ​   ​​ ​ = __
2
​   ​ 
2
Given f (x) = x2 – 4x + 9 fi ​( x – __
​   ​  )​ = ± ___
121. 1 1
​    ​  __
2  
​÷2 ​  
1 1
fi x = __
​   ​  ± ___
​  __  ​ 
2 ÷   
​ 2 ​
Hence, the solutions are ​ __
1
​   ​  + ___
2 ÷ { 
1 1 ___
​  __  ​ , __
1
}
​   ​  – ​  __  ​   ​.
    2 ​÷2 ​
​ 2 ​   
123. Since the f is invertible, so f is one-one function.
Given f (x) = (m + 2)x2 – 3mx2 + 9mx – 1
fi f ¢(x) = 3(m + 2)x2 – 6mx + 9m
fi f ¢(x) = 3(m + 2)x2 – 6mx + 9m > 0
Real Function  1.53

Thus,  (m + 2) > 0 and 36m2 – 108m(m – 2) < 0 ex is defined for all x in R.


fi m > – 2 & m2 – 3m(m + 2) < 0 1
Also, ​ _____
   ​ is defined for x Œ R –{1}
fi m > – 2 & m2 + 3m > 0 |x – 1|
fi m > – 2 & m Œ (– •, – 3) » (0, •) Thus, Df = [0, 4] » R » R – {1}
fi m Œ (– 2, – 3) » (0, •) = [0, 4] – {1}
124. Given f (x) = f –1 (x) = [0, 1) » (1, 4]

(  )
______________________
|x| – 3
128. Here, cos–1 ​ ​ _____
3
  3 + 3bx2 – 3bx 
Now, y = ​÷a – x     +  b3 ​ + b 3
 ​  
 ​ is defined for

( 
3 3 2 3


(y – b) = a – (x – 3bx + 3bx – b )
(y – b)3 = a – (x – b)3

|x| – 3
– 1 £ ​ ​ _____
3 )
 ​ £ 1
 ​  

fi –3 £ |x| – 3 £ 3
fi (x – b)3 = a – (y – b)3
__________ fi 0 £ |x| £ 6
(x – b) = ​÷a  – (y –  
3
fi b)3
__________ fi –6 £ x £ 6
x = b + ​÷a  – (y –  
3
 ​ fi b)3 ​ x Œ [–6, 6]
__________
f  (x) = b + ​÷ 
–1 3
Thus, b)3
a – (x –    ​ 1
Also, ​ _____
   ​ is defined for all x in R
ex + 1
125. We have f (x) = sin–1(3x + 4) Thus, Df = [–6, 6] « R = [–6, 6]
fi – 1 £ 3x + 4 £ 1
fi – 5 £ 3x £ – 3 ( 
1 – |x|
129. Here, sin–1 ​ ​ _____
2
 ​   )
 ​ is defined for

5
fi ​ __ ​  £ x £ – 1
3
fi ( 
1 – |x|
–1 £ ​ ​ _____
2 )
 ​ £ 1
 ​  


5
[ 
x Œ ​ – ​ __ ​ , – 1  ​
3 ] fi

–2 £ (1 – |x|) £ 2
–3 £ (– |x|) £ 1
5
[ 
Thus, Df = ​ – ​ __ ​ , – 1  ​
3 ] fi

–1 £ |x| £ 3
–3 £ x £ 3
–1
126. Given f (x) = cos  (4x + 5) e –1 x
Also, ​ _____  ​ is defined for all x in R.
fi – 1 £ 4x + 5 £ 1 ex + 1

fi – 6 £ 4x £ – 4 Thus, Df = [–3, 3] « R = [–3, 3]


3
fi – ​ __ ​  £ x £ – 1
2 130. Here, the given function is defined for


3
[ 
x Œ ​ – __
​   ​ , – 1  ​
2 ] ( 
​ 
3
–1 £ ​ ________
   
4 + 2sinx )
 ​  ​ £ 1

3
Thus, Df = ​ – __ [ 
​   ​ , – 1  ​
2 ] fi –1 £ ​( ​  ________
4 + 2sinx
3
 ​  )​ £ 1

(  )
 x
127. Given f (x) = cos–1 ​ __
2
1
​   ​  – 1  ​ + ex + _____
​     ​ 
|x – 1| fi –3 £ (4 + 2sinx) £ 3

Here, cos ​( __


​   ​  – 1 )​ is defined for
 x –1
2 fi –7 £ (2sinx) £ –1

–1 £ ​( __ ​   ​  – 1 )​ £ 1


 x
fi 7 1
2 fi – __
​   ​  £ (sin x) £ – __
​   ​ 
x 2 2
fi 0 £ __
​    ​ £ 2
2 1
fi –1 £ (sin x) £ – ​ __  ​
fi 0£x£4 2
– p p
fi x Œ [0, 4] fi ​ ___ ​ £ x £ – ​ __ ​ 
2 6
1.54  Differential Calculus Booster

p p
[ 
Thus, Df = ​ – ​ __ ​ , – ​ __ ​   ​
2 6 ] fi

– 1 £ ​( ​| x2 – 1 |​ )​ £ 1
– 1 £ (x2 – 1) £ 1
1+x
2x( 
131. Here, sin–1 ​ ​ _____
    
2
)
​  ​ is defined for fi 0 £ (x2) £ 2
__

( 
fi 0 £ |x| £ ​÷2 ​
)
  
1 + x2
– 1 £ ​ ​ _____   
 ​  ​ £ 1 __ __
2x fi     £ x £ ​÷2 ​
– ​÷2 ​   

| 
__ __
1 + x2
fi ​ ​ _____
2x
|1 + x2|
   ​  ​ £ 1

| fi
Thus,
x Œ Ζ ​÷2 ​
Df = Ζ ​÷2 ​
  ˚ 
  ,  ​÷2 ​
__
  ˚ 
  ,  ​÷2 ​
__

fi ​ ______  £ 1
 ​  135. We have (fog) (x) = f (g(x))
2|x|
fi 1 + x2 £ 2 |x| = f (x – 1)

fi |x|2 – 2|x| + 1 £ 0 = (x – 1)2 + 1


= x2 – 2x + 2
fi (|x| – 1)2 £ 0
Also, (go f ) (x) = g ( f (x))
fi (|x| – 1)2 = 0
= g (x2 + 1)
fi (|x| – 1) = 0
= g (b), b = x2 + 1
fi |x| = 1
=b–1
fi x = – 1, 1
Also, the function cos (sin x) is defined for all x in R. = (x2+1) – 1
= x2
Thus,  Df = {– 1, 1} « R {–1, 1}
Note:  Clearly, fog π go f, so composition of functions
132. Here, the given function is defined for is not commutative.

– 1 £ log2 (x) £ 1 136. We have (fog)(x) = f(g(x))


–1 1
_____
fi 2 £ (x) £ 2 = f ​( ÷ )​
  –  2 ​  
​ x  
_____
1
fi ​__    ​  £ x £ 2 = sin (​÷x    –  2 ​) 

[  ]
2
1
Thus, Df = ​ __ ​   ​ , 2  ​ Also, (go  f ) (x) = g( f(x))
2
133. Here, the given function is defined for = g(sin x)
2 _______
– 1 £ log4 (x ) £ 1   x – 2 ​ 
= ​÷sin
fi 4 –1 £ (x2) £ 4 137. We have ( fo g) (x)
1
__ = f (g(x))
fi ​   ​  £ x2 £ 4
4__ = f ( 2
+ 1)

÷ 
__ __ __________
1
fi ​ __ ​   ​ ​   £ ​÷x  2 ​  £ ​÷4 ​
   = ÷ 
​ (x2 + 1)   – 3 ​
4
______
1
fi ​ __  ​ £ |x| £ 2 = ​÷x  2 – 2 ​ 
2
Thus, the domain of the function fo g
When |x| £ 2 fi – 2 £ x £ 2 __ __
= ​( – •, – ​÷2 ​
  ˚   ​ ​ » ​ Î​   ,  • )​
​ ÷2 ​
When
1
|x| ≥ ​__
1
]  ​​​[ __​ 12 ​  ​,​ • )​
( 
   ​  fi x Œ ​ ​ – •, –  ​ __  ​ ​  ​ »
2 ​ 2​ 138. Here, Df = [0, •) and Rf = [0, •)

D = ​[ – ​ __ ​ , – 2 ]​ » ​[ __


​    ​, 2 ]​.
 1 1 Also, Dg = R and Rg = [– 1, •)
Thus, f
2 2 As we know that fo g is defined only when
134. Here, the function f is defined for if Rg Õ Df

(  1
– 1 £ ​ ______
​  2     ​  
|x –  1|
​£ 1 ) But [– 1, •) [0, •)
Therefore,  fog is not defined.
Real Function  1.55

139. Now,  f ( f(x)) = f ​( ​( a – xn )1/n


​  )​ Ï(– x )2 : x≥0
n 1/n Ô
= f(b), where b = (a – x ) Ô(– x ) : x ≥ 0 and x<0

2
= (a – bn)1/n Ôx : x ≥ 0 and x<0
Ôx
( 
= ​​ a – (a –
1
__
) 1/n
xn​)​ ​n ​ × n​  ​​ ​ Ó : x<0

Ï x2 : x ≥ 0
= (xn )1/n Ô
=x Ô– x : x = 0

( (  ( (  ) ) ) )
2
1
Also,  f  ​ f  ​ __ (  (  ) )
​ x ​  ​  ​ = f  ​ ​​ a  – ​​ ​​ a – __
1 1/n n n
​  n   ​  ​​ ​  ​​ ​  ​​ ​  ​
x
Ôx : x = 0
Ôx : x < 0
Ó

(  (  ) )
= f ​​ a – ​ a – __
1
​  n   ​  ​  ​​ ​
x
1/n
142. we have (fo  f ) (x) = f ( f (x))

Ï1 + (1 + x ) :1 + x ≥ 0 and x ≥ 0
= ​​( __
​     ​ )​​ ​ = __
1 1/n 1 Ô1 – (1 + x ) :1 + x < 0 and x ≥ 0
​ x ​ Ô
x n =Ì
Ô1 + (1 - x ) :1 – x ≥ 0 and x £ 0
1
Therefore,  f (f (x)) + f  ​ f  ​ __ (  (  ) )
1
​ x ​  ​  ​ = x + __
​ x ​.
ÔÓ1 - (1 - x ) :1 - x < 0 and x < 0

140. We have  f ( f ( f (x))) = 0 Ï 2+ x:x ≥0


Ô1 - (1 + x ) : no common value
fi f ​( f ​( x2 – 1 )​ )​ = 0 Ô

fi f ​( (x2 – 1)2 – 1 )​ = 0 Ô1 + (1 - x ) : no common value
ÔÓ1 – (1 - x ) : x < 0
fi f ​( ​( x4 – 2x2 + 1 – 1 )​ )​ = 0
fi f (x4 – 2x2) = 0 Ï2 + x : x ≥ 0

Óx : x < 0
4 2 2
fi (x – 2x ) – 1 = 0
fi (x4 – 2x2)2 = 1 143. we have f (g (x))
fi (x4 – 2x2) = ± 1 = f ({x} + 1) : 1 £ x £ 2
When (x4 – 2x2) = – 1 fi x = ± 1 = ({x} + 1)2 – ({x} + 1) + 2 : x Œ R + and 1 £ x £ 2
= {x}2 + {x} + 2 1 £ x £ 2
Thus, domain of f (g (x)) = [1, 2]
and the range of f (g (x)) = [2, 4)
b d
Thus, the value of __
​ a ​ + __
​ c ​
2 4
= __
​   ​  + __
​   ​  = 2 + 2 = 4
1 2
144. Given g is the inverse of f
From the graph it is clear that, it has two solutions i.e. g(x) = f  –1(x)
Thus, the number of distinct real solutions of the fi f (g (x)) = x
given equation is 4.
fi f ¢(g (x)) ◊ g¢(x) = 1
ÏÔ x 2 : x ≥ 0
141. We have f (x) = Ì 1
ÔÓ x : x < 0 fi g¢(x) = ______
​       ​
f ¢(g(x))
Ï – x, x ≥ 0 1
and g(x) = Ì fi g¢(x) = ________
​       ​ = cosec (g(x))
Ó x, x < 0 sin (g(x))
145. Now, g2 (x) = g(g(x))
Ï f (– x ): x ≥ 0
Now,  f (g (x)) = Ì
Ó f ( x ): x < 0 (  )
1
= g ​ _____
​     
1–x
 ​  ​
1.56  Differential Calculus Booster

​ 
1
= ________
   
1
_____
1 – ​      ​
1–x
 ​ = ​ ________
1–x–1
  
x–1
 ​ = ​ _____
x   ​ 
1
= __
2 [  (  2p
3 ( 
​   ​ ​  (1  –  cos 2 x)  + ​ 1  –  cos ​ 2x + ​ ___ ​   ​  ​ ))
(  (  ) ]
1–x p p
Also, g3 (x) = g (g(g(x)))
   + cos ​ 2x + __
​   ​   ​ + cos ​ __
3 )
​   ​   ​  ​
3


1
= ​ ________
   
x  –
1 – _____
  
1
​  x    ​ 
​ 
x –
x
 ​ = ________
   
x  +  1
 ​ = x
1
= ​ __ ​  ​
2 [  1
  1  +  1 + __
2 3
p
​    ​ – cos 2 x  +  cos ​ 2x +​ __ ​   ​ (  )
Again, g4 (x) = g (g (g (g (x))))
2p
(  )]
– cos ​ 2x + ​ ___ ​   ​  ​
3

1
= _____
​     
1–x
 ​
Thus, the period of g(x) is 3.
1 __
= __
​   ​  ​
2 2
5
[  p
  ​   ​  – cos 2x  +  2 sin ​ 2x + __ (  p
​   ​   ​ sin ​ __
2 ) (  ) ]
​   ​   ​  ​
6

Now, g2016 (x) = g3 (x) = x


1 5
= __ [ 
​   ​  ​ __
2 2
1
​   ​  – cos 2x + 2 cos 2x × __
​   ​   ​
2 ]
[ 
Thus, the range of g2016 (x) is R.
146. We have = __
1 5
​   ​  ​ __
2 2
​    ​ – cos 2x + cos 2 x  ​ ]
(c – 1) (x2 + x + 1)2 – (c + 1) (x4 + x2 + 1) = 0 5
= __​   

(  )
fi (x2 + x + 1)[(c – 1)(x2 + x + 1) 4
5
 ​ Also g ​ __ ​   ​   ​ = 1

(  )
– (c + 1)(x2 – + 1)] = 0 4
5
Thus, (go f ) (x) = g (f (x)) = g ​ __ ​    ​  ​ = 1
4
fi (c – 1) (x2 + x + 1) – (c + 1)(x2 – x + 1) = 0
Hence, the result.
fi x2 (c – 1 – c – 1) + x(c – 1 + c + 1) 148. We have f (g(x)) = 1 + x2 – 2x3 + x4

+ (c – 1 + c + 1) = 0 = 1 + (x4 – 2x3 + x2)

fi – 2x2 + 2cx – 2 = 0 = 1 + x2 (x2 – 2x + 1)

fi x2 – cx + 1 = 0 = 1 + x2 (x – 1)2

fi x2 + 1 = cx = 1 + (x(x – 1))2
1–x
Now,  f ( f (x)) = f  ​ ​ _____ 
1+x ( 
 ​  ​ ) Thus, g(x) = ± x(x – 1)
149. we have f (g(x))
1- x
1- = f (1 + x – [x])
1 + x 1 + x - 1 + x 2x
= = = =x
1- x 1+ x +1- x 2 = 1, since the Rg = [1, 2)
1+
1+ x
150. Given f (f (x)) = x
1
Also,  f ​ f  ​ __(  (  ) ) (  x–1
​ x ​  ​  ​ = f ​ ​ _____ 
x+1
 ​  ​ ) fi (  ax
f ​ ​ _____
   )
 ​  ​ = x
x+1
x -1
=
1-
x +1 x +1- x +1 2 1
= = = fi ​ ___________
 ax 
a ◊ ​ ​ _____  
 x +1
     
( 
 ​  ​
 ​ = x
)
( 
x - 1 x + 1 + x - 1 2x x

1+
x +1
 ax 
​ ​ _____
 x + 1
   ​  ​  +  1  )
1
Therefore, f (x (x)) + f ​ f  ​ __ (  (  ) )
​ x ​  ​  ​ a2 x
fi ​ _________
  
ax + x + 1
 ​ = x
1
= x + __ ​ x ​ fi a2 x = (a + 1)x2 + x
x2 + 1 cx
= ​ _____
x   ​=
  ​ __
x ​  = c
Comparing the co-efficients of x and x2
we get, a2 = 1 & a + 1 = 0
147. We have f (x)
fi a2 = 1 & a = – 1
1
= __ [  (  p
) (  p
)]
​    ​  ​ 2 sin2 x  +  2 sin2 ​ x  + ​ __ ​   ​ + 2cos x cos ​ x  + ​ __ ​   ​  ​
2 3 3


a = ± 1 & a = – 1
a=1
Real Function  1.57

151. Given ( 3x – 1


f (x) = x ​ ​ _____
3x + 1

  ​  ​ ) fi (  ) (  )
3–x
f(x) + f(– x) = log ​ ​ _____ 
3+x
3+x
 ​  ​ + log ​ ​ _____ 
3–x
 ​  ​

f (x) + f (– x) = log ​( ​ _____ 


fi –x( 
3 –x – 1
f (– x) = – x ​ ​ ______
3 +1

  )1 – 3x
 ​  ​ = – x  ​ ​ ______x 
1  +  3 ( 
 ​  
​ ) fi
3 – x 3_____
3+x
 ​ × ​ 
3 – x)
+x
  ​  ​ = log1

fi ( 
3x – 1
f (– x) = x ​ ​ _____
3x + 1

 ​  ​ = f(x))
fi f (– x) = – f(x)
=0

fi f (x) is an even function. fi f (x) is an odd function.


2 _____
152. Given f (x) = x sin (x + 1) 157. Given f(x) = log ​( x + ​÷x  2 + 1 ​  
)​
fi f  (– x) = – x sin ((– x)2 + 1) _____
fi f (– x) = log ​( – x + ÷ )​
​ x  2 + 1 ​ 

fi f (– x) = – x sin (x2 + 1) = – f (x) _____
fi  – x )​
f (– x) = log ​( ​÷x  2 + 1 ​
fi f (x) is an odd function. _____
fi f (x) + f(– x) = log​( x + ÷ )​
​ x  2 + 1 ​ 

153. Given f (x) = tan–1 (sin (cos–1(x))) _____
fi f(– x) = tan–1 (sin (cos–1(–x))) + log​( ​÷x  2 + 1 ​ – x )​
_____
fi f(– x) = tan–1 (sin (p – cos–1 x)) fi f (x) + f (– x) = log ​{ ​( x + ÷ )​
​ x  2 + 1 ​ 

fi f (– x) = tan–1 (sin (cos–1 x)) _____
 – x )​ }​
  ​( ​÷x  2 + 1 ​
fi f (x) is an even function.
fi f (x) + f(– x) = log (x2 – x2 + 1) = 0
154. Given f(x) = sin x + cos x
fi f (– x) = – f (x)
fi f (– x) = sin (– x) + cos (– x)
fi f (– x) = – sin x + cos x fi f (x) is an odd function.
________ ________
fi π f (x), – f (x) 158. Given ​ x  2 – x  +  1 
f(x) = ÷ ​ x  2 + x  +  1 
 ​ – ÷  ​
________ ________
fi f (x) is neither even nor odd function. fi ​ x  2 + x  +  1 
f (– x) = ÷ ​ x  2 – x  +  1 
 ​ – ÷  ​ = –f (x)
155. Given (  x
f (x) = ​ _____
​  x    
e –1 2
 ​ + __
x
​    ​ + 1  ​ ) fi f (x) is an odd function.

–x x 159. we have f (x)


fi f (– x) = ​ ______
–x     ​ – __ ​    ​ + 1
e –1 2 =O+O+O
x
______ x
= ​    –x   ​– __ ​    ​ + 1 = Odd function
1–e 2
160. we have f (x)
xex x
= ​ _____  ​ – __
3
x    ​    ​ + 1 (tan ( x 5 )) e x sgn x7
e –1 2

( 
= Ø Ø Ø Ø

xex
= ​ ​ _____
x   
e –1 ) x
 ​ – x  ​ + __
​    ​ + 1
2 O O O O

= (​ ​ ___________
= even function
e –1 ) 2
x x
xe – xe + x x
 ​  
  x ​ + __
​    ​ + 1 161. We have

( 
Numerator = E × O = O
= ​ _____
x
​  x     ​  
e –  1
​+ ) x
__
​    ​ + 1
2 Denominator = O + O + O = O
O
= f (x) Thus,  f(x) = __ ​    ​ = E = even function.
O
fi f (x) is an even function. 162. We have
Numerator = O(O(O)) = O
156. Given ( 
3–x
f(x) = log ​ ​ _____ 
3+x
 ​  ​ ) Denominator = E + E + E = E
O
Thus,  f (x) = __ ​   ​  = O = Odd function.
E

3+x
3–x ( 
f(– x) = log ​ ​ _____ 
 ​  ​ ) 163. We have f (x)
1 1
= __
​   ​  ( f(x) + f(– x)) + __
​   ​  ( f (x) – f(– x))
2 2
1.58  Differential Calculus Booster

1 1  Thus, the period of f (x) is


= __ ​   ​  (​  4sinx – 4 – sin x )​
​   ​  (4sinx + 4 –sinx) + __
{ 
2 2
= (Even Function) + (Odd Function)

p 2p
= L.C.M of ​ __
​   ​ , ​ ___ ​   ​
2 3 }
Hence, the result. L.C.M of {p, 2p}
= ________________
​    
   ​
164. We have f(x) H.C.F of {2, 3}
1 1 2p
= __
​   ​  ( f(x) + f (– x)) + __ ​   ​  (f (x)) – f (– x)) = ​ ___ ​ = 2p
2 2 1
1
​   ​  (​  (1  +  x)
= __ 2015
+ (1 – x)2015 )​ 169. Here, the period of 3 sin 4x is = p /2 and the period
2 p
1 of 4 ​| sin 4x |​ is = __
​   ​ 
+ __​   ​  (​  (1 + x)2015 – (1 – x)2015 )​ 4
2 Thus, the period of f (x) is
= (Even function) + (Odd function)
Hence, the result.
p
= L.C.M of ​ __
2 {  __​ p4 ​  }​
​   ​ ,
165. Even extension of f (x) is
L.C.M of {p, p}
Ï x + x2 : 0 £ x < 3 = _______________
​    
   ​
Ô H.C.F of {2, 4}
Ô x +1 :3 < x £ 5
f ( x) = Ì p
2 = __
​   ​ 
Ô– x + x : - 3 < x £ 0 2
Ô- x + 1 : - 5 £ x < - 3
Ó 170. We have f (x) = sin x . cosec x
and odd extension of f (x) is = 1
Thus, the function is periodic but it has no funda-
Ï x + x2 : 0 £ x < 3
Ô mental period.
Ô x +1 :3 < x £ 5 171. We have f (x) = tan x . cot x
f ( x) = Ì
2
Ô x - x : -3 < x £ 0 =1
Ô x - 1 : - 5 £ x < -3 Thus, the function is periodic but it has no funda-
Ó
mental period.
Hence, the result. __
166. Even extension    x )​ is
172. Here, the period of 5 sin ​( 2​÷2 ​ 
2p__ ___ p
Ï f ( x) : 0 £ x £ 2
=Ì = ​ ____  ​ = ​  __  ​ 
    ​÷2 ​
2​÷2 ​   
Ó f (- x ) : - 2 £ x < 0 __
and the period of 7 cos ​( 3​÷2 ​    x )​ is
ÏÔ x + e x + sin x : 0 £ x £ 2
2p__

-x = ​ ____  ​ 
ÔÓ- x + e - sin x : - 2 £ x < 0   
3​÷2 ​
We can not find the odd extension of f (x) Thus, the period of f (x) is
since f (0) = 1.
In case of odd function, the value of f (0) must be

÷
p ____
= L.C.M of ​ ​ ___ { 
2p
__  ​,  ​  __  ​  ​
    3​÷2 ​
​ 2 ​    }
zero. L.C.M of {p, 2p}
167. Even extension = ​  _________________
__  
   __  ​
   , 3​÷2 ​
H.C.F of {​÷2 ​    }
Ï f ( x) : 0 £ x £ 1 2p __
f ( x) = Ì = ​ ___
__ 
 ​ = p/​÷2 ​
  
Ó f (- x ) : - 1 £ x < 0 ÷   
​ 2 ​
__ 2p__ ___ p
ÏÔ x 2 + x + sin x - cos x + log(1 + x ) : 0 £ x £ 1   x   )​ is ​ ____
173. Here, the period of 3 sin ​( 2​÷3 ​   ​ = ​  __  ​ 
f ( x) = Ì     ​÷3 ​
2​÷3 ​   
2
ÔÓ x - x - sin x - cos x + log(1 - x ) : - 1 £ x < 0 __ 2p__
and the period of 2 cos ​( 5​÷3 ​   x   )​ is ​ ____  ​ 
We can not find the odd extension, since f (0) = – 1.   
5​÷3 ​
2p p Thus, the period of f(x) is
168. Here, the period of 3 sin 4x = ​ ___ ​ = __
{ 
​   ​  and the period
2p
___
of 4 cos 3 x = ​   ​ .
3
4 2

÷
p__ ____
= L.C.M of ​ ​ ___
2p
  ​,  ​  __  ​  ​
    5​÷3 ​
​ 3 ​    }
Real Function  1.59

Since, two irrational roots are not of the same kind, 178. Here, the period of 3 sin{ 2x} is
so we can not find its L.C.M 1 1
= __
​   ​  and the period of 2 cos {3x} is = __
​   ​ 
So, the period of the given function is not exist. 2 3
174. Here, the period of sin2 x is p, the period of Hence, the period of f (x) is

(  p
sin2 ​ x + __ )
​   ​   ​ is also p.
3
= L.C.M of ​ __
1 1
{  }
​   ​ , ​ __  ​  ​
2 3
L.C.M of {1, 1}
Finally, the period of cos x cos ​  x + __
p
​   ​   ​
3 (  ) = ​ ______________   
   ​
H.C.F  of  {2, 3}

( 
1

1
= __
​   ​  × ​ 2 cos x cos  ​ x + __
2
p
( 
​   ​   ​  ​
3 )) = __​   ​  = 1
1

(  (  (  ) )
p p 179. We have, f (x) + f ( x + 3) = 5 ...(i)

1
= ​   ​  × ​ cos ​ 2x + ​   ​   ​ + cos ​ __
__
2
__
3 )
​   ​   ​  ​
3 Replacing x by x + 3, we get,
2p
= ​ ___ ​ = p f ( x + 3) + f ( x + 6) = 5 ...(ii)
2
Subtracting (ii) from (i), we get,
Thus, the period of f (x) is
fi f (x) + f ( x + 3) – f ( x + 3) – f ( x + 6) = 0
= L.C.M of {p, p, p} fi f (x) – f (x + 6) = 0
=p
fi f (x + 6) = f (x)
175. Here, the period of |​ sin x + cos x |​ is

|  ( 
Thus, f (x) is periodic with period 6.
)|
__ p
=÷     sin ​ x + __
​ 2 ​ ​ ​   ​   ​  ​
4 180. We have f (x + 4) + f (x – 4) = f (x) ...(i)
= 2p Replacing x by x + 4, we get,
p f (x + 8) + f (x) = f (x + 4) ...(ii)
and the period of |​ sin x |​ + ​| cos x |​ is __
​   ​ 
2 From (i) and (ii), we get,
Hence, the period of f(x) is
f (x + 8) + f (x + 4) + f (x – 4) = f (x + 4)

p
= L.C.M of ​ 2p, ​ __ ​   ​
2 {  } fi f (x + 8) + f (x – 4) = 0
L.C.M of {p, 2p} Replacing x by x + 4, we get,
= ________________
​    
   ​
H.C.F of {1, 2}
f (x + 12) + f (x) = 0 ...(iii)
2p
= ​ ___ ​ = 2p Again replacing x by x + 12, we get,
1
176. Here, the period of |​ sin x + cos x |​ is f (x + 24) + f (x + 12) = 0 ...(iv)

|  (  )|
__ p
    sin ​ x + __
= ​÷2 ​ ​ ​   ​   ​  ​ Subtracting (iii) from (iv), we get,
4 f (x + 24) + f (x + 12) – f (x + 12) – f (x) = 0
= 2p
fi f (x + 24) – f (x) = 0
and the period of |sin x| – |cos x| is p
fi f (x + 24) = f (x)
Hence, the period of f (x) is
= L.C.M of {2p, p} Thus f (x) is periodic with period __ 24.
= 2p     f  (x)
181. We have f ( x – 1) + f ( x + 1) = ​÷2 ​ ...(i)
1 |sin x|
177. Here, the period of __
​    ​ ​ _____
2 cos x
​   ​   ( 
​ is ) Replacing x by x + 1 and x by x – 1, we get,
__
f (x) + f (x + 2) = ÷     f (x + 1)
​ 2 ​ ...(ii)
__
= L.C.M of {p, 2p} = 2p and the period
and f (x – 2) + f (x) = ÷     f (x – 1)
​ 2 ​ ...(iii)
1 sinx
2 |cos x| ( 
of ​ __ ​  ​ ​ ______  )
 ​  ​ is = {2p, p} = 2p
  Adding (ii) and (iii), we get,
f (x + 2) + f (x – 2) + 2f (x)
__
Thus, the period of f (x) is =÷     (f (x – 1) + f (x + 1))
​ 2 ​
= L.C.M of {2p, 2p} __ __
= ÷     ​( ​÷2 ​
​ 2 ​     f (x) )​
= 2p
= 2 f(x)
1.60  Differential Calculus Booster

f (x + 2) + f (x – 2) = 0 ...(iv) Multiplying (i) by 2 and (ii) by 3, we get,

Replacing x by x + 2, we get, 4f (sin  x) + 6f (cos x) = 10 ...(iii)

f (x + 4) + f (x) = 0 ...(v) and 9f (sin x) + 6f (cos x) = 15 ...(iv)


Again replacing x by x + 4, we get, Subtracting (iii) from (iv), we get,
f (x + 8) + f (x + 4) = 0 ...(vi) 5 f (sin x) = 5
Subtracting (v) from (vi), we get, fi f (sin x) = 1
f (x + 8) + f (x + 4) – f (x + 4) – f (x) = 0 fi f (x) = 1, " x Œ (0, 1)
fi f (x + 8) – f (x) = 0 185. Given f (1) = 0
fi f (x + 8) = f (x) = 0 f (2) = f (1) + 1 = 0 + 1 = 1
Hence, the function f (x) is periodic with period 8. f (3) = f (2) + 1 = 1 + 1 = 2

182. We have f (x + 1) + f (x + 5) = f (x + 3) + f (x + 7) f (4) = f (3) + 1 = 2 + 1 = 3, ...


...(i) Now, f (2) – f (1) = 1
Replacing x by x + 2, we get, f (3) – f (2) = 2
f (x + 3) + f (x + 7) = f (x + 5) + f (x + 9) f (4) – f (3) = 3
...(ii) ... ... ...
Adding (i) and (ii), we get, ... ... ...
f (x + 1) + f (x + 5) + f (x + 3) + f (x + 7) ... ... ...
= f (x + 3) + f (x + 7) + f (x + 5) + f (x + 9) f (x + 1) – f (x) = x
On addition, we get,
fi f (x + 1) = f (x + 9) ...(iii)
f (x + 1) – f (1) = 1 + 2 + 3 + ... + x
Finally replacing x by (x – 1) in (iii), we get,
fi f (x + 1) = f (1) + 1 + 2 + 3 + ... + x
f (x) = f (x + 8)
fi f (x + 1) = 0 + 1 + 2 + 3 + ...
Hence, the period of f (x) is 8.
x(x – 1)
183. Given
1
(  )
f (x) + 3f ​ ​ __
x ​  ​ = 5x ...(i)

Replacing x by x – 1, we get,
+ x = ​ _______
2
 ​  

Replacing x by 1/x in (i), we get,


(x – 1)(x – 2)
f(x) = ​ ____________
 ​
     for all x in N.
(  )
1
f  ​ __
1
​ x ​  ​ + 3f (x) = 5 ​ __
x ​ ...(ii)
2
186. Given f (x + f (x)) = 4f (x)
Multiplying (ii) by ‘3’ and subtracting from (i)
we get, Put x = 1, we get, f (1 + f(1)) = 4f(1)


1
f (x) + 3f  ​ __ (  ) 1
​ x ​  ​ – 3f   ​ __ (  ) 15
​ x ​  ​ – 15f (x) = ___
​  x ​  fi f (1 + 4) = 4 × 4

15 fi f (5) = 16
fi – 14f (x) = ___
​  x ​ 
187. Given P (x + 1) – P (x) = 2x + 1
15
fi f (x) = –  ​ ____  ​  Put x = 1, 2, 3, ..., n, we get,
14x
P(2) – P(1) = 3
184. Given 2f (sin x) + 3f (cos x) = 5 ...(i)

( 
p P(3) – P(2) = 5
Replacing x by ​ __
2 )
​   ​  – x  ​ in (i), we get,
P(4) – P(3) = 7
2f  ​ sin ​ __ (  ( 
p
)) p
​   ​  – x  ​  ​ + 3f  ​ cos ​ __
2 (  (  ))
​   ​  – x  ​  ​ = 5
2


...
...
...
...
...
...
... ... ...
fi 2f (cos  x) + 3f  (sin x) = 5 ...(ii)
P(n + 1) – P(n) = 2n + 1
Real Function  1.61

On addition, we get, Since, the equation is valid for all x except 0, we


compare the co-efficients of various powers of x on
P(n + 1) – P(1) = 3 + 5 + 7 + ... + (2n + 1)
both sides
fi P(n + 1) = P(1) = 3 + 5 + 7 + ... + (2n + 1) x2n : ao = an ao fi an = 1, where ao π 0
fi P(n + 1) = 1 + 3 + ... + (2n + 1) = n(n + 1) x2n – 1 : a1 = aoan–1 + a1an fi an–1 = 0
fi P(n + 1) = n(n + 1) x2n–2 : an–2 = 0 
fi P(n) = n(n – 1) x2n–3 : an–3 = 0
Hence, the polynomial P(x) is P(x) = x(x – 1) ... ... ...
188. Given f (x + y) = f (x). f (y) ... ... ...
... ... ...
Put x = 1 = y, f (2) = f (1) ◊ f (1) = 4 = 22
x 2 : a2 = 0
Put x = 2, y = 1, f (3) = f (2) ◊ f (1) = 23
x : a1 = 0
Put x = 2 = y, f (4) = f (2) ◊ f (2) = 24
n Now, comparing the co-efficients of xn, we get,
Now, Â f (a + k )
k =1 2an = a 02 + an2
= f (a + 1) + f (a + 2) + f (a + 3) + ... + f (a + n) fi 2 = a 02 + 1
= f (a) ◊ f (1) + f (a) ◊ f (2) + ... + f (a) ◊ f (3) fi a 0 = ± 1
= f (a) ( f (1) + f (2) + f (3)+ ... + f (n)) Thus, f (x) = ± xn + 1
= f (a) (2 + 22 + 23 + 24 + ... + 2n) 190. Given f (x) f  ( y) = f (x) + f (y) + f (xy) – 2
1
Also, f (a) (2 + 22 + 23 + ... +2n) = 16 (2n – 1) Put y = __
​ x ​

fi ( 
2 n – 1
f (a) ◊ 2 ​ ​ ______ 
2–1 )
 ​  ​ = 16 (2n – 1) fi (  )
1
f (x) ◊ f  ​ __ (  )
1
​ x ​  ​ = f (x) + f ​ __
​ x ​  ​ + f (1) – 2

​ x ​ )​ = f (x) + f  ​( __


f (x) ◊ f  ​( __ ​ x ​ )​ + 2 – 2
fi f (a) = 8 1 1

fi 2a = 8 = 23
​ x ​ )​ = f (x) + f  ​( __
f (x) ◊ f  ​( __ ​ x ​ )​
1 1
fi a =3 fi

189. Let f(x) = a 0xn + a1xn–1 + a2 xn – 2 + ... + an fi f(x) = xn + 1


1
Given relation is f(x) + f  ​ __ (  )
1
​ x ​  ​ = f(x) ◊ f  ​ __
​ x ​  ​ (  ) Also, it is given that, f (2) = 5

​( aoxn + a1xn–1 + a2 xn–2 + ...  + an  ​ ) fi



2n + 1 = 5
2n = 4

ao
( 
+ ​ ​ __n  ​ +
x
a1
____
​  n–1  ​ +
x
a2
​ ____
xn–2 )
  ​ + ...  + an  ​ fi
Thus,
n=2
f (6) + 2009
= (​  aox + a1x
n n–1
+ a2 x n–2
+ ... + an  ​ ) = 36 + 2009

( 
= 2045.

ao
. ​ ​ __n  ​ +
x
a1
____
​  n–1  ​ +
x
a2
​ ____
xn–2 )
  ​ +  ...  + an  ​
191. Given P(x2) + 2x2 + 10x = 2x  P (x + 1) + 3
Put x = 0, we get, P(0) = 3.
fi ​( aox2n + a1x2n–1 + a2 x2n–2 + ...  + anxn )​
Put x = –1, we get,
+ (​  ao + a1x + a2 x2 +  ...  +  anxn )​ P(1) + 2 – 10 = – 2P(0) + 3 = – 6 + 3 = – 3
P(1) = – 3 – 2 + 10 = 5
= (​  aoxn + a1xn–1 + a2 xn–2 + ...  + an )​
Let P(x) = a x + b
. ​( ao + a1x + a2 x2 + ... + anxn )​
Then P(0) = b fi b = 3
1.62  Differential Calculus Booster

Also,

P(1) = a + b fi a + b = 5
a+3=5fia=2
Now,
9u + v
20 ( 
f (u, v) = 20 × ​ ​ ______
 ​  
 ​ = 9u + v)
fi f (x, y) = 9x + y
Thus, P(x) = 2x + 3
4. As we know that, if a polynomial satisfying
192. Let f (x) = ax2 + b x + c
Put x = 0 fi c = f (0) = 1.
(  )
1
f (x) + f  ​ __ (  ) __1
x​   ​   ​ = f (x) ◊ f   ​ x​   ​   ,​ then the polynomial
f(x) = xn + 1
Also, f (x + 2) – f(x) = 4x + 2
Also, f (3) = 28
fi a (x + 2)2 + b(x + 2) + c – ax2 – bx – c
fi 3n + 1 = 28
= 4x + 2 n=3
fi a(x2 + 4x + 4) + b(x + 2) + c – ax2 – bx – c Thus, f (4) = 43 + 1 = 65
= 4x + 2
5. Given f (x + y, x – y) = xy
fi a(4x + 4) + 2b = 4x + 2
Let x + y = u and x – y = v
fi 4a = 4 and b = – 2
1 1
Then x = __​   ​  (u+v) & y = __ ​   ​  (u – v)
fi a = 1 and b = – 1 2 2
f (x) = x2 – x + 1 1
Thus, Therefore, f(u, v) = __ ​   ​  (u2 – v2)
4
1
f (x,y) = __ ​   ​  (x2 – y2)
  (Problems for JEE-Advanced) 4
1
f (y, x) = __ ​   ​  (y2 – x2)
1. Given
1
2 f (x) + 3 f  ​ __ (  )
​ x ​  ​ = x – 3, x π 0 ...(i)
Also,
4

Replacing x by 1/x in (i), we get, f(x, y) + f (y, x)


Now, ​ _____________
 ​    
2
(  )
1
2f ​ ​ __
1
__
x ​  ​ + 3f(x) = ​ x ​ – 3 ...(ii) 1
​ __ ​  (x2 – y2 + y2 – x2) = 0
8
Multiplying (i) by 2 and (ii) by 3 and subtracting (i)
from (ii), we get, 2F(n) + 1
6. Given F (n + 1) = _________
​   ​   
, n Œ  N
2
3
9f (x) – 6f (x) = __ ​ x ​ – 9 – 2x + 6 1
fi F(n + 1) – F(n) = __
​   ​  ...(i)
3 2
fi 3f (x) = __
​ x ​ – 2x – 3

( 
Put n = 1, 2, 3, ..., 2014 in (i). we get,

1 2
f (x) = ​ __
​ x ​ – __ )
​   ​  x – 1  ​
3 F (2) – F (1) = 1/2
2. Given f (x) + 2f(1 – x) = x2 + 2 ...(i)
F (3) – F (2) = 1/2
Replacing x by 1 – x, we get, F (4) – F (3) = 1/2
f (1 – x) + 2f (x) = (1 – x)2 + 2 ...(ii) ... ... ...
Multiplying (ii) by 2 and (i) by 1 and subtracting (i) ... ... ...
from (ii), we get, F(2015) – F(2014) = 1/2
fi 3f (x) = 2(1 – x)2 + 4 – x2 – 2 On addition, we get,
1
fi 3f (x) = x2 – 4x + 4 = (x – 2)2 F(2015) – F(1) = ​ __ ​  × 2014 = 1007
2
1 fi F(2015) = F(1) + 1007 = 1009
fi f (x) = __
​   ​  (x – 2)2
3
3. Let 2x + 3y = u and 2x – 7y = v 7. Now,
1
2 {  (  ) x
f (x) ◊ f (y) – ​ __ ​ ​  f ​ __ }
​ y ​  ​ + f (x y)  ​

= ​ __  ​ ​( 2 f (x) f (y) – f ​( __


​ y ​)​  – f (x y) )​
9u + v u–v 1 x
Then x = ​ ______  & y = ​ _____
 ​   ​ 

20 10 2
Real Function  1.63


1
= __ [ 
​   ​  ​ 2 cos (log x) . cos (log y)
2 (  (  ) (  ) ) (  )
1007
+ ... +​ f  ​ _____
​ 
2015
1008
 ​  ​ + f  ​ _____
​ 
2015
1008
 ​  ​  ​ + f ​ _____
​ 
2015
 ​  ​

(  (  ) )
x
– cos ​ log ​ __ ]
​ y ​  ​  ​ – cos(log(xy))  ​ = 1 + 1 + 1 + ... + 1 (1007 times)

1
[ 
= ​ __ ​  ​ cos (log(xy)) + cos  ​ log​ __
x
​ y ​  ​  ​

(  (  ) ) (  )
1008
+ f ​ _____
​ 
2015
 ​  ​

(  )
2
1008
= 1007 + f ​ _____ ​   ​  ​
– cos ​ log ​ __
x
(  (  ) )
​ y ​  ​  ​ – cos(log(xy))  ​ ] 2015

=0
10. Given (  ) 1
f  ​  x  + ​ __ 2 1
__
x ​  ​ = x + ​ x2  ​ , x π 0

8. Given f (x + 1) + f (x – 1) = 2f (x) (  ) (  )


f  ​ x + __
1
​ x ​  ​ = ​​ x + __
1 2
​ x ​  ​​ ​– 2

Replacing ​( x + __
Put x = 1, f (2) + f (0) = 2 ◊  f (1) fi f (2) = 2f (1)
​ x ​ )​ by y, we get,
1
Put x = 2, f (3) + f (1) = 2 ◊  f (2)
fi f (3) = 2 f (2) – f (1) = 4 f (1) – f (1) = 3 f (1) f (y) = y2 – 2

Put x = 3, f (4) + f (2) = 2 ◊  f (3) fi f (x) = x2 – 2


fi f (4) = 6 f (1) – 2 f (1) = 4 f (1) 11. *
12. 2
Therefore by similarity, we can say that, 13. Given f (x – 1) = x2 – 1
f (n) = n f (1), n Œ N
Replacing x – 1 by x, we get,
4x
9. Given f(x) = ​ _____
x     ​
4 +2 f(x) = (x – 1)2 – 1 = x2 – 2x

( 
41–x
Also, f(1 – x) = _______
​  1–x   
4 +2
 ​ 14. Given
8
f  ​ _______
​  __   __ 
÷    + ÷
​ 1 ​ ​ x 
  ​ 
)
 ​  ​ = x, for all x ≥ 0

Now, f(x) + f(1 – x)


Put x = 9, we get

( 
x 1–x

)
4 4
= ​ _____
   ​ + _______
​    
8
f  ​ _______
​  __   __  ​  ​ = 9
4x + 2 41–x + 2    + ÷   
÷
​ 1 ​ ​ 9 ​
4x 41–x 4x

( 
 ​ = ​ _____
x     ​ + _______
​  1–x    ​ × ​ __x ​
4 +2 4 +2 4
4x 4

÷
​ 1  
8
f  ​ ______
​  _____ )
   ​  
  +  3 ​ 
​= 9
= ​ _____
x     ​ + _______
​      ​
4 + 2 4 + 2.4x fi f (4) = 9
_____
x
4 2 15. The function ÷ ​ is defined for (1 – x) ≥ 0
​ 1  – x  
= ​ ______
    ​ + _____
​      ​
4x + 2 2 + 4x fi x£1
x
4 +2
= ______
​  x  
 ​ fi x Œ (– •, 1]
4 +2
1
=1 Also, the function ​ ______    
_____  ​ is defined for
​ x  + 1 ​
÷  

Now, (  ) (  ) (  )
1
f  ​ _____
2015
2
​     ​  ​ + f  ​ _____
​     ​  ​ + f  ​ _____
2015
3
​     ​  ​
2015


x+1>0
x > – 1
(  ) 2014
+ ... + f  ​ _____
​ 
2015
 ​  ​ fi x Œ(– 1, •)
Thus, domain of the function f
(  (  ) (  ) )
1
_____
= ​ f  ​ ​     ​  ​ + f  ​ ​ 
2015
2014
_____
2015
 ​  ​  ​ = (–1, 1]
_____ _____
​ x  – 2 ​ + ÷
f (x) = ÷ ​ 5  – x  
(  (  ) (  ) )
16. Given ​
2 2013
+ ​ f  ​ _____
​     ​  ​ + f  ​ _____
​   ​  ​  ​ _____
2015 2014 Now, ​÷ x – 2 ​ is defined for x ≥ 2
_____

+ ​( f  ​( _____ ​ + f  ​( _____
2015 ) 2015 ) )
3 2012 ​ is defined for x £ 5
and ​÷ 5 – x  
​     ​   ​   ​  ​  ​
Thus, Df = [2, 5]
1.64  Differential Calculus Booster
_____
17. Here, ​÷ x – 2 ​ is defined for x ≥ 2 23. The function f is defined for

( 
_____
Also, ​÷ 4 – x  
Thus,
​ is defined for x £ 4
Df = [2, 4]

1 + x2
– 1 £ ​ ​ _____
2x
   
  )
​  ​ £ 1

18. Here,
Also
x2 – 3x + 2 = 0 gives x = 1, 2
log (3 – x) is defined for 3 – x > 0
1+x2
fi ​ ​ ____   
2x | 
​  ​ £ 1

|
fi x<3 |1 + x2|
fi ​ ______  £ 1
 ​ 
fi x Π(Х, 3) |2x|
Thus, Df = (–•, 3) – {1, 2}
_____ fi ​| 1 + x2 |​ £ |2x|
19. Here, ​÷ |x| – 1 ​
 is defined for |x| – 1 ≥ 0
fi 1 + ​| x2 |​ £ 2 |x|
fi |x| ≥ 1
fi |x| 2 – 2 |x| + 1 £ 0
fi x Œ (–•, 1] » [1, •)
_____ fi (|x|  –  1)2 £ 0
is defined for 3 – |x| ≥ 0
Also, ​÷ 3  –  |x| ​ 
fi |x| £ 3 fi (|x| – 1)2 = 0
fi – 3 £ x £ 3 fi |x | – 1 = 0
Thus, Df = [ – 3, – 1] » [1, 3] fi x = ± 1
20. The function f is defined for Thus, Df = { – 1, 1}
|​ x |​ – 1
​ _______   ​ ≥ 0 |x| π 4 24. The function f is defined for
4 – |​ x |​
|​ x |​ – 1
fi ​ _______ ​ 
|​ x |​ – 4
£ 0, ​| x |​ π 4
x2
2 (  )
– 1 £ log2 ​ ​ __ ​   ​ £ 1



1 £ |x| £ 4, |x| π 4
x Œ (– 4, – 1] » [1, 4)

x2
(  )
2–1 £ ​ ​ __ ​   ​ £ 2
2

Thus, Df = (– 4, – 1] » [1, 4) 1


fi ​__
x2
(  )
   ​  £ ​ ​ __ ​   ​ £ 2
2 2
21. The function f is defined for
fi 1 £ x2 £ 4
[x] – 1
​ ______   ​ ≥ 0, [x] π 4 __ __ __
4 – [x] fi ​÷ 1 ​  £ ​÷x  2 ​  £ ​÷4 ​
  
[x] – 1
fi ​ ______   ​ £ 0, [x] π 4 fi 1 £ |x| £ 2
[x]  –  4
fi Œ [– 2, – 1] » [1, 2]
fi 1 £ [x] £ 4, [x] π 4
Thus, Df = [– 2, – 1] » [1, 2]
fi 1 £ x < 5, x œ [4, 5)
25. The function f is defined for
fi 1£x<4
log5  ​( log3 ​( 18x – x2 – 77 )​ )​ > 0
fi x Œ [1, 4)
fi ​( log3  (​  18x – x2 – 77 )​ )​ > 1
Thus, Df = [1, 4)
fi (​  18x  – x2 – 77 )​ > 3
22. The function f is defined for
fi ​( 18x– x2 – 80 )​ > 0
– 1 £ log2 (x) £ 1
fi x2 – 18x + 80 < 0
fi 2–1 £ x £ 2
(x – 8) (x – 10) < 0
1
fi ​ __ ​  £ x £ 2
2 fi 8 < x < 10
Thus,
1
Df = ​ __ [  ]
​   ​ , 2  ​
2
Thus, Df = (8, 10)
Real Function  1.65

26. The function f is defined for Case-I:   When x ≥ 1


x
1 – logx log2 (4 – 12) ≥ 0 Also, x a > xb fi a > b
fi logx log2 (4x – 12) £ 1 Thus, (x12 – x9) + (x4 – x) + 1 > 0
fi log2 (4x – 12) £ x Case-II:   when 0 £ x < 1
fi x
(4 – 12) £ 2 x Also, x a < xb fi a > b
Thus, (x9 – x4) + (x – 1) < 0
fi ((2x)2 – 2x – 12) £ 0
So, x12 – (x9 – x4) + (x – 1) > 0
fi (2x – 4) (2x + 3) £ 0
Case-III:  when x < 0
fi – 3 £ 2x £ 4
Also, a > b fi x a < xb
fi 2x £ 4 = 22
Thus, (x9 – x4) + (x – 1) < 0
fi x£2
Therefore, x12 – (x9 – x4) + (x – 1) > 0
Also, log2 (4x – 12) > 0
Thus, the domain of the function is
fi (4x – 12) > 20 = 1
= Df = R
fi 4x > 13
30. Given y = f (x) = x2 + x + 1
fi x > log4 (13)

Thus, Df = (log4 (13), 2]


(  ) 1
2
3
= ​ x + ​ __ ​   2​ + __
​   ​ 
4

27. Given
4
3y + 2​ x​ ​ = 2​ 4x

2 
– 3
​ Thus, [  )
3
Rf = ​  ​ ​__ ​ , •  ​ ​
​4 ​
y = log3  ​( ​24x
​ –3​ – 2​ x​ ​ )​
2 4
fi x2
31. Given y = f (x) = ​ _____   
 ​
x2 + 1
It is defined for (​  ​24x
​ –3​ – ​2x​ ​ )​ > 0
2 4

fi ​2​x ​ < 2​ 4x
​ –3​
4 2
( 
x2 + 1 – 1
= ​ ​  ________
x2 + 1
 ​ 

  ​ )
( 
4 2
fi x < 4x – 3
fi x4 – 4x2 + 3 < 0

1
= ​ 1 – _____
​  2    
x +1
 ​  ​
)
fi (x2 – 1) (x2 – 3) < 0 Here, y Æ 1, x Æ •
__ __
fi (x + 1) (x – 1) ​( x + ​÷3 ​ 
   )​ ​( x – ​÷3 ​ 
   )​ < 0 Thus, Rf = [0, 1)
__ __
fi x Œ ​( – ​÷3 ​
  ,  –1 )​ » ​( 1, ​÷3 ​ 
   )​ x2 1
__ __ 32. Given y = f(x) = ​ _____    ​ = _______
​       ​
4 1
Thus,   ,  –  1 )​ » ​( 1, ​÷3 ​ 
Df = (​  –​÷3 ​    )​ x +1 x + __
2
​  2  ​ 

(  (  ))
x
_____ 1 1
​ 4  – x2  fi y = f(x) = ______
​       ​ £ ​__
   ​ 
f(x) = sin ​  loge ​ ​ ÷______  
 ​ 1 2
28. Given   ​  ​  ​ 2 __
x + ​  2  ​ 
1–x
x
______
Now, ​÷ 4  – x2 

 ​ is defined for x2 < 4
2
x –4<0
Thus, [  ]
1
Rf = ​ 0, __
​   ​   ​
2
x2 + x + 1
fi (x + 2) (x – 2) < 0 33. Given f (x) = ​  __________
    ​
x4 + x2 + 1
fi – 2 < x < 2
(x2 + x + 1)
Also, 1–x>0 = ​  _____________________
   
    ​
(x2 + x + 1) (x2 – x + 1)
fi x<1
1
= _________
​  2      ​
Thus, Df = (– 2, 1). (x – x + 1)
29. Given f (x) = (x12 – x9 + x4 – x + 1) –1/2 Let
1
g(x) = (x2 – x + 1) = ​ x – __ (  ) 3
​   ​   2​ + __
2
​    ​
4
1.66  Differential Calculus Booster

Thus, [  ]
4
Rf = ​ 1, ​ __ ​   ​
3
Applying, A.M ≥ G.M, we get,
y ≥ 2 + 10 = 12
x2 – x + 1 Thus, Rf = [12, •)
34. Given f (x) = ​ _________  
x2 + x + 1
38. Let g(x) = (x2 + 4x + 4) = (x + 2)2
2
(x + x + 1) – 2x Dg = R & Rg = [0, •)
 ​ = ​ ______________
  
    ​ Thus
(x2 + x + 1)
So, – • < log1/2 (x2 + 4x + 4) < •
2x
= 1 – ​ __________
   
2
(x + x + 1) fi 0 < log1/2 (x2 + 4x + 4) < •
2 – • < log2 (log1/2 (x2 + 4x + 4)) < •
 ​ = 1 – __________
​       ​ fi
1
( 
__
​ x + ​ x ​ + 1  ​ ) Thus, Rf = (–•, •)

Let ( 1
g(x) = ​ x + __
​ x ​ + 1  ​ ) 39. Let ( 
cos2 x + 1
g(x) = ​ ​  _________
 ​  
 ​ )
( 
2
g¢(x) = ​ 1 – __
x
1
​  2  ​   ​
) Thus,
1
[  ]
Rg = ​ __
​   ​ , 1  ​
2

[  (  )
Now, g¢(x) = 0 gives x = ± 1 Therefore,
  At the nbd of x = – 1, g¢(x) changes from positive
to negative, so g (x) will provide us the max value
and the max value of g(x) is – 1.

40. we have
2 ]
​ 1  ​  ​, sin–1 (1)  ​ =
Rf = ​ sin–1 ​ __ [​  __​ p6 ​ , ​ __p2 ​  ]​
f (x) = 2x + 3x + 4x + 2– x + 3 – x + 4 – x + 10
  At the nbd of x = 1, g’(x) changes from negetive
to positive, so g(x) will provide us the min value and = (2x + 2–x) + (3x + 3 –x) + (4x + 4 –x) + 10
the min value of g(x) is 3. ≥ 2 + 2 + 2 + 10 = 16, since A.M ≥ G.M
Thus, the range of the function is = ​ __
1
​   ​ , 3  ​
3 [  ] Thus, Rf = [16, •)
_____ _____
35. Given y = f(x) = log2 ​( ​÷x  – 2 ​ + ​÷4  – x  
​ )​ 41. Given f (x) = 3 tan2 x + 12 cot2 x + 5
= (3 tan2 x + 12 cot2 x) + 5
Clearly, domain of the function = [2, 4]
__ 1 1 = (3tan2 x + 12cot2 x) + 5
At    ) = __
x = 2, y = log2 (​÷2 ​ ​   ​   (log2 2) = __
​   ​ 
2 2 ≥ 12 + 5 = 17, since A.M ≥ G.M
__ 1 1
At   )  = __
x = 4, y = log2 (​÷2 ​ ​   ​  (log22) = __
​   ​  Thus, Rf = [17, •)
2 2 __
At x = 3, log2 (2) = 1 42. Given f (x) = ÷     sin x + cos x + 4
​ 3 ​

[  ]
_____
1 ​ 3  + 1 ​
Max value of f (x) = ÷  + 4 = 2 + 4 = 6
Thus, Rf = ​ __
​   ​ , 1  ​
2 _____
1 Min value of f (x) = – ​÷3  + 1 ​
 + 4 = – 2 + 4 = 2
36. Given y = f(x) = x2 + _____
​  2    
 ​
x +1 Thus, Rf = [2, 6]
1
= (x2 + 1) + ​ _______
2
   
 ​ – 1 43. Given ​    
1
f (x) = ________________________
     ​
(x + 1) 2
2 cos x + 4 sin x cos x + 4
Applying, A.M ≥ G.M, we get,
Let g(x) = 2 cos2 x + 4 sin x cos x + 4
y≥2–1=1
= 1 + cos 2x + 2 sin 2x + 4
Thus, Rf = [1, •)
= cos 2x + 2 sin 2x + 5
1
y = f (x) = x + 4x + ___________
4 2 __ __
37. Given ​  4      ​ + 10 Thus, Rg = Î5 + ​÷5 ​   ˚ 
  ,  5 – ​÷5 ​
x + 4x2 + 9

​  4
1
= (x4 + 4x2 +9) + ___________
     ​ + 10
(x + 4x2 + 9)
​ 
1
Therefore, Rf = ​ ________
[ 
 __  
(​  5 – ​÷5 ​ 
) (
1
 ​, ________
​ 
    ​ ​  5 + ÷
 __  
   )​
​ 5 ​ 
 ​  ​
]
Real Function  1.67

44. Given
x2
f(x) = cos–1 ​ ​ ______
1 + x2 (  )
   ​  ​
Therefore,

p p
f (x) = sin–1(1) + cos–1 (0) = __
​   ​  + ​ __ ​  = p
2 2
Let 2
x2
g(x) = ​ _____
x +1
   2
x +1
1
 ​ = ​ 1– ​ _____  
( 
 ​  ​
) Also,
Thus,
f(x) = sin–1(0) + cos–1(–1) = 0 + p = p
Rf = {p}
Thus, Rg = [0, 1) x2 + 2x + 3
47. Given f(x) = ​ __________
x     ,  ​x>0
Now,
p
cos–1(0) = cos–1 ​ cos​ __
2 (  (  ) )
p
​   ​   ​  ​ = __
​   ​ 
2 ( 3
= ​ x + __
​ x ​ + 2  ​ )
–1 –1
and cos (1) = cos (cos(0)) = 0 __
≥ 2 ​÷3 ​
    + 2, since A.M ≥ G.M
p
Thus, Rf = 0, ​ __ ​  __
2 Thus,   ,  • )​
Rf =  ​Î2 + 2​÷3 ​

( 
______

45. Given f(x) = cos–1


  2 + 1 ​ 
÷​ 2x
​ ​  ________
x2 + 1
 ​ 
  ​ ) 48. Given
x2 + x + 2
f(x) = ​ _________
x2 + x + 1
  ​

( ÷ 
____________

–1 2x2 + 1
= sin ​ ​ 1 – ​  _______
    

(x2 + 1)2
 ​ ​  ​
)
(x2 + x + 1) + 1
= ​  ______________
  
  
(x2 + x + 1)
 ​

( ÷ 
1
= 1 + __________

)
________________ ​  2      ​
2 2 2
(x + 1) – 2x – 1 (x + x + 1)
= sin–1 ​ ​ ​ ________________
  
   ​ ​ 
   ​
(x2 + 1)2 Let g(x) = (x2 + x + 1)

(  )
( ÷ 
1 2 3
____________________ = ​​ x + __
​   ​   ​​ ​ + __
)
​   ​ 
4
x + 2x + 1 – 2x – 1 2 2 2 4
= sin ​ ​ ​ ____________________
–1
   
     ​ ​ 
   ​
(x2 + 1)2 4 7
Thus, max value of f(x) = 1 + __
​   ​  = __
​   ​ 

( ÷ 
________ 3 3

x4
= sin ​ ​ ​ _______
–1
(x + 1)2
2
   
 ​ ​  ​
) 7
Therefore, Rf = ​ 2, __ [  ]
​   ​   ​
3


( x2
= sin ​ ​ _______
–1
(x2 + 1)
   ​  ​
) 49. Given
ex
f(x) = ​ ______
  
1 + [x]
 ​

Clearly, Rf = R
x2
Let g(x) = ​ _____  
 ​ {x}
2
x +1 50. Given y = f (x) = _______
​     ​
1 + {x}
Thus, Rg = [0, 1) 1
fi y = 1 – _______
​      ​

​ 2​
p
Therefore, Rf = [sin (0), sin (1)) =  ​​ 0, __
–1
​   ​  ​  ​ –1
[  ) Also,
1 + {x}
0 £ {x} < 1

46. Given [  ] [  ] 1
f (x) = sin–1​ x2 + __
2
1
​   ​   ​ + cos–1​ x2 – __
​   ​   ​
2
fi 1 £ 1 + {x} < 2
1 1
fi ​__
   ​  < _______
Now, ​[ x + __ ​   ​  ]​ = [​  x – __ ​   ​  + 1 ]​ = ​[ x – __
​      ​ £ 1
​   ​  ]​ + 1
1 2 1 2 1 2 2 1 + {x}
2 2 2 1 1
fi – 1 £ – ​ _______ < – ​ __ ​ 
   ​ 
f(x) = sin ​[ x + __ ​   ​  ]​ + cos ​[ x – __
1  +  {x} 2
​   ​  ]​
–1 1 2 1 –1 2
Thus,
2 2 1 1
fi 1 – 1 £ 1 – _______
​  < 1 – __
   ​  ​    ​
1  +  {x} 2
= sin [​  ​( x – ​ __ ​  )​ + 1 ]​ + cos ​[ x – ​ __ ​  ]​
( 
1 1
)
–1 2 –1 2
1 1
2 2 fi 0 £ ​ 1 – _______
​      ​  ​ < __
​   ​ 
1 + {x} 2

It is defined for [​  x – __ ​   ​  ]​ = 0, – 1


1 1
0 £ y < __
2
fi ​   ​ 
2 2
1.68  Differential Calculus Booster

[  )
1 2
Thus, ​ __
Rf = ​  0, ​   ​   ​ ​ sin4 (x) . ​e(x)​ ​
​ 2 ​ = ​ __________    ​  
​ __[  ]
x 1
​ p ​ ​ + __
​   ​ 
51. Given
p
f (x) = 2 cos x + sec2 x, x Œ ​ 0, ​ __ ​   ​
2 (  ) = –  f (x)
2
2
= cos x + cos x + sec  x
Thus, f (x) is an odd function.

(  [ 
Applying, A.M ≥ G.M, we get,
cos x + cos x + sec2 x 3
​ __________________
3
 ​       ≥ ÷  
​ cos x. cos x sec
  
2
 x ​ = 1
______________
54. Given
2x (sin17 x + tan99 x)
f (x) = ​ ​ ________________
x + 21p
2​ ​ _______
p    



  

   ​  ​
– 41 ] )
( 
2
cos x + cos x + sec
fi ​ _________________ ≥1

)
 ​     
3 2x (sin17 x + tan99 x)
= ​ ​ ________________   
   ​  ​
fi 2 cos x + sec2 x ≥ 3
2​ __
x
[ 
​ p ​ + 21  ​ – 41 ]

( 
Thus, Rf = [3, •)


p
52. Given f (x) = 3 tan x + cot3 x, x Œ ​ 0, ​ __ ​   ​
2
= tan x + tan x + tan x + cot3 x
(  )
2x (sin17x + tan99x)
= ​ ​ ________________
2​ __
x
[  ]
  
   ​  ​
​ p ​ ​ + 42 – 41 )
Applying, A.M ≥ G.M, we get,
tan x + tan x + tan x + cot3 x
fi ​ ________________________
   
4
 ​   
___________________

( 
2x (sin17 x + tan99 x)
= ​ ​ ________________
2​ __
x   
[  ]
​ p ​ ​ + 1
 ​ 
   ​
)

4

≥ ​÷tan x ◊ tan  
x ◊ tan x ◊ cot
   
3
 x ​ = 1 Nowt, f ( – x)

(  )
tan x + tan x + tan x + cot3 x
fi ​ ________________________
 ​
       ≥ 1 2 (– x)(sin17(– x) + tan99 (– x))
4 = ​ ________________________
​  x     ​ 
   ​
fi 3tan x + cot  x ≥ 4 3 2​ – ​ __[  ]
p ​ ​ + 1

Thus,

53. Given
Rf = [4, •)

sin4x ​ex​ ​
f(x) = ​ ____________     ​
2

( 
2(x)(sin17(x) + tan99 (x))
= ​ ​ ___________________
– 2​ __
x    
[  ]
  
​ p ​ ​ – 2 + 1
 ​  ​
)
[  ]
( 
+ 3p
x______ 5
​ ​  p    ​  ​ – __
​   ​ 


2
sin4 x ◊ ​ex​ ​
= ​ ___________       ​
2
2 (x)(sin17(x) + tan99 (x))
= ​ ​ ____________________
   
– ​ 2​ __
x
  
(  [  ] )
 ​  ​
​ p ​ ​ + 1  ​ )
( 
[ 
__x
] 5
__
​ ​ p ​ + 3  ​ – ​    ​

)
2 2(x) (sin17(x) + tan99 (x))
= – ​ ​ ____________________
     ​ 
   ​
(  [  ] )
4 x2 x
sin  x ◊ ​e​ ​ ​ 2 ​ __
= ​ ___________       ​ ​ p ​ ​ + 1  ​
[  ]
​ __
x
​ p ​ ​ +  3 – __
5
​   ​ 
2
sin4 x. ​ex​ ​
2
Thus, f (x) is an odd function.
= ​ ________   ​ ______
[  ]
x 1 __
​ __
​ p ​ ​ + __
​   ​  55. Given ​ x  + ÷
f (x) = ÷ ​ x 
   ​ ​ 

( 
2

Now,
sin4 (– x) ◊ ​e(–x)
f (– x) = ​ ____________
​ ​
  
   ​
2
1
f ¢ (x) = ________
​  ______
  __ 
2​÷x  + ÷​ x 
 ​ ​
  
1
 ​ × ​ 1 + ____
​  __
   ​  ​
2​÷x 
  ​  )
[  ]
​ – __
x 1
​ p ​ ​ + __
​    ​
2
>0
2
Thus f (x) is strictly increasing function.
sin4 (x) ◊ ​e(x)
​ ​
= ​ ___________   
    ​ Thus f is one-one function.
[  ]
________
x 1
–​ __
​ p ​ ​ – 1  + ​ __ ​  Also, let y = ​÷x  + ÷
​ x 
   ​ ​ 
2 __
fi y2 = x + ​÷x 
  ​ 
2
__ __
sin4 (x) ◊ ​e(x)
​ ​ fi (​÷x  2
  ​)  + ​÷x 
2
  ​  – y = 0
= ​ __________    ​  
[  ]
x 1 _______
–​ __
​ p ​ ​ – __
​   ​  __ – 1 ± ​÷1  + 4y2 ​ 
2 fi (​÷x  ____________
  ​)  = ​   ​    
2
Real Function  1.69

( 
_______


__
(​÷x 
– 1
   ​) = ​ 
+÷​ 1  + 4y2 ​ 
_____________
2
 ​    
ax + 1
a​ ​ ______ ​  ​  + 1
x+3
____________
fi ​    
   ​= x
)
( 
​ – 1 + ​÷1  + 4y2 
______
 ​  2​ )
ax +1
______
​ ​ 
x+3 ( 
 ​ 
 ​ + 3 )
fi x = ​  _______________
 ​    
4 fi a2 x + a + x + 3 = x (ax + 1 + 3x + a)
Clearly, x is defined for [0, •) fi (a2 + 1) x + a + 3 = (a + 3) x2 + 10x
So it is onto function fi a + 3 = 0 and a2 + 1 = 10
Thus, f is a bijective function. fi a = – 3 and a = ± 3
So, its inverse is exist. a = – 3
______
(​​  1 – ÷​ 1  + 4x  
 ​ )​​
2 ​
2
59. Given f (x + y) + f  (x – y) = 2f (x) ◊ f(y) ...(i)
Hence, f  –1 (x) = ​ ______________ ​
    
4 Putting y = x, we get,
______
1 – 2​÷1  + 4x   2
 ​ + 1 + 4x 2 f (2x) + f (0) = 2(f (x))2 ...(ii)
= ​ _____________________
   
 ​   
______
4 Putting y = – x in (i), we get,
2 – 2​÷1  + 4x2 
 ​ + 4x2 f (0) + f (2x) = 2 f (x) f (– x) ...(iii)
= ​ __________________
 ​      
4
Subtracting (iii) from (ii), we get,

( 
_______


1 – ​÷1  + 4x2 ​  2
= ​ ​ ___________
2
 ​
     + x  ​ )

2( f(x))2 = 2 f (x) ◊ f (–  x)
f (x) (f (x) – f (– x)) = 0
56. Given y = f  (x) = 4x3 – 3x fi f (x) = 0 or f(– x) = f (x)
fi y = f (cos q) = 4 cos3 q – 3 cos q Thus, in both the cases, function is an even function.
fi y = f (cos q) = cos 3 q 60. Given 2 + f (x) ◊ f (y) = f (x) + f (y) + f (xy) ...(i)
Clearly f is one-one and onto 1
Put y = ​ __
x ​, we get,
So, its inverse exist.

Thus,
1
( 
f  –1(x) = cos ​ ​ __ ​  cos–1 x  ​ )
1
(  )
2 + f (x) ◊ f  ​ __
1
​ x ​  ​ = f (x) + f  ​ __ (  )
​ x ​  ​ + f (1) ...(ii)
3
Again put x = 1 in (ii), we get,
57. Here y = f(x) fi x = f  –1(x)
2 + f (1) ◊ f (1) = f (1) + f (1) + f (1)
____________________
3 fi (f (1))2 – 3 f (1) + 2 = 0
Now, y= ​÷a  – x3 + 3x   
2
– 3bx + b3 ​ + b
fi (f (1) – 1) (f (1) – 2) = 0
fi (y – b)3 = a – (x – b)3 fi f (1) = 1 or 2

fi (x – b)3 = a – (y – b)3 But f(1) π 1, thus, f (1) = 2


__________
(x – b) = ​÷a  – (y –  
3
b)3 ​
Thus, (  )
1
f (x) ◊ f  ​ __ (  )
1
​ x ​  ​ = f (x) + f  ​ __
​ x ​  ​

__________
fi f  (x) = xn + 1
x = ​÷a  – (y –  
3
fi b)3 ​ + b
__________ Also, f (2) = 5 fi 2n + 1 = 5
f  –1(x) = ​÷a  – (x –  
3
Thus, b)3 ​ + b
fi n=2
–1
Therefore, f(x) = f   (x) Therefore, f (f (2)) = f(5) = 52 + 1 = 26
So, f (f (x)) = x
61. Since f (x) is symmetrical about the lines
fi b ŒR
x = a and x = b
58. Given f (x) = f  –1(x)
Thus, f (a – x) = f (a + x)
fi f (f (x)) = x
and f (b – x) = f (b + x)
Now, f (x) = f (a + (x – a))
1.70  Differential Calculus Booster

= f (a – (x – a)) fi ((tan–1 x) – 1) ((tan–1 x) – 1) > 0


= f (2a – x) fi x < – tan1 and x > tan1 ...(ii)
= f (b + (2a – x – b)) Again, (tan–1 x)2 > 0
= f (b – (2a – x – b)) fi (tan–1 x) > 0
= f (2b – 2a + x) fi x > 0 ...(iii)
= f (x + (2b – 2a)) From (i), (ii) and (iii), we get,
Thus, f(x) is a periodic with period (2b – 2a) x > tan2
62. The function f is defined for Thus, Df = (tan 2, •)
log1/3 (log7 (sin x + a)) > 0 65. Here, sin2 x + sin x + 1 > 0, which is true for all x
fi 0 < (log7(sin x + a)) < 1 in R.

fi 1 < (sin x + a) < 7 Also,


(  2
1
sin–1 ​ ​ ______
|x – 1|
   
)
 ​  ​ is defined for
fi 1 – sin x < a < 7 – sin x
1
fi 2<a<6 fi – 1 £ ​_______
  2    ​ 
£1
|x – 1|
63. Given
(  1
x1/4( 
f (x) = log2 ​ – log1/2 ​ 1 + ​ ___
) )
   ​  ​ – 1  ​

1
0 < ______
​     
|x2 – 1|
 ​ £ 1

(  1
It is defined for  ​ – log1/2  ​ 1  + ​ ___
(  ) )
   ​  ​  –  1  ​ > 0
x1/4
fi |x2 – 1| ≥ 1 and x2 –1 £ – 1

(  ( 
fi (x2 – 1) ≥ 1 and x2 £ 0
fi ​ log1/2 ​ 1 + ___
1
​  1/4   ​  ​ + 1  ​ < 0
x ) ) fi x2 ≥ 2 and x = 0

( 
__

(  ))
1 fi |x| ≥ ​÷2 ​
    and x = 0
fi ​ log1/2 ​ 1 + ___
​  1/4   ​  ​  ​ < – 1
x __ __
fi x Œ ​( – •, –​÷2 ​
  ˚   ​ » ​ Î​   ,  • )​ » {0}
​ ÷2 ​
1
fi ​ 1 + ​ ___
x1/4
   ​ 
  ( 
​ > ​​
1 –1
__
​ 
2 ) (  )
  ​  ​​ ​ Thus, Df = (​  – •, – ​÷2 ​
__
  ˚   ​ ​ » ​ Î​
__
  ,  • )​ » {0}
​ ÷2 ​

fi ​ 1 + ___
1
( 
​  1/4   ​  ​ > 2
x ) 66. The function f is defined for

4x – ​| x2 – 10x + 9 |​ > 0


1
fi ​___
  1/4   ​ > 1 fi ​| x2 – 10x + 9 |​ < 4x
x
fi – 4x < (x2 – 10x + 9) < 4x
fi 0<x<1
fi (x2 – 10x + 9) < 4x
Thus, Df = (0, 1)
and – 4x < (x2 – 10x + 9)
64. The function f is defined for
fi x2 – 14x + 9 < 0 and x2 – 6x + 9 > 0
log3 (log4 (tan–1 x)2) > 0 ___ ___
fi    ) x – (7– ​÷40 ​
x – (7 + ​÷40 ​    ) < 0
–1  2
fi (log4 (tan x) ) > 1
and (x – 3)2 > 0
–1  2
fi (tan x) > 4 ___ ___
fi x Œ (7 – ​÷40 ​
   , 7 + ÷    ) – {3}
​ 40 ​
​( (tan–1x) + 2 )​ ​( (tan–1 x) – 2 )​ > 0
___ ___
fi Thus,    , 7 + ​÷40 ​
Df = (7 – ​÷40 ​    ) – {3}
fi x < – tan2 and x > tan2 ...(i) 67. The function f is defined for
Also, log4 (tan–1 x)2 > 0 sin–1 |sin x| ≥ cos–1 (cos x)
fi (tan–1 x)2 > 1
Real Function  1.71

Ï p fi ​( 3x–1 + 5x–1 + 7x–1 )​ ≥ ​( 32 + 52 + 72 )​


Ôx :0 £ x £
2 fi x–1≥2
Ô
p
Ôp – x : £ x £ p
Ô fi x≥3
–1 2
Now, sin |sin x| = Ì
Ô x - p : p £ x £ 3p Thus, Df = [3, •)
Ô 2 71. The function f is defined for
Ô 3p
Ô2p - x : £ x £ 2p 3 – 2x – 21–x ≥ 0
Ó 2
fi 3.2x – (2x) 2 – 2 ≥ 0
Ïx :0 £ x £p
Also, cos–1 (cos x) = Ì fi (2x)2 – 3.2x + 2 £ 0
Ó2p - x : p £ x £ 2p
fi (2x – 1) (2x – 2) £ 0
From the above relation, it is clear that

[  ] [  ]
fi 1 £ 2x £ 2
p 3p
​   ​   ​ » ​ ___
x Œ ​ 0, __ ​   ​ , 2p  ​
2 2 fi 20 £ 2x £ 21

​   ​  ]​ » ​[ ___
D = [​  0, __ ​   ​ , 2p ]​
p 3p fi 0£x£1
Thus, f
2 2
Thus, Df = [0, 1]
68. The function is defined for 72. The function f is defined for

fi ( 
3x – x
log0.5 ​ ​ ______ ​ 
x–1
2
3x – x
)
 ​ ≥ 0 and ​ ​ ______ ​ 

x–1

 ​ > 0 (  2
)

(1 – log7 (x2 – 5x + 13)) > 0
log7  (x2 – 5x + 13) < 1
3x – x
fi ​ ​ ______ 
x–1 (  x – 3x 2
 ​  ​ £ 1 and ​ ​ ______ ​ 
x–1 )
 ​ < 0
  (  2
) fi (x2 – 5x + 13) < 7
fi (x2 – 5x + 6) < 0
2 x(x – 3)
x – 2x – 1
fi ​ __________
  
 ​ ≥ 0 and ________
​   ​ 

<0 fi (x – 2) (x – 3) < 0
x–1 (x – 1)
__ __ fi 2<x<3
fi   ,  1 )​ » ​​ Î1 + ÷
x Œ ​​ Î1 – ​÷2 ​   ,  • )​
​ 2 ​
Thus, Df = (2, 3)
and x Œ (– •, 0) » (1, 3)
__ __ 73. The function f is defined for
Thus,   ,  0 )​ » ​ Î1
Df =  ​Î1 – ​÷2 ​   ,  3 )​
​ + ​÷2 ​
24 – [x2] ≥ 0 and |x| – 4 ≥ 0
69. The function f is defined for
fi [x2] £ 24 and |x| ≥ 4
( 
log5 ​ log3 ​( log2 ​( 2x3 + 5x2 – 14x )​ )​  ​ > 0 ) fi 24 £ x2 < 25 and |x| ≥ 4
__
( 
fi ​ log3 ​( log2 ​( 2x3 + 5x2 – 14x )​ )​  ​ > 1 ) ___
fi ​÷ 24 ​ £ ​÷x  2 ​  < ​÷25 ​
___
   and |x| ≥ 4
__
fi ​( log2 ​( 2x3 + 5x2 – 14x )​ )​ > 3 fi     £ |x| < 5 and x Œ (– •, – 4] » [4, •)
2 ​÷6 ​
__ __
fi (​  2x3 + 5x2 – 14x )​ > 8 fi x Œ ​( – 5, – 2​÷6 ​
  ˚   ​​ » Î2​÷6 ​
  ,  5)

(​  2x3 + 5x2 – 14x – 8 )​ > 0 and x Œ(– •, – 4] » [4, •)



fi (x – 2) (x + 4) (2x + 1) > 0 Thus, Df = (–5, –4] » [4, 5)

( 
74. Clearly, Df = {–1, 1}

1
x Œ ​ – 4, – ​ __  ​  ​ » (2, •)
2 ) Thus,
p
Rf = ​ __ {  }
​   ​   ​
2
Thus,
1
2 ( 
Df = ​ – 4, – ​ __  ​  ​ » (2, •)

70. The function f is defined for


) 75. Given (  )
x2
y = f(x) = cos–1 ​ ​ _____
1 + x2
  
 ​  ​

fi 3x–1 + 5x–1 + 7x–1 – 83 ≥ 0 2


x2
Clearly, range of the function ​ ​ _____
x +1
  (  )
 ​  ​ is [0, 1)

fi 3x–1 + 5x–1 + 7x–1 ≥ 83 fi ​ __


y Œ ​  0,
p
(  ]
​   ​   ​ ​
​ 2 ​
...(i)
1.72  Differential Calculus Booster

( 
_____
Also, 0 £ y £ p
From (i) and (ii), we get,
...(ii) ÷​ 4  – x2 
1–x
 ​
81. Since the range of ​ ​ ______ ​  
  ​ is R )
(  ]
p
y Œ ​ ​ 0, ​ __ ​  ​  ​ Thus, Rf = [– 1, 1]

(  )
​ 2​ 1 2 3
82. We have, x2 + x + 1 = ​​ x + ​ __ ​   ​​ ​ + __
​    ​
Thus, (  ]
Rf =  ​​ 0, __
​ 2​
p
​   ​  ​  ​
Also, the function is defined for
2 4

76. Given x2 + 1 
y = f(x) = sin–1 ​ ​ ______
x2 + 2
 ​  

(  ) fi x2 + x + 1 £ 1
3
Therefore, __
​   ​  £ x2 + x + 1 £ 1
4

(  ) ÷ 
__
x2 + 1 ________
__
Clearly, the range of the function ​ ​ _____   ​  ​ 3
fi ​ __
​    ​ ​  £ ​÷x  2 + x + 1 ​
  £ ​÷1 ​
  
x2 + 2 4

[  ) ÷​  2  ​ ​ £ ​÷x  + x + 1 ​ £ 1


__ _________
1
is ​  ​ ​__ ​ , 1  ​ ​ 3
fi ​ __  
2

​2 ​

Thus,
​ [  (  )
​ 1 ​   ​,  sin–1(1) ​  ​
Rf = ​ ​ sin–1 ​ __
2 ​ ) fi sin ​( ​ __
÷​  2 ​ ​  )​ £ sin ​( ​÷x  + x + 1 ​ )​ £ sin
3 –1 
__

–1
_________
2

–1
(1)

[  ) p p
=  ​​ __
​   ​ , __
​ 6 2​
​   ​  ​  ​ p
   ​  £ sin–1 ​( ​÷x  2 + x + 1 ​  
fi ​__
3
_________
)​ £ __​ p ​ 
2
D = ​[ – __
3 3]
p p
77. Clearly, f ​   ​ , __
​   ​   ​

p
(  )
log ​ __ (  (  _________
​   ​   ​ £ ln ​ sin–1​ ​÷x  2 + x + 1 ​  
3
) 2 (  )
)​  ​ £ log ​ __​ p ​   ​
R = ​[ tan​( – __ ​   ​  )​, tan​( __
​   ​  )​ ]​
R = ​[ log ​( __
p p
​   ​  )​, log ​( __
​   ​  )​ ]​
Thus, p p
f
3 3 Thus, f
__ __
3 3
= Ζ ​÷3 ​   ˚ 
  ,  ​÷3 ​
83. Clearly, 5x2 – 8x + 4 > 0, " x Œ R
78. Clearly, Df = ​ – __
p p
​   ​ , __
4 4 [  ]
​   ​   ​
Also,
4 2 4
(  )
5x2 – 8x + 4 = 5 ​​ x – ​ __ ​   ​​ ​ + __
​   ​ 

[  (  ) (  ) ]
p p 5 5
Thus, Rf = ​ sin ​ – ​ __ ​   ​, sin ​ __​   ​   ​  ​
4 4 Thus, the range of log (5x2 – 8x + 4) is

÷
1__ ___
= ​ – ​ ___
   ÷
​ 2 ​ [  1
  ​,  ​  __  ​  ​
​ 2 ​   ] (  (  ) )
4
= ​ log ​ __
​   ​   ​,  •  ​
5
79. The function f is defined for
Hence, Rf = [–1, 1]
(4x – x2) > 0
84. We have (x4 – 2x2 + 3) = (x2 – 1)2 + 2 ≥ 2
fi (x2 – 4x) < 0
Let g (x) = (x4 – 2x3 + 3)
fi x(x – 4) < 0
and h(x) = log (x4 – 2x2 + 3)
fi 0<x<4
Thus, Df = (0, 4) Now, Rg = [2, •)
and Rh = (log0.5 (•), log0.5 (2)] = (–•, –1]
Therefore, Rf = (log2 (0), log2 4) = (–•, 2).
Also, Range of cot–1 x is (0, p)
80. Given 2
+e
y = f(x) = log ​ ​ x_____  ​  ​ (  x +1
2

) and cot–1 x is a decreasing function.


p
[  )
( 
Thus, Rf = [cot–1(–1), p) =  ​​ 3__
​   ​ , p ​  ​

x2 + 1 + e – 1
= log ​ _____________
​ 
x2 + 1
  
 ​ 
   ​ ) 85. Given
​ 4 ​

___________________________

( 
___________________
____________

)
____

e–1
= log ​ 1 + ​ _____   ​  ​ y = f(x) = ÷
​ 1 – cos x       
​÷1 – cos x ​      
÷1 – cos x ​ ​ ​ 
÷... • 
    

x2 + 1
= (1 – cos x)1/2 ◊ (1 – cos x)1/4 ◊ (1 – cos x)1/8...
Thus, Rf = (0, 1]
Real Function  1.73
b
1 1 1 1
__
​    ​ + ​ __  ​ + ​ __  ​ + ​ ___  ​  ... to • = ​ _______
   
______  ​
= (1 – cos x​)​2 4 8 16 ​ ÷​ 1  + b2 
 ​
1
__
​    ​ x
________
2
____
​     ​  ​  ______
     ​
1
1 – ​ __ ​  ÷
​  
1 +
=​  ____________
2x 2
  ​

÷ 
= (1 – cos x​)​ 2 ​ __________       ​
2
x
​ 1 + ​ ______   
    ​ ​
= (1 – cos x) 1 + 2x2
x
= ________
(  )
​  ______      ​
​ x ​   ​
= 2 sin2 ​ __ ÷​ 1  + 3x2   ​
2
Hence, the result.
Thus, Rf = [0, 2]
88. The function f is defined for
86. Given f (x) = ax + b
log10 a – log10 (4 – a) – log10 3 ≥ 0
Now, f (f (f (x)))
fi log10 a – log10{3(4 – a)} ≥ 0
= f (f (ax + b))
= f (a(ax + b)) + b) fi ​ 
( a
log10 ​ ________
   
3 (4 – a) )
 ​  ​ ≥ 0

( 
= f((a2 x + ab + b))
2
= a(a x + ab + b) + b
a
fi ​ _______
​     
3(4 – a)
 ​  ​ ≥ 1 )
= (a3x + a2b + ab + b)
3 2
​ 
a
fi ​ _______
   
12 – 3a ( 
 ​  –  1  ​ ≥ 0 )
( 
Thus, (a x + a b + ab + b) = 27 x + 26

fi a3 = 27, a2b + ab + b = 26
a – 12 + 3a
fi ​ ​ ___________
  
12 – 3a
​  ​ ≥ 0
    )
fi ​( ​ _______ 
3a – 12 )
fi a = 3, 9 b + 3 b + b = 26 4a –12
 ​  ​ £ 0

fi a = 3, 13 b = 26

fi ​( ​ _____ 
a – 4)
fi a = 3, b = 2 a–3
 ​  ​ £ 0

Therefore, a2 + b2 + 2
=9+4+2 fi 3£a<4
= 15 fi 3 £ log10 x < 4
x
87. Given f (x) = _______
​  _____
    ​ fi 103 £ x < 104
÷​ 1  + x2 
 ​

(  ( 
Thus, Df = [103, 104)
Now,
x
f (f (f (x))) = f ​ f ​ _______
​  _____
    ​  ​  ​
÷​ 1  + x2 
 ​ )) 89. The function f is defined for

= f (f (a)), a = ​ _______ _____


2
   
 ​ log1/2 (log2 [x2 + 4x + 5]) ≥ 0
÷​ 1  + x2   ​ fi (log2 [x2 + 4x + 5]) £ 1

(  a
= f ​ ​ _______     ​  ​
)
 ( ÷  )
______
fi ([x2 + 4x + 5]) £ 2
÷​ 1  + a2   ​
x
_______ fi 2 £ (x2 + 4x + 5) < 3
​  _____
     ​
​ 1  + x2 
÷
___________  ​ when 2 £ (x2 + 4x + 5)
= f  ​​  _________      ​  ​
x2
_____ fi (x2 + 4x + 5) ≥ 0
​ 1 + ​         ​ ​
1 + x2


(   x
= f ​ ________
​  ______
     ​  ​
÷​ 1  + 2x2 
 ​ )


(x + 1) (x + 3) ≥ 0
Œ (–•, –3] » [–1, •)

x when (x2 + 4x + 5) < 3


= f (b), b = ________
​  ______
     ​
÷​ 1  + 2x2 
 ​ fi (x2 + 4x + 2) < 0
1.74  Differential Calculus Booster
__ __
fi ​( x + 2 + ÷    )​ ​( x + 2 – ÷
​ 2 ​     )​ < 0
​ 2 ​  fi 4(y + 1)2 –  4 (2y – a) (y – 1) ≥ 0
__ __
fi – ​( 2 + ​÷2 ​ 
   )​ < x < (​  – 2 + ÷    )​
​ 2 ​  fi y2 – (a + 4) y + (a – 1) £ 0
Since the function is onto,
Thus, Df
__ __ so, Rf = Cf = [–1, 2]
= ​( –​( 2 + ​÷2 ​     )​
   )​, –3˚ » Ζ1, –2 + ​÷2 ​ 
when y = 2, 4 – 2(a + 4) + (a – 1) £ 0
90. Given f (g(x)) = 1 + x2 – 2x3 + x4
fi 4 – 2a – 8 + (a – 1) £ 0
= 1 + (x4 – 2x3 + x2) fi – a – 5 £ 0
= 1 + x2 (x2 – 2x + 1) fi a≥5
when y = – 1, 1 + (a + 4) + (a – 1) £ 0
= 1 + x2 (x – 1)2
fi 2a + 4 £ 0
= 1 + (x (x – 1))2 fi a £ – 2
= g(x) = ± (x (1 –  x)) Thus, Œ [–  5, –  2]

p
91. Given f(x) = sin2 x + sin2 ​ x + __
​   ​   ​
3 (  ) 93. Given

|||x2 – x + 4| – 2| – 3| = x2 + x – 12
|||x2 – x + 2| – 3|| = x2 + x – 12
+ cos x ◊ cos ​ x + __
p
(  )
​   ​   ​ fi
3 |x2 – x – 1| = x2 + x – 12

[ 
fi x2 – x – 1 = x2 + x – 12
1
= __
2
p
​   ​   ​ 2sin2 x + sin2 ​ x + __(  ) p
​   ​   ​  + 2cos x . cos ​ x + __
3
​   ​   ​  ​
3 (  ) ]
fi 2x = 11

​   ​ ​ [ 1 – cos 2  x + 1 – cos 2 ​( x + __


p
​   ​  )​
1 fi x = 11/2
= __  
2 3
Hence, the value of x is 11/2.
  + cos ​ 2x + __
p
(  p
​   ​   ​ + cos ​ __
3 ) (  ) ]
​   ​   ​  ​ 94. Given
3 f (x) = x3 + (a + 2)x2 + 3ax + 5

1 __
= __
2 2
5
(  2p
(  )
  ​   ​  – cos2 x –  cos ​ 2x + ​ ___ ​   ​  + cos ​ 2x + __
​   ​  ​
3
p
(  ) )
​   ​   ​  ​
3
Since the function is one-one, so is monotonic.
Now, f ¢ (x) = 3x2 + 2(a + 2) x + 3a > 0

​   ​  ​( __
​   ​  – cos 2 x + cos ​( 2x + __​   ​  )​  – cos​( 2x + ​ ___ ​  )​ )​
1 5 p 2p
= __ fi 4(a + 2)2 – 36 < 0
2 2 3 3

​   ​  ​( __
​   ​  – cos 2 x + 2 sin (​  2x + __ ​   ​  )​ sin ​( __
fi (a + 2)2 – 9 < 0
​   ​  )​ )​
1 5 p p
= __
2 2 2 6 fi (a + 2)2 – 32 < 0

​    ​ ​( __
​   ​  – cos 2 x + 2 sin ​( 2x + __ ​    ​ )​
p
​   ​  )​ × __
1 5 1
= __ fi (a + 2 + 3) (a + 2 – 3) < 0
2 2 2 2

​   ​  ​( __
fi (a + 5) (a – 1) < 0
​   ​  – cos 2 x + sin ​( 2x + __
​   ​  )​ )​
1 5 p
= __ fi – 5 < a < 1
2 2 2

​    ​ ​( __
​   ​  – cos 2 x + cos 2 x )​
1 5 95. Given f (x) = g(x) + 1
= __
2 2
Also, f (x) + f (1 – x) = 2
5
= __
​   ​  fi g(x) + 1 + g (1 – x) + 1 = 2
4
Now,
5
g (f(x)) = g ​ __
4 (  ) 5 1 __
​   ​   ​ = __
​   ​  + __
4 4 2
3
​   ​  = ​    ​ fi g(x) + g (1 – x) = 0 ...(i)

a x2 + 2x + 1 Replacing x by (1 – x) in (i), we get,


92. Let y = ​ ____________
  
  
2x2 – 2x + 1 g(1 – x) + g(– x) = 0 ...(ii)

 ​ fi 2x2 y – 2xy + y – ax2 – 2x – 1 = 0 From (i) and (ii), we get,



fi (2x2 – a) y – (2y + 1) x + (y – 1) = 0 g(x) – g(– x) = 0

since x Œ R, so D ≥ 0 fi g(x) = g (– x)


Real Function  1.75

Thus,  (x) is symmetrical about the line x = 0. From (i) and (ii), we get,
96. Given f (2 + x) = f (2 – x) fi (f(x + 2a) – b)3 – (f (x) – b)3 = 0
and f (7 – x) = f (7 + x) fi (f (x + 2a) – b)3 = (f(x) – b)3
Now, f (x) = f (2 + (x – 2)) fi (f(x + 2a) – b) = (f(x) – b)
= f (2 – (x – 2)) fi f(x) = f(x + 2a)
= f (4 – x) Thus f (x) is periodic with period 2a.
= f (7 + (4 – x – 7)) 98. We have 4x – 2x+2 + 5 + ||b –1|  – 3| = |sin y|
= f (7 – (4 – x – 7)) fi (4x – 4.2x + 4) + 1 + ||b – 1| –3| = |sin y|
= f (10 + x) fi (2x – 2)2 + 1 + || b – 1| – 3| = |sin y|
Thus, f (x) is periodic with period 10. L.H.S ≥ 1 & R.H.S £ 1
Therefore, the number roots of f (x) = 0 is 5. Thus, (2x – 2) = 0, |b – 1| – 3 = 0
97. We have f (a + x) – b fi |b – 1| – 3 = 0
= {1 + (b – f(x))3}1/3 fi |b – 1| = 3
3 3
fi ( f (a + x)– b) = 1 + (b – f (x)) fi (b – 1) = ± 3
3 3
fi ( f (a + x) – b) + ( f (x) – b) = 1 ...(i) fi b = – 2, 4
Replacing x by x + a, we get, Hence, the values of b are – 2, 4
( f (x + 2a) – b)3 + (f (x + a) – b)3 = 1 ...(ii)

Hints and solutions

(Tougher Problems for JEE-Advanced)

1. We have
log3 (log4/p (tan–1 x) –1) > 0

fi (log4/p (tan–1 x) –1) > 0


4
(tan–1 x) –1 > ​ __ (  )
​ p ​ ​


p
(tan–1 x) < ​ __(  )
​   ​   ​
4
x<1
1
Thus, 0, ______  ​ £ 1
​  2    
Also, f (x) is defined only when x > 0 |x – 1|
Thus, 0<x<1 fi |x2 – 1| ≥ 1
__ __
x Œ (0, 1) fi x Œ ​( – •, – ​÷2 ​
  ˚   ​ » ​ Î​   ,  • )​ » {0}
​ ÷2 ​
1 3. f is defined for
2. f is defined for – 1 £ ​ ______      ​ £ 1 and
|x 2 – 1| 4x – |x2 – 10x + 9| > 0
sin2 x + sinx + 1 > 0 is true for every x in R. fi |x2 – 10x + 9| < 4x ...(i)
1.76  Differential Calculus Booster

3x – x2
( 
Now, ​ ​ ______ ​  
x–1
 ​ £ 1 )
3x – x2
( 
fi ​ ​ ______ ​ – 1 
x–1
  ​£ 0
  )
x–1 ( 
3x – x2 – x + 1
fi ​ ​ ______________
 ​ 
     ​£ 0 )
2x – x2 + 1
x–1
  ( 
fi ​ ​ __________ ​£ 0
 ​   )
x2 – 2x – 1
x–1
  ( 
fi ​ ​ __________ ​£ 0
 ​   )
__ __
We draw y = x2 – 10x + 9 and y = 4x fi   ,  1 )​ » ​( 1 + ÷
x Œ ​​  Î1 – ​÷2 ​   ,  1˚ ​
​ 2 ​ ...(i)

and solving x2 – 10x + 9 = ± 4x


we get, x = 7 ± 4 ​÷3 ​
  ,  3
__
3x – x2
x–1 ( 
Again ​ ​ _______ ​  
 ​> 0 )
since |x2 – 10 x + 9| < 4x
__ __ x–1( 
x(x – 3)
fi ​ ​ _______ ​   
 ​ < 0 )
so x Œ (7 – 4​÷3 ​
  ,  7 + 4​÷3 ​
   ) – {3}
__ __ x Œ (– •, 0) » (1, 3) ...(ii)
Thus,   ,  7 + 4​÷3 ​
Df = (7 – 4​÷3 ​   )  – {3}
From (i) and (ii), we get,
4. f is defined for __ __
  ,  0 )​ » ​( Î1 + ​÷2 ​
x Œ​​  Î1 – ​÷2 ​   ,  3 )​
–1 –1
sin (|sin x|) – cos (|cos x|) ≥ 0 __ __
   , 0 )​ »  ​Î1 + ​÷2 ​
Df =  ​Î1 – ​÷2 ​   ,  3 )​
fi sin–1 (|sin x|) ≥ cos–1 (|cos x|)
7. f is defined for
It is possible only when
(1 – log10 (x2 – 5x + 16)) > 0
[  ] [  ]
p
x Œ ​ 0, __
2
3p
​   ​   ​ » ​ ___
​   ​ , 2p  ​
2 fi log10 (x2 – 5x + 16) < 1
fi (x2 – 5x + 16) < 10
​   ​  ]​ » ​[ ___
D = ​[ 0, __ ​   ​ , 2p ]​
p 3p
Thus, f fi (x2 – 5x + 6) < 0
2 2
fi (x – 2) (x – 3) < 0
5. f is defined for fi 2<x<3
2
2{x} – 3{x} + 1 ≥ 0 Thus, Df = (2, 3)
(2{x} – 1)({x} – 1) ≥ 0 8. f is defined for
(2{x} – 1) £ 0, ({x} – 1) ≥ 0 – log 0.3 (x – 1) ≥ 0, (x – 4)(x + 2) < 0
1
{x} £ __
​   ​ , {x} ≥ 1 (x – 1) > 0 and x π – 2, 4
2
1
__ fi x ≥ 2, –2 < x < 4, x > 1, x π – 2, 4
{x} £ ​   ​ , since {x} ≥ 1 is not possible
2 fi x Œ [2, 4)
Thus, [ 1
2 ] [  ] 1
x Œ ​ –1, – ​ __ ​   ​ » ​ 0, __
​   ​   ​ » {1}
2
fi Df = [2, 4)

÷ 
________

D = ​[ –1, ​ __ ​  ]​ » ​[ 0, __


​   ​  ]​ » {1}
x–1

1 1 9. Given f (x) = ​ ​ _______
   ​  ​

f
2 2 x – 2{x}
x–1
6. f is defined for f is defined for ​ _______
   ​ ≥ 0

(  ( 
x – 2{x}

3x – x2
log0.3 ​ ​ ______ ​ 
x–1
  )
3x – x2
 ​ ≥ 0 and ​ ​ ______ ​ 
x–1

 ​ > 0 ) case-I:  when (x – 1) ≥ 0 and (x – {2x}) > 0
x ≥ 1 and x > 2{x}
3x – x2
​ ​ ______ ​ 
x–1
  (  3x – x2
 ​ £ 1 and ​ ​ ______ ​ 
x–1
  )
 ​ > 0 (  )

x≥2
case-II:  (x – 1) £ 0 and (x – 2{x}) < 0
Real Function  1.77

x £ 1 and x < 2{x} 1 1


14. Given f (x) = ​ ___  ​ + log(2{x}–5) (x2 – 3x + 10) + _______
​  _____
   ​ 
[x] ÷   –  |x| ​ 
​ 1  
x £ 1 and x π 0
x Œ (– •, 0) » (0, 1] f is defined for

Hence, the domain of the given function R – [0, 1), |x| < 1, 2 {x} – 5 > 0, π 1
5
= (– •, 0) » (0, 1] » [2, •) R – [0, 1), – 1 < x < 1, {x} > ​ __ ​ , π 3
2
10. Given f (x) = log[​x​2​] (4 – |x|) Thus, Df = j.

f is defined for 15. f is defined for

(4 – |x|) > 0, [x2] > 0 and [x2] π 1 (i) sin (cos x) ≥ 0

– 4 < x < 4, [x2] > 0 and [x2] π 1 (ii) – 2 cos2 x + 3 cosx – 1 > 0

– 4 < x < 4, [x2] = 2, 3, 4, ...


2sin_____
x+1
(iii) – 1 £ ​ _________ £1
 ​ 

– 4 < x < 4, [x ] ≥ 2 2  
2​÷2sinx 

– 4 < x < 4, x2 ≥ 2 Now, from (i), we get,


__ __
– 4 < x < 4, x ≥ ​÷2 ​
    and x £ –​÷2 ​
   0 £ cosx £ p
__ __
Thus, Df = (​  – 4, –​÷2 ​   ,  4 )​
  ˚   ​ » ​ Î​÷2 ​ as p >1

( 
[x 2 – 3]
11. Given f (x) = cos –1 ​ ​  _______
5
 ​ 
  )
 ​ + log2 (|x| – 1)


0 £ cosx £ 1

( 
p
2np £ x £ ​ 2np + __ )
​   ​   ​, n Œ I ...(1)
f is defined for 2

( 
[x2 – 3]
– 1 £ ​ ​ _______
5 )
 ​ £ 1, (|x| – 1) > 0
 ​  
Also, from (ii), we get,
– 2 cos2 x + 3 cos x – 1 > 0
fi – 5 £ [x2 – 3] £ 5, |x| > 1 2 cos2 x – 3 cos x + 1 < 0


– 5 £ [x2] – 3 £ 5, |x| > 1
2
– 2 £ [x ] £ 8, |x| > 1
(  1
​ cosx – ​ __ ​   ​ (cosx – 1)
2 ) <0
1
fi – 2 £ (x2) < 9, |x| > 1 ​ __ ​  < cosx < 1
2
p p
fi – 3 < x < 3, x < –1, x > 1 2np – __​   ​  < x < 2np + __
​   ​ , n Œ I ...(2)
3 3
Thus, Df = (–3, –1) » (1, 3) Again from (iii), we get,
__________ ____
12. Given f (x) = ÷   – 1 +  x2 ​ f is defined for
​ [x]    x + 1 ≥ 0
2 sin x + 2 ​÷2sin ​
_____
[x] – 1 + x2 ≥ 0   ​+ 1 £ 0
and 2 sin x – 2​÷2sinx 

_____ _____
fi [x] ≥ 1 – x2   ​ + 1)2 ≥ 0 and (​÷2 sinx 
(​÷2sinx 
    ​ – 1)2 £ 0

fi 1 – x2 £ [x] It is possible only when sinx = __


1
​   ​ 
__ 2
It is possible only when ÷   
​ 3 ​
__ ___
cos x = ​   ​ 
x ≥ 1 and x £ –​÷3 ​
   2
__ p
Thus, Df = ​( – •, –​÷3 ​
  ˚   ​ » [1, •) x = 2np ± __ ​   ​ , n Œ I ...(3)
_____________ 6
13. Given f (x) =   x)  –  (cos
÷​ (sin    x) ​ From (1), (2) and (3), we get,
f is defined for (sin x) – (cos x) ≥ 0 p
x = 2np + __
​   ​ , n Œ I
6
(sin x) ≥ (cos x)
p
Thus, Df = 2np + __​   ​ , n Œ I
3p
It is true for x Œ ​ 0, ___
2 (  )
​   ​  ​
16. Given
6
___________________
​ 1  – logx log   
f (x) = ÷ x
2 (4 – 12) ​
Thus, (  3p
Df = ​ 2np, 2np + ___
​   ​   ​, n Œ I
2 ) f is defined for
1.78  Differential Calculus Booster

(i) (4x – 12) > 0 Again, from (iii), we get,


(ii) log2 (4x – 12) > 0 [x2 + 4x + 5] > 0
(iii) logx (log2 (4x – 12)) £ 1 x2 + 4x + 5 ≥ 1
From (i), we get, x2 + 4x + 4 ≥ 0
4x > 12 (x + 2)2 ≥ 0
x > log4 (12) ...(1) which is true for all x in R. ...(3)
Also, from (ii), we get, From (1), (2) and (3), we get,
__ __
x
4 – 12 > 1 x Œ ​( – 2 – ​÷2 ​
  ,  – 3˚ ​​ »  Ζ 1,
​​    )​
– 2 + ​÷2 ​ 
__ __
4x > 13 Thus, Df = (​  –2 –​÷2 ​
   , –3˚ ​​ »  Ζ1,
​​    )​
–2 + ​÷2 ​ 
x > log4 (13) ...(2) x2 – 3x + 2
18. Let y = ​ __________
   ​ 
Again, from (iii), we get, x2 + x – 6
log2 (4x – 12) £ x (x – 1)(x – 2) (x – 1)
fi y = ____________
​    
   ​ = ​ ______  ​, x π 2
(x + 3)(x – 2) (x + 3)
(4x – 12) £ 2x
3y + 1
(4x – 2x – 12) £ 0 x = ​ ______ ​

1–y
((2x​)2​ ​ – 2x – 12) £ 0 1
Also, when x = 2, then y = __​   ​ 
(2x – 4)(2x – 3) £ 0 5
3 £ 2x £ 4 1
Hence, the range is R – ​ 1, ​ __  ​  ​.
5 {  }
log2 (3) £ x £ log2 (4) ...(3)
19. Clearly, the domain of the given function is [0, •)
From (1), (2) and (3), we get,
x Œ [log4 (13), 2]
Thus, Rf =  ​  f(0), ​  

lim ​ f (x)  ​
x Æ • )
Thus, Df = [log4 (13), 2]


____________________
​ 
17. Given f (x) = ÷ 2
log1/2  log2 [x   
+ 4x + 5] ​

​ [  )
p 2
=  ​​ 2, ___
​   ​  ​  ​
4 ​
_________________
f is defined for 20. Let y = ​÷a  2  cos2 x  + b2    
sin2  x ​
________________
(i) log1/2 (log2 [x2 + 4x + 5]) ≥ 0 ​ a  2sin2 x + b2    
+÷ cos2  x ​
(ii) log2 [x2 + 4x + 5] > 0 __ ___________
2
fi y = ​÷l    2 + b2)   
  ​  + ​÷(a – l ​, where
(iii) [x + 4x + 5] > 0
Now from (i), we get, l = a2 cos2 x + b2 sin2 x
_____________
(log2 [x2 + 4x + 5]) £ 1 fi  
y2 = l + (a2 + b2) – l + 2​÷l( 2
+ b2)   
– l2 ​
_____________
[x2 + 4x + 5] £ 2
fi y2 = (a2 + b2) + 2​÷  l(a2 + b2)   
– l2 ​
x2 + 4x + 5 < 3
Max value of

( 
x2 + 4x + 2 < 0
)

a2 + b2
__ __ y2 = 2(a2 + b2), when l = ​ ​ _______
 ​  
 ​
x Œ (– 2 –​÷2 ​
  ,  – 2 + ​÷2 ​
  )  ...(1) 2
Min value of
Also, from (ii), we get, _____________
log2 [x2 + 4x + 5] > 0 ​ a  2  +  b2 +   
y2 = ÷ 2ab ​, when l = b2
______
[x2 + 4x + 5] > 1 Thus, Rf = Î(a + b), 2​÷a  2 + b2 
 ​˚
x2 + 4x + 5 ≥ 2 p
21. As 0 < x < ​ __ ​ , so 0 < x < 1
2
x + 4x + 3 ≥ 0 2
Now, range of


(x + 1)(x + 3) ≥ 0
x £ – 3, x ≥ – 1 ...(2)

p
y = ln {(cos x) cos x + 1} in ​ 0, __(  )
​   ​   ​
2
Real Function  1.79


Let
Ry = ln (xx + 1) in (0, 1)
g(x) = x + 1 x
fi 0 £ sin–1 ​ ​ _______
( 
x2   
(​  x + 1 )​ 2
2
p
 ​  ​ < __
​   ​ 
)
Thus, the range of a function is
fi g¢(x) = xx (1 + ln x)
For max / min, g¢(x) = 0 gives
​ 2​
p
R f =  ​​ 0, __ [  )
​   ​  ​  ​

fi xx (1 + ln x) = 0
(  ​ ​( 16 sin2 x + 1 )​  ​
23. Given f (x) = log2 ​ 2 – lo​g​÷​ __2 ​
   )
fi (1 + ln x) = 0
f is defined for
fi ln x = – 1
1 ​ (​  16 sin2 x + 1 )​ > 0
(i) 2 – lo​g​÷​ __2 ​
x = e–1 = __
  
fi ​ e ​
(ii) ​( 16 sin2 x + 1 )​ > 0
1
Now, g(x) is minimum at x = __
​ e ​
which is true for all x in R
Minimum value of g(x) is
Now, from (i), we get,
(  ) (  )
1
= g ​ __
1 1/e
​ e ​  ​ = ​​ __
​ e ​  ​​ ​ + 1
(16 sin2 x + 1) < 2
x
Also, lim ​  (x + 1)
g (0) = ​  
x Æ 0 0 £ (16 sin 2 x) < 1
lim ​ (​  e x log x + 1 )​
= ​   1 £ (16 sin2 x + 1) < 2
x Æ 0

= ​e ​x Æ 0 (  )
logx
lim ​ ​ ____
​   ​   ​  ​
1/x ​ + 1
​log​​÷__2 ​
     
​ (16 sin2 x + 1) < lo​g ​​÷__2 ​ 
​ (1) £ lo​g​÷​ __2 ​   
​ (2)

(  )
1 0 £ lo​g ​​÷__2 ​ ​(16 sin2 x + 1) < 2
​ __
x ​
  
lim ​ ​ ___
​  
x Æ 0 ___
​    ​  ​
– 1
​   ​ 
–2 < –lo​g​÷​ __ ​ (16 sin2 x + 1) £ 0
2    ​
= ​e​ x2 ​ + 1
​(16 sin2 x + 1) £ 2
2 – 2 < 2 – lo​g ​​÷__2 ​ 
  
​  
lim ​  (– x)
= e​ ​x Æ 0 ​ + 1 ​(16 sin2 x + 1) £ 2
0 < 2 – lo​g ​​÷__2  ​
   
=1+1
= 2. ( 
log2 (0) < log2 ​ 2 – lo​g ​​÷__2 ​
    
(​​  16 sin2 x + 1 )​  ​ £ log2 2 )
Thus, (  )
1 1/e
1 + ​​ __
​ e ​  ​​ ​ < x < 2
( 
– • < log2 ​ 2  –  lo​g ​​÷__2 ​
   )
(​​  16 sin2 x + 1 )​  ​ £ 1
Thus, R f = (– •, 1]
fi (  (  ) )
ln ​ 1 + ​​ __
1 1/e
​ e ​  ​​ ​  ​ < ln (xx + 1) < ln (2)
24. Let g (x) = (5x2 – 8x + 4)

ln ​( 1 + ​​( __
​ e ​ )​​ ​ )​< ln ​( (cos x)
1
(  )
1/e cos x
+ 1 )​< ln (2) 8 4
fi = 5 ​ x2  – ​ __  ​ x + __
​   ​   ​
5 5
R = (​  ln ​( 1 + (​​  __ ​ e ​ )​​ ​ )​,  ln (2) )​ = ​( 5 ​​( x  – ​ __ ​  )​​ ​+ __
1 1/e
fi ​   ​  )​
4 2  4
f

( 
_______ 5 5
22. Given f (x) = cos ​ ​ ÷
  2 + 1 ​
2x
​________
–1
2
x +1

 ​  
 ​ ) Thus, Rg =  ​​[ __
​5 ​
​    ​, • ​ )​
4

( ÷ 
___________

)
4
2x2 + 1 ​ __ ​  £ g(x) < •
= sin –1
​ ​ 1  – ​ _______
      ​ ​  ​ 5

(  )
(x2 + 1)2 4
log5/4 ​ __
​   ​   ​ £ log5/4 ​( g (x) )​ < log5/4 (•)

( ÷ 
________

)
4 5
x
= sin–1 ​ ​ ​ _______
     ​ ​  ​ –1 £ log5/4 ​( g (x) )​ < •
(x2 + 1)2


2
x2   
= sin–1 ​ ​ _______
( 
 ​  ​
(​  x + 1 )​ )


tan–1 (– 1) £ tan–1 ​( log5/4  ​( g (x) )​ )​< tan–1 (•)
p p
– ​ __ ​  £ tan–1  ​( log5/4 (​( g (x) )​ )​ < __
​   ​ 
As we know that 4 2
x2 p p
0 £ ​ _______
     ​<1 – __
​   ​  £ f (x) < __ ​   
(x2 + 1)  4 2

(  ) [  p p
fi x2   
sin–1(0) £ sin–1 ​ ​ _______  ​  ​ < sin–1 (1)
(​  x2 +1 )​
 ​ Thus, ​ __ ​ , __
R f = ​  – ​ 
​ 4 2 ​ )
​   ​   ​ ​
1.80  Differential Calculus Booster

25. Given f (x) = 6x + 3 – x + 6 – x + 3x + 2. xŒ [– 1, 1]


= (6x + 6 – x) + (3x + 3 – x) + 2 Df = [– 1, 1]
≥2+2+2=6 Thus, Rf = [f (–1), f (1)]
Thus, R f =  ​​​[ 6, • ​ )​
[  ] 3
= ​ – ​ __ ​ , 1  ​
(  )
______ 4
1 + x2
26. Given f (x) = cos–1 ​ ​ _____
   
    –  x2 
​  ​ + ​÷2   ​
f (x) = sin  ​[ x + __ ​   ​  ]​ + cos  ​[ x – __ ​   ​  ]​
2x –1 1 2 1
– 1 2
28. Given
f is defined for 2 2

Now, ​[ x + __ ​    ​ ]​ = [​  x – __ ​   ​  + 1 ]​ = ​[ x – __


​    ​ ]​ + 1
( 
1 1 1
)
2 2 2
1 + x2
(i) – 1 £ ​ ​ _____ ​  ​ £ 1
   
  2 2 2
2x
f (x) = sin  ​[ x + __ ​   ​  ]​ + cos  ​[ x – __​    ​ ]​
–1 1 2 1
–1 2
(ii) 2 – x2 ≥ 0 Thus,
2 2

= sin  ​[ ​( x – ​ __ ​  )​ + 1 ]​ + cos  ​[ x – ​ __ ​  ]​


Now, from (i), we get, 1 1

|  |
–1 2 –1 2

2 2 2
1+x
​ ​ _____ ​  ​ £ 1
   

It is defined for [​  x – __ ​    ​ ]​ = 0, – 1
2x 1 2

2 2
|1 + x | £ 2 |x|
Therefore,
(1 + x2) £ 2 |x|
p p
f (x) = sin–1 (1) + cos– 1(0) = __
​   ​  + __
​   ​  = p
|x|2 – 2 |x| + 1 £ 0 2 2
Also, f (x) = sin–1 (0) + cos– 1(– 1) = 0 = p = p
(|x| – 1)2 £ 0
Thus, Rf = {p}
(|x| – 1) = 0

{  ÷ 
_______

x = ± 1 ...(1) 29. Given


p2
f (x) = log ​ cos ​ ___
​   ​ – x2 ​  
9
​ }
Also, from (ii), we get, f is defined for

( ÷ 
2 – x2 ≥ 0 _______

)

p2
2 (i) cos ​ ​  ​ ___ ​ – x2 ​  
​> 0
x –2£0 9
__

​( x + ÷
x2 – ​( ​÷2 ​ 
   )​ ​( x  – ​÷2 ​ 
​ 2 ​ 
   )2​ £ 0
__
   )​ £ 0
__ ( 
p2
(ii) ​ ___
9 )
​   ​ – x2  ​ ≥ 0
__ __
    £ x £ ​÷2 ​
–​÷2 ​    ...(2) Now, from (ii), we get,

From (1) and (2), we get, ​ x2 – ___


​   ​   ​ £ 0
9 (  2
)
x = {– 1, 1}
Thus, Df = {– 1, 1}
​ x + __
3 ( 
p x–p
​   ​   ​ ​ ​ _____
3 ) ( 
 ​  
 ​ £ 0 )
p p
Rf = { f (1), f (– 1)} – ​ __ ​  £ x £ ​ __ ​ 
3 3

[ 
= {1, 1 + p}
1 x+1
Thus,
p p
Df = ​ – ​ __ ​ , __
3 3
​   ​   ​ ]
27. Given (x) = __
​ p ​ ​( tan –1x + sin–1x )​ + ​ __________

   ​

÷ 
_______
2
x + 2x + 5 p2 p
Also, 0 £ ​ ___ £ __
​   ​ – x2 ​  ​   ​ 
f is defined for 9 3

( ÷ 
_______


(i) – 1 £ x£1
(ii) x2 + 2x + 5 π 0

p 
cos ​ __ (  ) p2
​   ​   ​ £ cos ​ ​ ___
3
​   ​ – x2 ​  
9 )
​ £ cos (0)

( ÷ 
_______
which is true for all x in R
From (i) and (ii), we get
1
2
p2
​ __ ​  £ cos ​ ​ ___
​   ​ – x2 ​  
9
​£ 1 )
Real Function  1.81

(  ( ÷  ) ) (  )
_______

(  )
3p

1
log ​ __
p2
​   ​   ​ £ log ​ cos ​ ​ ___
​   ​ – x2 ​  
​  ​ £ log (1) log ​ ___
​   ​   ​ £ f (x) £ log (2p)
2 9 2

[  (  )
3p
]
( 
Rf = ​ log ​ ___
( ÷  ) )
_______ Thus, ​   ​   ​, log (2 p)  ​
p2 2
– log (2) £ log ​ cos ​ ​ ___
​   ​ – x2 ​  
​  ​ £ 0
9 32. Find the range of the function
Thus, Rf = [– log (2), 0] f (x) = loge (2 sin x + tan x – 3x + 1),
p p
30. Given f (x) = [1 + sin x] + [cos x – 1] + [tan–1 x] where ​ __ ​  £ x £ __
​   ​ 
6 3

__
= 1 + [sin x] + [cos x] – 1 + [tan–1 x] 33. Given ​ ​{ ​÷2 ​ 
f (x) = lo​g​​ __5 ​    (sin x – cos x) + 3 }​
   ÷
__
= [sin x] + [cos x] + [tan  x] –1 Now, ​{ ​÷2 ​
    (sin x – cos x) + 3 }​

Ï 1: x = 0
Ô 0 : 0 < x < tan (1)
{  (  ) } p
= ​ 2 sin ​ x – __
​   ​   ​ + 3  ​
4
Ô
Ô p
(  ) p
– 1 £ sin ​ x – ​ __ ​   ​ £ 1
4
Ô 1 : tan(1) £ x <
Ô
Ô p
2
(  )
– 2 £ sin ​ x – __
p
​   ​   ​ £ 2
4
Ô 2:x = 2
Ô
=Ì p
(  )
– 2 + 3 £ sin ​ x – __
p
​   ​   ​ + 3 £ 2 + 3
4
Ô 0: 2 < x <p
Ô
Ô –1 : p £ x £ 3p
(  )p
1 £ sin ​ x – ​ __ ​   ​ + 3 £ 5
4

lo​g​ ​ (1) £ lo​g​ ​{​  sin (​  x – __


Ô
​   ​  )​+ 3 }​£ ​log​ ​(5)
2 p
Ô __ __ __

3p ÷   
​ 5 ​ ÷   
​ 5 ​ 4 ÷   
​ 5 ​
Ô 0: £ x £ 2p
0 £ lo​g​ ​ ​{ sin (​  x – __
Ô 2
​   ​  )​ + 3 }​ £ 2
p
Ô 2 : x = 2p __

Ó   
​÷5 ​ 4

Hence, the range of the function is 0 £ f (x) £ 2


= {–1, 0, 1, 2} Rf = [0 2]

{  }
_____ 3p
31. Given  ​ )​  + ​ ___ ​   ​
f (x) = log10 ​ sin–1 ​( ​÷x  –  5  34. Given equation is
2 __________
f is defined for ​÷ 3 – 4 cos  
2
 x ​ > 2 sin x + 1
_________
(i) (x – 5) ≥ 0 ​÷ 4 sin2 x – 1 ​   > 2 sin x + 1
Let sin x = t
x ≥ 5
So, the given equation becomes
_____ ______
(ii) – 1 £ ​÷x  – 5 ​ £ 1
​÷ 4t 2 – 1 ​
  > 2t + 1
0 £ (x – 5) £ 1 It is defined for 4t 2 – 1 ≥ 0
5 £ x £ 6 fi (2t + 1) (2t – 1) ≥ 0
Thus, Df = [5, 6] 1 1
fi t ≥ __
​   ​  and t £ – ​ __ ​ 
_____ 2 2
p
0 £ sin–1 ​÷x  – 5 ​ £ ​ __ ​ 
Now, Case-I:   when (2t + 1) ≥ 0
2
3p _____ 3p p ___ 3p The given equation becomes
​ ___ ​ £ sin–1 ​( ​÷x  – 5 ​  
)​ + ___
​   ​ £ __
​   ​  + ​   ​ 
2 2 2 2 fi (4t 2 – 1) > (2t + 1)2
3p _____ 3p
​ ___ ​ £ sin–1 ​( ​÷x  – 5 ​  
)​ + ___
​   ​ £ 2p fi (4t 2 – 1) > 4t 2 + 4t + 1
2 2
fi 4t < – 2
(  ) (  )
3p _____ 3p
log ​ ___
​   ​   ​ £ log ​ sin–1 (​÷ x – 5 ​) + ___
​   ​   ​ £ log (2p) fi
1
t < – ​ __ ​ 
2 2 2
1.82  Differential Calculus Booster

So, it has no solution 1 1


fi (x – [x]) + __
​ x ​ – ___
​    ​ = 1
Case-II:  when 2t + 1 < 0 [x]
1 1

1
t < – ​ __ ​  fi x + __
​ x ​ = [x] + ___
​    ​ + 1
2 [x]

(  ( 
1
fi sin x < – ​ __ ​ 
2
1
fi ​ x + __ ) 1
​ x ​  ​ – ​ [x] + ___
​    ​  ​ = 1
[x] ) ...(i)
5p p
fi 2nx – ___
​   ​ < x < 2np – __
​   ​ , n  Œ I Since R.H.S is an integer, so L.H.S is also an
6 6 integer.
35. Given
1
m = ​ __[  ] [  1
​   ​   ​ + ​ __
4
1
​   ​   + ​ ____
4 200 ] [  1
   ​  ​ + ​ __
2
​   ​  + ____
4 200 ]
​     ​  ​ Let ​ [x] + ___ ( 
1
)
​    ​ + 1  ​ = t (Integer)
[x]


1
+ ... + ​ __ [ 
​   ​  +
4
199
____
​   
200 ] 1
 ​  ​ fi ​ x + __ (  )
​ x ​  ​ = t, from (i)

[ 1
= ​ __ ]
​   ​  × 200  ​ = 50
4
fi x2 – tx + 1 = 0
______
Hence, the value of m + 50. t±÷ ​ t  2 – 4 ​ 
fi x = ​ __________
 ​    
= 50 + 50 2

= 100 For real x, t 2 – 4 ≥ 0


36. Given equation is {x} + sin {x} = 2 fi (t + 2) (t – 2) ≥ 0
Now, {x} < 1 and {sin x} < 1 fi t £ – 2 and t ≥ 2
fi {x} + {sin x} < 2 fi t = 2 and – 2 does not satisfy the equation (i)
Hence, it has no solution. So, the equation (i) has infinitely many solutions.
37. The given equation is 39. Given equation is {x2} + [x4] = 1
x2 – 4x + [x] + 3 = 0 ...(i) It can be solved graphically, it has only 2 real
fi 2
x – 4x + x – {x} + 3 = 0 solutions.

fi x2 – 3x + 3 = {x} 40. Given f (x) = |x – a| + |x – b| + |x – c| + |x – d|

fi 0 £ x2 – 3x + 3 < 1
Now, x2 – 3x + 3 ≥ 0
It is true for all x, since D < 0
Also, x2 – 3x + 3 < 1
fi x2 – 3x + 2 < 0
fi (x – 1) (x – 2) < 0
fi 1<x<2
fi [x] = 1
Now, from (i), we get, Hence, the least value of the function is
x2 – 4x + 4 = 0 = (c + d – a – b) at x = b or c.

(  )
89
fi (x – 2)2 = 0 1
41. Let S = Â ​ ___________
​     2 ​  ​
n = 1 1 + (tan n°)
fi x=2
which does not satisfy 1 < x < 2 1 1 1
= + +
Thus, the given equation has no solution. 1 + tan (1∞) 1 = tan (2∞) 1 + tan 2 (3∞)
2 2

38. Given F(x) = x – [x] 1 1 1


+ 2
+ 2
+
1 1 + tan (87∞) 1 = tan (88∞) 1 + tan 2 (89∞)
Now, F (x) + ____
​     ​ = 1
F(x)
...(i)
Real Function  1.83

1 x
=
1
+
1
+
1 fi ​__   ​  < __ ​    ​ £ 1
2 3
1 + cot (89∞) 1 + cot (88∞) 1 + cot 2 (87∞)
2 2
3
fi ​ __ ​  < x £ 3
1 1 1 2

1 + cot 2 (3∞) 1 + cot 2 (2∞) 1 + cot 2 (1∞) 4


and ​ __
​ x ​  ​ = 4 [  ]
...(ii) 4
Adding (i) and (ii), we get, fi 4 £ ​__  x ​ < 5
4
89 1 2 fi ​__    ​  < x £ 1
fi 2S = Â 2
+ 5
n = 1 1 + tan ( n∞) 1 + cot 2 (n∞)
This is not possible simultaneously,
89 1 tan 2 (n∞) So, it has no solution.
fi 2S = Â +
2
n = 1 1 + tan ( n∞) 1 + tan 2 (n∞) From (iii), we get,

(  ) [  ] [  ]
3 4
89 1 + tan2 (n°) ​ __​ x ​  ​ = 2 and ​ __​ x ​  ​ = 3
fi 2 S = Â = ​ ​ ___________
  
   ​  ​
n =1 1 + tan2 (n°) 3 4
fi 2 £ ​__  x ​ < 3 and 3 £ __
​ x ​ < 4
89
fi 2 S = Â  1 = 1 + 1 + 1 + ... + 1 (89 times) 1 x __ 1 1 x __ 1
n =1 fi ​__
   ​  < __
​    ​ £ ​   ​  and __
​   ​  < __
​    ​ £ ​   ​ 
3 3 2 4 4 3
fi 2S = 89
3 4
1 fi 1 < x £ __ ​   ​  and 1 < x £ __
​   
fi S = 44 ​ __ ​  2 3
4
4
 ​ fi 1 < x £ __
[  ] [  ]
3 4 ​   ​ 
42. Given equation is ​ __
​ x ​  ​ + ​ __
​ x ​  ​ = 5 3

when x is negative, the value of fi (  ]


x Œ ​  1,
4
​ ​ __ ​   ​ ​
​ 3 ​
3
​ __ [  ] [​  __​ 4x ​ ]​ can never be equal to 5
​ x ​  ​ + Thus, = 1, b = 4, c = 3

when x is positive, there are 3 possibilities Hence, the value of a + b + c + abc + 30


= 1 + 4 + 3 + 12 + 30
[  ] 3
(i) ​ __ [  ]
4
​ x ​  ​ = 0 and ​ __
​ x ​  ​ = 5 = 50.

(ii) ​[ __
​ x ​ ]​ = 1 and [​  __ ​ x ​ ]​ = 4
3 4 43. Given equation is

(iii) ​[ __
​ x ​ ]​ = 2 and [​  __
3
​ x ​ ]​ = 3
4 4(  ) (​​  __​ 12 ​  )​​ ​ + b = 0
1 x
​​ __
​   ​   ​​ ​ +
x–1

fi ​​( ​​( __
​   ​  )​​ ​ )​​ ​ + ​​( __
​   ​  )​​ ​ + b = 0
x 2
1 1 x–1

From (i) ​[ __ ​ x ​ ]​ = 0, then 0 £ ​__


3 3 2 2
 x ​ < 1

t + 2t + b = 0, t = (​​  __ ​    ​ )​​ ​


x
2 1
fi x>3 fi
2
4
and ​ __
​ x ​  ​ = 5 [  ] where 0<t<1
Sum of the roots = – 2 < 0
4
fi 5 £ __ ​ x ​ < 6 So, there is a + ve root in (0, 1)
1 x __ 1 Thus, f (0), f (1) < 0
fi ​__   ​  < __ ​    ​ £ ​    ​
6 4 5
fi b (b + 3) < 0
2 4
fi ​__    ​  < x £ ​__
   ​  fi – 3 < b < 0
3 5
This is not possible simultaneously, fi bŒ (– 3, 0)
So, it has no solution. 44. Given f (x) = px + q
3
[  ] 3
​ x ​  ​ = 1, then 1 £ ​__
From (ii), ​ __  x ​ < 2
Now,

f  (f  (f (x)))
= f (f (px + q))
1.84  Differential Calculus Booster

= f (f (a)), a = px + q fi (a2 + 46)2 – {2(9 + 8a)2} < 0


= f (pa + q) fi (a2 + 16a + 64) (a2 – 16a + 28) < 0
= f (p(px + q) + q) fi (a + 8)2 (a – 2) (a – 14) < 0
2
= f (p x + pq + q) fi (a – 2) (a – 14) < 0
= f (c), c = (p2 x + pq + q)
fi 2 < a < 14
= pc + q
Hence, the value of a is (2, 14).
= p(p2 x + pq + q) + q
2 (ex – e–x) (sin x + tan x)
= p3x + p2 q + pq + q 47. Given f (x) = ​ _____________________
       ​
Given f ( f ( f (x))) = 8x + 21
+ 2p
x______
2 ​ ​  p   [ 
​  ​ – 3 ]
Thus, p3 = 8, p2 q + pq + q = 21
2 (ex – e– x) (sin x + tan x)
= _____________________
​         ​
fi p = 2, q = 3
[ 
2 ​ 2 + __
x
]
​ p ​ ​ – 3
Hence, the value of p2 + q2 + p + q
2 (ex – e–x) (sin x + tan x)
=4+9+2+3 = ​ _____________________
x     ​
  
= 18.
2 ​ __ [  ]
​ p ​ ​ + 1

45. Given f (x) = x3 – 2x2 – px + 1 2 (ex – e–x) (sin x + tan x)


f (– x) = ​ _____________________
     ​
  
f ¢(x) = 3x2 – 4x – p
Since f is one-one function
p
x
[  ]
2 ​ – ​ __ ​ ​ + 1

So, f is either strictly increasing or decreasing 2 (ex – e–x) (sin x + tan x)


= ​ _____________________
     ​
  
function
Thus, f ¢(x) > 0 (since, a = 3 > 0)
[  ]
– 2 ​ __
x
​ p ​ ​ – 1

fi D<0 2 (ex – e–x) (sin x + tan x)


= – ​ _____________________
x     ​
   = – f (x)
fi 16 + 12p < 0 [  ]
2 ​ __
​ p ​ ​ + 1
fi 4 + 3p < 0
4 Thus, f (x) is an odd function.
fi p < – __
​   ​  48. Given equation is
3

÷(   )
______
1 3
46. Since the given function is onto, so the range of a x – x + 1 = __
2
​   ​  + ​ ​ x – __
​   ​    
​  ​
function is equal to its co-domain 2 4

​   ​  + ​ ​   x – __
_______
​​( x – __
4 2 ÷(
​   ​  )​​ ​ + __ ​   ​  )​ ​
Thus, Rf = R 1 3 1 2 3
​   ​  = __  
2 4
ax2 + 6a – 8
Let y = ​ ___________
    ​
​   ​  = ​ ​(   x – __
_______
​​( x – __
​   ​  )​​ ​ + __ ​   ​  )​ ​
4 ÷
a + 6x – 8x2 1 1 2 3

2 4
fi (a + 8y) x2 + 6 (1 – y) x – (8 + ay) = 0

since x is real, so D ≥ 0
fi 36 (1 – y)2 + 4 (a + 8y) (8 + ay) ≥ 0
fi 9 (1 – y)2 +  (a + 8y) (8 + ay) ≥ 0
fi (9 + 8a) y2 + (a2 + 46)y + (9 + 8a) ≥ 0
Since the range of a function is R, so the given equal-
ity holds good for all real values of y
Thus, (9 + 8a) > 0 and D < 0
9
fi a > – ​ __ ​  and D < 0
8
Now, D<0 Clearly, it has a solution at x = 1.
Real Function  1.85
_____ _____
p
49. We have f (x) = ​ g ​ ___ [ 
​    ​  ​  ​
10 (  ) ] 2. Given f (x) = log2 ​( ​÷x  – 2 ​
Clearly, Df = [2, 4]
​ )​
  – x  
 + ​÷4


p
[  (  ) p
= ​ sin ​ ​ ___  ​  ​ + cos ​ ___
18
​    ​  ​  ​
18 (  ) ] Thus, a = 2, b = 4
Hence, the value of (2b – 3a)
= [sin (18°) + cos (18°)] =8–6=2

÷  (  )
__________

[ 
_________

]
__ __ x–2
   – 1
​÷5 ​   + 2 ÷​ 5 ​ ​
​÷__________
10      3. Given f (x) = ​ log2 ​ ​ _____
    ​  ​ ​
= ​ ​ ______ ​ 
 + ​   ​ 
    
​ 3–x
4 4
Thus, ​( ​ _____  ​  ​ > 0 and (​ ​ _____  
3 – x) 3 – x)
x–2 x–2
= [1. 26] ​  ​ ≥ 1
=1
fi ​( ​ _____   ​  ​ < 0 and (​ ​ _____ 
x – 3) 3 – x)
x–2 x–2
 ​  ​ – 1 ≥ 0
fi sin2016 x – cos 2016 x = 1
fi ​( ​ _____   ​  ​ < 0 and (​ ​ ______ ​  
x – 3) x–3)
x–2 2x – 5
​£ 0
fi sin 2016 x = cos 2016 x + 1 ≥ 1
It is possible only when cos 2016 x = 0 fi 2 < x < 3 and __
5
​   ​  £ x £ 3
p 2

50. Given
x = (2n + 1) ​ __ ​ , n Œ I
2
f (x, y) = 7x + 4xy + 3y2
2

5
​ 2​ [  )
Df =  ​​ ​ __  ​ ​, 3  ​
Thus, a = 5/2, b = 3
Put x = cos q, y = sin q Therefore, the value of (2a – b)
Thus, y = f (q) =5–3=2
4. Let f (x) = |x – 2| – |x + 1|
= 7 cos 2 q + 4 sinq cosq + 3 sin2 q
Clearly, Range of the function f (x) is [– 3, 3]
= 3(sin 2 q + cos 2 q) + 4 sinq cosq + 4 cos2 q Thus, – 3 £ p £ 3
= 3 + 2 sin 2q + 2(1 + cos 2q) Therefore, the number of integral values of p is 7,
where p = – 3, – 2, – 1, 0, 1, 2, 3.
= 5 + 2 sin 2q + 2 cos 2q
x2 + x + a
5. Let y =​ _________     ​
= 5 + 2 (sin 2q + cos 2q) x2 + 2x + a
__
Max value = ​( 5 + 2​÷2 ​ 
   )​ a
x + __
​ x ​ + 1
__
= ​  _________   ​
Min Value = (​  5 – 2​÷2 ​ 
   )​ a
x + __​ x ​ + 2
INTEGER TYPE QUESTIONS Let g(x) = x + __
a
​ x ​
1. We have a
fi g¢(x) = 1 – __
(  ) (  )
x–p x–p ​  2  
f (x) = 4 cos4 ​ ​ _____  ​  ​ – 2 cos ​ ​ _____  ​  ​ x
4p 2 2p 2  ​ For max or min, g¢(x) = 0
= ​​  2cos  ​ ​ _____ ​  ​  ​​ ​ – 2 cos ​ ​ _____ ​  ​

{ ( x4p – p ) } ( x2p – p )
2
2
2
2 fi 1 – __
a
​  2  ​  = 0
x
= ​​  1 + cos ​ ​ _____ ​  ​  ​​ ​ – 2 cos ​ ​ _____ ​  ​
{ ( x2p – p ) } ( x2p – p ) 2 __
fi x = ± ​÷a     ​
2 2
__
Minimum value at x = ​÷a 
  ​  is

= 1 + cos  ​ ​ _____ ​  ​
2
( x2p – p ) 2
2​÷a 
= _______
   ​ + 1
​  __
2 ​÷a 
   ​ + 2

 ​
__

__
Thus, the period of f (x) is Maximum value at x = – ​÷a 
  ​  is
__
2 ​
÷ a 
  ​
  – 1

p
= ____
​     ​ = 2p 3 = _______
​  __  
1 2 ​÷a 
   ​ – 2
____
​   2 ​  __
2p  2 ​÷a 
   ​+ 1 5
 ​ Clearly, ​ _______
__ = __
 ​  ​   ​ 
Clearly, m = 2 and n = 3 2 ​÷a 
   ​+ 2 6
__ __ __
Hence, the value of (m + n) is 5. fi 12 ​÷a 
  ​  + 6 = 10 ​÷a 
  ​  + 10 ​÷a 
  ​  = 2
1.86  Differential Calculus Booster
__
fi 2 ​÷a 
  ​  = 4 10. Clearly, f (x) ≥ 0 for any x in R
__
fi ​÷ a ​  = 2 When x is non zero,
fi a=4 1 1
f (x) = ______  ​ £ __

(  )
​      ​   ​ 
2 1 2
a +a
Hence, the value of ​ ​ ______
 ​  
 ​ is 4 x2 + __
​  2  ​ 
5 x
6. Given equation is But at x = 0, f (0) = 0

( [  ] )
15
sgn ​ ​ _____
​  2    
x +1
 ​  ​  ​ = [1 + {2x}]
1
Thus, the range of the given function is ​ 0, __
​   ​   ​
2 [  ]
( [  ] )
Clearly, a = 0, b = 1 and c = 2
15
fi sgn ​ ​ _____
​  2    
 ​  ​  ​ = 1 Hence, the value of (a + b + c + 2) is 5
x +1
ex – 1
Thus, 1 + x2 £ 15 y = ​ _____
11. Let  
 ​
ex + 1
fi x2 £ 14 1+y
Hence, the number of integral values of x are 7 fi ex = ​ _____   ​
1–y
where x = – 3, – 2, – 1, 0, 1, 2, 3. 1+y
_____ _____ fi ​ _____   ​ = ex > 0
7. Given equation is 3÷ ​ x  – 3 ​ 
​ x  + 1 ​ = ÷ 1–y
y+1
Y fi ​ _____   ​ < 0
y–1
fi – 1 < y < 1


Thus, the range is (– 1, 1)
X
O
Clearly, a = – 1, b = 1
Hence, the value of (a2 + b2 + 2) is 4.
12. Given f (x) = ax + b

We have f ( f (x))
From the graph, it is clear that, the number of = f (ax + b)
solutions is 1.
= a (ax + b) + b
8. Given equation is  sgn (x + 1) = 2x2 – x
= a2 x + (ab + b)
Y
Also, f ( f ( f (x)))
= f (a2 x + (ab + b))
y=1 = a(a2 x + (ab + b)) + b
X¢ X
y = –1
O = a3x + a(ab + ) + b
= a3x + a2b + ab + b
= 8x + 21

Clearly, a = 2, b = 3
Clearly, the number of solutions is 2. Hence, the value of (a + b + 3) is 8.
1
9. Given function is f (x) = ________
​  _______
 ___ ​ 
–3
÷​    ​÷81 ​ ​
x – 1
     questions asked in
It is defined for Past JEE Main Exams (2002 to 2014)
___
x – 1
x – 1 ≥ 2, x – 1 Œ N and    > 3
 ​÷81 ​ 1. Clearly the option is (b)
x – 1 __
fi x ≥ 3, x Œ N and 81 > 3 Let f (x) = cos x and g(x) = ÷
​ x 
   ​
__ __
fi x ≥ 3, x Œ N and 34 > 3x – 1 Thus, f (g(x)) = f (​÷x 
  ​)  cos ​÷x 
   ​ is non-periodic.



x ≥ 3, x Œ N and x < 5
x = {3, 4}
2. we have – 1 £ ​ log3 ​ __(  (  ) )
x
​    ​  ​  ​ £ 1
3

Thus, the integral values of x is 2 fi


x
3 –1 £ ​ __ (  )
​    ​  ​ £ 3
3
Real Function  1.87

1 x
fi ​ __ ​  £ ​ __
3 (  )
​    ​  ​ £ 3
3
Ans. (c)

6. The function sin–1 (x – 3) is defined for


fi 1£x£9
– 1 £ (x – 3) £ 1
fi x Œ [1, 9]
fi 2£x£4 ...(i)
Thus, Df = [1, 9] _____
Also, the function ​÷9  – x2 
 ​ is defined for
Ans. (a)
9 – x2 > 0
3. The expression log10 (x3 – x) is defined for
(x3 – x) > 0 fi x2 – 9 < 0
fi (x + 3) (x – 3) < 0
x (x2 – 1) > 0
fi – 3 < x < 3 ...(ii)
x (x + 1) (x – 1) > 0
From (i) and (ii), we get, x Œ [2, 3)
x Œ (– 1, 0) » (1, •)
Thus, Df = [2, 3)
3
Also, the function ​ _____
   ​ is defined only when 7. Since the function is onto, so Rf = Cf = S
4 – x2 _____
x π ± 2   +  3 ​ 
Max value of f = ​÷1  +1=2+1=3
_____
Thus, Df = (– 1, 0) » (1, •) – {2} Min value of f = – ​÷1  + 3 ​
 + 1 = – 2 + 1 = – 1

= (– 1, 0) » (1, 2) » (2, •) Thus, S = Rf = [– 1, 3]


Ans. (c) Ans. (d)
8. Since f (x) is symmetrical about the line x = 2
4. Given  f (x + y) = f (x) + f (y)
f (2) = 7 + 7 = 14 then f (2 + x) = f (2 – x)

f (3) = 14 + 7 = 21 Ans. (b)






f (4) = 21 + 7 = 28
... ... ...

p p
2 2 ( 
9. The range of the function f is ​ – ​ __ ​ , __
​   ​   ​ )
... ... ... Since the function f is onto, so Rf = Cf = B

( 
... ... ... p p
f (n) = 7 ◊ n Thus, B = ​ – ​ __ ​ , __
2 2 )
​   ​   ​
n
Thus, ​S  ​ ​   ​f (r) 10. Given f (x – y) = f (x) f(y) – f (a – x) f (a + y)
r =1
Put x = 0 = y, we get,
= (7 + 14 + 21 + 28 + ... + 7 ◊ n)
f (0) = f(0) f (0) – ( f (a))2
= 7(1 + 2 + 3 + ... + n)
1 = 1 – (f (a))2
7n(n + 1)
= ​ ________
 ​   f (a) = 0
2
Ans. (c) Also,  f  (2a – x) = f (a – (x – a))
7 – x
5. The function Px – 3 is defined for = f (a)f (x – a) – f (a – a)f (a + x – a)
x – 3 ≥ 0, 7 – x > 0 & x – 3 £ 7 – x = f (a)f (x – a) – f (0)f (x)
fi x ≥ 3, x < 7 & x £ 5 = 0 ◊ f (x – a) – 1◊ f (x)
  

fi x = 3, 4, 5
= – f (x)
Thus, Df = {3, 4, 5} Ans. (b)
when x = 3, y = 4P0 = 1
11. The given expression
when x = 4, y = 3P1 = 3
3x2  + 9x + 17
when x = 5, y = P1 = 2 2 = ____________
​  2   
   ​
3x + 9x + 7
Thus, Rf = {1, 2, 3}
1.88  Differential Calculus Booster
_____
(3x2 + 9x + 7) + 10 fi ​÷ x + 1 ​ 2
 – 1 = (x + 1) – 1
= ​ _________________
  
    ​
(3x2 + 9x + 7) _____
2
fi ​÷ x + 1 ​
 = (x + 1)
10 _____
= 1 + ____________
​        ​ fi (x + 1)2 – ​÷x  + 1 ​
 = 0
(3x3 + 9x + 7) _____
2
( (x + 1)3/2 – 1 )​ = 0
fi ​÷ x + 1 ​​ 
Let g(x) = (3x + 9x + 7)

_____
fi ​÷ x + 1 ​  = 0, (​  (x + 1)
3/2–1
)
  ​= 0
D fi x = – 1, 0
Min value of g(x) = ___
​     ​
4a Thus, the solution set is {– 1, 0}
81 – 84 __ 1 Hence, the statement-II is a correct explanation for
= – ​ _______
 ​ 
 = ​   ​  statement-I
12 4
Thus, the max value of the given expression Ans. (a)
16. The function f is defined for |x| – x > 0
= 1 + 10 × 4 = 41
fi |x| > x
Ans. (b)
​ ​2 fi x Œ (– •, 0)
12. Here, 4​ –​ x ​​ is defined for all R.
Thus, Df = (– •, 0)
x
(  ) x
cos–1 ​ ​ __  ​ – 1  ​ is defined for – 1 £ ​ __
2 ( 
​    ​ – 1  ​ £ 1
2 ) Ans. (a)

(  )
x 17. No questions asked in 2012.
fi 0 £ ​ __
​    ​  ​ £ 2 18. Ans. (2)
2
fi 0£x£4 Here A × B has 8 elements respectively
Now, total subsets of A × B is 28
Also, log (cos x) is defined for cos x > 0
Thus, the number of subsets of 28 having 3 or more
p p
fi – ​ __ ​  < x < __
​   ​  elements
2 2 = 28 – 8C0 – 8C1 – 8C2
Thus, the function f is defined for = 256 – (1 + 8 + 28)
[  )
​ 2​
p
x Œ ​ ​ 0, __
​   ​  ​  ​

= 256 – 37
= 219

D =  ​​[ 0, __
​   ​  ​ )​
p 19. Ans. (3)
fi f
​ 2​ We have
Ans. (b) X = {4n – 3n – 1 : n Œ N}
13. Since f is invertivle, so its inverse is exist. = {(1 + 3) n – 3n – 1}
Let y = 4x + 3 = {(1 + nC1◊3 + nC2◊32 + nC3◊33 + ... + nCn ◊3n)
y–3 – 3n – 1}
fi x = ​ _____
 ​ 

4 n 2 n 3
= ( C2 ◊ 3 + C3 ◊ 3 + ... Cn ◊ 3 ) n n

x–3
Thus, f  –1(x) = ​ _____ ​ 
  = 9(nC2 + nC3 ◊ 3 + ... nCn ◊ 3n – 2)
4
and
Ans. (a)
Y = {9(n – 1) : n Œ N}
14. Since f is a bijection, so its inverse is exist.
= {0, 9, 18, 27, 36,...}
Given y = f (x) = (x – 1)2 – 1 fi YÃX
fi (x – 1)2 = y + 1 fi X»Y=X
_____ 20. Ans. (4)
fi (x – 1) = ​÷y  + 1 ​
  Here A × B has 8 elements respectively
_____
fi x = ​÷y  + 1 ​ – 1 Now, total subsets of A × B is 28
_____ Thus, the number of subsets of 28 having 3 or more
Thus, f  –1(x) = ÷
​ x  + 1 ​
 – 1 elements
It is also given that, f  –1(x) = f(x) = 28 – 8C0 – 8C1 – 8C2
Real Function  1.89

= 256 – (1 + 8 + 28) = (xn)1/n


= 256 – 37 =x
= 219 So, it is true.
5. Here, we can consider any value of f (x) other than 1
questions asked in Past Iit-JEE Examinations Let f (x) = 2
1. Given f (x) = cos x – x(x + 1)
x2 + 4x + 30
f ¢(x) = – sin x – 2x – 1 Then ​ ___________
  
   ​ = 2
x2 – 8x + 18
= – (sin x + 2x + 1)
fi 2x2 – 16x + 36 = x2 + 4x + 30

p p
< 0, " x Œ ​ __
​   ​ , __
6 3 [  ]
​   ​   ​
fi x2 – 20x + 6 = 0
Clearly, f is a decreasing function ________
20 ± ÷  
​ 400 – 24 ​ 
Thus, the range of the function f = f (A) fi x = ​ ______________
 ​
    

[  (  ) (  ) ]
2
p p
= ​ f ​ __
​   ​   ​, f ​ __
​   ​   ​  ​ ___
3 6 fi x = 10 ± ​÷94 ​
   

[ 
__

1 p 2 __
= ​ __
​   ​  – ___
2 9
p ÷     p 2 p
​ 3 ​
​   ​ – ​   ​ , ___
​   ​ , ​ ___ ​ – __
3 2 36 6 ]
​   ​   ​ Thus, for x = 10 + ÷
___
   and 10 – ​÷94 ​
​ 94 ​    
___

the value of f (x) = 2


2. Case-I:  Let f (x) = 1 is true
So, f is not one-one function. It is false.
Then f(y) = 1 and f (z) = 2 6. Ans. (d)
So, f is not one-one, since f (x) = 1 = f (y) Now, f (x2) = |​ x2 – 1 |​ π |x – 1|2
Case-II:  Let (y) π 1 is true f (x + y) = |x + y – 1| π |x – 1| + |y – 1|
Then f (x) π 1 and f(z) = 2
f (|x|) = ||x| – 1| π ||x – 1||
So, f is not one, since f (x) = 2 = f(y) 7. Ans. (d)
or f (x) = 2 = f (z), f (y) = 3 Given f (x) = cos (log (x))
Case-III:  Let f (z) π 2 is true
Then f (x) π 1, f(y) = 1
Now,
1 x
f (x) f (y) – __
2 y [  (  )
​    ​ ​ f ​ __ ]
​   ​  ​ + f (xy)  ​

Thus,
since
f is one-one,
f (x) = 2, f(y) = 1, f(z) = 3
[  1
(  ) x
= ​ __ ​ ​  2f (x) f (y) – f ​ __
2 ]
​ y ​  ​ – f (xy)  ​

​   ​ ​ [ 2 cos (log x) cos (log y)


1
Therefore, f –1(y) = 1 = __
[  ]
p 2
3. Clearly, the domain of the function is ​ 0, ​ __ ​   ​ and it
  – cos ​( log ​( __
4
​ y ​ )​ )​ – cos (log (xy)) ]​
x
is a decreasing function.
Hence, the range of the function is
​   ​ ​ [ cos (log x + log y) + cos (log x – log y)
[  (  )
1
= __

p
= ​ f  ​ __
​   ​   ​, f (0)  ​
4 ]
2

  – cos ​( log ​( __
[  ] ​ y ​ )​ )​ – cos (log (xy)) ]​
x
3__
= ​ 0, ​ ___  ​  ​
÷   
​ 2 ​
​   ​ ​ [ cos (log (xy))  +  cos ​( log ​( __​ y ​ )​ )​
1 x
= __
4. Given f (x) = (a – xn)1/n 2
f ( f (x)) = f (a – xn)1/n
  – cos ​( log ​( __

​ y ​ )​ )​ – cos (log (xy)) ]​
x
= f (b), b = (a – xn)1/n
=0
= ​( a – bn )1/n

8. Ans. (c)
= ​​( a – ​​{ (a – an)1/n }​​ ​ )​​ ​
n 1/n
_____
1
1/n Given f (x) = __________
​  ​ x  + 2 ​
     ​ + ÷  
= ​( a – (a – x ) )​ n
log10 (1 – x)
1.90  Differential Calculus Booster

f is defined for (1 – x) > 0, (1 – x) π 0, 1; x ≥ – 2 16. Let f (x) = ax + b, where f : [– 1, 1] Æ [0, 2]
fi x < 1, x π 1, 0; x ≥ – 2 Clearly, f (– 1) = 0, f (1) = 2
fi [– 2, 1) – {0}
or f (– 1) = 2, f (1) = 0
9. Ans. (a)
Here, f(x) = x – [x] , [.] = G.I.F ÔÏ – a + b = 0 Ï – a + b = 2
fi Ì or Ì
is a periodic function with period 1. ÓÔa + b = 2 Óa + b = 0
10. We have
Ïa = 1, b = 1

(  (  ) ) 2 fi Ì
x  ​   ​  ​
f (x) = sin–1 ​ log2 ​ ​ __ Óa = –1, b = 1
2
fi f (x) = x + 1, 1 – x

(  )
f is defined for – 1 £ log2 ​ ​ __
2
2
x  ​   ​ £ 1 17. Ans. (b)
Now, (g 0  f) (x)


x
(  )
2
2–1 £ ​ ​ __ ​   ​ £ 21
2
= g(f(x))


1
( 
= g ​ __
p x
​   ​  – tan ​ ___ (  ) )
​   ​  ​  ​

1
fi ​__
x2
(  )
   ​  £ ​ ​ __ ​   ​ £ 2
2 2
2 2

÷  ( 
_______________________________
fi 1£x £4 2
__

1
= ​ 3 + 4  ​ __
​   ​  – tan ​
2
    
px
___
2 (  ) ) (  1
​   ​   ​  ​ – 4​​ __
​   ​  – tan​ ___
2 (  ) )
px 2
​   ​   ​  ​​ ​ ​
2
fi 1 £ ​÷x  2 ​  £ 2

÷  (  ( 
______________________
fi 1 £ |x| £ 2
p x
(  ) ) )
1 2
= ​ 4 – ​​ 2​ __
​   ​  – tan ​
    ___
​   ​  ​  ​ – 1  ​​ ​
fi |x| ≥ 1, |x| £ 2 2 2

÷ 
_____________
fi x ≥ 1, x £ – 1 ; – 2 £ x £ 2
 ​
p x
___
= ​ 4 + 4 tan2 ​  
​   ​  ​ ​
2 (  )
fi x Œ [– 2, 1] » [1, 2]

÷  (  (  ) )
______________

(  ( 
_____
÷​ 4  – x  
12. Given f(x) = sin ​ ln​ ​ ______  
1–x
 ​

2
​  ​  ​ )) ​ p x ​  ​  ​ ​
___
= ​ 4​ 1 + tan2​  
2

÷ 
f is defined for __________

(  )
​ px
= ​  4 sec2 ​ ___
( 
_____  ​   ​ ​
)
  


÷​ 4  – x2 
​ ​ ______ 
 ​
 ​  ​ > 0
2
(1 – x)

(4 – x2) ≥ 0, 1 – x > 0


|  px
= 2​ sec ​ ___(  ) |
​   ​   ​  ​
2
(x2 – 4) ≥ 0, x <1 It is defined for, x Œ R – (2n + 1), n Œ I
– 2 £ x £ 2, x < 1 Also, 3 + 4x – 4x2 ≥ 0
Thus, Df = [– 2, 1) fi 4x2 – 4x – 3 £ 0
Clearly, the range of the function f is
fi (2x – 3) (2x + 1) £ 0
= [– 1, 1]
1 3
13. No questions asked in 1986. fi – ​ __ ​  £ x £ ​__
   ​ 
2 2
14. Clearly, f (x) = 0 Thus,
which is an even function as well as it has a 1 3
value at x = 0 – ​ __ ​  £ x £ ​__
   ​  ; – 1 < x < 1 ; x Œ R – (2n + 1), n Œ I
2 2
15. The domain of ( f 1(x) + f 2 (x)) is D1 « D 2 not
D1 » D2. It is false.

1 1
Therefore, the domain of (g 0  f ) is ​ – ​ __ ​ , __
2 2
​    ​  ​ [  ]
Real Function  1.91

18. No questions asked in 1991. fi (x + 1) ((x + 1)3 – 1) = 0


19. Ans. (b)
fi (x + 1) x ((x + 1)2 + (x + 1) + 1) = 0
Given f (x) = bx2 + cx + d
and f  (x + 1) – f (x) = 8x + 3 fi x (x + 1) (x2 + 3x + 3) = 0
b(x + 1)2 + c(x + 1) + d – (bx2 + cx + d) __
– 3 ± i​÷3 ​
  
=8x+3 fi x = 0, –1, ​ ________
 ​   
2
b((x + 1)2 – x2) + c(x + 1 – x) = 8x + 3
Hence, the solutions are
b(2x + 1) + c = 8x + 3
{ 
__ __
2bx + (b + c) = 8x + 3
2
    – 3 – i ​
– 3 + i ​÷3 ​
 ​ – 1, 0, ​  ________
 ​ 
2
  
÷3 ​
, ​ _______
 ​   ​ }
= 4 and c = – 1
20. Ans. (b)
Let y = 1 + ax
24. Given
x+1
x+2( 
f (x) = f  ​ ​ _____ 
 ​  ​ )


y–1
x = ​ _____
a    ​
  fi
x+1
x+2( 
x = ± ​ ​ _____ 
 ​  ​ )
x–1
fi f  –1(x) = ​ _____
a   
It is given that, f (x) = f   (x)
​ 
–1
fi ( 
x+1
x = ​ ​ _____ 
x+2 ) x+1
 ​  ​ and x = – ​ ​ _____ 
x+2
 ​  ​ (  )
fi x2 + x – 1 = 0 and x2 + 3x + 1 = 0
x–1
fi 1 + ax = ​ _____
a     __ __
– 1 ± ​÷5 ​
   – 3 ± ​÷5 ​
  
fi x = ​ _______
 ​  and x = ​ _______
 ​  
 ​ Comparing the co-efficients, we get, 2 2
a 2 = 1 and a = – 1 25. Ans. (d)
fi a = – 1 We have
21. No questions asked in 1993.
22. Ans. (d)
p
f (x) = sin2 x + sin2 ​ x + __ (  )p
​   ​   ​ + cos x cos ​ x + __
3
​   ​    ​
3 (  )
We have (f o g) (x) = f (g (x))
= f (ln |x|) = sin (ln |x|)

1
= __ [  p
​   ​  ​ 2 sin2 x + 2 sin2 ​ x + __
2 (  ) p
​   ​   ​ + 2 cos x cos ​ x + __
3
​   ​   ​  ​
3 (  )]
Thus,
Also,
R1 = {u : – 1 £ u £ 1}
(g o f ) (x)

1
= __ [  2p
( 
​   ​  ​ 1 – cos 2 x + 1 – cos ​ 2x + ​ ___ ​   ​
2 3 )
= g(f(x)) = g(sin x)= ln|sin (x)|
p
( 
3
p
+ cos ​ 2x + ​ __ ​   ​ + cos ​ __)
​   ​   ​  ​
3 (  ) ]


Since

0 £ |sin(x)|, £ 1 we get,
R2 = {v: – • < v £ 0}
1
= ​ __ [  1
​   ​  1 + 1 + __
2
p
​   ​  – cos2 x + cos ​ 2x + __
2
​   ​   ​
3 (  )
23. Ans. (a)
Let y = (x + 1)2 – 1
(  2p
3 )]
– cos ​ 2x + ​ ___ ​   ​  ​

fi (x + 1)2 = y + 1
_____
1 5
= ​ __ ​ ​  __
2 2 [  1
​   ​  – cos 2 x + 2 cos 2 x × __ ]
​    ​  ​
2

​   ​  ​[ __
fi (x + 1) = ​÷y  + 1 ​
​   ​  – cos 2 x + cos 2 x ]​

1 5
_____ = __
fi ​ y  + 1 ​
x=÷  – 1 2 2
5
Thus, f  –1(x) = ÷
_____
​ x  + 1 ​
 – 1
= ​ __ ​ 
4
Now, f (x) = f  –1(x) Now, (g0  f) (x)
_____
fi (x + 1)2 – 1 = ​÷x  + 1 ​
 – 1 = g (f (x))

(  )
_____
fi (x + 1)2 = ​÷x  + 1 ​ 5
  = g ​ __
​    ​  ​
4
4
fi (x + 1) = (x + 1) =1
1.92  Differential Calculus Booster

ax2 + 5x – 8 29. Ans. (b)


26. Let y = ​ ___________
    ​
a + 5x – 8x2 Let y = 3x – 5

fi y (a + 5x – 8x2) = a x2 + 5x – 8 y+5


fi x = ​ _____ ​  
3
(a + 8y) x2 + 5 (1 – y) x –(8 + ya) = 0
x+5
fi f  –1(x) = ​ _____
 ​ 

Since for all x in R, so D ≥ 0 3
fi 25 (1 – y)2 + 4(a + 8y)(8 + ya) ≥ 0 30. Ans. (b)

fi 25(y2 – 2y + 1) + 4 (8a + 64y a 2y + 8ay 2) ≥ 0 Let y = f (x) = 2x(x–1)


2

fi (25 + 32a) y2 + (4a 2 + 256 – 50)y + (25 + 32a) ≥ 0 fi y = 2​ x​ – x​


fi x2 – x = log2 (y)
fi (25 + 32a)y2 + (4a 2 + 206)y + (25 + 32a) ≥ 0
fi x2 – x – log2 (y) = 0
Since the range of a function is R, so
__________
fi (25 + 256a) > 0 and 1 ± ​÷1  + 4log  
2 (y) ​
fi x = ​  _______________
 ​     f(x)
2
fi (4a2 + 206)2 – 4 (25 + 32a) (25 + 32a) < 0
since f is defined for [1, •) to [1, •)
fi (4a2 + 206 – 50 – 64a) (4a2 + 206 + 50 + 64a) < 0
__________
fi (4a2 – 64a + 156) (5a2 + 64a + 256) < 0 1+÷ ​ 1  + 4log  
2 (y) ​
so, x = ​ ________________
 ​    
fi (a2 – 16a + 39) (a2 + 16a + 64) < 0 2
__________
fi (a – 3) (a – 13) (a + 8)2 < 0 1+÷ ​ 1  + 4log  
2 (x) ​
Thus, f –1(x) = ​ _______________
 ​    
2
fi (a – 3) (a – 13) < 0
31. Given equation is 2x + 2y = 2
fi 3 < a < 13
fi 2y = 2 – 2x
Hence, the value of is a (3, 13). fi y = log2 (2 – 2x)
2
3x
  + 5x – 8
when a = 3, f(x) = ___________
​      ​ It is defined for (2 – 2x) > 0
3 + 5x – 8x2
fi 2x < 2 = 21
3x2 + 5x – 8
= – ​ ___________
  
   ​ fi x<1
8x2 – 5x – 3
Thus, Df = (– •, 1)
(3x + 8)(x – 1)
= – ​ _____________
  
   ​ Ans. (d)
(8x + 3)(x – 1)
32. Ans. (d)
(3x + 8)
= – ​ _______   ​, x π 1
(8x + 3) Here, x2 + 3x + 2 = 0 gives x = – 1, – 2
55
f ¢(x) = – ​ ________
   The function log2 (x + 3) is defined for x + 3 > 0
(3x + 8)2
fi x > – 3
8
 ​ Thus, f ¢(x) is not defined at x = – ​ __ ​  fi x Œ (– 3, •)
3
So f is neither increasing nor decreasing function Thus domian of the function f is

Therefore, f is not one-one function. = (– 3, •) – (– 1, – 2)


27. No questions asked in 1997. 33. Here, g(x) = 1 + x – [x] = 1 + {x} ≥ 1
28. Ans. (a)
Thus, f (g(x)) = 1
Given g (f (x)) = |sin(x)|
__ 2
Ans. (b)
and f (g(x)) = (sin(​÷x 
  ​)  )
34. Clearly f is bijective.
__
fi f (x) = sin2 x and g (x) = ÷
​ x 
  ​  So, its inverse exists.
Real Function  1.93

1 Ans. (d)
Let y = x + __
​ x ​
38. Clearly g(x) is the inverse of f(x)
fi x2 – xy + 1 = 0 Let y = (x + 1)2
__
_____   ​ 
(x + 1) = ​÷y 
y ± ​÷y  2 – 4 ​  __
fi x = __________
​   ​      x=÷    ​ – 1
​ y 
2
__
_____ ______ Thus, f  –1(x) = g(x) = ÷    ​ – 1, x ≥ 0
​ x 
y+÷ ​ y  – 4 ​ __________
2
y – ​÷y  – 4 ​ 
2
fi x = ​ __________
 ​
    
, ​   ​
     39. Given f (x) = 2x + sinx
2 2
Since the values of x and y are positive, fi f ¢(x) = 2 + cos x > 0
_____ fi f is monotonic function
y + ​÷y  2 – 4 ​  fi f is one-one function
so x = ​ __________
 ​    
2 Also, Rf = R = Co-domain
_____
x+÷ ​ x  – 4 ​ 
2
Thus, f is one-one and onto function.
fi f  –1 (x) = ​ __________
 ​
    
2 Ans. (a)
Ans. (a) x 1
40. Given f(x) = _____
​      ​ = 1 – _____
​      ​
35. Given f (x) = (1 + b2)x2 + 2bx + 1 x+1 x+1
1
Now, m(b) = Min value of f (x) f ¢(x) = _______
​       ​ > 0
(x + 1)2
D
= – ___
​     ​
4a f is monotonic function
f is one-one function
(4b2 – 4(1 + b2))
= – ​ _______________
    
 ​ x
4(b2 + 1) Also, let y = _____
​      ​
x+1
1
= _______
​  2      ​ x y + y = x
(b + 1)
( – 1) x = – y
Thus, the range of m(b) = (0, 1]
Ans. (d) – y y
x = ______
​   ​ = ______
   ​       ​
36. The number of onto function (y – 1) (1 – y)
= 24 – 2
Thus, Rf = R – {1} π Co-domain
= 14
Ans. (a) Therefore f is not onto.
ax Ans. (b)
37. Given f(x) = _____
​      ​, x π – 1
x+1 x2 + x + 2
41. Given f (x) = ​ _________
     ​
x2 + x + 1
Now, f (f (x)) = x
(x2 + x + 1) + 1
fi (  ) ax
f ​ _____
​    
x+1
 ​  ​ = x = ​ ______________
  
  
(x2 + x + 1)
 ​
1
= 1 + __________
a ​( _____
​  2      ​
x + 1)
ax
​      ​  ​ (x + x + 1)
fi ​ _________    ​ = x
ax
_____ Let g(x) = (x2 + x + 1)
​      ​ + 1
x+1
Thus, the min value of g(x)
a2 x
fi ​ __________
     ​ = x D (1– 4) 3
ax + x + 1 = – ​ ___   ​ = – ​ ______
 ​ 
 = __
​   ​ 
4a 4 4
fi (a + 1)x2 + (1 – a 2) x = 0 4 7
Thus, max value of f(x) is = 1 + __
​   ​  = __
​   ​ 
fi (a + 1) = 0 & (1 – a ) = 0 2 3 3
Also, f(x) Æ 1 as x Æ •
fi a = – 1 & a = ± 1
fi a = – 1 Thus, (  ]
Rf = ​  1,
7
​ ​ __ ​   ​ ​
​ 3 ​
1.94  Differential Calculus Booster

Ans. (c) Now, if a à X fi f (a) à f (x) = d

÷ 
____________
p fi f  –1(f(a)) = a
42. Given f(x) = ​ sin – 1 (2x)    + __​   ​ 
6
Ans. (d)
p
 ​ It is defined for sin  (2x) + __
 – 1
​   ​  ≥ 0
6 47. The function f is defined for
p
fi sin – 1 (2x) ≥ – ​ __ ​   
8.3x
– 1 £ ​_________
   ​ £1
6
1 – 32(x–1)

p
6(  ) 1
2x ≥ sin ​ – ​ __ ​   ​ = – ​ __  ​
2 fi
3x – 3x–2
– 1 £ ​ ________  
   ​ £ 1
1 – 32(x–1)
1
fi x ≥ – __
​   ​ 
4 3x – 3x–2
Case-I: When ​ ________  
   ​ £ 1
Also, – 1 £ (2x) £ 1 1 – 32(x–1)
1 1 3 – 3x–2 –1 + 32x–2
​   ​  £ x £ ​__
– __    ​  fi ​ _________________
     £ 0
 ​
2 2 1 – 32x–2
Thus, [ 
1 1
Df = ​ – __
​   ​ , __
4 2 ]
​   ​   ​

(3x – 1)(3x–2 – 1)
fi ​_______________
    
   ​ £ 0
Ans. (a) (32x–2 – 1)
43. Given f (x) = sin x + cos x, g(x) = x2 – 1 fi x Œ (– •, 0] » (1, •)
Now, g(f (x)) = (sin x + cos x)2 – 1 3x – 3x–2
Case-II: when ________
​   
  ​ ≥ – 1
= sin 2 x 1 – 32(x–1)
Clearly g(f(x)) is invertible in the domain of 3x – 3x–2
fi ​ _______   ​+ 1 ≥ 0
p p 1 – 32x–2
– ​ __ ​  £ 2 x £ ​__
   ​ 
2 2
3x – 3x–2 + 1 – 32x–2
p p fi ​ _________________  
 ​   ≥ 0
– ​ __ ​  £ x £ ​__
   ​  1 – 32x–2
4 4


Thus,
Ans. (b)
[ 
p p
Dg( f(x)) = ​ – ​ __ ​ , __
4 4 ]
​   ​   ​
(3x–2 – 1)(3x + 1)
fi ​ _______________  
  
(3x . 3x–2 – 1)
 ​ ≥ 0

Ï x : If x is rational fi x Œ (– •, 1) » [2, •)


44. Given f(x) = Ì
Ó x : If x is irrational Thus, Df = (–•, 0] » [2, •)
where f  :  R Æ 48. Given f (x) = 2x3 – 15x2 + 36x + 1
Ï0, if x is rational fi f ¢(x) = 6x2 – 30x + 36
and g(x) = Ì
Ó x, if x is irrational fi f ¢(x) = 6(x2 – 5x + 6)
where g  :  R Æ R = 6(x – 2) (x – 3)
Now, f (x) – g (x)
Thus, f ¢(x) is positive at x < 2
Ï – x : x Œ Rational fi f ¢ (x) is negative at 2 < x < 3

Ó x : x Œ Irrational fi f ¢(x) is positive at x > 3
where ( f – g) : R Æ R Therefore f(x) is many-one function.

Clearly, (f – g) is one-one and onto. Also, Rf = [1, 29] = Co-domain

Ans. (a) Thus f is many-one onto function.


Ans. (b)
45. Given that X and Y are two sets and f : X Æ Y,
2 
49. Given f (cos 4q) = _________
​       ​ for
{f (c) = y; c à X, y à Y} and 2 – 2 sec2 q

(  ) ​( __​ p4 ​ , __​ p2 ​  )​
{f  –1(d) = x ; d à Y, x à X} p
q Œ ​ 0, __
​   ​   ​ »
since   –1
f (d) = x fi f (x) = d 4
Real Function  1.95

1 50. No question asked in 2013.


Let cos (4q) = __ ​   ​ 
3
1 51. We have f(x) = (log(sec x + tan x))3
fi 2 cos (2q) – 1 = __
2
​    ​
3 f(– x) = {log(sec(– x) + tan(– x))}3
2 4
__
fi 2 cos (2q) = ​    ​
3 = {log(sec(x) – tan(x))}3
2 2
__
fi cos (2q) = ​    ​

{  (  )}
3 sec(x) – tan(x)) (sec(x) + tan(x)) 3
= ​​ log ​ ​ ___________________________
÷ 
__
2           ​  ​  ​​​
cos(2q) = ± ​ ​ __ ​ ​   (sec(x) + tan(x))
3

Now, (  )
1
f  ​ __
3
​ 
2
​   ​   ​ = _________
2 – sec2q 
     ​ {  (  ​ 
1
= ​​ log​ ______________ )}
3
     ​  ​  ​​ ​
(sec(x) + tan(x))

2cos2q = {–  log (sec(x) + tan(x))}3


= __________
​      ​
2cos2q – 1 = – {log (sec (x) + tan(x))}3
cos 2 q + 1
= ​ _________   

  = – f (x)
cos 2 q
1
_____
= 1 + ​     ​  Thus, f(x) is an odd function.
cos2q
52. No questions asked in 2015, 2016.

(  ÷  )
__
3
= ​ 1 ± ​ __
​   ​ ​    ​
2
Ans. (a, b)
Chapter

3 The Limit

5. Concept of infinity
Concept Booster
Let us consider that n assumes sucessively the values 1, 2, 3,
... Then n gets larger and larger and there is no limit to
1.  Meaning of xÆa
the extent of its increase. It is convenient to that n tends to
The symbol x Æ a is called as x tends to a or x approaches infinity. When we say that n tends to infinity, we simply
to a. It implies that x takes values closer and closer to a mean it that n is supposed to assume a series of values which
but not equal to a. increas beyond limit.
A function y = f (x) which approaches infinity as x Æ a
2. Neighbourhood of a Point
does not have a limit in the ordinary sense.
Any open interval containing a point a as its mid-point is
called a neighbourhood of a. A positive number d is called 6. Concept of limit
a neighbourhood of a, if a – d < x < d + a. The concept of limit is used to discuss the behaviour of a
function close to a certain point
3.  Limit of a function
x2 – 1
If there is a number l such that x approaches to a, either Let f (x) = ​ ______ ​ 
,xπ1
from the right or from the left, f (x) approaches l, then l is x–1
called the limit of f (x) as x approaches to a. Clearly the given function is not defined at x = 1.
A number l is said to be a limiting value only if it is It is defined only when x π 1, that means, either x > 1
finite and real, otherwise we say that limit does not exist. or x < 1.
In a piece-wise defined function or more than one Case I:   When x > 1 (just slightly more than 1)
function, we shall use the concept of right hand limit as
well as left hand limit. In a function(s), if right hand limit x>1 f (x) = x + 1
as well as left hand limit both are exist and their values are 1.1 2.1
same, then limit exist, otherwise not. 1.01 2.01
4. Formal definition of a limit 1.001 2.001
1.0001 2.0001
A number l is said to be limit of the function f(x) at x = a,
for any positive number Œ > 0, there corresponds a positive Thus, x Æ 1+ fi f (x) Æ 2+
number d such that | f (x) – l | < Œ, " x Œ Df , x π a, |x – a|
we define it the right hand limit of a function
< d.
Case II:   When x < 1 (just slightly less than 1)
x<1 f (x) = x + 1
.9 1.9
.99 1.99
.999 1.999
.9999 1.9999
3.2  Differential Calculus Booster

x Æ 1– fi f (x) Æ 2–. m
Thus, (iii) ​ lim 
  ​ ( f (x) ÷ g(x)) = ​   lim ​ g(x) = __
lim ​ f (x) ÷ ​   ​ n ​ , n π 0
x Æ a x Æ a x Æ a
We define it the left hand limit of a function.
Both the concept simultaneously known as limit of a   ​ ​ (  f (x) g(x)  )​ = ​  
(iv) ​ lim  lim ​  ( f (x)​)​ ​x Æ a
  lim ​  g (x)
​ = mn

(  )
x Æ a x Æ a
x2 – 1
lim ​ ​ ​ ______ ​  
function. i.e. ​   ​= 2   ​  (kf (x)) = k ​( ​  
(v) ​ lim  lim ​  ( f(x)) )​ = km
x Æ 1 x – 1 x Æ a x Æ a

| 
  Note:   ​ ​| f(x) |​ = ​ ​  
1. The value of a function at any x Œ Df is the the exact
(vi) ​ lim 
x Æ a
lim ​  ( f (x)) ​ = |m|
x Æ a |
value of a function.
  ​  e f(x) = e​  ​​  
(vii) ​ lim  lim ​  ( f(x))
x Æ a ​ = e m
2. But the limiting value of a function is the closer x Æ a

value of the exact value of a function.   ​ f(g(x)) = f ​( ​  


(viii) ​ lim  lim ​( g(x)) )​ = f(n)
x Æ a x Æ a
3. If the limiting value of a function is a real and finite
___ _______
÷ x Æ a
then limit is exist, otherwise not. n __
n
  ​ ​÷
(ix) ​ lim     =
 f(x) ​
x Æ a
n
​ ​  
lim ​ f (x) ​ 
= ​÷   ​ 
m 
4. We shall use the concepts of R.H.L as well as L.H.L
only when, we get a piece-wise defined function or   ​ ( f(x)) n = (​​  ​  
(x) ​ lim  lim ​ f (x) )n​​ ​ = mn
x Æ a x Æ a
more than one function.

Example 1. ​ lim 
  ​  [x] Some Important expansions to remember
x Æ 1
n(n – 1) 2
(i) ​ lim 
  + ​ [x] = 1 (i) (1 + x) n = 1 + nx + ​ _______  ​   x
x Æ 1 2!
(ii) ​ lim 
  – ​ [x] = 0 n(n – 1)(n – 2) 3
x Æ 1 + ​ _____________  ​     
x + ...
3!
5. In a function, if the R.H.L and L.H.L both are exist
x2 x3 x4
and their values are same, then limit is exist. (ii) ex = 1 + x + ​ __  ​ + ​ __  ​ + ​ __  ​ + ...
2! 3! 4!
Ï x2 : x >1
Example 2.  Let f (x) = ÔÌ (logea)2 2 _______
(iii) ax = 1 + (logea)x + ​ _______
 ​ x

(logea)3 3
  + ​   ​ x

  + ...
ÔÓ2 – x : x < 1 2! 3!
Then ​ lim 
  +​ f (x) = 1 x2 x3 x4
x Æ 1 (iv) log (1 + x) = x – ​ __ ​  + ​ __ ​  – ​ __ ​  + ...
2 3 4
and ​ lim 
  –​ f (x) = 1.
x Æ 1
x2 x3 x4
Thus ​ lim 
  ​ f (x) = 1. (v) log (1 – x) = – x – ​ __ ​  – ​ __ ​  – ​ __ ​  – ...
x Æ 1 2 3 4
6. In a function, if the R.H.L and L.H.L both are exist x 3 x5 x 7
(vi) sin x = x – ​ __  ​ + ​ __  ​ – ​ __  ​ + ...
and their values are not same then limit does not 3! 5! 7!
exist.
x2 x4 x6
(vii) cos x = 1 – ​ __  ​ + ​ __  ​ – ​ __  ​ + ...
Example 3.  Let ​   |x|
lim ​ ​ ​ __ (  )
x ​   ​.
x Æ 0
2! 4! 6!
x3 2
Then R.H.L = ​ lim 
  +  (  )
|x|
​ ​ ​ __
x ​   ​ = ​ lim 
  + 
x Æ 0 x
x
(  )
​ ​ __
​   ​  ​ = 1 (viii) tan x = x + ​ __ ​  + ___
3
​    ​  x5 + ...
15

( 
x Æ 0

and L.H.L = ​   ​ ​( ​ __


lim  
|x|
x Æ 0 – x ​  )​ = ​   ​ ​( __
lim  
x Æ 0 –
x
​ x ​ )​ = –1
(ix) (1 + x)1/x = e ​ 1 + __
x 11 2
​    ​ + ___
2 24
​    ​ x + ...  ​ )
x2 x3
Thus, limit does not exist at x = 0. (x) f (x + 0) = f(0) + xf ¢(0) + ​ __  ​  f ≤(0) + ​ __  ​  f ¢≤(0) + ...
2! 3!
7. Algebra of limits which is known as Maclaurin Series.
Let f and g be two real functions defined in the domain
D. 8. Evaluation of limit
(i) Algebraic Limit
If ​  
lim ​    f (x) = m and ​  
lim ​  g (x) = n, where m and n are real
x Æ a x Æ a (a) Direct Substitution Method (DSM)
and finite, then (b) Factorisation Method (FM)
  ​  ( f(x) ± g(x)) = ​  
(i) ​ lim  lim ​ f(x) ± ​  
lim ​ g(x) = m ± n (c) Rationalisation Method (RM)
x Æ a x Æ a x Æ a
(d) Standard Result Method (SRM)
(ii) ​ lim 
  ​ (  f (x) × g(x)) = ​  
lim ​ f (x) × ​  
lim ​ g(x) = m × n
x Æ a x Æ a x Æ a (e) Infinity Method (IM)
The Limit  3.3

(ii) Non-algebraic Limit


(a) Trigonometric limit x Æ 0
x​( ÷
_________
( 
x2 + x + 1 – x2 – 1
lim ​ ​ ​ ______________________
  = ​      
   
​ x  2 + x + 1 ​ 
______  ​  ​
)​
​ x  2 + 1 ​  
+÷ )
( 
(b) Inverse trigonometric limit
(c) Exponential limit x Æ 0
​  _________
(
x​  ÷
  
x
lim ​ ​ ______________________
  = ​      
​ x  + x + 1 ​
2

______  ​  ​
)​
​ x  2 + 1 ​  
+÷ )
( 
(d) Logarithmic limit


(e) Miscellaneous limit.
(i) L’Hospital Rule
  = ​  
x Æ 0 (
​  _________
​  ÷
  
1
lim ​ ​ _____________________
   
​ x  + x + 1 ​ 
2
______  ​  ​
)​
​ x  2 + 1 ​  
+÷ )
(ii) Advances Exponential limt (1• ) 1
  = __
​   ​ 
2
(iii) Sandwitch Theorem
(iv) Newton and Leibnitz limit (iv) Standard Result method
(v) Definite integral as the limit of a sum. In this method we shall use the formula


(vi) Limits of the form of (00)
(vii) Limits of the form of (1• ).
​ lim 
xn – an
  ​ ​ _______ ( 
​  ​ = nan – 1.
​  x – a   

x Æ a
)
8.1  Algebraic limit    
x – 32
​ ​ _______
Example 9. ​ lim  ​ 
x Æ 2
(  )
 ​  


5

x–2

(  )
(i) Direct substitution method
x5 – 25
In this method, we can directly sustitute the number lim ​ ​_______
  = ​   ​  ​ = 5.25 – 1 = 5 × 16 = 80
 ​  

x Æ 2 x–2

( 
at which the limit is to be find. After substitution,
if we get a finite value, that is the limiting value of
x – 243
lim ​ ​ ​ _______
Example 10. ​    
5
 ​  ​ )
(  )
a function. x Æ 3 3
x – 27
Example 4. ​  
lim ​ (x2 + 3x + 4) = 8.
x5 – 35
x Æ 1
​ ______ ​ 
Example 5. ​  
lim ​ (x3 – 5x + 4) = 0. x–3 5.35 – 1

x Æ 1   = ​   ​ ​ ​ ______
lim   3 3
 ​  ​ = ______
  ​  3 – 1 ​ 
= 15
x Æ 3 x – 3 3.3
______
​   ​ 
(ii) Factorisation method x–3

( 
In this method, we shall find out a common factor _____
from the numerator as well as a denominator. Cancel
the common factor and then directly substitute the

3 – ​÷5  + x  
lim ​ ​ ​ __________
Example 11.  Evaluate: ​  
x Æ 0
  

   ​  ​
_____
1 – ​÷5  – x  
​)
number at which the limit is to be find.

( 
_____

(  )
x –1 2
lim ​ ​ ​ ______ ​ 
Example 6. ​  
x Æ 1 x–1
  ​
We have ​  
lim  
3 – ​÷5  + x  
​ ​ ​ __________
x Æ 0 1 – ​÷5
  
_____
  – x  

   ​  ​

__
)
  
3 – ​÷5 ​
  = ​  
x Æ 1
(x + 1)(x – 1)
lim ​ ​ ____________
​ 
x–1
  
 ​   ( 
​ = ​  
lim ​( x + 1) = 2
x Æ 1 )   = _______
​  __ ​   
  
1 – ​÷5 ​
.

(  )
(v) Infinity Method
​ •
1
 – ​ __  ​
1_______
– ​x​ 3 ​ (a) Form: ​( __
• ​ )​. In this method, we shall write down
Example 7. ​  
lim ​ ​​  2

 ​  

x Æ 0 – ​ __ ​  the given expression in the form of a rational
1– ​x​ 3 ​

( (  )(  ) )
f(x)
1
 – ​ __ ​ 
function. i.e. ____​     ​. 
1 – ​x​ 3 ​ g(x)
lim ​ ​​ ________________
  = ​          ​  ​
1 1
x Æ 0 – ​ __ ​  – ​ __  ​ Then divide the numerator and denominator by
​ 1 + ​x​ 3 ​  ​​ 1 – ​x​ 3 ​  ​
the highest power of x and then use

(  )
1
  = __
​   ​  1
2 ​ lim 
    ​ ​ __
​     ​  ​ = 0, x > 1
x Æ • xn
(iii) Rationalisation method
In this method, our first aim will be, remove the
radical sign. This is particularly used when either Ï• : x >1
Ô0 : 0 < x <1
the numerator or denominator or both involve the Ô
  ​ xn = Ì
Note: ​ lim 
fractional powers. x Æ •
Ô1 : x =1
_________ ______

Example ( 
÷​ x  2 + x +  1 ​ ​ x  2 + 1 ​ 
–÷
lim ​ ​ ​ ___________________
8.  Evaluate: ​  
x Æ 0
   
x     ​  ​ ) ÔÓnot defined : x<0
3.4  Differential Calculus Booster

(b) Form (• ± •): In this method, first we reduce the


function f(x) into a rational function by rationalisation
i.e. multiply the numerator and denominator by its
​ lim 
   
x Æ 0
f(x)
​ ​ ​ ____  
g(x) (  )
 ​  ​ = ​ l im  
x Æ 0
f ¢(x)
​ ​ ​ ____  
g¢(x) (  )
 ​  ​ = ​ l im  
x Æ 0
f ≤(x)
​ ​ ​ _____ 
g≤(x)
  (  )
 ​  ​,

conjugate and then apply ​   (  )


1
lim ​ ​ ​ __   ​  ​ = 0, x > 1.
x Æ • xn removed.
0
0
• ​ )​ is
untill and unless the form of ​ ​ __  ​  ​ or ​( ​ __
• (  )
8.2  Non-algebraic Limit L’Hospital rule is applicable only when the lim-
Before solving the value of a limit, please read throughly
each of the following formulae.
0
its are in the form of ​ __ ​ •
​   ​   ​ or (​  __
0 • ​ )​. (  )
(a) Trigonometric limit
Note:  Indeterminate form:

(  )
(i) ​ lim 
  ​ sin x = 0
0
x Æ 0
(i) ​ __
​    ​  ​ ​ •
(ii) ​( __
• ​ )​
(ii) ​ lim 
  ​ cos x = 1 0
x Æ 0
(iii) (•, – •) (iv) (0, •)
(iii) ​ lim 
  ​ tan x = 0 0
x Æ 0 (v) (• ) (vi) (00)
(iv) ​ lim 
x Æ 0
(  )
sin x
  ​ ​ ​ ____ ​  ​ = 1
x    (vii) (1• ) (viii) (•0)

(v) ​ lim 
x Æ 0
( 
sin (x – a)
  ​ ​ ​ _________
x – a  ​  
 ​= 1 ) (ii) Advanced Exponential limit
1
__
  ​  (1 + x​)​ ​x ​ ​= e
(a) ​ lim 
(vi) ​ lim  (  )
tan x
  ​ ​ ​ ____ ​  ​ = 1
x   
x Æ 0
x Æ 0
( 
1 x
   ​ ​​ 1 + __ )
( 
(b) ​ lim  ​ x ​  ​​ ​ = e

x Æ a
tan (x – a)
  ​ ​ ​ _________
(vii) ​ lim 
(x – a)
 ​  
  )
​= 1

x Æ •

(iii) Sandwich Theorem


(viii) ​ lim 
x Æ •
(  )
sin x
  ​ ​ ​ ____
x  ​  
 ​ = 0 Let f (x), g(x) and h(x) be three real functions
and a Œ R such that g(x) £ f(x) £ h(x).
(ix) ​ lim 
x Æ • ( 
cos x
  ​ ​ ​ ____ ​  ​ = 0
x    ) If ​   lim  
​ g (x) = m and ​  
x Æ a
lim ​ h (x) = m, then
x Æ a
​ lim 
  ​ f (x) = m.
(x) ​ lim 
x Æ •
(  )
tan x
   ​ ​ ​ ____ ​  ​ = 0
x   
x Æ 0

(b) Inverse Trigonometric limit

(i) ​ lim 
x Æ 0
(  )
sin–1x
  ​ ​ ​ _____
x   ​  ​ = 1

  ​ ​(​  ______ )​ = 1
–1
tan  x
(ii) ​ lim  x  ​  

x Æ 0

(c) Exponential limit


  ​ ax = 1, a π 1, a > 0.
(i) ​ lim 
x Æ 0
  ​ ex = 1
(ii) ​ lim  (iv) Newton and Leibnitz Rule
x Æ 0
If the function j (x) and Y(x) be defined on [a,
(iii) ​ lim (  )
x Æ 0
ax – 1
  ​ ​ ​ ______
x  ​  
 ​ = logea b] and differentiable at a point x Œ (a, b) and
f(t) is a continuous for j (a) £ t £ j (b) then​
  ​ ​(​  _____ )​ = 1
{ 
x

}
e –1 Y(x)
(iv) ​ lim  x  ​  
  d
x Æ 0
​     ​   ​Ú  ​ ​ ​f (t)dt  ​ = f(Y(x))Y¢(x) – f(j(x))f¢(x)
__
dt j (x)
(d) Logarithmic limit
j(x)

(i) ​ lim 
x Æ 0
( 
log(1 + x)
  ​ ​ ​ _________
x    )
​  ​ = 1 Note: 1. If y = Ú​  ​   ​ ​ f x(t)dt
a
dy
(e) Miscellaneous limit then ​ ___  ​ = f (j (x))f ¢(x)
dx
(i) L’Hospital Rule a

Let f(x) and g(x) be two real functions and 2. If y = Ú​    ​ ​  f(t)dt


a Œ R. If f(a) = 0 = g(a) or f(a) = • = g(a), then y (x)
The Limit  3.5

1
dy (ii) ​ __
n ​ is replaced by dx
then ​ ___  ​ = –f(y (x))y ¢(x)
dx
  ​  
(iii) ​ lim S is replaced by Ú

{  }
n Æ •
x7


d
 ​ ___   ​ ​ ​Ú ​ ​ ​logt dt  ​
dx x3
lim ​ ​ ____
(iv) a = ​  
n Æ •
(  )
j (x)
​  n  ​  
 ​, & b = ​  
n Æ •
Y(x)
lim ​ ​ ____
​  n   
​  ​. (  )
(vi) Form (0 0): In this method, we shall get the
  = {​  log ​( x 7 )​ ◊ 7x6 – log​( x3 )​ ◊ 3x2 }​

{  }
x5 lim ​ (f(x)) g(x).
function is in the form of ​  
x Æ a
d
 ​ ___   ​ ​ ​Ú ​ ​et dt  ​ If ​  
lim ​  ( f(x)) = m, m > 0 and ​  
lim ​  (g(x)) = n
dx x3 x Æ a x Æ a

  = {​ ​e  x​ ​ ◊ 5x4 – e​ x​ ​ ◊ 3x2}   ​ a finite quantity, then ​  


lim  ( f(x)) g(x) = mn.
​  
5 3
x Æ a

(v) Definite Integral as the limit of a sum We write it in the form of


b
1 Y(x)    ​ ( f(x)) g(x) = ​e​​ l 
im  ​( g(x) log ( f(x)))
  ​ __​ n ​  ​S   ​ ​ f ​ __
r
(  )
​ n  ​  ​ = Ú​  ​  ​ f (x) dx,
​ lim  ​. Where the
x Æ a
​ lim  x Æ a
n Æ • r = j (x) a function f(x) and g(x) are defined in the
where neighbourhood of a and f (x) > 0.
r
(i) ​ __
n ​  is replaced by x (vii) Form (•0)

Exercises
_____ _____
÷   + x ​ 
​ 1    – x ​ 
– ​÷1 
lim ​ ​ ______________
10. Evaluate: ​          ​
(Problems Based on Fundamentals) x Æ 0 2x
______ ______
– ​÷5  + 2x ​ 
​ 1  + 4x ​ 
÷
Direct Substitution Method (DSM) lim ​ ​ ________________
11. Evaluate: ​    ​
    
x Æ 0 x–2
2

( 
1. Evaluate: ​  
lim ​  (x – 6x + 10) ______ __ __

)
x Æ 1
​ 7  + 2x ​ 
÷    – ​÷2 ​
– ​÷5 ​   
lim ​ ​ ( x2016 – x2017 + 2 )​
2. Evaluate: ​   12. Evaluate: ​   
lim   ​ _______________
___​ ​   
 ​ 
   ​
x Æ 1    
x Æ ​÷10 ​ x2 – 10
______ ___
Factorisation Method (FM) ​ a  + 2x  
÷   ​ 
​ – ​÷3x 
lim ​ ​ _____________
13. Evaluate: ​   ______    __ ​

(  )
x Æ a ÷   + x  
​ 3a ​ – 2​÷x 
   ​
x2 – 3x + 2
lim ​ ​​ __________
3. Evaluate: ​       ​  ​
x Æ 1 x 2 – 5x + 4
3 _____

(  )
​ 7  – x  
÷ ​– 2
1    ​ ​ _________
14. Evaluate: ​ lim     ​ 
– ​ __ ​  x Æ –1 (x + 1)
1 – ​x​ 3 ​
lim ​ ​ ​  ______2 
4. Evaluate: ​      ​  ​

(  )
x Æ 1 – ​ __  ​ _______ ______
1 – ​x​ ​
3
÷​ x  ______
2
+ 8 ​ – ÷   _____
​ 10 – x2 
 ​
lim ​ ​ ​ _________________

( 
15. Evaluate: ​     
   ​  ​

) ÷​ x  2 + 3 ​ –÷ ​ 5  – x2 
x Æ 1
x3 + 7x2 – 36  ​
lim ​ ​ ​ ____________
5. Evaluate: ​     
   ​  ​
x Æ 2 x2 + 2x – 8

(  )
Standard Result Method (SRM)
x2 + x log x – log x – 1
lim ​ ​ ___________________
6. Evaluate: ​   ​       ​     ​ __ __
x3 – 1    ​ – a​÷a ​
x​÷x     
lim ​ __________
x Æ 1
16. Evaluate: ​   ​  x – a       ​
x Æ a
Rationalisation Method (RM) 5
​ __ ​ 
5
​ __ ​ 
(x + 2​)​3 ​ – (a + 2​)​3 ​
x lim ​ _________________
17. Evaluate: ​   ​  x – a        ​
lim ​ ______________
7. Evaluate: ​   ​  _____      ​
_____ x Æ a
x Æ 0 ​÷a   +  x ​ 
– ​÷a  – x ​ 
_____ xm – a m
8. Evaluate: ​  
x Æ 0
÷
( 
​ 1  + x ​ 
–1
lim ​ ​ ​ __________
x    ​  

_____
) lim ​ _______
18. Evaluate: ​   ​  n
x Æ a x – an

  

3 – ​÷5  + x ​  (1 – x) n – 1
lim ​ ​ __________
9. Evaluate: ​       ​ lim ​ ___________
19. Evaluate: ​   ​  x       ​.
_____
x Æ 0 1 – ​÷5   – x ​  x Æ 0
3.6  Differential Calculus Booster

20. Evaluate: ​  


lim  
x Æ 1
(x + x2 + x3 +  ...  + xn) – n
​ ​  ______________________
   
x–1
 ​    37. If ​  
lim 
x Æ 0
( 
x2 + x  +  1
​ ​ ​  _________
x + 1
 ​
  )
 – ax – b  ​ = 4, then find the

xn + 1 – (n + 1)x + n values of a and b.

( 
​ _________________

)
21. Evaluate: ​  
lim   ​   ​
    
x Æ 1 (x – 1)2 1
x4sin ​ __ (  )
​ x ​  ​ + x2

( 
______ 38. Evaluate ​   lim ​ ​ ​  ___________   
 ​   

)
3 4 ______
1  + x2 ​ 
​÷________________
– ​÷1  – 2x ​  x Æ – •
1 + |x|3
22. Evaluate: ​  
lim ​ ​ ​   ​ 
  
   ​
x Æ 0 x + x2 Trigonometric Limit

( 
______ ______

23. Evaluate: ​  


lim ​ ​ ​ 
x Æ 1
3
7  + x3 ​ 
2
– ​÷3  + x2 ​ 
​÷________________
x –  1
 ​ 
  
   ​ ) 39. Evaluate: ​  
lim 
x Æ 0 (  )
sin3x
​ ​ _____
​     
5x
​  ​

(  ) lim ​ ​( _____
__ __ __

sin bx )
​3 x 
÷
4
  ​  + ​÷x 
5
  ​  + ​÷x 
   ​ – 3 sin ax
lim ​ ​ _______________
24. Evaluate: ​   ​   ​ 
      ​ 40. Evaluate: ​   ​   ​  

x Æ 1 x–1 x Æ 0

25. Evaluate: ​  


( S  )
( 
100
​ ​  ​ ​ ​xk  ​ – 100
lim ​ ​ ​ ___________
k = 1
  
 ​  ​ ) 41. Evaluate: ​    ​ sin 2x
lim ​ ​(__________
x Æ 04x + sin 6x )
+ 3x
  ​   ​
  

lim ​ ​(​  ____________
x–1
sin 4x + sin 6x )
x Æ 1
sin 2x + sin 5x
42. Evaluate: ​      ​   ​
x Æ 0
Infinity Method (IM)

lim ​ ​(​  _________ 
1 – cos 6x )
1
__
​   ​ 1 – cos 4x
lim ​ ​​( 4n + 5n )​​n ​
26. Evaluate: ​   43. Evaluate: ​    ​  ​
n Æ • x Æ 0

27. Evaluate: ​   ( 


n
​ ​ ​ ________________
lim         ​  ​
n Æ • 1 + 2 + 3 + ... + n
2
) 44. Evaluate: ​  
x Æ 0 ( 
​ 1  – cos 2x  
÷
lim ​ ​ ​ __________
2
 ​ 

    
_________
​ )
( 
x

)
n n
x +y
​ ​ ​ _______ 
( 
28. Evaluate: ​  
lim    ​  ​, where 0 < x < y.
n Æ • xn – yn

______
45. Evaluate: ​  
cos 7x – cos 9x
lim ​ ​ ​ _____________
x Æ 0 cos 3x – cos 5x
    ​  ​ )
(  x )​
( 
29. Evaluate: lim ​ ​ ​÷x  2 + 2x  
​   ​– ______

30. Evaluate: ​   


x Æ •

( ​÷x  2 + 8x  
lim ​ ​ 
______
​ + x )​
x Æ 0
1 – cos x ÷
x 2

​ cos 2x 
lim ​ ​ ​ _______________
46. Evaluate: ​    ​      


​ )
( 
x Æ – •
tan x – sin x
lim ​ ​ ​ __________
)
( 
47. Evaluate: ​    ​ 
    ​
)
2 2x + 5
_____
31. Evaluate: ​  
x + 2x – 1 ​ 2x – 7 
lim ​ ​​ ​ ___________
  
    ​  ​​
 ​

x Æ 0 x3

( 
x Æ • 2

)
2x – 3x – 2 1 – cos x cos 2x cos 3x
lim ​ ​ ​ ___________________
48. Evaluate: ​            ​  ​
32. Evaluate: x Æ 0 sin2 2x

​ lim 
x Æ • ( 
(x + 1)10 + (x + 2)10 + ... + (x + 2012)10
  ​ ​ ​ __________________________________
    
x10 + 201210
 ​ 
    ​ ) 49. Evaluate: ​  
x Æ 0 ( 
sin–1x – tan–1x
lim ​ ​ _____________
​ 
x3
 ​ 
    
​ )
33. Evalaute: 50. Evaluate: ​  
x – sin x
lim ​ ​ ​ _______
( 
 ​   ​
)
( 
x Æ 0 x3
​ lim 
n Æ •
1 ◊ n + 2 ◊ (n – 1) + 3 ◊ (n – 2) + ... + n ◊ 1
  ​ ​ ​ __________________________________
    
n3
 ​     ​ ) 51. Evaluate: ​  
x Æ 0
x – tan x
lim ​ ​ ​ _______
x3 ( 
 ​  
 ​
)
34. Evaluate:
52. Evaluate: ​   ( 
x + sin x
lim  ​ ​ ________
​  ​  
   ​ )
{ ÷(  
__________________________________

}
________________________

)÷(   
______________ x Æ • x + cos x

n Æ •
1
  ​n 2 ​ ​ ​ 1 – cos ​ __
​ lim 
1
– cos ​ __
n ​  ​​ ​ 1     n ​  ​​ ​   
1
1 – cos ​ __
n ​     ) ÷(   
​ ... • ​ ​ ​  ​ ) lim ​ ​( ​[ ​ _____
53. Evaluate: ​  
n sin x
​  ]​ + ​[ ​ ______
n tan x
​  ]​ )​
x    x   

( 
x Æ 0

35. If ​  
x2 – 1
lim ​ ​ ​ ______ ​ 
x Æ • x + 1 )
– ax – b  ​ = 2, find the values of a 54. If = min {x2 + 2x + 3, x2 + 4x + 10}

and b. and b = ​  


x Æ 0
1 – cosq
lim ​ ​ ​ ________
q 2 ( 
 ​  
)
 ​, then find the value

36. If ​  
x Æ • x + 1 ( 
x +1
lim ​ ​ ______
​   ​ 
2
)
– ax – b  ​= 0, find the values of a and b. n
​   ​ ​ ​(arbn – r ).
of S
r = 0
The Limit  3.7

55. If ​  
x Æ 0 ( a cos x + bx sin x – 5
lim ​ ​ ​ _________________
x4
 ​   
)
   ​ exists and finite, then 73. Evaluate: ​  
x Æ 2
( 
(cos a) x + (sin a) x – 1
lim ​ ​ ​ ___________________
   
x–2
 ​     ​ )
( 
find the value of a + 2b + 10.
ex cos x   – 1 – x
lim ​ ​ ​ ____________
)
( 
74. Evaluate: ​    ​ 
     ​

) sin ​( x2  )​
2 x Æ 0
sin(p cos  x)
lim ​ ​ ​ __________
56. Evaluate: ​    ​ 
    ​
x Æ 0 x2 75. If a = min {x2 + 4x + 6, x2 + 2x + 8}

( 
Exponential Limit
sin x cos x
lim ​ ​ ​ ________ )
( 
and b = ​    
​  ​, then find the value of
  
57. Evaluate: ​  
x Æ 0
e –1
lim ​ ​ ______
​ 
5x
4x
   
​  ​ ) x Æ 0
n
ex – e–x

​S  ​ ​ (​  arbn – r )​
58. Evaluate: ​  
x Æ 0 e – 1 (  )
e3x – 1
lim ​ ​ ______
​  5x  
 ​  ​
r = 0

( 
Aex – B cos x + Ce–x
lim ​ ​_________________
76. If ​   ​       )
  ​  ​ = 2 then find the value

( 
x sin x
)
x Æ 0
ex + e–x – 2
lim ​ ​ ​ __________
59. Evaluate: ​    ​ 
    

x Æ 0 x2 of A + B + C + 10.

e3 + x – sin x – e3
lim ​ ​ ______________
60. Evaluate: ​   x     ​ 
Logarithmic Limit

( 
x Æ 0


ex – esin x
lim ​ ________
61. Evaluate: ​   ​   ​
  x Æ 0
log(1 + 3x)
lim ​ ​ ​ __________
77. Evaluate: ​  
sin2x
  
   ​  ​ )
( 
x Æ 0 x – sin x

e –e
( 
x
lim ​ ​ ​ _________ ​  
62. Evaluate: ​    ​
x cos x
) x Æ 0
log(1 + 3x)
lim ​ ​ ​ __________
78. Evaluate: ​       ​  ​.
log(1 – 2x) )
x Æ 0 x + sin x
79. Evaluate: ​  
log x – 1
lim ​ ​ ​ ________
x – e   ( 
​  
​ )
(  )
x Æ e
ex – 1 – x
lim ​ ​ ​ _______
( 
63. Evaluate: ​    ​  
 ​
x Æ 0 x2 log x – log 5
lim ​ ​ ​ __________
80. Evaluate: ​     
   ​  ​. )
( 
x Æ 5 x–5
)
x x x
8 –4 –2 +1
lim ​ ​ _______________
( 
64. Evaluate: ​   ​   ​ 
     ​
x Æ 0 x2
9x – 2.6x + 4x
x Æ 5
log(x + 5) – log(5 – x)
lim ​ ​ ​ ___________________
81. Evaluate: ​      
x–5
 ​     ​ )
lim ​ ____________
( 
65. Evaluate: ​   ​   ​
    
x Æ 0 x2
82. Evaluate: ​  
ex – log(x + e)
lim ​ ​ ​ _____________
ex – 1
  
 ​  
​ )
( 
x Æ 0
a x – aa
)
( 
lim ​ ​ _______
66. Evaluate: ​   ​  x – a   

​ ​, a > 0
ln(cos x)
)
x Æ a
lim ​ ​ __________
83. Evaluate: ​   ​ 4 _____       ​  ​.

(  ) ÷​ 1  + x2 
x Æ 0
x + h
a  + a – 2a x – h  x  ​ – 1
lim ​ ​ ​ _____________
67. Evaluate: ​    ​ 
    ​

( 
h2
)
x Æ 0
ln(cos(sin x))
lim ​ ​ ​ ___________
84. Evaluate: ​    ​ 
    ​
(  ) x2
x x x Æ 0
8 – 7 
lim ​ ​ _______
68. Evaluate: ​   ​  ​   ​

x Æ 0 6x – 5x
85. Evaluate: ​   (  1
lim ​ ​ x – x2   ln ​ 1 + __ ( 
​ x ​  ​  ​ ))
( 
x Æ •

)
x x

(  )
(5 – 1) (4 – 1)
lim ​ ​ ______________
69. Evaluate: ​   ​  x   
   ​  ​ x x – 1
lim ​ ​ ​ _____ ​  
86. Evaluate: ​   ​
x Æ 0 (3 – 1)(6x – 1)
x Æ 1 x ln x

lim ​ ​( ___
70. Evaluate: ​   ( 
x ◊ 2x – x
lim ​ ​ ​ ________ 
x Æ 0 1 – cos x
  )
 ​  ​ 87. Evaluate: ​  
x Æ 1ln x x – 1)
1
​      
1
​ – _____
​       ​ ​

lim ​ ​(​  _______ 
lim ​ ​( ​ ___________ 1 – cot x )
ln cot x
x – sin x )
x –x 88. Evaluate: ​     ​  ​
e – e – 2x p
71. Evaluate: ​         
​ ​ x Æ ​ __ ​
x Æ 0 4

( 
L’Hospital Rule

)
3

( 
e​x​ ​ – 1 – x3
lim ​ ​ ​ __________
72. Evaluate: ​  
x Æ 0
  
sin6 2x
   
​  ​
89. Evaluate: ​  
x Æ 0
ex – 1 – x
lim ​ ​ ​ _________
x2
 ​  

​ )
3.8  Differential Calculus Booster

( 
n __ n
90. Evaluate: ​  
x Æ 0
e​x​ ​ – 1 – x3
lim ​ ​ ​ __________
64x6
  
 ​  ​
3

) 111. Evaluate: ​  


lim  
n Æ •
(    ​ 
a – 1 + ​÷b 
​ ​​ ​ __________
a     ​   ​​ ​ )
( 
Sandwitch Theorem
91. Evaluate: ​  
x Æ 0
ex – e–x – 2x
lim ​ ​ ​ ___________
  
x – sin x
  ​  
​ ) (  (  )) 1
lim ​ ​ x2sin ​ ​ __
112. Evaluate: ​  
x Æ 0 x ​  ​  ​
92. Evaluate: ​  
x Æ e
ln x – 1
lim ​ ​ ​ _______
x – e ​  
 ​ (  ) lim ​ ​( ​ ____
113. Evaluate: ​  
sin x
x Æ • x    ​  )​

93. Evaluate: ​  


x Æ 0
x + tan 2x
lim ​ ​ ​ _________ 
x – tan 2x ( 
​   ​ ) lim ​ ​( ___
114. Evaluate: ​   ​  x   ​ )​
[x]
x Æ •

94. Evaluate: ​  


lim 
x Æ e
sin x – x
​ ​ ​ _______
x3
 ​  
 ​
(  ) lim ​ ​( ___
115. Evaluate: ​   ​    ​  )​
[x]
x Æ •
x

x Æ 0 ( 
ex + e– x – x2 – 2
lim ​ ​ ​ _______________
95. Evaluate: ​     
  
sin2 x – x2
 ​  ​ ) 116. Evaluate: ​  
n Æ • ( 
[x] + [2x] + [3x] + ... + [nx]
lim  ​ ​ ​ ________________________
   
n2
 ​     ​ )
(  )
x
x e – log(1 + x) Newton and Leibnitz Rule
lim ​ ​ ​ ______________
96. Evaluate: ​    ​ 
    

(  )
x Æ 0 x2 x

x Æ •
ln x
lim ​ ​ ​ ___
97. Evaluate: ​   x    (  )
​   ​ x – ​Ú ​  ​ cos(t 2)dt
lim ​ ​ ____________
117. Evaluate: ​   ​ 
0
       ​ ​
x
(  ) x3 – 6x
x Æ 0
lim ​ ​ __
98. Evaluate: ​   ​     ​  ​

(  )
x Æ • ex
x _____

x Æ •
x
lim ​ ​ ____
99. Evaluate: ​   ​  x   
e

​  ​ (  ) 2014

118. Evaluate: ​  


x Æ •
​Ú ​  ​ ​÷4  + t4 
__________
0
lim ​ ​ ​ 
x3
 ​dt
 ​ 
     ​

(  )
(  )
x2 + 2 cos x – 2
lim ​ ​ ​ _____________
100. Evaluate: ​     
    ​  ​ x2
Ú​  ​  ​ ​ ​e– t 
2
x Æ 0 x sin3 x ​ ​dt
119. Evaluate: ​   _______
x
lim ​ ​ ​   ​ 
 ​

101. Evaluate: ​  
lim 
p
​ (tan x loge sin x) x Æ 1 x –  1
x Æ ​ __ ​ 
2
Definite Integral as the limit of a sum
Advanced Exponential Limit

(  )
x + 6 x + 4
lim ​ ​​ ​ _____ 
102. Evaluate: ​  
x Æ • x+1
 ​  ​​ ​ 120. Evaluate: ​  
1
lim ​ ​ _____
​     
n Æ • n + 1 [  1
 ​ + _____
​     
n+2 n+3
1
 ​ + _____
​     
1
 ​ + ... + ___
​     ​  ​
2n ]
lim ​ ​​( ​ ______ 
103. Evaluate: ​  
x Æ •2x + 3)
2x + 4
 ​  ​​
2x + 10
​ 121. Evaluate: ​  
n Æ •
1
lim ​ ​ ______
[ 
​  2   2 ​ 
n +1 n +2 n +3 2
1
+ ​ ______
  2 ​  2
1
+ ​ ______
  2 ​  
1
+  ...  + ___
]
​     ​  ​
2n

(  (  ) (  )) ( 
1 1 x
lim ​ ​​ sin ​ __
​ x ​  ​ + cos ​ __
)
2n
r
104. Evaluate: ​  
x Æ •
​ x ​  ​  ​​ ​ lim ​ ​S   ​ ​ ​ ________
122. Evaluate: ​   ​  _______
   ​  

n Æ • r = 1
÷​ n  + r 2 ​ 
2

(  )
x x x
a +b +c
lim ​ ​ ​ __________
105. Evaluate: ​    ​ 
    ​
(  )
2n
2 2r
x Æ 0 3 lim ​ ​S   ​ ​ __
123. Evaluate: ​   ​​ n ​ ​ __
​ n ​  + 1  ​
n Æ • r = 1

106. Evaluate: ​  


1
lim ​ ​​ 2 – __ ( 
​ x ​  ​​
(  )​
px
tan ​  ​ ___ ​   ​
) 2
x Æ 1
(  )
(2n)! 1/n
lim ​ ​​ ​ _____ 
124. Evaluate: ​    
 ​  ​​ ​
(  (  ))
1
 ​ ___     n Æ • n!×nn
p ​
lim ​ ​​ tan ​ __
107. Evaluate: ​   ​   ​  + ln x  ​  ​​ln x ​
x Æ 1 4
Advanced limit in the form of (00 and •0)

(  (  (  ) ) )
p
ln ​ tan ​ __ ​   ​  + 2x  ​  ​ 2
​​​( x – x2 )​​ ​
4 ​x​ ​
lim ​ ​ ​ _______________
108. Evaluate: ​       
  ​  ​ 125. Evaluate: ​ lim 
  + 
x Æ 0
x Æ 0 sin3 x
1
________
​       ​
109. Evaluate: ​  
x Æ 0
tan x 1/x
lim ​ ​​ ​ ____ (  )
​  ​​ ​
x   
126. Evaluate: ​  
lim 

​ ​​( 1 – x2 )​​log(1 – x) ​
x Æ 1

( 
1
__

)
​   ​
1 + tan x cosec x lim ​ ​​( 3x + 4x )​​x ​
lim ​ ​​ ​ ________ ​  
110. Evaluate: ​   ​​ ​ 127. Evaluate: ​  
x Æ •
x Æ 0 1 + sin x
The Limit  3.9

(  __1 )
lim ​ ​ ​x​ ​x ​​  ​
128. Evaluate: ​  
x Æ •
(a) a – b (b) b – a
(c) 1 (d) None

( 
x + 1 x  + 2014
) (  )
2
lim ​ ​ ​ ______ 
129. Evaluate: ​    
​ ​  ​​ ​ sin (p cos2 x)
x Æ • 2x + 1   ​ ​ ​ __________
7. ​ lim   ​ 
     ​=
x Æ 0 x2
ln 2
______
​     
lim ​ ​x​
130. Evaluate: ​   1 + ln x ​ ​ (a) – p (b) p
x Æ •
(c) p /2 (d) None
lim ​ (2x + 3x)1/x
131. Evaluate: ​  
x Æ •
x x x x 1/x
8. ​ lim 
x Æ • 4x
p
   ​ x cos ​ ___ (  ) (  ) 4x
p
​    ​  ​ sin ​ ___
​   ​   ​ =
132. Evaluate: ​  
lim ​ (1 + 2 + 3 + ... + 99 )
x Æ • (a) p /4 (b) p /3
lim ​ ​(1  cose​c​ ​x + 2cose​c​ ​x + 3cose​c​ ​x + ... + ncose​c​ ​x )si​ (c) p (d) 0
2 2 2 2 2
133. Evaluate: ​   ​ n​ ​x
(  )
x Æ 0 b
   ​ ax sin ​ __
9. ​ lim  ​  x   ​  ​, where a > 1 is
x Æ • a
(a) b log a (b) a log b
(Mixed Problems)
(c) b (d) a
Choose the most appropriate one.
(a + x)2 sin (a + x) – a2 sin a

÷ 
________   ​ ​ ________________________
10. ​ lim     
x    ​ is
x – sin x
1. If f(x) = ​ ​ ________   
 ​ ​, then ​  
  lim ​ ( f (x)) is x Æ 0
x + cos2 x x Æ 0
(a) a2cos a + a sin a (b) a2cos a + 2a sin a
(a) 0 (b) • (c) 2a2cos a + a sin a (d) None
(c) 1 (d) None
sin nx [(a – n) nx – tan x]
2. If f (x) = x sin (1/x), x π 0, then ​  
lim  
​ ( f (x))   ​ ​ ____________________
11. ​ lim      ​   , where n is a non-zero
x Æ 0 x Æ 0 x2
(a) 1 (b) 0 positive integer, then a is
(c) – 1 (d) None
________ ( 
n+1
(a) ​ ​ _____
n  ​  
 ​ ) (b) n2
3. ​ lim 
x Æ 0
÷
( 
​ 1  – cos2x  
  ​ ​ ​ _________

x    ​  ​= ) (c) 1/n (d) n + 1/n

(  (  ) (  ) (  ) (  ) )
(a) 1 (b) 0 x x x x
lim  ​​cos ​ __
12. The value of ​   ​    ​  ​ ◊ cos ​ __
​    ​  ​ ◊ cos ​ __
​    ​  ​... cos ​ __
​  n   ​  ​  ​
(c) – 1 (d) Non-existent x Æ • 2 4 8 2
4. Let a and b be the roots of ax2 + bx + c), then is
(a) 1 (b) sin x/x
1  – cos(ax2 + bx + c)
lim ​ ​ ___________________
​        ​
   is (c) x/sin x (d) None
x Æ 0 (x – a)2
1 __________
______
__ __
(a) 0 (b) ​ __ ​  (a – b)2    ​ ​[ ​÷x  + ÷
13. ​ lim  ​ x  +  
​ x 
÷    ​ ​ ​
    ​  ]​ is equal to
– ​÷x 
2 x Æ •
a2 a2
(c) ​ __ ​  (a – b)2 (d) – ​ __ ​  (a – b)2. (a) 0 (b) 1/2
2 2
(c) log 2 (d) None
sin2a  – sin2b
   ​____________
5. ​ lim  ​    
   ​ is
a Æ b a 2 – b 2 x4sin(1/x) + x2
   ​ ​ ____________
14. ​ lim     ​ =
(a) 0 (b) 1 x Æ – • 1 + |x|3
(a) 1 (b) 2
sin b sin2 b
(c) ​ ____
   ​   (d) ​ _____
   ​.
  (c) – 1 (d) None
b 2b
x (1 + a cos x) –  b sin x
6. If a and b be the roots of x2 – ax + b = 0, then 15. If ​   ​ ​ ____________________
lim       ​    = 1, then a, b are
2 
x Æ 0 x3
​ex​ – ax + b​ – 1
​   ​ __________
lim   ​   ​
    =
x Æ a x–a (a) 1/2, –3/2 (b) 5/2, 3/2
(c) –5/2, –3/2 (d) None
3.10  Differential Calculus Booster
cos(sin x) –  cos x (a) (2, 1) (b) (1, 2)
lim ​ ​ ______________
16. The value of ​    ​     is equal to
x Æ 0 x4 (c) (1, R) (d) (1, 1)



(a) 1/5
(c) 1/4
(b) 1/6
(d) 1/2
26. If a, b, c, d are positive, then ​  
x Æ •
1
lim  ​ ​​ 1 + ______
​     
a + bx ( 
c + dx
 ​  ​​ ​ )
[ 
d/b c/a
(a) e (b) e
17. If ​  
x Æ 1
l2
lim ​ sec  ​ ____ –1 l2
​    ​ – _____
​    
log x x – 1 ]
 ​  ​ exists, then l lies in (c) e (c + d)/(a + b) (d) e

lim ​ (cos x + a sin bx)1/ = e2, then (a, b) is equal to


27. If ​  
__ __
   ​​ ]​
(a) ​​ ​( – •, ​÷2 ​    ,  • ​​ )​
(b) ​​ ​[ ​÷2 ​ x Æ 0

__ __ (a) (1, 2) (b) (2, 1/2)


   ]​​  ​» ​​ ​[​÷
(c) ​​ ​( – •, – ​÷2 ​    ,  • ​​ )​ (d) None
  2 ​
( 

)
__ 1
  ,  ___
(c) ​ 2​÷3 ​ ​  __  ​  ​ (d) (4, 2)
[  (  ) ]
x m ÷   
​ 3 ​
​ lim  ​ ​​ cos ​ __
18. The value of   ​ m  ​   ​  ​​ ​ is equal to

(  )
m Æ •
x + 6 x + 4
   ​ ​​ ​ _____ 
28. ​ lim   ​  ​​ ​ =
(a) 1 (b) e x Æ • x + 1
(c) 1/e (d) None
(a) e (b) e3

[  { 
p–4
   ​ ​​ tan ​​ ​ _____
19. ​ lim 
n Æ • 4
1 a n
 + ​1 + __
 ​  (  )} ]
​ n ​  ​  ​​ ​  ​​ ​ is equal to (c) e5

   ​ (4n + 5n)1/n is equal to


29. ​ lim 
(d) e7

a 2a x Æ •
(a) e (b) e
(c) 1 (d) None (a) 4 (b) 5

[  ]
a a (c) e (d) None
1
  ​ __​   ​ ​ ​Ú ​  ​ esi​n​ ​t dt – ​Ú   ​ ​  esi​n​ ​t dt  ​ is equal to
2 2
20. ​ lim  30. If a and b are the roots of the equation ax2 + bx + c = 0,
x Æ 0 x y x + y 2
​ ______
   ​ 
2 2 lim ​ ​​( ax2 + bx + c + 1 )​​(x – a) ​ is
then ​  
(a) esi​n​ ​y (b) sin2ye si​n​ ​y x Æ 0

(c) 0 (d) None (a) 2a(a – b) (b) 2 log |a(a – b)|

(  )
x x
1 – 3 – 4 + 12x
  ​ ​​  ________________
21. ​ lim  __________     ​  ​ is
   (c) e2a(a – b) (d) None
x Æ 0 (​  ​ (2 cos x 7) ​ – 3 )​
÷  +  
x +y n n
   ​_______
31. ​ lim  ​    ​, where x > y > 1 is equal to
(a) 2 log4 ◊ log3 (b) –  6 log4 ◊ log3 n Æ • xn – yn

(c) 3 log4 ◊ log3 (d) None (a) 0 (b) 1


x x x x x (c) – 1 (d) None
64 –  32 –  16 +  4 + 2 – 1
  ​ ​ _________________________
22. ​ lim     
   
_____________   ​=

( 
(​  ​÷(15 –  4 ​ )​ sin x
  + cos x)   
)
x Æ 0 x
x2 +  5x + 3
32. If f(x) = ​​ ​ __________
   ​  
​​ ​, then ​  
lim  ​ f (x) is
(a) – 96 (log 2)3 (b) 48(log 2)2 x2 + x + 2 x Æ •

(c) log 2 (d) None (a) e4 (b) e3


8 x – 4x – 2 x + 1 (c) e2 (d) 24
  ​ _______________
23. ​ lim  ​  __   
   ​ =

( 
_______

)
x Æ 0 (​  ÷ ​ )​
​ 1  + cos x  
   – ÷
​ 2 ​ x
x2 – 2x + 1
__ __
lim  ​ ​​ ​ __________
33. The value of ​     
    ​  ​​ ​ is equal to
x Æ • x 2 – 4x + 1
(a) ​( 4​÷2 ​ 
   )​ log 2 (b) ​( 8​÷2 ​ 
   )​ log 2
__
(a) e2 (b) e–2
(c) ​( 8​÷2 ​ 
   )​ (log 2)2 (d) None
(c) e6 (d) None

x Æ 0 {  ( 
p
  ​ ​​ tan ​ __
24. ​ lim 
4
1/x
​   ​  + x ​  ​​ ​ = )} 34. ​ lim  ( 
1 + tan x cosec x
  ​ ​​ ​ ________ 
 ​  ​​ )
​ is equal to
2 x Æ 0 1 + sin x
(a) e (b) e
__ 1__ (a) e (b) 1/e
(c) ​÷e 
  ​   (d) ​ ___  ​  (c) 1 (d) None
​ e 
÷    ​

[  (  )
sin x

]
_______
a b 2x sin x ​ x – sin x   

lim  ​ ​​ 1 + __
25. If ​   ​ x ​ + __
​  2  ​   ​​ ​ = e2, then (a, b) is   ​ ​​ ​ ____
35. ​ lim  x ​  ​ ​
   ​
x Æ • x Æ 0
x
The Limit  3.11

(a) 1/e (b) e (a) 3/5 (b) – 3/5


(c) 1 (d) None (c) 5/3 (d) – 5/3

36. The integer n for which ​  


x Æ 0
(cos x – 1) (cos x – ex)
lim ​ ​ __________________
xn
       ​ is
x Æ •
(ln x)10
lim  ​ ​ ______
45. The value of ​   ​  4 ​  
x (  )
 ​ is

a finite non-zero number is (a) 1 (b) 0


(a) 1 (b) 2 (c) 1/2 (d) – 1/2
___________

(  ÷ 
(c) 3 (d) 4
37. Let f : R Æ R be such that f (1) = 3 and f ¢(1) = 6, lim  ​ ​ x + ​ x2 + x2sin ​
46. The value of ​  
x Æ •
  
1
__
​ x ​  ​ ​  ​ is (  ) )
x Æ 0 (  f(1) )
f (1 + x) 1/x
lim ​ ​​ ​ _______
then ​    ​  

​​ ​ is


(a) 0
(c) – 2
(b) 2
(d) None

(  ÷ 
_____________


(a) 1
(c) e2
(b) e1/2
(d) None
47. The value of ​   lim  ​ ​ x + ​ x2 + 3x sin ​  
x Æ •
__1
(  ) )
​    ​   ​ ​  ​ is
|x|
(a) 3/2 (b) – 3/2
38. For x > 0 ​   (  1 sin x
lim ​ ​ (sin x)1/x + ​​ __ (  ) )
​ x ​  ​​ ​  ​ is equal to (c) – 1 (d) 0

(  )
x Æ 0
2n n
x  sin  x
(a) – 1 (b) 0 48. If ​   ​ ​ ​ _________ 
lim    
 ​  ​ is a non-zero finite number, then
x Æ 0 x 2n – sin n x
(c) 2 (d) 1
the value of must be
39. Suppose f : R Æ R is a differentiable function and
(a) 1 (b) 2
f(x)
(c) 3 (d) 4
Ú​   ​  ​ ​2t dt
( 
__ __
lim ​ ​ ______ ​ 
)
4
f(1) = 4. Then the value of ​   is ​÷x p  x
  ​  + ​÷   ​ 
q 
x Æ 1 x – 1 lim  ​ ​ ​ _______
49. The value of ​    ​    ​, where p > 0, q > 0 is
x Æ – • 2
(a) 8 f ¢(1) (b) 4 f ¢(1) equal to
___
(c) 2 f ¢(1) (d) f ¢(1) (a) 1 (b) ​÷pq    ​
(c) pq (d) pq / 2
40. If f (x) is differentiable increasing function, then
x2
f(x2)  – f(x)
lim ​ __________
​   ​     ​  
is equal to Ú​  ​  ​ ​(sin x2)dx
x Æ 0 f(x) – f(0)
lim  ​ ​ _________
0
50. If ​        
​ is a non-zero finite number, then
(a) 2 (b) 1 x Æ • xn
(c) –1 (d) 0 the value of n is

[ 
(a) 1 (b) 3
41. The value of ​  
is
x2
lim ​ ​ ​ ________
x Æ 0 sin x tan x
    ]
 ​  ​, where [, ] = G.I.F.,

(c) 5 (d) 4
51. If a, b, c are real numbers, then the value of

(  )
t
1 __e
(a) 0 (b) 1 ​   ​   ​ ​ Ú ​  ​  (1 + a sin bx​)​ ​x ​​  dx  ​ is
lim ​ ln ​ __
x Æ 0 t 0
(c) – 1 (d) does not exist
2
__ (a) abc (b) ab/c
lim ​ {1 + x​}​ ​x ​​, where {,} = F.P.F, is
42. The value of ​   (c) bc/a (d) ac/b
x Æ 0

(  )
2
(a) (e – 7) (b) (e2 – 8) sin x – (sin x) sin x
52. The value of ​  
lim   
​ ​
​  _______________
    ​  ​ is
  
(c) (e2 – 6) (d) (e2 – 10) p
x Æ ​ __ ​  1 – sin x (log sin x)
2

x Æ 0 ( 
2(tan x  – sin x) – x3
lim ​ ​ ​ ________________
43. The value of ​  
x5
 ​      ​ is )

(a) 1
(c) 2
(b) 0
(d) 2/3



(a) 1/4
(c) 1/3
(b) 1/2
(d) None
53. The value of ​  
x Æ • na
1
lim  ​ ​ ___ ( 
1
​     ​ + ______
​     
1
 ​ + ______
​     
na + 1 na + 2
 ​ + ...
nb
1
)
​ ___   ​  ​ is

(  ( 
2x2  + 1
lim  ​ x2 × ​ tan–1 ​ ​ _______
44. The value of ​  
x Æ • 2
x +2 )
​ – tan–12  ​ is
 ​ 
  )
a
(  )
(a) log ​ __
​   ​  ​
b
b
(b) log  ​ __ (  )
​ a ​  ​
3.12  Differential Calculus Booster

(c) log (ab) (d) None lim ​ ​ ________


64. The value of ​   ​  ( 
ex – esin x
​   
​ is )
( [  ] [  ])
x Æ 0 x + sin x
2000x 13 sin x
54. The value of ​  lim ​ ​ ​ ​ ______    ​  ​ + ​ ______
​  x     
​  ​  ​ is, where
x Æ 0 sin x
(a) 0 (b) 1
[,] = G.I.F
(c) – 1 (d) 2
(a) 2013 (b) 2012
(c) 2011
__
(d) None
( 
ex – ex cos x
lim ​ ​ _________
65. The value of ​   ​     
​ ​ is )
(  ) x + sin x
n
​ n! ​  x Æ 0
÷ 
lim  ​ ​ ​ ___
55. The value of ​   n   
​   ​ is
n Æ • (a) 0 (b) 1
(a) e (b) 1/e (c) 2 (d) – 1
(c) 1/2e (d) 1/3e
(  ( 
1
____

(  ))
​     ​ 
p
56. The value of ​  
x Æ 0
1 +  sin x – cos x + log(1 – x)
lim ​ ​ ​ ________________________
   
x tan2 x
    ​  ​ is ) (a) e
lim ​ ​​ tan ​ __
66. The value of ​  
x Æ 1
​   ​  + log x ​  ​​log x ​ is
4
(b) 1/e
(a) – 1/2 (b) 1/2 (c) e2 (d) 1/2e
(c) – 1/3 (d) 1/4

lim  ​ ​ __
57. The value of ​  
x Æ • x 2
1
(  1
​    ​  – _____
​     
 ​  ​ is
)
67. The value of ​  
n Æ • ( (  )
n+1
n
lim  ​ ​​ ​ _____
​     
1 n
 ​  ​ + sin ​ __ (  ) )
​ n ​  ​  ​​ ​ is
tan2 x
(a) ea – 1 (b) ea + 1
(a) 2/3 (b) 3/2
(c) e –a + 1 (d) e– a – 1
(c) 2/5 (d) 5/2

(  )
1
__
1
___ e – (1 + x​)​ ​x ​​
58. The value of ​  
​      

lim ​ (cosec x​)​ln x ​ is lim ​ ​ ​ __________
68. The value of ​         ​  ​ is
x Æ 0 x Æ 0 tan x
(a) 1/e (b) e (a) e/2 (b) 2/e
(c) 2/e (d) e/3 (c) 3/e (d) e/3
n __ n
59. The value of ​  
1
lim ​ ​ ____
​      
x Æ 0 sin x
​– (  1
__
)
​ x ​  ​ is 69. The value of ​  
__
lim  
n Æ •
a–1+÷
​  (    ​
​ b 
​ ​​__________
a     ​   ​​ ​, n Œ N is )
a __
(a) 1 (b) – 1   ​  
(a) ​÷b  (b) ​÷b a 
  ​ 
__ __
(c) 0 (d) 2   ​  
(c) ​÷b  (d) ​÷ a ​. 

p
lim ​ ​​ tan ​ __
60. The value of ​  
x Æ 1 (  ( 
​      ​
))
​   ​  + ln x  ​  ​​ln x ​ is
4
1
___
70. The value of ​  
lim  
x Æ 0
n sin x
​ ​ ​ _____
x     [  ] [ 
n tan x
​  ​ + ​ ​ ______
x    
​  ​, where [,] = ]
G.I.F, is
(a) e4 (b) e2
(a) 2n (b) 2n + 1
(c) e3 (d) e5
(c) 2n – 1 (d) does not exist
1
________
​       ​
61. The value of ​  
lim 
p
​ (1 + [x]​)​log(tan x) ​, where [, ] = G.I.F, nksin2 (n!)
x Æ ​ __ ​ 
4 71. The value of ​   ​ ​ ________
lim    ​ is, where 0 £ k < 1, is


is equal to n Æ • n+1
(a) 0 (b) 1 (a) 0 (b) 1
(c) e (d) 1/e (c) – 1 (d) – 2

x Æ •
logxn – [x]
lim  ​ ​ ​ _________
62. The value of ​  
[x]
 ​  ( 
​, n Œ N, is
  ) ( 
(1 +  x)1/x + ex – e
lim ​ ​ ________________
72. The value of ​  
x Æ 0
​ 
sin–1x
  ​  ​ is
     )
(a) – 1 (b) 0
(c) 1 (d) 2 (a) – e/2 (b) e/2

( 
(c) 1 (d) 0
63. The value of ​  
x Æ 0
x cos x – log(1 + x)
lim ​ ​ ​ ________________
x2
 ​ 
     ​ is equal to )   +  ​ (– ln({x} + [x])){x} is
73. The value of ​ lim 
x Æ 0

(a) 1/2 (b) 0 (a) 0 (b) 1


(c) 1 (d) – 1 (c) ln 2 (d) ln (1/2)
The Limit  3.13

74. The value of ​  


x Æ •
xn  + nxn – 1 + 1
lim ​ ​ ​ _____________
[x]
 ​     ( 
​, n Œ I is ) 17. Evaluate: ​    ​ ​[ sin –1 (sin x) ]​
lim  
p
(  )
x Æ ​ __
​   ​   ​
2



(a) 1
(c) n
(b) 0
(d) n (n – 1)
18. Evaluate: ​  
x Æ 0 ( [  ] [  ] )
100x
lim ​ ​ ​ _____
​ 
sin x
99sin x
 ​  ​ + ​ ​ ______
x   ​  ​  ​

( 
n __ n

75. The value of ​ lim 


   + 
ln (x – a)
​ ​​ _________
x a 
x Æ a  ln (e   –  e )

 ​  ​ is ) 19. Evaluate: ​    ​ a  – 1 a +  ​ b ​ ​ )​​ ​, where a, b > 0
lim ​ ​​(__________
n Æ •
÷   

(  )
ex
(a) 1 (b) – 1 (1  – x)1/x – e + ​ __ ​ 
2
(c) 0 (d) None lim ​ ​ ​  _______________
20. Evaluate: ​   2
 ​ 
     ​.
x Æ 0 x

  (Problems for JEE-Advanced) x Æ 0


sin{x}
lim ​ ​ ______
21. Evaluate: ​   ​ 
{x}
 ​   (  )
 ​, where {,} = F.I.F


x Æ 0
sin x
lim ​ ​ ​ ____
1. Evaluate: ​   [  ]
​  ​
x    n Æ •
p–4
lim ​ ​​ tan ​​ ​ _____
22. Evaluate: ​  
4 [  {  1 a n
 + ​ 1 + __
 ​  (  )} ]
​ n ​ ​  ​​ ​  ​​ ​

lim ​ ​{c  os ​( __ ​    ​ )​  ...  cos ​( __


​    ​ )​ cos ​( __
​    ​ )​ cos ​( __ ​     ​ )​ }​
[  ]
tan x x x x x
lim ​ ​ ​ ____
2. Evaluate: ​   ​  ​ 23. Evaluate: ​  
x Æ 0 x    n Æ • 2 4 8 2 n


x Æ 0
[  ]
sin–1x
lim ​ ​ ​ _____
3. Evaluate: ​   x   ​  ​ 24. Evaluate: ​ lim 
  +​ 
x Æ p ( 
2cot x + 3cot x – 51 + cot x + 10
​ ​ ____________________________
    
   
(4 )1/2  + (27cot x)1/3 – 5cot x + 20
cot x
 ​  ​
)
lim ​ ​[ ​ _____​  ]​
–1
tan x 25. Find a polynomial of the least degree such that
4. Evaluate: ​   x   

(  )
x Æ 0
x2   + f (x) 1/x
5. Evaluate: ​ lim 
    ​ [sin x + cos x] ​ lim    ​ ​​ 1  + ​ ________
 ​   ​​ ​ = e2.t
5p
x Æ ​ ___ ​ 
x Æ 0 x2
4 26. Evaluate:
6. Evaluate: ​  
x2
x Æ 0 sin x tan x[ 
lim ​ ​ ​ ________
    ​  ​ ] ​​{(​​   n6 + 6n5 + 12n4 + 1 )1/3
​ lim 
   
n Æ •
​  }​
​​ ​– (​  n4 + 4n2 + 6n + 1 )1/2

( [  ] [  ] )
n sin x n tan x 27. Evaluate:
lim ​ ​ ​ ​ _____
7. Evaluate: ​   x   ​  ​ + ​ ​ ______
x   ​  ​  ​

x Æ 0
( 
(1 + 3x + 2x2)1/x  – (1 + 3x – 2x2)1/x
  ​ ​ ​  ______________________________
​ lim       x    ​  ​ )
lim ​ ​( ​[ 2016 ​ ____ ​  ​ + [​  ​ ____ ​  ]​ )​
sin x ]
x tan x x Æ 0
8. Evaluate: ​        x   
(  )
x Æ 0 1 – cos(1 – cos x)
lim ​ ​​ ______________
28. Evaluate: ​    ​ 
     ​
( [  ] [  ] ) x4
x Æ 0
sin–1x x
lim ​ ​ ​ 2016 ​ _____
9. Evaluate: ​   x   ​  ​ + ​ ____
​      
​  ​  ​
(  )
tan x sin x
_______
x Æ 0 sin x ​ x – sin x   

lim ​ ​​ ​ ____
29. Evaluate: ​   ​  ​​
x    ​
lim ​ ​( ​[ 2016 ​ _____ ​  ]​ + [​  ​ ____ ​  ]​ )​
–1 x Æ 0
tan x tan x
10. Evaluate: ​  
x Æ 0 x    x    30. Let f (x) be a real function such that ​  
f (x)
lim ​ ​ ____
​  x   
​  ​ = 1 (  )
( 
x Æ 0

)
( 
sin[cos x]
lim ​ ​ _________
)
11. Evaluate: ​   ​    
 ​  ​ (1  + a cos x) – b sin x
x Æ 0 1 + [cos x]
lim ​ ​ ​ ___________________
If ​        ​ 
   ​ = 1, then find the
x Æ 0 {f (x)}3
log x
lim ​ ____
12. Evaluate: ​   ​   ​  
x Æ • [x] values of (a + b + 10)

x Æ • [x] ( 
log(xn) – [x]
lim ​ ​ ​ __________
13. Evaluate: ​    ​ 
    ​, n Œ N ) (Tougher Problems for JEE-Advanced)

(  ) (  ( 
m x m n – x
lim ​ nCx ​​ __
14. Evaluate: ​  
x Æ •
​ n ​   ​​ ​ ​​ 1 – __
​ n ​   ​​ ​ )
xx – aa
lim ​ ​ _______
1. Evaluate: ​   ​  x
x Æ a a – aa
​   ​
  )
n sin(n!)
(  k
)
( 
​ ​ ​ ________ ​, (0 £ k £ 1) _____

)
15. Evaluate: ​  
lim    ​  

n Æ • n+1 x +  ln (​÷x  2 + 1 ​
 – x)
lim ​ ​ _________________
2. Evaluate: ​   ​   ​ 
      ​

(  )
x Æ 0 3
sin [x] x
lim ​ ​ _____
16. Evaluate: ​   ​   ​  
 ​
x Æ 0 [x]
3.14  Differential Calculus Booster

(  (  (  ))
2
__ n

)
​  3  ​  n Ck
2 ​ ​ ​S   ​ ​ ​ ​ ________
lim ​ ​​ __
3. If ​   ​  3  ​ ​ ( sin– 1 x – tan– 1 x )​  ​​x ​ = e– M, find the value 15. If ​  
lim  
n Æ • k = 0 k
   ​  ​  ​ = Le + M, find the value
x Æ 0 x n  (k + 3)
of 2M + 2013. of L + M + 10.

(  )
n + ​xn​ ​
1
4. For n Œ N, let xn be defined as ​​ 1 + __
​ n ​  ​​ ​ = e, 16. If ​  
lim  
1
​ f (x) = ___
​    ​ such that
n Æ • 60
find ​  
lim  
​ (xn)
n Æ •
(1a + 2a +  3a + .... + na) ÷ (n + 1) a–1
5. If a and b are the roots of the equation ax + bx + c = 0, 2 f (x) = ​ ________________________________
    
      ​
((na + 1)  + (na + 2) + ..... + (na + n))
find the value of
where n π – 1 and a π 0, find the value of a.

(  )
__________________

( 
÷ 
)
2
​ ___________________
1 –  cos (ax   
+ bx + x) ​
  ​ ​ ​ 
​ lim 
x Æ a
   
(x + a)2
 ​ 
   ​
17. If L = ​    ________
lim ​ ​ ​ 
1
x4 sin ​ __
​ x ​  ​
 ​ 

  ​
(  )
x Æ – •
1 + |x|3
3 3 3

(  )
6. Find the value of a + b + c + 10

(  )
x2
t2
(  ) Ú​__ ​  ​ tan– 1 ​ ​ _____
x – x
a e –  b cos x + ce     ​  ​ dt
  ​  ​ ​ ________________
​ lim    ​  ​ = 2
     ​÷________________
  ​ 
x  1 + t2
x Æ 0 x sin x and M = ​  
lim ​ ​ ​         ​  ​, find the value
sin 2x

( 
x Æ 0

7. If ​  
1 –  cos x + 2sin x – sin3 x – x2 + 3x4
lim ​ ​​ _______________________________
          ​  ​ = M, ) of L + M + 11.

( 
x Æ 0 tan3 x – 6sin2 x + x – 5x3

) [  ]
2n
n3 x 1
find the value of M + 10. lim ​ ​ ​Ú ​   ​ ​ ​ _____
18. If ​   ​ dx = k, find the value of ​ __
  ​   ​   ​  ​+ 2013.,
n Æ • n 5
x +1 k
8. If L = ​  
x2
x Æ 0 sin x tan x[ 
lim ​ ​ ​ ________
     ]
​  ​, find the value of L + 2013. where [ ] = G.I.F
______________ ______________
9. Let P(x) = a1x + a2 x + ....... + a100x 2 100
, where a1 = 1 19. Evaluate: ​   ​÷
lim ​ ​ (x
x Æ •
( 
  2 + a2) (x2 +  
b2) ​– ÷   2 + c2) (x2 +  
​ (x d2) ​  ​ )

{  }
and a1, a2 ... a100 ΠR such that
(1 –  x) (1 – x2) ... (1 – x2n)
lim ​ ​ _________________________
*

20. Evaluate: ​   ​     


    ​  ​
(  )
100
________
÷​ 1  + P (x) ​  x Æ 1 {(1 –  x) (1 – x 2) .... (1 – xn)}2
lim ​ ​ ​ __________
L = ​   x     ​  – 1  ​

( 
x Æ 0

then find the value of (1000 L + 2007) 21. Evaluate: ​  


4x  + 4 – x
lim ​ ​ ________
​ 
x Æ • 4x – 4 – x
  ​  ​ )
100x
lim ​ ​ _____
10. Let L = ​   ​ 
x Æ 0 sin x [  ]
 ​  ​ and M = ​  
x Æ 0
99 sin x
lim ​ ​ ​ ______
x   ​  ​, find the [  ] 22. Evaluate: ​  
n
r3 – 8
​ ​P   ​ ​ ​ ​ ​ _____
lim  
n Æ • r = 3
r3 + 8
  (  )
 ​  ​
value of L + M + 2.
23. Find the value of

11. If ​  
x Æ • ( 729x  – 243x – 81x + 9x + 3x – 1
lim ​ ​__________________________
​     
x3
 ​     ​= K (log M) N, ) ​ lim 
x Æ 0 x
32
  ​ ​  ___ {  (  (  ) (  ) (  ) (  ))}
x2
2
x2
4
x2
2
x2
​  8 ​ ​ 1 – cos ​ ​ __ ​   ​ – cos ​ ​ __ ​   ​ + cos ​ ​ __ ​   ​ cos ​ ​ __ ​  ​ ​  ​
4
find the value of K 2 + M 3 + N 2.
24. Find the value of

12. If ​  
lim 
x Æ 1– ( 
(3 +  ax)5/2 –  b ln x + c sin (x – 1)
​ ​ ​ ___________________________
    
(x – 1)2
2
 ​ 

2
   ​ = 2, find

2
)   ​ ​[ {(x + p) (x + q) (x + r) (x + s)}1/4  –  x ]​
​ lim 
x Æ •

25. Let f (x) be a function such that ​  


f (x)
lim ​ ​ ____
​  x   
​  ​ = 1 (  )
the value of a + b + c . x Æ 0

13. Find the smallest integral value of n for which If ​   ​ 


( 
x (1  +  a cos x) – b sin x
lim ​ ​____________________
     ​ 
   ​ = 1, find the value
)
( 
x Æ 0 {f (x)}3
​  
lim  
x Æ 0
1 – cos x) (ex – cos x)
​  ​​  ___________________
   
xn
    ​  ​ is a non-zero finite ) of (a + b + 10).
number.
( 
ax ex –  b log (1 + x) + cx e– x
lim ​ ​ ​ ________________________
26. If ​           ​  ​ = 2, find the )
(  ) x2 sin x
x Æ 0
cos 4x + a cos 2x + b
lim ​ ​ ​  _________________
14. If ​    ​ 
     ​ is a finite quantity,
x Æ 0 x4
find the value of a2 + b2 + 10.
8( 
a  + b + c
value of ​ ​  ________  ​   ​ )
The Limit  3.15

( 
sin x  – (sin x) sin x
lim ​ ​ _________________
27. Evaluate: ​   ​        ​  ​
x Æ 0 1 – sin x + log (sin x) )
( 
lim ​ ​
6. If ​  
x Æ 1
tan (x – 1) loge (x x– 1)
 __________________
​ 
(x – 1)3
 ​ 
  
)
   ​ exists and equal to L,

28. Let f (x) = cos 2x cos 4x cos 8x cos 10x and find the value of (L + 3).

M = ​  
x Æ 0
1 – (f (x))3
lim ​ ​ ​  _________
5 tan2 x
_____
( 
  
  )
​  ​, where M is finite, find the
x Æ • ( 2 (tan x  –  sin x) – x3
lim ​ ​ _________________
7. If ​   ​ 
x 5
 ​ 
   ) m
   ​ = ​ __
n ​ , find the value of
(m + n + 2).
value of (​÷M   – 2 ​  + 1)

29. If ​  
x Æ 0
( [ 
sin– 1 x
lim ​ ​ ​ ​ ______
x     
​  ​ 


+ ] [ 

22 sin– 1 (2x)
__________
​  x       
​  ] [ 
​ + ​
32 sin– 1 (3x)
__________
​  x     ​  ​

] x Æ 0 (  1  + sin x – cos x – + log (1 – x)
lim ​ ​  ​ __________________________
8. If ​      
x tan2 x
  )
   ​  ​ exists and

[  ])
m
n2 sin– 1 (nx) is equal to – ​ __
n ​ , find the value of (m + n).
+ ... +  ​​ ​ __________
x     ​  
​  ​ = 100, find the value of n,

where [,] = G.I.F. 9. If ​  
x Æ 0 (  ex + e– x + 2 cos x – 4
lim ​ ​ ​  __________________
x4
 ​ 
      ​ = ) p
__
​ q ​, where p, q Œ N,

x Æ 0
​ ( 
x  + sin x – x cos x – tan x
lim ​  ​ _____________________
30. If ​      
xn
  )
  ​  ​ exists and have a find the value of (q – p).

non-zero finite value, then find n. lim ​ (4n + 3n)1/n


10. If P = ​  
n Æ •

31. Find the value of and ​  


1
lim ​ ​ ___
​      (  1
​ – ______
​      )
 ​  ​ =
m
__
​ n ​ , where

(  )
n x Æ 1 ln x (x – 1)
n ◊ 1 +  (n – 1) (1 + 2)  + (n – 2) (1 + 2 + 3)  + ... + 1 ​S  ​ ​ ​
m, n, p Œ I +, find the value of (m + n + p)
​ ​ ​ _____________________________________
r = 1
​  
lim               ​  ​

(  (  ) )
n Æ •
n4
2x3
32. Find the value of 2x – sin 2x – tan– 1 ​ ​ _____   ​  ​
1 + x6 p
lim ​ ​ ​ ________________________ ​ = – ​ __

( 
11. If ​        ​     q ​
)
3
14 + 24 + 34 + ... + n4 x Æ 0 x
  ​  ​ ​ ______________________________________
​ lim      
       ​  ​
n Æ • (​  1 + 2 + 32 + ... + n2  )​ ​( 13 + 23 + 33 + ... + n3  )​
2 2 
where p, q Œ N, find the value of ( p + q)

Integer Type Questions 12. If ​  


x Æ • {  (  (  ) x+2
lim ​ ​  x ​ tan– 1 ​ _____
​ 
x+1 (  x
 ​  ​ – tan– 1 ​ _____
​     
x+2 ))}
 ​  ​  ​  ​

1. If ​  
x Æ 0
( 
ln (1 +  x + x2 + ... + xn)
lim ​ ​ ____________________
​     
n x    ​  ​ = ) 1
__
​   ​ , find the value
5
exists and finite and equal to ​ __
m
n ​ , find (m + n).

of n. Comprehensive Link Passages


2. If ​ l im  
x Æ 0 ( 
tan x  – sin x
​ ​ ​  __________
x 3
 ​ 
    
a
b )
​ = ​ __ ​, find the value of
(For JEE-Advanced Exam Only)
In these questions, a passage (paragraph) has been given
(a + b + 3) followed by questions based on each of the passage. You
3. Let P (x) = a1x + a2 x2 + ... + anx n, where a1 = 1, have to answer the questions based on the passage given.
Passage I
(  )
________
64
÷
​  
1 + P (x) ​
 – 1
ai Œ R, i = 1, 2, 3, ..., 64 such that ​   lim ​ ​ _____________
​  x     ​   ​ Let f (x) and g (x) be two real functions and a Œ R such that
x Æ 0
f (a) = 0 = g(a) and f (a) = • = g (a), then
_________

(  )
a

(  )
__
= ​   ​, find the value of ​÷b  +  a – 1 ​ 
b f (x) f ¢(x)
  ​ ​ ____ lim ​  ​ _____

( 
​ lim  ​     ​  ​ = ​   ​   ​  ​

) g (x) g¢(x)
x Æ a x Æ a
x2n sinn x
___________
4. If ​  
lim   ​ ​ ​  2n      ​  ​ is a non-zero finite number

(  )
x Æ 0 x – sin2n x f ¢¢(x)
find n. lim ​ ​ ​ _____ 
= ​    ​  ​
x Æ a g≤(x)
____________
5. If ​  
x Æ •
( ÷  1
​ __
lim ​ ​ ​ x2 + x2 sin   (  ) ) a
​ x ​  ​ ​ – x  ​ exists and equal to __
​   ,​
b
0
untill and unless the form of ​ __ (  )
​   ​   ​ and
0
​ •
• ​  )​ is removed.
(​  __
find the value of (a + b).
On the basis of above information, answer the following
questions:
3.16  Differential Calculus Booster

1. The value of ​   ________


lim ​ ​ ​  ( 
1
x2 sin ​ __
​ x ​  ​
     
(  )
​  ​ is ) On the basis of above information, answers the
following questions:

(  )
x Æ 0 sin x
x2
(a) 0 (b) 1 x – ​Ú ​  ​   ​ cos (t 2) dt
(c) 1/2 (d) 1/4 lim ​ ​ ​ ______________
0
1. The value of ​        ​  ​ is
  

( 
x3 – 6x
)
x Æ 0
2
x + 2 cos x – 2
lim ​ ​ ​ _____________
2. The value of ​     
    ​  ​ is
x Æ 0 x sin3 x (a) – 1/6 (b) 1/8
(a) 1/6 (b) 1/12 (c) – 1/10 (d) 1/12

(  )
(c) 1/8 (d) 1/16 x2 ___
sin (p cos  x) 2 Ú​  ​  ​ ​ sin  ​÷tdt 
   ​
lim ​ __________
3. The limit of ​   ​   ​
     is lim ​ ​ ​ ________
2. The value of ​  
0
 ​   
 ​
x Æ 0 x2 x Æ 0 x3
(a) – p (b) p
(a) 1/2 (b) – 1/2
p
(c) ​ __ ​   (d) None (c) 1/3 (d) Non-existent
2

(  )
x
1 t log (1 + t)
Passage II 3. The value of ​   ​    ​  × Ú​  ​  ​ ​ __________
lim ​ __ ​  4     ​   
​ dt is
x Æ 0 x 3 0 t +4
Let f (x) and g (x) be two real functions and
(a) 0 (b) 1/12
a ŒR such that ​   lim ​ (g (x)) = •
lim ​  ( f (x)) = 0 and ​   (c) 1/24 (d) 1/64
x Æ a x Æ a

Then we can write ​  


lim ​ ( f (x).g (x)) Matrix Match
x Æ a

​ 
x Æ a 1/g (x)(  ) (  )
f (x)
lim ​  ​ ______
= ​     ​  
​ form
0
​ __
​   ​   ​
0

(For JEE-Advanced Exam Only)
1. Match the following columns
or ​   (  )
​ 
x Æ a 1/f (x)
g (x)
lim ​   ​ _____  
 ​   ​ •
• ​  )​
​ form ​( __
( 
a sin x – bx + cx2 + x3
lim ​ ​​ _____________________
If ​      
     ​  ​
x Æ 0 2x 2 log  (1  +  x)  –  2 x 3  + x 4 )
e
On the basis of above information, answer the following
questions: exists and is finite.
1. The value of ​   
lim  
​ (cot x ◊ log (sec x)) is Column I Column II
x Æ p/2
(a) 0 (b) 1
(A) a (P) 6
(c) –1 (d) None
2. The value of ​  
x Æ 0 sin x
1
lim ​ ​ ____ ( 
1
​ – __
​       ​ x ​  ​ is ) (B)

(C)
b
c
(Q)

(R)
0

12
(a) 0 (b) 1
(c) 2 (d) –1 (D) a+b+c (S) 5

3. The value of ​  


1
lim ​ ​ _____
​     
x Æ 0 tan2 x ( 
 ​ – __
1
​  2  ​   ​ is
x ) 2. Match the following columns

(a) 2/3 (b) – 2/3 Column I Column II



Passage III
(c) 4/3 (d) – 4/3
(A) ​   (  )​ 
x Æ 0 tan {x}
{x}
lim ​ ​ ______   ​  
​ (P) – log16 e

( 
_________ _____
Let j (x) and y (x) be two differentiable functions, then    
  – x  + x2 ​ 
÷​ 1   – ​÷1  + x2 
 ​
)
( 
(B) lim ​ ​ ​  __________________    ​ (Q) e– 1

)
y (x) ​   x  ​ 
  
d x Æ 0 4 –1
​ ___   ​ ​ ​Ú  ​ ​ ​   f (t) dt  ​

(  )
dx j(x)
2esin x – (1 + sin x)2
(C) lim ​ ​ ​  ________________
​     
    ​  ​ (R) 1
= f (y (x)) y ¢ (x) – f (j (x)) j ¢(x) x Æ 0 2 [tan– 1 (sin x)2]

(  ) (  )
sin x
_______
x3 sin x ​ x – sin x   

d
Now ​ ___   ​ ​ ​Ú ​ ​ ​ et dt  ​ = (​ex​  ​◊ 3x2 – e​ x​  ​◊ 2x)
3 2
lim ​ ​​ ​ ____
(D) ​   x ​  ​​
   ​ (S) 0
x Æ 0
dx x2
The Limit  3.17

3. Match the following columns 6. Match the following columns


Column I Column II Column I Column II

(A) ​  
x Æ 0 {  log (1 + x) x____
lim ​ ​  ​ _________
x 2
 ​ 

–1
​   ​ (P)
+ ​  x    } 1
__
​   ​ 
8 x Æ • { 
3 – 2x2
lim ​ ​  ​  ______ ​
(A) ​  
x+1
  + ax + b  ​ = 1 } (P)
a = 1,
b = – 2

(  (  )
X
4t 2 dt ____ a = 1,
(B)
x Æ 0
27x__– 9x ________
÷    – ÷
​ 2 ​
– 3x + 1
lim ​ ​ ​  _____________
​       ​  ​
  +  cos x  
​ 1   ​ ) (Q)
3
__
​   ​ 
2
lim ​  ​  ​Ú ​  ​  ​ _______________
(B) ​          ​ = 1  ​ (Q)
x Æ 0 0 (bx – sin 2x) ​÷t  – a   ​ b=2

(  (  )
1
__ 1
__

)
a​e– ​ 
​ x ​​ – 2​e​ ​x ​​ a = 2,
lim ​ ​ ​ _________
2
e​ x​ ​ – cos x __ (C) ​    
 ​= 1  ​ (R)
(C) lim ​ ​ _________
​   ​   ​ 
    
​    (log3)2
(R) 8​÷2 ​  x Æ • 1
__ __ 1 b = –1
x Æ 0 x2 ​e– ​ 
​ x ​​ + b​e​ ​x ​​

(D) ​   ( 
1 – cos (1 – cos x)
lim ​  ​  ​  _______________
 ​ 
     ​ (S) ) 1
__
​   ​ 
2
( 
b sin x – a sin 2x
lim ​ ​ ​ ______________
(D) ​  
x Æ 0 e– x × cos 2x × x 3
    ​  = 1  ​
  
) (S)
a = –1,
b = 2.
x Æ 0 x4

4. Match the following columns


Assertion (A) and Reason (R) Code
(A) Both A and R are true and R is the correct
Column I Column II
explanation of A

(  )
a a
1 (B) Both A and R are true and R is not the correct
​ x ​ ​ ​Ú ​    ​ ​e​sin t​ dt – ​Ú   ​ ​   ​ ​e​sin t​ dt  ​ (P)
lim ​ __
2 2
(A) ​   1

 ( )
x Æ 0
explanation of A.
y x + y
(C) A is true and R is true.

( Ú  )
x + y 2 (D) A is false and R is true.
2
​​ ​  ​  ​   ​ ​e​t ​ dt  ​​ ​
lim ​ ​ ​ __________
(B) ​  
x Æ 0

0
x + y
      ​  ​ (Q) e​ sin
​ ​y
2
1. Assertion (A): The value of ​ lim 
   
x Æ 0 +
sin x
​ ​ ​ ____ ​  ​ = 0
x    [  ]
​Ú ​   ​   ​ ​e​2t ​ dt
2
Reason (R): sin x £ x for all x in R

( 
0
________

)
(  )
​ x  –   sin x  
÷ ​
x 3 2. Assertion (A): The value of ​   lim ​ ​ ​ _________  
 ​  ​ is 1.
_ x Æ • 2
(C) Ú​  ​  ​   ​ sin ​÷t   ​d  t (R) 0
x + cos x
​  
x Æ 0
y
lim ​ ​ ​ _________
x3
 ​    
​ Reason (R): ​  
x Æ •
sin x
lim ​ ​ ​ ____ ​  ​ = 0 = ​  
x    (  )
x Æ •
cos x
lim ​ ​ ​ ____ ​  ​
x    (  )
3. Assertion (A): The limiting value of

(  )
x _________
(D)
​  
​Ú ​    ​ cos t 2 dt
y
lim ​ ​ ​ _________ ​  ​
(S)
2
__
​    ​
3
​ lim 
x Æ 0
÷
​ 1  
( 
  –   cos 2x  
  ​  ​  ​ __________

x     ​   )
​ does not exist.
x   
x Æ 0
Reason (R): The limiting value  
​ lim 
|x|
​  ​ __
​ x ​   ​does not exist.
x Æ 0
(  )
5. Match the following columns 4. Assertion (A): The limiting value of

Column I Column II
( 
tan–1 x – sin– 1x
  ​ ​ ​ _____________
​ lim   ​ 
    
1
​ is – ​ __ ​  )
(  )
__ x Æ 0 x 3 2
sin ​÷q 
   ​
lim ​ ​ ​ ______
(A) ​   ___    ​  ​ (P) – 2 Reason (R): The limiting value of
q Æ 0 ÷    q
​ sin ​ 

(  )
loge cos x
lim ​ ​ ________
(B) ​   ​   ​    ​ (Q) 1
​  
lim 
x Æ0  
( 
cos (sin x) – cos x
​  ​  ​ ______________
x4
 ​ 
    
1
​ is __
​    ​
6 )
x Æ 0 x2

( 
5. Assertion (A): The limiting value of

)) (  )
3
tan  x – 3 tan x sin x – x 1
lim ​ ​ ​  ____________
(C) ​      ​  ​
   1   ​ ​ ________  ​ is – ​ __  ​
(  – ​ __ ​ 
p (R) ​ lim  ​   ​  
x Æ 0
cos ​ x + __
​   ​   ​ 2 x Æ 0 x 3 6
6

( 
Reason (R): The limiting value of

(D) ​  
lim ​  ​ ​ 
x Æ •
   
1
(3x4 + 2x2) sin ​ __
​ x ​  ​+ |x|2 + 5
______________________
   
|x|3 + |x|2 + |x| + 1
 ​  ​
(  )
) (S) – 24. ​ lim 
x Æ 0
​  ( 
  ​  ​ _________
x2
2
​ex​ ​– cos x
 ​   
​ is 3/2 )
3.18  Differential Calculus Booster

Problems asked in JEE Main Exams. 8. The value of


From 2002-2014
​ lim 
    ( 
14 + 24 + ... + n4 1______________
​ ​ ​ ______________
 ​
     – ​ 
3
+ 23 + ... + n3
 ​ 
     ​ is )
( 
n Æ • 5
n4
)
1p + 2p + 3p + ... + np n
1. ​ lim 
    ​ ​ ​ __________________
     ​     ​ equals
n Æ •
np + 1 (a) 0 (b) 1/4
1 1 (c) 1/5 (d) 1/30
(a) ​ _____    ​ (b) ​ _____
   ​
p+1 1–p
[JEE Main, 2003]

( 
1
(c) ​ __
1
​ p ​ – _____
​     
p–1 )
 ​  ​
1
(d) ​ _____
p+2
   ​
9. Let f (a) = g (a) = k and their nth derivatives f n (a),
gn (a) exist and are not equal for some n. Further,
[JEE Main, 2002] if

(  ) (  )
n f (a) g (x) – f (a) – g (a) f (x) + g (a)
log x – [x]
  ​  ​ __________
2. ​ lim  ​   ​ 
    ​, n Œ N, [,] = G.I.F   ​ ​_____________________________
​ lim  ​          
 ​  ​ = 4
x Æ • [x] x Æ a g (x) – f (x)
(a) –1 (b) 0 then the value of k is
(c) 1 (d) does not exist. (a) 2 (b) 1
[JEE Main, 2002] (c) 0 (d) 4
3. Let f (2) = 4 and f ¢(2) = 4, then

( 
[JEE Main, 2003]

( 
xf (2) – 2f (x)
  ​ ​ ___________
​ lim 
x Æ 2
​ 
x–2
  
 ​   ​ equals to ) 10. If ​  
x Æ •
a __
lim ​ ​​ 1 + ​ __ ​
x x2 + ​ 
b 2x
  ​ 
  ​​ ​ = e
)2
, then the values of a and

b are
(a) 2 (b) – 2
(a) a Œ R, b = 2 (b) a = 1, b ŒR
(c) – 4 (d) 3
(c) a Œ R, b Œ2 (d) a = 1, b = 2.
[JEE Main, 2002]

( 
[JEE Main, 2004]

)
x
x2 + 5x + 3

(  (  ) (  ) )
  ​  ​​ ​ __________
(  )
4. ​ lim     ​  
​​ ​ is equal to
x Æ •
x2 + x + 3 1 1 2 4 1 n2
  ​ ​ __
11. ​ lim  ​  2  ​  sec2 ​ __
​  2  ​   ​  + ​ __2  ​  sec2 ​ __
​  2  ​   ​ + ... + __
​  2  ​  sec2 ​ ​ __2  ​  ​  ​
n Æ •
4 2 n n n n n n
(a) e (b) e
(c) e3 (d) e is equal to
1
[JEE Main, 2002] (a) ​ __ ​  tan (1) (b) tan (1) – log (sec 1)

(  )
x2
2
Ú​  ​  ​ ​ sec2 t dt
1
(c) ​ __ ​  cosec (1)
1
(d) ​ __ ​  sec (1)
lim ​  ​  ​ ________
5. The value of ​  
0
    
​  ​ is 2 2
x Æ 0 x sin x [JEE Main 2005]
(a) 3 (b) 2 12. Let f : R Æ R be a differentiable function having
(c) 1 (d) 0
(  )
f (x)
1 4t 3
[JEE Main, 2003] lim ​ ​Ú ​   ​ ​ ​ _____
f (2) = 6, f  ¢ (2) = ​ ___  ​,  then ​   ​      ​  ​ dt equals
48 x–2
(  )
x Æ 2 6
log (3 + x) – log (3 – x)
lim ​ ​ _____________________
6. If ​   ​     
x    ​  ​ = k then the value
x Æ 0 (a) 18 (b) 12
of k is (c) 36 (d) 24
(a) 0 (b) – 1/3 [JEE Main 2005]
(c) 2/3 (d) – 2/3 13. Let a and b be the distinct roots of ax2 + bx + c = 0,
then the limiting value of

( 
[JEE Main, 2003]

(  )
( 
)
1 – cos (ax2 + bx + c)

7. ​ lim 
  p ​  ​ ​  ____________
(  ) )
​ 1 – tan ​ __
x
​    ​  ​  ​
2 (1 – sin x)
   ​ × _________
   ​   ​  
  ​ is
  ​  ​  ​ __________________
​ lim 
x Æ a
   
(x – a)2
 ​ 
   ​ is

2 (  (  ) )
x Æ ​ __ ​  ​ 1  +  tan ​ __
2
x
​    ​  ​  ​ (p – 2x)
3


1
(a) ​ __ ​  (a – b)2
a2
(b) – ​ __ ​  (a – b )2
(a) 1/8 (b) 0 2 2
(c) 1/32 (d) 1/16 a2
(c) 0 (d) ​ __ ​  (a – b )2
[JEE Main, 2003] 2
[JEE Main 2005]
The Limit  3.19

14. No questions asked in 2006-2009.


15. Let f : R Æ R be a positive increasing function
3. Evaluate: ​  
x Æ 0 ​ (1 ( 
2x – 1
lim ​ ​ ​ __________
_______  
   ​  ​
÷  + x) ​ –1 )
(  ) (  )
f (3x) f (2x) [IIT-JEE, 1982]
lim ​  ​ _____
with ​   lim ​ ​ ​ _____ ​  ​ is
​   ​  ​ = 1. Then ​   _______
x Æ • f (x) x Æ • f (x)   –x 
4. If G (x) = – ​÷25  ​, then 2



(a) 1
(c) 3/2
(b) 2/3
(d) 3 x Æ 1
​ ( 
G (x) – G (1)
  ​ ​ ___________
​ lim 
x–1
  
 ​   )
​ has the value

[JEE Main, 2010] 1 1


(a) ​ ____
___   ​   (b) ​ __ ​ 

( 
_____________ 5
)
÷    
​ 24 ​
​÷1  – cos 2 (x   
_______________– 2) ​ ___
16. ​ lim 
  ​ ​ ​    
 ​ 
   ​ is    
(c) – ​÷24 ​ (d) None
x Æ 2 (x – 2)
__
[IIT-JEE, 1983]
1__
  
(a) – ​÷2 ​ (b) ​ ___  ​  5. If f (a) = 2, f ¢ (a) = 1, g(a) = – 1, g ¢ (a) = 2, then the
÷   
​ 2 ​
__ value of


(c) does not exist (d) ​÷ 2 ​ 
[JEE Main, 2011]
​ lim 
x Æ a
( 
g (x) f (a) – g (a) f (x)
  ​  ​_________________
​  x – a        ​  ​ is )
17. No questions asked in 2012. (a) – 2 (b) 1/5

( 
(c) 5 (d) None
x Æ 0
(1 – cos 2x) (3 + cos x)
  ​ ​ ___________________
18. ​ lim  ​     
x tan 4x
     ​  ​ is equal to ) [IIT-JEE, 1983]



(a) 1/2
(c) 2
(b) 1
(c) –1/4
6. ​ lim 
   
n Æ • [  1
​ ​ _____
​     
2
 ​ + _____
​     
1 – n2 1 – n2
n
 ​ + ... + _____
​     
1 – n2 ]
 ​  ​ is equal to

[JEE Main, 2013] (a) 0 (b) – 1/2

(  )
2
sin (p cos  x) (c) 1/2 (d) None
  ​  ​  ​ __________
19. ​ lim   ​ 
     ​ is equal to
x Æ 0 x2 [IIT-JEE, 1984]
p
(a) ​ __ ​   (b) 1 Ï sin[ x ]
, [ x] π 0
2 Ô
7. If f (x) = Ì [ x ] , where [,] = G.I.F,
(c) – p (d) p Ô0, [ x ] = 0
Ó
[JEE Main, 2014]

(  (1 – cos 2x) (3
  ​ ​ ​ ___________________
20. ​ lim 
x Æ 0
   
x tan 4x
+ cos x)
)
     ​  ​ is equal to

(a) 1
then ​  
lim ​    f (x) is equal to
x Æ 0
(b) 0
(a) 3 (b) 2 (c) 1 (d) None
(c) 1/2 (d) 4 [IIT-JEE, 1984]

__ ___ 1
​     ​
[JEE-Main, 2015]
8. ​ lim 
x Æ 1(  p x
  ​ ​ (1 – x) tan ​ ___ (  ) )
​   ​  ​  ​ = ...
2
[IIT-JEE, 1984]

( 
tan2 ​÷x  ______________

)
21. If p = ​ lim 
  +  ​ (1 +   2x ​,
  ​​)​ then the value of log (p)
x Æ 0 ​÷1  – cos 2 (x   
_______________ – 1) ​
is 9. The limiting value of ​  
lim ​  ​  ​   ​ 
  
   ​ is
x Æ 1 x–1
(a) 1 (b) 1/2 __ __
(c) 1/4 (d) 2   
(a) ​÷2 ​   
(b) – ​÷2 ​
[JEE-Main, 2016] (c) Non-existent (d) None
[IIT-JEE, 1985]
Questions asked in Past IIT-JEE Exams.
Ïsin x, x π np , n Œ Z
1. Evaluate: 10. If f (x) = Ì

( 
Ó2, otherwise
​ lim 
   
h Æ 0
(a + h)2 sin (a + h) – a2 sin a
​ ​ ​ ________________________
   
h
    ​  ​ ) Ï x 2 + 1, x π 0, 2
Ô
[IIT-JEE, 1980]
and g (x) = Ì4, x = 0 ,
2. Prove that:
Ô5, x = 2

( 
Ó
​ lim 
   
1
​ ​ _____
​     
n Æ • n + 1
1
 ​ + _____
​     
n+2 n+3
1
 ​ + _____
​     
1
 ​ + ... + ___
​     ​  ​= loge 6
6n ) then ​  
lim ​  (g ( f (x))) is...
x Æ 0
[IIT-JEE, 1981] [IIT-JEE, 1986]
3.20  Differential Calculus Booster

(  (  )
) ( 
______________

11. ​ lim 
1
x4 sin ​ __
​   ​   ​ + x2
4
  ​  ​ ​ ____________   
  
 ​  ​ = ... [IIT-JEE, 1987] x Æ 1
÷​ 1  – cos 2 (x   
  ​ ​ ​ _______________
23. ​ lim 
x–1
– 1) ​
 ​ 
     ​ )
x Æ •
(1 + |x|3) __
(a) exist and equals to ÷   
​ 2 ​

(  )
__
x2   
(b) exist and equals to – ​÷2 ​
Ú​  ​  ​ ​ cos2 tdt (c) does not exist.
  ​  ​  ​ ________
0
12. ​ lim      
​  ​ = ... [IIT-JEE, 1987] (d) None of these.
x Æ 0 x sin x
[IIT-JEE, 1998]
13. If f (9) = 9, f  ¢ (9) = 4, then 24. ​ lim 
x Æ 0 ( 
x tan 2x – 2x tan x
  ​  ​  ​ _______________
  
  
(1 – cos 2x)2 )
 ​  ​ equals to

( 
____

​ lim 
x Æ 9
   – 3
​÷f (x) ​
  ​ ​ _________
​  __
​ x 
÷    ​ – 3
 ​  
 ​ is equal to ... ) [IIT-JEE, 1988]

(a) 2
(c) 1/2
(b) – 2
(d) – 1/2
14. ABC is an isoceles triangle inscribed in a circle of [IIT-JEE, 1999]
radius r. If AB = AC and h is the altitude from A to
BC, then the triangle ABC has perimeter
25. For x Œ R, ​   (  )
x Æ • x + 2
–3 x
x_____
lim ​ ​​ ​   
 ​  ​​ ​ is equal to

________ (a) e (b) 1/e


___
P = 2 ​( ​÷2hr
  – h2 ​     ​ )​ and area A is equal to ...
+ ​÷2hr  5
(c) e (d) e– 5

(  )
[IIT-JEE, 2000]

(  )
A
lim ​ ​ __
Also ​   ​    ​   ​ = ... [IIT-JEE, 1989] 2
sin (p  cos  x
h Æ 0 p3   ​ ​ ​ __________
26. ​ lim   ​ 
     ​ is equal to
x Æ 0 x2
15. No questions asked in 1990.
(a) – p (b) p

( 
÷  )
___________
1 p
​ __ 2
​   ​  (1 – cos  x) ​ (c) ​ __ ​   (d) 1.
2 2
lim ​ ​ ​ ____________
16. The value of ​   x      
​  ​ is [IIT-JEE, 2001]
x Æ 0



(a) 1
(c) 0
(b) – 1
(d) None x Æ 0
​  ( 
(cos x – 1) (cos x – ex)
lim ​ ​ __________________
27. The integer n for which ​      
xn
    ​  ​ )
[IIT-JEE, 1991] is a finite non-zero number is
17. ​ lim  (  )
x + 6 x + 4
  ​ ​​ _____
​ 
x Æ • x + 1
 ​  ​​ ​ = ... [IIT-JEE, 1991]

(a) 1
(c) 3
(b) 2
(d) 4.

  ​ ​  _______
[IIT-JEE, 2002]

( ​ tanx  2x 
18. ​ lim 
x Æ 0 )
 ​  ​ equals
– 1
[IIT-JEE, 1992]
28. Let f  :  R Æ R be such that f (1) = 3 and f ¢ (1) = 6,
p
(  (  )) (  )
1/x f (1 + x) 1/x
lim ​ ​​ tan ​ __
19. Evaluate: ​   ​   ​  + x  ​  ​​ ​. [IIT-JEE, 1993] lim ​  ​​ ________
then ​   ​   ​  

​​ ​ is
x Æ 0 4 x Æ 0 f (1)

(  )
1
5x2 + 1 ​ x2  ​ 
__
(a) 1 (b) e1/2
  ​  ​​ _______
20. ​ lim  ​  2  ​  
​​ ​ = ... [IIT-JEE, 1996]
x Æ 0 3x + 1 (c) e2 (d) e3
[IIT-JEE, 2002]

h Æ 0 ( 
log (1 + 2h) – 2 log (1 + h)
lim ​   ​_______________________
21. ​   ​     
h2
    ​  ​ = ... ) h Æ 0 ( 
f (2 + h + h ) – f (2)
  ​ ​ __________________
29. ​ lim  ​    
   ​  ​
f (h – h2 + 1) – f (1)
2

)
[IIT-JEE, 1997]

( 
(given that f ¢ (2) = 6, f  ¢ (1) = 4
22. The value of ​  
1 2n ______
n Æ • n  r = 1

__
​ 
r
​   ​  ​S   ​ ​ ​ _______
lim ​  ​ __     ​  ​ is
÷​ n  +  r 2 ​ 
2   ) __


(a) Does not exist
(c) 3/2
(b) – 3/2
(d) 3
  
(a) 1 + ​÷5 ​   
(b) – 1 + ​÷5 ​ [IIT-JEE, 2003]

(  )
__ __
  
(c) –1 + ​÷2 ​   
(d) 1 + ​÷2 ​ [(a – n)nx – tan x] sin nx
lim ​ ​ ​ _____________________
30. If ​       ​     ​ = 0,
x Æ 0 x2
[IIT-JEE, 1997]
where n is non-zero real number, then a is
The Limit  3.21
n
(a) 0 (b) ​ _____
    ​
n+1 (a) 0 (b) 1/12
1
(c) n (d) n + __​ n ​ (c) 1/24 (d) 1/64
[IIT-JEE, 2010]

(  )
[IIT-JEE, 2003] 1
x 2 __
40. If ​  
lim ​  (1 + x log (1 + b2)​)​ ​x ​​ = 2b tan2q, b > 0 and
​Ú ​  ​ ​ sec2 tdt x Æ 0

lim ​ ​ ​ ________ q Œ (– p, p), then the value of q is


0
31. The value of ​       
​  ​ is
x Æ  0 x sin x
p p
(a) 3 (b) 2 (a) ± ​__
   ​   (b) ± ​ __ ​ 
4 3
(c) 1 (d) 0
p p
[IIT-JEE, 2003] (c) ± ​ __ ​   (d) ± ​ __ ​ 
6 2
32. If f (x) is differentiable and strictly increasing [IIT-JEE, 2011]

(  ) (  )
2 2
f (x ) – f (x) x +x+1
lim ​ ​ __________
function, then the value of ​   ​      ​  
​ is 41. If ​   ​  ​ ​ _________
lim    ​ 
 – ax – b  ​ = 4, then
x Æ 0 f (x) – 0 x Æ 0 x+1
(a) 1 (b) 0 (a) a = 1, b = 4 (b) a = 1, b = – 4
(c) –1 (d) 2. (c) a = 2, b = – 3 (d) a = 2, b = 3
[IIT-JEE, 2004] [IIT-JEE, 2012]
33. Find the value of 42. If
(  (  ) )
( 
2 1


  ​ ​ __
​ lim 
n Æ • p
​   ​ (n + 1) cos– 1 ​ __
​ n ​  ​ – n  ​
[IIT-JEE, 2004]
​ lim 
n Æ •
1a + 2a + ... + na
  ​ ​ _________________________________________
     
    
(n + 1) a – 1 ((na + 1) + (na + 2) + ... + (na + n))
 ​  ​
)
34. No questions asked in 2005. 1
= ___
​    ​ , where a Œ R, then the value of a is
35. For x > 0, ​  
x Æ 0
( 
lim ​  ​  sin x​)​ ​+ ​​ __
1
1 sin x
__
​ x ​ 
​ x ​  ​​ ​  ​ is(  ) )
60
(a) 7 (b) 1/7
(a) 0 (b) –1 (c) – 7 (d) 1/5
(c) 1 (d) 2

(  )
[JEE Advanced, 2013]
[IIT-JEE, 2006]
43. The largest value of the non-negative integer a for
sec2 x
which
​Ú ​   ​ ​ f (t) dt

{  }
1 – x
lim ​  ​  ​ ________ – ax + sin (x – 1) + a _____
2 ​  __   ​  1
36. The value of ​    ​  
​ is equal to
x Æ 0 p2   ​ ​​ ___________________
​ lim  ​          ​  ​​1 – ​÷x   ​ ​ = __
​    ​ is ...
2
x – ​   ​ ___ x Æ 1 x + sin (x – 1) – 1 4
16
[JEE-Advanced, 2014]
8 2
(a) ​ __
p ​  f (2) (b) ​ __
p ​  f (2) p
44. Let f (x) = sin ​ __ (  ( 
p
​   ​  sin ​ __ ))
​   ​  sin x  ​  ​ for all x in R

(  )
6 2
2 __ 1
(c) ​ __
p ​  f  ​ ​ 2 ​   ​
(  )
(d) 4 f (2).
( 
p f (x)

37. No questions asked in 2008.
[IIT-JEE, 2007] and g (x) = ​ __ ) lim ​  ​ ____
​   ​  sin x  ​, " x Œ R, then ​  
2 x Æ 0 g (x)
​     ​  ​ is ...

(  )
______ [JEE-Advanced, 2015]
x2
a – ​÷a  2 – x2  ​ – ​ __ ​ 
4 45. Let m and n be two positive integers greater than 1.
38. Let L = ​  
lim  ​ ​ ​  _______________
4
 ​ 
     ​, a > 0
x Æ  0 x

(  ) (  )
n
​ecos (a
​ ) – e
​ e
If L is finite, then If ​ lim    ​ ​ ​ ________
m   ​    ​ = – ​ __
​    ​  ​
a Æ 0 a 2
(a) a = 2 (b) a = 1 m
then the value of ​ ​ __ (  )
n ​   ​ is ... [JEE-Advanced, 2015]
1 1
(c) L = ___
​    ​   (d) ​ ___  ​  46. Let a, b, Œ R be such that
64 32

(  )
[IIT-JEE, 2009] x2 sin (b x)
  ​ ​ ​ _________ 
( 
​ lim   ​  ​ = 1. Then 6 (a + b) is ....
)
x
1 t log (1 + t) x Æ 0 a x – sin x
​    ​  ​Ú ​  ​ ​__________
lim ​ __
39. The value of ​   ​  4     ​  ​ dt is
x Æ 0 x 3 0 t +4 [JEE-Advanced, 2016]
3.22  Differential Calculus Booster

Answers
Level II Integer Type Questions
1. (c) 2. (b) 3. (d) 4. (c) 5. (d) 1. 5 2. 6 3. 8 4. 2 5. 3
6. (a) 7. (b) 8. (a) 9. (c) 10. (b) 6. 4 7. 7 8. 3 9. 5 10. 7
11. (d) 12. (b) 13. (b) 14. (c) 15. (c) 11. 5 12. 3
16. (b) 17. (a,b,c) 18. (a) 19. (b) 20. (a)
Comprehensive Link Passages
21. (c) 22. (b) 23. (a) 24. (c) 25. (b)
Passage I : 1. (a) 2. (b) 3. (b)
26. (c) 27. (a,c) 28. (c) 29. (c) 30. (b) Passage II : 1. (a) 2. (a) 3. (b)
31. (b) 32. (a) 33. (a) 34. (c) 35. (a) Passage III : 1. (a) 2. (d) 3. (b)
36. (c) 37. (c) 38. (d) 39. (a) 40. (c)
Matrix Match
41. (a) 42. (a) 43. (a) 44. (b) 45. (b)
1. (A)Æ(P), (B)Æ(S), (C)Æ(Q), (D)Æ(R)
46. (d) 47. (b) 48. (b) 49. (b) 50. (d)
2. (A)Æ(Q), (B)Æ(A), (C)Æ(S), (D)Æ(Q)
51. (a) 52. (b) 53. (b) 54. (b) 55. (b)
3. (A)Æ(S), (B)Æ(R), (C)Æ(Q), (D)Æ(P)
56. (a) 57. (a) 58. (a) 59. (c) 60. (b) 4. (A)Æ(Q), (B)Æ(R), (C)Æ(S), (D)Æ(P)
61. (b) 62. (a) 63. (a) 64. (b) 65. (a) 5. (A)Æ(Q), (B)Æ(R), (C)Æ(S), (D)Æ(P)
66. (*) 67. (c) 68. (a) 69. (a) 70. (c) 6. (A)Æ(R), (B)Æ(S), (C)Æ(P), (D)Æ(Q)
71. (a) 72. (b) 73. (d) 74. (b) 75. (a)
Assertion and Reason
1. A 2. A 3. B 4. B 5. B

Hints and solutions

(  )
1
– ​ __ ​
1 – ​x​ 3 ​
​  ​ ​ _______2 
2
1. ​  
lim ​  (x – 6x + 10) 4. ​  
lim   ​  

x Æ 1 x Æ1 – ​__

(  (  ) )
   ​
1 – ​x​ ​3
= 1 – 6 + 10 1
– ​ __ ​ 
1 – ​x​ 3 ​
= 11 – 6 lim ​ ​ ​ ___________
= ​     
   ​  ​
x Æ 1 1 2
– ​ __ ​ 
=5 2
(1) – ​​ ​x​ ​  ​​ ​
3



lim ​   (x2016 – x2017 + 2)
2. ​  
x Æ 1

=1–1+2
=2

x Æ 1
 ( (  (  )(  ) ) )
​ 1 – ​x​ 3 ​  ​
lim ​  ​ ​ ________________
= ​  
1
– ​ __ ​ 
3
      ​  ​
1
1
– ​ __ ​ 

– ​ __ ​ 
​ 1 + ​x​ ​  ​ ​ 1 – ​x​ 3 ​  ​

3. ​   ( x2 – 3x + 2
lim ​ ​ ​ __________
  
   ​  ​
x Æ 1 x 2 – 5x + 4 )
x Æ 1
( ( 
1
lim ​ ​ ________
= ​   ​     
1
– ​ __ ​ 
​ 1 + x​ ​ 3 ​  ​ ) ) 1
 ​  ​ = __
​   ​ 
2

( (x – 1) (x – 2)
lim ​ ​ ​ ____________
= ​     
   ​  ​
x Æ 1 (x – 1) (x – 4) ) 5. ​  
x Æ 2 (  x3 + 7x2 – 36
lim ​  ​ ​ ____________
      
x2 + 2x – 8
​  ​ )
= ​   ( (x – 2)
lim ​ ​ ​ ______ 
x Æ 1 (x – 4) )
 ​  ​
( 
(x – 2) (x + 3) (x + 6)
lim ​ ​___________________
= ​  
x Æ 2
​        
(x + 4) (x – 2)
 ​  ​ )
(  )
1  –  2
= ​ _____
​ 
1–4

–1 1
 ​ ​ = ___
​    ​ = __
– 3 3
​   ​
= ​   ( 
(x + 3) (x + 6)
lim ​ ​ ​ ____________
   ​  
​ )
x Æ 2 (x + 4)
The Limit  3.23
_____ _____ _____ _____
​ 1  + x ​ 
÷ –÷​ 1  – x   ​÷1  + x  ​ + ​÷1  – x ​
​ _______________  

4 × 7 ___
=  ​_____
   
28
​ = ​   ​  lim ​  ​  _____________
= ​          ​ × ​  _____   
   ​
_____
5 5 x Æ 0 2x ​÷1  + x   ​ 1  – x  
​  + ÷ ​

6. ​  
x Æ 1 ( x2  +  x log x – log x – 1
lim ​ ​ ___________________
​     
x3 – 1
 ​     ​ ) = ​  
lim ​ 
x Æ 0 2x (​÷1 ( 
1 +_____
x – 1 + _____
​ ​ ________________
  + x    
  
x
    ​  ​
  –  x  
​  +  ​÷1   ​ )
= ​  
x Æ 1 ( 
(x2 – 1) + (x – 1) log x
lim ​ ​ ​ ___________________
   
x3 – 1
 ​    ​ ) x Æ 0 2x ​ ÷ (  ( 
____
2x
lim ​ ​ ​ _________________
= ​        
​ 1  + x  
____  ​  ​
​+ (​÷1  – x  
​  ​ ))

= ​  
x Æ 1 ( 
(x + 1) + log x
lim ​ ​ ​ _____________
   ​  ​
  
(x2 + x + 1) ) x Æ 0 ​ ÷ ( ( 
1
lim ​ ​ _______________
= ​   ​  _____  _____
​ 1  + x  
    ​  ​
​ )​
​ 1  – x  
​+ ÷ )
2 1
= ​ __ ​ = __
​   ​ 
3 2

( 
______ ______
7. ​ lim 
x Æ 0 ÷( 
​  _____  
​ a  + x  
x
  ​  ​  ______________    ​  ​
_____
​  – ​÷a  – x  
​ ) 11. ​ lim 
x Æ 2
​  – ​÷5   + 2x  
​ 1  + 4x  
÷
  ​ ​​  ________________
x  – 2
 ​      ​

)
( 
______ ______ ______ ______

(  )
_____ _____
= ​  
x Æ 0 ÷ ​ a  + x  
x
lim ​ ​ ______________
​  _____  _____
​– ÷
​ a  – x  
​  a
÷   + x  
_____
​ ​÷a  + x  
​+ ÷
​ a  – x ​
    ​ ×   ​ _______________
​+ ÷
  
_____

   ​  ​
​ a  – x  
​ ) = ​  
lim ​ ​ ​ 
x Æ 2
​  – ​÷5  + 2x  
​÷1  + 4x  
_________________
x  – 2
 ​  
​÷1  + 4x  
     × ​  ______
​  + ÷
​ _________________
​÷1  + 4x  
​ 5  + 2x  
  

    ​ ​
______
​  + ​÷5  + 2x  

(  (  )
_____ _____
x (​÷ a
  + x 
​ + ​÷ a
    – x 
lim ​ ​ __________________
= ​  
x Æ 0
​    
(a + x – a + x)
  
​)

 ​  ​ ) = ​  
lim ​  ​ 
1 + 4x – 5 – 2x
​  _______________________
x Æ 2 (x –  2) ​( ÷
   
______  ______  ​  ​
​ )​
​ + ​÷5  + 2x  
​ 1  + 4x  

x Æ 0
​  ( 
x (​÷ a
  + x 
​+ ÷

2x
​  a
_____
  – x 
lim ​  ​ __________________
= ​       
​)

  ​  ​
_____

) = ​  
x Æ 2 (x – 2) (÷ (  2x – 4 ______
lim ​ ​ ​ _______________________
  
     ​  ​
______
​ + ​÷5  + 2x  
​ 1  + 4x   ​) )
(  (  )
_____ _____ 2 (x – 2)
(​÷a  + x  
​ + ​÷a  – x  
lim ​ ​ ​ _______________
= ​  
x Æ 0 2
 ​      
​)
​ ) lim ​ ​ ​ ________________________
= ​  
x Æ 2 (x – 2) (​÷1
  
   
______ ______  ​  ​
​ + ​÷5  + 2x  
  + 4x   ​)

(  ( 
__ __ __
= ​  
x Æ 0
​ a 
÷
​ 
   ​ + ÷
lim ​ ​ ________
2
​ a ​
   
 ​   ) 2 ​÷a 
 ​ = ____
  ​ 
​   ​ 
2
 = ÷
__
​ a 
  ​ 
x Æ 2 ÷
2 ______
lim ​ ​ ________________
= ​   ​  ______  
​ 1  + 4x ​   
   ​  ​
  +  2x  
+  ​÷5   ​ )
_____
  ​  ​ ( ​ __________ )​
​÷1  + x ​ – 1 2 2 1
8. ​ lim 
  = ​ _______
__   __  ​ = __
​   ​  = __
​    ​
x Æ 0 x     ​      + ÷
​÷9 ​    6 3
​ 9 ​

( 
_____ _____

) ( 
______ __ __

)
​ 1  + x ​ 
÷ 1  + x  
– 1 ​÷__________ ​+ 1
lim ​  ​  ​ __________
= ​   x     ​  
× ​  _____    ​  ​ ​ 7  + 2x  
÷    + ÷
​ – (​÷5 ​   ) 
​ 2 ​
x Æ 0 ​ 1  + x ​+
÷  1 12. ​   lim ___​ ​ ​ __________________
2
 ​ 
     ​
   
x Æ ​÷10 ​ x – 10

= ​  
x Æ 0 x (​÷1 ( 
1 _____
+x–1
lim ​ ​ ​  ____________
  
  + x  
   ​  ​
​ + 1) ) = ​   
lim 
( 
___​ 
÷
______
​ 7  + 2x  
​ – (​÷5 ​
​ ​ __________________
2
 ​
  
__
   + ÷
__
   ) ​÷7
​ 2 ​   + 2x  
______
   + ÷
​ + (​÷5 ​
   × ___________________
​  ______     __   
  ) 
​ 2 ​
__  ​  ​
__ __

)
   
x Æ ​÷10 ​ x – 10 ​÷7  + 2x      + ÷
​ + (​÷5 ​    )
​ 2 ​

( 
x
lim ​  ​  ___________
)
( 
= ​   ​  _____
     ​  ​

)
__ __
x Æ 0 x (​÷1   + x ​ 
+ 1) 7 + 2x – (​÷5 ​    + ÷   )  2
​ 2 ​
= ​   
lim  ​ ​ __________________________________
___​  ___    
    
___ ______ __ __  ​  ​

(  )
1     (x – ÷
x Æ ​÷10 ​   )  (x + ÷
​ 10 ​   )  ​÷7  
​ 10 ​   +  2x      + ÷
​ + (​÷5 ​    )
​ 2 ​
lim ​ ​ ___________
= ​   ​  _____     ​  ​
x Æ 0 (​÷1   + x  
​ + 1)

( 
___
1
= __
​    ​
2
_____
= ​   
lim ___​ ​ 
    (x – ÷
x Æ ​÷10 ​
7 + 2x – (5 + 2 + 2 ​÷10 ​
___          
  ) (x + ÷
​ 10 ​
___
​ 10 ​
__
  )  (​÷7 ​
  ) 
​ ___________________________________ __
   + ÷
    + 2x + (​÷5 ​
__  ​  ​
   )
​ 2 ​ )
( 
3 – ​÷5  + x  
​ ___
  ​ __________
9. ​ lim  ​     ​ 
)
_____ 2 (x – ÷   ) 
​ 10 ​
  – x  
x Æ 0 1 – ​÷5 ​ = ​   
lim ___​ ​ ​ ___________________________________
___        
___ __ __ __  ​  ​
    (x – ÷   )  (x + ÷   ) (​÷7 ​    + ÷
    + 2x + (​÷5 ​    ))

( 
__ x Æ ​÷10 ​ ​ 10 ​ ​ 10 ​ ​ 2 ​
​ 
  
3 – ​÷5 ​
= ​ _______
__ ​  
  
1 – ​÷5 ​
​ ) = ​   
lim 
(  2
 ____________________________
___​ ​ ​  ___       
______ __ __  ​  ​
)
    (x + ÷   )  (​÷7
​ 10 ​   + 2x      + ÷
​ + (​÷5 ​    ))
​ 2 ​
( 
_____ _____ x Æ ​÷10 ​

10. ​ lim 
x Æ 0
​ 1  + x  
÷
​ 
​– ÷
2x
​ 1  – x ​ 
  ​ ​ _______________
       ​  ​ )
3.24  Differential Calculus Booster

( 
   (÷
2 ​÷10 ​ 
2
= ​ __________________________
​  ___ ________       
___
​ 7  + 2 ​÷10 ​ ​
   
__
   + ÷
 + (​÷5 ​
__  ​  ​
   )
​ 2 ​ ) (  – 1
= ​ __________________
​  2/3    1/3  
(8) + 2 (8) + 82/3
 ​  ​
)
(     + ÷
   ((​÷5 ​
(​÷10 ​  
1
= ​ _________________________
​  ___ __       
__
​ 2 ​
__
   + ÷
   ) + (​÷5 ​
__  ​  ​
   )
​ 2 ​ ) ​ 
–1
= _________
4 + 4 + 4 12
–1
= ___
   ​  ​   ​ 

(  (  )
_____ ______

   (÷
2​÷10 ​ 
1
= ​ _____________
​  ___ __   
   + ÷
​ 5 ​
__  ​  ​
   )
​ 2 ​ ) x Æ 1
÷​ x  2 + 8 ​  – ​÷10  
  ​  ​  _________________
15. ​ lim  ​  _____
 _____
– x2 
  
 ​
   ​  ​
÷​ x  + 3 ​ – ÷​ 5  – x  
2 2
 ​

(  )
( 
1 ___
= ​ ____________

)
​  ___      ​  ​ _____ _____
   + ÷
2 (​÷50 ​    )
​ 20 ​ (x2 + 8 – 10 + x2) (​÷x  2 + 3 ​  + ÷​ 5  – x ​2 
lim ​ ​ ​ ________________________________
= ​       
     _____ ______  ​  ​
x Æ 1
(x2 + 3 – 5 + x2) (​÷x  2 + 8 ​ + ÷   – x2 
​ 10  ​)

( 
______ ___

)
( 
a  + 2x     ​ 
​ – ​÷3x 
​÷_____________

)
13. ​ lim 
  ​  ​​  ______       _____ _____
__ ​  ​
x Æ a ​ 3a
÷  + x   ​ – 2​÷x 
  ​  (2x2 – 2) (​÷x  2 + 3 ​ ​ 5  – x2 
 + ÷  ​
lim ​ ​ ​  _________________________
= ​      
   
_____ ______  ​  ​

 ( (2x – 2)(​÷x  + 8 ​  + ÷   – x2 


______ x Æ 1 2 2

)
__ ​ 10  ​
  + x  
(a + 2x – 3x) (​÷3a ​ + 2​÷x 
  ​) 
lim ​ ​ ​ ___________________________

( 
= ​      
    ______ ___  ​  ​

)
x Æ a (3a + x  –  4x)  ×  (​ a _____ _____
÷  + 2x   ​+ ÷   ​) 
​ 3x 
(​÷x  2 + 3 ​
 + ÷​ 5  – x2 
 ​)
lim ​ ​ __________________
= ​   ​  _____   
    ​  ​
______

 (
______

) (​÷x  + 8 ​ + ÷   – x 
__ x Æ 1 2
​ 10 2
 ​)
  + x  
(a – x) (​÷3a ​ + 2 ​÷x 
   ​)
= ​   ​  ________________________
lim ​ ​     
   
______ ___  ​  ​
x Æ a 3 (a – x) × (​ a   ​) 
÷  + 2x  
​+ ÷ ​ 3x  2+2 4 2
= ______
​   ​ = __
​   ​  = __
​   ​ 

( 
______ 3+3 6 3

)
__
  + x  
(​÷3a ​ + 2​÷x 
________________   ​)  __ __
= ​  
lim ​  ​​ 
x Æ a 3 (​ a
______   
÷  + 2x  
  
​+ ÷
___  ​  ​
  ​) 
​ 3x  ( 
   ​ – a​÷a ​
x​÷x 
  ​  ​ __________
16. ​ lim 
x Æ a
​  x – a  
   
    ​  ​ )
( 
__ __
1 2​___
= __
3 (​÷3a 
   ​ + 2​÷a 
÷a 
​   ​ ​  ​ ___________
  
  
  ​ 
___ ​  ​
  ​ + ​÷3a 
  ​  ) ( 
x2/3 – a3/2
lim ​  ​ ​ _________
= ​  
x Æ a x – a    
​  ​ )
1 4__
= __
​    ​ ​ ____
3 2​÷3 ​ (  )
​     ​  ​ =
  
2
____
​  __
   ​ 
  
3​÷3 ​
3 __​ 3 ​  – 1
= __
​   ​  (a​)​2 ​
2
__
3​÷a 
   ​
​ 7  – x  
÷
3
​– 2
_____
= ____
​   ​  
   ​ ​ __________
14. ​ lim    
 ​  2
x Æ –1 (x + 1)

(  )
5 5
​ __ ​  ​ __ ​ 

(  )
(x + 2​)​3 ​ – (a + 2​)​3 ​
(7 – x)1/3 – (8)1/3   ​  ​ _________________
17. ​ lim  ​  x – a        ​  ​
   ​ ​ ​ ______________
= ​ lim    
 ​ 
   ​ x Æ a
x Æ – 1 (x + 1)

(  (  )
5
__ 5
__
​    ​ ​   ​ 
(7 – x)1/3 – (8)1/3 (x + 2​)​3 ​ – (a + 2​)​3 ​
   ​ ​ ​ ______________
= ​ lim    
 ​   = ​    lim ​ ​ _________________
​    
    ​  ​
x Æ – 1 (x + 1) x + 2 Æ a + 2 (x + 2) – (a + 2)


(7 – x)2/3 + 2 (7 – x)1/3 + 82/3
 ​   ×  ​ ​_________________________
2/3
   
    1/3
​ (7 – x) + 2 (7 – x) + 8 )
2/3
 ​  ​
) 5
= __
5
__
​   ​  – 1
​   ​  (a + 2​)​3 ​
3

( 
2
__
5
= __
​   ​ 
(7 – x – 8) ​   ​  (a + 2​)​3 ​
   ​ ​ ​ __________
= ​ lim     ​ ​  3
x Æ – 1 (x + 1) ​
1
 ​ × _________________________
​          ​  ​
) ( 
xm – a m
  ​  ​ ​ _______
18. ​ lim 
x Æ a xn – an
 ​  ​ )
(  )
2/3 1/3 2/3
​ (7 – x) + 2 (7  –  x) + 8

( 
xm – a m
– (1 + x) _________________________
   ​ ​ ________
= ​ lim  ​ 
x Æ –1 (x + 1)
 ​ 

× ​    
1
     ​  ​
(7 – x)2/3 + 2 (7 – x)1/3 + 82/3 ) = ​  
x Æ a x – a
​  ______
x – a  
lim ​  ​ ​ _______
n
______
n
​  x – a  



 ​  ​

= ​ lim  ​    
( 
–1
   ​ ​ ________________________
     ​  ​
x Æ –1 (7 – x) 2/3 + 2 (7 – x)1/3 + 82/3 ) mam – 1
= ​ ______
na n – 1
 ​ 
m
= ​ __ (  )
​ n ​   ​ am – n
The Limit  3.25

( 
(1 – x) n – 1
) (  )
2 1/3
{(1 + x ) – 1} _____x2
  ​ ​ ​ ___________
19. ​ lim  x     ​  
​ lim ​ ​ ______________
= ​   ​   ​
    × ​     ​  ​
x Æ 0 x Æ 0 x2 x + x2

(  ) ( 
n

)
(1 – x) – 1
lim ​ ​ ​ __________
= ​       ​  ​ {(1 – 2x) – 1} _____ –2 x
1/4
1 – x Æ 1 (1 – x) – 1 lim ​ ​ ______________
  – ​   ​        
​ × ​    
 ​  ​
x Æ 0 – 2x x + x2
= n (1) n – 1 = n
1 1 1
= __
​   ​  × 0 + __
​   ​  × 2 = __
( 
​   ​ 
(x + x2 + x3 + ... + xn) – n
  ​ ​ ​ _______________________
20. ​ lim       ​     ​ ) 3 4 2

(  )
x Æ 1 x–1 3
_____
2
_____
÷​ 7  + x3   ​  – ÷​ 3  + x2   ​
= ​  
x Æ 1
( 
(x – 1) + (x 2
– 1) +
______________________________________
lim ​ ​ ​       
x–1
(x 3
– 1)
 ​ 
+ ... 
    ​
+ (x n

) 1)   ​ ​ ​ ________________
23. ​ lim 
x Æ 1 x – 1
 ​       ​

= ​   ( 
(x
lim ​ ​ ______
​ 
x Æ 1 (x – 1)
– 1) (x 2

 ​ + ​ _______ ​ 

(x – 1)
1) (x
+ _______
​ 
3

(x – 1)
1) (x n

) 1)
+ ... + ​ _______ ​  
 ​ 
x – 1)

= ​ 
 l im 
x Æ 1
(   
​ ​
(7
​ 
+ x3)1/3 – (3 + x2)1/2
____________________
x–1
    )
 ​     ​

= (1 + 2 + 3 + 4 + ... + n)
n (n + 1)
= ​  
x Æ 1
(  {(7 + x3)1/3 – 2} – {(3 + x2)1/2 – 2}
lim ​ ​ _______________________________
​      
x–1
 ​     ​ )
= ________
( 
​   ​   
2 {7 + x3)1/3 – (8)1/3}
)
( 
= ​ 
 l im   
​ ​ ​ _________________  ​ 
      ​
21. ​ lim 
  ​  ​  ​ 
xn + 1 – (n + 1) x + n
__________________      ​ 
)
   ​
x Æ 1 x–1
(x  –  1)2
(  )
x Æ 1
{(3 + x2)1/2 – (4)1/2}
lim ​ ​ __________________

( 
  – ​   ​   ​ 
      ​
= ​  lim ​ ​ ​ 
x n + 1 – x – nx + n
_________________
)
 ​ 
      ​
x Æ 1 x–1

(  )
x Æ 1 (x – 1)2
{(7 + x3 – 8)}
lim ​ ​ ​  __________________________________

( 
= ​             ​  ​

)
x Æ 1 (x – 1){(7 + x) 2/3 + 82/3 + 81/3 (7 + x)1/3}
x (xn – 1) – n (x – 1)
__________________
= ​  lim ​ ​ ​   ​ 
      ​

( 
x Æ 1 (x – 1)2

= ​  
( 
2
______________________________________
lim ​ ​ ​       
3
x (x  –  1) (1  +  x  +  x   +  x   +  ...  +  x
 ​ 
n–1
    ​
)
) –  n (x  –  1)
  – ​  
x Æ 1
(x –
{(3 + x2 – 4)}
lim ​ ​ ​  __________________
÷
1) (​
_____
3  +)  
x
   ​  ​
2
   ​ + 4)

( 
x Æ 1 (x – 1)2

= ​   ( 
lim ​ ​ ​ 
x (1 + x + x2 + x3 + ... + xn – 1) – n
_____________________________
      ​  )
   ​
= ​ 
 l im   
​ ​
​ 
x Æ 1 {(7 + x) 2/3 + 82/3 + 81/3 (7 + x)1/3}
(x2 + x + 1)
_____________________________
        ​  ​
)
( 
x Æ 1 (x – 1)

= ​   ( 
(x + x 2
+ x 3
lim ​ ​ ​ _______________________
+
   
.... +
 ​ 
x n
)
)
   ​
– n
  – ​   lim ​ ​ ​ ____________
x Æ 1
(​÷ 3
(x + 1)
_____   
  + )x 2
   ​ +
  ​  ​
2)
x Æ 1 (x – 1)
3 1
= _____ ​  2/3    ​ + __ ​   ​ 
n (n + 1) 3.8 2
= ________
​   ​   
2
1 1
= __ ​   ​  + __

( 
_____ ​   ​ 

)
3 4 ______ 4 2
÷​ 1  + x  
  ​ ​ ​ ________________
22. ​ lim 
2
 ​ – ÷ ​ 1  – 2x  
 ​ 
      ​

x + x2 3
= __
x Æ 0
​   ​ 
4

(  )
2 1/3 1/4
(1 + x ) – (1 – 2x)
= ​   lim ​ ​ ​ ____________________
x Æ 0
   
x + x2
 ​     ​
24. ​ lim ( 
x Æ 1
3 __
​ x 
÷    ​ + ÷
  ​ ​ ​ ________________
4 __
​ x    ​ + ÷
x–1 )5 __
​ x 
 ​ 
   ​ – 3
      ​

(  ) (  )
__ __ __
{(1 + x2)1/3 – 1} – {(1 – 2x)1/4 – 1} (3​÷x  4
   ​ – 1) + ( ​÷x 
5
   ​ – 1) + ( ​÷x 
   ​ – 1)
lim ​ ​ ​ ______________________________
= ​         ​     ​ lim ​ ​ ___________________________
= ​   ​        ​     ​
x Æ 0 x + x2
x Æ 1 x–1

(  ) (  )
1 1 __ 1 20 + 15 + 12 ___ 47
{(1 + x2)1/3 – 1} {(1 – 2x)1/4 – 1} = __
​   ​  + __
​    ​ + ​   ​  = ​ ____________
 ​
     = ​    ​
lim ​ ​ ______________
= ​   ​    
 ​ 
   ​ – ​ 
 l im  
​ ​_______________
​    
 ​ 
   ​ 3 4 5 60 60
x Æ 0 x + x2 x Æ 0 x + x2
3.26  Differential Calculus Booster
_______

((    )
100
​ ​S  ​ ​ ​ x   ​ – 100
  ​  ​  ​ ____________
25. ​ lim 
k = 1
k

  
 ​  ​ ) y Æ •
( 
lim ​ ​ ​÷y  2 – 8y ​ 
= ​   + y  ​,

where x = – y, x Æ – •, y  Æ •


)
x–1

( 
x Æ 1

= ​  
x Æ 1
( (x + x2 + x3 + ... + x100) – 100
lim ​ ​ ​ __________________________
   
x–1
 ​     ​ ) = ​  
y Æ •
y2 – 8y – y2
​ ​  ___________
lim ​     
__________    ​  ​
​÷y  2 – 8y   
+ y ​ )
( (x – 1) + (x2 – 1) + (x3 – 1) + ...  +  (x100 – 1)
)
( 
lim ​ ​ ______________________________________
= ​   ​         ​      ​
x Æ 1 (x – 1) – 8y
lim ​ ​ ___________
= ​   ​  ______       ​  ​
)
(  ÷​ y  2 – 8y  
y Æ •

)
​+ y

(÷   
2 3 100
(x – 1) (x – 1) (x – 1) (x   –  1)
lim ​ ​ ​ ______ 
= ​    ​  + ​ ______ ​ 
+ ​ ______ ​ 
+ ... + ​ ________ ​  
 ​

)
x Æ 1 (x – 1) (x – 1) (x – 1) (x – 1) – 8
lim ​ ​ _________
= ​   ​  _____     ​  ​ = – 4
y Æ • 8
= (1 + 2 + 3 + .... + 100) ​ 1 – __
​ y ​ ​  
+  1

( 
100 × 101
= ​ _________
)
 ​  = 5050 2x + 5
_____
x2 + 2x – 1 ​ 2x – 7 
 ​
2   ​ ​​ ​ __________
31. ​ lim      ​  ​​ ​
x Æ •
2x2 – 3x – 2
1
__
  ​  
26. ​ lim (4 + n
5n​)​ ​n ​​
n Æ • 1
= __
2x + 5
lim ​ ​ ______
​   ​ , where ​   ​   ​   ( 
​= 1 )
(  (  (  ) ) )
2 x Æ • 2x – 7
4 n __​ 1 ​
lim ​ ​​ 5  ​ 1 +
= ​   n
​​ __
​    ​  ​​ ​  ​  ​​n ​

(  )
n Æ • 5
x2 + 2x – 1 1
lim ​ ​ ​ ___________
and ​     
    ​  ​ = __
​   ​ 
= 5. x Æ •
2x2 – 3x – 2 2

27. ​ lim  (  n2
  ​ ​ ​ _________________
      ​  ​
n Æ • 1 + 2 + 3 + ... + n ) ( 
(x + 1)10 + (x + 2)10 +  ...  + (x + 2012)10
  ​ ​ ​ _________________________________
32. ​ lim 
x Æ •
    
x10 + 201210
   
 ​  ​ )
= ​  
2n2
lim ​ ​ ​ _______
n Æ • n (n + 1) ( 
   ​  ​ ) (  (1 + 1/x)10 + (1 + 2/x)10 +  ...  + (1 + 2012/x)10
lim ​ ​ ______________________________________
= ​  
x Æ •
​       
1 + (2012/x)10
   
 ​  ​
)
​ 
2
lim ​ ​ ________
= ​  
n Æ • (1 + 1/n)
    ( 
 ​  ​ ) = 2012

33. Let tr = (n – r + 1) ◊ r


​ 
2
lim ​ ​ ________
= ​  
n Æ • (1 + 1/n)
    ( 
 ​  ​ = 2 ) = (nr – (r – 1) r)
= ((n – 1) (r – r 2)

(  )
n n
x +y n
  ​ ​ ​ _______
28. ​ lim   ​  ​, where 0 < x < y.
  Now, ​   ​ ​ ​ tr
Sn = S
n Æ • xn – yn

(  ) ( 
 r – 1
n
x n
​​ __ (  )
​ y ​  ​​ ​+ 1 ​   ​ ​ ​ ((n – 1) r – r 2)
=S
lim ​ ​ ​ _______
= ​  
n Æ • __ x n  
(  )
​​ ​ y ​  ​​ ​ –  1
0+1
 ​  ​ = ​ ​ _____ 
0 –1
 ​  ​ = –1 ) r – 1
n n
​   ​ ​ ​ (n – 1) r – ​S  ​ ​ ​ r 2
=S
 r – 1 r = 1
______
​ – x )​
( ​÷x  + 2x  
(  )
2
  ​ ​ 
29. ​ lim  n (n   + 1) n (n  +  1) (2n  +  1)
x Æ • = (n  –  1) ​________
​   ​   ​ – ______________
​   ​     

( 
2 6
= ​  
x Æ •
x2______
+ 2x – x2
lim ​ ​ ​ ___________     ​  ​
÷​ x  2 + 2x  
​+ x ) ( 
n (n + 1)
= ​ ________
​   ​  
2n + 1
 ​ ​ n – 1 – _______
​   ​  ) ( 
 ​ )
( ÷ 
2 3

x Æ •
2
lim ​ ​ ___________
= ​  
2
​ 1 + __
​ x ​ 
​  + 1
2
​  ______    ​  ​ = __
​    ​ = 1
2
) ( 
n (n + 1) (n – 2)
= ​ ​ ______________
6
 ​ 
     ​ )
  ​ ​ 
30. ​ lim 
x Æ •
_______
+ x )​
( ​÷x  2 + 8x ​  lim ​ ​ ______________
Therefore, ​  
n Æ •
​ 
6n3
 ​ ( 
n (n + 1) (n – 2)
     ​ )
The Limit  3.27

= ​  
n Æ •
( 
1
​ 1 + __ ( 
______________
lim ​ ​ ​ 
6
 ​ 
2
​ n ​  ​ ​ 1 – __
    
) ( 
​ n ​  ​

) ) fi ​ lim 
x Æ •
   
x+1
 ​  ( 
x2 + 1 – ax2 – ax – bx – b
  ​ ​ ​ ______________________
   ​ = 0 )

1
= __
​   ​ 
6
x Æ •
   
x+1 ( 
(1 – a) x2 – (a + b) x + (1 – b)
  ​ ​ ​ __________________________
fi ​ lim   ​     ​ = 0 )
1 It is possible only when, 1 – a = 0 and (a + b) = 0
34. Put ​ __
n ​ = x
Thus, a = 1 and b = –1

{  }
________________________________
_______________________
_____________
​÷  ÷      
( 
1 – cos x) ​ (1          
– cos x) ​÷(1 – cos x) ... • ​ ​ ​
)
  
  ​  ​  _________________________________
​ lim  ​   ​     ​
x2 + x + 1
  ​ ​ ​ _________
37. Given ​ lim   ​ 
 – ax – b  ​ = 4
x Æ 0 x2 x Æ 0 x+1

(  ) ( 
1 1 1

)
​ __ ​  + __
​    ​ + __
​    ​ + ... •
(1 – cos​)​2 22 23 ​ x2 + x + 1 – ax2 – ax – bx – b
lim ​ ​ ​ ___________________
= ​        ​    ​   ​ ​ ​ __________________________
fi ​ lim       ​     ​ = 4
x Æ 0 2 x Æ 0 x+1
x

( 
(1 – cos x)
lim ​ ​ ​ _________
= ​  
x Æ 0 x2
 ​  

​ ) fi
x Æ 0
( (1 – a) x2 + (1 – a – b) x + (1 – b)
lim ​​ _____________________________
​     ​      
x+1
 ​     ​ = 4 )
It is possible only when

(  )
2
2 sin  (x/2) fi (1 – a) = 0, (1 – a – b) = 4
lim ​ ​ _________
= ​   ​   ​ 
    ​
x Æ 0 x2 fi a = 1, a + b = – 3

( 
fi a = 1 and b = – 4
2 sin2 (x/2)
lim ​ ​ _________
) 38.
We have

( 
= ​   ​  2     ​  

(  )
)
x Æ 0 (x /4)/4 1
x4 sin ​ ​ __
x ​  ​ + x
2

1 ​    lim ​  ​ ​  ____________   


 ​   ​
= __
​   ​  x Æ – •
1 + |x|3
2
x2 – 1
x Æ • x + 1 ( 
lim ​ ​ ​ _____ 
35. Given ​    ​ – ax – b  ​ = 2 ) lim ​ ​
= ​     ​  ( 
1
x4 sin ​ __
____________
(  )
​ x ​  ​ +  x2
  
 ​   
​ )
( 
x Æ – •
1 – x3

(  )
)
x2 – 1 – ax2 – ax – bx – b
lim ​ ​ ​ ______________________
fi ​  
x Æ •
   
x+1
 ​     ​ = 2
= ​   
1
x sin ​ __ (  )
1
​ x ​  ​ + __
lim ​  ​ ​ ___________   
​ x ​
 ​   ​
x Æ – • 1
( 
__
)
​  3  ​ – 1
(1 – a) x2 – (a + b) x – (1 + b)
lim ​  ​ ​ __________________________
fi ​        ​     ​ = 2 x
x+1
( 
x Æ •

It is possible only when 1 – a = 0 fi a = 1



1+0
= ​ ​ _____ 
0–1
 ​  ​ )
= –1
Also, ​  
x Æ •
​  ( 
– (a + b) x – (1 + b)
lim ​ ​ _________________
x+1
 ​ 
     ​ = 2 ) 39. Limit = ​   (  )
sin 3x
lim ​ ​ _____
​      ​  ​

( 
x Æ 0 5x

) lim ​ ​( _____ ​ × ​   ​  )​


(1 + b) sin 3x ___ 3x
– (a + b)  –  ​ ______
x   ​  = ​   ​     
lim ​ ​ ________________
fi ​   ​   ​ 
  
   ​ = 2 x Æ 0 3x 5x
x Æ • 1
1 + __​ x ​ 3
= __
​   ​ 
5
fi – (a + b) = 2
sin ax
(  )
lim ​ ​ ​ _____  
40. Limit = ​   ​  ​

(  )
fi (a + b) = – 2 x Æ 0 sin bx

sin ax
fi b = – a – 2 = – 1 – 2 = – 3 ​ _____
ax    
​ × ax
lim ​ ​ ​  _________
= ​        ​  ​

( 
sin bx
)
2 x Æ 0 _____
​      ​ × bx
x +1
  ​ ​ ______
36. Given ​ lim  ​   ​ 
– ax – b  ​ = 0 bx
x Æ • x + 1
a
= __
​   ​
b
3.28  Differential Calculus Booster

41. Limit = ​  


lim   ( 
sin 2x + 3x
​ ​ ​ _________ 
4x + sin 6x
 ​  ​ ) Ï
Ô lim Á
Ê 2 | sin x | ˆ

(  )
x Æ 0
x ˜
Ô x Æ0+ Ë ¯
sin 2x =Ì
​ _____
x    ​ + 3 Ê 2 | sin x | ˆ
lim ​ ​ ​ ________ 
= ​    ​  ​ Ô
sin 6x Ô lim– Á ˜
4 + ​ _____
x Æ 0 x
x ​ 
    Ó x Æ0 Ë ¯

( 

)
sin 2x Ï
​ _____
   
​× 2 + 3 Ê 2(sin x ) ˆ
2x Ô lim Á ˜
lim ​ ​ ​  ____________
= ​       ​  ​ Ô x Æ0+ Ë x ¯
sin 6x
4 + ​ _____
x Æ 0
   
​× 6 =Ì
6x Ô Ê 2(sin x ) ˆ
Ô lim– Á
( 
˜

2+3
= ​ ​ _____ 
4+6
 ​  ​ ) Ó x Æ0 Ë
ÏÔ 2
x ¯


1
= __
​   ​  ÔÓ – 2
2

( 
sin 2x + sin 5x
lim ​ ​​ ____________
42. Limit = ​      ​   ​
x Æ 0 sin 4x + sin 6x ) Since R.H.L π L.H.L, so limit does not exist.

( 
( 
cos 7x – cos 9x
lim ​ ​ ​ ____________ )
)
45. Limit = ​       ​  ​
sin 2x _____ sin 5x  Æ 0 cos 3x – cos 5x
​ _____  
​ + ​  x   
x    ​ 
= ​  
lim ​ ​ ​  ____________     ​  ​
lim ​ ​( ​ __________
2 sin 4x sin x )
x Æ 0 _____ sin 4x _____sin 6x 2 sin 8x sin x
​  x    
​ + ​  x    ​  = ​      ​  ​

( 
x Æ 0

)
sin 2x sin 5x
​ _____ ​ × 2  + _____
lim ​ ​( ​ _____  
  ​      ​ × 5
sin 4x )
   sin 8x
2x
____________________ 5x = ​   ​  ​
= ​  
lim ​ ​ ​     
     ​  ​

(  )
x Æ 0
sin 4x
_____ sin 6x
​  × 4 + ​ _____
x Æ 0
​          
​× 6
4x 6x
sin 8x
​ _____
   
​ × 8x
(  )
2+5
= ​ ​ _____ 
4+6
 ​  ​
x Æ 0 _____
8x
lim ​ ​ ​ _________
= ​  
sin 4x
​     
    ​  ​
​ × 4x
4x
7
= ​ ___  ​  =2
10

(  ( 
______
43. Limit = ​  
1 – cos 4x
lim ​ ​ ​ ________ 
x Æ 0 1 – cos 6x
 ​  ​ ) 46. Limit = ​  
x Æ 0 x 2

1 – cos x ​÷cos 2x 
lim ​ ​​ ______________
 ​ 
    


​ )
(  ) (  )
______ ______
2 sin2 2x (​  1 – cos x ​÷cos 2x 
  ​ )​ ​( 1 + cos x​÷cos 2x 
  ​ )​
= ​   ​ ​ ​ _______
lim     ​  ​

lim ​ ​ _______________________________
= ​   ​          
______  ​  ​

(  )
x Æ 0 2 sin2 3x 2
x Æ 0  
x (1 + cos x ​÷cos 2x  ​)

(  )
sin2 2x
​ ______  × 4x2
 ​  (1 – cos2 x (cos 2x))
4x 2 lim ​ ​​ __________________
= ​           ​  ​
______
lim ​ ​ ​ ___________
= ​       ​  ​ x Æ 0 x 2 ​( 1 +cos x ​÷ cos 2x 
  ​ )​

x Æ 0 sin2 3x
​ ______ 9x2

( 
 ​

  ×

)
2
9x (1 – cos2 x (2 cos2 x – 1))
4 lim ​ ​ ​  ___________________
= ​      
    ______  ​  ​
= __
​   ​  x Æ 0 x2 ​( 1 + cos x ​÷ cos 2x 
  ​ )​

9

(  (  )
_________
44. Limit = ​  
lim 
xÆ 0

​ 1  – cos 2x  
÷
​ ​ ​ __________
x2
 ​ 

     ​ ) = ​  
(1 – 2 cos4x + cos2 x)
lim ​ ​ ​ ___________________
        ​  ​
______
x Æ 0 x 2 ​( 1 + cos x ​÷cos 2x 
  ​ )​

( 
______

= ​  
x Æ 0
( 
    2x  
÷​ 2 sin
  ​ _______
lim ​ ​ x  

​  ​ ) = ​  
x Æ 0
(2 cos4x – cos2 x – 1)
lim ​ ​ – ​  _________________
  
    ​ ​
______
x2 (1 + cos x ​÷cos 2x 
   ​ )
( 
______
= ​  
x Æ 0
÷
( 

​ 2 |sin x| ​
lim ​ ​ ​ _______
x   


​  ​)
sin2 x(2 cos2 x + 1)
lim ​ ​​  ___________________
= ​         
)
______  ​  ​
x Æ 0 x 2 ​( 1 + cos x ​÷cos 2x 
  ​ )​

The Limit  3.29

= ​  
x Æ 0 ( 
sin2 x
lim ​ ​ ​ _____
x2
 ​

  × ​ 
(2 cos2 x + 1)
________________       ​  ​
______
(​  1 + cos x ​÷cos 2x 
  )​
​   ) 49. Limit = ​  
x Æ 0
sin– 1x – tan– 1x
lim ​ _____________
​ 
x3
 ​     


2
= __
​   ​ 
3 = ​   ( 
x
tan– 1 ​ ​ ______
( 
   
_____
÷​ 1  – x   2
lim ​ ​ ​  ____________________
   
 ​
 ​ 
)
 ​  ​  – tan– 1x
   ​ )
( 
( 
3

)
x Æ 0

(  ))
tan x – sin x x
lim ​ ​ ​  __________
47. Limit = ​    ​ 
    
​ x
x Æ 0 x3 ______
​  _____
     ​ – x
​ 1  – x2 
÷
( 
 ​
)
– 1 ____________
sin x(1 – cos x) tan  ​ ​         ​ ​
lim ​ ​ ​ ____________
= ​     
    ​  ​ 1 + x ◊ ​ ______
_____
x
     ​
x3cos x

( 
x Æ 0
÷​ 1  – x2   ​
= ​   lim ​ ​ ​  __________________
    ​     ​
(  )
)
x Æ 0 3
x x
2 sin2 ​ __
​    ​  ​

(  (  ))
sin x __________
____ 2 _____
= ​  
lim ​ ​ ​  x   
​ × ​  2      ​    ​
x Æ 0 x x  – x​÷1  – x2   ​
__
​   ​  × 4 cos x tan  ​ ____________
– 1
​  _____       ​  ​
4 ÷​ 1  – x  
2
 ​  + x 2

 ( ( 
lim ​  ​  ​ __________________
= ​   3
 ​        ​
1 x Æ 0 x

)
= ​ __ ​ 

( 

)  
2 _____
x    – x ​÷1  – x2   ​
tan– 1 ​ ____________
​  _____       ​ ​

( )
_____
48. We have cos x cos 2x cos 3x ÷​ 1  – x   2
 ​+ x 2
x  – x​÷1  – x2   ​
= ​   __________________
lim ​ ​ ​      ____________
______   ​× ​ ​  _____        ​ ​  ​

1
= __
​   ​  (2 cos 3x cos x) cos 2x
2
x Æ 0
x  – x​÷1  –  x  
​ ​ ___________
_____     ​  ​ x 3
÷​ 1  – x   2
 ​ + x
2
 ​
2 )
​÷1  – x  2
 ​ + x 2

(  ( 
1
= __
​   ​  (cos 4x + cos 2x) cos 2x

)
_____
2
x (​  1 – ​÷1  – x2 
 ​ )​
1 lim ​ ​​  ______________
= ​   _____       ​  ​
= ​ __ ​  (2 cos 4x cos 2x + 2 cos2 2x)
4
x Æ 0
x3​ ​÷1  – x2 
 ​ + x2  ​ )
(  )
_____
(1 –  1 + x2) ​( ​÷1  – x2   ​ – x2  )​
1
= __
​   ​  (cos 6x + cos 2x + 1 + cos 4x)
4 lim ​ ​ ​ ________________________
= ​   _____   
     ​  ​
x2 ​( 1  + ÷  ​ )​ (1 – x2 – x2)
​ 1  – x2 
x Æ 0

(  ( 
1
= __

)
​    ​ (1 + cos 2x + cos 4x + cos 6x) _____
(x2) ​( ​÷1  – x2 
 ​ – x2  )​
4
lim ​ ​ ​ ______________________
= ​      
   
_____  ​  ​
Thus x Æ 0
 ​ )​ (1 – 2x2)
( 
​ 1  – x2 
x2​ 1  + ÷
1 –  cos x  cos 2x  cos 3x
lim ​ ​ ​ ___________________
)
( 
Limit = ​            ​  ​

)
_____
x Æ 0 sin2 2x (​  ​÷1  – x2 
 ​ – x2  )​
lim ​ ​ ​ ____________________

(  )
= ​      
_____     ​  ​
1 (1  + ​÷1  – x2 
1 – __
x Æ 0
​   ​ (1 + cos 2x + cos 4x + cos 6x  ​) (1 – 2x2)
4
= ​   ​ ​ ​  ___________________________
lim             ​  ​ 1
 xÆ 0 sin2 2x = __
​   ​ 
2
= ​  
x Æ 0 ( 
3 – (cos 2x + cos 4x +  cos 6x)
lim ​ ​ ​ _________________________
   
4 sin2 2x
     ​  ​ ) 50. Limit = ​  
x Æ 0 ( 
x  – sin x
lim ​ ​ ________
 ​ 
x3
 ​   

)
(  )
 ( (  )
(1 – cos 6x)  + (1 – cos 4x) + (1 – cos 2x)
lim ​ ​ ​  __________________________________
= ​  
x Æ 0
    
4 sin2 2x
      ​  ​ x 3 x5 x 7
)
x – ​ x – ​ __  ​ + ​ __  ​ – ​ ___   ​+ ...  ​
3! 5! 7!
im   ​ ​ ​  _______________________

( 
= ​l       ​     ​
2 sin2 3x +  2 sin2 2x + 2 sin2 x
lim ​ ​ ​ ________________________
= ​            ​  ​ ) x Æ 0 x3

( ( 
4 sin2 2x

)
x Æ 0

(  2
lim ​ ​ ​  ____________________
= ​      
2
  sin  3x + sin  2x + sin  x
     ​  ​
2

) = ​  
x 3 x5 x 7
3! 5!
​ ​ ​  ________________
lim  
7
 ​      
)
​ ​ __  ​ – ​ __  ​ + ​ __ ​  – ...  ​

x Æ 0 2 sin2 2x x Æ 0 x 3

1
9 +  4 + 1 ___
= ​ _________
 ​  
14 7
= ​   ​ = __
​    ​ = __
​   ​ 
2×4 8 4 6
3.30  Differential Calculus Booster

51. Limit = ​  


x Æ 0 ( 
x – tanx
lim ​ ​ ​  _______
x3
 ​  
 ​
) = __
2 ( 
1 4n + 1 – 1
​  n   ​ ​ ​ _______
3
 ​  
 ​ )
( 
(  (  ) )
4n + 1 – 1
x3 2
x  – ​ x + ​ __ ​  + ___
3
​    ​x 5 + ...  ​
15
lim ​ ​ ​ _____________________
) = ​ ​ _______
3.2n
   
​  

( 
= ​       ​     ​
x Æ 0 x3 55. Given ​ lim 
a cos x  + bx sin x – 5
  ​ ​ ​ _________________
 ​       ​
)
 ( (  )
x Æ 0 x4


x3
3
lim ​ ​ _________________
= ​   ​ 
15
2
–  ​ ​ __ ​  + ___
​    ​ x5 + ...  ​
 ​ 
      ​
) 0
As x Æ 0, x4 Æ 0, the limit must be in the form of ​ __
​   ​   ​ (  )
0
x Æ 0 x3
Thus, ​ lim 
  ​ (a cos x + bx sin x – 5) = 0
1
= – __
x Æ 0
​   ​ 
3 fi a=5=0
lim ​  
52. Limit = ​   ( 
x + sin x
​ ​ _______ 
x Æ • x + cos x

 ​  ​ ) fi a=5

(  )
Now, Limit

lim ​ ​
= ​    ​ 
sin x
1 + ​ ____
________x   
cos x 
​ 
  ​  ​

x Æ 0 ( 
a cos x  + bx sin x – 5
lim ​ ​ ​ _________________
= ​  
x4
 ​       ​
)
x Æ •
1 + ​ ____
x ​ 
   

( 
1+0
= ​ ​ _____ 
1+0
 ​  ​ )
= ​  
x Æ 0
​ 
( 
5 cos x  + bx sin x – 5
lim ​ ​ _________________
x4
 ​       ​
)
=1
53. As we know that, sin x £ x and tan x ≥ x ( 
x2 4
) ( x3
)
5​ 1  – ​ __  ​ +  ​x    ​ – ...  ​ + bx ​ x – ​ __  ​ + ...  ​  –  5
2! ​4! ​
lim ​ ​  ____________________________________
= ​         ​
3!
   
x Æ 0 4
sin x tan x x
Thus, ​ ____ x   ​ £ 1 and ​ ____ x   ​ ≥ 1
Therefore,

Limit = ​  
n sin x
lim ​ ​ ​ ​ _____ ( [ 
n tan x
​  ​ + ​ ​ ______ ​  ​  ​ ] [  ])
5
2(  ) ( 
​ b – __
24
5
​   ​   ​ x2  + ​ ___
lim ​ ​ _________________________
= ​        ​
b
​    ​ – __
6 )
​   ​   ​ x4 + ...
  
x Æ 0 x    x    x Æ 0 x 4

= (n – 1) + n
= 2n – 1. It will provide finite limit only when, co-efficient of
x2 is zero.
54. We have a = min {x2 + 2x + 3, x2 + 4x + 10}


a = min {(x + 1)2 + 2, (x + 2)2 + 6}
a=2
Thus, ​ b – __
5
(  )
​   ​   ​ = 0
2

(  )
5
Also, b = ​  
1 – cos q
lim ​ ​ ​ ________
   ​  ​ fi b = __
​   ​ 
2

 ( (  ) (  ) )
x Æ 0 q2
Also, the limit value
q
2 sin  ​ __ 2
​   ​   ​
lim ​ ​ ​ ________ 
= ​  
q Æ 0 q
___
2
2

​ ​   ​   ​ × 4
 ​  ​
= ​  
lim ​ ​ 
( 5
​ ___
24 6 )
b
​    ​ – ​ __  ​  ​ x4 +  ... 
_______________  ​     
4 x Æ 0 x4

(  )
1
__ 5 b
= ​   ​  = ​ ___
​    ​ – __​    ​  ​
2 24 6
n

= ​( ___
​   ​ ​ ​(arbn – r)
Therefore, S
24 12 )
5 5
r = 0 ​    ​ – ___
​    ​  ​

(  (  ) )
n
1 n – r
= ​S  ​ ​ ​ 2r​ ​ __
​   ​   ​​ ​  ​
5
= – ​ ___  
r = 0 2 24
n
1
​  n   ​ ​S  ​ ​ ​(4r )
= __  ​ Therefore, the value of a + 2b + 10
2 r = 0
= 5 + 5 + 10
1
= __
​  n   ​ (1 + 4 + 42 + ... + 4n)
2 = 20.
The Limit  3.31

x Æ 0 ( 
sin (p cos2 x)
lim ​ ​ ​ __________
56. Limit = ​  
x2
 ​ 
     ​ ) = ​ lim 
x Æ 0 
(  3
  ​​ ​ ________
x     
x
e (e – 1) ____
sin x
​ – ​  x   
​  ​ )
(  )
sin (p (1 – sin2 x) = (e3 – 1)
lim ​ ​ ______________
= ​   ​   ​ 
     ​
x2 ex – esin x
lim ​ ​ ________ 
x Æ 0
61. Limit = ​    ​
x Æ 0 x – sin x

= ​  
lim 
x Æ0 ( 
sin ((p – p sin2 x))
​ ​ ​  ______________
x2
 ​ 
     ​ ) = ​  
x Æ 0
( 
ex–sin x – 1
lim ​ esin x ​ ​ ________ ​  
x – sin x
​ )
= ​  
x Æ 0 ( 
sin (p sin2 x)
lim ​ ​ ​ __________
x2
 ​ 
     ​ )

= 0×1
=1

= ​  
x Æ 0 ( 
sin (p sin x) p sin
lim ​ ​ ​ __________
2
  
p sin  x
   
​ × ​ 
2
______
x 2
2
x
 ​   ​ ) 62. Limit = ​  
x Æ 0
​ ( 
ex  – ex cos x
lim ​ ​ _________
x + sin x
 ​
 ​   )
=p ( 
ex cos x (ex – x cos x – 1)
lim ​ ​ ​  _______________
= ​  
x + sin x
      ​  ​ )
( 
x Æ 0
e –1
lim ​ ​​ ______
4x
)
( 
57. Limit = ​     ​  
 ​
x Æ 0 5x
= ​  
ex cos x (ex – x cos x – 1)
lim ​ ​ ​  _______________   
x(1 – cos x)
 ​ × ​ __________
        
​ ​ )
( 
x Æ 0 (x – x cos  ) x + sin x
4x
)
( 
e –1
= ​   ​ ​ ​ ______
lim      
​  ​

)
x Æ 0 5x e x co x (ex – cos x  – 1) (1 – cos x)
lim ​ ​ ​ _______________
= ​       ​ × ​ _________ ​  
    ​
= ​  
x Æ 0 ( 
e4x – 1 ___
lim ​ ​ ​ ______
4x
   
4x
​ × ​    ​  ​
5x ) x Æ 0 (x –  cos x) sin x
1 + ​ ____
x   


=0

( 
4
= ​ __ ​ 

5 63. Limit = ​  
ex  – 1 – x
lim ​ ​ ​ _________
 ​  

​ )
(  )
x Æ 0 x2
e3x – 1

( 
lim ​  ​ _______
(  )
58. Limit = ​   ​   ​  
  ​
)
(  )
x Æ 0 e 5x – 1 x2 x3
​ 1+  x + ​ __  ​ + ​ __  ​ + ...  ​ – 1 – x
2! 3!
3x = ​   ​ ​ ​ __________________________
lim       ​     ​
e –1
​ ______   ​ × 3x x Æ 0 x2

 ( (  )
3x
lim ​ ​ ​  __________
= ​  
x Æ 0 e5x – 1
​ ______
5x
   
    ​  ​
​ × 5x
= ​  
x2 x3
2! 3!
lim ​ ​ ​  ____________  ​ 
)
​ ​ __  ​ + ​ __  ​ + ...  ​
     ​
3
x Æ 0 x2
= __
​   
5 1
= __
( 
​   ​ 

x Æ 0
ex  + e– x – 2
lim ​ ​ ​ __________
 ​59. Limit = ​  
x2
 ​ 
    ​ ) 2

8x  – 4x – 2x + 1
lim ​ ​  _____________

(  )
64. Limit = ​    ​
    
1 x2
ex + __
x Æ 0
​  x   ​ – 2
e

( 
lim ​ ​ ​  _________
)
= ​    ​    
​ 4x (2x – 1) – (2x – 1)
x Æ 0 x2 lim ​ ​ ​ _________________
= ​    ​ 
      ​

( 
x2
)
x Æ 0
x 2
(e – 1)

( 
lim ​ ​ ​ _______
= ​    ​  


x Æ 0 e x x2 = ​  
(4x – 1) (2x – 1)
lim ​ ​ ​  _____________
 ​ 
     ​ )
( ( 
x2
)
x Æ 0
)
x
– 1 2 __
e_____ 1
=​ lim   ​ ​ ​​ ​ 
x   
​  ​​ ​ × ​  x   ​  ​
( ( 
e
)(  ))
x Æ 0
4x – 1 2_____ x
–1
lim ​ ​ ​ ​ _____
= ​   x   

​  ​​ ​  x   
​  ​  ​
= 1. x Æ 0

60. Limit = ​  


x Æ 0
( 
e3 + x – e3 ____
lim ​ ​ ​ ________
x   
sin x
​ – ​  x   
​  ​ )


= log 4 × log 2

= 2(log 2)2
3.32  Differential Calculus Booster

65. Limit = ​  


x Æ 0
9x  – 2.6x + 4x
lim ​ ​ ____________
x2
 ​
     ​  x ( 
(5x  – 1) (4x – 1)
lim ​ ​ ______________
69. Limit = ​      ​  ​
x Æ 0 (3   – 1) (6x – 1) )
= ​  
x Æ 0 ( 
(3x)2 –  2.3x ◊ 2x + (2x)2
lim ​ ​ ​ ___________________
   
x2
 ​     ​ )
( 
5x  – 1 ______
​ ______
​  x   
lim ​ _______________
= ​   ​  x   x   
) ( 
4x  – 1
​  ​​ ​  x   
   ​

​  ​ )
(  ) (  )
( 
x Æ 0 3 – 1 6 –1
= ​  
(3x – 2x)2
lim ​ ​ ​ ________
 ​  

​ ) ​ ​ _____
x   ​  ​​ ​ _____
x   ​  ​

( 
x2
)
x Æ 0
log 5 ◊ log 4
= ​ _________
(  )
3x  – 2x 2 ​   ​  

lim ​ ​​ _______
= ​   ​  x    ​  ​​ ​ log 3 ◊ log 6

( 
x Æ 0

lim ​ ​​(​  ______
= ​  
x
3   – 1 ______
x    
2  – 1
​ – ​  x   ​ )​​​

x 2
70. Limit = ​  
x ◊ 2x  – x
lim ​ ​ ________
​ 
x Æ 0 1 – cos x
​   ​
  )

 ( )
x Æ 0

= (log 3 – log 2)2 = ​   ​ ( 


x(2x  – 1)
lim ​ ​ ________
x Æ 0 1  – cos x
 ​  

​ )

3
= log 2 ​ __(  )
​   ​   ​
2 ( 
2x – 1
x2​ ​ _____ x    ​  ​ )
lim ​ ​ ​  ____________
= ​          ​  ​
x Æ a
( 
ax  – aa
lim ​ ​ _______
66. Limit = ​   ​  x – a   

​ ​, a > 0 ) x Æ 0
2 sin2​ __
​ ________
2
 ​ 
x
(  )
​    ​  ​

x 2
× ​ __ ​ 
x 2 4
​ __ ​ 
= ​  
x Æ a
( 
aa (ax – a – 1)
lim ​ ​ ​  __________
x – a  
    ​  ​ ) 4
= 2 log 2
= (aa ◊ log a)
( 
e x  – e– x – 2x
)
( 
lim ​ ​ ​ ___________

 (( 
71. Limit = ​        ​  ​
ax + h  + ax – h – 2ax
lim ​ ​ ​ _______________ ) x Æ 0 x – sin x

)
67. Limit = ​    ​      ​
h2
) (  )
x Æ 0
x x2 x3 x x2 x3
​ 1 +  __
​    ​ + ​ __ ​ + ​ __ ​ + ...  ​ – ​ 1 – __
​    ​ + ​ __ ​ – ​ __ ​ + ...  ​– 2x
(  )
x h – h 1! 2! 3! 1! 2! 3!
a (a   + a – 2) lim ​ ​ ​  _________________________________________
lim ​ ​ _______________ = ​               ​  ​
( 
= ​   ​   ​ 
    ​
x Æ 0 h2
x Æ 0
x
__
3
x
__
5
x7
__
x – ​ x – ​   ​ + ​   ​ – ​   ​ + ...  ​ )
( 
3! 5! 7!

)
 (( 
ax (a2h  – 2ah + 1)
lim ​ ​_______________
= ​   ​    
 ​ 
   ​

) ( 
)
ah × h 2
)
x Æ 0
x3 x4 x3 x4
​ ​ __  ​  + ​ __  ​ + ...  ​ – ​ – ​ __  ​ + ​ __  ​ – ...  ​
= ​  
h Æ 0 a ( 
ax
( 
(a2h  – 2ah + 1)
lim ​ ​ ​ __h ​ × ​ ​ _____________
h2
 ​ 
     ​  ​ )) = ​  
x Æ 0
3! 4!
lim ​ ​ ​  ____________________________
(  x
    
3
x 5
3! 4!
   
x7
 ​  ​
– ​ – ​ __  ​ + ​ __  ​ – ​ __  ​ + ...  ​ )
( 
3! 5! 7!
(  ))
(  )
ax ah – 1 2
lim ​ ​ ​ __h ​ × ​​ ​ _____
= ​      

​  ​​ ​  ​
h Æ 0 a h __1 1
​    ​ + __ ​    ​ 
3! 3!
= a x log a = ​ ​ ______  ​    ​
__1
​    ​ 
( 
8x – 7x
lim ​ ​ ​ ______
68. Limit = ​  
x Æ 0 6x – 5x
  ​  ​ ) 3!

=2
= ​  
lim   ( 
(8x  – 1) – (7 x – 1)
​ ​ ​ _______________
   ​  ​
x Æ 0 (6x  – 1) – (5x – 1) ) lim ​ ​ ​ _________
72. Limit = ​   ( 
e​x​​  – 1 – x3
    
3

​  ​ )
 (((    (( 
x Æ 0 sin 62x

) (  )
)
x x

)
8 –1 7 –1
​ ​ _____ ​  ​  – ​ ​ _____
x    x   
​  ​ 3

lim ​ ​ ​ _________________ e​x​ ​  – 1 – x3


= ​       ​  ​ = ​   ​ ​ ​ ______________
lim         ​  ​
) (  ) )
x x
x Æ 0 6_____
–1 5_____
–1 x Æ 0 sin6 2x
​ ​  x    ​  ​  – ​ ​  x    ​  ​ ______
​ ​  6  ​  × (2x6)  ​

( 
(2x )
log 8  – log 7
= ​___________ )
(  ) ( 
​    
   ​  ​

)
log 6 – log 5 3
e​x​ ​ – 1 – x3 sin62x
= ​   ​ ​ ​ _________
lim   ​ ​ ​ ______
 ​  
   ​ 
= 1  ​

(  )
6
log (8/7) x Æ 0 (2x) (2x) 6
= ​ ​ _______   ​  ​
log (6/5)
The Limit  3.33

= ​  
lim 
t Æ 0 ( 
e t  – 1 – t
​ ​ ​ ________
26 ◊ t 2
 ​  

​ )
2
1
= ​ __n   ​  (1 + 4 + 42 + ... + 4n)

(put x3 = t, when x Æ 0, t Æ 0) = __
2 ( 
1 4n + 1  – 1
​  n   ​ ​ ________
​ 
3
 ​  
 ​ )
= ​  
lim  ​ ​ __
t Æ 0 2
1
(  (  et  – 1 – t
​  6  ​ × ​ _________
​ 
t2
 ​   
​  ​ )) ( 
4n + 1 – 1
= ​ ​ _______
3 ◊ 2n
   

​  ​ )
( 
1 1
= __
​  6  ​ × __
​   ​  Aex  – Bcos x + Ce– x
  ​ ​ ​ _________________
76. We have ​ lim    ​  ​
     )
 ( ( 
2 2 x Æ 0 x sin x

(  ) ) (  ) (  )
)
1
= ​ ____
​     ​  ​ x2 x3 x2 x2
128 A ​ 1 + x + ​ __  ​+ ​ __ ​ + ...  ​  – B​ 1 – ​ __ ​ + ...  ​ +C ​ 1 – x – ​ __ ​ + ...  ​
2! 3!
_________________________________________ 2! 2!
= ​  
lim ​ ​ ​         ​ 
    ​

( 
(cos a) x  + (sin a) x – 1
lim ​ ​ ​ __________________
73. Limit = ​        ​     ​ )
x Æ 0
( x3
__
x ​ x – ​   ​ + ...  ​ )

( 
x Æ 2 x–2 3!

(  (  )
)
A  + B + C 2
(cos a) x  + (sin a) x – (cos2 a + sin2 a)
lim ​ ​ ​ _______________________________
= ​         ​     ​ ) = ​  
(A –  B + C ) + (A – C) x + ​ ​  _________
lim ​ ​ ​ ________________________________________
     
2
   
 ​  
 ​  ​

​x + ...

( 
x–2
)
x Æ 2
x Æ 0
x3
x​ x – ​ __  ​ + ...  ​

( ( 
​ (cos a)   – cos  a) )​ + (sin a) – sin  a
x
lim ​ ​ ​ ________________________________
= ​  
x Æ 2
    
x–2
 ​      ​
2 x 2

) 3!

It will provide finite limit only when, A – B + C = 0,

= ​  
lim 
t Æ 0
( ( 
​ (cos a)   – cos  a) )​ + (sin a)
t

t + 2
– sin  a
​ ​ ​ ___________________________________
        ​  ​
2 t + 2 2
)
A  + B + C
A – C = 0 and ​ _________
2
 ​
    
=2

= ​  
lim 
t Æ 0
(  (  (cos a) t – 1
​ ​ cos2 a ​ ​  __________
t
     ​  
t
 )
(sin a) t – 1
​ + sin2 a ​ ​ _________  
​  
​  ​ (  )) On solving, we get, A = 1, B = 2, C = 1
Hence, the value of A + B + C + 10 = 14.
= (cos2 a log (cos a) + sin2 a log (sin a))
log (1 + 3x)
(  )
( 
lim ​ ​ ​ __________
77. Limit = ​     
   
​  ​
)
x cos x
e  – 1 – x sin 2x

( 
lim ​ ​ ​ ____________
x Æ 0
74. Limit = ​    ​  
   ​

)
x Æ 0 sin (x2)
log (1 + 3x)

( 
​ __________
)
     ​  × 3x
e x cos x  – 1) _______
x 3x
lim ​ ​__________
= ​   ​    
 ​ 
– ​     ​  
​ lim ​ ​ ​  ______________
= ​         ​  ​
sin (x2) (sin (x2) sin 2x
​ _____
x Æ 0 x Æ 0
  ​ × 2x

2x
75. We have a = min {x2 + 4x + 6, x2 + 2x + 8}  3
= __
​   ​ 
a = min {(x + 2)2 + 2, (x + 1)2 + 5} 2
a = 2
sin x cos x
lim ​ ​ ​ ________ (  )
log (1 + 3x)
lim ​ ​ ​ __________
78. Limit = ​  
x Æ 0 log (1 – 2x) ( 
    ​  ​ )

( 
Also, b = ​   ​ ​
  

e x – e– x

( ( 
x Æ 0

)) )
log (1 + 3x)
sin x
​ ____ ​ × cos x ​ __________
     ​ × 3x
x   
________________ 1 3x
lim ​ ​​  ________________
= ​  
lim ​ ​ ​  x        ​  ​ = __
​   ​  = ​       ​  ​
  
) (  x Æ 0 log (1 – 2x)
– x 2
x Æ 0 –1
e_____ –1
e______ ​ __________
     
​ × – 2x
​ ​  x    ​  ​– ​ ​  – x   ​  ​
– 2x
n 3
Thus, ​S  ​ ​ ​(ar bn – r ) = – ​ __ ​ 
r = 0
2


r = 0
n

(  (  ) )
1 n – r
= ​S  ​ ​ ​ 2r  ​​ __
​   ​   ​​ ​  ​
2 x Æ e
log x – 1
lim ​ ​ ​ _______
79. Limit = ​   (  )
x – e   

​  ​

lim ​ ​(​  __________ ​ )​
1
n log x – log e
​  n   ​ ​S  ​ ​ ​(4r)
= __ = ​  
x Æ e
   
x – e  
2 r = 0
3.34  Differential Calculus Booster

( 
(  {  ( 
} )
x
log ​ 1 + __ )
= ​  
y Æ 0
log (y + e) – log e
lim ​ ​ ​  ______________
y+e–e
    
  ​  ​ ) ex – 1
​ _____
x   

​  – ​ ​ _________
x
__
​ e ​  ​
  
​ e ​ × e
    ​  ​

lim ​ ​ ​ ____________________
= ​   x      ​     ​

(  (  ) )
y+e x Æ 0 –1
e_____
log  ​ ​ _____ ​  ​
e   
​  x    ​ 
lim ​ ​ ​ __________
= ​   y      
​  ​
(  )
y Æ 0

 ( (  ) )
1
= ​ 1 – ​ __
e ​  ​
y
log ​ 1 + __ ​ e ​  ​

( 
_________
)
= ​  
lim ​ ​ ​  y        ​ ​ ln (cos x)
y Æ 0 __
​ e ​ × e lim ​ ​ __________
83. Limit = ​   ​  _____       ​  ​
x Æ 0 4
÷​ 1  + x2  ​– 1

(  )
1
= ​ __
e ​
( 
log ​ 1 – 2 sin2​ __
________________
(  ) )
x
​    ​  ​  ​
2
– 2sin2​ __
_________
(  )
x
​    ​  ​
2
(  )
= ​  
lim ​ ​ ​  x     
 ​ × ​ 4 _____      ​  ​
(  )
log x – log 5
80. Limit = ​  
lim  ​ ​ ​ __________
  
 ​  ​ x Æ 0
– 2 sin2​ __
​    ​  ​ ÷​ x  2 + 1 ​ – 1
xÆ 5 x–5 2

(  ) (  (  )
)
x
log (y + 5)  – log 5 – 2sin2​ __
​    ​  ​
lim ​ ​ ________________
= ​   ​      
 ​ ​ 2
y Æ 0 y+5–5 lim ​ ​ ​ __________
= ​   _____      ​  ​
x Æ 0 4 2
÷​ x  + 1 ​ – 1

(  (  ) )
y+5
log ​ ​ _____
5
lim ​ ​ ​ _________
= ​  
y Æ 0 y 
 ​  
  
 ​
​  

( 
– 2sin2 ​ __
x
2 (  )
lim ​ ​ ________________________
4
_____
​    ​  ​  × ​( ​÷x  2 + 1 ​ 
 + 1 )​

)
 ( (  ) )
= ​   ​  _____    
    _____  ​  ​
y
x Æ 0
(​  ÷​4 x  2 + 1 ​ –  1 )​ ​( ÷​4 x  2 + 1 ​ + 1 )​
log ​ 1 + __ ​    ​  ​

( 
5
lim ​ ​ ​ __________

)
= ​      ​   ​ _____ _____
y Æ 0
5
y
​ __  ​ × 5
= ​  
– 2 sin2 ​ __
2
x
(  )
​    ​  ​ × ​( ​÷x  2 + 1 ​ 
4

lim ​ ​ ​  ________________________________
_____     

    
+ 1 )​ ​( ​÷x  2 + 1 ​ 
_____
2
+ 1 )​
 ​  ​

1
= __
​   ​ 
x Æ 0
(​  ÷​2 x  2 + 1 ​ + – 1 )​ ​(  ÷​2 x  2 + 1 ​ + 1 )​
5

(  (  )
_____ _____

x Æ 5
log (x + 5)  – log (5 – x)
lim ​ ​ ____________________
81. Limit = ​   ​     
x–5
 ​     ​ ) = ​  
lim ​ ​ ​ 
2
x
– 2sin2 ​ __ (  )
​    ​  ​  × ​( ​÷x  2 + 1 ​
4
 + 1 )​ ​( ​÷x
___________________________________
      ​ 
  + 1 ​ + 1 )​
2 2

    ​
x Æ 0 (x2)
( 
(log (x + 5)  – log 5) – (log (5 – x) – log 5)
)
( 
lim ​ ​​ _________________________________
= ​         ​      ​
x–5

)
x Æ 5
_____ _____
​    ​  ​  × ​( ​÷x  2 + 1 ​
x
– 2 sin2 ​ __
4
(  )  + 1 )​ ​( ​÷x
  + 1 ​ + 1 )​
2 2

= ​  
lim ​ ​ ​ ​  ( ( 
log (x + 5) – log 5)
_______________
 ​ 
     ​ – ​ ​ 
log (5 –
) ( 
x) – log 5)
_______________
 ​ 
  
   ​  ​ = ​   ))
lim ​ ​ ​ 
2
___________________________________
      ​      ​
x Æ 5 x–5 x–5 x Æ 0 x2
​ ​ __ ​   ​ × 4
4 (  )
= ​  
lim ​ ​ ​ ​ 
x Æ 0 ( ( 
(log (x + 5)  – log 5
________________
x–5
 ​ 
     ​ + ​ ​ 
5–x ) ( 
log (5 – x)  – log 5
_______________     
   ​ ​  ​ = – 2 ))
1 1 __
= ​ __ ​  + __
5 5 5
2
​    ​ = ​    ​
84. Limit = ​  
x Æ 0
​  ( 
ln (cos (sin x))
lim ​ ​ ____________
x2
 ​ 
     ​ )
x Æ 0 ( 
ex  – log (x + e)
lim ​ ​ ​ _____________
82. Limit = ​  
ex – 1
  
 ​   
​ ) = ​    ( ( 
__________________
lim ​ ​ ​     
(  ) )
sin x
log ​ 1 – 2 sin2 ​ ​ ____

(  )
sin x
2
   ​× ​ 
sin x
 ​  ​ – 2 sin2​ ​ ____
 ​  
____________
x2
 ​ 
2
  
 ​  
 ​

(  )

)
( 
x Æ 0
– 2 sin2 ​ ​ ____
{  ( 
)
 ​  
 ​
= ​  
lim   _________________
​ ​ ​   ​ 
  
x
ex –  log ​ e ​ 1 + __
​ e ​  ​  ​
   ​
)} 2
ex – 1

(  )
x Æ 0

(  )
( 
sin x

)
– 2 sin2 ​ ​ ____
= ​  
{  ( 
ex  – 1 – ​ log ​ 1 + __
____________________
lim ​ ​ ​       ​ 
x
   ​
)}
​ e ​  ​  ​
= ​  
lim   ____________
​ ​ ​   ​ 
2
  
 ​  
 ​
  ​
x Æ 0 ex – 1
x Æ 0 x2
 ( ( 
The Limit  3.35

(  )
) (  (  ) )
sin x
– 2 sin2 ​ ​ ____  ​  
 ​ y 2 y 3 y4
2 sin2 x __ 1 y  – ​ y – ​ __ ​  + ​ __ ​  – ​ __ ​  + ...  ​
lim ​ ​ ​ ____________
= ​     
 ​  × ​ _____  ​ 
 × ​    ​  ​ 2 3 4
x Æ 0 sin x
4
2
​ ​ _____
 ​  
 ​ ) x 2 4 lim ​ ​ ________________________
= ​  
y Æ 0
​     
y2
 ​     ​


1
= – ​ __ ​ 
2
= ​  
y Æ 0
(  (  1
lim ​ ​ – ​ – ​ __ ​  +
2
y
__
​    ​ –
3
y2
))
​ __ ​  + ...  ​  ​
4

85. Limit = ​  


x Æ •
(  1
lim ​  ​ x – x2ln ​ 1 + __ ( 
​ x ​  ​  ​ ))
1
= __
​   ​ 
2
lim ​ ​ __
= ​  
y Æ 0 y ( 
1 1
​   ​ – __
​  2  ​  log (1 + y)  ​
y ) lim ​ ​
88. Limit = ​    ​ 
x Æ p/4 1 – cot x( 
ln cot x
 _______   ​  ​
  )
(  (  ))
(  (  ( 
)
y2 y3 y4
= ​  
y Æ 0
1 1
lim ​ ​ __
​ y ​ – __
​  2  ​  ​ y – ​ __ ​  +
y 2
​ __ ​  –
3
​ __ ​  + ...  ​  ​
4
= ​  
p
log ​ cot ​ __
4
lim ​ ​ ​ ______________   
))
​   ​  + y  ​  ​
    ​  ​
y Æ 0
1 – cot ​ __
p
(  )
​   ​  + y  ​
(  (  ))
 ( ((   )))
y y2 4
1
lim ​ ​ – ​ – ​ __ ​  +
= ​   __
​    ​ – ​ __ ​  + ...  ​  ​
y Æ 0 2 3 4
cot y – 1
log ​ ​ _______   ​  ​
1 cot y + 1
= __
​   ​  lim ​ ​______________
= ​   ​    
    ​  ​
2 y Æ 0 cot y – 1
1 – ​ ​ _______   ​  ​
( 
 ( (  (  )))) )
86. Limit = ​  
xx  – 1
lim ​ ​ ______
​ 
x ln x
  
​  ​ ) cot y + 1

  ( 
x Æ 1

= ​   ( 
(1 + y) –1
lim ​ ​ ​ ______________
  
1 + y
    ​  ​ ) = ​  
lim ​ ​ ​ 
( cot y – 1
log ​ 1  + ​ ​ _______ 
cot y + 1
_____________________
      
 ​ – 1  ​  ​
 ​  ​
y Æ 0 (1 + y)log (1 + y)

( 
y Æ 0 cot y –1
_______
– ​ 1 – ​ ​   
 ​  ​  ​

)
cot y + 1
y2
1  + y (1 + y) + y (1 + y)​ __  ​ + ... –1 = –1
2!
lim ​ ​ ​ ______________________________
= ​              ​  ​

(  )
y Æ 0 log (1 + y) ex  – 1 – x
(1 + y)​ _________
y    
​ × y lim ​ ​​ _________
89. Limit = ​    ​  


x Æ 0 x2

= ​  
y Æ 0
(     
(1 + y)y
y2
y (1 + y)  + y (1 + y) ​ __  ​ + ...
2!
lim ​ ​ ​ _______________________
     ​  ​ )


x Æ 0

1
= __
​   ​ 
( 
e x  – 1
lim ​ ​ ______
= ​   ​ 
2x
    
​  ​ )

( 
2

y Æ 0
y2
2!
y3
lim ​ ​ 1 + ​ __  ​ + (y – 1) ​ __  ​ + ...  ​
= ​  
3! ) 90. Limit = ​   ( 
3
e​x​ ​  – 1 – x3
lim ​ ​ ​ __________   
 ​  
​ )
x Æ 0 64x6
= 1.

87. Limit = ​  


x Æ 1 ln x
1
lim ​ ​ ___ ( 
​      
1
​ – _____
​     
x–1
 ​  ​ ) = ​  
y Æ 0 ( 
ey  – 1 – y
lim ​ ​ ​ _________
64 ◊ (y2)
 ​  


)
( 1 1
) (  )
y
lim ​ ​ _________
= ​   ​       ​ – __
​ y ​  ​
1 e   – 1  – y
lim ​ ___
= ​   ​    ​ ​ _________
​   ​ 
    

y Æ 0 log (1 + y)
y Æ 0 64 (y2)

​ ( 
y –  log (1 + y)
lim ​ ​ _____________
= ​     
    ​  ​ ) 1 ey – 1
lim ​ ___ (  )
​    ​ ​​ _____

 (
y Æ 0 ylog (1 + y) = ​     ​  
 ​
y Æ 0 64 2y

)
y – log (1 + y)
lim ​ ​​  _____________
= ​     
    ​  ​ 1 1
y Æ 0
2 log (1 + y)
_________ = ___
​    ​ × __
​   ​ 
y × ​  y   ​  64 2

( 
1
= ​  
y Æ 0
y  – log (1 + y)
lim ​ ​ ​ ____________
y2
 ​ 
     ​
) = ____
​     ​ 
128
3.36  Differential Calculus Booster

91. Limit = ​  


lim  
x Æ 0 ( 
ex  – e– x – 2x
​ ​ ​ ___________
  
x – sin x
  ​  
​ ) ( ex – e– x
lim ​ ​​ ________ 
= ​  
x Æ 0 – 4sin 2x )
​   

​ ​( ​ __________ lim ​ ​(​  ________ 
1 – cos x ) – 8cos 2x )
x – x x –x
e  + e – 2 e +e
= ​  
lim        ​  
​ = ​     
​  ​
xÆ 0 x Æ 0

lim ​ ​(​  _______
sin x )
x – x
e –e 2 1
= ​     ​  
 ​ = – ​ __ ​  = – ​ __ ​ 
x Æ 0 8 4


x Æ 0
( 
ex + e– x
lim ​ ​​  _______
= ​   cos x  ​  
  ​ ) ( 
x ex  – log (1 + x)
lim ​ ​ ​ ______________
96. Limit = ​    ​ 
     ​ )
x Æ 0 x2
=2

92. Limit = ​  


x Æ e
( 
ln x  – 1
lim ​ ​ ​ _______
x – e   

​ ​ ) = ​  
x Æ 0
(  2x
1 
lim ​ ​ ​ ______________
1
ex  + x ex – _____
​     ​ 
+ 
      
x
)
​  ​

(  )
1
__
​ x ​

(  )
__ 1
= ​  
lim ​ ​ ​   ​   ​
x Æ e 1 ex  + ex + xex + _______
​       ​
(1 + x)2

1 = ​  
lim ​ ​ ​ ____________________
   
 ​     ​
= __
​ e ​ x Æ 0 2
3
= __
( 
​   ​ 
93. Limit = ​  
x  + tan 2x
lim ​ ​_________
​  ​
 ​   ) 2
x Æ 0 x  – tan 2x ln x
lim ​  ​ ​ ___ (  )
(  )
97. Limit = ​   ​   ​
x   
x Æ •

(  )
tan 2x
1 + ​ _____
x   ​  1
__
lim ​ ​ ​  _________ 
= ​    ​  ​ ​ x ​
__
tan 2x lim ​  ​ ​   ​   ​
= ​  
1 – ​ _____
x Æ 0
x ​ 
    x Æ • 1

(  )
=0
tan 2x
1 + 2 ◊ ​ _____   ​ 

= ​  
lim ​ ​ ​ 
x Æ 0
___________ 2x
    ​  ​
tan 2x
1 – 2 ◊ ​ _____   

lim ​ ​ __
98. Limit = ​  
x Æ • ex
x
(  )
​     ​  ​
2x
= ​   (  )
1
lim ​  ​ __
​     ​  ​
(  )
1  + 2 x Æ • ex
= ​ ​ _____ 
 ​  ​ = – 3
1–2
=0

94. Limit = ​  


x Æ 0 ( 
sin x – x
lim ​ ​ ​ _______
x3
 ​  
 ​
) 99. Limit = ​  
x Æ •
x2014
lim ​  ​ ____ (  )
​  x   
e

​  ​


( 
cos x  – 1
lim ​ ​ ________
= ​  
x Æ 0
​ 
3x2
 ​   

) = ​  
x Æ • ( 
2014 ◊ 2013 ... 3 ◊ 2 ◊ 1
lim ​  ​ ​ _________________
ex
       ​  ​ )
= ​  
x Æ 0 ( 
– sin x
lim ​ ​ ​ _____
6x
    
​  ​) = ​  
x Æ • ( 
(2014)!
lim ​ ​ _______
​  x   
e

​  ​ )
1
= – ​ __ ​  =0
6

x Æ 0 ( 
ex  + e– x – x2 – 2
lim ​ ​ ​ _______________
95. Limit = ​     
  
sin2 x – x2
 ​  ​ ) 100. Limit = ​  
x Æ 0 ( 
x2  + 2 cos x – 2
lim ​ ​ ​ _____________
x sin3x
  
    ​  ​ )

x Æ 0 ( 
ex  – e– x – 2x
lim ​ ​ ​ ___________
= ​     
sin 2x – 2x )
   ​  ​ = ​  
x Æ 0 ( 
x2  + 2cos x – 2 _____
lim ​ ​ ​ _____________
x 4
 ​
    
x3
× ​  3    
sin x
​  ​ )
lim ​ ​( ​ ___________
2 cos 2x – 2 ) (  )
x –x
e   + e – 2x x2  + 2cos x – 2
= ​      ​ ​
   lim ​ ​ ​ _____________
= ​    ​ 
     ​
x Æ 0
x Æ 0 x4
The Limit  3.37

= ​  
x Æ 0
​ 
4x3 ( 
2x  – 2sin x
lim ​ ​ __________
 ​ 
     ​
)

= ​e​y Æ 0
=e
​  
lim ​  ((cos y – sin y))

= ​  
x Æ 0 12x2( 
2  – 2cos x
lim ​ ​ ​ _________
 ​  


) x Æ 0 (  )
ax  + bx  + cx 2/x
lim ​ ​​ ​ __________
105. Limit = ​  
3
 ​ 
     ​​ ​

= ​  
x Æ 0
2sin x
24x ( 
lim ​ ​ ​ _____   

​  ​ ) lim ​ ​​(1  + ​( ​ __________
= ​  
a   + b +c
 ​ – 1 )​ )​​ ​
    
x x x 2/x

x Æ 0 3
1
= ___
​    ​ 
lim ​ ​​( 1 + (​ ​  ______________ )​ )​​ ​
12 a  + b + c – 3 x x x 2/x
= ​    ​ 
    
x Æ 0 3
101. Limit = ​  
lim 
p
​ (tan x loge sin x)
x Æ ​ __ ​ 
2
= e​ ​x Æ 0
​   ( (  a x  + bx + cx – 3
lim ​ ​ ​ ______________
​ 
3
 ​ 
    
2
​ × __ ) )
​ x ​  ​

= ​   
lim  
x Æ p/2
loge sin x
​ ​ ​ ________
cot x ( 
​  
    ​ ) ​   ( (  (a x  – 1) + (bx – 1) + (cx – 1)
lim ​ ​ ​ ​  ________________________
   
x 
2
  ​  ​ × __
​   ​   ​ ) )
= ​e​x Æ 0 3 ​

= ​   
lim   ​ 
cot x
​ ​ ________
x Æ p/2 – cosec2 x
   
( 
​  ​
) = ​e​ 3 ( 2
​ __ )​
​   ​  log (abc)  ​

=0
= (abc)2/3
lim ​ ​
102. Limit = ​  
x  + 6 x + 4
 ​ ​ _____ 
 ​  ​​ ​ (  ) 1 tan​( ​  2 ​  )​
p x
x+1
( 
___
)
x Æ •
lim ​ ​​ 2 – ​ __
106. Limit = ​   x ​  ​​ ​

( 
x Æ 1

)
x + 1 x + 4
5 ____  × _____
​   ​  ​    
 ​  ×  5

( 
lim ​ ​​ 1 + _____
= ​   ​      ​  ​​ 5 x + 1 ​ 1 tan​( (1 + y) ​ 2 ​  )​
p

)
__
x Æ • x+1 lim ​ ​​ 2 – _____
= ​   ​      ​  ​​ ​
y Æ 0 1+y
4
1 + ​ __
x ​

( ___
( 
____
py
)
​   ​  
× 5

)
1
__
1 + ​ x ​
lim ​ 
​​   ​ 1 – cot ​ ​  2 ​  ​
= ​e​ x Æ • ​ lim ​ ​​ 2 – _____
= ​   ​      ​  ​​ ​
y Æ 0 1+y
= e5
(  ( 
– cot ​( ​   ​  )​
pr

))
___
1
lim ​ ​​ 1 + ​ 1 – ​ _____
( 
2
= ​      ​  ​  ​​ ​
lim ​ ​
103. Limit = ​  
2x  + 4 2x + 10
 ​ ​ ______ 
x Æ • 2x + 3
 ​  ​​ ​ ) y Æ 0 1+y

( 

(  ) ) ( )
py
y – cot ​ ___
​   ​  ​

(  )
y  + 4 y + 10 lim ​ ​​ 1 + ​ _____
= ​   ​      ​  ​  ​​ 2

1+y
 ​ ​ _____ 
y Æ 0
lim ​ ​
= ​    ​  ​​ ​
y Æ • y + 3

(  )y p y
(  )
( 
y + 3 _____
____ y + 10 lim ​ ​ _____
​   ​      ​  ​ × – cot ​ ___
​   ​  ​
1 ​  1 ​ × ​ 
)

   ​  1+y 2 ​
lim ​ ​ 1 + _____
= ​   ​      ​  ​​ y + 3
​ = e​ ​y Æ 0
y+3
(  )
y Æ • py
​ ___
(  ) (  )
​   ​  ​
–1
_____ 2
________ 2
​  
lim ​ ​ ​      ​  ​ × ​      ​ × __
​ p ​
y + 10 y Æ 0 1 + y py
_____
lim ​  
​  tan ​ ___
​   ​  ​
​  
​y Æ • y + 3 ​
 ​  = ​e​ 2 ​
= ​e
2
__
=e ​ p ​​
= e​ – ​ 
lim ​ ​​
1
  sin ​ __ (  (  )
1 x
​ x ​  ​ + cos ​ __ (  ) )
(  ( 
104. Limit = ​   ​ x ​  ​  ​​ ​ 1
___
p
))
​      ​
x Æ •
lim ​ ​​ tan ​ __
107. Limit = ​   ​   ​  + ln x  ​  ​​ln x ​
1
__
x Æ 1 4
​ y ​
= ​  
lim ​ (sin y + cos y​)​ ​

(  )
y Æ 0 1
1 + tan (log x) ____
​     ​ 
1
__
​   ​ lim ​ ​​​ ____________
= ​       ​  ​​log x ​
= ​  
lim ​ (1 + (sin y + cos y – 1)​)​y ​ x Æ 1 1 – tan (log x)
y Æ 0

(  )
1
1
​ __ ​ 2tan (log x) ____
​     ​ 
lim ​ ​​ 1 + ​ ___________
​  
lim ​( (sin y + cos y – 1)​)​y ​
= ​e​ y Æ 0 ​ = ​       ​  ​​log x ​
  
x Æ 1 1 – tan(log x)


​  
= e​ ​y Æ 0
( 
(sin y + cos y – 1)
lim ​ ​ ​  ______________
y     ​   ​ )​
3.38  Differential Calculus Booster

= e​ ​x Æ 1 (  2tan (log x)
lim ​ ​ ​ ___________
​  
1 – tan (log x) )
1
    ​  ​ × ____
   ​     ​ 
log x ​ = ​  
x Æ 0
​ (  ( 
tan x  – sin x
lim ​  ​​ 1 + ​ __________  
1 + sin x
  
cosec x
​ – x  ​  ​​ ​ ))
= e2
= ​e​x Æ 0
​   ( ( tan x  – sin x
lim ​ ​ ​ __________
​      
1
​  ​ × ____
​      
​  ​
sin x ​ ) )
( 
1 + sin x

(  (  )) )
p
ln ​ tan ​ __
​   ​   + 2x  ​  ​
108. Limit = ​  
x Æ 0
​ 
4
lim ​ ​ _______________
sin 3x
   
  ​  ​
= e​ ​x Æ 0
(  ( 
​   ​  )
sec x  – 1
lim ​ ​ sin x ​ ________
1 + sin x

1
 ​  ​ × ​ ____
    
sin x ​
​  ​ )
lim ​ ​ ( ________ ​  )​
sec x  – 1

( 
​   ​   

(  )) = ​e​ 1 + sin x ​
x Æ 0
1  + tan 2x
log ​ ​ _________   ​  ​
1 – tan 2x
______________
= ​  
lim ​ ​ ​     
  ​  ​ = 1
x Æ 0 sin 3x
n __ n
( 
a  – 1 + ÷    ​
​ b 
 ​ ​ __________ )
(  (  (  )
lim ​ ​
111. Limit = ​   a     ​   ​​ ​
1  + tan 2x
log​ 1  + ​ ​ _________ 
1 – tan 2x
 ​ – 1  ​  ​ )) n Æ •
__
 ​( 1 + ​( ​ ______
lim ​ ​ ​ ______________________
)​ )​​ ​
= ​           ​  ​ n n
sin 3x ​ b ​ – 1 ÷   
x Æ 0 lim ​ ​
= ​   a    ​  

( 
n Æ •

(  (  ) ) (  )
)
__
2tan 2x 2tan 2x ( (  ) )​
n
log ​ 1 + ​ ​ ________     ​  ​  ​ ​ ​ ________    ​  ​ ​  
  ​  – 1
​÷b 
lim ​  ​ ​ ​ ______
a    ​  ​ × n  ​
1 – tan 2x 1 – tan 2x = ​e​ n Æ •

lim ​ ​ ​ _________________
= ​         ​ × ​  __________   
  ​   ​
x Æ 0
( 
2tan 2x
​ ​ ________  
1 – tan 2x
  ​  ​ )
sin 3x

= ​e ​n Æ •
( (  ) )
​ ​ ​ ______
lim ​  
​  
1
​ __ ​
​b​n ​ – 1
a    ​  ​ × n  ​

(  ( 
2tan 2x
) ) (  )
1
__
​ ​ _________
​   ​
  ​  
​ ​  
​b​n ​ – 1
​ ​ ______
lim ​  
1
 ​ × __
 ​   ​ a ​
1  – tan 2
lim ​ ​ ​ __________
n Æ • 1
__
= ​     
  ​   ​ = ​e​ ​ n ​

x Æ 0 sin 3x
1
__

( (  )
​   ​ log b

) = e​ ​a
tan 2x 3x 2x ​
lim ​ ​ ​ ​ _____
= ​    
  ​  ​ × _____ × ​ ___  ​ × 2  ​
​    ​ 
x Æ 0 2x sin 3x 3x 1
__
​ a ​
= e​ log ​
​ b​ ​​
4
= __
​    ​. 1
__
​   ​
3 = b​ ​a ​
109. Limit = ​  
x Æ 0
tan x 1/x
lim ​ ​​ ​ ____ (  )
​  ​​ ​
x   
1
112. We have, –1 £ sin ​ __
​ x ​  ​ £ 1 (  )
(  (  )) £ x sin ​( __
​ x ​ )​ £ x
tan x 1/x 1
lim ​ ​​ 1 + ​ ​ ____
= ​   x  ​ – 1  ​  ​​ ​
    fi – x2 2 2
x Æ 0

​ ​​(1  + (​  ________ )​ )​​ ​   ​ (x2) = 0


tan x  – x 1/x Now, ​ lim 
= ​  
lim   ​  x    ​   x Æ 0
x Æ 0

  ​ (– x2) = 0
and ​ lim 
= ​e ​x Æ 0 x( tan x  – x
lim ​ ​ ________
​   ​      ) 1
​  ​ × __
​ x ​ x Æ 0

​  
(  tan x  – x
lim ​ ​ ________
​   ​  
)
 ​
Thus, ​ lim 
1
  ​ ​ x2sin ​ __
​ x ​  ​  ​ = 0
x Æ 0
(  (  ) )
​ = e​ ​x Æ 0 x2 ​
113. We have, – 1 £ sin x £ 1
(  sec 2 x  – 1
lim ​ ​ _________
​   ​ 
e​ ​x Æ 0 2x ​
  ​   
​ ) 1 ____ sin x __ 1
= fi – ​ __
x ​ £ ​  x   
​ £ ​ x ​

= ​e​x Æ 0 ( 2sec2 x tan x


lim ​ ​ ​ __________
​  
2
 ​ 
     ​
​ ) Now, ​ lim  (  )
1
  ​ ​ – ​ __
x ​  ​ = 0 x Æ •

​ ​( __
=1
​ x ​ )​ = 0
1

( 
and ​ lim 
   
lim ​ ​​ ________
110. Limit = ​   ​  )
1 +  tan x cosec x
​   ​​
  ​
x Æ •

  ​  ​( ​ ____
x Æ 0 1 + sin x
​  )​ = 0
sin x
Thus, ​ lim  x   
lim ​ ​​( 1 + (​  ​ ________  ​ – 1 )​ )​​
cosec x x Æ •
1  + tan x
= ​   ​
x Æ 0 1 + sin x 114. We have, x – 1 < [x] £ x
The Limit  3.39

x–1
fi ​ _____
x   ​ <
[x] x
___
​  x   ​ £ __
​ x ​ = ​  
x Æ •
​  ( 
n (n  + 1)x __
lim ​ ​ _________
2n2
 ​  
1
– ​ n ​  ​ )
fi (  1
​ 1 – __ )
​ x ​  ​ <
[x]
___
​  x   ​ £ 1
lim ​ ​ ​ 1 + __
= ​  
1 x 1
( ( 
​ n ​  ​ ​ __  ​ – __
​   ​  ​ ) )
x Æ • 2 n
Now, ​ lim 
x Æ •
1
  ​  ​ 1 – ​ __( 
x ​  ​ = 1 ) = __
x
​    ​
2
​ (1) = 1
and ​ lim 
    (1 +  2 + 3 + ... + n)x
x Æ • and ​ lim 
    ​ ​ __________________
     ​   
x Æ •
n2
[x]
(  )
( 
  ​ ​ ___
​  x   ​  ​ = 1
)
Thus, ​ lim  n (n  + 1)x
lim ​ ​ _________
x Æ •
= ​   ​   ​   

x Æ •
2n2
115. We have, x – 1 < [x] £ x
1 ___
fi ​ __
1 _____1
x ​ £ ​ [x]  ​ < ​ x  –  1 ​ 
= ​  
x Æ • ( (  ) )
1 x
lim ​ ​ ​ 1 + __
​ n ​  ​ ​ __  ​  ​
2
x
x x x = __
​    ​
fi ​ __x ​ £ ___
​    ​ < _____
​     ​  2
[x] x  –  1


x
[x] x  –  1
x
1 £ ​ ___  ​ < _____
​     ​  x Æ •
n2 ( 
[x]  + [2x] + [3x] + ... + [nx])
  ​ ​ ​ _________________________
Thus, ​ lim      ​ 
x
   ​ = __
​    ​
2 )
(  )
x
  ​  (1) = 1
Now, ​ lim 
x Æ •
x  – ​Ú ​    ​ cos (t 2)dt
lim ​ ​ ______________
0

( 
117. Limit = ​   ​      ​  ​
  
x
  ​ ​ _____
and ​ lim  ​     
x Æ • x – 1
 ​  ​ = 1 ) x Æ •
x3 – 6x

x
  ​ ​ ___
Thus, ​ lim 
x Æ • [x] (  )
​    ​  ​ = 1 ( 
1  – cos (x2)1
lim ​ ​ ​ ___________
= ​  
x Æ •
  
3x2 – 6
 ​  
​ )
116. We have, x – 1 < [x] £ x
2x – 1 < [2x] £ 2x
( 
1  – 1
= ​ _____
​ 
0–6

 ​  ​ )
3x – 1 < [3x] £ 3x =0

(  )
... ... ...  x _____

... ... ...  Ú​  ​  ​ ​÷4  + t4 


 ​dt
0_________
... ... ...  lim ​ ​ ​ 
118. Limit = ​    ​    

x Æ •
x3
nx – 1 < [nx] £ nx

(  )
_____
By adding we get, = ​  
÷​ 4  + x ​ 
lim ​ ​ ​ _______
4
   ​
  ​
x Æ • 2
3x
(1 + 2 + 3 ... + n)x – n < ([x] + [2x]

( ÷  )
______
+ [3x] + ... + ([nx]) £ (1 + 2 + 3 + ... 4 + x4
+ n)x – n lim ​ ​ ​ ​ _____
= ​    ​ ​ 
  
 ​
x Æ •
9x4

( ÷ 
_______
(1  + 2 + 3 + ... + n)x – n
fi ​ ______________________

   
n2
  ​ = ​  
x Æ •
9x
4
lim ​ ​ ​ ___
1
​   4 ​ + __
​   
9
  ​ ​  ​
)
1
([x]  + [2x] + [3x] + ... + [nx]) = __
​   ​ 
< ​ _________________________
    ​    3

(  )
n2 x 2

Ú​  ​  ​ ​e– ​t​ ​dt
2
(1  + 2 + 3 + ... + n)x
£ ​ __________________
    ​    ______
x
n2 lim ​  ​ ​ 
119. Limit = ​    ​ 
 ​

x Æ 1 x–1

Now, ​ lim 
x Æ •
(1  + 2 + 3 + ... + n)x – n
  ​ ​  ______________________
   
n2
 ​    = ​  
lim  
x Æ 0
​ ( 
e– x​​ 4​ ◊ 2x  – e–x ◊ 1
​ ​ _____________
1
 ​ 
     ​ )
= –1
3.40  Differential Calculus Booster

120. Limit = ​  


n Æ • n + 1 [ 
1
lim ​   ​ _____
​     
1
 ​ + ​ _____
  
n+2 n+3
1
 ​ + ​ _____
  
1
 ​ + ... + ___
​    ​ ​
2n ] 2n
2 2r
lim ​ ​S   ​ ​ __
123. Limit = ​   ​​ n ​ ​ __
n Æ • r = 1
( 
​ n ​  + 1  ​ )
lim ​  ​( _____
n + n) (  ) ( 
1 1 1 1 1 2n r r
= ​  
n Æ •
​      ​ + ​ _____
n+1 n+2 n+3
   ​ + _____
​      ​ + ... + _____
​      ​  ​ = ​  
n Æ •
​ n ​ ​S   ​ ​ ​2 ​ __
lim ​ __
r = 1
​ n ​   ​ ​ 2 ​ __
n  ​ + 1  ​ )
( 
2
1 n
= ​  
n Æ • n r = 1 n + r
n
​   ​ ​S  ​ ​ ​ _____
lim ​ __ ​      ​  ​ ) = ​Ú ​  ​  2x (2x + 1)dx

( 
0

)
n
1 1 2
= ​   ​   ​  ​S  ​ ​ ​ _______
lim ​ __ ​   r    ​  ​ = ​Ú ​  ​  (4x2 + 2x)dx
n Æ • n r = 1
1 + ​ __ (  )
​ n ​   ​

0
1

(  )|
dt
= ​Ú ​  ​ ​ _____
2
  
 ​ 4x2
0 1 + x
= ​​ ​ ​ ___ ​  + x2  ​  ​​ ​​ 
​ 3 0

= log 2 ( 
16
= ​ ___
28
)
​   ​ + 4  ​ = ​ ___ ​ 

[ 
3 3
lim ​  
121. Limit = ​  
n
​ ______
​  2   2 
n
 ​ + ​ ______
n
 ​ + ​ ______
  2 
1
 ​ + ... + ___
  2  ​     ​  ​
]
2n
(  )
n Æ • 2 2
n +1 n +2 n +3 (2n!)! 1/n
lim ​ ​​ ​ _____n ​  ​​ ​
124. Let A = ​  

[ 
n Æ • n! ◊ n

= ​  
n Æ •
n
lim ​ ​ ______
​  2   2 
n +1 n +1
n
 ​ + ______
n
 ​ + ... + ______
​  2   2  ​  2   2 
n +n
 ​  ​
] = ​   ( 
(2n(2n – 1) (2n – 2)...(2n – (n – 1)))n! 1/n
lim ​ ​​ _____________________________
​            ​  ​​ ​ )
n Æ • n!nn

(  )
n
n
lim ​ ​S  ​ ​ ​ ______
( 
= ​   ​  2   2 
 ​  ​
n Æ • r = 1
n +r = ​  
(2n(2n  – 1)(2n – 2)....(2n(n – 1))) 1/n
​ _____________________________
lim ​   ​           ​  ​​ ​ )
( 
n Æ • nn
= ​  
1 n
n Æ • n r = 1
n2
​   ​ ​S  ​ ​ ​ ​ ______
lim ​ __
n + r2
2
  
 ​  ​ ) fi Log A

( 
( 
= ​  
1 n – 1 2n  – r
​ n ​  ​S ​ ​ ​log  ​ ​ ______
lim ​ __ n    ​  ​ )
)
n
1 1
​   ​ ​S  ​ ​ ​ _______
n Æ •
lim ​ __
= ​   ​   r 2   ​  ​ r = 0
n Æ • n r = 1
1 + ​​ __ (  )
​ n  ​  ​​ ​
1

1 n – 1
lim ​ __
= ​  
n Æ • n r = 0 ( 
​   ​  ​S ​ ​ ​log  ​ 2 – __
r
​ n ​   ​ )
dt
= Ú​  ​  ​   ​ _____  2 
 ​ 1
0 1 + x
= ​Ú ​  ​  log (2 – x)dx
0
= (tan–1x)​|10​ ​​ 
1

= (tan–1(1) – tan–1(0)) = – ​Ú ​  ​  log y dy, where (2 – x) = y


2
p
= __
​   ​  1
4 ​ – y log y) |​​2​​ 
= ​​ (y

(  )
2n
r
122. Limit = ​    S   ​ ​ ​ _______
lim ​ ​ ​  ______
    ​  ​ = 1 – 2 + 2 log 2
n Æ • r = 1
÷​ n  + r 2 
2
 ​ = log 4 – 1

(  )
2n
1 nr
= ​   ​   ​  ​S   ​ ​ ​ ​ _______
lim ​ __ ______     ​  ​ = log 4 – log e
n Æ • n r = 1
÷​ n  + r 2 ​ 
2

( ÷  (  ) )
__r
​ n ​ 

4
= log ​ ​ __
e ​  ​ (  )
1 2n _______
= ​   ​ n ​ ​S   ​ ​ ​ ​ _________
lim ​ __      ​  ​ 4
Thus, A = ​ __
n Æ • r = 1
​ 1 + ​​ ​ __
r 2
  
​  ​​
  ​ ​ e ​
n
​ ​x​2​log (x – x 2)
​ lim 
  + 

( 
2
2 ​(x – x2​)x​ ​ = ​e​x Æ 0
125. ​ lim 
  +  ​

0 ÷
x
= ​Ú ​  ​  ​ _______
​  _____
    ​ ​ dx
​ 1  + x2 
 ​ )
x Æ 0

  + 
= ​e​x Æ 0
(  ) x
​ ​ _____
​ lim  ​((x – x​ 2​ ​) log (x – x​ 2​ ​))
​    ​  
x – 1 ​
_____
= ( 
 ​​​​ ​÷1  + x2  )|
2
 ​  ​  ​​0​​ 
_____ = e0   ( ​ lim 
  +  ​ x log x = 0) = 1
x Æ 0
= ​( ​÷5  – 1 ​  
)​
The Limit  3.41
1
 ​ _________
     ​ lim ​  (1x + 2x + 3x + ... + 99 x )1/x
132. Limit = ​  
  –​ (1 – x2​)​log (1 – x) ​
126. ​ lim  x Æ •
x Æ 1
lim ​  (1x + 2x + 3x + ... + 99 x )1/x
= ​  
log (1 – x 2) x Æ •
​  
lim  ​ _________
​         ​
= ​e​x Æ 1 log (1 – x) ​
(  ( (  ) (  ) (  ) (  ) ))

1 x 2 x 3 x 99 x 1/x
lim ​ ​ ​ 99x ​ ​​ ___
= ​   ​    ​  ​​ ​ + ​​ ___
​    ​  ​​ ​ + ​​ ___
​    ​  ​​ ​ + ... + ​​ ​ ___  ​  ​​​ ​  ​​ ​
​  
l im  ​ ________
​   ​  
log (1 + x)
log (1 – x) ________
+ ​ 
log (1 – x) log (1 – x) ​
 ​  x Æ • 99 99 99 99
e​ ​x Æ 1

= ​e​ ​
= 99(0 + 0 + 0 + ... + 1) 0
1+10
=e =e = 99
1
__ 133. Put y = cosec2 x
  ​ (3x +
127. ​ lim  4x​)​ ​x ​​
x Æ • when x Æ 0, y Æ •

lim ​ ​
= ​   (  (  (  ) ) )
3 x __​ 1 ​
 ​ 4x ​ 1 + ​​ __
​   ​   ​​ ​  ​  ​​x ​
4
lim ​  (1y + 2y + 3y + ... ny)1/y
Limit = ​  
y Æ •

(  ((   ) (  ) (  )
x Æ •
lim ​ ​
= ​  
1 y
 ​ ny​​​ __
2 y 3 y
​ n ​  ​​ ​  + ​​ __
​ n ​  ​​ ​ + ​​ __
​ n ​  ​​ ​ + ... + ​​ __ (  ) ) )
n y 1/y
​ n ​  ​​ ​  ​  ​​ ​

(  (  ) )
y Æ •
3 x __​ 1 ​
lim ​ 4 ​​ 1 +
= ​   ​​ __
​   ​   ​​ ​  ​​x ​ = 4
x Æ • 4 = (n (0 + 0 + ... + 0 + 1))
=n
 ( ​x​ ​x ​​  ​ )
1
__
  ​ ​
128. ​ lim 
x Æ •

(  ) = e​ ​ ​ lim ​  ​( __​ 1x ​  )​= ​e​ ​ = 1​
1
log x ​ __ ​
0
​   ​ ____
lim ​   ​  x   
​  ​
= ​e ​x Æ • ​ x Æ •
sin x
1. When x > 0, sin x < x fi ​ ____ ​ < 1
x   
x + 1 x2 + 2014
 ​​ ______ 
lim ​ ​
129. Limit = ​  
x Æ • 2x + 1
 ​  
​ ​ ( 
​ ) Thus, R.H.L = ​ lim 
  + 
x Æ 0
sin x
​ ​ ​ ____ [  ]
​  ​ = 0
x   

(  )
sin x
1 x  + 2014
2
When x < 0, sin x < x fi ​ ____
x   ​ < 1
1 + __
​ x ​
 ​ ​ _____ 
lim ​ ​
= ​  
x Æ •
2 + __
1
 ​  ​​
​ x ​

Thus, L.H.L = ​  
lim 


sin x
​ ​ ​ ____ [  ]
​  ​ = 0
x   
x Æ 0

= ​​ __(  )
1 •
​   ​   ​​ ​
2
x Æ 0
sin x
  ​ ​ ​ ____
Hence, ​ lim  ​  ​ = 0
x    [  ]
tan x
2. When x > 0, tan x > x fi ​ ____
x    ​ > 1
=0

130. Limit = ​     1​ + ln x ​


lim ​ ​
x Æ •
ln 2
​ ______  
 ​
Thus, R.H.L = ​ lim 
  + 
x Æ 0
tan x
​ ​ ​ ____ [  ]
​  ​ = 1
x   

= ​e​
(  log 2
lim ​  ​ ________
​   ​ 
x Æ • 1 + log x
  
 ​  ​log x)

When
tan x
x < 0, tan x > x fi ​ ____
x   ​ > 1

​   (  )
log 2
____
​   ​ 
1
​ _____
lim ​   ​ 

 ​ + 1  ​
Thus, L.H.L = ​  
lim  
x Æ 0 –
tan x
​ ​ ​ ____ [  ]
​  ​ = 1
x   
x Æ • log x
= ​e​ ​

= elog 2 = 2 x Æ 0
tan x
  ​ ​ ​ ____
Hence, ​ lim  ​  ​ = 1
x    [  ]
lim ​  (2x + 3x)1/x
131. Limit = ​   sin–1x
x Æ • 3. When x > 0, sin–1 x > x fi ​ _____
x   ​> 1

lim ​ ​
= ​  
x Æ • (  ( 
2 x 1/x
 ​ 3x​ 1 + ​​ __
​    ​  ​​ ​  ​  ​​ ​
3 (  ) ) ) Thus, R.H.L = ​ lim 
  + 
sin–1x
​ ​ ​ _____
x     [  ]
​  ​= 1
((   2 x 1/x
(  ) ))
x Æ 0
lim ​  3 × ​​ ​ 1 + ​​ __
= ​   ​   ​   ​​​  ​  ​​ ​
x Æ • 3 sin–1x
When x < 0, sin–1x > x fi ​ _____
x   ​> 1

= (3 (1 + 0) 0)
=3 Thus, L.H.L = ​ lim 
  – 
x Æ 0
sin–1x
​ ​ ​ _____
x    [  ]
​  ​ = 1
3.42  Differential Calculus Booster

Hence, ​ lim 
sin–1x
  ​ ​ ​ _____
x   ​  ​ = 1
x Æ 0
[  ] x
fi ​ ____
    
sin x
tan x
​ > 1 and ​ ____  ​ > 1
x   

4. When
tan–1x
x > 0, tan–1x < x fi ​ _____​ < 1
x    x Æ 0
x
  ​ ​ ​ 2016 ____
Thus, ​ lim  ( [ 
sin x
tan x
​  ​ + ​ ​ ____
​       ] [  ] )
​  ​  ​
x   

Thus, R.H.L = ​ lim 


  + 
tan–1x
​ ​ ​ _____ [ 
​  ​ = 0
x   
x Æ 0
]
= 2017
= 2016 + 1

tan–1x 9. We have
x < 0, tan–1x < x fi ​ _____
When ​ < 1
x    sin–1x
  ​ ​ ​ 2016 ​ _____
​ lim  x    ( [  ____x
​  ​ + ​ ​ tan x ​  ​  ​
     ] [  ] )
[ 
x Æ 0

Thus, L.H.L = ​ lim 


  – 
x Æ 0
tan–1x
​ ​ ​ _____​  ​ = 0
x    ] As we know that,
sin–1x > x and tan x > x
[ 

x Æ 0
tan–1x
  ​ ​ ​ _____
Hence, ​ lim  ​  ​ = 0
x    ] sin–1x x
fi ​ _____
x   ​ > 1 and ____
​      
​< 1
tan x
5. ​ lim 
    ​ [sin x + cos x]
5p
x Æ ​ ___ ​ 
4 Thus, ​ lim 
x Æ 0
( [ 
  ​ ​ ​ 2016 ​ _____
x   ​  ​
x
sin–1x + ​ ​ ____
    
​  ​ 
tan x ​ ] [  ] )
[  (  )]
__ p
= ​ lim 
        sin ​ x + __
​ ​ ​÷2 ​ ​   ​   ​  ​ = 2016 + 0
5p
___
x Æ ​   ​  4
4
= 2016

[  (  ) ]
__ 3p 10. We have
  s  in ​ ___
= ​ ​÷2 ​ ​   ​   ​  ​

= – 2
2
​ lim 
x Æ 0
( [ 
​  ​
x   
tan x
tan–1x + ​ ​ ____
  ​ ​ ​ 2016 ​ _____  ​  ​  ​
x    ] [  ] )
[ 
As we know that

x2
  ​ ​ ​ ________
6. ​ lim     
x Æ 0 sin x tan x
 ​  ​ ] tan–1x < x but tan x > x

[ 
tan–1x tan x
​ _____  < 1 but ​ ____
= ​  
x ____
lim ​ ​ ____
​      
x
​ ​      
x Æ 0 sin x tan x
​  ​ ] x ​   ​ > 1
x   

7. As we know that, sin x < x


x Æ 0
( [ 
tan–1x
  ​ ​ ​ 2016 ​ _____
Thus, ​ lim   
x   
tan x
​  ​ + ​ ​ ____ ] [  ] )
 ​  ​  ​
x   
sin x
fi ​ ____
x   ​ < 1 = 2015 + 1
= 2016.
x Æ 0
n sin x
  ​ ​ ​ _____
fi ​ lim  x    
​  ​ = n – 1 [  ] 11. As we know that

(  )tan x
Also, ​ ​ ____ ​  ​ > 1
x   
​ lim 
  + 
​ [cos x] = 1 = ​  
x Æ 0
lim 

 ​ [cos x]
x Æ 0

  ​ ​[ ​ ______
fi ​ lim 
n tan x
x    ​ ]​ = n
x Æ 0
  ​ ​ _________
Thus, ​ lim  ​ 
x Æ 0 1 + [cos x]( 
sin [cos x]
  
 ​  ​ )
Thus, ​ lim 
   
x Æ 0
n sin x
​ ​ ​ ​ _____
x    ( [ 
n tan x
​  ​ + ​ ​ ______
x    ​  ​  ​
  ] [  ])
sin (1)
= ______
​ 
1+1

sin (1)
 ​ = ______
​   ​ 
2

=n–1+n 12. As we know that


= 2n – 1. x £ [x] < (x – 1)

8. We have ​  
x Æ 0
x
lim ​ ​ ​ 2016​ ____
sin x
    ( [  tan x
​  ​ + ​ ​ ____ ] [  ] )
​  ​  ​
x   
1
fi ​ ______
   
(x – 1)
 ​ <
1
___
​    ​ £
[x]
1
__
​ x ​

As we know that, sin x < x and tan x > x log x log x ____log x
fi ​ ______   ​ < ____
​   ​  £ ​  x    ​
sin x tan x (x – 1) [x]
fi ​ ____
x   ​ < 1 and ​ ____  ​ > 1
x   
sin x
fi ​ ____
x   
tan x
​ < 1 and 1 < ​ ____  ​ < 2
x   
Now, ​ lim  ​ 
log x
  ​ ​ ______
x Æ • (x – 1)
   ( 
 ​  ​ )
The Limit  3.43

= ​  
x Æ • 1
1
__
(  )
​ x ​
__
lim ​ ​ ​   ​   ​ = 0
16. when

when x < 0, L.H.L


sinx
lim ​ ​ ​ ____
x > 0, R.H.L = ​   ​  ​ = 1
x    x Æ 0
(  )
Also, ​ lim 
x Æ •
log x
  ​ ​ ____
​  x   
​  ​ (  ) = ​  
lim   
sin x
​ ​ ​ ____
x (  ) ( 
sin (–1)
​  ​ = ​ _______
   ​   ​  
  )
​ = sin (1)
x Æ 0 –
(–1)

= ​  
x Æ • 1
1
__
(  )
​ x ​
__
lim ​ ​ ​   ​   ​ = 0 Thus, ​ lim 
x Æ 0
sin [x]
  ​ ​ _____
​ 
[x]
 ​   (  )
 ​ does not exist.

By Sandwitch theorem, we get, 17. As we know that

​ lim 
x Æ •
log x
  ​  ​ ____
​  x   
 ​  ​ = 0 (  ) Ï
ÔÔ x : x<
p
–1 2
sin (sinx) = Ì
13. Let x = m + f, 0 £ f < 1 p
Ôp – x : x >
Then [x] = m ÔÓ 2
when x Æ •, then m Æ • Now, L.H.L

Now, ​ lim 
x Æ •
log  (xn) – [x]
  ​ ​ ​ ___________
[x]
 ​ 
    ​, n Œ N(  ) [sin–1(sin x)]
lim –​ 
= ​   
p
(  )
x Æ ​​ __
​   ​   ​​ ​
2

= ​ lim  ​ 
m Æ •
( 
n log (m  + f ) – m
  ​ ​ _______________
m     ​   ​ ) lim –​ 
= ​   
x Æ ​​( __
p
​   ​  )​​ ​
[  ]
p
[x] = ​ __
​   ​   ​ = 1
2

(  )
2
n
​ ______
    ​ – 1 Also, R.H.L
(m + f)
__________
  ​ ​ ​ 
= ​ lim   ​     ​
m Æ • 1 [sin–1(sin x)]
lim +​ 
= ​   
(  )
(  )
p
0–1 x Æ ​​ __
​   ​   ​​ ​
= ​ ​ _____
 ​  
 ​ = –1 2

[  ]
1 p
lim +​ 
= ​    [x] = ​ __
​   ​   ​ = 1
x Æ ​​( ​   ​  )​​ ​ 2
p
(  ) ( 
__
m x m n – x
  ​  Cx​ __
14. ​ lim 
x Æ •
​ n ​   ​​ ​ ​ 1 – __
n
​ n ​   ​​ ​ ) 2

[sin–1(sin x)] = 1
lim ​ 
Thus, ​   
(  m n
)
1  – __
(  (  )
p
m x ​​________ ​ n ​   ​​ ​
()   )  
x Æ ​ __
​   ​   ​
n!
________ __ 2
lim ​ ​ ​ 
= ​        ​  ​ ​​ ​ n ​   ​​ ​ ​  m x   ​
x Æ • n!(n – x)!
( )
​​ 1 – __​ n ​   ​​ ​ 18. As we know that sin x < x

( 
m n
1 – ​ __ ) sin x
fi ​ ____
( 
mx ​​________ n ​   ​​ ​ ________________________ x   ​ < 1
lim ​  
= ​   ___
​ ​   ​   ​ ​  ) ( 
m x 
n(n– 1) (n – 2) ... (n – x + 1)
 ​ ​ 
)
   
nx
    ​
​​ 1 – __
[  ]
x Æ • x!
​ n ​   ​​ ​ 99sin x
fi ​ lim 
    ​ ​ ​ ______
x   ​  ​ = 98
xÆ 0

(  ) [ (  ) (  ) (  )]
x
m 1 2 x–1
= ​ ​ ___ ​   ​ ◊ e–m ◊ ​   ​ ​ 1 – __
​ n ​  ​ ​ 1 – __
​ n ​  ​ ... ​ 1 – ​ _____
Also, ​( ____
lim ​   n    
​  ​  ​
sin x )
x! x Æ • x
​      
​  ​ > 1

= ​( ​ ___ ​  )​ ◊ e  ◊ 1
x
m –m
x!
fi ​ lim 
100x
​  [  ]
  ​ ​ _____
x Æ 0 sin x
 ​  ​ = 100
 ◊ ​( ​ ___ ​  )​
x
–m m
​ ​( ​[ _____
=
  ​  + ​[ ​ ______ ]​ )​
sin x ]
x! 100 x 99 sinx
Thus, ​ lim 
    ​   ​   
x ​  
xÆ 0
nksin (n!)
  ​  
15. ​ lim ​ ________ ​ , (0 £ k £ 1)

  = 100 + 98
n Æ • n + 1
= 198
(  nk
lim ​ ​ _____
= ​ ​      )
 ​  ​ × (​  ​   sin (n!) )​
lim ​   19. We have

( 
__ n
n Æ • n + 1
)
n Æ • n
  
a  – 1 + ​÷b ​
​ lim    ​ ​​ ​ __________
a  ​
    
  ​​ ​, where a, b > 0
n Æ •
= 0 × (limit does not exist) _
(  )
n n
=0 ÷    – 1
​ b ​
= ​    ​ 1 + ​ ______
lim ​ ​ a ​   ​​ ​
n Æ •
3.44  Differential Calculus Booster

[  { (  )
__
(  ) p
}]
n n
÷     – 1
​ b ​ 1/x
= ​e ​n Æ • ​​ _____
lim ​  
​   ​  a ​    ​​ ​
​ lim ​ ​​ tan ​ ​ __
= ​   ​   ​  – 1  ​ + (1 + x) a  ​  ​​ ​
x Æ 0 4
( 
1 b1/n – 1
)
lim ​ ​​[ tan ​{ ​( __
​   __
​    ​ ​ ​ ______
lim ​      
​  ​
​   ​  – 1 )​ + (1 + x)  }​ ]​​ ​
= e​ ​n Æ • a 1/n ​ p a 1/x
Let A = ​  
1
x Æ 0 4
__
​   ​  log b 1/a
= ​e​a ​ = e​ logb
​ ​= b1/a
log ​[ tan ​{ ​( __
20. We have
log A = ​   { 4   ​   ​  – 1 )​ + (1 + x)  }​ ]​
p
_______________________
lim ​ ​ ​      ​
   
x ​
a

}
(  )
ex x Æ 0
(1  + x) – e + ​ __ ​  1/x
2
  ​ ​ ________________
​ lim  ​   ​ 
     ​ Applying L’ Hospital rule, we get,

 { [ 
x Æ 0 2
x

= ​  
lim   ( 
e ​ 1 + __ ( 
x 11 2
​    ​ + ___
2 ) ex
​    ​ x + ...  ​  – e + ​ __ ​ 
24
​ ​ ___________________________
​        ​     ​
2
)
log A = ​  
x Æ 0
​ 
p
__
  
1
lim ​ ​ ______________________
     ​  ​
{ (  )
​ tan ​ ​ ​   ​  – 1  ​+ (1 + x) a  ​  ​
4 }] }
{ 
xÆ 0 2
x
p
× sec2 ​ __ }
​   ​  – 1 + (1 + x) a  ​ × a (1 + x) a – 1

(  )
4
11
e (​ ___  ​ x2 + (...)x3 + (...)x4 + ...) __
24 log A = 1 ◊ ​​( ​÷2 ​ 
   )​​​ ◊ a = 2a
2
= ​  
lim  ​ ​ ​  ________________________
    ​     ​
xÆ 0 x2
A = e2a
11e
= ____
​   ​  23. We have
24

x
​ cos ​ __
lim ​  
= ​   {  (  ) (  ) (  )
x
​    ​  ​cos ​ __
x
​    ​  ​cos ​ __
​    ​  ​ ... cos ​ __
x
​  n  ​ ​  ​(  )}
(  )
n Æ • 2 4 8 2
sin {x}
  ​ ​ ______
21. We have ​ lim  ​   ​  
 ​ As we know that
x Æ 0 {x}
Now, L.H.L. cos A ◊ cos 2A ◊ cos 22 A ◊ cos 23A ... cos2nA

(  )
n
sin {x} 1 sin (2 A)
= ​  
lim   ​ ​ ______
​   ​  
 ​ = __
​  n  ​  ◊ ​ ________ ​ 

x Æ 0 – {x} 2 sin A

(  )
x
sin (0  – h) Let ​ __n   ​ = A
lim ​ ​ _________
= ​   ​    
 ​  
​ 2
h Æ 0 (0 – h)
When n Æ •, A Æ 0
(  )
sin (–1)
= ​ _______
​ 
(–1)
 ​  

​ = sin (1)
  ​  
Thus, ​ lim 
n Æ •
x
​ cos ​ __ {  (  ) (  ) (  ) (  )}
x
​    ​  ​cos ​ __
2
x
​    ​  ​cos ​ __
4
​    ​  ​ ... cos ​ __
8
x
​  n  ​  ​  ​
2

lim ​​ c  os ​(__ ​      ​  c​ os ​ ____


​      ​  ​ ... cos ​( __
{ ​ 2 x   ​) c​ os ​(____
And, R.H.L
= ​  
x
) ( x
) x
}
​    ​ )​  ​

(  )
n
sin {x} A Æ 0 2 n – 1
2 n – 2 2
= ​ lim 
    ​ ​ ______
​   ​  
 ​
x Æ 0 + {x}
lim ​ {cos (A) cos (2A) cos (22 A) ... cos
= ​  

( 
A Æ 0

= ​  
h Æ 0
​  )
sin (0  + h)
lim ​ ​ _________  
(0 + h)
 ​ 
 ​= 1 (2n – 1 A)}

(  )
n
1 sin (2 A)
Since R.H.L = L.H.L, so limit does not exist. = ​   ​  n  ​  ◊ ​ ________
lim ​ ​ __    

​  ​
A Æ 0 2 sin A

 {
22. We have

= ​  
n Æ • [  {  (  ) } ]
p–4
lim ​  ​​ tan ​ ​ _____
4
 ​ 
1
 + ​​1 + __
a
​ n ​  ​​ ​  ​  ​​ ​
n = ​–  
 lim 
n Æ •
 ​ 
(n4  + 4n2 + 6n)
​  _____________________
   
  
(n4 +  4n2 + 6n + 1)1/2 + 1
 ​  ​
}
= ​  
n Æ • [  { (  ) (  ) } ]
p
lim ​  ​​ tan ​ ​ __
4
1 a n
​   ​  – 1  ​ + ​​ 1 + __
​ n ​  ​​ ​  ​  ​​ ​

A Æ 0 ( 
A sin (x)
​ x ​  ◊ ​_____
lim ​ ​ __
= ​    
sin A
 ​ 
 ​ )
Put n = 1/x = ​  
A Æ 0 ( 
sin (x) ____
lim ​ ​ _____
​  x ​ ◊ ​

A
      ​ 
sin A
 ​ )
when n Æ •, then x Æ  0 sin (x)
= _____
​  x ​   
The Limit  3.45

24. We have Hence, the polynomial f(x) = – x2 + 2x3.

   +​ ​ ​ ____________________________
​ lim 
x Æ p 
    
   
( 
2cot x  + 3cot x – 51 + cot x + 10
(4 )1/2  + (27cot x)1/3 – 5cot x + 20
cot x
 ​  ​
) 26. We have

  ​ ​{(  n6 + 6n4 + 12n3 + 1)1/3 – (n4 + 4n2 + 6n + 1)1/2 }​
​ lim 
n Æ •  

+
when x Æ p , cot x Æ + •
​{​(  n6 + 6n4 + 12n3 + 1 )1/3
lim ​  
= ​   ​ – n2}  ​

( 
n Æ •

)

2cot x +  3 cot x – 5.5cot x + 10 lim ​ ​{​(  n4 + 4n2 + 6n + 1 )1/2


​ – n2  }​
= ​ lim 
   +​  ​ ​ __________________________
   
     ​  ​ –  ​  
n Æ •
x Æ p  (22cot x)1/2 + (33cot x )1/3 – 5cot x + 20


( 
2cot x  + 3cot x – 5.5cot x + 10
   +​ ​​ ________________________
= ​ lim     
    ​  ​
x Æ p  (2cot x )  + (3cot x )– 5cot x + 20 ) n Æ • {  (n6 + 6n4 + 12n3 + 1) – n6
lim ​  ​ __________________________________________
= ​       
      ​  ​
(n6 + 6n5 + 12n4 + 1)2/3 + n4 + n2 (n6 + 6n5 + 12n4 + 1)1/3

(  (  ) (  )
) {  }
2 cot x 3 cot x 10 (n4  + 4n2 + 6n + 1) – n4
​​ __
​   ​   ​​ ​  + ​​ __
​   ​   ​​ ​ – 5 + ____ ​  cot x  ​  – ​   lim ​   ​ _______________________
​  4    
     ​  ​
5 5 n Æ • 2 1/2 2
___________________________ 5 (n   + 4n + 6n + 1) + n

( (  ) )

= ​ lim 
   +  ​ ​​   ​  ​
( (  ) )
        

{ 
2 cot x 3 cot x 20
​ ​​ __
​   ​   ​ ​  ​  + ​ ​​ __ ​    ​  ​ ​  ​ – 1 + ​ ____
x Æ p 

}
  ​

5 5 5cot x (6n4 + 12n3 + 1)
= ​   ​ __________________________________________
lim ​   ​  6             ​  ​
n Æ •
(n  + 6n4 + 12n3 + 1)2/3 + n4 + n2 (n6 + 6n4 + 12n3 + 1)1/3

( 

0  + 0 – 5 + 0
= ​​ ____________ )
{ 
  
   ​  ​ = 5

}
0  + 0 – 1 + 0
(4n2 + 6n + 1)
_______________________
25. We have – ​ 
  l im  ​
  ​​     
    ​  ​
n Æ •

{ ( 
(n4 + 4n2 +  6n + 1)1/2 + n2

(  )

1/x
x2 + f (x)
  ​ ​​ 1 + ​ _______  ​​ ​ = e2.

}
​ lim   ​  

(  )
x Æ 0 2
x 12 1
​ 6 + ___ ​ n ​ + ​ __4  ​   ​
It is possible only when n
​​  ________________________________________
( 
lim ​  
= ​        
     ​  ​
n Æ • 6 12 __ 1 2/3
) 6 12 __ 1 1/3
)
( 
__ ___ __ ___

)
​​ 1   + ​     ​  
  +  ​   ​ 
 +  ​    ​ 
  ​​ ​ 
+  1 + ​​ 1 + ​    ​
  + ​   ​
  + ​    ​ 
  ​​ ​

x2  + f (x) n2 n3 n 4 n 2 n3 n 4

 { (  ( 
​ ​  ________  ​    ​ = 0

}
x2

So, the least degree of f (x) is 2


– ​  
​ 4 + __
lim ​ ​ ​  ______________________
6
​ n ​ + __
   
n
   1/2
1
)
​  2  ​   ​
 ​  ​
Let 2
f (x) = a1x + a2 x + ... 3 n Æ • 4
__ __6 1
__
​​ 1  + ​  2  ​  + ​  3  ​ + ​  4  ​   ​​ ​ + 1

n n n )
(  + f (x) 1/x
)
2
x_______
Now, ​ lim    ​ ​​ 1 + ​   ​​ ​ = e2
 ​   6 4
x Æ 0 x2 = __ ​   ​  – __
​   ​ 
3 2

(  ) = 2 – 2
2
x  + f (x) __ 1
lim ​ ​ ​ _______
​    ​  
 ​ × ​ x ​
​e​x Æ 0 x2 ​= e2 = 0

(  )
x2  + f (x) 27. We have
  ​ ​ ________
(  )
​ lim  ​   ​    
​= 2
x Æ 0 x3 (1  + 3x + 2x2)1/x – (1 + 3x – 2x2)1/x
  ​ ​ ​ ____________________________
​ lim       x    ​  ​

(  )
x Æ 0
2 2 3
x   + a1x + a2 x + ...
  ​ ​ ​ ___________________
(  )
​ lim      ​     ​ = 2
x Æ 0 x3 {(1  + 3x + 2x2)1/x – e3} – {(1 + 3x – 2 x2)1/x – e3}
lim ​ ​ ​  ____________________________________
= ​         x     ​  ​

( 
x Æ 0

x Æ 0
(1  + a1)x2 + a2 x3 + ...
  ​ ​ ​  ___________________
​ lim     
x3
 ​     ​ = 2 ) = ​  
x Æ 0
( 
{​  (1  + 3x + 2x2)1/x – e3  ​
lim ​ ​ ​ ____________________
   
x    ​  ​
})
It is possible only when


(1 + a1) = 0, a2 = 2
a1 = –1, a2 = 2
= ​      
x Æ 0x 
( { 
​ (1  + 3x – 2x2)1/x – e3  ​
lim ​ ​ ​  ____________________
  ​  ​
})
3.46  Differential Calculus Booster

= (L1 – L2) (say) Now, L1 – L2

5e3 13e3
Now, = – ​ ___ ​  + ____
​   ​ 

2 2

L1 = ​  
x Æ 0
( 
​{ (1  + 3x + 2x2)1/x – e3  ​
lim ​ ​ ​  ____________________
   
x    ​  ​
} )
8e3
= ​ ___ ​  
2
= 4e3

​{  ln (1 + 3x
​e​ ​
+ 2x 2)
_____________
x     ​ 
e ​
lim ​ ​  ________________
= ​  
​– 3 }
28. We have
x     ​ 

( 
x Æ 0

= ​  
{​  ​e​ ​ – e  }​
    
2
x
(3 + 2x) – ​ __  ​(3 + 2x) 2
lim ​ ​  __________________
  ​
3 x Æ 0
1 –  cos (1 – cos x)
  ​ ​ ________________
​ lim  ​ 
x4
 ​ 
     ​ )
( 
x
(  (  ) )
)
x Æ 0
x
1  – cos ​ 2sin2 ​ __
​    ​  ​  ​


e​ 3 {  ​e​ 2
x
2x – ​ __  ​(3 + 2x) 2
1  ​
lim ​ ​  ________________
= ​          ​
​– } lim ​ ​ ________________
= ​  
x Æ 0
​ 
x4
 ​ 
    
2

x

(  (  (  ) )
)
x Æ 0
x
2sin2 ​ sin2  ​ __
{  }
x ​    ​  ​  ​
2x  – __ 2
e {​  ​e​ 2 ​ – 1 }​
x ​    ​ (3 + 2x)
32x – ​ __  ​(3 + 2x) 2
_____________ 2
2

lim ​ ​  ________________
= ​       ​× ​ ​ ______________
   = ​  
lim ​ ​ ​   ​      ​
    ​  
{  x4

( 
x x Æ 0
x Æ 0 x
​ 2x – __
​    ​ (3 + 2x)2  ​ }
(  (  )) ​ ________
sin   ​( __

)
2
​    ​ )​
x x
2sin2 ​ sin2  ​ __ ​    ​  ​  ​ 4

{  x
1 }​
2x – ​ __  ​(3 + 2x) 2 lim ​ ​ ​ ____________
= ​   x   
2
  
 ​ ×
2
   
 ​  ​
= ​  
e3​ ​e​ 2
_________________
lim ​​  
​– 1
    ​ × ​ 2 – __
   ​   ​  (3 + 2x)2  ​ {  } x Æ 0
sin4  ​ __
​    ​  ​ (  ) ​​( ​ 2  ​ )​​ ​ × 2
x
__ 4 4

}
2 2
{ 
x Æ 0
x
​ 2x  – __
​    ​ (3 + 2x)2  ​
2
2 1
= __
​  4  ​  = __
​   ​ 
(  ) 8
3
9 5e 2
= e3​ 2 – ​ __ ​   ​ = – ​ ___ ​  
2 2
29. We have
sin x
______
(  )
Again, sin x ​ x – sinx  
 ​
  ​ ​​ ​ ____
​ lim     
​ 
  ​​ ​

(  )
x
​{ (1 + 3x  – 2x2)1/x – e3  }​ x Æ 0
lim ​ ​ ​  ____________________
L2 = ​      
x    ​  ​
x Æ 0
sin x

(  ( 
_______
))
​     ​
sin x
lim ​ ​​ 1 + ​ ​ ____ ​​x – sin x ​
{​  ​e​   ​   – e  }​
= ​      

  – 1  ​ 
2
ln (1 + 3x – 2x ) x
​ ____________
x     ​  3 x Æ 0
lim ​ ​  ________________
= ​   ​ x
x Æ 0 sin x

(  ( 
_______
sin x – x ​ x – sin x   
))

lim ​ ​​ 1 + ​ ​ _______
= ​       ​  ​  ​​ ​

= ​  
{ 
​ ​e​ 2
x
(3 – 2x) – ​ __  ​(3 – 2x) 2
​ – e3  ​
lim ​ ​  __________________
       ​
} x Æ 0 x

x Æ 0 x
= e​ ​x Æ 0
( sin x – x ______
lim ​ ​ ​ _______
​   x    )
​  ​ × ​ 
sin x
  
 ​
x – sinx ​
{ 
e3​ ​e​ 2
x
– 2x – ​ __  ​(3 – 2x) 2
1  ​
lim ​ ​  _________________
​– }
= ​   x       ​ ​   (   sin x
lim ​ ​ – ​ ____ )
​  ​
x   
x Æ 0
= e​ ​x Æ 0 ​

} {  }
x
{  x
__
3 – 2x – ​ 2  ​(3 – 2x)
​ ​e​
e_________________
2
– 2x – ​ __  ​ (3 – 2x)2  ​
​ – 1  ​ ​_______________
2 1
= ​  
lim ​ ​     
   ​ × ​  ​ = e–1 = __ ​ e ​
x      
x Æ 0
{  x
__
​ – 2x – ​    ​ (3 – 2x)   ​
2
2
}
{  x
__ 2
} 30. Given ​ lim 
f (x)
  ​ ​ ____
​  x   ​  ​ = 1 (  )
{ 
3 – 2x – ​ 2  ​ (3 + 2x) – 1

}
 ​ ​e​
e________________ ​  ​ x Æ 0
1
= ​  
lim ​ ​      ​ × ​ – 2 – ​ __ ​  (3 – 2x)2  ​
x Æ 0
{  2
x
​ – 2x – ​ __  ​ (3 – 2x)2  ​ 2
} We have

3
(  9
__
)
= e ​ – 2 – ​   ​   ​ = – ​   ​ 
2
13e3
____
2

​ lim 
x Æ 0 ( 
x (1  + a cos x) – b sin x
  ​ ​ ​ ___________________
   
{f (x)}3
 ​     ​ = 1
)
The Limit  3.47

(  ) (  )
_____
x (1 + a cos x) – b sin x
  ​ ​ ​  __________________ (​  ​÷x  2 + 1 ​  – 1 )​
​ lim    
    ​  ​ = 1 lim ​ ​ ​ ____________
= ​     
   ​  ​
{  }
_____
f (x) 3
x Æ 0
____
​​ ​  x   
 ​  ​​ ​ × x 3 x Æ 0 (​  3x2÷​ x  2 + 1 ​  )​

​ lim 
x Æ 0 ( 
x +  ax cos x – b sin x
  ​ ​ ​ _________________
x3
 ​       ​ = 1
)
x Æ 0 ( 
x2  + _____
3x ÷
2
1–1
lim ​ ​ ​ __________
= ​  
​ x  + 1 ​
2
1
 ​ × ___________
    ​  _____     ​  ​
  (​  ​÷x
  + 1 ​ + 1 )​
2 )
​  (  ( 
x2 x4
2! 4!
  ​ ​ ___________________________________
​ lim       3
 ​ 
) ( 
x3
x + ax ​ 1 – ​ __  ​ + ​ __  ​ + ...  ​ – b​ x – ​ __  ​ + ...  ​
3!
    ​ = 1
) )
x Æ 0 ( 1
lim ​ ​ ________
= ​   ​  _____
   
3​÷x  + 1 ​
2
1
 ​ × ___________
​  _____     ​  ​
  (​  ​÷x
  + 1 ​ + 1 )​
2 )
( 
x Æ 0 x
1 1
) 1

( 
= ​ __ __ __
( 
)
​    ​ × ​   ​   ​ = ​   ​ 

​        ​ 
2!
a
(1 +  a – b)x + x  ​ – ​ __  ​  + __
  ​ ​ _____________________________
​ lim 
3
3!
b
​    ​   ​ + ...
   ​ = 1
) 3 2 6
2

( 
__
​  3  ​ 
x3
)
x Æ 0 2 x
3. Given ​ lim  ​  3  ​  (sin–1x – tan–1x)  ​​ ​ = e– m
  ​ ​​ __
x Æ 0 x
It is possible only when
x3 3x5

a
(1 + a – b) = 0 and ​ –  ​ __  ​  +
2! (  __b
)
​    ​   ​ = 1
3!
Now, sin–1x = x + ​ __ ​  + ___
6
​   ​ + ...
40
x 3 x5 x 7
5 3
On solving, we get, a = – ​ __ ​ , b = – ​ __ ​  and tan–1x = x – ​ __ ​  + ​ __ ​  – ​ __ ​  + ...
2 2 3 5 7
Now,


(a + b + 10)
5
= – ​ __ ​  –
3
__
​   ​  + 10

x3 __
6 3
x3
sin–1x –  tan–1x = ​ ​ __ ​ + ​ 
3x5
   ​   ​ + ​ ___
​   ​  –
5 (  ) (  x5
)
​ __ ​   ​ + ...
5

( 
2 2 2
__

)
​  3  ​ 
2
= – 4 + 10   ​ ​​ __
Now, ​ lim  ​  3  ​  (sin–1x – tan–1x)  ​​x ​ (1 • – from)
x Æ 0 x
=6

= ​e​
__
lim ​  
​  
x Æ • 2
2 2
x x3 ( 
​    ​ ​  __
​    ​  (sin–1 x – tan –1 x) –1  ​
​ )

( 
xx  – aa
  ​ ​ _______
1. ​ lim  ​  )
​   ​
  =
​2 
​e​
lim 
x Æ • ( 
 ​  
(sin–1 x – tan –1 x)– x 3
​​  ________________
x5
 ​       ​
​ )
x Æ a a x – aa

( 
1 1
2 × – ​ __ ​  __
– ​   ​  1__
) = ​e​ ​e​ 2 ​ = ​ ___
4​ =
xx (1 + log x)  –  0   ​ 
= ​   ​ ​ ​ ______________
lim     
    ​  ​ ​ e 
÷    ​
x Æ a ax log a 1
Thus, M = – ​ __ ​ 
2

( 
aa (1 + log a)
= ​ ​ ___________
  
aa log a
  ​  
​ ) Hence, the value of 2M + 2013 = 2012

(  1 n + ​xn​ ​
4. We have ​​ 1 + __ )
​ n ​  ​​ ​ = e

( 
(1  + log a)
= ​ ​ _________
log a
   
​  
​ ) (  1 n + ​xn​ ​
log ​​ 1 + ​ __ )
n ​  ​​ ​ = log (e) = 1

( 1
= ​ ____
​     ​ + 1  ​
log a )
1
(n + xn)log ​ 1 + __
​ n ​  ​ = 1 (  )
1
(n + xn) = __________
​       ​
= (logae + 1) 1
log ​ 1 + __
​ n ​  ​ (  )
(  )
_____
x  + ln​( ÷  – x )​
​ x  + 1 ​
2 1
  ​ ​ ​ _________________
2. ​ lim   ​ 
     ​ xn = __________
​       ​ – n
x Æ 0 x 3 1
log ​ 1 + __ ( 
​ n ​  ​ )

(  )
1


1   –  ​ _______
÷

_____

x
lim ​ ​ ​ ___________
= ​  
2
+
   
 ​
1 ​
 ​ 

     ​
1
Let ​ 1 + __ (  )
​ n ​  ​ = y fi ny = n + 1
x Æ 0 3x2 1
fi n = _____
​      ​
y–1
3.48  Differential Calculus Booster

  ​  
Now, ​ lim (xn)
n Æ •
( 
a ex + (a + c) sinx – c e– x
  ​ ​ ​ ____________________
​ lim        
  ​  ​ = 2 )
( 
x Æ 0 x cos x + sinx

n Æ 1 logy
l
lim ​ ​ ____
= ​  
1
​     ​ – _____
​     
y–1
 ​  ​ ) lim ​,  numerator (a – c) Æ 0 and the denomina-
As ​  
x Æ 0

= ​   ( 
y –  1 – log y
lim ​ ​ ​ ___________
   ​  ​
  
(y – 1)logy ) tor Æ 0

( 
n Æ1 So, a–c=0

)
a=c
1
1 – ​ __
y ​
= ​   ___________
lim ​ ​ ​ 
n Æ1 y – 1
_____
​   ​ 

    ​  ​
 + log y
(    ​ ​ ​ _________________
Thus, ​ lim 
x Æ 0
  
  
x cos x + sin x)
a ex + 2a sinx – a e– x
  ​  ​ = 2

(  )
1
  ​ ​ ( _________________
cos x – x sin x + cos x )
x  – x
a e + 2a cos x + a e
​ lim  ​     ​  ​ = 2
__1 x Æ 0
​  2  ​ 
y 1
lim ​ ​ ​ ______
= ​       ​  ​ = __
​    ​ a + 2a + a
n Æ1 __ 1 1
__ 2 ​ _________
 ​   
=2
​  2  ​  + ​ y ​ 2
y
4a = 4

( ÷  )
_________________
1 – cos (ax2 + bx + c) a=1
  ​ ​ ​ ​  _________________
5. We have ​ lim      
 ​ ​ 
   ​ c = 1, b = a + c = 2
x Æ a
(x – a)2
Hence, the value of a3 + b3 + c3 + 10

( ÷  )
_____________________
1 – cos {a (x – a) (x – b)}
lim ​ ​ ​ ​  _____________________
= ​      
     ​ ​ 
   ​ = 1 + 8 + 1 + 10
x Æ a
(x – a)2
= 20

( ÷  {  )
____________________
7. We have


a (x – a) (x – b)
2sin2 ​ ​  _____________
2
 ​ 
    
lim ​ ​ ​ ​  ____________________
= ​        ​ ​ 
  


} ​ lim  ​ ( 
1 – cos x + 2 sin x – sin3 x – x2 + 3x4
  ​ ​ _______________________________
          ​ ​ )
    x Æ 0 tan3 x – 6 sin2 x + x – 5x3

 (÷ 
x Æ a
(x – a)2

( 
) )
______________________
sin x + 2 cos x – 3 sin2 x cosx – 2x + 12x3

= ​  
{ 
a (x – a) (x – b)
2sin  ​ ​  _____________
2

   
2
   
   
 ​ 
lim ​ ​ ​ ​  ______________________
     ​
 ​ ​  ​
} lim ​ ​ ​ _________________________________
= ​  
x Æ 0
        
3 tan2 x sec2 x – 6 sin 2x + 1 – 10x
  ​  ​

– a)2 (x – b)2
(  )
x Æ a = 2
(x_____________ –b 2
x_____
​   ​
     × ​​ ​   ​   ​​ ​
4 2 Hence, the value of M + 10 = 12.

÷  (  ) [  ]
_________
x2
a–b 2 lim ​ ​ ​ ________
8. We have L = ​       ​  ​
= ​ 2 ​​ ​ _____
 ​  
 ​​ 
​ ​ x Æ 0 sin x tanx
2


1
= ___
​  __  ​  | (a – b)|
÷   
​ 2 ​
= ​   [ 
x Æ 0 sin x
x
lim ​ ​ ____
x
​ × ____
​       ​      
tan x ]
​  ​

As we know that
6. We have ​ lim 
x Æ 0
​  ( 
aex – b cos x + ce– x
  ​ ​ _______________
x sin x
  ​  ​ = 2
     ) ​ ____
x
sin x
​ > 1 and ____
    
x
​      
tan x
​< 1

As x Æ 0, numerator (a – b + c) Æ 0 and the x ____


fi ​ ____
x
​ ◊ ​      
     ​< 1
denominator Æ 0 sin x tan x

So, (a – b + c) = 0
x ____
fi ​ ____
x
[ 
​ ◊ ​      
​      
sin x tan x
​  ​ = 0 ]
b=a+c
x
  ​ ​ ____ [ 
x
​  × ____ ]
( 
Thus, ​ lim  ​       ​      
​  ​ = 0
Thus, ​ lim 
   
x Æ 0
aex –  (a + c) cosx + ce– x
​ ​ ​ ____________________
   
x sin x
    ​  ​ = 2 ) x Æ 0 sin x tan x

Hence, the value of L + 2013 is 2013.


The Limit  3.49

9. Given P (x) = a1 x + a2 x2 + ....... + a100  x100


________
=  ( (27x  – 1) (9x – 1) (3x – 1)
​ lim ​ ​ ______________________
 ​      ​     ​ )
(  ) x3
x Æ •
÷​ 1  + P (x) ​ 
100
​ lim ​ ​ ___________
L=  ​  x       ​ – 1  ​
((   ) (  ) (  ))
x x
x Æ 0 27x – 1 9_____ – 1 3_____ –1
​ lim ​ ​ ​ ​ ______
=     

​  ​ ​ ​  x   
​  ​ ​ ​  x  ​  
 ​  ​
( 
x
)
x Æ •
1
​ 1 + ____
​     ​  P(x) +  ...  ​  –  1
100 = log(27) ◊ log(9) ◊ log(3)
​ lim ​ ​  ____________________
=     
x    ​
x Æ 0
= log(33) ◊ log(32) ◊ log(3)
​ lim ​ ​  ____
= 
x Æ 0 100
( 
a1x  + a2 x2 + a3 x3 + ... + an xn
1 _________________________
   ​ ​ ​     
x     ​  ​ ) = 6{log(3)}3
1
= ​ lim ​ ​  ____
   ​  (a1 + a2 x + a3 x2 + ... + an xn – 1) So, K = 6, M = 3, N = 3
x Æ 0 100
Hence, the value of K2 + M2 + N2.
a1
= ____
​    ​  = 36 + 9 + 9
100
1 = 54.
= ____
​     ​ 
100 12. We have
Hence, the value of (1000 L + 2007)
= 10 + 2007
  –​ ​__________________________
​ lim 
x Æ1
​     
(x – 1)2 ( 
(3 + a x)5/2  – b ln x + c sin (x – 1)
 ​     ​ = 2
)
= 2017
It is possible only when (3 + a)5/2 = 0
10. We have L =  
100 x
​ lim ​ ​ _____
​ 
x Æ 0 sin x [  ]
 ​  ​

(a + 3) = 0
a = – 3
= 100, since ​
(  ​  x   ​ > 1  ​
____
sin x ) ( 
(3  – 3x)5/2 – b ln x + c sin (x – 1)
  –​  ​ _________________________
​ lim 
x Æ1
​     
(x – 1)2
 ​     ​ = 2
)
[  ]
(  )
99 sin x
and ​ lim ​ ​ ______
M=  ​  x    ​  ​ 5 b
​ __ ​  (3  + ax)3/2 – __
​ x ​ + c cos (x – 1)
x Æ 0 2
  –​ ​ __________________________
​ lim  ​       ​     ​ = 2
x Æ1 2 (x – 1)

sin x
= 98, since ​ ____
​  x   
​ < 1  ​ (  ) It is possible only when – b + c = 0
Hence, the value of L + M + 2 b=c



= 100 + 98 + 2
= 200. ​ lim 
5 3
__
  –​ ​ ​ 
x Æ1
( 
​   ​  ◊ ​ __  ​ ◊ a2 (3  + ax)12 + __
2_______________________________
2
    
2
x
 ​
b
​  2  ​  + c sin (x – 1)
     ​ = 2 )
x Æ • ( 
729x  –  243x – 81x + 9x + 3x – 1
  ​ ​ ____________________________
11. We have ​ lim  ​      
x3
 ​     ​ ) b
It is possible only when __
​   ​  = 2
2
b=4
= 
x Æ • (  243x (3x – 1)  – 9x (9x – 1) + (3x – 1)
​ lim ​ ​ ______________________________
​      
x3
 ​     ​ ) Thus, a = – 3, b = 4 = c
Hence, the value of a2 + b2 + c2

= 
x Æ • (  243x (3x  – 1) – 9x (3x – 1) + (3x – 1)
​ lim ​ ​ ​ _______________________________
    
x3
 ​     ​ )

= 9 + 16 + 16
= 41

(  ) ( 
x  x x x
(243 – 9  (3 + 1) + 1) (3 – 1)
​ lim ​ ​ ___________________________
= 
x Æ •
​      
x3
 ​     ​ (cos x  – 1) (cos x – ex)
​ lim ​ ​ ​ __________________
13. Limit =  
x Æ 0
   
xn
    ​  ​ )
(  )
(  )
243x  – 9x ◊ 3x – (9x – 1) (3x – 1)
​ lim ​ ​ ___________________________
= 
x Æ •
​      
x3
 ​     ​
 ​ ________
(  ) ( 
2 sin2 ​ __
2
 ​ 
x
​    ​  ​ x

(e – 1) 1_______
​ ​ _______
    ​ + ​ 
– cos x
)
​  ​
2 x x   

( 
x
__
)
x x
(27  (9   – 1)  –  (9 – 1))(3 – 1) x x ​   ​ 
​ lim ​ ​ ​  __________________________ 4
=      ​     ​ ​ lim ​​ ​  __________________________
=       ​     ​
x Æ •
x3 x Æ 0 xn – 3
3.50  Differential Calculus Booster
1
It will provide us a finite limit only when n – 3 = 0
= (x2 e x​)10​ ​​  – 2 ​Ú ​  ​ xex dx
Thus n = 3. 0

14. We have = (x2e x​)10​ ​​  – 2 (xex – e x​)10​ ​​ 

x Æ 0
​ 
x4
 ​   
( 
cos 4x  + a cos 2x + b
  ​ ​ __________________
​ lim     ​
)

= e – 2 (e – e) + 2 (0 – 1)
= (e – 2).
Thus, L = 1, M = – 2
Since it has a finite limit, so a + b + 1 = 0 ...(i)
Hence, the value of L + M + 10 is 9.

​ lim 
x Æ 0 4x3
 ​ 
  
( 
– 4 sin 4x – 2a sin 2x
  ​ ​ ​ ________________   ​
) 16. Limit

= 
( 
– 16 cos 4x  – 4a cos 2x
​ lim ​ ​ ___________________
​       ​     ​
)
=  
​ lim 
n Æ • ( 
a – 1 
1a + 2a + ... + na
​ ​ ​ ___________________________________
     ​  ​
   
(n + 1) ((na + 1) + (na + 2) + ... + (na + n)) )
x Æ 0 12x2

(  )
n
Since, it has a finite limit, so – 16 – 4 a = 0 2 ​S  ​ ​  r a
=   ______________________
​ lim ​ ​  ​ 
r = 1
     ​  ​
  
4a = – 16 n Æ •
(n  +  1)a – 1 (2n2 a+ n2 + n)

( 
fi a = – 4

)
n
when a = – 4, b = 3 2 ​S  ​ ​ ​ ​​ __
r a
(  )
​ n ​   ​​ ​
​ lim ​ ​ _____________________
r = 1
=   ​     
    ​  ​
Hence, the value of a2 + b2 + 10
= 16 + 9 + 10
n Æ •
( 
1 a – 1
) ( 
​​ 1  + ​ n ​  ​​ ​ ​ 2a + 1 + __
__ 1
​ n ​  ​ )

(  )
= 35. 1

(  ( 
2 ​Ú ​  ​ xa dx
)
n n
Ck
  ​ ​ ​S  ​ ​ ​ ​ ________
15. We have ​ lim  ​  k   
 ​  ​  ​ = ​ _______
​ 
0
 ​ 
  ​
n Æ • k = 0
n (k + 3) 2a + 1

(  )
Ck 1
n
n 2 1
​ lim ​  ​ S  ​ ​ ​ ​ ___
=  ​  k ​  ​ ​Ú ​  ​  x k + 2dx Thus, ​ _____________
     ​ = ___
​    ​ 
n Æ • k = 0 (2a + 1) (a + 1) 60
n 0

(  )
1 n n fi (2a + 1) (a + 1) = 120
Ck
= ​Ú ​  ​  ​  
lim ​  ​ S  ​ ​ ​ ​ ___
​  n  ​  ​ ◊ x k + 2dx
0
n Æ • k = 0 k fi 2a2 + 3a – 119 = 0

{  (  ) }
1 n
fi 2a2 + 17a – 14a – 119 = 0
n Ck
= ​Ú ​  ​  x2 ​  
​ lim ​  ​ S  ​ ​ ​ ​ ___
​  k ​  ​ ◊ x k   d​ x fi a(2a + 17) – 7 (2a + 17) = 0
n Æ • k = 0
0 n
fi (2a + 17) (a – 7) = 0

{  (  (  ) )}
1 n
x k
= Ú​  ​  ​   x  ​  
2
​ lim ​    ​S  ​ ​ ​ Ck ​​ __
n
​ n ​  ​​ ​  ​  d​ x 17
0
n Æ • k = 0 fi a = 7, – ​ ___ ​ .
2
1

{  ( 
( 
17. Given
x n
)}
)
= Ú​  ​  ​  x2 ​  
​ lim ​ ​​ 1 + ​ __

0
n Æ • n  ​  ​​ ​   ​dx
L=  ________
​ lim ​ ​ ​ 
1
x4 sin  ​ __
​ x ​  ​
 ​ 

  ​
(  )
1 x Æ •
1 + |x|3
{   ​ lim ​ ​​ ( 1 + ​   ​ )​​​​ }​dx
x n
 __

= Ú​  ​  ​  x2 ​ ​e n Æ •
​ n

0 Put x = – y
1
= Ú​  ​  ​  x2 ​{ ​en Æ •
​ lim  ​ 

​ (  x
n  ​ 1 + __ ) }​dx
​ n ​ – 1  ​
​ when x Æ – •, than y Æ •

(  )
0


1
= Ú​  ​  ​  x  e   dx
2 x
Thus, ​ lim ​ ​ _________
L=  ​ 
1
y4 sin ​ – ​ __
y ​  ​
 ​ 

  ​
(  )
0 y Æ •
1 + |– y|3
The Limit  3.51

( 
​ lim ​  ​ _________
=  ​ 
1
– y4 sin ​ __
 ​ 
(  )
​ y ​  ​

  ​ ) x Æ • ( 
((x2 +  a2 )(x2 + b2)  – (x2 + c2) (x2 + d2))
​ lim ​ ​ ​ _________________________________
=   ______________    
  2 + a2) (x2  
÷​ (x
    
+ b2) ​ + ÷
______________  ​  ​
  2 + c2) (x2   
​ (x + d2) ​ )
 ( )
y Æ •
1 + y3

– y sin ​ __
1
(  )
​ y ​  ​
( 
((a2 + b2) x2 + a2b2  – ((c2 + d2) x2 + c2d2))
​ lim ​ ​ __________________________________
=   ​  ______________
     ______________  ​  ​
)
 (
​ lim ​ ​ _________
=  ​   ​   
– 1
​ = ___
​   ​ = – 1
x Æ •
  2 + a2) (x2  
​÷(x + b2) ​  + ÷   2 + b2) (x2  
​ (x + d2) ​
y Æ • __1 1

)
​  3  ​  + 1
y
( 
​ (a2 + b2)  + ____ ( 
a2b2
))
 – ​ (c2 + d 2) + ____
c2d 2

(  )
​  2 ​  ​  2 ​  
 ​  ​

(  )
x2 x
___________________________________ x

÷(   ) (  ) ÷(   )(  )


2
t  ​ lim ​ ​ ​  _______________
=       
     ______________  ​  ​
​Ú__ ​  ​ ​tan– 1 ​ ​ _____    ​  ​ dt x Æ •
a2
__ b2 c2 d2
Also, M= 
​÷_______________
​ lim ​ ​ ​ 
  ​ 
x  1 + t2
  ​  ​
     + ​  2 ​    ​ ​  + ​ ​ 1 + __
​ ​ 1 + ​  2 ​    ​ ​ 1    __ + __
​  2 ​     ​​ 1    ​  2 ​   ​ ​
x Æ 0 sin 2x x x x x

(  (  ) )
= (a2 + b2) – (c2 + d2)
x2
tan–1 ​ _____
​   2 
x
 ​  ​ 2x – tan– 1 ​ _____
​     
1
( 
​   ​ ◊ ​ ____ )
   ​ 
__

{ 
1+x 1 + x 2​÷x    ​ 

}
= ​ lim ​ ​ ​  ______________________________
      ​ 
    ​ (1  – x) (1 – x2) ... (1 – x2n)
x Æ 0 2 cos (2x)   ​ ​ ​  ________________________
20. ​ lim     
    ​  ​
x Æ 1 {(1  – x) (1 – x2) ... (1 – xn)}2
0
= __
​   ​  = 0

{ 
2
Hence, the value of L + M + 11 = – 1 + 0 + 11 = 10. = ​   x Æ 1
(1 –  x) (1 – x2) ... (1 – xn) (1 – xn +2) ... (1 – x2n)
lim ​ ​ _____________________________________
​            
{(1 – x) (1 – x2) ... (1 – xn)}2
 ​  ​
}
(  ) { 
2n
18. Given k= 
n Æ • n 5

1
n3x
​ lim ​ ​ ​Ú ​  ​ ​ ​ _____
x +1
   ​  ​ dx
=  
x Æ 1
(1  – xn + 1) (1 – xn + 2) ... (1 – x2n)
​ lim ​ ​  ____________________________
​          
(1  – x) (1 – x2) ...(1 – xn)
 ​  ​
}
n = __
{ 
put ​ y ​

when n Æ •, then y Æ 0 x Æ 1


(1  – xn + 1) (1 – xn + 2) ... (1 – x2n)
​ lim ​ ​  ​  ___________________________
=           
(1  – x) (1 – x) ... (1 – x)
 ​  ​ }
{ 
(  ) }
(1  – x)(1 – x) ... (1 – x)

( 
2/y
​ lim ​ ​ _____________________
×   ​     
    ​  ​

​ lim 
1/y x + 1
____________
  ​ ​  ​ 
5
x
​Ú ​ ​ ​  ​ _____
   
 ​  ​ dx
 ​ 
     ​
) x Æ 1 (1  – x) (1 – x2) ... (1 – xn)

1 1 1
y Æ 0 y3 = (n + 1) (n + 2) ... (2n) × 1 × __
​   ​  × __
​    ​ ×...× __
​ n ​
2 3

(  (  ) (  ) )
2/y 1/y (n  + 1) (n + 2) ... (2n)
 ​ ________
  
 2
 ​ ​ – ​ __2  ​   ​ – ________
​  5    
1
 ​ ​ – ​ __2  ​  ​ = ​ __________________       ​
5 1 ◊ 2 ◊ 3 ... n
32/y   + 1 y (1/y ) + 1 y
  ​ ​ ​  _____________________________
​ lim        ​     ​
y Æ 0 3y 2 {(1 ◊ 2 ◊ 3 ... n ◊ (n  + 1) (n + 2) ... (2n)}
= ​ ______________________________
         ​
{1 ◊ 2 ◊ 3 ... n}2

(  )
– 4   _____
______ 1
​   5 
 ​+ ​    5 
 ​ (2n)!
32 + y 1+y
_______________ = _____
​  2 ​ 
​ lim ​ ​ ​ 
=   ​       ​ (n!)
y Æ 0 3


7
= ___
​    ​ 
24
4x  + 4– x
  ​ ​ ________
21. We have ​ lim  ​  x
x Æ • 4   – 4– x
 ​  
​ (  )
1
Now, ​ __ [  ]
​   ​  ​ + 2013 =  ( 
1  + 4– 2x
​ lim ​ ​ ________
​    ​  ​)
 ( ((   )) )
k x Æ •
1 – 4– 2x

= [​  ___
​   ​  ]​ + 2013
1 2x

24 1  + ​​ __
​   ​   ​​ ​
4
7 ​ lim ​ ​ _________
=  ​   
 ​  ​
x Æ • 1 2x
__
= 3 + 2013 = 2016 1 – ​​ ​    ​  ​​ ​
4
19. We have

​ lim 
x Æ •
( 
______________
  2 + a2) (x2   
  ​ ​ ​÷(x + b2) ​ – ÷   2 + c2) (x2   
​ (x
______________
+ d2) ​  ​ )
= 
x Æ • 1 – 0( 
1  + 0
​ lim ​ ​ ______
​    )
 ​  ​ = 1
3.52  Differential Calculus Booster


  ​ ​
22. We have ​ lim 
n

n Æ • r = 3

= 
r3 – 8
 P   ​ ​ ​ ​ _____
r3 + 8

n
 ​  ​(  )
P  (  )
r3 – 8
​ lim ​ ​   ​ ​ ​ ​​ _____   ​  ​
x Æ •
 { (  (  ) )(  (  ) ) }
2 sin2 ​ __
​ lim ​ ​  32 ​ ​ _________
=   ​  4
x
___
x2
​   ​   ​ 2 sin2 ​ __

​    ​ × 16
16
4
 ​ 

  ​ ​ _________
​  4
x
___
x2
​   ​   ​

​    ​ × 64
64
8
 ​ 

  ​  ​  ​

n Æ • r = 3
r3 + 8
1 1
= 32 × 4 × ___
​    ​ × ___
P ( 
​    ​ 
= 
n Æ • r =3 3
r –2n

r +2 )
    ​ ​ ​​ ______3 
​ lim ​  ​  ​  ​
3 3

1
= __
​   ​ 
16 64

8
P (  ) (  r2  + 2r + 4
)
n
r – 2 __________
​ lim ​ ​   ​ ​ ​ ​ _____ 
=   ​  ​ ​ ​  2    ​  ​ 24. We have
n Æ • r = 3 r + 2
r – 2r + 4
  ​   ​[ ​{ (x + p) (x + q) (x + r) (x + s) }1/4
​ – x ]​
(  ) (  )
​ lim 
n
r–2 n
r2  + 2r + 4
lim ​  ​ P   ​ ​ ​ ​​ _____   P   ​ ​ ​ ​​ __________
x Æ •
= ​   lim ​ ​
 ​  ​ × ​     
   ​  ​
n Æ • r = 3 r + 2 n Æ • r = 3 r  – 2r + 4
(x +  p) (x + q) (x + r) (x + s) – x4

{ 
​ lim ​ ​  ________________________________
}
1 2 3 (n  – 3) (n – 1) =       
      ​
_________________________
​ lim ​ ​ __
=  ​   ​  ◊ ​ __ ​  ◊ ​ __ ​  ... ​ _____________
n Æ • 5 6 7
  
   ​  ​
(n  + 1) (n + 3)
x Æ •
{​  ÷​  (x 
  + p)(x + q) (x
    + r) (x + s) ​ + x2  }​

{ 
1
19 28 39 (n2 + 3)   ×  ​ __________________________________
    
    ​
  × ​ ___
​   ​ × ​ ___ ​ × ​ ___ ​ × ... × ​ ___________
  
  ​  [​  {​  (x  + p) (x + q) (x + r) (x + s) }1/2
​ + x ]​
7 12 19 2
(n – 2n + 7)


(n2  +  2n + 4)
× ____________
​  2   
   ​  ​ =  
(n – 2n + 7) x Æ • } ​{ x2 +  (p + q) x + pq }​​{ x2 + (r + s) x + rs }​– x4
​ lim ​ ​  _____________________________________
         
_________________________
​{ ​  (x 
  +  p) (x + q) (x
    ÷
 ​
+ r) (x + s) ​  + x2  }​ 

= 
n Æ • { 
1 ◊ 2 ◊ 3 ◊ 4 ... (n2  + 3)(n2 + 2n + 4)
​ lim ​​ ​ ___________________________
         ​ ​
(n  – 1) n (n + 1) (n + 2) ... 7.12 } 1
  × _________________________________
​      
  
[​  {​  (x  + p) (x + q) (x + r) (x + s) }1/2
 ​
​ + x ]​


2
= __
​   ​ ​    {  (n2  + 3)(n2 + 2n + 4)
lim ​ ​ ​ ____________________
   
    ​  ​
7 n Æ • (n  – 1) n (n + 1)(n + 2) } (p  + q + r + s) x3 + (...)x2 + (...)x + pqrs
​ lim ​ ​  __________________________________
=       
      ​

{ 
x Æ • _________________________
{​  ÷​  (x  }
}
  + p) (x + q) (x 2

(  ) ( 
    + r) (x + s)  ​+ x   ​


2
= __
​ 1  + __
3
​  2  ​     ​ ​ 1 + __
n
lim ​ ​ ____________________
​   ​ ​    ​      ​  ​
2
​ n ​ + __
4
)
​  2  ​   ​
n 1
  × _________________________________
​      
    ​
7 n Æ •
( 
1
​ 1  – __ )( 
​ n ​  ​​ 1 + __
1
)( 
​ n ​  ​​ 1 + __
2
)
​ n ​  ​ [​  {​  (x  + p) (x + q) (x + r) (x + s) }12​ + x ]​
p +  q + r + s
= __ { 
2 (1  + 0)(1 + 0 + 0)
​   ​ ​  ​  __________________
       ​  ​
7 (1  – 0)(1 + 0)(1 + 0) } = ____________
​ 
2×2
 ​
    

(p  + q + r + s)
2 = _____________
​   ​     
= __
​   ​  4
7
23. We have 25. We have ​ lim 
f (x)
  ​ ​ ____
​  x    (  )
​  ​ = 1

{  (  (  ) (  ) (  ) (  ))} 2 2 2 2 x Æ 0
32 x x x x

( 
  ​ ​  ___
​ lim    1 – cos ​ __
​   ​  ​ ​   ​    ​ – cos ​ __
​   ​    ​ + cos ​ __
​   ​    ​cos ​ __
​   ​   ​ ​  ​
x Æ 0 x8 2 4 2 4 x (1  +aq cos x) – b sin x
​ lim ​ ​ ___________________
and   ​       ​ 
   ​ =1
)
{  (  (  ) (  ) {  (  )})}
x Æ 0 {f (x)}3

 (
2 2 2
32 x x x
​ lim ​ ​ ___
=   ​  8 ​ ​ 1 – cos ​ ​ __ ​    ​ – cos ​ __
​   ​   ​ ​ 1 – cos ​ __
​   ​  ​  ​  ​  ​
2 4  4 

)
x Æ •
x
x + a cos x – b sin x
  ​ ​ ​ ________________

{  ( (  (  ) ) (  (  ) ) ) } {  }
​ lim    
    ​  ​ =1
32 x 2
x 2 x Æ 0 f (x) 3
____ 3
​ lim ​ ​  ___
=   ​  8 ​ ​  ​ 1 – cos​  ​ __ ​    ​  ​ ​ 1 – cos ​ __
​   ​    ​  ​  ​  ​ ​​ ​  3 ​  ​​ ​ × x
x Æ • 2 4 x

( 
x

x Æ •
32
​ lim ​ ​  ___
=  
x {  ( (  (  ) ) (  (  )))}
x2 
​  8 ​​   ​ 2 sin2 ​ __
4
x2
​   ​    ​  ​ ​ 2 sin2 ​ __
​   ​    ​  ​  ​  ​
4 ​ lim 
x Æ 0
​ 
x2 x4
x + ax ​ 1 – __
2! 4! ( 
​    ​ + __
  ​ ​ ___________________________________
    
f (x) 3
​​ ____ {  }
​   ​​ ​× x3
​  x   
) ( 
 ​ 
   
x3 x5
​    ​– ...  ​– b ​ x – __
​    ​+ __
3! 5!
​    ​ –  ​ )
​= 1
)
The Limit  3.53

(  (  )
)
a b Put sin x = t
1  +  a – b) x + ​ – ​ __  ​ + __ ​    ​   ​x3 +  ...
2! 3! when x Æ 0, t Æ 0
  ​ ​ ​ ____________________________
​ lim           ​  ​ = 1
x Æ 0 f (x)
____
​​ ​  x    {  }
3
​   ​​ ​ × x3
​  
lim 
t – tt
​ ​​ __________   ​   ​
  (  )
t Æ 0 1  – t + log t

( 
It is possible only when


a
(1 + a – b) = 0 and ​ – ​ __  ​ + __
2! 3!
b
​    ​   ​ = 1 (  ) = 
​ lim 
t Æ 0
1  – tt (1 + log t)
​ ​ ______________
​ 
1
– 1 + __
    ​ 
​   ​ 
   ​
)
( 
On solving, we get, t

)
5 3 1
a = – ​ __ ​ , b = – ​ __ ​  0 – tt × ​ __ ​  – tt (1 + log t)2
2 2 t
_____________________
= 
​ lim ​ ​ ​       ​     ​
1
– ​ __2  ​ 
t Æ 0
Hence, the value of (a + b + 10)
t
= – 4 + 10 = 6.
– 1 – 1
= ​ ______ ​ 
 = 2
26. We have – 1
28. We have

( 
ax ex  – b log (1 + x) + cx e– x
  ​ ​ ________________________
​ lim  ​          ​   ​ = 2 ) ( 
1 –  (f (x))3
​ lim ​ ​ _________ )
 (
x Æ 0 x2 sin x M=  ​      ​  

 (
x Æ 0 5 tan2 x

) )
ax ex  – b log (1 + x) + cx e– x
  ​ ​ ________________________
​ lim  ​       ​ 
   ​ = 2 (1  – f (x)) (1 + f (x) + f 2 (x))
x Æ 0
sin x
3 ____
x ​ ​     
x
​  ​ (  )
​ lim ​ ​ ​  _______________________
=  
x Æ 0
   
tan2 x 2
5 ​ _____
​  2 ​  
x
 ​ x (  )
 ​ 
   ​

(  )
x – x
ax e   – b log (1 + x) + cxe

( 
  ​  ​ ​  _______________________
)
​ lim      ​     ​ = 2 3 (1  – f (x))
x Æ 0 x3 ​ lim ​ ​ __________
=   ​   ​ 
     ​

(  )
x Æ 0 5 x2
(  x2 x3
ax ​ 1 + x + __
​   ​ + __
2! 3!   ) ( x2 x3
​   ​ + ...  ​– b ​ x – __
​   ​  + __
2
​   ​  –   ​
3 ) 3
= __
( 
1 –  cos 2x cos 4x cos 6x cos 8x cos 10x
lim ​ ​ ________________________________
​   ​ ​    ​        ​     ​
)
( 
 ​          ​ 5 x Æ 0 x2
x__2 __ x3
+ cx ​ 1 – x + ​   ​  – ​   ​ ! + ...  ​ )
x Æ 0
 ​ ​
2
  ​​ ____________________________________
​ lim       
3
x3
 ​    ​  ​= 2 = __ ( 
3 22 +  42 + 62 + 82 + 102
​   ​ ​  _____________________
5
​     
2
 ​     ​ )

(  (  )
) (  )
b 3 12  + 22 + 32 + 42 + 52
(a – b + x) x + ​ a    + __ ​    ​ – c  ​ x2 = __
​   ​  × 22 ​ ​ ___________________
    ​     ​
  2      5 2
 ​ ​
( 
  

​ lim 
a __
__
2 3 2
b __
  ​ ​ ​ ​_________________________
c 3
   
)
+ ​ ​   ​  – ​   ​  + ​    ​  ​ x + ...
 ​    ​ ​ = 2
12 5 ◊ 6 ◊ 11
= ___
​   ​ × _______
​   ​ 

5 12
x Æ 0 x3
= 66
It is possible only when ______
(a – b + c) = 0, ​ a + __
b
​   ​  – c  ​ = 0
2 (  ) Hence, the value of (​  ​÷ M  + 1 )​
  – 2 ​
______
a b c   – 2 ​
= ​÷ 66   +1=8+1=9
and ​ __ ​  – __
​   ​  + __
​    ​ = 0
2 3 2 29. As we know that
On solving, we get, a = 3, b = 12, c = 9 sin– 1 x
sin– 1 x > x fi ______
​  x    ​ > 1
a+b+c
Hence, the value of ​ ​ ________
8
 ​   ( 
3 + 12 + 9
 ​ = ​ _________
8
 ​  =3 ) we have

27. We have ​ lim 


x Æ 0
( [ 
sin– 1 x
  ​ ​ ​ ______
​  x   
​ ] [ 
22 sin– 1 (2x)
​  ​  ​+ ​ __________
​  x       
​  ​ + ​ ] [ 
32 sin– 1 (3x)
__________
​  x     ​  ​ ]
   ( 
sin x – (sin x) sin x
  ​ ​ ​ _________________
​ lim      ​  ​
x Æ 0 1 – sin x + log (sin x) )   + ... +  ​​ __________

​  [ 
n2 sin– 1 (nx)
x       ​  ​  ​ = 100 ])
3.54  Differential Calculus Booster

(  {  }
)
(13 + 23 + 33 + ... + n3) = 100 1
__ n (n + 1) (2n + 1) n (n + 1)
​    ​ (n + 1)​ ​  ______________ ​      + ​ _______
 ​   ​
2 6 2
{  }
   

{ ( 
 ​      ​
n (n + 1) 2
​​ ​ _______
2
 ​  
 ​​ ​ = 100
=   ​ ​
1
__
– ​   ​  ​
2
n (n
  ​​ ​ 
+ 1) 2 n (n
_______
2
 ​   )
 ​​ ​ + ​ 
​ lim ​ ​ ___________________________________
      ​ 
+ 1)(2n + 1)
______________
6
 ​ 
   
     ​
​ ​
}
​​{ ​ _______
n Æ •
n4
n (n + 1)
2 }​​ ​ = 10
 ​  

2

1 1 4–3 1
= __
​   ​  – __
​   ​  = ​ _____
 ​ 
 = ___
​    ​. 
n (n + 1) = 20 6 8 24 24
n = 4 32. We have

( 
30. Now, x + sin x – x cos x – tan x


= x (1 – cos x) – tan x (1 – cos x)
= (x – tan x) (1 – cos x)
​ lim 
n Æ •
14 +  24  +  34  +  ...  +  n4
  ​ ​ ​ _______________________________________
    
       ​  ​
(12 + 22  + 32 +....+ n2) (13 + 23 + 33 +.....+ n3) )
x Æ 0
​ 
xn ( 
(x  – tan x) (1 – cos x)
  ​ ​ __________________
So, ​ lim         ​  ​ ) INTEGER TYPE QUESTIONS

( (  ) (  ) ) ( 
ln (1  + x + x2 + ... + xn)
)
x – tan x 1_______– cos x
​ ​ _______
 ​  
 ​ ​ ​   ​   ​   ​ ​ _____________________
1. We have ​ lim  ​     

1
  ​  ​ = __
​    ​
x3
x2 x Æ 0 n x 5
​ lim 
  ​ ​ ​  __________________ ​ 
      ​

(  )
x Æ 0 x n – 5 1 +  2x  + 3x3 +...+ nxn – 1
______________________
​     
     ​
(1 +  x + x2 + ... + x n)
______________________ 1
= a non-zero finite value. ​ lim 
  ​ ​  ​     
n    ​  ​ = __
​   ​ 
x Æ 0 5
It is possible only when n = 5
31. We have 1 __ 1
​ __
n ​ = ​ 5 ​ 

(  )
n
n ◊1 +  (n – 1)(1 + 2) + (n – 2)(1 + 2 + 3) + ... + 1​S   ​ ​ ​r n = 5.
  ​ ​ ___________________________________________
(  )
 r = 1
​ lim  ​         ​      ​ tan x – sin x a
n Æ • 4   ​ ​ ​ __________
2. We have ​ lim   ​ 
    ​ = __
​   ​
n x Æ 0 x3 b

(  )
Let t r = (n – (r – 1))(1 + 2 + 3 + ... + r) tan x (1 – cos x) a
  ​ ​ ​ _____________
​ lim   ​ 
     ​= __
​   ​

( 
x3 b
( 
x Æ 0
r (r + 1)
= (n + 1 – r)​ ​ _______ )
)
 ​  
 ​
2
tan x
  ​ ​  ​ ____
​ lim 
2 sin2 ​ __
​  × ________
​ 
x
​    ​  ​
2

 ​  ​ =
(  ) a
__
​   ​
x   
(  )
1 1
= __
​   ​  (n + 1)(r (r + 1) – __
​   ​  (r2 (r + 1)) x Æ 0 x2 b
2 2 ​ __
​   ​   ​ × 4
4
1 1
= ​ __ ​  (n + 1) (r2 + r) – __
​   ​  (r3 + r2)
2 2 a 1
​ __ ​ = __ ​   ​ 
n n b 2
1 1
Thus, Sn = ​ __ ​  (n + 1) ​S   ​ ​ ​(r2 + r) – ​ __ ​ ​ S   ​ ​  (r3 + r2) Hence, the value of a + b + 3 = 1 + 2 + 3 = 6
2  r = 1 2 r = 1
________
Now
( 
  + P (x) ​ 
÷​  1
64
  ​ ​ ​ _____________
3. We have ​ lim 
–1
x    ​   ​ )
(  )
x Æ 0
n
n ◊ 1  +  (n – 1)(1 +  2) + (n –  2)(1 + 2 + 3) + ... + 1 ​S   ​ ​ r​ 
(  )
1
___
(1  + P (x)​)​ ​64  ​ ​ – 1
  ​ ​ __________________________________________ ​ lim ​ ​ ______________
 r = 1
​ lim  ​        4
 ​      ​ =  ​  x       ​  ​
n Æ • x Æ 0
n

=  
1
2
( 
n

​ lim ​ ​ _________________________________
​ 
 r = 1
      ​ 
1
​   ​  (n  + 1) ​S   ​ ​ ​ (r2 + r) – ​ __ ​  ​
__
2  r = 1
n
  S   ​ ​ ​ (r3 + r2)
   ​ ) = 
x Æ 0
( (  64
1
​ 1  + ___
​ lim ​ ​ ​ _______________
x      
)
​    ​  P (x)  ​ – 1
​  ​ )
n Æ • 4
n
= 
x Æ 0 64 ( 1
​ lim ​ ​ ___
P(x)
​    ​ × ____
​  x  ​  
 ​)
The Limit  3.55

= 
x Æ 0 64
(  2 3
1 a1x  + a2 x + a3 x +...+ an x
​ lim ​ ​ ___
​    ​ × ________________________
​     
x    
n
​  ​ )
1
= ​ __ ​ 
2
Hence, the value of (a + b) is 3
( 1
​    ​ × ​( a1 + a2x + a3x2 +.....+ anxn– 1  )​ ​
​ lim ​ ​ ___ )
( 
= 

)
x Æ 0 64
tan (x – 1) loge (x x – 1)
a1 ​ lim ​ ​ ​ _________________
6. We have     
 ​ 
   ​
1 (x – 1)3
= ___ ___ x Æ 1
​    ​ = ​    ​ 
64 64
So, a =1, b = 64
Hence, the value of ÷
_____
  + a  
​  b ​– 1

x Æ 1 ( 
tan (x  – 1) (x – 1) loge (x)
​ lim ​ ​ ​ _____________________
=     
(x – 1)3
 ​    ​
)
_________ ___

( 
=÷   + 1 – 1 ​
​  64    = 8
= ​÷ 64 ​
)

tan (x – 1) loge (x)

( 
​ lim ​ ​ ​ ______________
=    
 ​ 
   ​
  ​ ​ ___________
4. We have ​ lim  ​    
2n
x  sin   x
   ​  ​
n

) x Æ 1 (x – 1)2

 ( )
x Æ 0 x2n –  sin2n x

= 
x Æ 0
​ 
sin n x
​ lim ​ ​ _________   
sin2n x
______
 ​  ​ = 
x Æ 1 ( 
tan (x – 1)
​ lim ​ ​ ​ _________ ​
(x – 1)


loge (x)
× ​ ______ 
(x – 1)
 ​  ​ )

(  ( 
1 – ​  2n    ​ 
x

(  )
) (  )
n tan (1  + h – 1) loge (1 + h)
x3 x5 __ x7 ​ lim ​ ​ _____________ ​ __________
​​ x  – __
​    ​ + __ ​    ​ – ​    ​ + ...  ​​ ​ = 
h Æ 0
​    
(1 + h –1)
   ​ ×     ​  ​
(1 + h – 1)
3! 5! 7!
​ lim ​ ​ ________________________
=  ​     
   2n ​  ​
x Æ 0
1 – ​​ 1 – __
x2 x4 __
​    ​ + __
x6
)
​    ​ – ​    ​  + ...  ​​ ​
( 
tan (h) log e (1 + h)
)
 ( (  (  ) ) )
3! 6! 7! ​ lim ​ ​ ______
=  ​   × ​ __________
 ​   ​ 
    

h Æ 0 (h) (h)
3 n
x
​​ x – __
​    ​  ​​ ​ =1
3!
____________
= 
​ lim ​ ​ ​       2n ​  ​
x Æ 0 x2
__ Thus, L=1
1 – ​​ 1 – ​    ​  ​​ ​

 ( ( (  ) ) )
3! Hence, the value of (L + 3) is 4

( 
n
x2

xn ​​ 1 – __
​    ​  ​​ ​
3!
​ lim ​ ​ ​ ____________
= 
x Æ 0
    2n ​  ​
x2
7. We have ​ lim 
x Æ 0
​ 
2 (tan x  – sin x) – x3
  ​ ​ _________________
x5
 ​       ​ )
1 – ​​ 1 – __ ​    ​  ​​ ​

(  ( {  } (  )
3!

It provides a non-zero definite number only when n = 2. =  


​ lim ​ ​ ​ 
x3 2x5
2 ​ ​ x  + __
​   ​ + ___
3 15
__________________________________
      ​ 
x3 x5
​   ​ + ...  ​– ​ x – __
​    ​+ __
3! 5!
    ​
​    ​– ...  ​  ​ ))
x Æ 0 x5
5. We have

   ( ( 
( ( ) (  ) )) )
____________
  ​ ​ ​  x2 + x2 sin ​
​ lim 
x Æ •
   ( ÷ 
1
__
​ x ​  ​ ​ – x  ​ (  ) ) x2 x3
2 ​ ​ ​ ​ __ ​   + __
3 3!
​ lim ​ ​ __________________________________
15
2
​    ​   ​ + ​ ___
x5
​    ​  x5 – __
​    ​  ​ + ...  ​  ​– x3
5!

(  ÷ 
=   ​       5
 ​      ​
x Æ 0 x
(  )
)
1
x2 + x2 sin ​ __
​ x ​  ​ – x2
​ lim ​ ​ ​  ________________
= 
x Æ •
____________
2 2
​  x + x  sin ​
  
  
   ​   ​
1
(  )
__
​ x ​  ​ ​ + x
 ( ( ( 
x3 x5
2 ​ ​ __
​   ​   + __
2 ))
​   ​  + ...  ​  ​ – x3
8
​ lim ​ ​ ____________________ )
( ÷ 
=   ​       ​      ​
x Æ 0 x5
(  )
)
1
x2 sin ​ __
​ x ​  ​

( ( (  )
​ lim ​ ​ ________________
= 
x Æ •
​  ____________
      ​  ​
​  x2 + x2 sin ​
  
1
__ (  )
​ x ​  ​ ​ + x
x5
​ ​  x3  + __ ))
​   ​  + ...  ​  ​ – x3
4
​ lim ​ ​ ​  _________________

( ÷ 
=    ​        ​
x Æ 0 x5
(  )
)
1
x sin ​ __
​ x ​  ​
______________
​ lim ​ ​ ​  __________ 1
=        ​  ​ = ​ __ ​ 
(  )
x Æ •
1
__ 4
​  1 + sin ​   ​ x ​  ​ ​ + 1
3.56  Differential Calculus Booster

Thus, m = 1, n = 2
(  (  (  ) ) )
1/n
3 n
Hence, the value of m + n + 2 = 5 ​ lim ​ ​​ 4 ​ 1 + ​​ __
=  n
​    ​  ​​ ​   ​  ​​ ​
n Æ • 4

4 ​​(​ 1 + (​​  __


8. We have
​   ​  )​​ ​ )​ )​​ ​
n 1/n
3

( 
​ lim ​  
)
1  + sin x – cos x – + log (1 – x) = 
  ​ ​ ​  __________________________
​ lim          ​  ​
n Æ • 4
x Æ 0 x tan2x

 (
= 4 ((1 + 0))0 = 4

( 
) )
1 +  sin x – cos x – + log (1 – x) 1 1
​ lim ​ ​ ___________________________
=  ​        ​     ​   ​ ​ ___
Also, ​ lim  ​     ​ – ______
​       ​  ​
x Æ 0 tan2x
x Æ1 ln x (x – 1)
x3 ​ _____ ​

( 
x2
= 
​ lim 
(x  – 1) – ln x
​ ​ ____________
​      ​  ​
   )
(  )
(  )
1 +  sin x – cos – + log (1 – x) x Æ1 (x – 1) ln x
​ lim ​ ​ ​ _________________________
=      ​     ​
x Æ 0 x3 __1
​  2  ​ 

(  )
1 x
cos x  + sin x – ______
​       ​ =  ​ ​ ______
​ lim  ​      ​  ​
(1 – x) x Æ1 __ 1 1
​ lim ​ ​ ___________________
=  ​      ​     ​ ​ x ​ + __ ​  2  ​ 
x

( 
x Æ 0 3x2

(  ) )
1
2
– sin x  + cos x – _______
​    2 
 ​ 1 – __​ x ​
=  ​ ​ ​ ____________
​ lim   
    ​  ​
( 
(1 – x)
=  _____________________
​ lim ​ ​ ​ 
x Æ 0
   
6x
    ​  ​ x Æ1 x  – 1
ln x + ​ _____
​  x    ​  ​ )

(  ) (  ))
2 1
– cos x  – sin x – _______
​       ​ 1 – __
​ x ​
(1 – x)3 = 
​ lim 
​ ​ ​  ___________  
    ​  ​
_____________________
= 
​ lim ​ ​ ​      ​     ​ x Æ1
ln x + ​ 1 – __
1
( 

(  )
x Æ 0 6 ​ x ​  ​

3 1 __1
= – ​ __  ​ = – ​ __ ​  ​  2  ​ 
6 2 x
=  ​ ​ ______
​ lim  ​      ​  ​
x Æ1 __ 1 1
Thus, m = 1, n = 2 ​ x ​ + __ ​  2  ​ 
Hence, the value of (m + n) is 3. x
1 1
= _____ __

( 
​     ​ = ​   ​ 

x Æ 0
ex  + e– x + 2 cos x – 4
  ​ ​ __________________
9. We have ​ lim  ​     
x4
 ​     ​ ) Thus
1  +  1
m = 1 and
2
n=2

( 
Hence, the value of (m + n + p)
= 
x Æ 0
ex  – e– x – 2 sin x
​ lim ​ ​ ______________
​ 
4x3
 ​ 
     )


=4+1+2
= 7.

( 
ex  + e– x – 2 cos x
​ lim ​ ​ ​ _______________ )
( 
11. We have

(  ) )
=   ​ 
     ​
x Æ 0 12x2 2x3
2x  – sin 2x – tan– 1 ​ _____
​    
 ​  ​
1 + x6
(  )    ​ ​ ________________________
​ lim  ​      ​     ​
x – x
e   – e + 2 sin x
​ lim ​ ​ ​ ______________
=       ​  
​ x Æ 0 x3

( 
x Æ 0 24x

​ lim ​ ​ (​  _______________
= 
x – x
e   + e + 2 cos x
 ​ 
     )​
= 
x Æ 0
2x  – sin 2x – 2 tan– 1 (x3)
​ lim ​ ​ ____________________
​     
x3
 ​     ​ )
(  )
x Æ 0 24
6x2
2  – 2 cos 2x – _____
​    
 ​
4 1 1 + x6
= ___
​    ​ = ___
​    ​  ​ lim ​ ​ __________________
=  ​      ​     ​
24 16 x Æ 0 3x2
Thus, p = 1, q = 6
Hence, the value of (q – p) is = 5.

(4n + 3n)1/n
​ lim ​  
10. We have P =  
n Æ •
= 
​ lim ​ ​ ​ 
x Æ 0
(     
3x2
 ​ 
​ 
6x2
2 (1  – cos 2x) – _____
1 + x6
___________________
  
   ​
 ​
)
The Limit  3.57

(  ) {  (  ) }
n
6x2 1 r p
4 sin2 x  – _____
​    
 ​ =  ​ n ​ (​S  ​ ​ ​​​ __
​ lim ​ ​  __ ​ n ​   ​​ ​  ​
x Æ •
1 + x6  r =1
=  ​ ​ ______________
​ lim  ​   ​      ​ 1
x Æ0 3x2 = ​Ú ​  ​  x p dx

(  )
0
4 sin2x ________
(  )
6
​ lim ​ ​ __
=  ​   ​ ​  _____
 ​ 
 – ​       ​  ​ x p + 1 1
x Æ 0 3 x2 3 (1 + x6) = ​​ ​ _____  ​  ​​ ​​ 
p+10
4 2
= ​ __ ​  – 2 = – ​ __ ​ 
1
= _____
​     ​ 
3 3 p  +  1
Thus, p = 2 and q = 3 2. Ans.(a)
Hence, the value of (p + q) is 5. We have
12. We have ​ lim 
x Æ •
log x n  – [x]
  ​ ​ __________
​ 
[x]
 ​ 
     ( 
​, n Œ N, [,] = G.I.F )
​ lim 
x Æ • {  (  ( 
x+1
  ​ ​  x ​ tan– 1 ​ ​ _____ 
x+2 ) x
 ​  ​ – tan– 1 ​ _____
​     
x+2
 ​  ​ ​  ​ (  ))} ​ lim ​ ​ _____
=  ( 
log xn ___
​   ​ 
[x]
 – ​   ​  ​ )
{  (  ( 
x Æ • [x] [x]

)}
x+2 x =0–1
​ _____   ​ – _____
​       ​
x+2 x+2
__________________
– 1 = –1.
​ lim ​ ​  x ​ tan ​   ​
=          ​  ​  ​  ​
x Æ • x + 1 _____
1  + ​ ​ _____ 
x+2 x+2 (  x
 ​× ​       ​  ​ ) 3. Ans. (c)

( 
xf (2)  – 2f (x)
)
{  (  ( 
  ​ ​ ___________
​ lim  ​    
 ​   ​
(x + 2)
​ lim ​ ​  x ​ tan  ​​ ________________
=   – 1
       ​ ​  ​  ​
)} x Æ 2 x–2

( 
x Æ •
(x + 2)2  + x (x + 1)
= 
f (2)  – 2f ¢(x)
​ lim ​ ​ ___________
​   ​ 
     ​ )
{  (  ( 
x Æ 2 1
=  
x Æ • 2
x+2
​ lim ​ ​  x ​ tan  ​ ​ __________
– 1

    ​  ​  ​  ​
2x + 5x + 4 ) )} = f (2) – 2f ¢(2)

(  )
=4–8

( 
x + 2  
tan–1​ ​ __________
   ​  ​
)  

4. Ans. (a)
= – 4

) ( ) ( 
2
2x + 5x +4 x (x  + 2)
​ lim ​ ​ ​  ________________   ​  ​ × ​ ____________
)
x

( 
=      ​  2   
   ​  ​ x2 +  5x + 3
x Æ • x + 2
__________ 2x   + 5x + 4   ​ ​​ __________
​ lim  ​  2   
   ​  ​​ ​
​ ​  2      ​  ​ x Æ •
x +x+3
2x + 5x + 4

(  )
x
4x
1 1 ​ lim ​ ​​ 1 + __________
=  ​  2     ​  ​​ ​
= 1 × __
​    ​ = __
​   ​  x Æ •
x  + x + 3
2 2
Thus, m = 1 and n = 2
(  )
x
4x
​ lim ​ ​​ ________
  ​       ​  ​​ ​
Hence, the value of (m + n) is 3. = ​e
x Æ • 2
​ x + x + 3 ​

Problems asked in Jee Main Exams. =



​ x Æ •
​e​
(  ​  2
x
4x2
lim ​ ​ ________
+
   )
 ​  ​
x +3 ​

( 
From 2002-2014
1. Ans. (a) =
x Æ •

​e​
​ 
1 + __
4
​ lim ​ ​ _________

1 3
   
​ x ​+ __ )
 ​  ​
​  2  ​ 
x ​
We have
= e4
( 

1p  + 2p + 3p + ... + np
  ​ ​ ​  ___________________
​ lim       ​    ​ ) 5. Ans.(c)

(  )
n Æ •
np + 1 x2

Ú​  ​  ​ ​sec2t dt
= 
x Æ • {  ( 
1 1p  + 2p + 3p + ... + np
​ lim ​ ​  __
​ n ​ ​ ​ ___________________
   
np
    ​  ​  ​ )}   ​ ​ ________
​ lim 
x Æ 0
0
​ 
x sin x
    
​  ​

​ lim ​ ​  __
= 
x Æ • {  ( (  ) (  ) (  )
1 1 p __
​ n ​ ​ ​​ __
2 p 3 p
​ n ​  ​​ ​+ ​​ ​ n ​  ​​ ​+ ​​ __ (  ) )}
n p
​ n ​  ​​ ​+ ... + ​​ __
​ n ​  ​​​ ​  ​ ( 
sec2 (x2) 2x
​ lim ​ ​ ___________
=  ​ 
x Æ 0 sin x + x cos x
    ​  ​
   )
3.58  Differential Calculus Booster

(  ) (  (  ) ​​( __​ nr  ​ )​​ ​ )​
n
2 sec2 (x2) 1
__
lim ​ ​  
r 4
 ​ ​ ​S   ​ ​ ​​​ __
3
​ lim ​ ​ __________
=  ​       ​  ​ = ​  
n Æ • n  r = 1 n
​   ​   ​​ ​ –
x Æ 0 ____ sin x
​  x   ​ + cos x
1
2 = ​Ú ​  ​ (x4 – x3) dx
= ​ __ ​  = 1
2 0

( 
6. Ans. (c)

( 
log (3 + x)  – log (3 – x)
  ​ ​ ____________________
Given ​ lim  ​        ​  ​ = k )

x5
= ​​ __
​   ​  –
5
x4 1
__
4 0 )
​   ​   ​​ ​​ 

x Æ 0 x 
1 1 4–5 1
= __
​   ​  – __
​   ​  = ​ _____  = – ​ ___  ​ 
 ​ 

k=  ( 
x
log ​ 1  + __
​ lim ​ ​ ​ 
(  ) (  (  ) ) )
x
​    ​  ​ – log ​ 1 + ​ – ​ __  ​  ​  ​
3
_________________________
   
3
  ​  ​ 9. Ans. (d)
5 4 20 20

x 

 ( ( 
x Æ 0
We have

) (  (  ) )
)
x x

( 
log ​ 1  + __ ​    ​  ​ log ​ 1 + ​ – ​ __  ​  ​  ​
​ lim ​ ​ __________
k= 
x Æ 0
​  x
__
  
​    ​ × 3
3
3
 ​  – ​ _____________
x
(  )
3
  
   ​  ​
​ – ​ __  ​  ​ × (– 3)
3
​ lim 
x Æ a
f (a) g(x) – f (a) – g(a) f (x) + g(a)
  ​  ​ ​ __________________________
   
g(x) – f (x)
 ​     ​ = 4 )

1 1 __
k = __
​   ​  + __
3 3 3
2
​    ​ = ​    ​
​ lim 
x Æ a
f (a) g¢(x)  – g(a)f ¢(x)
  ​  ​ __________________
​    
  
g¢(x) – f ¢(x)
 ​  ​ = 4 (  )
7. Ans. (c) f (a) g¢(a)  – g(a) f ¢(a)
​ __________________
​         ( 
 ​  ​ = 4 )
 ( ((  
We have g¢(a) – f ¢(a)

(  ))
) ( 
x
​ 1 – tan ​ __
)
​    ​  ​  ​ kg¢(a)  – k f ¢(a)
2 (1 – sin x)
  p ​ ​ ​ ___________
​ lim      ​ × ​ ________3  ​  ​ ​ _____________
​    
   ​  ​ = 4
   ​  ​ 1 + tan ​ __
x Æ ​__
2
x
​    ​  ​  ​
2 (  )) (p – 2x) g¢(a) – f ¢(a)

= 
​ lim 
p
__ (  p x
​ ​ tan ​ __
  ( 
​   ​  – __
4 )
(1 – sin x)
​    ​  ​ × ​ ________3 
2
 ​  ​
)
(  g¢(a) – f ¢(a)
k ​ ​ __________
    ​  ​ = 4
g¢(a) – f ¢(a) )
x Æ ​   ​  (p – 2x)

(  ( 
2
k=4

(  (  ) )
)
p 10. Ans. (b)
​ 1 – sin ​ __ ​   ​  + h  ​  ​
)     ( 
p p __
)
h 2
​ lim ​ ​ tan ​ __
=  ​   ​  – __
​   ​  – ​   ​   ​× ​ ______________     ​  ​ Given ​ lim 
a
  ​ ​​ 1 + __
​ x ​ + __b 2x
​  2  ​   ​​ ​ = e2
h Æ 0 4 4 2
( ( )) p
​ p – 2 ​​ __
3
​   ​  + h  ​​ ​  ​
2
x Æ •
x
(  )
( 
a b
​ lim ​ ​ __
  ​ x ​ + __
​  2  ​   ​

(  ) )
h (1 – cos h) ​e​x Æ • x ​= e2
​ lim ​ ​ tan ​ – ​ __ ​   ​ × ​ _________
=   ​  


( 

( 
h Æ 0 2 (– 2h)3 ​
​ex Æ •

2b
​ lim  ​ 2a + ___
​  x ​   ​
​=) e2

tan ​( – ​ __  ​ )​ 2 sin  ​( __


​   ​  )​

)
h h2 e2a = e2
2 2
​ lim ​ ​ ___________
=  ​        ​ × ____________
​         ​  ​ a = 1, b Œ R
​( – ​   ​  )​ × (– 2) (– 2)  ​( __ ​   ​  )​ × 4
h Æ 0 h
__ h 3
2
11. We have
2

(  (  ) (  ) (  ) )
4
1 1 2 4 1 n2
1   ​ ​ __
​ lim  ​  2  ​  sec2 ​ __
​  2  ​   ​ + __
​  2  ​  sec2 ​ __
​  2  ​   ​ + .... + __
​  2  ​  sec2 ​ __
​  2  ​  ​  ​
= ___
​    ​  n Æ •
n n n n n n
32

( S  (  ) )
8. Ans. n
__r 2 __ r2
We have ​ lim ​ ​ ​
=     ​ ​ ​    ​  sec  ​ ​    ​   ​  ​ 
n Æ •  r = 1 2
n2

( 
n

n Æ •
14 + 24 + ... + n4 _____________
  ​ ​ ​ _____________
​ lim 
n 5
 ​ – ​ 
    
13 + 23 + ... + n3
n4
 ​      ​
1
__
lim ​ ​  
= ​    ​ ​ ​
n
__r
) 2 __ r2
  ​ ​ ​ ​   ​   ​ sec  ​ ​    ​   ​  ​
( S  (  ) (  ) )
n Æ • n r = 1 n
n2
​ lim ​ ​  
= 
n Æ • n
 ​
 ​ ​  ( 
4 
+ 24 + ... + n4 1____________
1 1____________
__
n 4
 ​    
– ​ 
3 
+ 23 + ... + n3
n 3
 ​ 
     ​
=
1
Ú
​ )
 ​  ​ x sec x dx
2
0
The Limit  3.59

|
1
f (2x)
​ lim ​ _____
1
1£  ​   ​ £ 1
= x ​​ ​Ú ​  ​ sec  x dx  ​​ ​ ​ – ​Ú ​  ​ tan x dx
2 x Æ • f (x)
​ x 0
Thus, by Sandwitch theorem, we get,
=  ​​​x tan x |1​​0​​  – (log sec x​​ )  1
​ |​​0​​ 
f (2x)
= tan (1) – log (sec 1)   ​ _____
​ lim  ​   ​ = 1
x Æ • f (x)
12. Ans. (a)
We have 16. Ans. (c)

(  )
f (x) We have
4t3

( 
  ​  ​Ú ​   ​ ​ _____ ______________

)
​ lim  ​      ​  ​ dt   – cos2 (x   
​÷ 1  – 2) ​
x Æ 2 6 x – 2
  ​ ​ _______________
​ lim  ​    
 ​ 
   ​

(  )
x Æ 2 (x – 2)
f (x)
Ú​  ​   ​ ​(4t3) dt
( 
__________

​ lim ​ ​ _________
= 
x Æ 2
6
​ 
(x – 2)
 ​ 

 ​ = 
x Æ 2
  2 (x –  2) ​
​÷ 2sin
​ lim ​ ​ ​ ____________
  
(x – 2)
 ​   ​ )

x Æ 2
( 
4 (f (x))3 f ¢(x)
​ lim ​ ​ ___________
=  ​ 
1
 ​      ​ ) ( 
__
   |​sin ​​ (x – 2) |​
 ​​÷2 ​ 
​ lim ​ ​ ​ ____________
= 
x Æ 2 (x – 2)
  
 ​   ​ )
= 4 (f (2))3 f ¢(2)

( 
__


1
= 4 × 216 × ___
​    ​ 
48
Now, L.H.L =  
x Æ 2
÷    sin (x – 2)
​  2 ​ 
​ lim ​ ​ – ​ ___________   
(x – 2)
 ​  
__
   
​ = – ​÷ 2 ​ )
( 
__

13. Ans. (d)
= 18 R.H.L = ​ lim 
x Æ 2
÷    sin (x – 2)
​  2 ​ 
  +​ ​ ​ ___________   
(x – 2)
​= ÷
 ​  
__
   
​  2 ​ )
Since R.H.L π L.H.L, so limit does not exist.
We have

( 
17. No questions asked in 2012.

x Æ a
1 –  cos (ax2 + bx + c
  ​ ​ __________________
​ lim  ​     
(x – a)2
 ​ 
   ​
) 18. Ans. (c)

( 
(1  – cos 2x) (3 + cos x)
  ​ ​ ​ ___________________
We have ​ lim          ​  ​ )
(  )
x Æ 0 x tan 4x
1 –  cos {a (x – a) (x – b)}

( 
​ lim ​ ​ ______________________
=   ​       ​     ​

)
x Æ a
(x – a)2

 ( { 
sin2 x (3 + cos x)
​ lim ​ ​ 2 × _____
=  ​  2 ​  × ​ __________   
    ​  ​
( 
})
{ 
a (x – a) (x – b)
2 sin2 ​ ​  _____________
 ​ 
     ​ a2 (x – b)2 }
x Æ 0 x tan 4x
​ _____
​     
4x
​  ​ × 4)
2
​ lim ​ ​ ​  ____________________
=      
     ​  ​ × ​ _________
 ​    1+3
x Æ a
a (x – a) (x – b) 2 4 = 2 × ​ _____
 ​ 
 = 2
​ ​  ______________
 ​ 
     ​ 4
2 19. Ans. (d)

(  )
2 2
a  (a  – b) sin (p cos2 x)
= __________
​   ​       ​ ​ __________
We have ​ lim  ​   ​ 
     ​
2 x Æ 0 x2
14. No questions asked in 2006-2009.
15. Since f (x) be an increasing function, then we can
= 
x Æ 0
​  ( 
sin (p (1  –  sin2 x))
​ lim ​ ​ _______________
x2
 ​       ​ )
(  )
write
sin (p – p sin2 x)
f (x) £  f (2x) £  f (3x) ​ lim ​ ​ ​ _____________
=   ​ 
     ​
x Æ 0 x2

f (2x)
1 £ _____
​ 
f (x)
 ​ £
f (3x)
_____
​ 
f (x)
 ​  = 
x Æ 0 ( 
sin (p sin2 x)
​ lim ​ ​ __________
​ 
x2
 ​ 
     ​ )
(  )
f (2x) f (3x) 2
​ lim  ​ lim ​   ​ _____ ​ £  
  ​ 1 £   ​ lim ​ _____
​   ​  ​ lim ​ ​ __________
=  ​    
   
​ ×
p sin2 x
sin (p sin  x) ______
​   ​ = p
 ​  
x Æ • x Æ • f (x) x Æ • f (x)
x Æ 0 p sin2 x x2
3.60  Differential Calculus Booster
______
20. Ans. (b) Now,   – x2 
G(x) = –  ​÷25  ​
21. Ans. (b)
(– 2x)
__ ___
1
​    ​  G¢(x) = – ​ _________
______   ​
We have p = ​ lim 
  +​  (1 + tan2 ​​÷ x ​)  ​2x ​   – x2 
2 ​÷25  ​
x Æ 0
x
= _______
1
___
  +​  
​ lim (tan2 ​÷x 
__ ​    ​ 
2x
  ​​)​
  ​ ​  ______     ​
= e​ ​ x Æ 0 ​   – x2 
÷​ 25  ​
(  )
__
1 ______ tan2 ​÷x 
   ​ 1 1
G¢(1) = _______  ​ = ____
​ lim 
  __
​ ​   ​ ​  ​  x    ​  ​
x Æ 0+ 2 Thus, ​  ______
    ​  __
   ​ 
= e​ ​ ​   – 1 ​
​÷25   2​÷6 ​   
1
​ __ ​  5. Ans. (c)
2​
= e​ ​

Now,
1
log (p) = log (e1/2) = __
​    ​
2 x Æ a
​  ( 
g(x) f (a)  – g (a) f (x)
​ lim ​  ​ _________________
Limit =   x – a       ​  ​ )
Questions asked in Past Iit-Jee Exams
x Æ a ( 
g¢(x) f (a)  – g (a) f ¢(x)
​ lim ​  ​ __________________
=  ​ 
1
 ​       ​ )
= g¢(a) f (a) – g (a) f ¢(a)

h Æ 0
( 
(a  + h)2 sin (a + h) – a2 sin a
​ lim ​ ​ ​  _______________________
1. Limit =      
h
    ​  ​ ) = 2 ◊ 2 + 1 ◊ 1

=5
=   ( 
2 (a + h) sin (a
​ lim ​ ​ _____________________________________
​       
+ h) + (a
 ​ 
+ h) 2
+ cos (a
    ​
+
)
h) 6. Ans. (b)

[  ]
h Æ 0 1 1 2 n
Limit =   ​ lim ​ ​ _____
​    2   ​+ _____
​    2   ​+ ... + _____
​    2 
 ​  ​
n Æ •
2 1–n 1–n 1–n
= (2a sin a + a  cos a)
2. Limit = 
n Æ • ( 
1  + 2 + 3 + ..... + n
​ lim ​ ​ ​ _________________
1 – n2
 ​ 
  
)
   ​

(  )
( 
1 1 1 1
​ lim ​ ​ _____  ​ + _____  ​ + _____  ​ + ...... + ___
)
=   ​      ​      ​      ​    ​  ​ n (n + 1)
n Æ • n + 1 n+2 n+3 6n =  ​ lim ​ ​ ​ ________2     ​  ​

( 
n Æ •
2 (1 – n )
_____ 1 _____ 1 1
_____ 1
______
)
( 
=   ​ lim ​ ​ ​       ​ + ​       ​ + ​       ​ + .... + ​       ​  ​

)
n Æ • n + 1 n+2 n+3 n + 5n (n2 + n)
=  ​ lim ​ ​ ​ ________2     ​  ​

(  (  ))
(  )
5n n Æ •
1 2 (1 – n )
=   ​ lim ​  ​ S   ​ ​ ​ _____    ​   ​
n Æ •  r = 1 n + r
( 
​ 1 + ​ __
_________
)
1
n ​  ​
(  ​
5n = ​ 
  l im   
 ​ ​       ​  ​
=   ​ lim ​  __
1
n Æ • n  r = 1 n + r
n
​   ​  ​S   ​ ​ ​ _____
​      )
​   ​ n Æ •
2 ​ __
1
​  2  ​ – 1  ​
n
=  
1 5n _______
n Æ • n  r = 1 1 + (r/n) ( 
​ lim ​ ​   ​  ​S   ​ ​ ​ ​ ​ 
__ 1
    )
 ​  ​
( 
(1 + 0)
= ​​ _______    )
 ​  ​
5
2 (0 – 1)
dx
= Ú​   ​ ​ ​ _____
    ​ = log 6
01 + x
1
(  )
= ​ – ​ __ ​   ​

( 
2
3. Limit =  
x Æ 0 ​ (1
2x – 1
​ lim ​ ​ ​ __________
______   
÷  + x) ​ – 1
   ​  ​
) 7. Ans. (d)

( 
Limit =   ​ lim ​   f (x)
=  
2x – 1 ( _____
​ lim ​ ​ ​ _____ x     ​ × ​  ÷ ​ 1  + x   ​ + 1 )​  ​ ) x Æ 0

(  )
x Æ 0
sin [x]
= 2 log 2 =  ​ lim ​ ​ _____
​   ​    ​
x Æ 0 [x]

4. Limit =  
x Æ 1 (  G (x) – G (1)
​ lim ​ ​ ​ ___________
x–1
  
 ​    ​ ) Ï Ê sin[ x ] ˆ
Ô lim+ ÁË [ x ] ˜¯
=   ​ lim ​  G ¢(x) Ô x Æ 0
x Æ 1 =Ì
Ô lim Ê sin[ x ] ˆ
= G¢(1) ÔÓ x Æ0 – ÁË [ x ] ˜¯
The Limit  3.61

Ï Ê sin(0) ˆ Since R.H.L π L.H.L, so limit does not exist.


Ô lim+ ÁË =1
Ô x Æ0 (0) ˜¯ 10. Limit =  
​ lim ​  (g (f (x)))
=Ì x Æ 0
Ô lim Ê sin[ x ] ˆ = Ê sin(–1) ˆ = sin(1)
Ô x Æ0 – ÁË [ x ] ˜¯ ÁË (–1) ˜¯ = 
​ lim ​  (g(sin x))
Ó x Æ 0

Since R.H.L π L.H.L, so limit does not exist. ​ lim ​  ((sin x)2 + 1)


= 
x Æ 0

8. Limit =   (  p x
​ lim ​ ​ (1 – x) tan ​ ___
​   ​  ​  ​(  ) ) =1

( 
x Æ 1 2

= 
y Æ 0
(  p
​ lim ​ ​ (1 – 1 – y) tan​ (1 + y) ​ __ ( 
r ​   ​  ​ )) ​ lim ​ ​ ​ 
11. Limit =  
x Æ •
1
x4 sin ​ __
____________
(1 + |x|3)
(  )
​ x ​  ​ + x2
  
  
 ​ ​ )
( 
p p y
)
( 
​ lim ​  (– y) tan ​ __
​ r ​  + ___

)
=  ​   ​  ​
y Æ 0 2
​ lim ​ ​ ​ 
= 
1
x4 sin ​ __ (  )
​ x ​  ​ + x2
____________   
 ​  ​
​ lim ​  (– y) × – cot ​( ___ ​   ​  )​
p y

(  (  ) )
= 
x Æ •
(1 – x3)
y Æ 0 2

( 
x sin ​ __(  )
1 1
​ x ​  ​ + __
​ x ​
​( ___

)
___________
​   ​  )​
p y ​ lim ​ ​ ​ 
=     ​   ​
x Æ • 1
2 2 ​ __
​  3  ​  – 1  ​
​ lim ​ ​ ________
=  ​    ​  × __
​ p ​ ​
tan ​( ___
x
​   ​  )​
y Æ 0 p y
2


2
= __
​ p ​
( 
1+0
= ​ ​ _____ 
0–1
 ​  ​ )
= –1
9. Ans. (c)

( 
(  )
_____________

Limit =  
​ lim 
x Æ1
​÷1  – cos 2 (x  
x–1
– 1) ​
​ ​ ​  ______________
 ​ 
     ​ ) x2

Ú​  ​  ​ ​cos2 tdt
​ lim ​ ​ _________
0
12. Limit =   ​     ​  

( ÷ 
_________

)
​ 1 – cos 2y  

x Æ 0 x sin x
​ lim ​ ​ ​ __________
=  y     ​  

( 
y Æ 0
cos2 x ◊ 2x
​ lim ​ ​ ____________ )
( 
______

)
=  ​        ​  ​

÷​   2 sin2 y  
​ lim ​ ​ _______
=  ​  y    ​  ​
​ x Æ 0 sin x  + x cos x

( 
y Æ 0

= 
y Æ 0
÷
( 
__
   | sin y |​
​ 2 ​​
​ lim ​ ​ ________
​  y     
​  ​ )
= 
x Æ 0 ___
​ 
sin
2cos2 x
​ lim ​ ​ _________  
​  x   ​ + cos x

 ​  ​
)
Ï
Ô lim Á
Ê 2 sin y ˆ
˜
(  2
= ​ _____
​     ​  
1  +  1
​ )
+
ÔÔ yÆ0 Ë y ¯
=Ì =1
Ô Ê 2 sin y ˆ
Ô lim– Á
( 
˜ ____
ÔÓ yÆ0 Ë y ¯
x Æ 9
   –  3
÷​  f (x) ​
​ lim ​ ​ _________
13. Limit =   ​  __
​ x 
÷    ​ – 3
 ​  

​ )

( 
Ï Ê sin y ˆ

)
Ô lim+ Á 2 ¥
Ô yÆ0 Ë y ˜¯ 1
______
​  ____   ​ × f ¢(x)
= Ì  
Ô lim Ê – 2 ¥ sin y ˆ
2​÷f (x) ​

​ lim ​ ​ ​ ____________
=   ​ 
    

Ô yÆ0 – ÁË y ˜¯
x Æ 9 1
____
​  __   ​ 
Ó 2​÷x    ​

(  )
__
ÏÔ 2    ​
f ¢(x)​÷x 
=Ì ​ lim ​ ​ ______
=  ​  ____ ​   ​
ÔÓ – 2 ÷   
x Æ 9 ​  f (x) ​
3.62  Differential Calculus Booster

(  ) (  (  ))
p 1
​ lim ​ ​​ tan ​ __
__
​   ​
3f ¢(9) 19. Limit =   ​   ​  + x  ​  ​​x ​
= ​ _____
​  ____ ​  ​ x Æ 0 4
   
÷​  f (9) ​
(  )
3 ◊ 4
= ​ ____
​  __ ​  ​
  
​÷9 ​
=  ( 
x Æ 0 1 – tan x )
1 + tan x __​ 1x ​
​ lim ​ ​​ ​ _______ 
​   ​​ ​

​ lim ​ ​​ ( 1 + (​  ​ ________ 
=4
 ​– 1 )​ )​​ ​
1
__
1 + tan x ​ x ​
=   
14. x Æ 0 1 – tan x
15. No questions asked in 1990.
(  (  ))
1
__
2 tan x ​   ​
16. (d) ​ lim ​ ​​ 1 + ​ _______
=  ​    ​  ​  ​​x ​

( ÷  )
___________ x Æ 0 1 – tan x
1
​ __ 2
​    ​ (1 – cos    x) ​
2
​ lim ​ ​ ​ ____________
Limit =  
x Æ 0 x     ​  

= ​e

​ (  ​ 
2 tan x
​ lim ​ ​ _________
x Æ 0 x(1 – tan x)
   )
  ​  ​

=  ( 
1
​ lim ​ ​ ___
|sin x|
​  __  ​ × _____
​  x     
​  ​ ) = e2

( 
x Æ 0 ÷   
​ 2 ​

)
1
__
5x2 + 1 ​ x2  ​ 
Ï Ê sin x ˆ ​ lim ​ ​​ ​ ______
20. Limit =     ​  ​​ ​
1 x Æ 0 3x2 + 1
Ô lim+ Á ¥
x ˜¯

(  )
Ô x Æ0 Ë 2
= Ì
Ô Ê 1 sin x ˆ 1 __1
¥ 5 + __​  2  ​  ​ x2  ​ 
Ô lim– ÁË 2 x ˜¯ x
Ó x Æ0 ​ lim ​ ​​ ​ ______ 
=   ​  ​​ ​
1
3 + __
x Æ 0
​  2  ​ 
Ï Ê 1 sin x ˆ x
Ô lim+ Á ¥
Ô x Æ0 Ë 2 x ˜¯

(  )
= Ì y+5 y
Ô lim Ê – 1 ¥ sin x ˆ ​ lim ​ ​​ ​ _____ 
=   ​  ​​ ​
y Æ • y + 3
Ô x Æ0 – ÁË 2 x ˜¯
Ó

Ï 1
Ô–
Ô 2
= 
y Æ • ( 2 y
​ lim ​ ​​ 1 + _____
​     
y+3
 ​  ​​ ​ )

Ô 1
ÔÓ 2 = ​e ​y Æ • (  ) 2y
​ lim ​ ​ _____
  ​    
y+3​
 ​  ​

Since R.H.L π L.H.L, so limit does not exist. = e2

x Æ • x + 1(  )
x + 6 x+4
​ lim ​ ​​ ​ _____ 
17. Limit =    ​  ​​ ​
( 
log (1  + 2h) – 2 log (1 + h)
​ lim ​ ​ _______________________
21. Limit =  
h Æ 0
​     
h2
 ​     ​ )
(1  + _____
x + 1)
5 x+4

 ( ( (  )
​ lim ​ ​​ 
=  ​      ​  ​​ ​
x Æ • (2h)2 _____
​ ​ 2h – _____
​   ​ 
2
(2h)3
 + ​   ​ 
3
h2 h3
 – ...  ​– 2 ​ h – __ ) ( 
​   ​ + __
2
​   ​ – ...  ​  ​
3 ))
=
(  5(x + 4)
)
​ lim ​ ​  ​ _______ ​  


​ex Æ • x+1 ​


lim ​ ​ ________________________________________
= ​  
h Æ 0
​       
h 2
 ​      ​

= e5
x
( 
​ lim ​ ​ _______
18. Limit =   ​      
x Æ 0 tan– 1 2x
​  ​
)  ( ( ( 
(2h)2 _____
​ ​ – ​ _____
2
 ​  
(2h)3
 + ​   ​ 
3
​ lim ​ ​ ​  ____________________________________
= 
h Æ 0
     
h2
 ​ 
h2 h3
) ( 
 – ...  ​ – 2​ – ​ __ ​  + __
2
   
3 )) )
​   ​  – ...  ​  ​

( ( 
8h
​ lim ​ ​ ​ – 2 + ___
2h
​   ​ – ...  ​ – ​ – 1 + ___ ) (  ))
( 
=   ​   ​ – ...  ​  ​
= 
x Æ 0 2
1
​ lim ​ ​ __
2x
​    ​ × _______
​  – 1    
tan  2x
​  ​
) h Æ 0 3 3

= – 2 + 1
1
= __
​    ​
2 = – 1
The Limit  3.63

22. Ans. (b)


( 
x tan 2x  – 2x tan x
​ lim ​ ​ ​ ______________
)
( 
Limit =     
    ​  ​

) 1 – (cos 2x)2
2n x Æ 0
1 r
​   ​ ​S   ​ ​ ​ ​ _______
​ lim ​ ​ __
Limit =      
______  ​  ​

(  )
n Æ • n  r = 1
÷​ n  + r2 
2
 ​ 2x tan x
________
​     ​ – 2x tan x
1 – tan2x

(  S 
​ lim ​ ​ _________________
=  ​        ​  ​
= 
n Æ •
1 2n ______
​ lim ​ ​ __
 r = 1 ÷
r
​ n ​ ​   ​ ​ ​ ​ _______
   
​ n  + r2 
2
 ​  ​
) x Æ 0 4 sin4 x

(  ( 
 ​
) )
1

( 
2x tan x ​ ________
​       ​ – 1  ​

)
1 – tan2 x
​ lim ​ ​ ​ ____________________
= 
1 2n _________ (  )
​ __
r
​ n ​   ​
​ lim ​ ​ ​ n ​ ​S   ​ ​ ​ ​  _______
__     ​  ​
= 
x Æ 0
   
4 sin4 x
    ​  ​

÷  (  )
( 
n Æ • r 2
​ 1 + ​​ ​ __
 r = 1

)
n  
​ 
  ​​
  ​ ​ 2x tan x × tan2 x
​ lim ​ ​ ​ _______________
=      ​  ​
  

 (
x Æ 0 4 sin4 x(1 – tan2 x)
2

)
x dx
= Ú​  ​  ​  ​ _______
_____   ​
tan2x
tan x _____
0 ÷ ​ 1  + x2  ​ 2x4 × ____
​  x   
​ × ​  2 ​ 

x
_____ ​ lim ​ ​ ​  __________________
=          ​  ​
 ​ )​ |​​0​​ 
= ​ ​​​( ​÷1  + x2  sin4x
2 x Æ 0
4 _____
4x × ​  4 ​ (1
  2
  – tan x)

( 
x

)
__
= ​(​÷     – 1 )​
  5 ​
tan2 x
tan x _____
2 × ____
​  x   
​ × ​  2 ​  
23. Ans. (c) _________________ x
= 
​ lim ​ ​ ​        ​  ​
x Æ 0 sin4 x
( 
4 × _____
_____________

)
1  – cos2 (x  
​÷_______________
– 1) ​ ​  4 ​ (1  – tan2 x)

Limit =  
​ lim 
​ ​ ​   ​ 
  
   ​ x
x Æ1 x–1
1
= __
​   ​ 

( ÷ 
_________

)
2
​ 1 – cos 2y  

​ lim ​ ​ ​ __________
=  y      
​  ​ 25. Ans. (d)
y Æ 0

=  ( 
​÷2 sin
​ lim ​ ​ _______
​  y   
______
  2y ​ 
​  ​ ) x Æ • x + 2 (  )
x–3 x
​ lim ​ ​​ ​ _____ 
Limit =    ​  ​​ ​

( 1 + _____
y Æ 0

x + 2)
x
– 5
​ lim ​ ​​ 
=  ​      ​  ​​ ​

( 
__

)
x Æ •
÷    |sin y|
​ 2 ​
​ lim ​ ​ _______
=  ​  y     
​  ​
y Æ 0


= e​ x Æ •

(  )
​ 
– 5x
​ lim ​ ​ _____
x+2 ​
  
 ​  ​

Ï Ê 2 sin y ˆ
Ô lim Á = e– 5
˜
+
ÔÔ yÆ0 Ë y ¯ 26. Ans. (b)

Ô
Ô lim– Á
ÔÓ yÆ0 Ë
Ê 2 sin y ˆ
y
˜
¯
Limit =  
x Æ 0
​ 
x2( 
sin (p cos2x)
​ lim ​ ​ __________
 ​ 
    ​ )
Ï Ê
Ô lim+ Á 2 ¥
Ô yÆ0 Ë
sin y ˆ
y ˜¯

x Æ 0
​ ( 
cos (p cos2x)  × – p sin 2x
​ lim ​ ​ ____________________
=     
2x
    ​  ​ )
= Ì
Ô lim Ê – 2 ¥ sin y ˆ
Ô yÆ0 – ÁË
Ó y ˜¯
= 
x Æ 0 (  sin 2x
​ lim ​ ​ – p cos (p cos2x) × _____
​     
2x
​  ​ )
= (– p cos (p))
ÏÔ 2
=Ì =p
ÔÓ – 2
27. Ans. (c)

Since R.H.L π L.H.L, so limit does not exist.


24. Ans. (b) x Æ 0 ( 
(cos x  – 1) (cos x – ex)
​ lim ​ ​ ​ __________________
Limit =      
xn
    ​  ​ )
3.64  Differential Calculus Booster

(  (  ) ( 
)
x
2sin2 ​ __​    ​  ​  1
(a – n) = __
) ​ n ​
x
________ 2 (e – 1) 1_______
_______ – cos x
​   ​  
  ​ ​  x    ​  + ​  x     
​  ​
= 
​ lim ​ ​  __ x 2
1
x Æ 0 ​   ​ 
4 a = n + __
​ n ​
​ ___________________________
      ​     ​
xn – 3
31. Ans. (c)

(  )
It will provide us a finite limit only when x2

n–3=0 Ú​  ​  ​   ​ sec2 t  dt


​ lim ​ ​ _________
0
Thus n=3 Limit =   ​    ​  
   ​
x Æ 0 x sin x
28. Ans. (c)

Limit =  
x Æ 0 ( 
 f (1 + x) 1/x
​ lim ​ ​​ ​ _______
f (1)
 ​  

​​ ) =  ​ ( 
sec2x ◊ 2x
​ lim ​ ​ ______________
      ​  ​
x Æ 0 sin x ◊1 + x ◊ cos x )

​ = 
x Æ 0 ( 
f (1 + x)
​ lim ​ ​​ 1 + ​ _______
f(1)
 ​ 

1/x

– 1  ​​ ​ )
x Æ 0 ____
( 2sec2x
​ lim ​ ​ __________
=  ​ 
sin x
​  x   
  
 ​   ​
​ + cos x )


= e​ x Æ 0
​ ( 
f (1  + x) – f(1)
​ lim ​ ​ ____________
​ 
x f (1)
 ​ 
    ​
​ ) ( 
1+1
2
= ​ _____
​      ​  ​ = 1)
  ( 
 f ¢(1  + x)
​ lim ​​ ________
​   ​  

​ ) 32. Ans.(c)

( 
= e​ x Æ 0
​ f (1) ​


f ¢(1)
​ ____ ​ 
= e​ ​ f (1) ​
Limit =  
x Æ 0
f (x2)  – f (x)
​ lim ​ ​ __________
​    
f (x) – 0
 ​  
​ )

6
​ __ ​ 
= e​ ​3 ​ = e2 = 
x Æ 0 ( 
f ¢(x2) ◊ 2x  – f ¢(x)
​ lim ​ ​ ______________
​ 
f ¢(x)
 ​ 
     ​ )
( 
29. Ans. (c)
f (2  + h + h2) – f (2)
​ lim ​ ​ _________________
Limit =   ​    
    ​  ​
h Æ 0 f (h – h2 + 1) – f (1) ) = 
x Æ 0 ( 
0  – f ¢(0)
​ lim ​ ​ ________
​ 
f ¢(0)
 ​  

​)
( 
= –1

f ¢(2  + h + h2) ◊ (1 + 2h)
​ lim ​ ​ ____________________
=  ​     
    ​  ​
h Æ 0 f ¢(h – h2 + 1) ◊ (1 – 2h) ) 2
​ lim ​ ​ __
33. Limit =   (  1
​ p ​ (n + 1) cos– 1 ​ __
n Æ •
(  ) )
​ n ​  ​ – n  ​

​ lim ​ ​ ____
=  ​ 
h Æ 0 f ¢(1) (  )
f ¢(2)
 ​  ​ =   (  (  )
y Æ 0 p y
2 1
​ lim ​ ​ __
​   ​ ​ __ )1
​   ​ + 1  ​ cos– 1 (y) – __
​ y ​  ​

​ lim ​ ​ ( __
6 3
= __
​    ​ = __
​    ​ =  
2
​ y ​ (cos  (y) – (1) )​
1
​ p ​ cos  (y) + __
– 1 – 1
4 2 y Æ 0
30. Ans. (d)

We have, ​  
x Æ 0
   
x2 ( 
[(a – n) nx – tan x]sin nx
lim ​ ​ ​  ____________________
 ​     ​ = 0 ) =  
2
​ lim ​ ​ __
y Æ 0
(  (cos– 1 (y) – 1)
​ p ​ cos– 1 (y) + ​ ____________
y       ​  ​ )
  ​ ​ ​ _______________
​ lim 
x Æ 0
​  x       ( ( 
(a  – n) n x – tan x _____
sin nx
​ × ​  x   ​  ​  ​ = 0 )) 2
​ lim ​ ​ __
=   (  (cos– 1 (y) – cos– 1(0))
​ p ​ cos– 1 (y) + ​ _________________
y Æ 0 y       ​  ​ )
(( (   ))
)
( 
_____ ____

)
tan x sinnx
  ​ ​ ​ ​ (a – n)n – ​ ____
​ lim  ​  ​ × _____
x    ​  nx   
​ × n  ​  ​ = 0
  (    2 ​ ​ 
cos– 1 ​ y ◊ 0 + ​÷1 – y ÷ 1 – 0 ​  ​ )
2
​ lim ​ ​ __
​ p ​ cos– 1 (y)  + ​ ______________________
x Æ 0
=      
y    ​  ​
y Æ 0
((a – n)n – 1)n = 0

( 
_____


((a – n)n – 1) = 0
(a – n)n = 1
=  
y Æ 0 
2
​ lim ​ ​ __
cos– 1 ​ ​÷1  – y2 
​ p ​ cos– 1 (y) + ​ _____________
y     
( 
 ​  ​
​  

)
)
The Limit  3.65

2
​ lim ​ ​ __ (  sin– 1 y
​ p ​ cos– 1 (y) + ______ ) 38. Ans. (a, c)

( 
=   ​  y    ​  ​

)
______
y Æ 0 
x2
 ​– __
a  – ​÷a  2 – x2  ​   ​ 
( 
4
2
= ​ __
​ p ​ × cos– 1 (0) + 1  ​ ) ​ lim ​ ​ ​  ______________
L =  
x Æ 0 x4
 ​ 
     ​

(  )
(  (  ) )
2 p
= ​ __
​ p ​ × __
​   ​  + 1  ​ x2 x2
1/2
2 a –  a ​​ 1 – __
​  2  ​  ​​ ​– __
​   ​ 
a 4
= 1 + 1 ​ lim ​ ​ ​  __________________
=    ​        ​
= 2
x Æ 0 x4

(  (  (  )(  ) ) )
34. No questions asked in 2005.
1 x2 __ 1 1 x2 2 x2 1
35. Ans. (c) a –  a ​ 1 – __
​   ​  __
​  2  ​+ ​   ​ ​  __
​   ​  – 1  ​​​ __
​  2  ​  ​​ ​– ...  ​ – ​ __ ​  × __
​    ​ 
2 a 2 2 a 4 2!
​ lim ​ ​ ​ ________________________________________
Limit =   (  1 sin x
​ lim ​ ​ (sin x​)​ ​x ​​ + ​​ __
x Æ 0 
1
__
​ x ​  ​​ ​  ​ (  ) ) = 
x Æ 0
     
x4
 ​      ​

(  ) + e​  ​ ​lim ​ ​ ( ______ )​​
(  (  (  )(  ) ) )
log sin x – log x
​ lim ​ ​ _______ 1 x2 1 __ 1 x4 x2
a  – ​ a – __
​   ​ ​  __2  ​+ __
​   ​ ​  ​    ​ – 1  ​​ ___
​  3  ​  ​ – ...  ​ – __
  ​  x    ​  ​ ​ cosec x  ​  
= e​ x Æ 0 
​ ​ x Æ 0  ​   ​ 
2a 2 2 2a 4
​ lim ​ ​ ​ ___________________________________
=         ​      ​
=e – •
+ e​ ​ x Æ 0 
(  – log x
​ lim ​ ​ ______
  ​ cosec x )​
 ​  

x Æ 0 x 4

( x Æ 0+, log (sin x) Æ – •) 1


​ ​ ___
2a 4((   ( 
1
​    ​  – __
1 x4
​   ​   ​ x2 + __ )
​   ​ ​  __
8 a3 (  ) )
​    ​  ​ – ...  ​
)
= e– • +
(  1
 ​ __
x ​
​ lim ​ ​ – ​ ___________

e​ ​
x Æ 0 
    ​  ​
– cosec x cot x ​
) =  
​ lim ​ ​ ​ 
x Æ 0
_______________________    
x4
 ​     ​

It will provide a finite limit only when


– •   ( 
tan x
​ lim ​ ​ ​ ____ ​ × sin x  ​
x    ​ )
( 
=e + e​ ​ x Æ 0 

=e +e – • 0 2a
1
​ ___
​    ​ –
1
__
4 ) 1
​    ​  ​ = 0 and L = ___
​   3 ​ 
8a
=0+1 1
=1 fi 2a = 4 and L = ​ ___ 3 ​ 
8a

(  )
36. Ans. (a)
1 1
sec2x fi a = 2 and L = ​ ______  ​ = ___
  3  ​    ​ 
​Ú  ​  ​ ​  f (t) dx 8×2 64
Limit =  
​ lim  _________
2
​  ​  ​     ​  
​ 39. Ans. (b)
p
x Æ ​ __ ​  p2

( 
2 ___
)
4 x – ​    ​ x
16 1 t log (1 + t)
​    ​  ​ Ú ​  ​  ​ ​ _________
​ lim ​ __
Limit =    ​   
 ​ dt
x Æ 0 x3 0 t4 + 4
( 
f (sec 2
 x) ◊ 2sec2 x tan x – 0
)
( 
____________________

)
= 
​ lim   
​ ​ ​     
    ​  ​

(  )
x
p
x Æ ​ __ ​  2x t log (1 + t)
4
Ú​  ​  ​  ​ ​ _________
 ​   


= 
​ lim 
p

​ ​
x Æ ​ __ ​ 
( 
f (sec2
 x) ◊ sec2 x tan x
________________
​  x     ​   ​ ) = 
x Æ 0
0_____________
​ lim ​ ​ ​ 
t4 + 4
x3
 ​ 
     ​

(  ) ((   
4


2f (2)
= ​ _____
​  p  ​  
__
​   ​ 
 ​
= 
​ lim ​ ​ ​ 
x log (1 + x)
​ ​ __________  
(x4 + 4)
____________
 ​  
 ​ 

     ​
))
4 x Æ 0 3x2

(  )
8f (2)
= ​ _____
​  p   ​​
  
​ lim ​ ​ __
= 
x Æ 0 3 ( 
1 log (1 + x) _______
​   ​  × ​ _________
x    
1
​ × ​  4    
 ​  ​
)
(x + 4)
( 
8
= ​ __
​ p ​ f (2)  ​ ) = ___
1
​    ​ 
12
37. No questions asked in 2008. 40. Ans. (d)
3.66  Differential Calculus Booster
1
__ 2 1
​ lim ​  (1 + x log (1 + b2)​)​ ​x ​​ = 2b sin2q
Limit =   Thus, ​ _____________
     ​ = ___
​    ​ 
x Æ 0 (2a + 1) (a + 1) 60

fi ​e​
(  x log (1 + b2)
​ lim ​ ​ ​ ___________

x Æ 0 x     ​  
​ )​ = 2b sin q 2
fi (2a + 1) (a + 1) = 120
fi 2a2 + 3a – 119 = 0
fi ​e​log(1 + b )​ = 2b sin2q
2
fi 2a2 + 17a – 14a – 119 = 0
fi a (2a + 17) – 7 (2a + 17) = 0

fi (1 + b2) = 2b sin2q
fi (2a + 17) (a – 7) = 0


1 + b2
sin2q = ​ ​ ______
2b ( 
   
​  ​ ) fi
17
a = 7, – ​ ___ ​ 
2

( 
43. We have

1
sin2q = __
1
​   ​ ​  b + __
​   ​  ​ ≥ 1 )
{ 
2 b
}
1–x
– ax +  sin (x – 1) + a ​ ______
__ 
   ​ 1
2   ​ ​​  __________________
​ lim  ​         ​ ​​1 – ​÷x   ​ ​ = __
​   ​ 
fi sin q = 1 x Æ 1 x + sin (x – 1) – 1 4
p
fi q = ± __
​   ​ 
{ 
__ __

}
(1  + ÷
​ x 
  ​)  (1 – ÷
​ x 
  ​) 
2 sin (x  – 1) – a (x – 1) ​ ______________
__  
 ​
   1
  ​ ​​  ​  _________________
​ lim      ​  ​​ 1 – ​÷x   ​  ​ = __
   ​   ​ 
41. Ans. (b) x Æ 1 (x – 1) + sin (x – 1) 4

{ 
x2 +  x + 1
(  )
}
  ​ ​ ​ _________
Given ​ lim   ​  
– ax – b  ​ = 4 sin (x – 1) (1 + ​÷x 
  ​) 
__
x Æ 0 x+1 ​ ________ ​   –a
(x – 1) 1
(    ​ ​​  ​  ____________ __
​ lim 
x Æ 0
x2 + x  + 1 – ax2 – ax – bx – b
  ​ ​ __________________________
​     
x+1
 ​     ​ = 4 ) ​ lim 
x Æ 1
    ​  ​​
sin (x – 1)
1 + ​ ________ ​
(x – 1)


​= ​   ​ 
4

  ​ ​ ___________________________
​ lim 
x Æ 0
​     
x+1
 ​  ( 
(1 – a) x2 + (1 – a – b) x + (1 – b)
   ​ = 4 ) (  ) __​ 14 ​ 
1–a 2
​​ ​ _____ 
1+1
 ​  ​​ ​ =

​​( ​ _____
2 )
It is possible only when 1–a 12
 ​​ ​ = __
 ​   ​   ​ 
fi (1 – a) = 0, (1 – a – b) = 4 4
fi a = 1, a + b = – 3 (1 – a)2 = 1
fi a = 1 and b = – 4
(1 – a) = ±1
42. Ans. (a)
Limit a = 1 + 1


n Æ • (  a – 1
1a + 2a + ... + ◊ na
lim ​ ​ ​ _____________________________________
= ​        
   
(n + 1)  ((na + 1) + (na + 2) + ... + (na + n))
 ​  ​
a = 0, 2
)
The largest value of a is 2.

(  ) (  )
n
f (x)
2 ​S   ​ ​ ​ ra ​ lim ​ ​ ____
44. We have   ​     ​  ​
x Æ 0 g (x)
​ lim ​ ​ _____________________
 r = 1
=   ​       ​  ​
  

 ( ( 
n Æ •
(n + 1)  (2n2 a + n2 + n)
a – 1

(  ))
)
p p
sin ​ __
​   ​  sin ​ __

( 
​   ​  sin x  ​  ​

)
6 2
n ​ lim ​ ​ ________________
=   ​  p      ​  ​
2 ​S   ​ ​ ​ ​​ __
r a
​ n ​   ​​ ​ (  ) x Æ 0 __
​   ​  sin x
2

(   
​ lim ​ ​ ____________________
 r = 1
=   ​     
    ​  ​
(  ) (  )
( (  ​( __
​   ​  sin ​( __

)
n Æ • 1 a – 1 1
​​ 1 + ​ n ​  ​​ ​ ​ 2a + 1 + __
__ ​ n ​  ​ ​ __
​ 
p
6
​ lim ​ sin ​ _____________
=  
p
​   ​ sin ​ __
  
   ​ ×
) ) ​ _____________
​   ​ sin x  ​  ​
2
p
6
​   ​  sin x )​ )​
p
2
  
  
 ​  ​
p
__ p
__
(  ) ​( ​ 2 ​  sin x )​
p
__

(  )
x Æ 0
1 ​   ​ sin ​ ​   ​  sin x  ​
6 2
2 ​Ú  ​ ​  x  dx a
p
= ​ ________
​ 
0
 ​  
 ​ = __
​   ​ 
2a = 1 6
The Limit  3.67

(  ) (  ) (  )
n
​ a ​ ​)​ – e
e​ cos(​ e x2 sin b x b x
​ lim ​ ​ ​ _________
45. We have       ​  ​ = – ​ __
  ​    ​  ​   ​ ​ ​ _________ 
46. ​ lim   ​ × ___
  ​    ​  ​ = 1
a m

 (
a Æ  0 2 x Æ 0 a x – sin x b x

(  ) )
n
) – 1
e(​e cos(a 
​ ​ – 1 cos (​a n​ ​) – 1 e x3b
  ​ ​ ​ ____________
​ lim    
    ​ × ​  __________ ​ 
    ​ = – ​ __  ​   ​ ​ ___________________
  
[ 
​ lim  ​       ​  ​ = 1
cos(​a n​ ​) – 1 ​a m
]
( 
a Æ  0 ​​ 2 x Æ 0  x3 __ x5
__
a x – ​ x – ​    ​ + ​    ​– ....  ​

(  )
)  (
3! 5!
​___ n​ ​ ​ ​
a 2

)
2 sin  ​ ​ 
e​( ​e​ ​ – 1 )​ _________
n
cos(​a​ )
​  – 1
2 a 2n e x3b
  ​ lim ​ ​ ______________
​        ​ × ​   ​  
  × ____
​  m  ​  ​ = __
   ​    ​
(  )   ​ ​ ____________________
​ lim  ​    
[ 
     ​  ​ = 1
 n n 2
cos (​a​ ​) – 1 ​a ​ ​ 2
]
a Æ  0 4a
​​ ___

( 
​   ​  ​​ ​ x Æ 0 x3 x5
2 (a – 1) x + ​ __
​    ​– __
​    ​+ ...  ​

)
3! 5!


e​( ​e​
cos(​an

​ lim ​ ​ ​ _____________
)
​  – 1
  
    ​ ×
 sin
​ – 1 )​ ________
​ 
2 ___
 ​ ​  (  )
​a n​ ​
2
 ​   ​
 ​  
  ×
a 2n – m
______
​   ​    ​ = __
e
​    ​
It is possible only when

(  )
 n n 2
a Æ  0 cos (​a​ )​ – 1 ​a ​ ​ 2 2 1
​​ ___
​   ​  ​​ ​ a – 1 = 0, b = __​   ​ 
2 6
  ​  ​ __
​ lim 
e
( 
​    ​ × a 2n – m e
   ​ = __ )
​    ​
Thus, the value of 6 (a + b)

( 
a Æ  0 2 2
It is possible only when (2n – m) = 0
1
= 6 ​ 1 + __ )
​   ​   ​ = 7
6
m
​ __
n ​  = 2
Chapter

4 The Continuity and


Differentiability

(ii) ​ lim 
  ​ f(x) exists
Concept Booster x Æ c
(iii) ​ lim 
  ​ f(x) = f(c)
x Æ c
1.  Introduction
i.e ​ lim 
  + ​ f(x) = ​  
lim 

 ​ f(x) = f(c)
x Æ c x Æ c
Graphically, a function is continuous at a point if its graph
can be drawn at this point without raising the pen or pencil, It should be noted that continuity of a function at x = c is
otherwise it is discontinuous at that point. meaningful only if the function is defined in the immediate
neighbourhood of x = c, not necessarily at x = c.

3. Reasons of Discontinuity
(i) f(x) is not defined at x = c.
(ii) ​ lim 
  ​ f(x) does not exist.
x Æ c
  ​ f(x) π f(c)
(iii) ​ lim 
x Æ c
Geometrically, the graph of the function will exhibit
(Continuous function) a break at x = c. The graph as shown is discontinuous
at x = 1, 2 and 3.

(Discontinuous function)
4. Types of discontinuities
But only graphical approach is not sufficient, because
sometimes it is quite time taking (and in some cases it is even (i) Removal Discontinuity
impossible) to draw the complete graph of a function. (ii) Non-Removal Discontinuity
So we must have an analytical approach to analyse the
continuity of the function at any given point. 4.1  Removal Discontinuity
2. Continuity In case ​  
lim ​ f(x) exists but is not equal to f(c), then the func-
x Æ c

A function f(x) is said to be continuous at x = c if tion is said to have a removal discontinuity or discontinuity
of the first kind. In this case we can redefine the function
(i) f(c) is defined
such that ​  
lim ​ f(x) = f(c) and make it continuous at x = c
x Æ c
4.2  Differential Calculus Booster

are known as non-removal discontinuity or discontinuity of


the 2nd kind.
(a) Finite Discontinuity:
  + ​ f(x) π ​  
​ lim  lim 

 ​ f(x) π f(c)
x Æ c x Æ c

For examples:

(a) Missing Point Discontinuity: Where ​  


lim ​ f(x) exists
x Æ c
finitely but f(c) is not defined.
For examples:
(1 – x)(9 – x2)
(i) f(x) = ​ ____________
  
 ​  
has a missing point dis-
(1 – x)
continuity at x = 1.
(i) f(x) = x – [x] at all integral x


1
(ii) f(x) = tan–1​ __ (  )
​ x ​  ​ at x = 0
1
(iii) f(x) = ______
​    1   ​ at x = 0
__
1 + ​2​ ​x ​​
(Note that f(0 +) = 0, f(0 –) = 1).
(b) Infinite Discontinuity:
​ f(x) = ​  
​ lim 
  +  lim 

 ​ f(x) = f(c) = •
x Æ c x Æ c
sin x
(ii) f(x) = ​ ____ ​ has a missing point discontinuity at
x   
x = 0.
(b) Isolated Point Discontinuity: Where ​  
lim ​ f(x) exists
x Æ c
lim ​ f(x) π f(c)
and f(c) is also exists but ​  
x Æ c
For examples:
x2 – 16
(i) f(x) = ______
​  , x π 4 and f(4) = 9 has an isolated
 ​ 

x–4
point of discontinuity at x = 4
For examples:
1
(i) f(x) = _____
​       ​ at x = 4
x–4
1
(ii) f(x) = _______
​       ​ at x = 3
(x – 3) 4
p
(iii) f(x) = 2 tan x at x = __ ​   ​ 
2
cos x
(iv) f(x) = ​ _____
x    

  at x = 0.

(c) Oscillatory Discontinuity:


Ï0 , x Œ I For example:
(ii) f(x) = [x] + [– x] = Ì has an isolated
Ó- 1 , x œ I
point of discontinuity at all integer points.

1
(i) f(x) = sin​ __ (  )
​ x ​  ​ at x = 0

Note: In all these cases the value of f(c) of the function


4.2  Non-Removal Discontinuity f(x) at x = c (point of discontinuity) may or may not exist
In case ​  
lim ​ f(x) does not exist then it is not possible to make but in every case ​  
lim  
​ f(x) does not exist.
x Æ c x Æ c
the function continuous by redefining it. Such discontinuous
The Continuity and Differentiability  4.3

(f) If f(x) is continuous at x = c and g(x) is continuous


at x = f(c) then g(f(x)) is continuous at x = c.

In case of the discontinuity of the second kind, the non-


negative difference between the value of the R.H.L at x = c
p
and L.H.L at x = c is called the Jump of Discontinuity. Example-1.  Let f(x) = sin x is continuous at x = ​ __ ​  and
A function having a finite number of jumps in a given 2
interval is called a Piece-wise Continuous or Sectionally g(x) is also continuous at x = 1. Then the function (go f )(x)
p
Continuous function in this interval. = cos(sin x) is continuous at x = ​ __ ​ .
2
Very Important Points to Remember Example-2.  Let f(x) = |x – 2| is continuous at x = 2 and
(a) If f(x) is continuous at x = a and g(x) is discontinu- g(x) = x2 + 1 is continuous at x = 0.
ous at x = a, then both the functions f(x) + g(x) and Then (go f )(x) = |x – 2|2 + 1 is continuous at x = 2.
f(x) – g(x) are discontinuous at x = a.
(g) If f(x) is continuous, then | f(x)| is also continuous.
For example:
Let f(x) = x and g(x) = [x] Example-3.  Let f(x) = x is continuous everywhere
Clearly, f(x) is continuous at x = 0 and g(x) is dis- Then g(x) = |x| is also continuous everywhere.
continuous at x = 0.
x + 1
But both the function x + [x] and x – [x] are discon- Example-4.  Let f(x) = ​  ______
2
  ​ 
and g(x) = |x – 2| are con-
tinuous for all x. x +  4

| 
tinuous at x = 0.
(b) If f(x) is continuous at x = a and g(x) is discontinuous
at x = a then the product function h(x) = f(x)g(x) is
not necessarily be discontinuous at x = a.
all x.
x + 1
Then (go f)(x) = ​ ​  ______

x2 + 4

|
 ​ – 1  ​ is also continuous for

For example: 5. Continuity of an Interval

Let f(x) be a real function and a and b be two real numbers


Ï Ê 1ˆ
Ôsin , xπI such that a < b
Let f(x) = x and g(x) = Ì ÁË x ˜¯
(i) A function f(x) is said to be continuous in the open
Ô0 , x=0
Ó interval (a, b) if it is continuous to each point of
(a, b).
Clearly f(x) is continuous at x = 0 and
g(x) is discontinuous at x = 0.
Thus, the product x sin ​ __
1
(  )
​ x ​  ​ is continuous at x = 0.
(c) If f(x) and g(x) both are discontinuous at x = a, then
the product function h(x) = f(x) ◊ g(x) is not necessar-
ily be discontinuous at x = a.
For example:
Let f(x) = [x] and g(x) = [– x]
Clearly f(x) and g(x) are discontinuous at x = 0.
But the product f(x) ◊ g(x) = [x] ◊ [– x] is continuous at (ii) A function f(x) is said to be continuous in the closed
x = 0. interval [a, b] if
(d) Point functions are to be treated as discontinuous. (a) it is continuous in (a, b)
(e) A continuous function whose domain is in closed (b) ​ lim 
  +  ​ f(x) = f(a) (or f is right continuous at
interval must have a range also in a closed interval. x Æ a
x = a)
4.4  Differential Calculus Booster

(c) ​ lim 
  – ​  f(x) = f(b) (or f is left continuous
x Æ b
at x = b).
Y

(Continuous
from left)
(Continuous
from right)
X¢ X
O x=a x=b 8. Differentiability

8.1.  Introduction
6. Single Point Continuity In calculus (a branch of mathematics), a differentiable func-
A function which are continuous only at one point are said tion of one real variable is a function whose derivative exists
to be exihibit single point continuity. at each point in its domain. As a result, the graph of a dif-
ferentiable function must have a non-vertical tangent line at
For example:
each point in its domain, be relatively smooth, and cannot
Ï x , x ŒQ contain any breaks, bends, or cusps.
(i) f(x) = Ì is continuous at x = 0
Ó - x , x œQ 8.2 Differentiabilty of a function at a point
Ï x, x Œ Q (i) The right hand derivative of f(x) at x = a is denoted
(ii) g(x) = Ì is continuous at x = 0 by f  ¢(a +) and is defined as
Ó0, x œQ

7. Properties of Continuous Functions


f  ¢(a +) = ​   lim 
hÆ0 +
 (  f(a + h) – f(a)
​ ​ _____________
​ 
h
      
  
) ​  ​, provided the limit
exists.
Let f(x) and g(x) be two continuous functions at x = c and (ii) The right hand derivative of f(x) at x = a is denoted
k be a non-zero real number. Then by f  ¢(a–) and is defined as

(  )
(i) k f(x) f(a + h) – f(a)  
(ii) f(x) + g(x) f  ¢(a –) = ​ lim   +  ​ ​ _____________
​         ​  ​, provided the limit
h Æ 0 h
(iii) f(x) – g(x) exists.
(iv) f(x) ◊ g(x) are also continuous at x = c and Thus, a function is said to be differentiable (finitely)
(v) f(x)/g(x) is also continuous, provided g(c) is non-zero at x = a, if f  ¢(a +) = finite.
at x = c.
(vi) If f(a) and f(b) are of opposite signs, then there exists
By definition, f  ¢(a +) = ​ lim    – 
h Æ 0(  f(a + h) – f(a)
​ ​ ____________
​ 
h )     
       ​  ​

at least one solution of the equation f(x) = 0 in the


open interval (a, b). i.e. ​ lim 
  + 
h Æ a
 ​ ( 
f(a + h) – f(a)
​ ​_____________
h
  
     ​   )
​ = ​ lim 
  – 
h Æ a ( 
f(a – h) – f(a)   
​ ​ ​ ___________
h
     ​   ​ )
8.3  Geometrical Meaning of a Derivative

(vii) If k is any real number between f(a) and f(b),


f(a + h) – f(a)    _____________
f(a + h) – f(a)
then there exist atleast one solution of the equation m(PQ) = tan a = _____________
​    
     ​= ​       ​ 
f(x) = k in the open interval (a, b). Which is known (a + h) – a h
as Intermediate Value Theorem. when h Æ 0, the point Q moving along the curve tends to
P. i.e. Q Æ P
The Continuity and Differentiability  4.5

The chord PQ approaches the tangent line PT at the


point P.

So a Æ q

h Æ 0 ( 
  f (a + h) – f(a)
lim ​ ​ _____________
Thus, tan q = ​   ​ 
h
       )
​  ​

fi m = f  ¢(a) = ​  
h Æ 0 ( 
  f(a + h) – f(a)
lim ​ ​ ​ ____________
h )
     ​  ​

Thus, a function is differentiable at a point x = a, if there (f(x) is not differentiable at x = a)


is a tangent at x = a. 3. A function f(x) is differentiable if there exists a
unique tangent to each point on the given curve.
Note:
1. A function f(x) is differentiable in its domain if it
is a smooth curve.

8.4 Concept of tangent and its association with


derivability
2. A function f(x) is differentiable, if there is a no Tangent: The tangent is defined as the limiting case of a
break point, hole point or corner point or a kink chord or a secant.
point on the given curve.

Slope of the chord joining (a, f(a)) and


f(b) – f(a)
(f(x) is not differentiable at x = a) (b, f(b)) =​  _________
  ​ 

b–a
Slope of the line joining (a, f(a)) and

  f(a + h) – f(a)
(a + h, f(a + h)) =​  _____________        ​
h
  f(a + h) – f(a)
R.H.D = f(a +) = ​ l im  ​  ​ _____________
       ​ 
hÆ0 h
  f(a – h) – f(a)
L.H.D = f(a–) = ​   lim   ​  ​ _____________
     ​ 
hÆ0 – – h
A function will have a tangent at point x = a if f ¢(a +)
= f ¢(a–) (may or may be finite) and equation of tangent at
(f(x) is not differentiable at x = a) (a, f(a)) is given by y – f(a) = f ¢(a) (x – a)
4.6  Differential Calculus Booster

Notes: If f(x) is a function such that


1. y = x3 has x-axis as tangent at origin. R.H.D = f(a +) = l and
L.H.D = f(a–) = m
2. Tangent is also defined as the line joining two infi-
nitely small close points on a curve. Case I:   If l = m = some finite value, then the function
f(x) is differentiable as well as continuous.
3. y = sgn (x) will have a vertical tangent at x = 0
4. y = |x| does not have tangent at x = 0 as L.H.D
π R.H.D.
5. Discontinuous function can also have vertical tan-
gent, namely, y = sgn (x) at x = 0
6. A function is said to be derivable at x = a if
their exist a tangent of finite slope at that point
f ¢(a +) = f ¢(a–) = finite value. Case II:   If l π m = but both have some finite value, then
7. If a function f(x) is differentiable at x = a, the the function f(x) will not be differentiable but it will be
graph of f(x) will be such that there is a unique continuous.
tangent to the graph at the corresponding point. But
if f(x) is non-differentiable at x = a, there will not
be unique tangent at the corresponding point of the
given curve.

8.5  Relation between the Continuity and Derivability


(i) If f  ¢(a) exists, then f(x) is continuous at x = a Case-III:   If at-least one of the l or m is infinite, then
the function is non differentiable but we can not say about
(ii) If f(x) is derivable for every point of its domain, then
continuity of f(x)
it is continuous in that domain.
Proof: Let f(x) be a real function and a Œ R

Given f ¢(a) = ​  


x Æ a
(  f(x) – f(a)
lim ​  ​ ​ _________
x – a     
​  ​)
To prove f(x) is continuous at x = a.
i.e. ​ lim    ​   f (x) = f(a)
x Æ a

Now, ​  
lim ​    f (x) = ​  
lim ​  ((f(x) – f(a)) + f(a))
x Æ a x Æ a
Notes:
= ​  
x Æ a
( ( 
  f(x) – f(a)
lim ​  ​  ​ ​ _________
x – a      )
​  ​ × (x – a) + f(a)  ​ ) 1. Differentiable
2. Continuous


Continuous
Can be a differentiable.
lim ​ ​ ( ​  _________ ​ )​ ◊ ​  
  f(x) – f(a)
= ​   x – a      lim ​ × (x – a) + ​  
lim ​ f(a) 3. Discontinuous fi Non-differentiable.
x Æ a x Æ a x Æ a
4. Both one sided fi Continuous.
= f  ¢(a)0 + f(a)
derivative exists
= f(a)
8.6. Derivability over an interval
Thus, f(x) is continuous at x = a
(i) Open Interval: f(x) is said to be derivable over an
Therefore, every differentiable function is
open interval (a, b) if it is derivable at each and every
continuous.
point of (a, b).
Note: The converse of the above result is not true.
i.e. If ‘f ’ is continuous at x, then ‘f ’ is derivable at
x is not true.

For example:
the function f(x) = |x – 2| is continuous at x = 2 but
not derivable at x = 2.
(iii) If a function is not differentiable but is continuous
at x = a, it geometrically implies a sharp corner or
kink at x = a.
The Continuity and Differentiability  4.7

(ii) Closed Interval:  f(x) is said to be derivable over a


closed interval [a, b] if
(i) it is differentiable on (a, b)
(ii) it is right differentiable at left end.

i.e. f ¢(a +) = ​ lim 


  + 
h Æ 0
( 
f(a + h) – f(a)
​ ​ ​ ____________
h
     ​  
​ )
(iii) it is left differentiable at right end.

i.e. f ¢(b–) = ​   lim  


h Æ 0 – ( 
 f(b – h) – f(b)
​ ​ ​ ____________
– h
       )
​  ​ f(a + h) – f(a – h)
Slope of AB =​  _______________
h
     ​ 
(iv) for any point c such that a < c < b, f ¢(c +) and
f ¢(c–) exists finitely and are equal. f(a + h) – f(a)
Slope of CB =​  ____________      ​ 
h
Note: For checking the derivability in an interval, then
following points must kept in your mind.  f(a + h) – f(a – h)
(i) the limit of​  ________________       ​ as h Æ 0 is f ¢(a)
2h
1. All those point where discontinuity may arise
when f ¢(a) is exist.
2. Modulus functions are also non-differentiable and
(ii) it usually gives a better approximation of f ¢(a) for a
hence should be checked at their critical points.
given value of h then
3. Every sine and cosine functions are differentiable f(a + h) – f(a)
everywhere. Fermat’s difference quotient​  ____________      ​ 
h
4. tan–1 x, cot–1 x, all polynomials and exponential func-
f(a + h) – f(a – h)  
tions are differentiable everywhere. In fact, ​   lim ​ ________________
​         ​
h Æ 0 2h
5. logarithmic and trigonometric functions are differ-
ential in their domains.
6. Modulus function and signum function are non-
differentiable at x = 0.

1
= __ (  ( ( 
​   ​ ​  ​  
2 h Æ 0
f(a + h) – f(a)
lim ​ ​ ​ ____________
​ 
h ) ( 
     ​  
  f (a – h) – f(a)
​ + ​ _____________
​ 
– h
       )))
​  ​  ​  ​

1
Hence, y = |f(x)| and y = sgn(f(x)) should be checked = __
​   ​  ( f ¢(a +) + f  ¢(a–))
2
at points where f(x) = 0.
We notice that it is equal to f  ¢(a) if f is differentiable
7. Power function y = x p, 0 < p < 1 is non-differentiable
at x = a.
at x = 0. Hence y = (f(x)) p should be checked at
points where f(x) = 0.
8.8 Twice differentiability
8. The inverse trigonometric functions
y = sin–1x, cos–1x, cosec–1x, sec–1x A function f(x) is twice differentiable at x = a if its deriva-
are not differentiable at x = ± 1 tive f  ¢¢(x) is differentiable at x = a
Hence, y = sin –1( f(x)), cos –1( f(x)), cosec –1( f(x)),
sec–1(f(x)) h Æ 0
​  ( 
f ¢(a + h) – f ¢(a)
lim ​ ​ ______________
The limit f  ¢¢(a) = ​  
h
     ​   ​ exists.)
should be checked at points where f(x) = ± 1.
9. Greatest integer function and fractional part func-
tions are non-differentiable at all integral points of x.
Alternatively, f  ¢¢(a) = ​  
x Æ 0
( 
  f ¢(x) – f ¢(a)
lim ​ ​ ​  __________
x – a      ​  ​ )
Hence, y = [f(x)] and y = {f(x)} should be checked
Hence, if f ¢(a) is exists then
at points where f(x) = n, n ΠI.
__   f(a + h) – f(a – h)
10. The nth root function f(x) = n÷ ​ x 
   ​ is non-differentiable
  ​ ​  ________________
​ lim         ​ = f ¢(a).
at x = 0 h Æ 0 2h
11. If f(x) is differentiable at x = a, then the function
f(x)|f(x)| is also differentiable at x = a.
8.9 Differentiability of Composite functions
Theorem:  If f(x) is differentiable at x = a and g(x)
is differentiable at x = f(a) then the composite function
8.7  Centered difference Quotient (gof)(x) is differentiable at x = a.
f(a + h) – f(a – h) A function of a function composed of a finite number of
The centered difference quotient is ​ ______________
       ​ and is differentiable functions is a differentiable function.
2h
p
used to approximate f ¢(a) in numerical work. Example-5.  Let f(x) = sin x is differentiable at x = __
​   ​  and
2
4.8  Differential Calculus Booster

(  )
Ï x2 - 1 , x £ 1 p Ï x2 : x>0
g(x) = ÔÌ is differentiable at x = f ​ __
​   ​   ​ = 1 Ô
ÔÓ2 x - 2, x > 1 2 f(x) ◊ g(x) = Ì0 : x=0
Ô 2
Hence the composite function (go f)(x) is differentiable at Ó- x : x < 0
p
x = __
​   ​  is differentiable at x = 0.
2
sin x Example-9. Let f(x) = x and
Example-6.  Let f(x) = ​ ______    ​and g(x) = x|x| are differ-
x2 + 1 Ï Ê 1ˆ
Ô x sin Á ˜ : x π 0
entiable for all x. Hence the composite function (gof)(x) g(x) = Ì Ë 2¯

|  |
Ô0 : x=0
Ó
sin x _____ sin x
= ​ _____
2
  
 ​​ ​  2     ​  ​ is also differentiable for all x. 3. If f(x) and g(x) are two differentiable functions,
x +1x +1
Then the function f(x) is differentiable at x = 0 but
Theorem:  If the function f(x) is differentiable every where g(x) is non-differentiable at x = 0
and the function g(x) is also differentiable everywhere Hence the product f(x) ◊ g(x) is differentiable at
then the composite function (go f ) is also differentiable x = 0.
everywhere. Example-10. However, the function f(x) = x is differ-
Example-7.  Let f(x) = sin x and g(x) = cots x entiable at x = 1 and g(x) = [x] is non-differentiable
at x = 1 but the product f(x) ◊ g(x) is non-differentiable
Clearly, f(x) and g(x) is differentiable everywhere.
at x = 1.
Thus, f(g(x)) = sin(cos x) and g(f(x)) = cos (sin x) is alo
4. The quotient f(x)/g(x) is not necessarily differentiable
differentiable everywhere.
at x = a
Ï x + 1, x ≥ 0
8.10 Algebra of differentiable functions Let f(x) = x2 (x2 – 1) and g(x) = Ì
Ó x - 1, x < 0
1. If f(x) and g(x) are differentiable functions at x = a,
Here, f(x) is differentiable at x = 0 and g(x) is non-
then
differentiable at x = 0
(i) cf(x) is differentiable at x = a, where c is any
non-zero constant. ÏÔ x 2 ( x - 1), x ≥ 0
Therefore, f(x)/g(x) = Ì
(ii) f(x) + g(x) is differentiable at x = a 2
ÔÓ x ( x + 1), x < 0
(iii) f(x) – g(x) is differentiable at x = a is differentiable at x = 0.
(iv) f(x) ◊ g(x) is differentiable at x = a
5. If f(x) and g(x) both are non-differentiable at x = a,
(v) f(x) /g(x) is differentiable at x = a, provided g¢(a) then the follwing cases may arise:
is non-zero.
(i) The functions f(x) + g(x) and f(x) – g(x) are not
2. If f(x) is differentiable at x = a and g(x) is non- necessarily non-differentiable at x = a.
differentiable at x = a, then the following cases may
However, atmost one of f(x) + g(x)
arise.
and f(x) – g(x) can be differentiable at x = a.
(i) Both the functions f(x) + g(x) and f(x) – g(x) is
Thus both of them can not be differentiable at
non-differentiable at x = a.
x = a.
Let f(x) = x and g(x) = |x|.
Let f(x) = [x] and g(x) = {x}. Here both f(x) and
Obviously, f(x) is differentiable at x = 0 and g (x) g(x) are non differentiable at x = 0.
is non-differentiable at x = 0.
Thus the sum f(x) + g(x) is differentiable at
But the functions x + |x|, x – |x| are non-differ- x = 0, however the difference f(x) – g(x) is non-
entiable at x = 0. differentiable at x = 0.
(ii) f(x)◊ g(x) is not necessarily non-differentiable But this does not mean that one of the fun-
at x = a. We need to find the result by first tions f(x) + g(x) and f(x) – g(x) must be
principles. differentiable.
Example-8. Let f(x) = x2, g(x) = sgn(x) Let f(x) = 2[x] and g(x) = {x}. Here both f(x)
Obviously, f(x) is differentiable at x = 0 and g(x) is and g(x) are non-differentiable at x = 0.
non-differentiable at x = 0. Then the functions f(x) + g(x) and f(x) – g(x)
But the product of the functions are non-differentiable at x = 0.
The Continuity and Differentiability  4.9

(ii) f(x) ◊ g(x) is not necessarily non-differentiable at (ii) We manipulate f(x + h) – f(x) in such a way that the
x = a. given functional rule is applicable.
Let f(x) = [x] and g(x) = [– x]. Here, both the Now we apply the functional rule and simplify the
functions f(x) and g(x) are non-differentiable at R.H.S to get f ¢(x) as a function of x.
x = 0, but the product function [x] ◊ [– x] is dif- (iii) Then we integrate f ¢(x) to get f(x) as a function of x
ferentiable at x = 0. and a constant of integration.
Further more, let f(x) = [x] and g(x) = {x}. (iv) Finally, we apply the boundary conditions to deter-
Here both the functions f(x) and g(x) are non- mine the value of the constant of integration.
differentiable at x = 0, but the product [x]. {x}
is also non-differentiable at x = 0. ÔÏ x 2 - 1, x ≥ 0
Let f(x) = Ì and
(iii) f(x)/g(x) is not necessarily non-differentiable at ÔÓ x + 1 , x < 0
x = a.
Ï x + 1, x ≥ 0
8.11 Functional Equations g(x) = Ì
Ó x - 1, x < 0
We should follow the following steps to determine the func-
tions which are differentiable Then both the functions f(x) and g(x) are non dif-
  f(x + h) – f(x) ferentiable at x = 0, but the function f(x)/g(x) is dif-
lim ​ ​  _____________
(i) First we write f  ¢(x) = ​          ​ ferentiable at x = 0.
h Æ 0 h

Exercises

Ï xx -1
(Problems Based on Fundamentals) Ô : xπ0
6. Show that  (x) = Ì log(1 + 2 x )
Type-I Ô 7 : x=0
Ó
1. Test the continuity of the function f (x) at the origin is discontinuous at x = 0.
Ï | x| Ï sin 3 x
Ô , xπ0 : x<0
f (x) = Ì x Ô tan 2 x
ÔÓ 1, x = 0 Ô
Ô3
2. Show that the function f (x) is given by 7. Show that f (x) = Ì : x=0
Ô2
Ï sin x Ô log(1 + 3 x )
Ô + cos x, x π 0 Ô e2 x - 1 : x>0
f (x) = Ì x is continuous at x = 0.
Ó
ÔÓ 2 x=0
is continuous at x = 0.
3. Discuss the continuity of the function
Type-II
Ï ax + bx + cx - 3 8. For what value of k is the function
Ô : xπ0
f (x) = Ì x at x = 0. Ï sin 5 x
Ô log abc : x=0 Ô xπ0
Ó f (x) = Ì 3 x is continuous at x = 0?
ÔÓk x=0
4. Test the contnuty at x = 0 where
p
9. Find the value of k, if f (x) is continuous at x = __
​   ​ ,
Ï e3 x - 1 2
Ô : xπ0 Ï k cos x p
f (x) = Ì log(1 + 5 x ) ÔÔ p - 2 x x π 2
Ô5 : x=0 where f (x) = Ì .
Ó p
Ô3 x=
ÔÓ 2
Ï x - | x|
Ô : xπ0
5. Show that f (x) = Ì 2 is discontinuous Ï 2 x + 2 - 16
ÔÓ 2 Ô : xπ2
: x=0 10. If f (x) = Ì 4 x - 16 is continuous at
at x = 0. Ôk : x=2
Ó
x = 2, then find k.
4.10  Differential Calculus Booster

Ï cos2 x - sin 2 x - 1 18. If f (x) is continuous at x = 0 for which the


Ô : xπ0
11. If f (x) = Ì x2 + 1 - 1 e2x – 1 – x (e2x + 1)
function f (x) = ​ _________________  ​      , x π 0,
Ô x3
Ók , : x=0
then find f (0).
is continuous at x = 0, then find k. 19. Let f be a continuous function on R such that
Ï1 - cos kx
ÔÔ x sin x
12. If f (x) = Ì
: xπ0
is continuous at
1
4(  )
f ​  ​ __n   ​  ​ = (sin en) e– n​ 2
n +1
n2
​ ​ + ​ ______
2
   ​, then find f (0).

Ô1 : x=0 Ï 1
ÔÓ 2 Ô xn ¥ e x
Ô 1
, , x<0
x = 0, find k. Ô 1 + ex
ÔÔ
Type-III 20. Let f (x) = Ì 0 , x=0.
13. Determine the values of a, b, c for which the Ô
Ô x n sin ÊÁ ˆ˜
1
function: , x>0
Ô Ë x¯
Ï Ô
Ô sin (a + 1) x + sin x ÔÓ
Ô : x<0
ÔÔ x Find the smallest n in W such that f (x) is
f (x) = Ì c : x = 0 is continuous continuous.
Ô
Ô x + bx - x
Ô
2

3
: x>0 ( 
1 ______
21. Let f (x) = ​ ​ __
2
x ​ – ​ e2x – 1  )
 ​  ​.

ÔÓ bx 2 If f (x) is continuous at x = 0, then find f (0).


______
at x = 0 2 – 4​÷x  2 + 16 ​ 
22. Let f (x) = ​ ___________
  
   ​.
Ï ( x - 4) cos 2x – 1
Ô | x - 4| + a : x < 0
ÔÔ If f (x) is continuous at x = 0, then find f (0). 
14. If f (x) = Ì a + b : xπ0
Ô ( x - 4) Properties of Continuous Functions
Ô +b : x>0
ÔÓ | x - 4| 23. Prove that the equation x – cos x = 0 has a root in​
is continuous at x = 4, then find a, b. (  )
p
0, __
​   ​   ​.
2
15. Find the ordered pair (a, b) such that
24. Prove that the equation 2 tan x + 5x – 2 = 0 has at
Ï
Ô be x - cos x - x
Ô , x>0
p
least one root in ​ 0, __
4(  )
​   ​   ​.
Ô x2 25. Prove that the equation x ◊ 2x – 1 = 0 has only one
Ô
f (x) = Ì a , x=0 positive root in (0, 1).
Ô p
Ô 2 ÊÁ tan -1 (e x ) - ˆ˜ 26. Prove that the equation e2x + ex + 2 sin– 1 x + x – p
Ô Ë 4¯ = 0 has at least one real solution in [0, 1].
ÔÓ , x<0
x Disconinuity
is continuous at x = 0. 27. Prove that the function
Type-IV Ï2 - x : x ≥ 1
f (x) = Ì is discontinuous at x = 1

(  (  x
log ​ 1 + __ ) (  ) x
​ a ​  ​– log ​ 1 – __
16. Let f (x) = ​ ​ ____________________
    )
​   ​  ​
b
  ​  ​ x π 0. Find the
Óx + 2 : x < 1
x  28. Prove that the function
value of f (x) at x = 0 so that f becomes continuous
Ï sin 3 x + 2 x
at x = 0. Ô : xπ0
f (x) = Ì sin 7 x + sin 3 x
17. If f (x) is continuous at x = 0 such that
Ô 2 : x=0
sin 3x + A sin 2x + B sin x Ó
f (x) = ​ _____________________
    ​   , x π 0, is discontinuous at x = 0
x5
then find f (0).
The Continuity and Differentiability  4.11

29. Discuss the continuty of the function x2 – 7x + 2014


(xii) f(x) = _____________
​  __   
    ​
÷    cos x  – 1
​ 2 ​ 
Ï esin x - e x
Ô :xπ0 36. Discuss the continuity of the function
f (x) = Ì x - sin x at x = 0
Ô f(x) = [[x]] – [x – 1], where [,] denotes the greatest
Ó 2 :x=0
integer function.
30. Discuss the continuity of the function 37. Discuss the continuity of f(x) in [0, 1]
where f(x) = [sin p x].
ÏÊ 1 - cos 4 x ˆ
38. Discuss the continuity for the function
Ô
f (x) = ÌËÁ x ˜ : x π 0 at x = 0.
¯ f(x) = [x] + [– x].
Ô
Ó 2 : x=0 39. Discuss the continuity of f(x) in [– 2, 2], where
f (x) = x + {– x} + [x], x Œ I.
Discontinuity
40. Find the number of points of discontinuity of each
of the following functions.
Ï | x|
Ô : xπ0 (i) f (x) = [sin x], " x Œ [0, 2p]
31. Let f(x) = Ì x __
ÔÓ0 : x = 0    sin x], " x Œ [0, 2p]
(ii) f (x) = [​÷2 ​ 
(iii) f (x) = [sin x + cos x], " x Œ [0, 2p]
Find the length of the Jump.
32. Let ​  
x Æ 0
(  (  ) )
1
lim ​ ​ tan–1​ __
​ x ​  ​  ​


(iv) f (x) = [sin p x], " x Œ [0, 2]
(v) f (x) = [2 cos x], " x Œ [0, 2p]
Find the length of the Jump. 41. Find the number of points of discontinuity of the
33. Let ​  
x Æ 3 x
(  )
[x]
lim ​​ ___
​     ​  ​ function f(u) = _________
​  2
u +u–2
1
   
1
 ​ where u = _____
​     
1–x
 ​.

Find the length of the Jump. 42. Find the number of points of discontinuity of the
34. Let ​  
lim ​  [sin x] function f(f(f(x))),
x Æ 0
Find the length of the Jump. where f(x) = 1/1 – x.
35. Find the number of the points of discontinuity of 43. Discuss the continuity of the function
1
each of the following functions h(x) = f(g(x)), where f(x) = _____
​      ​and g(x) = x2 + 5.
x–6
1
(i) f(x) = _____
​      ​ Ï1 + x : 0 £ x £ 2
x–1 44. Let f(x) = Ì

1
(ii) f(x) = _____
​       ​ Ó3 - x : 2 < x £ 3
|x| – 1 Discuss the continuity of the function g(x), where
1
(iii) f(x) = _____
​       ​ g(x) = f(f(x)).
|x| + 2
Ï- 1 : x < 0
x2 + 1 Ô
(iv) f(x) = _____
​  2  
 ​ 45. If f(x) = Ì0 : x = 0 and g(x) = x(1 – x2)
x –4 Ô1 : x > 0
1 Ó
(v) f(x) = _____
​     ​ 
loge x then discuss the continuity of the function h(x),
1 where h(x) = f(g(x)).
(vi) f(x) = ______
​     ​ 
loge|x| 46. Discuss the continuity for the function f(x), where
1 f(x) = |x + 1|(|x| + |x – 1|).
(vii) f(x) = _________
​       ​ 47. Let f(x) = |x – 2| – 1, 0 £ x £ 4 and g(x) = 2 – |x|,
loge|x – 2|
– 1 £ x £ 3
1
(viii) f(x) = __________
​       ​ Then discuss the continuity of the function (fog)(x).
loge (x2 – 1) _____ 1
1 48. Prove that the equation ​÷x  – 5   ​= _____
​      ​ has at-least
(ix) f(x) = ___________
​       ​ x+3
loge |(x2 – 1)| one real root in (5, 6).
1 49. Show that the equation x5 + 3x4 + x – 2 = 0 has at-
(x) f(x) = __________
​  2      ​
x – 3|x| + 2 least one root in [0, 1].
50. Show that the equation x5 – 3x + 1 = 0 has a real
  x2 + 4x + 1
(xi) f(x) = ​ ___________
  
   ​ root in [1, 2].
2 sin x  – 1
4.12  Differential Calculus Booster

51. Show that the equation x3 + x2 – 3x – 3 = 0 has root If f (x) is differentiable at x = 1/2. Find the value of
in [1, 2]. a + b + 10.
Intermediate Value Theorem Ï Ê 1ˆ
Ô x sin Á ˜ : x π 0
52. Show that the equation x5 + x = 1 has a real root. 68. Let f (x) = Ì Ë x¯
Ô0 : x=0
53. Show that the equation x5 + 3x4 + x – 2 = 0 has Ó
atleast one root in [0, 1]. Examine the continuity and the differentiability at
54. Show that the equation x5 – 2x3 + x2 – 3x + 1 = 0 x = 0.
has atleast one root in [1, 2].
55. Show that the equation 2x3 + x2 – x – 5 = 0 has a Ï 1
Ô xe x
solution in [1, 2]. Ô 1
: xπ0
69. Let f(x) = Ì
Differentiability Ô 1 + e 2

56. Check the differentiability of the function Ô0 : x=0


Ó
f (x) = |x  –  2| at x = 2.
Examine the continuity and the differentiability at
57. Check the differentiability of x = 0.
ÏÔ x : x < 1 70. Let f (x) = |x – 1| ([x] – {x})
f (x) = Ì 2 at x = 1. Examine the continuity and differentiability at
ÔÓ x : x ≥ 1
x = 1.
58. Check the differentiability of the function
Ï Ê p xˆ
f (x) = ln2 x at x = 1. Ôsin : x<0
71. Let f (x) = Ì ÁË 2 ˜¯
59. Check the differentiability of the function
Ô[2 x - 3]x : x ≥ 0
f (x) = e|x| at x = 0. Ó
60. Check the differentiability of the function Examine the continuity and the differentiability at
x = 1.
ÔÏ3 x : -1 £ x £ 1
72. Check the differentiability of the function
f  (x) = Ì at x = 1.
ÔÓ4 - x : 1 < x < 4
f (x) = e–|x| in [– 2, 2]
61. Check the differentiability of the function
73. Check the differentiability of the function

|  |
f (x) = sin x + |sin x| at x = 0.
1
62. Let f (x) = [x] tan(p x), where [,] = G.I.F f (x) = ​ __​    ​  – 1  ​ in [–  4, 4]
Find the R.H.D at x = k, where k Π1 |x|
63. Let f (x) = [x] sin(p x), where [,] = G.I.F 74. Check the differentiability of the function
Find the L.H.D at x = k, where k Œ 1 f (x) = sin x + sin |x| in [– 2p, 2p].
64. Check the differentiability of the function 75. Check the differentiability of the function
Ï 2 Ê 1ˆ f (x) = min {|x + 1|, |x|, |x – 1|} in [– 4, 4].
Ô x sin Á ˜ : x π 0 76. Check the differentiability of the function
f (x) = Ì Ë 2¯ at x = 0.
Ô0 x
Ó : x = 0 f (x) = _____
​       ​ in R.
1 + |x|
65. Check the differentiability of the function
77. Check the differentiability of the function

|  |
Ï x
ÔÔ 1
: xπ0 x
f (x) = ​ _____
​      ​  ​ in R.
f (x) = Ì1 + e x at x = 0 . x–1
Ô
ÔÓ0 : x=0 78. Check the differentiability of the function
f (x) = |x| + |x2 – 1| in R.
Relation between Continuity and Differentiability
Ïax 2 + 1 : x£0 79. Check the differentiability of the function
66. If f (x) = ÔÌ is differentiable at
2 f (x) = |x2 – 1| + |x2 – 4| in R.
ÔÓ x + ax + b : x>0
x = 1, find a and b. 80. Let f (x) = sgn (x) and g(x) = x (1 – x2).
Ï1 1 Examine the differentiability of the function
ÔÔ | x | : | x| ≥ f(g(x)).
2
67. Let f (x) = Ì 81. Let f (x) = sin– 1 |sin x|. Examine the differentiability
Ôa + bx 2 : 1
| x| < of the function f (x) in R.
ÔÓ 2
The Continuity and Differentiability  4.13

Centered difference Quotient [,] = G.I.F, then ​  


lim ​ f (x) equals
x Æ 0
82. If f ¢(2) = 5, find the value of (a) 1 (b) 0

( 
(c) – 1 (d) None
​ lim 
h Æ 0
 f (2 + h) – f (2 – h
  ​ ​ ​ ________________
2h
       ​  ​ ) 2. ​ lim 
x Æ •
( 
log [x]
  ​​ ______
​  x    )
​  ​ where [,] = G.I.F, is
83. Given f ¢(2) = 6 and f ¢(1) = 4, find the value
(a) 1 (b) 0

(  )
2 (c) – 1 (d) None
 f(2h + 2 + h ) – f(2)
lim ​ ​ ___________________
of ​   ​         ​  ​
h Æ 0 f(h – h2 + 1) – f(1)
|x|3
3. The left hand limit of f (x) = ​ ​ ___ {  __
[  ] }
x 3
a   ​ – ​​ ​ a ​  ​​ ​  ​, where
Twice differentiability a > 0 and [,] = G.I.F, is
84. Let f (x) = |x3|. Examine whether the function is twice
(a) a2 (b) a2 – 1
differentiable or not.
(c) a2 – 3 (d) None
Ï 2 Ê 1ˆ
Ô x sin Á ˜ : x π 0 Êx ˆ
85. Let y = Ì Ë x¯
Ô0 Á 1Ú | t - 1 | dt ˜
Ó : x=0 4. lim Á ˜ is
x Æ t Á sin ( x - 1) ˜
Examine whether the function is differentiable or not ÁË ˜¯
at x = 0. (a) 0 (b) 1
ÏÔ xe x : x£0 (c) –1 (d) None
86. Let y = f (x) = Ì
2 3
ÔÓ x + x - x : x > 0 Ïx : x < 0
Ô
5. If f (x) = Ì1 : x = 0 , then ​   lim  ​ f (x) is
x Æ 0
Examine whether the function f (x) is twice differen- Ô 2
tiable or not. Óx : x > 0
(a) 0 (b) 1
Functional Equations (c) 2 (d) None
87. If f (x + y) = f (x), f (y), " x, y Œ R and f (x) is a dif- 6. If [x] denotes the greatest integer less than or equal
ferentiable function and f (0) π 0, find f (x). to x, then the value of ​   lim ​ (1 – x + [x – 1] + [1 – x])
x Æ 0
is
88. If f (x + y) = f (x) + f (y), " x, y Œ R and f (x) is a
differentiable function, find f (x). (a) 0 (b) 1
(c) – 1 (d) None
( 
x + y
89. Let f  ​ ​  _____
2
 ​   )
f (x) + f (y)
  ​ = ​  __________
2
 ​
    for all x, y in R.   ​ (1 + sin x)1/x is equal to
7. ​ lim 
x Æ 0
If f ¢(0) = – 1, f (0) = 1, find f (x). (a) 0 (b) infinity

( 
(c) e (d) Does not exist.
x + y
90. If f ​ ​  _____
3 )2 + f (x) + f (y)
 ​ = ​  _____________
 ​  
3
 ​
     for all x, y in R
8. ​ lim  ​ 
sin [cos x]
  ​ __________    
x Æ 0 1 + [cos x]  
 ​, where [,] = G.I.F
and f ¢ (2) = 2, then find f (x). (a) 0 (b) 1

( 
2x + 3y
91. If f ​ ​ _______
5 )
2f (x) + 3f (y)
 ​ = ​ ___________
 ​  
5
 ​
     for all x, y in R and

(c) Not exist (d) None
9. The number of points at which the function
f (0) = 1, f ¢ (0) = – 1, find f (x). 1
f (x) = _____
​     ​ is discontinuous is
log |x|
92. If f (x + y + z) = f (x) ◊ f (y) ◊ f (z) for all x, y, z in R
such that f (2) = 4, f ¢(0) = 3, find f ¢(2). (a) 1 (b) 2
(c) 3 (d) 4
log (1 + ax) – log (1 + bx)
(Mixed Problems) 10. The function f (x) = ​ ______________________
   
x    ​ is not
defined at x = 0. The value which should be assigned
Choose the most appropriate one to f at x = 0, so that it is continuous at x = 0 is
Ï sin[ x ] (a) a – b (b) 1 + b
Ô : [ x] π 0
1. If f (x) = Ì [ x ] , where (c) log a + log b (d) none
Ô0 : x=0
Ó
4.14  Differential Calculus Booster


cos2p x
11. If f (x) = _______
​  2x
e – 2ex
  
1
 ​, x π __
2
1
​   ​ , then the value of f ​ __ (  )
​   ​   ​ so
2
18. In order that the function f (x) = (1 + x) cot x is continu-
ous at x = 0, then f (0) must be defined as
that f (x) is continuous at x = 1/2 is (a) 0 (b) e
p p (c) 1/e (d) None
(a) ​ ___ 2 ​   (b) ​ ___   ​
2e 2e p
19. Let f (x) = (sin 2x) tan 2x is not defined at x = ​ __ ​ . If f (x)
p2 p2 4
(c) ​ ___2  ​   (d) ​ ___  ​
2e 2e
is continuous at x = __
p
(  ) p
​   ​ , then f ​ __
4
​   ​   ​ is equal to
4
ÏÔ(cos)1/ x : xπ0 (a) 1 (b) 2
12. If the function f (x) = Ì is continu-
ÔÓk : x=0 (c) e1/2 (d) None
ous at x = 0, then the value of k is Ï -p
Ô- 2sin x : x£
2
(a) 1 (b) – 1 Ô
Ô p p
(c) 0 (d) e 20. Let f (x) = Ìa sin x + b : - < x < .
Ô 2 2
1 – tan x
13. Let f (x) = ​ _______   
4x – p

p
​ : x π __
4
p
​   ​ , x Œ ​ 0, __
2 (  )
​   ​   ​. If f (x) is Ô
Ôcos x : x≥
p
Ó 2
p
2 (  )
continuous in ​ 0, ​ __ ​   ​, then the value of f ​ __ (  )
p
​   ​   ​ is
4 If f (x) is continuous everywhere, then (a, b) is
(a) 1 (b) 1/2 (a) (0, 1) (b) (1, 1)
(c) (– 1, 1) (d) (– 1, 0)
(c) – 1/2 (d) – 1
21. The value of f (0) so that the function
1
​ ____
   ​ 
14. Let f (x) = (x – 1​)​2 – x ​is not defined at x = 2. If f (x) 2x – sin–1 x
f (x) =​  __________    ​ is continuous at each point on its
is continuous then f (2) is 2x + tan–1 x
(a) e (b) 1/e domain is
1
(c) 1/e2 (d) 1 (a) 2 (b) ​ __ ​ 
3
2 1
15. If f (x) = x a log x and f (0) = 0, then the value of a (c) ​ __ ​   (d) – ​ __ ​ 
3 3
for which Rolles Theorem can be applied in [0, 1]
Ï 36 x - 9 x - 4 x + 1
is Ô : xπ0
22. If f (x) = Ì 2 - 1 + cos x is continuous
(a) – 2 (b) – 1
Ô
(c) 0 (d) 1/2 Ók : x=0
16. The value of p for which the function at x = 0, then k is __
Ï (4 x - 1)3 (a) 16 log 2 log 3     log 6
(b) 16 ​÷2 ​
__
Ô : xπ0     log 2 log 3
(c) 16​÷2 ​ (d) None
Ô Ê xˆ È Ê x2 ˆ ˘
f (x) = Ì sin Á ˜ log Í1 + Á ˜ ˙ , Ï x-4
Ô Ë p¯ ÎÍ Ë 3 ¯ ˚˙ Ô| x - 4 | + a : x < 4
Ô 3 ÔÔ
Ó12(log 4) : x=0
23. Let f (x) = Ìa + b : x = 4 . then f (x) is con-
may be continuous at x = 0 is Ô x-4
(a) 1 (b) 2 Ô +b : x>4
ÔÓ | x - 4 |
(c) 3 (d) 4
tinuous at x = 4, when
Ï
Ô1 - cos 4 x : x<0 (a) a = b = 0 (b) a = b = 1
Ô x2 (c) a = – 1, b = 1 (d) a = 1, b = – 1.
Ô
17. Let f (x) = Ìa : x=0.
Ô Ï sin (a + 1) x + sin x
Ô x Ô : x<0
: x>0 Ô x
Ô (16 + x ) - 4 Ô
Ó 24. If f (x) = Ìc : x = 0 is continuous
If the function is continuous at x = 0, then a is Ô 2
Ô x + bx - x
ÔÓ : x>0
(a) 4 (b) 6 bx 3/2
(c) 8 (d) 10 at x = 0, then
The Continuity and Differentiability  4.15

(a) a = – 3/2, b = 0, c = 1/2 32. The number of points where f (x) = [sin x + cos x],
(b) a = – 3/2, b = 1, c = – 1/2 where [,] = G.I.F., x = (0, 2p) is discontinuous is
(c) a = – 3/2, b = R, c = 1/2 (a) 3 (b) 4
(d) None (c) 5 (d) 6
33. Let f : R Æ R be any function. Define g : R Æ R by
Ï ae|sin x| - b cos x - | x | g(x) = |f (x)| for all x.
Ô : xπ0
25. Let f (x) = Ì x2 , then Then g is
Ôc : x=0
Ó (a) onto if f is onto
(a) discontinuous at x = 0 (b) one-one if f is one-one
(b) continuous at x = 0, if a = b = c = 1 (c) Continuous if f is continuous
(c) f ¢(0) = 1 (d) differentiable if f is differentiable.
(d) continuous but non-differentiable at x = 0.
Ï -1ÈÍ 1 + 1 ˘˙
Ï p Ê 1ˆ 34. If f (x) = ÔÌ xe Î |x| x ˚ : x π 0 , then f (x) is
Ô x sin Á ˜ : x π 0 ÔÓ0
26. Let f (x) = Ì Ë x¯ . Then f (x) is con- : x=0
Ô0 : x=0
Ó (a) continuous and diff for all real x
tinuous but not differentiable at x = 0 if (b) continuous for all x but not differentiable at
x=0
(a) p Œ (0, 1] (b) p Œ [1, •)
(b) neither differentiable not continuous at x = 0
(c) p Œ (– •, 0) (d) p = 0
(d) discontinous everywhere.
27. The value of k for which the function
35. The value of the derivative of |x – 1| + |x – 2| at
Ïsin (1/ x ) : x π 0 x = 2 is
f (x) = Ì makes continuous at x = 0 (a) – 2 (b) 0
Ók : x=0
is (c) 2 (d) not defined
36. Let f (x) = [tan2 x], where [,] = G.I.F., then
(a) 8 (b) 1
(a) ​  
lim ​ f (x) does not exist
(c) – 1 (d) None x Æ 0

1 (b) f (x) is continuous at x = 0


28. Given the function f (x) = _____
​     ​  
. The points of dis-
1– x (c) f (x) is not differentiable at x = 0
continuity of the composite function y = f ( f ( f (x)) are
(d) f ¢(0) = 1.
at x =
x
(a) 0 (b) 1 37. The set of all points where the function f (x) = ​ _____
    ​
1 + |x|
(c) 2 (d) – 1 is differentiable is
29. Let f (x) = x – |x – x2| , where x lies in [– 1, 1]. Then (a) R (b) (0, •)
the number of points of discontinuity is
(c) (– •, 0) (d) None.
(a) 0 (b) 1
(c) 2 (d) None 38. The set of all the points where the function
30. The function f (x) = [x] – [x2 ] , where [,] = G.I.F.,
2 Ï0 : x=0
Ô
is discontinuous as f (x) = Ì x , is differentiable is
Ô1 + e1/ x : xπ0
(a) all integers Ó
(b) all integers except 0 and 1 (a) (0, •) (b) (– •, •) – {0}
(c) all integers except 0 (c) (– •, 0) (d) R.
(d) all integers except 1.

( 
2x – 1
31. The function f (x) = [x] cos ​ ______
​   ​  
2 ) ​ p , where
39. The set of all points of differentiability of the func-
Ï 2 Ê 1ˆ
x sin Á ˜ : x π 0
[,] = G.I.F., is discontinuous at Ô
tion f (x) = Ì Ë x¯ is
(a) all x Ô0
Ó : x=0
(b) all integer points (a) (0, •) (b) (– •, •) – {0}
(c) no x (c) (– •, 0) (d) R.
(d) x which is not an integer.
4.16  Differential Calculus Booster

40. The function f(x) is defined as (a) {– 1, 1} (b) {– 1, 0}


Ï1 1 (c) {0, 1} (d) {– 1, 0, 1}
Ô3 - x : x<
3
Ô Ï 1
f (x) = Ì ,
2 Ô- : | x| ≥ 1
ÔÊ 1 - x ˆ : x ≥ 1 47. Let f (x) = Ì | x | . If f (x) is continuous
ÔÓÁË 3 ˜¯ 3 Ôax 2 - b : | x | < 1
Ó
then in the interval (0, 1), the mean value theorem and differentiable at any point, then
is not true, because
(a) a = 1/2, b = – 3/2 (b) a = 1/2, b = 3/2
(a) f (x) is not continuous
(c) a = 1, b = – 1 (d) None
(b) f (x) is not differentiable 51. The number of points at which the function

|  |
(c) f (0) = f (1) 1
f (x) = ​ x – __
​    ​  ​ + | x – 1| + tan x does not have a deriva-
(d) None. 2
41. Let f (x) = [x sin p x], where [ , ] = G.I.F., then f (x) tive in the interval (0, 2),
is (a) 1 (b) 2
(a) continuous at x = 0 (c) 3 (d) 4
(b) continuous in (– 1, 0)
52. The function f (x) = (x – 1) ​| x2 – 3x + 2 |​ + cos (|x|)
2
(c) differentable at x = 1 is not differentiable at x =
(d) differentable in (– 1, 1) (a) – 1 (b) 0
Ï| 2 x - 3 | [ x ] : x ≥ 1 (c) 1 (d) 2.
Ô
42. The function f (x) = Ì Ê p x ˆ x2 x2 x2
: x <1 53. If f (x) = x2 + ​ _______  ​ + ​ _______
 2 2   ​ + .... + ​ _______
 2 3      ​
Ôsin ÁË 2 ˜¯ (1 + x ) (1 + x ) (1 + x2) n
Ó
(a) is continuous at x = 2 + ..... to  – •, then at x = 0
(b) is differentiable at x = 1 (a) f (x) has no limit
(c) is continuous but not differentiable at x = 1 (b) f (x) is discontinuous
(d) None. (c) f (x) is continuous but not differentiable
43. The function f (x) is defined as under: (d) f (x) is differentiable.
54. Let f (x + y) = f (x) f (y) for all x and y. Suppose that
ÔÏ3 x : -1 £ x £ 1 f (3) = 3 and f  ¢(0) = 11, then f ¢(3) is given by
f (x) = Ì . The function is
ÔÓ4 - x : 1 < x < 4 (a) 22 (b) 44
(c) 28 (d) 33.
(a) continuous at x = 1
55. A function f : R Æ R satisfies the equation f (x + y)
(b) differentiable at x = 1
= f (x) f(y) for all values of x and y and for any
(c) continuous but not differentiable at x =1 x Œ R, f (x) π 0 . Suppose the function is differentiable
(d) None. at x = 0 and f ¢(0) = 2, then for all x = R, f (x) =
44. A function is defined as follows: (a) ex (b) e 2x
–x
ÏÔ x 2 : x 2 < 1 (c) e (d) None
f (x) = Ì
2
ÔÓ x : x ≥ 1
. The function is
(  (  ) )
x 2
56. Let F (x) = ( f (x)) 2 + ​ g​​ ​ __   ​  ​​ ​  ​, F(5) = 5 and
2
(a) continuous at x = 1 f ¢¢(x) = –  f (x) , g(x) = f  ¢(x) , then F(10) is equal to
(b) differentiable at x = 1 (a) 5 (b) 10
(c) continuous but not differentiable at x = 1 (c) 0 (d) None
(d) None. 57. Let f be a differentiable for every x. If f (1) = – 2 and
45. The left hand derivative of f  ¢ for all x in [1, 6] , then
f (x) = [x] sin (p x) at x = k, where k is an integer, is (a) f (6) < 5 (b) f (6) = 5
(a) (– 1) k (k – 1) p (b) (– 1) k – 1 (k – 1) p (c) f (6) ≥ 8 (d) f (6) < 8
k
(c) (– 1)  kp (d) (– 1) k – 1 kp. 58. If f is a real valued differentiable function satisfying
46. Let f : R Æ R be a function is defined | f (x) – f (y) £ (x – y)2 , for all x, y in R , then f (1) is
by f (x) = max {x, x3 }. The set of all points where equal to
the function f(x) is not differentiable is
The Continuity and Differentiability  4.17

(a) 2 (b) 1 Discuss the continuity for the function f (x) at


(c) – 1 (d) 0. x = 1.
12. Let f (x) = x – [x – x2]. Find the number of points of
discontinuity of f (x).

[  ] [  ]
1 2
13. Let f (x) = [x] + ​ x + __
​   ​   ​ + ​ x + __
​   ​   ​. Find the number
1. Discuss the continuity of the function f(x), where 3 3

f (x) = ​  
n Æ • (  sin x
lim ​ ​ ​ ___________ 
)
   ​  ​ in (0, p).
1 + (2 sin x)2n
of points of discontinuity of f (x) in [– 1, 1].
14. If the graph of the continuous function y = f (x) passes
2. Determine the set of all points where the function through (a, 0), find

(  )
x3
f (x) = ​ _____    ​ is continuous. ln(1 + 6f 2 (x) – 3f (x))
4 + |x|   ​ ​ __________________
​ lim  ​   ​ 
      ​
x Æ a 3f (x)
3. Examine the continuity at x = 0 of the function
x x x
f (x) = ​ _____     ​ + ​ ____________
    ​ + ​ _____________
     ​ + 15. Find the number of points of discontinuity of
x + 1 (x + 1)(2x + 1) (2x + 1)(3x + 1)
f (x) = [4x] + {3x} in [0, 5].
Ï Ê Ê 1 1ˆˆ
Ô x exp Á - Á + ˜ ˜ : x π 0 16. Find the number of points of discontinuity of

|  |
4. Let f (x) = Ì Ë Ë | x| x ¯ ¯ x
Ô f (x) = ​ _____
​  2    
 ​  ​.
Ó0 : x=0 x +1
Discuss the continuity of the function f (x) at x = 0.
17. Find the number of points of discontinuity of
Ï
Ô a (1 - x sin x ) + b cos x + 5 f (x) = sgn(x2 – 1).
Ô : x<0
x2 18. Find the number of points of discontinuity of the
ÔÔ
5. Let f (x) = Ì3 : x=0 function f (x) = [x] + {2x} + [3x] in [0, 1]
Ô 1x 19. Determine the set of all points, where the function
ÔÊ Ê cx + dx 3 ˆ ˆ x
ÔÁ 1 + Á 2 ˜˜ : x>0 f (x) = _____
​       ​ is continuous.
ÔÓË Ë x ¯¯ 1 + |x|
If f is continuous at x = 0 , then find the values of
ÔÏe x : x£0
a, b, c and d. 20. Let f (x) = Ì
ÔÓ| x - 1 | : x > 0
[  1
] [  1
]
6. Let f (x) = ​ x – ​ __ ​   ​ + [x] + ​ x + ​ __ ​   ​, x = [– 1, 0].
3 3 Discuss the continuity for the function f (x) at
x = 0, 1.
Discuss the continuity of the function f (x) at
[– 1, 0]. 21. Discuss the continuity of f (x) = Îtan –1x˚.

[ 
(x – 2)3
7. If the function f (x) = ​ ​  _______
a    ]
​  ​ sin (x – 2) + a cos
p
22. Discuss the continuity of f (x) = [x] sin ​ ______
​     
[x + 1]( 
 ​  ​ )
(x – 2) is continuous in (4, 6), then find the set of 23. Discuss the continuity of the function
all values of a, where [,] = G.I.F. f (x) = |x + 1| (|x| – |x – 1| in [– 2, 2].
p 
8. Let f (x) = min {1, cos x, 1 – sin x}, –  ​ __ ​  £ x £ p 24. Discuss the continuity of the function
2
Discuss the continuity for the function f (x) at Ï(1 - x ) : x <1
p  Ô
x = 0, ​ __ ​  f (x) = Ì(1 - x ) (2 - x ) : 1 £ x £ 2
2 Ô(3 - x )
______ Ó : x>2
{ 
9. Let f (x) = max ​ sgn (x), – ​÷9  – x2 ​, x3  ​ } x
25. Discuss the continuity of the function f (x) =  ​ _____
    ​.
Discuss the continuity of the function f (x) at x = 0. 1 – |x|
26. Discuss the continuity of the function
ÔÏ x 3 : x 2 < 1 f (x) = [x] + [– x].
10. Let f (x) = Ì . Discuss the continuity of
ÔÓ x : x > 1 27. Discuss the continuity of the function
the function f (x). f (x) = sgn (cos 2x – 2 sin x + 3).
ÔÏ3 x : x2 £ 1 28. Discuss the continuity of each of the following
11. Let f (x) = Ì functions.
ÔÓ4 - x : 1 < x < 4
(i) f (x) = [loge x]
4.18  Differential Calculus Booster

(ii) f (x)= Îsin–1 x˚ Ï 1


Ô ae |x + 2| - 1

[  ]
2
(iii) f (x) = ​ _____
​  2    
x +1
 ​  ​.
Ô
Ô
Ô2-e
1
| x + 2|
: -3 < x < -2

Ô
29. Discuss the continuity of the function 5. Let f (x) = Ìb : x = -2
Ô
Ôsin Ê x - 16 ˆ : - x < 0
4

2
x
f (x) = ​ __ [  ] [  ] [  ]
x
​    ​  ​ + ​ __
x
​    ​  ​ + ​ __
3
​    ​  ​ in [0, 4]
4
Ô ÁË x 5 + 32 ˜¯
Ô
ÔÓ
sin (3x) + a sin (2x) + b sin x
30. Let f (x) = ​ _______________________
    ​   , x π 0 If f (x) is continuous at x = – 2, find the value of
x5 (a + b + 2).
If f (x) is continuous at x = 0, find f (0) 6. Discuss the continuity of

(Tougher Problems for Jee-Advanced)


n Æ • ( 
x2n – 1
lim ​ ​ ​ ______
f (x) = ​  
x2n + 1

 ​  ​ )
7. Discuss the continuity of f (x), where

Ï1 + a cos x + b cos 4 x f(x) = ​   (cos2n x).


lim ​  
n Æ •
Ô : xπ0
1. Let f (x) = Ì x 2 sin 2 x 8. Discuss the continuity of f (x) in [0, 2] ,
Ôc : x=0
Ó
If f (x) is continuous at x = 0, find the value of​ where f (x) = ​  
n Æ •
px 2n
lim ​ ​​ sin ​ ___
​   ​   ​  ​​ ​
2 (  (  ) )
(  1
__
a + b + c + ​   ​   ​
3 ) 9. Discuss the continuity of the function

Ï
Ô(sin x + cos x )cosec x
Ô
1
: - £x<0
2
n Æ • ( 
log (2 + x) – x2n sin x
lim ​ ​ ​ _________________
f (x) = ​  
1 + x2n
     ​  ​ at x =1.
   )
Ô

(  )
2. Let f (x) = Ìa : x=0 x2n–1 + ax2 + bx
Ô 1/ x 2/ x 3/| x| lim ​ ​ ​ _____________
10. Let f (x) = ​      ​ 
   ​
Ôe + e + e 1 x2n + 1
n Æ •
: 0< x£
ÔÓ ae + be x|
2/ x 3/|
2
If f (x) is continuous for all x in R, find the value of
If f (x) is continuous at x = 0, find the value of {e (a 2a + 3b + 10.
+ b) + 2}.

Ï -1 Ê x + c ˆ 1
11. Let f (x) = ​   (    
n Æ • |​ a + sin px |n​ + 1 )
|n​ – 1  ​  ​, x Œ (0, 6)
|​ a + sin px   
lim ​ ​ ​ _____________

Ôb sin ÁË 2 ˜¯ : - 2 < x < 0 Find the number of discontinuity, when a = 0


Ô
Ô1 and 1.
3. Let f (x) = Ì : x=0
Ô2 12. Discuss the continuity of the function f (x),

( 
Ô e ax /2 - 1 1
Ô
Ó x
: 0< x<
2 n Æ •
​ 
x
lim ​ ​ ___________
where f (x) = ​       ​  ​.
1 + (2 sin x)2n )
If f (x) is differentiable at x = 0, find the value of a
and hence prove that 64 b2 + c2 = 4 13. Discuss the continuity of the function f (x) at x = 1,

Ï Ê 1 ˆ
Ô | x | Á 3e |x| + 4˜
n Æ • ( 
cos (p x) – x2nsin (x – 1)
lim ​ ​ ​ ____________________
where f (x) = ​           ​  ​
1 + x2n+1 + – x2n )
ÔÔ Ë ¯
4. Let f (x) = Ì 1
: xπ0 14. Discuss the continuity of the function f (x) in [0, 2],
Ô 2 - e | x| where
Ô
ÔÓa : x=0 Ï| 2 x - 3 | [ x ] : x ≥ 1
Ô
f (x) = Ì Ê p x ˆ
If f (x) is continuous at x = 0, find the value of (a2 Ôsin ÁË 2 ˜¯ : x <1
+ a + 10) Ó
The Continuity and Differentiability  4.19

15. Discuss the continuity of the function f (x) in [0, 2], 22. Let f (x) be defined in the interval [– 1, 1] such that
where
ÏÔ x - 1 : - 1 < x < 0
Ï[cos p x ] : 0 £ x £1 f (x) = Ì 2
f (x) = Ì ÔÓ x : 0 £ x £1
Ó| 2 x - 3 | [ x - 2] : 1< x £ 2
and g (x) =sin x , then discuss the continuity and dif-
16. Discuss the continuity of the function f (x) in [0, 2), ferentiability of h (x) in [– 1,1] , where h (x) = f (|g (x)|)
where + |f (g (x))|

ÏÔ|1 - 4 x 2 | : 0 £ x < 1
f (x) = Ì Integer Type Questions
2
ÔÓ[ x - 2 x ] : 1 £ x < 2
1. Let f : [1, 7] Æ Q be a continuous function such that
17. Discuss the continuity of the function f (x) in [0, 2], f (1) = 7, find the value of f (7).
where 2. Let f be a continuous function on [1, 3] which takes
rational values for all x. If f (2) = 5, find the value
Ï
Ô of f (2.5).
Ô|sin(p x ) | : -1 £ x < 0
3. Let m is the number of non-differentiable points
Ô
f (x) = Ì1 - {x} : 0 £ x <1 of f (x) = ||x| – 1| and n is the number of points
Ô
Ô1 + Ècos ÊÁ p x ˆ˜ ˘ : 1 < x £ 2 1
of differeniable of g (x) = ​ _____
   ​, find the value of
Í ˙
ÓÔ Î Ë 2 ¯ ˚ (m + n).
log |x|

4. If m is the number of differentiable points of f (x)


(  )
x+y
18. If f  ​ ​ _____
2
f (x) + (y)
 ​ = ________
 ​   ​ 
2
 ​  , x, y ŒR, 1
= ​ _________
     ​ and the value of n for which ​  
lim   ​ ​
log|x 2 – 4| x Æ 0
f (0) = 1, f ¢ (0) = – 1, then show that
2 2 2
{(f (0)) + (f (1)) + ( f (2)) + .... + ( f(n)) } 2
( xn – sin (xn)
​ __________
   ​  
x – sinn x )
​ has a non-zero finite value, find the

n (n – 1)(2n – 1) value of (m + n).


= 1 + ______________
​   ​     .
6 5. Find the number of points of discontinuity of
19. Let f (x) = x3 – x2 + x + 1 and f (x) = [3 + 4 sin x], x Œ [p, 2p]
6. If p is the number of discontinuity points of f (x) = [[x]]
Ïmax.{ f (t ) : 0 £ t £ x} : 0 £ x £ 1
g(x) = Ì – [x – 2], where [,] = G.I.F. and q is the limiting

( 
Ó3 - x : 1< x £ 2

Discuss the continuity and differentiability of the


function g(x) in (0, 2).
value of ​  
lim  
x Æ •

(p + q + 2).
3x4 – 2
​   ​​  ____________
  
__________
​÷ x8 + 3x   
+ 4 ​ )
   ​  ​, find the value of

20. Let f(x) = x4 – 8x3 + 22x2 – 24x – 55 and

Ïmin{ f (t ) : x £ t £ x + 1} : - 1 £ x £ 1
7. If m is the limiting value of ​  
x Æ 0 ( 
tan 2x – 2 sin x
lim ​ ​ ____________
​ 
x3
 ​ 
    
)
​ and

g(x) = Ì n is the number of points of disc of f (x) = [sin x


Ó x - 10 : x >1
+ cos x] in (0, 2p) and p is the number of non-
differentiable points of f (x) = |​ log |x|  |​, find the value
Discuss the continuity and differentiability of g (x) in
of (m + n– p).
[– 1, •).
8. If f be a differentiable function such that f ¢ (2) = 6,
21. Let a function y = f (x) be defined by
f ¢ (1) = 4, find the value of

( 
Ïj ( x ) : x£0
Ô x
f (x) = ÌÊ e + x ˆ
ÔÁ 1 + 2 x ˜ : x > 0
ÓË ¯
.
f (h2 + 2h +2) – f (2)
  ​ ​ _________________
​ lim  ​    
    ​  ​.
x Æ 0 f (h – h2 + 1) – f (1) )
If j (x) = a sin x + b cos x and f (x) is continuous ÏÔa + bx 2 : x<0
9. Let f (x) = Ì
and satisfies f ¢ (1) = f (– p /2), find the values of a ÔÓ3 ax - b + 2 : 0 ≥1
and b.
4.20  Differential Calculus Booster

If f (x) is differentiable at x = 1, find the value of (a 3. Number of points where | f (x)| is not differentiable
+ b + 1). is

(  )
2x
10. Let f (x) = sin– 1​ ​ _____
  ​  
1 + x2
​. If p is the number of discon-

(a) 1
(c) 3
(b) 2
(d) 4.
tinuity points and q is the number of non-differen- Passage III
tiable points of f (x), find the value of (p + q + 2).

[  )
In certain problem the differentiation of {f (x) ◊ g (x)} appears.
3
11. Let f (x) = [3x] – {2 x}, x = Œ ​ 0,  ​ ​__ ​   ​ ​. Find the number df dg
​2 ​ One student commits mistake and differentiate as ​ ___   ​ ___​     ​,
of points of discontinuity. dx dx
12. Let f (x) = (x – 2). Find the number of non-differen- but he gets correct result if f (x) = x 3 and g(x) is a decreas-
tiable points of g (x) = tan– 1(​|  f (|x|) |​). ing function for which g (0) = 1/3.

1. The function g (x) is


Comprehensive Link Passage 3 4
(a) ​ _______
     ​ (b) ​ _______
     ​
Passage I (x – 3)3 (x – 3)3
If two functions f (x) and g (x) are continuous at x = a, then
f (x) + g(x), f (x) – g (x), f ( ). g (x), ​|  f (x) |​ are continuous at
9
(c) ​ _______
     ​
27
(d) ​ _______
    ​.
f (x) (x – 3)3 (x – 3)3
x = a and ____
​     ​ is continuous at x = a , provided g (a) π 0.
g(x) 2. Derivative of {f (x–3) ◊ g(x)} with respect to x
If one of f (x) and g (x) is continuous but other is discontinu- at x = 100 is
ous at any point x = a, then f (x) + g (x) is discontinuous at (a) 0 (b) 1
x = a.
|x + 2| (c) – 1 (d) 2
1. Number of points where f (x) = ​ __________      ​ is dis-
continuous is/are tan – 1(x + 2) f (x) ◊ g (x)
  ​  ​  __________
3. ​ lim       ​ will be
x Æ 0 x (1 + g (x))
(a) 0 (b) 1
(c) 2 (d) infinite (a) 0 (b) – 1
2. f (x) = |x + 1| (|x| + |x – 1|) is discontinuous at (c) 1 (d) 2
(a) no where (b) x = – 1
Passage IV
(c) x = {0, – 1} (d) x = {– 1, 0, 1}
A function f (x) is said to have a jump discontinuity at a
3. Number of points in the interval (– 1, 1) where the point x = a, if both of the limits L.H.L and R.H.L exists
function f (x) = [x]/sin– 1 x – p/6) is discontinuous and finite at x = a, but not equal and f (a) may be equal
is/are to either of these limits. The value of ​| L.H.L – R.H.L |​ is
(a) 0 (b) 1 known as jump discontinuity.
(c) 2 (d) infinite 1. Jump of discontinuity of y = 2 [x] at x = 2, where
[,] = G.I.F, is
Passage II (a) 1 (b) 3
Let a function is defined as (c) 2 (d) – 2.
Ï -1 ÔÏ x + 1 : x £ 1
2
: -2 £ x £ 2. If f (x) = Ì , then jump of discontinu-
ÔÔ[ x ] 2 , where [, ] = G. I. F ÔÓ2 x + 5 : x > 1
f (x) = Ì
Ô2 x 2 - 1 : - 1 ity of f (x) at x = 1 is\
<x£2
ÔÓ 2 (a) 4 (b) 3
Answer the following question by using the above (c) 7 (d) None
information: 3. Number of jump discontinuities in y = f (x) ◊ g(x),
1. The number of points of discontinuity of f (x) is where
(a) 1 (b) 2 ÏÔ x + 1 : x ≥ 1
(c) 3 (d) None f (x) = Ì 2 and
ÔÓ x : x <1
2. The function f (x – 1) is discontinuous at the points
(a) – 1, – 1/2 (b) – 1/2, 1 ÏÔsin x : x < 1
(c) 0, 1/2 (d) 0, 1 g(x) = Ì 2
ÔÓ2 x : x ≥1
The Continuity and Differentiability  4.21

(a) 1 (b) 2 Matrix Match


(c) 3 (d) None 1. Observe the following Columns:
Passage V Column I Column II
Suppose that f is continuous on the closed interval (A) The Nnumber of points of discon- (P) 1
[a, b] and let k be any number between f (a) and f (b) where tinuity of f (x) = tan2 x – sec2 x in
f (a) π f (b). the interval (0, 2p) is
Then there exist a number c in (a, b) such that
(B) The number of points at which (Q) 2
f (c) = k.
– 1 – 1 – 1
On the basis of the above information answer the follow- f (x) = sin  x + tan  x + cot  x is
ing questions. non-differentiable in (– 1, 1) is
1. The number of real root of x.2x – 1 = 0 in (0, 1) is (C) Number of points of discontinuity (R) 0
(a) 2 (b) 3 of y = [sin x] x Œ [0.2p], where
(c) 0 (d) 1 [,] = G.I.F
2. The number of real root of 2 tan x + 5x – 2 = 0 in​ (D) Number of points where (S) 3

[  ] p
0, __
​   ​   ​ is
4
y = | (x – 1)3| + |(x – 2)5| + |x – 3|
is non-differentiable
(a) 1 (b) 2 2. Observe the following columns:
(c) 3 (d) 4
(  __​ p2 ​  )​
3. The number of real root of x – cos x = 0 in ​ 0,
is
Column I Column II

(A) Points of discontinuity of (P) – 1/2


(a) 1 (b) 2 1 1
y = _______
​  2      ​, where t = ____
​      ​
(c) 0 (d) 3 t –t–2 x +1
Passage VI (B) Points of continuity of y = [x] + (Q) – 2
A cos x + B x sin x – 5
Let f (x) =​  ___________________
    ​ ,    x π 0 is continuous at [– x]
x4
x = 0. Then (C) y = {sin (p x)} is non-differentiable (R) – 1
1. The value of A is at
(a) 2 (b) 5 (D) y = |2x + 1| + |x + 2| – |x + 1| – |x (S) 4
(c) 7 (d) 10 – 4| is non-differentiable at
2. The value of B is
(a) 1/2 (b) 3/2 3. Observe the following Columns:
(c) 5/2 (d) 7/2 Column I Column II
3. The value of f (0) is
(A) f (x) = {sin(p x} is disc for (P) [0, 1)
(a) – 1/24 (b) – 5/24
x Œ
(c) – 7/24 (d) – 11/24
Passage VII {  }
sin x
(B) g (x) = ​ ​ ____ ​   ​ is disc for
x   
(Q) {1, 2}

sin 3x + a sin 2x + b sin x xŒ


Let f (x) = ____________________
​   ​ ,
       x π 0 is continuous at
x5 {sin x}
x = 0. Then  (x) = ______
​   ​ 

is non-differentiable
1. The value of A is (C) {x} (R) {0}
for xŒ
(a) 2 (b) – 2
(c) 4
2. The value of B is
(d) – 4 (sin x)
(D) p (x) = _____
​ 
[x]
 ​ 
 is disc for xŒ (S) {  }
1
​ __
​   ​   ​
2
(a) 3 (b) 5
(c) 7 (d) 9 4. Observe the following Columns:
3. The value of f (0) is The number of points of discontinuity of the
(a) 1 (b) 0 functions
(c) – 1 (d) – 2
4.22  Differential Calculus Booster

Column I Column II 8. Draw the graph of the following function and discuss
its continuity and differentiability at x = 1
x
(A) y = f (x) = _____
​       ​ (P) 2
|x| + 1 ÔÏ3 x : -1 £ x £ 1
f (x) = Ì
ÔÓ4 - x : 1 < x < 4
x
(B) y = f (x) = _____
​       ​ (Q) 3 [Roorkee-JEE, 1994]
|x| – 1
9. Discuss the limit, continuity and differentiability of
1
(C) y = f (x) = _____
​     ​  (R) 1 the function at x = 0
log |x|
Ï x (3e1/ x + 4)
1 Ô : xπ0
(D) y = f (x) = _________
​       ​ (S) 0 f (x) = Ì (2 - e1/ x )
log |x2 + 1| Ô0 : x=0
Ó
[Roorkee-JEE, 1995]
Problems asked in Roorkee - JEE Exam 10. Find the value of f (0) so that the function
1 1
1. Sketch the function y = |x – 2| in [– 1, 2] f(x) = ​ ________
x ​ – , x π 0 is continuous at x = 0 and
​  2x    ​ 
e   –  1
Is the function (i) Continuous (ii) differentiable at
then examine the differentiability of f (x) at x = 0.
x = 2?
[Roorkee-JEE, 1984] [Roorkee-JEE, 1996]
11. Discuss the continuity and differentiability of the
Ïx - 1 : x < 0
Ô function
Ô1
2. Let f (x) = Ì : x=0 Ï2 + 1 - x 2 : |x| £ 1
Ô
Ô4 f (x) = Ì
ÔÓ x 2 (1 - x )2
: x>0 ÓÔ2e : |x| > 1

Discuss the continuity of f (x) at x = 0. [Roorkee-JEE, 1998]


[Roorkee-JEE, 1988] 12. Determine the constants a, b and c for which the
3. Draw the graph of the function f (x) = x – |x – x2|, function
– 1 £ x £ 1 and discuss the continuity and discontinuity Ï
of f in the interval [–1, 1]. Ô 1/ x
[Roorkee-JEE, 1989] Ô(1 + ax ) : x<0
Ô
4. If f (x) = – 1 + |x – 1|, –1 £ x £ 2, g (x) = 2 – |x + 1|, f (x) = Ìb : x=0
Ô
–2 £ x £ 2, then find (f o g) (x) and (g o f) (x). Draw their 1/ 3
Ô ( x + c) - 1 : x > 0
graphs. Discuss the continuity (f o g) (x) at x = –1, and ÔÓ ( x + 1)1/ 2 1
the differentiability of (g o f) (x) at x = 1.
[Roorkee-JEE, 1990] is continuous at x = 0.
[Roorkee-JEE, 1999]
5. The function f is defined by y = f (x), where
x = 2t – |t|, y = t 2 + t |t|, t Œ R. Draw the graph of f 13. Discuss the continuity and differentiability of the
for the interval – 1 £ x £ 1. Also, discuss the continu- function
ity and differentiability at x = 0. Ï x
[Roorkee-JEE, 1991] Ô1 + | x | : | x | ≥ 1
_____ Ô
f (x) = Ì
6. If f (x) = ÷   – 1| ​
​ |x  and g (x) = sin x, then calculate
Ô x : | x| < 1
(fog) (x) and (gof) (x) and discuss the differentiability ÔÓ1 - | x|
of (go f ) (x) at x = 1.
[Roorkee-JEE, 1992] [Roorkee-JEE, 2000]
7. A function is defined as follows: 14. Discuss the continuity of the function
Ï 1
Ï x3 : x2 < 1
f (x) = ÔÌ . Ô e x -1 - 2
Ô : x π1
2
ÔÓ x : x ≥ 1 f (x) = Ì 1 at x = 1
Ô e x -1 + 2
Draw the graph of the function and discuss limit, Ô1 : x =1
Ó
continuity and differentiability at x = 1
[Roorkee-JEE, 1993] [Roorkee-JEE, 2001]
The Continuity and Differentiability  4.23

Problems asked in Previous Years’ IIT-JEE Exam 9. Let g(x) be a polynomial of degree one and f(x) be
_____ defined by
__
1. If f (x) = x ​( ​÷x  )​, then
  + 1 ​  
  ​  – ​÷x
Ï g( x ) : x£0
(a) f (x) is continuous but not differentiable at ÔÔ 1
x=0 f ( x ) = ÌÊ 1 + x ˆ x
ÔÁ : x>0
˜
(b) f (x) is differentiable at x = 0 ÔÓË 2 + x ¯

(c) f (x) is not differentiable at x = 0
(d) None of these Find the continuous function f(x) satisfying
[IIT-JEE, 1985] f ¢(1) = f (– 1)
2. Let f (x) = x3 – x2 + x + 1 and [IIT-JEE, 1987]
10. Let R be the set of real numbers and f : R Æ R be such
Ïmax{ f (t ) : 0 £ t £ x : 0 £ x £ 1 that for all x and y in R, ​| f (x)  –  f (y) |​ £ (x – y)2.
g (x) = Ì .
Ó3 - x : 1£ x £ 2 Prove that f(x) is a constant.
[IIT-JEE, 1988]
Discuss the continuity and differentiability of the
function g(x) in (0, 2). Ï| x - 3| : x ≥1
Ô 2
[IIT-JEE, 1985] 11. The function f (x) = Ì x 3 x 13 is
3. The function f (x) = 1 + |sin x| is Ô - + : x <1
Ó4 2 4
(a) Continuous nowhere (a) continuous at x = 1
(b) Continuous everywhere (b) differentiable at x =1
(c) differentiable nowhere (c) continuous at x = 3
(d) not differentiable at x = 0. (d) differentiable at x = 3.
[IIT-JEE, 1986] [IIT-JEE, 1988]
4. Let [x] denote the greatest integer less than or equal 12. Find the values of a and b so that the function
to x. If f(x) = [x sin p x], then f(x) is
Ï p
(a) continuous at x = 0 Ô x + a 2 sin x : 0£ x<
4
(b) continuous in (– 1, 0) Ô
Ô p p
(c) differentiable at x = 1 f  (x) = Ì2 x cot x + b : £x£
Ô 4 2
(d) differentiable in (– 1, 1)
Ô p
[IIT-JEE, 1986] Ôa cos2 x - bsin x : 2 < x £ p
Ó
5. Let f (x) be defined in the interval [– 2, 2] such that
is continuous for 0 £ x £ p
Ï-1 : -2 £ x £ 0
f ( x) = Ì [IIT-JEE, 1989]
( 
Ó x - 1 : 0< x£2

and g(x) = f (|x|) + |f (x)|. Test the differentiability of
x
__
)
13. If f (x) = ​ ​    ​ – 1  ​, then on the interval [0, p]
2
g(x) in (– 2, 2) (a) tan (f(x)) and 1/f (x) are both continuous
[IIT-JEE, 1986] (b) tan (f(x)) and 1/f (x) are both discontinuous
x (c) tan (f(x)) and f –1 (x) are both continuous
_____
6. The set of all points where the function f(x) = ​       ​
1 + |x| (d) tan ( f(x)) is continuous but 1/f(x) is not
is differentiable is continuous.
(a) (– •, •) (b) [0, •) [IIT-JEE, 1989]
(c) (–•, 0) » (0, •) (d) (0, •) Ï
[IIT-JEE, 1987] Ô1 - cos 4 x : x<0
Ô x2
7. Let f (x) be a function satisfying the condition f (– x) Ô
= f (x) for all real x. If f ¢(0) exists, then find its
14. Let f (x) = Ìa : x=0
value. Ô
Ô x
: x>0
[IIT-JEE, 1987] Ô 16 + x - 4
Ó
8. Let f (x) be a continuous function and g(x) be a
discontinuous function, then prove that f(x) + g(x) Determine the value of a, if possible, so that the
is a discontinuous function. function is continuous at x = 0.
[IIT-JEE, 1987] [IIT-JEE, 1989]
4.24  Differential Calculus Booster

15. Draw a graph of the function y = [x] + |1 – x| for all (c) f (x) is not differentiable at x = 0
x in [–1, 2], (d) f ¢(0) = 1.
[,] = G.I.F, Determine the points, if any, where this [IIT-JEE, 1993]
function is not differentiable.
Ï a
p
[IIT-JEE, 1989] Ô(1 + |sin x |) |sin x | : – <x<0
16. A function f : R Æ R satisfies the equation Ô 6
Ô
f(x + y) = f (x). f(y) for all x, y in R and f(x) is non- 22. Let f (x) = Ìb : x=0
zero for any x in R. Let the function is differentiable Ô tan 2 x
at x = 0 and f ¢(0) = 2. Show that f ¢(x) = 2 f(x) for Ô tan3 x p
ÔÓe : 0< x<
all x in R. 6
Hence determine f (x). Determine a and b such that f is continuous at x = 0.
[IIT-JEE, 1990] [IIT-JEE, 1994]
17. The function f is defined by y = f (x) where
x = 2t –|t| and y = t 2+ |t|, t Œ R.
Draw the graph of f for the interval – 1 £ x £ 1.
(  )
23. Let f  ​ ​ _____
2
 ​  
f (x) + f (x)
x + y __________
 ​= ​ 
2
 ​
     for all real x and y. If f ¢(0)

Discuss the differentiability of the function y = f(x) exists and equals to –1 and f (0) = 1, find f(2).
at x = 0. [IIT-JEE, 1995]

( 
[IIT-JEE, 1991]
18. The following functions are continuous on (0, p)
p
24. Let f (x) = [x] sin ​ ______
​     
[x + 1] )
 ​  ​, where [,] = G.I.F. The
(a) tan x domain of f is ... and the points of discontinuity of
x f in the domain are ...

0
(  )
1
(b) ​Ú ​  ​ t sin ​ __
​   ​   ​ dt
t [IIT-JEE, 1996]
25. Let f (x) be a continuous function defined for
Ï 3p 1 £ x £ 3. If f (x) takes rational values for all x and
ÔÔ1 : 0< x£
4 f (10) = 10, then f (1.5) = ...
(c) Ì [IIT-JEE, 1997]
Ê 2 ˆ
Ô2sin Á ˜ x : 3p
< x<p
ÔÓ Ë 9¯ 4 Ï ÔÏÌ–Ê 1 + 1 ˆ Ô¸˝
Ô ËÁ | x | x ¯˜ ˛Ô
Ï p 26. Let f (x) = Ìe ÓÔ : xπ0
ÔÔ x + sin x : 0< x£
2
Ô0
Ó : x=0
(d) Ì Test whether
p p
Ô sin (p + x ) : < x < p
ÔÓ 2 (a) f (x) is continuous at x = 0
2
(b) f (x) is differentiable at x = 0
[IIT-JEE, 1991]
[IIT-JEE, 1997]
19. Each entry in Column-I is related to exactly in
Column-II. Write the correct letter from Column-II 27. Determine the values of x for which the following
against the entry number in Column-I. function fails to be continuous or differentiable
Ï1 - x : x <1
Column I Column II Ô
f (x) = Ì(1 - 2) (2 - x ) : 1£ x £ 2
(i) sin (p [x]) (A) differentiable everywhere Ô3 - x
Ó : x>2
(ii) sin {p (x – [x])} (B) nowhere differentiable
Justify your answer. [IIT-JEE, 1997]
(C) not differentiable at 1 2
and-1 where [,] = G.I.F 28. Let h(x) = min {x, x } for every real number of x.
Then
[IIT-JEE, 1992] (a) h is continuous for all x
20. Let f (x) = x |x|. The set of points where f(x) is twice (b) h is differentiable for all x
differentiable is ... (c) h¢(x) = 1 for all x > 1
[IIT-JEE, 1992] (d) h is not differentiable at two values of x.
21. Let f(x) = [tan2 x], where [,] = G. I. F, then [IIT-JEE, 1998]
(a) ​ lim 
  ​ f (x) does not exist
x Æ 0 29. The function f (x) = (x2 – 1) | x2 – 3x + 2| + cos (|x|)
(b) f (x) is continuous at x = 0 is not differentiable at
The Continuity and Differentiability  4.25

(a) – 1 (b) 0 37. Let f : R Æ R be a function is defined by f (x) = max
(c) 1 (d) 2 {x, x2}. The set of all points where f(x) is not dif-
[IIT-JEE, 1999] ferentiable is
2 2 (a) {–1, 1} (b) {–1, 0}
30. The function f (x) = [x] – [x ], where [,] = G. I. F.
is discontinuous at (c) {0, 1} (d) {–1, 0, 1}
(a) all integers [IIT-JEE, 2001]
38. The left hand derivative of f(x) = [x] sin (p x) at x = k,
(b) all integer except 0 and 1
where k is an integer, is
(c) all integers except 0
(a) (– 1) k (k – 1)p (b) (– 1) k–1 (k – 1)p
(d) all integers except 1.
[IIT-JEE, 1999] (c) (– 1) k kp (d) (– 1) k–1 k p
31. Let f : R Æ R be any function. Define g : R Æ R by [IIT-JEE, 2001]
g (x) = | f (x)| for all x. Then g is 39. Let a Œ  . Prove that a function f : R Æ R is dif-
(a) onto if f is onto ferentiable at x = a and only if there is a function
(b) one-one if f is one–one g : R Æ R which is continuous at x = a and satisfies
f (x) – f (a) = g (x) (x – a) for all x Œ R
(c) continuous if f is continuous
[IIT-JEE, 2001]
(d) differentiable if f is differentiable.
[IIT-JEE, 2000] 40. The domain of the derivative of the function

32. Let p (x) = a 0 + a1 x + a 2 x 2 + a3 x 3 +...+ a n x n Ïtan -1 x : | x| £ 1


Ô
If | p (x)| £ |ex–1 – 1| for all x £ 0, then prove that |a1 f (x) = Ì 1 is
Ô (| x | - 1) : | x| > 1
+ 2a2 + 3a3 + ... + nan | £ 1 Ó2
[IIT-JEE, 2000] (a) R – {0} (b) R – {1}
33. Discuss the continuity and differentiability of the (c) R – {–1} (d) R – {– 1, 1}
function [IIT-JEE, 2002]
Ï x Ïx + a : x < 0
Ô1 + | x | : | x | ≥ 1 41. Let f (x) = Ì and
Ô
f (x) = Ì Ó| x - 1| : x ≥ 0
Ô x : | x| < 1
ÔÓ1 - | x | ÏÔ x + 1 : x<0
g (x) = Ì where a and b are non-
[IIT-JEE, 2000] 2
ÔÓ( x - 1) + b : x ≥ 0
34. Let f (x), x ≥ 0, be a non-negative continuous function
negative real numbers.
and let
x Determine the composite function go  f.
F (x) = Ú​  ​  ​ f (t) dt, x ≥ 0. If (go f ) (x) is continuous for all real x, determine the
0 values of a and b. Further for these values of a and b,
If for some c > 0, f (x) £ cF (x) for all is goo differentiable at x = 0? Justify your answer
x > 0, then show that [IIT-JEE, 2002]
f (x) = 0 for all x ≥ 0. 42. If f  :  [– 2a, 2a] Æ R be an odd function such that left
[IIT-JEE, 2001] hand derivative at x = a is zero and f (x) = f (2a – x),
35. Discuss the continuity of the function x Π(a, 2a), then find left hand derivative of f at
x = – a.
Ï 1 [IIT-JEE, 2003]
Ô e x -1 - 2
Ô : x π1 Ï
f (x) = Ì 1   at x = 1 -1 Ê x + c ˆ 1
Ôb sin ÁË 2 ˜¯ : - 2 < x < 0
Ô e x -1 + 2 Ô
Ô1 : x =1 Ô1
Ó 43. If f (x) = Ì : x=0
[IIT-JEE, 2001] Ô2
36. Which of the following functions is differentiable at Ô e ax / 2 - 1 1
Ô : 0< x<
x = 0? Ó x 2
(a) cos (|x|) + |x| (b) cos (|x|) – |x| is differentiable at x = 0. Find the value of a and
(c) sin (|x|) + |x| (d) sin (|x|) – |x| also prove that 64 b2 = 4 – c2.
[IIT-JEE, 2001] [IIT-JEE, 2004]
4.26  Differential Calculus Booster

44. The function is given by y = ||x| – 1| is differentiable


for all real numbers except the points
(a) {–1, 0 , 1} (b) {– 1, 1}
( (x – 1) n
51. Let g (x) = ​​ ______________ )
      ​  ​ : 0 < 2, m and n are
log{cosm (x – 1)}
integers, m π 0, n > 0 and let p be the left hand
(c) 1 (d) – 1 derivative of |x – 1| at x = 1. If ​  
lim  
​ (g (x)), = p,
[IIT-JEE, 2005] x Æ 1
then
45. If f (x) is continuous and differentiable function and
f (1/n) = 0 for every n ≥ 1 and n Œ I, then (a) n = 1, m = 1 (b) n = 1, m = – 1
(c) n = 2, m = 2 (d) n > 2, m = n.
(a) f (x) = 0, x Œ (0, 1]
(b) f (0) = 0 and f ¢(0) = 0 [IIT-JEE, 2008]
(c) f (0) = 0 = f ¢(0), x Œ (0, 1] 52. Let f and g be real valued functions defined on (– 1, 1)
(d) f (0) = 0 and f ¢(0) need to be zero. such that g¢¢(x) is continuous, g (0) π 0, f ¢(0) = 0 and
[IIT-JEE, 2005] f (x) = g (x) sin x
46. Let f be a twice differentiable function satisfying Statement-I: ​ l 
im  ​ (g(x) cot x – g(0)cosecx x) =
x Æ 0
f (1) = 1,  f(2) = 4, f (3) = 9, then
f ≤(0)
(a) f ¢¢(x) = 2, " Œ R
(b) f ¢(x) = 5 = f ¢¢(x) for some x Œ (1, 3) Statement-II: f ¢(0) = g (0)
[IIT-JEE, 2008]
(c) there exists atleast one x Œ (1, 3) such that
f ¢¢(x) = 2 53. Let f (x) be a non constant twice differentiable
(d) None of the above. function defined on (– •, •) such that f (x) = f (1 – x)


[IIT-JEE, 2005]
47. If f (x – y) = f (x). g (y) – f (y). g (x) and g (x – y) =
1
(  )
and f  ​ __
​   ​   ​ = 0. Then
4
g (x). g (y) + f (x). f (y) for all x, y Œ R, if right hand (a) f ¢(x) vanishes at least twice on [0, 1]
derivative at x = 0 exists for f (x). Then find the
derivative of g(x) at x = 0.
[IIT-JEE, 2005]
(  )
1
(b) f ¢ ​ __
​   ​   ​ = 0
2

(c) ​Ú  ​ ​ ​ f  ​( x + __


2 3 1/2
​   ​  )​ sin x dx = 0
48. Let f (x) = min {1, x , x }, then 1

(a) f (x) is continuous for all x ŒR –1/2 2
(b) f ¢ (x) = 0 for every x > 1 1/2 1/2
(c) continuous but not differentiable for all x Œ R (d) ​Ú   ​ ​  f (t) esin(p t) dt = ​Ú ​   ​ ​ f (1 – t) esin(pt) dt
(d) differentiable everywhere. –1/2 0
[IIT-JEE, 2006] [IIT-JEE, 2008]
3
49. If f ≤ (x) = – f (x), where f (x) is a continuous double 54. If the function f (x) = x + e x/2
and g (x) = f  –1(x), then
differentiable function and g (x) = f ¢(x) the value of g¢(1). [IIT-JEE, 2009]
If (  (  ) ) (  (  ) )
2
x 2
F (x) = ​​ f ​ __
x 2
​    ​  ​  ​​ ​ + ​​ g​ __
​    ​  ​  ​​ ​
2
55. Let f be a real valued function defined on the interval
(0, •) by
and F(5) = 5 then F(10) is x
_________
(a) 0 (b) 5 f (x) = ln x + Ú​  ​  ​  ​÷1  
  +  sin t dt  

0
(c) 10 (d) 25.
[IIT-JEE, 2006] Then which one of the following statements is true.
50. Match the functions in Column-I with the properties (a) f≤(x) exists for all x Œ (0, •)
in Column-II (b) f ¢(x) exists for all x Œ (0, •) and
Column-I Column-II and f ¢ is continuous on (0, •) but
not differentiable on (0, •)
(a) x |x| (p) Cont. in (–1, 1)
__ (c) there exists a > 1 such that
(b)    
​÷|x| ​ (q) differentiable in (–1, 1)
| f ¢(x)| < | f (x)| for all x Œ (a, •)
(c) x + [x] (r) stictly inc in (–1, 1) (d) there exists b > 0 such that
(d) |x – 1| + (s) not differentiable atleast at |f ¢(x)| + | f (x)| £ b for all x Œ (0, •)
|x + 1| one point in (– 1, 1)
[IIT-JEE, 2010]
[IIT-JEE, 2007]
The Continuity and Differentiability  4.27

Ï p
Ô- x - 2 : x£-
p
2
( 1
(b) a unique point in ​ n + __ )
​   ​ , n + 1  ​
2
Ô (c) a unique point in (n, n + 1)
Ô p
56. If f (x) = Ì- cos x : - <x£0 (d) two points in (n, n + 1)
Ô 2
[IIT-JEE, 2013]
Ôx - 1 : 0 < x <1
Ôln x : x >1 61. Let f : [a, b] Æ [1, •) be a continuous function and
Ó
g : R Æ R be defined as
then
p
(a) f (x) is cont at x = – __
​   ​ 
2 Ï
(b) f (x) is not differentiable at x = 0 Ô0 : x<a
Ô
(c) f (x) is differentiable at x = 1 Ôx
g (x) = Ì Ú f (t )dt : x £ x £ b
(d) f (x) is differentiable at x = – 3/2
Ôa
[IIT-JEE, 2011] Ôb
ÔÚ f (t )dt : x > b
57. Let f : R Æ R be a function such that f (x + y) = f (x) Óa
+ f (y) for all x, y in R.
(a) g(x) is continuous but not differentiable at x = a.
If f (x) is differentiable at x = 0, then
(b) g (x) is differentiable on R.
(a) f (x) is differentiable only in a finite interval
(c) g (x) is continuous but not differentiable at x = b.
containing zero.
(d) g (x) is continuous and differentiable at either
(b) f(x) is continuous " x Œ R
x = a or x = b but not both.
(c) is constant " x Œ R
[IIT-JEE, 2014]
(d) f (x) is differentiable except at finitely many
points. 62. Let f : R Æ R and g : R Æ R be respectively given by
[IIT-JEE, 2011] f (x) = |x| + 1 and

Ï 2 Êpˆ g(x) = x2 + 1. Define h : R Æ R by


Ô x cos Á ˜ : xπ0
58. Let f  (x) = Ì Ë x¯
Ô0 Ïmax{ f ( x ), g ( x )} : x £ 0
Ó : x=0 h (x) = Ì .
then f is Ómin{ f ( x ), g ( x )} : x > 0
(a) differentiable both at x = 0, x = 2 Then the number of points at which h (x) is not
(b) differentiable at x = 0 but not differentiable at differentiable is...
x = 2. [IIT-JEE, 2014]
(c) differentiable at x = 2 but not differentiable at
x = 0. 63. Let g : R Æ R be a differential function
(d) neither differentiable at x = 0 nor at x = 2. g (0) = 0, g ¢ (0) = 0 and g ¢ (1) π 0
[IIT-JEE, 2012] Ï xg( x )
Ô : xπ0
59. Let f : [0, 1] Æ R be a function. Suppose the function Let f (x) = Ì | x |
f is twice differentiable, Ô0 : x=0
Ó
f (0) = 0 = f (1) and satisfies
and h (x) = e|x| for all x in R
f ¢¢ (x) – 2f ¢ (x) + f (x) ≥ ex, x Œ [0, 1]
Let ( f 0 h) (x) denote f (h (x))
Which of the following is true. and (h 0 f) (x) denote h ( f (x))
1 1
(a) 0 < f (x) < • (b) – ​ __  ​ < f (x) < __
​   ​  Then which of the following is (are) true?
2 2
1
__ (a) f is differentiable at x = 0
(c) – ​   ​  < f (x) < 1 (d) – • < f(x) < 0
4
[IIT-JEE, 2013] (b) h is differentiable at x = 0
60. Let f (x) = x sin (p x), x > 0. Then for all natural (c) ( f 0 h) is differentiable at x = 0
number n, f  ¢(x) vanishes at (d) (h 0 f) is differentiable at x = 0.
( 
(a) a unique point in ​ n, n + __
1
)
​   ​   ​
2
[IIT-JEE, 2015]
4.28  Differential Calculus Booster

Answers
Level-II Comprehensive Link Passages
1. (a) 2. (b) 3. (a) 4. (a) 5. (a) Passage I : 1. (b) 2. (a) 3. (c)
6. (c) 7. (d) 8. (a) 9. (c) 10. (d) Passage II : 1. (b) 2. (c) 3. (c)
11. (d) 12. (a) 13. (c) 14. (b) 15. (d) Passage III : 1. (c) 2. (a) 3. (a)
16. (d) 17. (c) 18. (b) 19. (c) 20. (c) Passage IV : 1. (c) 2. (a) 3. (a)
21. (b) 22. (c) 23. (d) 24. (c) 25. (b) Passage VI : 1. (d) 2. (a) 3. (a)
26. (a) 27. (d) 28. (a,b) 29. (a) 30. (d) Passage VI : 1. (b) 2. (c) 3. (b)
31. (b) 32. (c) 33. (c) 34. (b) 35. (d) Passage VII : 1. (d) 2. (b) 3. (a)
36. (b) 37. (a) 38. (b) 39. (b) 40. (b,c)
41. (a,b) 42. (c) 43. (a,c) 44. (a,c) 45. (a) Matrix Match
46. (d) 47. (b) 48. (c) 49. (c) 50. (b) 1. (A) Æ (S), (B) Æ (R),
51. (c) 52. (d) 53. (b) 54. (d) 55. (b) (C) Æ (S), (D) Æ (S)
56. (a) 57. (c) 58. (c) 2. (A) Æ (P, R), (B) Æ (Q, R, S),
(C) Æ (Q, R, S), (D) Æ (P, Q, R, S)
Integer Type Questions 3.
1. 7 2. 5 3. 5 4. 7 5. 8 4. (A) Æ (S), (B) Æ (P),
6. 5 7. 5 8. 3 9. 6 10. 4 (C) Æ (Q), (D) Æ (R)
11. 5 12. 3.

Hints and solutions

Type I = ​   ( 
ax + bx + cx – 3
lim ​ ​ ​  _____________
x     ​  
x Æ 0
​ )
1. (i) f (0) = 1

(ii) ​ lim 
|x|
lim ​ ​ ​ __ ​   ​
  ​   f (x) = ​  
x Æ 0 x Æ 0 x| (  ) (  (ax – 1) + (bx – 1) + (cx – 1)
lim ​ ​ ​  _______________________
= ​  
x Æ 0
   
x    ​  ​ )
lim ​ ​ ( ​ _______ )​
x x x
(a – 1) (b – 1) (c – 1)
Now, ​ lim    + 
x Æ 0 x
x
​ ​ __ (  )
​   ​  ​ = 1 = ​   x   
x Æ 0
​ + ​ _______
x   ​ + ​ _______
x    ​  

= log a + log b + log c


​ ​( ___
– x
and ​   
lim   –
​  x   ​ )​ = – 1
x Æ 0 = log (abc)
Thus, limit does not exist.
(iii) ​ lim 
  ​  f (x) = log (abc) = f (0)
Hence, f (x) is discontinuous at x = 0 x Æ 0

2. (i) f (0) = 2 Thus, f (x) is continuous at x = 0


4. (i) f (0) = 5

( 
(ii) ​ lim 
  ​   f (x)
x Æ 0

(  sin x
) (ii​ )  
lim   ​f (x) = ​  
e3x – 1
lim ​ ​ ​ __________
    ​  ​ )
( 
lim ​ ​ ​ ____
= ​   ​ + cos x  ​
x    x Æ 0    x Æ 0 log (1 + 5x)

)
x Æ 0

=1+1 e3x – 1
​ ______
    ​ 
3x 3x
=2 = ​  
lim  ​ ​ ​ __________   ​ × ___
   ​    ​  ​
x Æ 0 log (1 + 5x) 5x
(iii) ​ lim 
  ​   f (x) = 2 = f (0) ​ __________
    ​ 
x Æ 0 5x
3
Thus, f (x) is continuous at x = 0 = __
​   ​ 
5
3. (i) f (0) = log (abc)
3
  ​   f (x) = __
(iii) ​ lim  ​   ​  π f (0)
(ii) ​ lim 
  ​    f (x) x Æ 0 5
x Æ 0
The Continuity and Differentiability  4.29

Thus, f (x) is discontinuous at x = 0 Type-II


5. (i) f (0) = 2 8. Since f (x) is continuous at x = 0, so

(ii) ​ lim 
  ​  f (x) = ​  
x Æ 0 x Æ 0
x – |x|
lim ​ ​ _____
​   ​  
2 ( 
 ​ ) ​ lim 
  ​  
f (x) = f (0) = k
x Æ 0 

(  )
x–x fi k = ​  
lim ​  f (x)
Now,​    lim   ​ ​ ​ ____
 ​  
 ​ = 0 x Æ 0

(  )
x Æ 0+ 2 sin 5x
lim ​ ​ _____
(  )
x+x fi k = ​   ​      ​  ​
and ​   lim   ​ ​ ​ _____
 ​  
 ​ = 0 x Æ 0 3x
x Æ 0 – 2
lim ​ ​ ( _____ ​ × ​   ​  )​
sin 5x __ 5
Thus,​  
lim 
 ​  
 f (x) = 0 fi k = ​   ​     
x Æ 0
x Æ 0 5x 3
5
  ​   f (x) = 0 π f (0)
(iii) ​ lim  fi k = __
​   ​ 
x Æ 0  3
Thus, f (x) is discontinuous at x = 0 p
9. Since f (x) is continuous at x = ​ __ ​ 
2
6. (i) f (0) = 7 p
(  )
( 
so, ​  
lim   
​   f (x) = ​
f   __
​   
 ​  ​ = 3
(ii)​   
lim ​   f (x) = ​  
ex – 1
lim ​ ​ ​ __________
x Æ 0 log (1 + 2x)
  
  ​  ​ ) p
x Æ ​ __ ​ 
2
2

 ( (  )
x Æ 0
k cos x
  p ​ ​ ​ ______ 
fi ​ lim 

)
 ​  ​ = 3
e –1 x x Æ ​ __ ​  p – 2x
​ _____
x   ​  2

( ​ _______
1
– 2 )
lim ​ ​ ​ __________
= ​       ​ × __
​    ​  ​ fi ​ lim 
  ​ ​ 
– k sin x
 ​   ​ = 3
x Æ 0 log (1 + 2x) 2
​ __________
      
​ p
x Æ ​ __ ​ 
2x 2
1
= __
​   ​  fi ​ __  ​ = 3
k
2 2
1 fi k=6
lim ​   f (x) = __
(iii)​    ​   ​  π f (0)
x Æ 0 2 Hence, the value of k is 6.
10. Since f (x) is continuous at x = 2, so
Thus, f (x) is discontinuous at x = 0
​ lim 
  ​   f (x) = f (2) = k
3 x Æ 2
7. (i) f (0) = __
​   ​ 
2 k = ​  
x Æ 2 ( 
2 x +2 – 16
lim ​ ​ ​  ________
4x – 16
 ​  
  ​ )
( 
(ii) ​ lim 
    ​  f (x)
x Æ 0 4 (2x – 4)
lim ​ ​ ​ _____________ )
( 
k = ​          ​  ​

)
log (1 + 3x) x Æ 2 (2x – 4) (2x + 4)
​ ​ ​ __________
( 
Now, ​   lim        ​ 
  ​
)
( 
e2x – 1 4
lim ​ ​ _______
+
x Æ 0
k = ​   ​      ​  ​

)
x Æ 2 (2x + 4)
log(1 + 3x)
​ __________
     ​  4 1
3x 3 k = __
​   ​  = __
​   ​ 
= ​ lim 
  +  ​ ​ ​ __________  
 ​  × __
​   ​   ​ 8 2
x Æ 0 2x
–1
e______ 2 1
​     

​ Hence, the value of k is __ ​   ​ 
2x 2
3
= ​ __ ​  11. Since f (x) is continuous at x = 0, so
2

( 
​ lim 
  ​  
f (x) = f (0) = k
Also, ​ lim 
  – 
sin 3x
​​ _____
​ 
x Æ 0   tan 2x
  ​  ​ ) x Æ 0 

(  )
k = ​  
lim ​  
f (x)

( 
x Æ 0 

= ​ lim 
  – 
sin 3x
_____
​     
​ 
3x
​ ​ _____
​ 
  ​ ×
3
__
​   ​   ​
k = ​  
x Æ 0
cos2 x_____ – sin2 x – 1
lim ​ ​ ​  ______________   
÷​ x  2 + 1 ​ – 1
   ​  ​
)
( 
x Æ 0 tan 2x 2
​ _____
)
    
​ cos 2x –1
2x lim ​ ​ ​ __________
k = ​   _____      ​  ​
3
= ​ __ ​ 
x Æ 0
÷​ x  + 1 ​ – 1
2

( 

)
2 _____
sin2 x ( 2
3 lim ​ ​ – ​ _____
k = ​    ​  
 × ​ ÷
​ x  + 1 ​  + 1 )​  ​
(iii) ​ lim 
  ​   f (x) = __ ​    ​ = f (0) x Æ 0 x2
x Æ 0  2
k = 2
Thus, f (x) is continuous at x = 0
Hence, the value of k is 2.
4.30  Differential Calculus Booster

12. Since f (x) is continuous at x = 0, so ​ lim 


  +  
​  f (x) = ​ lim 
  –   
​ f (x) = (a + b)
x Æ 4 x Æ 4

(  ( 
1
  ​   f (x) = f (0) = __
​ lim 
x Æ 0

​ lim 
1
__
  ​   f (x) = ​    ​
​    ​
2 ​ lim 
  + 
(x – 4)
​​  ​ ______ ​ 
x Æ 4 (x – 4)
+ b  ​ =​  lim 
x Æ 4 –
  ) (x – 4)
​ ​ – ​ ______ 
(x – 4)
 ​ + a  ​ = (a + b) )
x Æ 0 2
1 + b = – I + a = (a + b)
​ lim 
  ​ ​ ​ 
x Æ 0 ( 
1_________
– cos kx
x sin x

    ) 1
​  ​ = ​ __ 
2 a = 1, b = – 1

( 
15. Since f (x) is continuous at x = 0, so
​ 
k sin (kx)
  ​ ​  ___________
 ​ ​ lim    
1
   ​  ​ = __
​   ​  ) ​ lim 
  ​  
f (x) = f (0) = a

( 
x Æ 0 sin x + x cos x 2

(  )
x Æ 0  

)
p
2 tan– 1 (ex) – __
( 
​   ​ 
)
k sin (k x)
________ be x
– cos x – x 4
​  x    ​  1 ​ lim 
  +  ​ ​ ​ ____________
 ​ 
     ​ = ​  
lim   ​ ​ ​ ____________
     ​   ​= a
  ​ ​ __________
​ lim  ​        ​  ​ = __
​    ​ x Æ 0 x 2 x Æ 0 – x
sin x
x Æ 0 ____ 2

) ( 
​  x   ​ + cos x
( 
(  )
2
k  sin (k x)
​ ________
    ​ 
​ lim 
  + 
x Æ 0
bex + sin x – 1
​ ​ ​ ____________
2x
       ​  ​ = ​  
lim  
x Æ 0 1 + e2x

2ex
​ ​ ​ ______
   ​  ​ = a )
​ ​( ​ _________ )​ = 1 = a
k x 1 x
  ​ ​ ​ __________
​ lim       ​  ​ = __
​   ​  ​ lim 
   
be + cos x
 ​   
sin x
x Æ 0 ____ 2 2
+
​  x  ​ +
  cos x
  x Æ 0

k2 1 b+1
​ __ ​  = __
​   ​  ​ _____
 ​ 
 = 1 = a
2 2 2
k2 = 1 a = 1 = b
l = ± 1 Type IV
Type III 16. Since f (x) is continuous at x = 0, so
13. Since f (x) is continuous at x = 0, so ​ lim 
  ​  
f (x) = f (0)
x Æ 0  
​ lim 
  ​   f (x) = f (0) = c
x Æ 0
x
log ​ 1 + __
(  (  x
)
​ a ​  ​ – log ​ 1 – __
lim ​ ​ ​ ____________________
f (0) = ​      
​   ​  ​
b
  ​  ​
(  ) )
​ lim 
  ​   f (x) = c
x Æ 0 x Æ 0 x 
​ lim 
  +   
​ f (x) = ​ lim 
x Æ 0
  –   
​ f (x) = c
x Æ 0
x
log ​ 1 + __
(  ( 
x
__
)
​ a ​  ​ log ​ 1 – ​ b ​  ​
lim ​ ​ ​ _________ – ​ _________
(  ) )
( 
______ f (0) = ​   x   
​  x   

​  ​

)
( 
__ x Æ 0
÷​ x  + bx2  ​ – ÷
​ x 
  ​ 

)
​ ​ ​  ____________
( 
​ lim 
  +   ​ 
     ​
x Æ 0 bx 3/2
log ​ 1 + __
lim ​ ​ ​ _________
f (0) = ​  
x
   
( 

)
​ a ​  ​ log ​ 1 – ​ b ​  ​
​ – ​ _________
x
__
  
​  ​
)
x  x
( 
__ – ​ __ ​ × – b
)
x Æ 0
sin (a + 1) x + sin x ​ a ​ × a
= ​  
lim 

 ​ ​ ​  _______________
x     ​   ​= c b
x Æ 0

( 
1 1
f (0) = __
​ a ​ + __
)
​   ​
x + bx2 – x b
​ lim 
  +  ​ ​​  _________________
    
______  ​  ​ = (a + 2) = c
bx3/2 ​( ÷    ​ )​
__
x Æ 0
​ x  + bx2 
 ​+ ÷
​ x 

( 
17. Since f (x) is continuous at x = 0, so

))
2
bx
​ lim 
  +  ​ ​ ​ _________________
     
______  ​  ​ = (a + 2) = c ​ lim 
  ​  
f (x) = f (0)
3/2 ( __
x Æ 0
bx  ​ ÷ ​ x  + bx2   ​+ ÷ ​ x    ​  ​
x Æ 0 

​ lim 
  + 
x Æ 0 ( ( 
​  ______ 
÷
​ ​ ______________
__
​ x 
   ​

 ​  ​÷x  + bx2 ​ 
  
)
 ​  ​ = (a + 2) = c
   ​ )​
__
x Æ 0 x5( 
sin 3x + A sin 2x + B sin x
lim ​ ​ ​ ____________________
f (0) = ​    ​ 
       ​
)
[  ( 
+÷​ x 
​ lim 
  + 
x Æ 0 (​ ÷ ( 
​ ​ ___________
​  ______
1
​ 1  + bx  
     ​  ​ = (a + 2) = c
​+ 1 )​ ) = ​   ​ ​ __
lim  
x Æ 0 x
1 (3x)3 _____
​  5  ​ ​  3x – _____
​   ​ 
3!
(3x)5
 + ​   ​ 
5!
 – ...  ​ ​

)
( 
1
​ __ ​  = (a + c) = c
2
3 1
a = –  ​ __ ​ , c = __
​   ​ , b Œ R – {0}

(2x)3 _____
+ A ​ 2x – ​ _____
3!
 ​ 
(2x)5
 + ​   ​ 
5!
 – ...  ​ )
2 2
14. Since f (x) is continuous at x = 4, so
​ lim 
  ​  
f (x) = f (4)
(  (x)3 (x)5
+ B ​ x – ____
​   ​ + ____
3!
​   ​ – ...  ​
5! )
x Æ 4 
The Continuity and Differentiability  4.31

(  )
Since f (x) is continuous at x = 0, it must 1
__

2A + 3 + B = 0
​ lim 
  + 
x Æ 0
1
​ xn sin ​ __ (  )
​ x ​  ​ = ​  
lim 
x Æ 0 –

xn ​e​ ​x ​​
​ ​ ​ ______  1

__
 ​  ​ = 0
1 + ​e​ ​x ​​
8A B 27
have ​ ___ ​ + ​ __ ​  + ___
​   ​ = 0 It is possible only when n is natural number.
6 6 6 Hence, the smallest value of n is 1.
On solving, we get, A = – 4, B = 5
21. Since f (x) is continuous at x = 0, so
35 A ◊ 25 __ B
Thus, f (0) = __
​   ​  + ​ _____
 ​ 
 + ​    ​  f (0) = ​  
lim ​   f (x)
5! 5! 5! x Æ 0


35
= __
​   ​  –
5!
27
__
​   ​  +
5!
__5
​    ​ 
5!
= ​  
x Æ 0 x
1
lim ​ ​ __
(  2
​   ​ – ______
​  2x    
e –1
 ​  ​
)
(  )
243 – 128 + 5
= ​ ____________
 ​
     e2x – 1 – 2x
120 lim ​ ​ ​ __________
= ​      ​  

( 
120
x Æ 0 x (e2x – 1)

)
= ____
​   ​ = 1
120 (2x)2 _____ (2x)3
18. Since f (x) is continuous at x = 0, so f (0) = ​  
lim ​  f (x) 1 + 2x + ​ _____
 ​ 
 + ​   ​   + .... – 1 – 2x
2! 3!
lim ​ ​ ​ _____________________________

( 
= ​            ​  ​
( 
x Æ 0

= ​  
x Æ 0
e2x – 1 – x (e2x + 1)
lim ​ ​ ​ ________________
x3
 ​      ​ )
x Æ 0
2 e
2x

2x
–1
______
2x  ​ ​     

​  ​ )

x Æ 0 ( 2e2x – (e2x + 1) – 2x e2x
lim ​ ​ ​ ____________________
= ​      
3x2
 ​     ​ ) = ​   ( (2x)2 _____
​ _____
2!
 ​ 
(2x)3
 + ​   ​ 
3!
 +....
lim ​​  ​  _______________
 ​ 
    
​ )
( 
x Æ 0 2x2
= ​  
x Æ 0
e2x – 2x e2x – 1
lim ​ ​ ​ ____________
3x2
 ​ 
     ​ ) =1

= ​  
x Æ 0 ( 2e2x – 2e2x – 4xe2x
lim ​ ​ ​ _______________
6x
       ​  ​ ) 22. Since f (x) is continuous at x = 0, so
​ lim 
  ​  
f (x) = f (0)

( 
x Æ 0 


x Æ 0
– 4 xe
lim ​ ​ ​ ______
= ​  
6x

   
2x
​  ​ ) f (0) = ​  
lim ​f  (x)
x Æ 0  

(  ( 
______

x Æ 0
– 4e2x – 8xe2x
lim ​ ​ ​ ___________
= ​  
6
 ​ 
     ​ ) = ​  
x Æ 0
2 – 4​÷x  2 + 16 ​ 
lim ​ ​ ​ ___________  
   ​  ​
cos 2x – 1 )
(  ÷ 
______

)
2
= – ​ __ ​  4 2
​ x + 16 ​ 
–2
3 lim ​ ​ ​  ___________
= ​     
    ​  ​
19. Since f is continuous in R, so it is continuous for all x Æ 0 1 – cos 2x

(  (  ÷  )
______
n in R.

  ​  
Now, f (0) = ​ lim f ​ __
n Æ •  4 n
1
(  )
​     ​  ​
x2
2​   ​ 1 + ​ ___  
16
4 

lim ​ ​ ​ ______________
= ​  
 ​ ​ – 1  ​
  
    ​  ​
)
x Æ 0 2 sin2 x

(   ( (  ÷ 
)
)
______
lim ​ ​ (​  sin ​​ (en)) e– n​
= ​  
2 n2
​ ​ + ​ ______
   ​  ​ x2
​ ​ 1 + ​ ___ 
4 
  ​ ​ – 1  ​ )
( 
n Æ •
n2 + 1 16
lim ​ ​ ​  ____________
= ​        ​  ​
1
) sin2 x
x Æ 0
​ ​ + ______
lim ​ ​ (sin (en)) e– n​
2
= ​   ​       ​  ​

( (  )
1
)
n Æ •
1 + __
​  2  ​  x2
n ​ 1 + ​ __ ​  + ...  – 1  ​
4
lim ​ ​ ​  _____________
( 
= ​         ​  ​

1
= ​ 0 + _____
​     
1+0
 ​  ​ ) x Æ 0 sin2 x

=1
20. Since f (x) is continuous at x = 0, so
= ​  
x Æ 0 4 sin2 x (  )
x2
lim ​ ​ ​ ______
    
1
​  ​ = __
​   
4

​ lim 
  ​  
f (x) = f (0)  ​Properties of Continuous Functions
x Æ 0 
23. Let f (x) = x – cos x
​ lim 
  +  
​ f (x) = ​ lim 
  –   
​ f (x) = f (0)
x Æ 0 x Æ 0
Now, f (0) = 0 – 1 = – 1
4.32  Differential Calculus Booster
p
and f ​ __
2 (  ) p
​   ​   ​ = __
p
​   ​  – 0 = __
2
​   ​  > 0
2 (  ) [x]
  ​ ​ ___
33. Let ​ lim 
x Æ 3
​  x   ​  ​
p
Thus, f (x) = 0 has a root in ​ 0, __
​   ​   ​
2 (  ) Then ​ lim 
    ​ ​( ___
​  x   ​ )​ = __
[x] 3
​   ​  = 1
24. Let f (x) = 2 tan x + 5x – 2 x Æ 3 + 3

​ ​( ___
​  x   ​ )​ = __
Now, f (0) = – 2 < 0 [x] 2
and ​ lim 
    ​    ​
x Æ 3 – 3
and
p
f ​ __ (  ) 5p
​   ​   ​ = 2 + ___
5p
​   ​ – 2 = ___
​   ​ > 0
4 4 5
Thus, (  )
2
Jump = ​ 1 – __
1
​   ​   ​ = __
​   ​ 
Thus, f (x) = 0 has a root in ​ 0, ​   ​   ​ (  )
p
__
4 34. Let ​ lim 
  ​ [sin x]
3 3

25. Let f (x) = x.2x – 1  Æ 0

Now, f (0) = – 1 < 0 Then ​ lim 


  + 
​ [sin x] = 0 and ​  
lim 

 ​ [sin x] = – 1
x Æ 0 x Æ 0
and f (1) = 1.2 – 1 = 1 > 0
Thus, f (x) = 0 has a root in (0, 1) Thus, Jump = (0 – (– 1)) = 1
26. Let f (x) = e 2x x  –1
+ e + 2 sin  x + x – p 35. A function is discontinuous at all such points where
it is undefined.
Now, f (0) = 1 + 1 – p = 2 – p < 0
1
and f (1) = e2 + e + p + 1 – p = e2 + e + 1 > 0 (i) f (x) = _____
​      ​
x–1
Thus, f (x) = 0 has a root in [0, 1] f (x) is discontinuous when x – 1 = 0
Disconinuity i.e. x = 1
27. Do yourself
Thus, the number of points of discontinuity is
28. Do yourself 1 at x = 1.
29. Do yourself 1
30. Do yourself. (ii) f (x) = _____
​       ​
|x| – 1
Ï | x| f (x) is discontinuous when |x| – 1 = 0
Ô : xπ0
31. Let f (x) = Ì x |x| = 1
ÔÓ0 : x = 0 x=±1
Thus, the number of points of discontinuity
is 2 at x = –1, 1.
1
(iii) f (x) = _____
​       ​
|x| + 2
Here, denominator of a function is defined for
all real values of x.
Thus, f (x) is continuous for all real x.
x2 + 1
(iv) f (x) = ______
​  2  ​ 
x –4
Thus, the graph of the function  (x) = sgn (x) makes
a jump of 2 units at the point x = 0 f (x) is discontinuous when x2 – 4 =0
Jump = (1 – (–1)) = 2 fi  x = ± 2

(  (  ) )
32. Let ​ lim 
1
  ​ ​ tan–1­ ​ __
​ x ​  ​  ​
x Æ 0
Thus, the number of points of discontinuity
is 2 at x = –2, 2.

​​( tan  ​( __​ x ​ )​ )​ = __
p 1
Then ​ lim 
   
1 –1
​   ​  (v) f (x) = _____
​     ​. 
x Æ 0 + 2 loge x

​​( tan  ​( __​ x ​ )​ )​ = – ​ __ ​ 


1 –1 p f (x) is discontinuous for
and ​ lim 
   
x Æ 0 – 2 x < 0, x = 0 and x = 1

Jump = ​( __
​   ​  – (​  – ​ __ ​  )​ )​ = p
p p Thus, the number of points of discontinuity is
Thus,
2 2 infinite.
The Continuity and Differentiability  4.33


1
(vi) f (x) = ______
​     ​ 
loge |x|
p
fi x = np + (–1) n ​ __
​   ​   ,​ n ŒI.
6 (  )
Thus, the number of points of discontinuity
f (x) is discontinuous for x = 0, x = –1, x = 1
is infinite.
Thus, the number of points of discontinuity is
3 at x = – 1, 0, 1. x2 –__ 7x + 2014
(xii) f (x) = ​ _____________  
    ​.
1 ÷    cos x – 1
​ 2 ​ 
(vii) f (x) = _________
​       ​ __
loge|x – 2| f (x) is discontinuous for ÷    cos x – 1 = 0
​ 2 ​ 
f (x) is discontinuous for |x – 2| = 0, |x – 2| = 1 1
fi cos x = ___ ​  __  ​ 
fi x = 2, x – 2 = ± 1 ÷   
​ 2 ​
p
fi x = 2, x = 3, 1 fi x = 2 np ± __ ​   ​ , n ŒI
4
Thus, the number of points of discontinuity is
Thus, the number of points of discontinuity is
3 at x = 1, 2, 3.
infinite.
1
(viii) f (x) = __________
​       ​. 36. We have f (x) = [[x]] – [x – 1]
loge (x2 – 1)
= [x] – [x – 1]
f (x) is discontinuous for (x2 – 1) < 0,
= [x] – ([x] – 1)
fi x2 – 1 = 0, x2 – 1 = 1
=1
fi – 1 < x < 1, x = ± 1, x = 0 Thus, f (x) is continuous for all real x.
Thus, the number of points of discontinuity is 37. As we know that [x] is discontinuous for all integral
infinite. values of x.
1 Thus, the function f(x) should be checked at all inte-
(ix) f (x) = ___________
​       ​ gral values of x for which [sin p x] Œ I.
loge|(x2 – 1)|
Now, sin p x = 0 fi p x = 0 fi x = 0 and sin px
f (x) is discontinuous for
p 1
|x2 – 1| = 0, |x2 – 1| = 1 = 1 px = 1 fi p x = __ ​ x ​ fi x = __
​   ​ 
2
fi x2 – 1 = 0, x2 – 1 = ± 1 Thus, f (x) = [sin px] is continuous for
fi x2 = 1, x2 = 1 ± 1
fi x = ± 1, x = 0 x = ± ​÷2 ​
  
__
all x Œ [0, 1] – ​ 0, __ {  }
1
​   ​   ​.
2
38. We have, f (x) = [x] + [– x]
Thus, the number
__
of points
__
of discontinuity is
  ,  –1, 0, 1, ÷
5 at x = –​÷2 ​   
​ 2 ​ Ï0, x ŒI

Ó-1, x œI
1
(x) f (x) = ___________
​  2      ​ Thus, f (x) is discontinuous for all x Œ I.
x – 3|x| + 2
39. We have f(x) = x + {– x} + [x], x Œ I
f (x) is discontinuous for x2 – 3 |x| + 2 = 0 = x + [x] + {– x}
fi |x2| – 3 |x| + 2 = 0 = {x} + {– x}
fi (|x| – 1) (|x| – 2) = 0 Ï1, x œI

fi ­­­­(|x| – 1) = 0, (|x| – 2) = 0 Ó0, x ŒI
fi x = ± 1, x = ± 2 Thus, f (x) is discontinuous at

Thus, the number of points of discontinuity is x = – 2, –1, 0, 1, 2.


4 at x = – 2, –1, 1, 2 40. (i) We have f (x) = [sin x]
2
x + 4x + 1 The function f (x) is discontinuous at all such
(xi) f (x) = ​ __________   ​  points where it is broken.
2 sin x – 1
f (x) is discontinuous for 2 sin x – 1 = 0 sin x = 0 fi x = 0, p, 2p
1 p
fi sin x = __ ​   ​  sin x = 1 fi x = __
​   ​ 
2 2
4.34  Differential Calculus Booster
3p
sin x = – 1 fi x = ___ ​   ​ 
2p 4p
2 cos x = – 1 fi x = ​ ___ ​ , ​ ___ ​ 
2 3 3
Thus, the number of points of discontinuity are 2 cos x = – 2 fi x = p
p 3p
4 at x = 0, __
​   ​ , p, 2p (it is continuous at x = ___
​   ​  2 cos x = 2 fi x = 0, 2p
2 2
__
(ii) We have f (x) = Î​÷2 ​     sin x˚, " x Œ [0, 2p] Thus, the number of points of discontinuity
p p ___ 2p 3p ___ 4p 5p
The function f (x) is discontinuous at all such = 8, at x = 0, __
​   ​ , __
​   ​ , ​   ​ , ___
​   ​ , ​   ​ , ___
​   ​ , 2p.
points where it is broken. 3 2 3 2 3 3
1
21. We have f (u) = _________
__
​÷ 2 ​  sin x = 0 fi x = 0, p, 2p ​  2      ​
u +u–2
__ p 3p
​÷ 2 ​  sin x = 1 fi x = __ ​   ​ , ___
​   ​ 
1
= ____________
​      ​
4 4 (u + 2) (u – 1)
__ 5p 7p
​÷ 2 ​  sin x = –1 fi x = ___
​   ​ , ___
​   ​  Now, u+2=0
4 4
fi u = – 2
Thus, the number of points of discontinuity
1
p 3p 5p 7p fi ​ _____   
 ​ = – 2
= 7 at x = 0, __
​   ​ , ___
​   ​ , p, ___
​   ​ , ___
​   ​ , 2p. x–1
4 4 4 4 1
fi x – 1 = – ​ __  ​
(iii) We have f (x) = [sinx + cosx], " x Œ[0, 2p] 2
1 1
fi x = 1 – __​    ​ = __
​    ​
[  (  (  ) ) ]
__ p 2 2
    sin ​ x + __
= ​ ​÷2 ​ ​ ​   ​   ​  ​  ​
4 Also, u–1=0
The function f (x) is discontinuous at all such fi u=1
points where it is broken. 1
fi ​ _____
   ​ = 1
(  (  ) )
__ p p 3p ___ 7p
​÷ 2 ​  ​  sin ​ x + ​ __ ​   ​  ​ = 0 fi x = – ​ __ ​ , ___
​   ​ , ​   ​  x–1
4 4 4 4
fi x–1=1
(  (  ) )
__ p p
​÷ 2 ​ ​  sin ​ x + __
​   ​   ​  ​ = 1 fi x = 0, __ ​   ​  fi x=2
4 2
Thus, the number of points of discontinuity
​÷ 2 ​ ​( sin ​( x + __
__
​   ​  )​ )​ = – 1 fi x = p, ___
p 3p
  ​   ​ 
4 4 1
= 3 at x = __ ​   ​ , 1, 2
Thus, the number of points of discontinuity 2
22. We have, f (f (f (x)))
p 3p 3p 7p
= 7 at x = 0, __
​   ​ , ___
​   ​ , p, ___
​   ​ , ___
​   ​ , 2p.


2 4
(iv) We have f (x) = [sin p x], " x Œ[0, 2]
2 4
(  (  ) )1
= f ​ f  ​ _____
​     ​  ​  ​
1  –  x
1
The function f (x) is discontinuous at all such = f  ( f (a)), where a = _____
​      ​
points where it is broken. 1–x
sin p x = 0 fi x = 0, 1, 2 ( (  ) )
1
= f  ​ ​ _____
​      ​  ​  ​

( 
1–a
1
sin p x = 1 fi x = __

)
​   ​ 
2 1
= f  ​ ________
​       ​  ​
3 1
sin p x = –1 fi x = __ ​   ​  1 – _____
​      ​
2 1–x
Thus, the number of points of discontinuity is
(  1–x
= f  ​​ ________
    ​  ​ )
1
4 at x = 0, __ (  3
​   ​ , 1, 2 ​ it is continuous at x  = ​ __ ​   ​. ) 1–x–1

( 
2 2
(v) We have f (x) = [2 cos x], " x Œ [0, 2p]

x–1
= f  ​ ​ _____
x    )
​  ​
x–1
The function f (x) is discontinuous at all such = f (b), where b = _____
​  x   
​ 
points where it is broken. 1
p 3p = _____
​     
2 cos x = 0 fi x = __
​   ​ , ___
​   ​  1–b
2 2 1
p 3p ___ 5p  ​ = ​ ________
   ​ 
2 cos x = 1 fi x = __
​   ​ , ___
​   ​ , ​   ​  x–1
1 – ​ ____
3 2 3 x   
​ 
The Continuity and Differentiability  4.35
x
= ________
​       ​ Ï1 : x Œ (-•, - 1) » (0, 1)
x–x+1 Ô
= Ì0 : x = 0, ± 1
=x Ô-1: x Œ (-1, 0) » (1, •)
Ó
Thus, the number of points of discontinuity
= 2 at x = 1 and x = 0 Thus, f  (g(x)) is discontinuous at x = – 1, 0, 1.

Note:  If x π 0, 1 then the function Note: g( f (x)) is continuous for all real x.
1 since g( f (x)) = sgn (x) (1– (sgn (x))2)
f ( f ( f (x))) = ​ ________
     ​ = x
x–1
1 – _____
​  x   
​  Ï | x| Ê Ê | x|ˆ 2 ˆ
is continuous everywhere. Ô Á1 - ˜ : xπ0
= Ì x Ë ÁË x ˜¯ ¯
23. We have h(x) = f  (g(x)) Ô
Ó0 : x=0
= f (x2 + 5)
Ï0 : x > 0
1 1 Ô
= __________
​  2      ​ = ______
​       ​ = Ì0 : x = 0
(x + 5) – 6 x2 – 1 Ô0 : x < 0
Ó
It is continuous at x Œ R – {– 1, 1}.
24. We have g(x) = f ( f  (x)) 26. We have f (x) = |x + 1| (|x| + |x – 1|)

Ï1 + f ( x ) : 0 £ f ( x ) £ 2 Ï( x + 1)(2 x – 1) : – 2 £ x < –1
=Ì Ô – ( x + 1)(2 x – 1)
Ó3 - f ( x ) : 2 < f ( x ) £ 3 Ô : –1£ x < 0
= Ì
Ï1 + (1 + x ) : 0 £ 1 + x £ 2, 0 £ x £ 2 Ôx + 1 : 0 £ x <1
Ô3 - (1 + x ) : 2 < 1 + x £ 3, 0 £ x £ 2 ÔÓ( x + 1)(2 x – 1) : 1£ x £ 2
Ô

Ô1 + (3 - x ) : 0 £ 3 - x £ 2, 2 < x £ 3
ÔÓ3 - (3 - x ) : 2 < 3 - x £ 3, 2 < x £ 3

Ï2 + x : - 1 £ x £ 1, 0 £ x £ 2
Ô2 - x :1 < x £ 2, 0 £ x £ 2
Ô

Ô4 - x : 2 £ x £ 3, 2 < x £ 3
ÔÓ x : 0 < x £ 1, 2 < x £ 3

Ï2 + x : - 1 £ x £ 1
Ô2 - x :1 < x £ 2
Ô Thus f (x) is continuous everywhere.
= Ì
Ô4 - x : 2 < x £ 3 Ï1 – x : 0 £ x £ 2
ÔÓ x : x Œj 27. We have f (x) = Ì
Óx – 3 : 2 < x £ 4
Ï2 + x : - 1 £ x £ 1
Ô Ï2 + x : – 1 £ x £ 0
= Ì2 - x :1 < x £ 2 and g(x) = Ì
Ô4 - x : 2 < x £ 3 Ó2 – x : 0 < x £ 3
Ó
Now (f 0 g) (x)
Thus, g(x) is discontinuous at x = 1 and x = 2.
= f (g(x))
25. Clearly f (x) is a signum function of x
i.e. f  (x) = sgn (x). Ï1 – g( x ) : 0 £ g( x ) £ 2
= Ì
Now, f (g(x)) = sgn (x(1 – x2)) Ó g( x ) – 3 : 2 < g( x ) £ 4

Ï1 : x(1 - x 2 ) > 0 Ï1 – (2 + x ) : 0 £ 2 + x £ 2, – 1 £ x £ 0
ÔÔ Ô2 + ( x – 3) : 2 < 2 + x £ 4, – 1 £ x £ 0
Ô
= Ì0 : x(1 - x 2 ) = 0 =Ì
Ô Ô1 – (2 – x ) : 0 £ 2 – x £ 2, 0 < x £ 3
2
ÔÓ-1: x(1 - x ) < 0 ÔÓ2 – ( x + 3) : 2 < 2 – x £ 4, 0 < x £ 3
4.36  Differential Calculus Booster

Ï –1 – x : – 2 £ x £ 0, – 1 £ x £ 0 34. Let f (x) = x5 – 2x3 + x2 – 3x


Ôx – 1 : 0 < x £ 2, –1 £ x £ 0 Clearly, f is continuous for all real x.
Ô

Ôx – 1 : 0 £ x £ 2, 0 < x £ 3 Now, f (1) = 1 – 2 + 1 – 3 = – 3 < –1
ÔÓ – x – 1 : – 2 < x £ 0, 0 < x £ 3 and f (2) = 32 – 16 + 4 – 6 = 16 > –1
Hence, from the Intermediate value theorem there
Ï –1 – x : –1 £ x £ 0 exist a number c in (1, 2) such that f (c) = –1.
Ôx – 1 : x Œf
Ô 35. Let f (x) = 2x3 + x2 – x
= Ì
Ôx – 1 : 0£ x£2 Clearly, f is continuous for all real x.
ÔÓ – x – 1 : x Œf Now, f (1) = 2 + 1 – 1 = 2 < 5
and f (2) = 16 + 4 – 2 = 18 > 5
Ï –1 – x : –1 £ x £ 0
=Ì Hence, from the Intermediate value theorem there
Óx – 1 : 0 £ x £ 2 exist a number c in (1, 2) such that f (c) = 5.
Thus, f (x) is continuous in [– 1, 2]. : x≥2
Ï( x – 2)
_____ 36. We have f (x) = |x – 2| = Ì
28. Let f (x) = ÷
1
​ x  – 5 ​ – _____
​      ​. Ó – ( x – 2) : x < 2

( 
x+3

Now,
_____ 1
f (5) = ​÷5  – 5 ​ – _____
​     
5+3
1
 ​ = – ​ __ ​  < 0
8
L.H.D = f ¢(2+) = ​ lim 
  + 
x Æ 2
f (x) – f (2)
​ ​ ​ _________
(x – 2)
 ​  

​ )
Also,
_____ 1
f (6) = ​÷6  – 5 ​ – _____
​     
6+3
 ​ = 1 – __
1 8
​   ​  = __
9 9
​   ​  > 0 = ​ lim 
  + 
x Æ 2 (  (x – 2) – 0
​ ​ ​  _________
(x – 2) )
 ​  

Thus, f (x) has a real root in (5, 6).


= 1.
29.

Let
Now,
f (x) = x5 + 3x4 + x – 2
f (0) = – 2 < 0
Also, R.H.D = f ¢(2–) = ​  
lim 
x Æ 2 –
  ( 
f (x) – f (2)
​ ​ ​ _________
(x – 2)
 ​ 

 ​ )
(  )
Also, f (1) = 1 + 3 +1 – 2 = 4 – 2 = > 0 – (x – 2) – 0
Thus, f (x) has a real root in [0, 1]. = ​  
lim   ​ ​ ​ ___________   ​  ​
x Æ 2 –
(x – 2)
30. Let f (x) = x5 – 3x + 1 = –1
Now, f (1) = 1 – 3 + 1 = 2 – 3 = –1 < 0 Since L.H.D π R.H.D, so, f (x) is not differentiable
Also, f (2) = 32 – 6 + 1 = 27 > 0 at x = 2.

( 
37. Now, R.H.D

31.
Thus,
Let
f (x) has a real root in [1, 2].
f (x) = x3 + x2 – 3x – 3
= f ¢(1+) = ​ lim 
x Æ 1
f (x) – f (1)
  +​ ​ _________
​ 
(x – 1)
 ​  

​)
( 
Now, f(1) = 1 + 1 – 3 – 3 = – 4 < 0


Also,
Thus,
f (2) = 8 + 4 – 6 – 3 = 3 > 0
f (x) has a real root in [1, 2].
= ​ lim 
x Æ 1
x2 – 1
  +​ ​ ​ ______ 
(x – 1)
  )
 ​  ​

= ​ lim 
  +​ ((x + 1))
32. Let f (x) = x5 + x x Æ 1

Clearly f is continuous for all real x. =2


Now, f (0) = 0 < 1 Also, L.H.D


and f (1) = 2 > 1
Hence, from the Intermediate value theorem there
= f ¢(1–) = ​ lim  (  )
x–1
  –​ ​​ ______ 
x Æ 1 (x – 1)

 ​  ​

exist a number c in (0, 1) such that f (c) = 1. = 1.

33. Let f (x) = x5 + 3x4 + x Since L.H.D π R.H.D, so, f (x) is not differentiable
at x = 1.
Clearly f is continuous for all real x.
38. We have, f ¢(1)
Now, f (0) = 0 < 2 and
f (1) = 1 + 3 + 1 = 5 > 2. = ​  
x Æ 1 ( 
f (x) – f (1)
​ ​ _________
lim   ​ 
x–1
 ​  

​)
Hence, from the Intermediate value theorem there
exist a number c in (0, 1) such that f(c) = 2. = ​  
lim  
x Æ 1
( 
ln2 x – 1
x–1 )
​ ​ ​ _______ ​  


The Continuity and Differentiability  4.37

= ​  
x Æ 1
(  )
_______
lim ​ ​ ​ 
1
 ​  
1
2ln x ◊ ​__
 x ​
 ​


x Æ 1

= 3 log 3
1(  )
3xlog3
  +​ ​ ______
= ​ lim  ​   ​   ​

= ​ lim 
x Æ 1+
( 
2lnx.
  ​ ​ ​ _____
x   ​  ​ ) Since L.H.D π R.H.D, so, f (x) is not differentiable
at x = 1.
=0 41. We have f (x) = sin x + |sin x|
Thus, f (x) is differentiable at x = 0. Ïsin x + sin x : x ≥ 0

39. We have Ósin x – sin x : x < 0
f (x) = e|x|
Ï2sin x : x ≥ 0

ÏÔe x : x≥0 Ó0 : x<0
=Ì Now, R.H.D
–x
ÔÓe : x<0
Now, R.H.D = f ¢(0 +)

= f ¢(0 +) = ​ lim 
   
x Æ 0 +
(  f (x) – f (0)
​ ​ ​ _________
x–0
 ​  
 )​

= ​ lim 
   
x Æ 0 +
( f (x) – f (0)
​ ​ ​ _________
x–0
 ​  

​ ) = ​ lim 
   
x Æ 0 +
( 
2sin x – 0
​ ​ ​ ________
x–0 )
 ​  
 ​

= ​  
lim 
x Æ 0+
( 
ex – 1
​ ​ ​ ______
x    
​  ​ ) =2
Also, L.H.D
= f ¢(0 –)
=1
Also, L.H.D
= f ¢(0 –)
= ​  
lim  
x Æ 0 – ( 
f (x) – f (0)
​ ​ __________
​ 
x–0
   ​  
​ )
= ​  
lim   ( f (x) – f (0)
​ ​ __________
​     ​  ​ )
= ​  
lim   (  )
0–0
​ ​ ​ _____ 
x Æ 0 – x – 0
 ​  ​ = 0
x Æ 0 – x–0
Since L.H.D π R.H.D, so, f (x) is not differentiable
= ​  
lim 
x Æ 0 –
  ( e–x – 1
​ ​ ​ ______
x  ​  
 ​) at x = 0.
42. We have f (x) = [x] tan(px)
= –1. Now, R.H.D
Since L.H.D π R.H.D, so, f (x) is not differentiable = f ¢(k +)

( 
at x = 0.
40. We have, R.H.D
+
= ​  
lim 
x Æ k +

f (x) – f (k)
​ ​ _________
​ 
x–k
  
  ​  
​ )
= f ¢(1 )
( 

( 
f (x) – f (1)
  +​ ​ _________
= ​ lim 
x Æ 1
​ 
x–1
 ​  

​ ) = ​ lim 
   
h Æ 0 +
f (k + h) – f (k)
​ ​ ​ _____________
  
  
(k  +  h) – k )
  ​  ​


x Æ 1
( 
4–x–3
  +​ ​ ​ ________
= ​ lim 
x–1
 ​  

​) = ​ lim 
  + 
h Æ 0
( 
f (k + h) – f (k)
​ ​ ​ _____________
h
    ​   ​ )
  ​ ​( ​ _____ 
x – 1) (  )
1–x [k + h]tan (kp + ph) – tan (p k)
= ​ lim   ​  ​
x Æ 1+ = ​ lim 
  +  ​ ​ ​ __________________________
   
    ​  ​
h Æ 0 h

( 
= – 1
Also, L.H.D = ​ lim 
  + 
h Æ 0
[k + h]tan (ph) – 0
​ ​ ​ ________________
h
     ​   ​ )
( 

= f ¢(1 )
= ​ lim 
  + 
k tan (ph)
​ ​ ​ ________
  ​  

​ )
= ​ ______

– ​ 
​ ​  ( 
f (x) – f (1)
lim _________
 ​  

​ ) h Æ 0 h

( 
x Æ 1 x–1
= ​ lim 
   
p k tan (ph)
​ ​ ​ _________
     
​  ​)
(  ) ph
x h Æ 0 +
3 –3
lim ​ ​ ​ ______ 
= ​    ​  ​
x Æ 1– x–1
= pk.
4.38  Differential Calculus Booster

43. We have f (x) = [x]sin (px) Ï Ê ˆ


Now, L.H.D Ô Á 1 ˜
Ô lim+ Á 1˜
= f ¢(k–) Ô x Æ0 ÁË 1 + e x ˜¯
Ô
= ​  
lim  
x Æ k – ( 
f (x) – f (k)
​ ​_________
​ 
x–k
  
  ​  
​ ) =Ì
Ô
Ô lim
Ê –1 ˆ
Á e x ˜
= ​ lim 
  – 
h Æ 0 ( f (k – h) – f (k)
​ ​ ​ ____________  
  
(k – h)/k
  ​  ​ ) Ô x Æ0 +
ÔÓ
Á –1 ˜
ÁË e x + 1˜¯

= ​ lim 
  – 
h Æ 0 ( 
f (k – h) – f (k)
​  ​ ​ ____________
– h
    ​  ​ ) ÏÊ 1 ˆ
ÔÁ •˜
ÔË 1 + e ¯
= ​ lim 
  – 
h Æ 0 ( 
[k – h] sin (kp – ph) – sin (pk)
​ ​ ​ __________________________
   
– h
    ​  ​ ) =Ì
ÔÊ e
–• ˆ

( 
ÔÁË e – • + 1˜¯
= ​ lim 
  – 
h Æ 0
(k – 1)(–1) k–1sin (ph)  –  0
​ ​ ​  ____________________
   
– h
 ​     ​ ) Ó
Ï1

= ​ lim 
  – 
h Æ 0
( 
p(k – 1)(–1) ksin (ph)
​ ​ ​ _________________
ph
       ​  ​ )
Ô•
Ô

ÔÊ 0 ˆ
Á
ÔË ˜
= (–1) k (k – 1)p Ó 0 + 1¯

44. We have f ¢(0) Ï0



= ​  
x Æ 0 ( 
f (x) – f (0)
lim ​ ​ __________
​ 
x–0
   ​  
​ )
Ó0
=0

(  1
x sin ​ __
2

lim ​ ​ ​ ___________
= ​  
x Æ 0 x–0
(  )
​ x ​  ​ – 0
  
 ​  ​ ) Thus,
46. Given
f (x) is differentiable at x = 0.
f (x) is differentiable at x = 1,

= ​  
x Æ 0
( 
________
lim ​ ​ ​  x   
1
x2sin ​ __ (  )
​ x ​  ​

​  ​ ) so, f ¢(1+) = f (1–)

( 
f (x) – f (1)
  +​ ​__________
fi ​ lim  ​     ​  ​ = ​ lim  ​  )
f (x) – f (1)
  –1​ ​ _________  ​  

​ (  )
(  (  ))1
lim ​ ​ x sin ​ ​ __
= ​  
x Æ 0 x ​ ​  ​
x Æ 1 x – 1 x Æ 1 x–1

=0
Thus, f(x) is differentiable at x = 0.
fi ​ lim 
x Æ 1
​ ( 
x2 + ax + b – a – 1
  +​ ​ __________________
x–1
 ​ 
      ​ )
45. We have f ¢(0)
= ​   ( 
f (x) – f (0)
lim ​ ​ __________
​    
 ​  ​ )
= ​ lim 
x Æ 1
( 
ax2 + 1 – a – 1
  –1​ ​ ​ ______________
x–1
 ​ 
     ​ )
x Æ 0 x–0

(  )
(  )
x (x2 – 1) + a (x – 1) + b
______
​    1   ​ – 0 fi   +​ ​​ ____________________
​ lim       ​     ​
__
​ x ​ x Æ 1 x–1
1 + ​e​ ​
lim ​ ​ ​  _________
= ​    ​  
  ​
x Æ 0 x–0
( 
a (x2 – 1)
)
(  )
  –1​ ​ ​ ________
= ​ lim   ​  


1 x–1
lim ​ ​ _____
= ​   ​    1   ​  ​ x Æ 1
x Æ 0 __

( 
​ x ​

Ï
1 + e​ ​

Ê ˆ

x Æ 1+ x–1
b
  ​ ​ a + (x + 1)+ ​ _____
​ lim    )
 ​  ​ = ​ lim 
  –1​( a(x + 1))
x Æ 1
Ô
  ​ ​(a  + (x + 1) + ​ _____
Á 1 ˜
x – 1)
Ô lim+ b
Á 1˜ fi ​ lim     ​  ​ = 2a ...(i)
Ô x Æ0 ÁË 1 + e x ˜¯ x Æ 1+
Ô

Ê ˆ As we know that, every differentiable function is
Ô
Ô lim Á 1 ˜ continuous.
Ô x Æ0 + Á 1˜
ÔÓ ÁË 1 + e x ˜¯ So, f (x) is continuous at x = 1.
The Continuity and Differentiability  4.39

fi ​ lim ​ f (x) = f (1)

x Æ 1 fi ​   
lim  
x Æ 1/2 – (  (  ) ) 1
​ ​ b ​ x + __
​   ​   ​  ​ = –1
2
fi ​ lim 
  +​ f (x) = ​  
lim ​ f (x) = f (1)
x Æ 1

x Æ 1 b = –1

fi   +​ (x2 + ax + b) = ​  
​ lim  lim 

​ (ax2 + 1) = a + 1 From (ii), we get,
x Æ 1 x Æ 1
1 9
fi (1 + a + b) = (a + 1) = a + 1 fi a = 2 + __
​   ​  = __
​   ​ 
4 4
fi b=0 Hence, the values of a and b are a = 9/4 and b = –1.
Put b = 0 in (i), we get, 48. To check continuity:
fi ​ lim 
  +​ (a + (x + 1)) = 2a ​ lim 
  ​ f (x) = f (0) = 0
x Æ 1 x Æ 0

fi (a + (1 + 1)) = 2a Now, ​ lim 


  ​ f (x)
x Æ 0
fi a = 2.
Hence, the values of a and b are 2 and 0.
= ​  
x Æ 0
(  (  ) )1
lim ​ ​ x sin ​ __
​ x ​  ​  ​

1 =0
47. Since f (x) is differentiable at x = __ ​   ​ ,
2 Thus, f (x) is continuous at x = 0.
so,
1+
__
(  ) (  ) 1–
__
f ¢ ​ ​   ​   ​ = f ¢​ ​   ​   ​ To check differentiability

(  ) (  )
2 2

(  )
f (x) – f ​ __
1
​   ​   ​
2 (  )
f (x) – f ​ __
1
​   ​   ​
2
x Æ 0 ( 
f (x) – f (0)
lim ​ ​ __________
f ¢(0) = ​   ​ 
x–0
    ​  
​ )
(  )
​ ​ ​ _________ ​ ​ ​ _________
fi ​   
lim  
x Æ 1/2 + 1
x – __
  
 ​  
​   ​ 
​ = ​   
lim – 
x Æ 1/2 1
x – __
  
 ​  
​   ​ 
​ 1
x sin ​ __ (  )
​ x ​  ​ – 0
__________

(  ) (  )
2 2 = ​  
lim ​ ​ ​     ​  ​
x Æ 0 x–0
__1
(  (  ) )
​    ​  – 2
|x| (a + bx2) – 2 1
lim ​ ​ sin ​ __
fi ​   
lim + ​ ​ ​ ______   ​  ​ = ​    lim – ​ ​ ​  __________
  
 ​   ​ = ​  
x Æ 0
​ x ​  ​  ​
1 1
x – __ x – __
x Æ 1/2 x Æ 1/2
​   ​  ​   ​ 

( 
2 2 = Oscillating a finite value between

)
–1 to 1.
(a + bx2) – 2
fi ​   
lim – ​ ​ ​  __________   
 ​   ​ = –1 ...(i) Thus, f (x) is not differentiable at x = 0.
1
x – __
x Æ 1/2
​   ​ 
2 49. To check the differentiability
As we know that every differentiable function is
continuous, so it is continuous at x = 1/2 f ¢(0 +) = ​ lim 
  + 
x Æ 0
​  ( 
f (x) – f (0)
​ ​ __________
x–0
   ​  
​ )
(  )
(  )
1
fi ​   
lim   ​ f (x) = f  ​ __
​    ​  ​ x​e​ ​x ​​
1
__
x Æ 1/2 2 ______
​     ​ 
–0
1

(  )
__
1 + ​e​ ​x ​​
1_________
fi ​  lim ​  
   f(x) = f ​ __
​  lim –​ 
f (x) =    ​   ​   ​ = ​ lim 
    ​ ​ ​ 
x–0
 ​ 

  ​
2 x Æ 0 +

(  )
x Æ 1/2+ x Æ 1/2

(  )
1
__
1 ​   ​
fi ​   
lim   ​ ​ __
​    ​   ​ = ​   
lim   ​ (a + bx2) = 2 ​e​x ​
x Æ 1/2 + |x| x Æ 1/2 – = ​ lim 
  +  ​ ​ ______
​   1   ​  ​
x Æ 0 __
​ x ​
1 + e​ ​ ​

(  )
(  )
b
fi ​ a + __
​    ​  ​ = 2 ...(ii) 1
4 = ​ lim 
    ​ ​ ______
​    1   ​  ​

(  )
x Æ 0 + – ​ __ ​ 
From (i) and (ii), we get, 1 + e​ ​ x ​

(  )
b
2  – __​    ​ + bx2 – 2 ​ 
1
= ​ _______
    ​ ​
4
_____________ 1 + e– •
​    lim   ​ ​ ​    
 ​   ​ = –1
x Æ 1/2 – 1
x – __
= ​( _____

 ( ((   )) )
​   ​ 
1 + 0)
2 1
​      ​  ​ = 1
2 __ 1
b​ x – ​   ​   ​

(  )
4
fi ​   
lim –  ​ ​ ​ ________ ​   ​ = –1 f (x) – f (0)
x Æ 1/2 1
__ Also, f ¢(0 –) = ​  
lim   ​ ​ __________
​     ​  

​ x – ​   ​   ​ x Æ 0 – x–0
2
4.40  Differential Calculus Booster

(  ) (  )
1
__ [2(1 + h) – 3](1 + h) + 1
x​e​ ​x ​​ lim ​  ​ ​ _____________________
= ​        ​ 
   ​
​ ______  1
 ​ 
–0 h Æ 0 1+h–1
__
​ x ​
1 + ​e​ ​
​ ​ ​ _________
( 
= ​  
lim     
 ​  

x Æ 0 – x– 0 [2h – 1](1 + h) + 1
lim ​ ​ ​  ________________ )
(  )
= ​          ​  ​
1
__
h Æ 0 h
​ x ​
​e​ ​
= ​  
lim  
x Æ 0 –
​ ​ ______
​   1 
__
 ​  ​
​ x ​
1 + ​e​ ​
= ​  
h Æ 0 ( 
– (1 + h) + 1
lim ​ ​ ​  ___________
h
     ​  
​ )
( 
= –1
​ 
e– •
= ​ _______  
​  ​ )
( 
1 + e– •
=0
Also, ​ lim 
x Æ 1
f (x) – f (1)
  –​ ​ _________
​ 
x–1
 ​  
 ​ )
So, f(x) is not differentiable at x = 0
Since, both one sided derivative exists (though they = ​  
h Æ 0 ( 
f (1 – h) – f (1)
lim ​ ​ ____________
​ 
1–h–1
  
    ​ ​ )
are unequal), so f (x) is continuous at x = 0.

(  (  )
p
50. To check the differentiability
________________ 2 )
sin ​ (1 – h) ​ __ ​   ​ + 1

(  )
f (x) – f(1) lim ​ ​ ​ 
= ​          ​  ​
  +​ ​_________
f ¢(1+) = ​ lim  ​     ​  
​ h Æ 0 – h

(  (  ) )
x Æ 1 x–1 p
cos ​ __
​   ​   ​ h + 1
= ​  
h Æ 0
​  ( 
f (1+ h) – f (1)
lim ​ ​____________
1+h–1
  
    ​  ​ ) = ​  
h Æ 0
​ 
2
lim ​ ​ ____________
– h
     ​  

(  )
f (1 + h) – f (1) = – •
lim ​ ​ _____________
= ​   ​         ​  ​ Thus, f (x) is not differentiable at x = 1.
h Æ 0 h

( 
52.

h Æ 0
|h| ([1 + h] – {1 + h}) – 0
lim ​ ​ ​ ______________________
= ​      
h
    ​  ​ )
= ​  
h Æ 0 ( 
h (1 – h)
lim ​ ​ ​ ________
h
   ​  ​ )
=1
Also, f ¢(1–) = ​  
lim 
x Æ 1–
​  ( 
f (x) – f (1)
​ ​__________
x–1
   ​  
​ )

h Æ 0
​  ( 
f (1 – h) – f (1)
lim ​ ​ _____________
= ​  
1–h–1
  
    ​  ​ ) Clearly f (x) is not differentiable at x = 0
So f (x) is not differentiable in [– 2, 2]

= ​  
h Æ 0 ( 
f (1 – h) – 0
lim ​ ​ ​ ___________
– h
     
​  ​ ) 53.


h Æ 0
​  ( 
|– h|([1 + h] – {1 + h})
lim ​ ​ ____________________
= ​      
– h
     ​  ​ )

lim – h(0
= ​ _____  ​ 
h Æ 0 (  – (1 + h))
​ ​  _____________
– h
     ​  
​ )
= –1

Thus, f (x) is not differentiable at x = 0. Clearly, f (x) is not differentiable at x = –1, 0, 1.


Since, both one sided derivative exists (though they Thus, f(x) is not differentiable in; – 4, 4]
are unequal), so f (x) is continuous at x = 0. 54. We have f (x) = sin x + sin |x|
51. To check the differentiability.
Ïsin x + sin x : x ≥ 0
+
f ¢(1 ) = ​ lim 
x Æ 1
​ 
x–1 ( 
f (x) – f (1)
  +​ ​ _________  ​  

​ ) f (x) = Ì
Ósin x – sin x : x < 0
Ï2sin x : x ≥ 0
= ​ lim 
x Æ 1
( 
[2x – 3]x – (–1)
  +​ ​ ​ ______________
x–1
 ​ 
     ​ ) =Ì
Ó0 : x<0
The Continuity and Differentiability  4.41

So f (x) is not differentiable in R.


59. We have f (x) = |x2 – 1| + |x2 – 4|

Clearly f (x) is not differentiable at x = 0.


55.
Clearly, f (x) is not differentiable at x = – 2,
–1, 1, 2.
Thus, f (x) is not differentiable in R.
60. We have f (g (x))
= f (x (1 – x2))
Ï1 : g( x ) > 0
Ô
= Ì0 : g( x ) = 0
Clearly, f (x) is not differentiable at x = –1, 0, 1. Ô –1 : g( x ) < 0
Thus, f (x) is not differentiable in; – 4, 4] Ó
x
56. We have, f (x) = _____
​       ​ Ï1 : x Œ (– •, – 1) » (0, 1)
1 + |x| Ô
= Ì0 : x = –1, 0, 1
Ô –1 : (–1, 0) » (1, •)
Ó

Clearly, f (x) is differentiable everywhere.


57.
Clearly, f (x) is not differentiable at x = – 1, 0, 1.
61. We have f (x) = sin–1|sin x|

Clearly f (x) is not differentiable at x = 0.


58. We have, f (x) = |x| + |x2 – 1|

p
Clearl, f (x) is not differentiable at x = (2 n + 1) ​ __ ​ 
2
Thus, f (x) is non-differentiable in R.

62. We have ​  


h Æ 0 ( 
f (2 + h) – f (2 – h)
lim ​ ​ ​ ________________
h
       )
​  ​
Thus, f (x) is not differentiable at x = – 1, 0, 1
4.42  Differential Calculus Booster



= f ¢(2)
= 5. x Æ 0
( 
f ¢(x) – f ¢(0)
lim ​​ ​ __________
Now f ≤(0) = ​  
x–0
  
 ​  ​ )
63. We have ​  
h Æ 0
​ 
( 
f (2h + 2 + h2) – f (2)
lim ​ ​ __________________
  
   ​  ​
f (h – h2 + 1) – f (1) ) = ​   (  (  )
1
2x sin ​ __
________________
lim ​ ​ ​ 
x Æ 0 x     ​  
(  )
1
​ x ​  ​– cos ​ __
)
​ x ​  ​

[ (  )
f (2h + 2 + h2) – f (2) Thus, the limit does not exist at x = 0.
lim ​ ​ ​ ​  __________________
= ​     
 ​ 
   ​ ​ Clearly, y = f (x) is not twice differentiable at
h Æ 0 2h + h2 ​ x = 0.

(  ) (  )]
ÏÔ xe x : x£0
h – h2
________________ 2h + h2
_______ 66. We have y = f (x) = Ì
× ​​        ​  ​ ×  ​​ ​   ​  ​  ​ 2 3
f (h – h2 + 1) – f (1) ​ h–h
2 ÔÓ x + x - x : x>0

= ​  
h Æ 0 (  ( 
h (2 + h)
lim ​ ​ f ¢(2) × ​ ​ _______ 
h (1 – h) )
1
 ​  ​ × ____
​      ​  ​
f ¢(x)  
) fi
ÏÔ xe x + e x
f ¢(x) = Ì
2
: x<0
ÔÓ1 + 2 x - 3 x : x > 0
1
= 6 × 2 × __​   ​  Clearly, f ¢(0 +) = 1 = f ¢(0 –)
4
= 3. Therefore, f (x) is differentiable at x = 0.
64. We have f (x) = |x3|
ÏÔ x 3 ÔÏ xe x + 2e x : x<0
: x≥0 Also, f ≤(x) = Ì
= Ì ÔÓ2 - 6 x : x>0
3
ÔÓ- x : x < 0
Clearly, f ≤(0 +) = 2 = f ≤(0 –)
ÏÔ3 x 2 : x≥0
fi f ¢(x) = Ì Thus, f (x) is twice differentiable at x = 0.
2
ÔÓ- 3 x : x < 0


Ï6 x
f ≤(x) = Ì
: x≥0
h Æ 0 ( 
f (x + h) – f (x)
lim ​​ ​ _____________
67. We have f ¢(x) = ​  
h
       ​  ​ )
Ó- 6 x : x < 0
Clearly, f ≤(0 +) = 0 = f ≤(0 –)
h Æ 0 ( 
f (x) ◊ f (h)– f (x)
lim ​​ _____________
= ​   ​ 
h
     ​  ​ )
( 
Thus, f (x) is twice differentiable at other values
of x.
Ï 2 Ê 1ˆ
= ​  
h Æ 0
f (x) ( f (h) – 1)
lim ​​ ​ _____________
h
       ​  ​)
( 
Ô x sin Á ˜ : x π 0
65. We have y = Ì
Ô0
Ó
Ë x¯
: x=0

h Æ 0 ( 
(f (h) – 1)
lim ​​ f (x)  × ​ _________
= ​   ​ 
h
   
​  
​  ​ )) ...(i)

h Æ 0 x–0 ( 
f (x) – f (0)
lim ​ ​ ​ _________
Here, f ¢(0) = ​    ​  

​ ) Also, f (x + y)= f (x) ◊ f (y), " x Œ R
Put x = 0 = y, then we get,

= ​   ( 
________
lim ​​ ​  x   
x Æ 0
1
x2sin ​ __ (  )
​ x ​  ​
​  ​)

f (0)= f (0) ◊ f (0)
f (0) = f 2 (0)

= ​  
x Æ 0
(  (  ) )
1
lim ​​ x sin ​ __
​ x ​  ​  ​


f (0)( f (0) – 1) = 0
f (0) = 1, ( f (0) π 0)
=0 From (i), we get,
Thus, y = f (x) is differentiable at x = 0 f ¢(x) =  
h Æ 0 (  (  ))(f (h) – f (0))
​ lim ​​ f (x) × ​ ​ __________
h
     ​  
​  ​

Ï
( ( 
Ê 1ˆ Ê 1ˆ
Ô2 x sin Á ˜ - cos Á ˜ : x π 0
y¢= Ì
Ô0
Ë x¯ Ë x¯
: x=0

f ¢(x)
​ ____ ​ = ​  
f (x) h Æ 0 ))
( f (h) – f (0))
lim ​​ ​ ​ ___________
h
     ​  
​  ​
Ó
f ¢(x)
fi ​ ____ ​ = f ¢(0) = k
f (x)
The Continuity and Differentiability  4.43

On integration, we get, Thus, f (x) = – x + 1


log |f (x)| = kx + c
put x = 0, log |f (0)| = log |1| = 0, so c = 0.
x+y
70. We have f ​ ​ _____
3 (  ) 2 + f (x) + f (y)
 ​ =​  ____________
 ​  
3
 ​
    

f ¢​( ​ _____
3 ) 5 5
fi log |f (x)| = kx x + y __ 2 2
fi   ​ ◊ ​   ​  = __
 ​   ​   ​  f ¢(x), y as a constant
fi f (x) = e kx.

f ¢​( ​ _____
3 )
Hence, the required function is f (x) = ekx. x+y
fi   ​ = f ¢(x)
 ​  
68. Given f (x + y) = f (x) + f (y)
Put x = 0 = y, f (0) = 0 Replacing x by 0 and y by 3x, we get,
f ¢(x) = f ¢(0) = k(say)
Now, f ¢(x) = ​  
h Æ 0(  )
f (x + h) – f (x)
lim ​​ ​ ____________
h
     ​  
​ when x = 2, then k = f ¢(2) = 2

= ​   ( 
h Æ 0 )
f (x) + f (h) – f (x)
lim ​​ ​ ______________
h
       ​  ​
Thus, f ¢(x) = 2
On integration, we get,
= ​   (  )
h Æ 0 h
f (h)
lim ​​ ____
​      ​  ​ f (x) = 2x + c

( 
when x = 0, then c = f (0) = 2
= ​  
h Æ 0 )
f (h) – f (0)
lim ​​ ​ _________
h
   
​  
​ Hence, the given function is f (x) = 2x + 2.

= f ¢(0) = k
On integration, we get, f (x) = kx + c
( 
2x + 3y
71. We have f ​ ​ _______
5 )
2f (x) + 3f (y)
 ​ =​  ____________
 ​  
5
 ​
    

If x = 0, f (0) = 0, then c = 0.
Thus, f (x) = kx
( 
2x + 3y __
fi f ¢​ ​ _______
5
2 2
)
 ​ ◊ ​   ​  = __
 ​  
5 5
​    ​ ◊ f ¢(x), y as a constant.

(  )
x+y
69. We have f ​ ​ _____
2
 ​  
f (x) + f (y)
 ​ =​  _________
2
 ​
    
( 
 2x + 3y
fi f ¢​ ​ _______
5
 ​ = f ¢(x)
 ​   )
2(  )
x + y __
fi f ¢​ ​ _____
 ​  
1 1
 ​ ◊ ​   ​  = __
2 2
​   ​  f ¢(x), y as a constant
5
Replacing x by 0 and y by ​ __ ​  x, we get,
3

(  )
x+y f ¢(x) = f ¢(0) = –1
fi f ¢​ ​ _____  ​  
 ​ = f ¢(x)
2 On integration, we get,
Replacing x by 0 and y by 2x, we get, f(x) = – x + c
f ¢(x) = f ¢(0) = – 1 when x = 0, then c = f (0) = 1
On integration, we get, Hence, the function is, f (x) = – x + 1
f(x) = – x + c 72. If f (x + y + z) = f (x) ◊ f (y) ◊ f (z) for all x, y, z in R such
When x = 0, c = 1. that f (2) = 4, f ¢(0) = 3, then find f ¢(2). Ans. 12

Hints and Solutions


p 5p
Thus f (x) is discontinuous at x = __
​   ​ or ___
​   ​ .
(Problems for JEE-Advanced) 6 6

( 
3
x
1. We have f (x) = ​  
n Æ •
sin x
​ ​ ___________
lim ​        ​ ​
1 + (2sin x)2n ) 2. We have f (x) = ​ ______  
4 + |x|
 ​

Here f (x) is defined for all real values of x.


Ï 5p 5p
Ôsin x : 0 < x < 6 or 6 < x < p Thus, f (x) is continuous for all real x.
Ô 3. We have f (0) = 0.
Ô1 p 5p
= Ì : x = or
Ô 4 6 6
Now, ​  
lim ​  
f (x)
Ô p 5p x Æ 0
Ô0 : <x<
Ó 6 6
4.44  Differential Calculus Booster

​   (  x
lim ​ ​ _____
​     
x Æ 0 x + 1
​ 
x
 ​+ _____________ ​ 
x
     ​+ _____________
     ​ + ...  ​
(x + 1) (2x + 1) (2x + 1) (3x + 1) ) = ​  lim  
x Æ 0 – (  a – ax sinx + b cos x + 5
​ ​ ​  ____________________
   
x2
    ​  ​
)
​  
x Æ 0 [ (  1
lim ​  ​ ​ 1 – _____
​     
x+1 ​ ) (  1
 ​ ​ ​ + ​ ______
​     
1
 ​ – _______
​ 
(x + 1) (2x + 1)
    ​ ​ ) (since it has a finite limit, so a + b + 5 = 0)

( 0 – a sin x – ax cos x – b sin x


​ ​ ​  ________________________ )
( 
= ​  
lim      
    ​  ​
​ 
1
+ ​ _______
    ​ 
1
 ​ – _______
(2x + 1) (3x + 1)
    ​ ​ ) x Æ 0 – 2x

​ ​( ​  ________________________________   )​
( 
0 – a cos x – a cos x + ax sin x – b cos x


​ 
((n –
1
+  ​​ ___________
1)x + 1)
1
     ​ – ______
​     
nx +1
 ​ ​  ​ )] = ​  
lim  
x Æ 0 –
    
2
 ​ 

– 2a – b
= ​  
x Æ 0 (  1
lim ​  ​ 1 – ______
​     
nx + 1
 ​ ​ ) =  ________
​ 
2
 ​  

Also, a + b + 5 = 0
= 1
– 2a – b
Thus, f (x) is discontinuous at x = 0. and ​ _______
 ​  = 3
2
4. We have
On solving we get, a = –, b = – 4, c = 0 and
Ï Ê Ê 1 1ˆˆ d = loge 3.
Ô x exp Á - Á + ˜ ˜ : x π 0
[  ] [  ]
f (x) =  Ì Ë Ë | x| x ¯ ¯ 1 1
Ô 6. We have, f (x) = ​ x – ​ __ ​   ​ + [x] + ​ x + __
​   ​   ​
Ó0 : x=0 3 3

Ï - Ê 1 1ˆ Ï 2
Ô xe ÁË x + x ˜¯ : x>0 Ô- 2 - 1 - 1 = - 4 : -1 £ x < -
3
Ô Ô
2 1
Ô Ê 1 1ˆ =  ÔÔ- 1 - 1 - 1 = - 3 : - £x < -
fi f (x) =  Ì xe - Á - + ˜ : x < 0 Ì 3 3
Ô Ë x x¯
Ô 1
Ô0 : x=0 Ô- 1 - 1 + 0 = - 2 : - £x <0
Ô Ô 3
Ó ÔÓ- 1 + 0 + 0 = - 1 : x=0
Ï -2 Thus, f (x) is discontinuous at
Ô xe x : x>0
Ô 2 1
fi f (x) =  Ì x : x<0 x = –1, –  ​ __ ​ , – ​ __ ​ , 0
Ô0 : x=0 3 3
Ô
Ó 7. As we know that [x] is discontinuous at all integral
values of x.
​  
lim  ​ f (x) = ​  
lim  ​  f (x) = f (0) = 0
x Æ 0+ –
x Æ 0 Therefore the function f (x) will be continuous in

Thus, f (x) is continuous at x = 0.


(x – 2)3
(4, 6) only when ​ ​ _______
a    [ 
​  ​ = 0 ]
5. Now, ​  
lim 
+

​  f (x) fi a ≥ 64
x Æ 0

(  ( 

))
1/x aŒ[64, •)
cx + dx3
= ​  lim  ​ 1 + ​ ​ ________
      
 ​ ​  ​ ​
x2
+
x Æ 0
8. We have f (x) = min{1, cos x, 1 – sin x}

= ​  
lim ​(1 +dx)1/x (since c = 0 for a finite limit)
  Ï p
+
x Æ 0
Ôcos x : - £x£0
2
Ô
= ed Ô p
= Ì1 - sin x : 0 < x £
2

( 
Ô
Also, ​  lim 
x Æ 0 –

a(1 – x sin x) + b cos x + 5
​ ​ ​ ______________________
   
x2
    ​  ​ ) Ô
Ôcos x
Ó
p
: < x£p
2
The Continuity and Differentiability  4.45

ÔÏ x
2
: x2 - x £ 0
fi f (x) = Ì
2 2
ÔÓ2 x - x : x - x > 0

ÏÔ x 2 : 0 £ x £1
fi f (x) = Ì
2
ÔÓ x - x x Œ -• •

p
Thus, f (x) is continuous at x = 0, __
​   ​ 
2
_______
{    – x2, ​ 
9. We have f (x) = max ​ sng (x), – ​÷9  }
x3  ​

Clearly f (x) is continuous for all real x.


1
13. We have f (x) = [x] + ​ x + __ [  ] [  ] 2
​    ​  ​ + ​ x + __
3
​   ​   ​
3
Clearly, f (x) is discontinuous at x = 0. Ï 2
Ô- 1 - 1 - 1 = - 3 : - 1 £ x < - 3
ÏÔ x 3 : x 2 < 1 Ô
10. We have f (x) =  Ì Ô- 1 - 1 = - 2 2 1
: - £x<-
ÔÓ x : x > 1 Ô 3 3
Ô
1
Ô- 1 + 0 + 0 = - 1 : - £ x < 0
Ô 3
Ô
=  Ì 1
Ô0 + 0 + 0 = 0 : 0£ x<
3
Ô
Ô0 + 0 + 1 = 1 1 2
: £x<
Ô 3 3
Ô 2
Ô0 + 1 + 1 = 2 : £ x <1
Ô 3
ÔÓ1 + 1 + 1 = 3 : x =1
Thus the number of points of discontinuity is 7.
Clearly f (x) is continuous at all real x. 14. Since f (x) is continuous at x = a, so f (a) = 0

x Æ a ( 
ln(l + 6f 2 (x) – 3f (x))
lim ​ ​ ​ __________________
Now, ​  
3f(x)
 ​ 
      ​ )
11.
= ​  
x Æ a ( 
log(l + 6f 2 (x) – 3f (x)) ____________
lim ​ ​ ​ ___________________
   
   
6f 2 (x)  –  3f (x) – 3f (x)
 ​ × ​ 
3f (x) )
6f 2 (x) – 3f (x)
 ​ 
     ​

= ​  
lim  
x Æ a ( 
6f 2 (x) – 3f (x)
​ ​ ​ ____________
3f (x)
 ​ )
    ​

= ​  
lim  
​ (2f(x) – 1)
x Æ a
Clearly f (x) is continuous at x = 1.
= (2f(a) – 1)
12. We have f (x) = x – |x – x2|
=0–1
ÏÔ x 2 : x - x2 ≥ 0
fi f (x) = Ì = – 1.
2 2
ÔÓ2 x - x : x - x < 0
4.46  Differential Calculus Booster

15. Let f (x) = g(x) + h(x) Y

where g(x) = [4 x] and h(x) = {3x}


Now, g(x) = [4 x] is discontinuous at

1
[  ) [  ) [  ) [  ) [  )
1 1
x Œ ​ 0,  ​ ​__  ​  ​ ​, ​ __
1 3
​   ​  ​,  ​ ​__ ​   ​, ​ __
3
​   ​  ​,  ​ ​__ ​   ​,  ​​ __
5
​   ​  ​,  1  ​, ​ 1, ​ ​ __ ​   ​ ​... 20. X ¢
O x=1
X

[  )
​4 ​ 4 ​ ​2 2 ​ ​4 ​ 4 ​ ​4 ​
21
​ 5,  ​ ​___ ​   ​ ​
​4 ​
There are 25 points, where g(x) is discontinuous

Also, h(x) = {3x} is discontinuous at x = 1, 2, 3,
4, 5 Clearly f (x) is continuous at x = 0, 1.
Thus, the number discontinuous points is 5. 21. We have f (x) = [tan–1 x]
Therfore, the number of discontinuous points of f (x)
is 30.

16.

22. The function f (x) is undefined only when


Clearly, the function f (x) is continuous everywhere. [x + 1] = 0
fi 0£x+1<1
fi –1 £ x < 0
Therefore, Df = R – [– 1, 0)
17. Thus, the function f (x) is continuous
at x Œ Df = R – [– 1, 0).
23. We have
f (x) = |x + 1|(|x| – |x – 1|)
Thus the number of points of discontinuity is 2 at Ï( x + 1) (2 x - 1) : -2 £ x < -1
x = – 1, 1. Ô- ( x + 1) (2 x - 1) : -1 £ x < 0
Ô
18. We have f (x) = [x] + {2x} + [3x] = Ì
Thus, the number of points of discontinuous is 4. Ô( x + 1) : 0 £ x <1
ÔÓ( x + 1) (2 x - 1) : -1 £ x < 0
Clearly, f is continuous everywhere but not
differentiable at x = – 1, 0, 1

19.

24.

Clearly, f (x) is continuous for all real x.

Clearly, f is discontinuous at x = 2.
The Continuity and Differentiability  4.47

25. Case-I : When x ≥ 0


Ï p
ÔÔ0 : x = 2 np + 2
= Ì
Ô1 : x π 2 np + p
ÔÓ 2

Case-II : When x < 0.

Thus the function has discontinuous


p
at x = 2np + __ ​   ​ , n Œ I
2
28.
(i) We have f (x) = [loge x]
It is discontinuous, where loge x is an integer.
i.e. loge x = k
i.e. x = ek, k Œ I
Thus, Thus, the number of points of discontinuity is
infinite at x = ..., e– 3, e– 2, e– 1, 1, e, e2, e3,...
(ii) We have f (x) = Î sin–1 x˚
It is discontinuous, where sin–1 x is an integer.
i.e. sin–1 x = – 1, 0, 1
i.e. x = – sin 1, 0, sin 1
Thus, the number of points of discontinuity is
3 at x = – sin 1, 0, sin 1

(iii) We have f (x) = ​ _____


[  ]
2
​  2    
x +1
 ​  ​
2
Thus, f (x) is discontinuous at x = – 1, 1. It is discontinuous, where ​ _____ 2
  
 ​ is an integer.
x +1
2
i.e. ​ _____     ​ = 1, 2
x2 + 1
i.e. x = 0, 1.
Thus, the number of points of discontinuity is
26. 2 at x = 0, 1.
x
29. We have f (x) = ​ __ [  ] [​  __​ 3x  ​ ]​ + [​  __​ 4x  ​ ]​
​    ​  ​ +
2
It is discontinuous, where f (x) is an integer.

Thus, the number of points of discontinuity is infinite Ï1 : 0£ x<2


at x Œ I. Ô1 : 2£ x<3
Ô
Thus, f (x) = Ì
27. We have f (x) = sgn (cos 2x – 2 sinx + 3) Ô2 : 3£ x< 4
= sgn(1 – 2 sin2 x – 2 sin x + 3) ÔÓ3 : x=2
= sgn(2 – 2 sin2 x – 2 sin x) Therefore, the number of points of discontinuity is
= sgn{2(1 – sin x)(2 + sin x)} infinite.
4.48  Differential Calculus Booster

Ê 2sin 2 ( x/2) ˆ
lim- ÁÁ 1 - ˜
(Tougher Problems for JEE-Advanced) xÆ0 Ë 2sin( x/2)cos( x/2) ˜¯
= e
Ê Ê xˆˆ
1. Since f is continuous at x = 0, so lim ÁË 1 - tan ÁË ˜¯ ˜¯
xÆ0
- 2
​ lim 
  ​ f (x) = f (0) = c = e
x Æ 0
=e

x Æ 0 ( 
1 + a cos 2x + b cos 4x
lim ​ ​ ​ ___________________
c = ​      
x2 sin2 x
     ​  ​
) Now, ​  
lim    ( 
e1/x + e2/x + e3/|x|
​ ​​  _______________
  
    ​  ​ )
ae2/x + be3/|x|

x Æ 0
Since it has a finite limit, so a + b + 1 = 0 ...(i)

( 
– a – b + a cos 2x + b cos 4x
lim ​ ​ ​ ________________________
Thus, c = ​  
x Æ 0
   
x4
 ​     ​
) = ​  
lim  
x Æ 0 – ( 
e1/x + e2/x + e3/x
​ ​​ ______________
  
  
ae2/x + be3/x
  ​  ​ )
( 
– a (1 – cos 2x) – b (1 – cos 4x)
lim ​ ​ ​ _________________________
= ​  
x Æ 0
   
x4
 ​     ​ ) = ​ lim 
  + 
x Æ 0 ( 
e3/x (e–2/x + e–1/x + 1)
​ ​ ​ __________________
    
e3/x (ae–1/x + b)
 ​ ​
)
(  (  ) (  ))
1
= __
​   ​
2sin2 x 2sin2 (2x) b
–  a ​ ​ ______
 ​ 

  ​ – b​ ________
​   ​   ​
x2
______________________ x2 Since f (x) is continuous at x = 0, so
= ​  
lim ​ ​ ​      ​     ​
x Æ 0 x2 ​ lim 
  + 
​  f (x) = = ​  
lim 

 ​  f (x) = f (0)
x Æ 0 x Æ 0

Since it has a finite limit, so (2a + 8b) = 0 1


​ __ ​ = e = a
a = – 4b ...(ii) b
1
On solving (i) and (ii), we get, Thus, a = e, b = __
​ e ​
4 1 Hence, the value of {e(a + b) + 2}
a = – ​ __ ​ ,  b = __
​   ​ 
3 3
{  (  1
= ​ e​ e + __ ) }
( 
​ e ​  ​ + 2  ​

x Æ 0
4(1 – cos 2x) – (1 – cos 4x)
lim ​ ​ ​ _______________________
Thus, ​      
3x4 
 ​     ​ ) = (e2 + 3)

( 
3. Since f (x) is differentiable at x = 0, so it is continuous
= ​  
x Æ 0
8 sin2 x – 8 sin2 x cos2 x
lim ​ ​ ​ __________________
3x4
 ​       ​ ) ​ lim 
  + 
at x = 0.
​  f (x) = ​  
lim   ​  f (x) = f (0)

( 

x Æ 0

)
x Æ 0
2 2
8 sin x(1 – cos x)
lim ​ ​ ​ _______________
= ​  
x Æ 0 3x4
 ​ 
     ​
fi ​ lim 
  + 
x Æ 0
( 
eax/2 – 1
​ ​ ​ _______
x    ​  ​ = ​ lim 
   
x Æ 0 –
) ( 
x+c
​ ​ b sin–1 ​ ​ _____
2 (  1
 ​  ​ = __
 ​   ​   ​ 
2 ))
= ​  
x Æ 0 3 ( 
8 sin2 x sin
lim ​ ​ __
​   ​  × ​ _____
x 2
 ​

  × _____
​ 
x
2

2
x
 ​  
 ​ ) a
fi __
​   ​ = b sin–1 ​ __
2
fi a = 1
2
c
(  )
​    ​  ​ = __
1
​   ​ 
2

8
= ​ __ ​  Also, it is differentiable at x = 0, so f ¢(0 +) = f ¢(0 –)
3
1
Hence, the value of ​ a + b + c + __
3 ( 
​    ​  ​ ) fi ​ lim 
  + 
x Æ 0
​ ​ _________
​ 
x – 0( 
f (x) – f (0)
   ​  
​ = ​ lim 
  – 
x Æ 0 x – )
f (x) – f (0)
​ ​ ​ _________
  
0
 ​  ​​  
lim 
x Æ 0 –
  ( 
​​e​​{ ​ x ​ }​​
2
__
)
( 4 1 __
= ​ – ​ __ ​  + __
3 3 3 3
8 1
​   ​  + ​   ​  + __ )
​   ​   ​ 1
eax/2 – __
________ (  ) (  ( 
​   ​ 
2
x+c
b sin–1​ ​ _____
2
_______________
1
 ​ – __
 ​   ​    ​
2 ) )
fi ​ lim 
  +  ​ ​ ​  x    ​  ​ = ​ lim 
  –  ​ ​ ​  x     ​   ​
x Æ 0 x Æ 0
= 2.
2. We have
  ​ (sin x + cos x) cosec x
​ lim 
x Æ 0 ​ ​ _______
fi ​ lim 
  +  ​  x   
1
ex/2 – __
2
x Æ 0
(  ) (  ( 
​   ​ 
x+c
b sin–1 ​ ​ _____
2
​ ​ ​ _______________
​  ​ = ​ lim 
  –  x      
x Æ 0
1
 ​ – __
 ​   ​    ​
2
​  ​
) )
= ​  
lim  ​ (1 + (sin x + cos x – 1)) cosec x

(  (  ) (  ) )

x Æ 0 x 1 x 2 1
1 + ​ __
​    ​  ​ + __
​   ​ ​​  __
​    ​  ​​ ​ + ... –  ​ __ ​ 
2 2 2 2
 (sin x
​ ​ ​ ______________________
+ cos x – 1)
=

​  
lim  
​e​x Æ 0 –
​​  ______________
sinx
       ​
​ fi ​ lim 
  +     
x    ​  ​
x Æ 0
The Continuity and Differentiability  4.49

(  ) {  (  ) }
1 1 4x3
b ​ ____________
     ​× __ ​   ​  ​​ sin​ ​ ___4 
÷  ( 
__________
= ​ lim 
      ​  ​  ​

= ​ lim 
    ​ ​ ​ 
+c 2 2
x_____
​ 1 – ​​ ​   ​ 
2
  
_________________
 ​     
)
 ​​ ​  ​


xÆ – 2 +
5x
x Æ 0 –

1 b
1 4 × – 8
= sin​ ​ ______ 
5 × 16 ( 
 ​  ​ )
fi __
​   ​  = _________
(  )
​  ______
     ​

÷ 
8 2
c2
__ = – sin​ __
​   ​   ​
2​ 1 – ​   ​    ​ 5
4
1 b
fi __
​   ​  = _______
​  ______
     ​ Also, ​ lim 
     ​ f (x)

÷ 
( 
8 c2

xÆ – 2
__

)
​ 1 – ​   ​   ​ –1
______
4 ​    ​ 
a​e​(x + 2) ​– 1
1 b = ​ lim 
     ​ ​ ​ _________  ​ 
  ​
fi __
​   ​  = _______
​  ______
     ​ –
xÆ – 2 –1 ______
​    ​ 
8 ​÷4  – c2  2 – e​ ​(x + 2) ​

( 
 ​

)
fi 64b2 = 4 – c2 1
______
​     ​ 
a – ​e​(x + 2) ​
Hence, the result.      ​​ _________
= ​ lim  ​  1    
 ​  ​
xÆ – 2 ______
​     ​ 
4. Since f (x) is continuous at x = 0, 2​e​(x + 2) ​ – 1
= – a
so ​ lim 
  ​   f (x) = f (0)
x Æ 0
Since f (x) is continuous at x = 0, so
​ lim 
  + 
​  f (x) = ​  
lim 

 ​   f (x) = f (0)
x Æ 0 x Æ 0 ​   
lim 

 ​ f (x) = ​   
lim 
+
 ​ f (x) = f (0)
xÆ – 2 xÆ – 2
​ lim 
  + 
​  f (x) = ​  
lim  ​  f (x) = a
2
(  )

x Æ 0
– a = – sin​ __
x Æ 0
​   ​   ​ = b
Now, ​ lim 
  + 
​  f (x) 5

(  ) (  )
x Æ 0
2 2
a = sin​ __
​   ​   ​, b = – sin​ __ ​    ​  ​
=0 5 5

= ​ lim 
  + 
x Æ 0 ( 
x(3e1/x + 4)
​ ​ ​ __________
  
2 – e1/x
   
​ ​ ) Hence, the value of (a + b + 2) = 2.
Note: As we know that

( 
x ​( 3 + 4​e​ ​ )​
)
1
– ​ __x ​
= ​ lim 
    ​ ​ ​ __________    ​  
​ Ï0 : 0 < x < 1
x Æ 0 –
1
__ Ô
​ x ​​ – 1
2​e– ​    ​ x2n = Ì1 :
​ lim  x =1 .
n Æ •
=0 Ô• : x >1
Ó
Also, ​ lim 
  – 
​  f (x) x2n – 1
(  )
( 
6. We have f (x) = ​  
lim  ​ ​ ​ ______   ​  ​

)
x Æ 0

– x ​( ​3e​– ​ x ​​+  4 )​
1
__ n Æ  •
x2n + 1
= ​ lim 
    ____________
​ ​  ​    
    ​  ​
​( 2 – e–1/x )​ Ï- 1 : 0 £ x 2 < 1

x Æ 0

( 
ÔÔ
= ​ lim 
  – 
x Æ 0
– x ​( 3 + 4e1/x )​
​ ​ ​ ____________
  
  
(2e1/x – 1)
 ​  ​
) = Ì0 : x 2 = 1
Ô1 : x > 1
ÔÓ
=0 Ï- 1 : -1 < x < 1
Thus, f (x) is continuous at x = 0 Ô
= Ì0 : x = ±1
Therefore, the value of a is 0. Ô1 : x < - 1, x > 1
Ó
Hence, the value of (a2 + a + 10) is 10.
5. Since f (x) is continuous at x = – 2 Thus, the number of discontinuity is 2 at x = –1, 1.
so, ​ lim 
     ​ f (x) = f (– 2) 7. We have f (x) = ​   ​ (cos2n x)
lim  
n Æ •
xÆ – 2

​   
lim 
+

​ f (x) = ​ lim 
     –
​ f (x) = f (– 2) ÏÔ0 : 0 £ cos2 x < 1
xÆ – 2 xÆ – 2
= Ì
ÔÓ1 : cos2 x = 1

{  ( 
Now,
= ​   
lim 
xÆ – 2 +
 ​ f (x) = ​   
lim 
xÆ – 2 +

x4 – 16
​​ sin​ _______
​  5
x + 32
  ​  ​  ​ )}
Ï0 : x π np , n Œ I
= Ì
Ó1 : x = np , n Œ I
4.50  Differential Calculus Booster

Thus, the number of discontinuity is infinite at For continuity at x = – 1


x = np, n Œ I. a – b = – 1 ...(ii)
8. We have f (x) = ​  
lim  
n Æ • (  (  ) )
px 2n
​ ​​ sin​ ​ ___ ​   ​  ​​ ​
2
On solving (i) and (ii), we get, a = 0, b = 1
Thus, the value of 2a + 3b + 10
Ï Ê p xˆ = 0 + 3 + 10 = 13.
Ô0 : 0 £ sin ÁË 2 ˜¯ < 1
Ô 11. Given function is discontinuous when
= Ì
Ô1 : Ê p xˆ a + sin px = 1
sin Á ˜ = 2
ÔÓ Ë 2 ¯
Now, if a = 1

Ï0 : x π (2 n + 1), n Œ I fi sinp x = 0
= Ì
Ó1 : x = (2 n + 1), n Œ I fi x = 1, 2, 3, 4, 5.
When a = 0
Here x Π[0, 2]
fi sinp x = 1
Thus, f (x) is discontinuous at x = 1.
1 5 __ 9
9. Case-I : When x < 1 fi x = __
​   ​ , __
​   ​ , ​    ​.
2 2 2

We have ​ lim 
   
n Æ •  ( 
log(2 + x) – x2n sinx
​​ ​ _________________
x2n + 1
 ​ 
     ​ ) Thus, the total number of points of discontinuity
= 8.

( 
log(2 + x) – 0
= ​ ____________
​ 
1+0
  
 ​   
​ )
12. We have f (x) = ​ lim
    
n Æ • 
​ 
x
​​ ___________
     ​  ​
1 + (2sinx)2n (  )
= log(2 + x)
Case-II : When x > 1
= ​ lim
    
n Æ • 
​ 
x
​​ ___________
     ​  ​
1 + (4sin2 x) n (  )
( 
ÏÔ0 : 0 £ 4 sin 2 x < 1
We have ​ lim 
   
log(2 + x) – x2n sinx
​​ ​ _________________
 ​ 
   ​
   ) = Ì
2
ÔÓ1 : 4 sin x = 1

( 
nÆ • 
x2n + 1

)
Ï p
  log(2 + x) ÔÔ0 : x π np ± 6 , n Œ I
​ _________     
​ – sin x = Ì
x2n
= ​ lim 
    _______________
​​ ​      ​ 
   ​ Ô1 : x = np ± p , n Œ I
n Æ •  1 ÔÓ
1 + ___
​  2n   ​ 6
p
x Thus, f (x) is discontinuous at x = n p ± __ ​   ​ , n Œ I

( 
6

0 – sinx
= ​ ​ _______ ​    )
​ = – sinx 13. Case-I : When x < 1

(  )
1+0
cos(px) – x2n sin(x – 1)
Thus, f (x) is discontinuous at x = 1. We have f (x) = ​ lim 
  ​  ​ ​  ___________________
         ​​ 
n Æ •
1 + x2n+1 – x2n
10. We have f(x) = ​ lim
    
nÆ •  ( 
x2n – 1 + ax2 + bx
​​ ​ ______________
 ​ 
  
x2n + 1
   ​ ) = ​ lim 
 x Æ 1
cos(px) – 0
  –​ ​ ​ __________
1+0–0
   ​ 
 ​ (  )
= cosp = –1
Ïax 2 + bx : -1 < x < 1
Ô Case-II : When x > 1
Ôa - b - 1

( 
: x = –1 We have f (x)
Ô 2

)
Ô
= Ìa + b + 1 cos(px) ________sin(x – 1)
Ô : x =1 ​ _______  ​ 
  – ​  x    ​ 
Ô 2 x 2n + 1
_________________
lim ​ ​ 
= ​       ​ ​
Ô1 n Æ • 1 1
1 + _____    ​ – __
                                                                                                                                                                                       

ÔÓ x : x > 1 or x < - 1 ​  2n + 1 ​ x ​

( 
x

)
For continuity at x = 1, sin(x – 1)
0 – ________
​  x     

a+b+1   +​ ​ ​ ____________
​ _________
 ​   =a+b = ​ lim    
    ​  ​
2 1
1 + 0 – __
 x Æ 1
​ x ​
a + b = 1 ...(i)
The Continuity and Differentiability  4.51

= ​ lim 
 x Æ 1
( sin(x – 1)
  +​ ​ – ​ ________
x–1
 ​  

​ ) Ï 2 1
Ô(1 - 4 x ) : 0 £ x < 2
Ô
f (x) = ÔÌ(4 x 2 - 1) : £ x < 1
= – 1 1

Ô 2
Thus, f (x) is continuous at x = 1.
Ô- 1 : 1£ x < 2
Ï| 2 x - 3 |[ x ] : x ≥ 1 Ô
Ó
14. We have f (x) = ÔÌ Ê p x ˆ Thus, f (x) is discontinuous at x = 1.
Ôsin ÁË 2 ˜¯ : x < 1
Ó 17. We have
Ï Ê p xˆ Ï
Ôsin ÁË 2 ˜¯ : 0 £ x <1 Ô
Ô Ô| sin (p x ) | : -1 £ x < 0
Ô
Ô 3 f (x) = Ì1 - {x} : 0 £ x <1
f (x) = Ì(3 - 2 x )[ x ] : 1 £ x <
2 Ô
Ô Ô1 + Ècos ÊÁ p x ˆ˜ ˘ : 1 < x £ 2
Ô 3 ÔÓ ÍÎ Ë 2 ¯ ˙˚
Ô(2 x - 3)[ x ] : 2 £ x £ 2
Ó
Ï- (sin (p x )) : - 1 £ x < 0
Ô
Ï Ê p xˆ fi f (x) = Ì1 - x : 0 £ x <1
Ôsin ÁË 2 ˜¯ : 0 £ x < 1 Ô1 - 1 : 1< x £ 2
Ô Ó
Ô 3 Ï- (sin (p x )) : - 1 £ x < 0
f (x) = ÔÌ(3 - 2 x ) : 1 £ x < 2 Ô
fi f (x) = Ì1 - x : 1£ x <1
Ô 3
Ô(2 x - 3) : £ x < 2 Ô0 : 1< x £ 2
Ó
Ô 2
ÔÓ2 : x=2 Clearly, f (x) is discontinuous at x = 0.
Clearly, f (x) is discontinuous at x = 2.
15. We have
(  )
x+ y
18. Given f ​ ​ _____
2
 ​  
f (x) + f (y)
 ​ = ​ _________
2
 ​   

Ï[cos p x ] : 0 £ x £1 Differentiate w.r.t x where y as constant


f (x) = Ì
Ó
Ï1
2 x - 3 [ x - 2]
: x=0
: 1< x £ 2
(  )
x+y
f ¢​ ​ _____
2
1 f ¢ (x)
 ​ × __
 ​   ​   ​  = ​ ____
2 2
 ​ 

Ô 1 Put y = – x, we get,


Ô0 : 0< x£
Ô
Ô 1
2
(  )
x– x
f ¢​ ​ ____
2
 ​ = f ¢ (x)
 ​  
Ô- 1 : < x £1
Ô 2
f (x) = Ì f ¢ (x) = f ¢ (0) = – 1
Ô(2 x - 3) 3
: 1< x £
Ô 2 On integration, we get,
Ô 3
Ô(3 - 2 x ) : <x<2 f (x) = – x + c
Ô 2 when x = 0, f(0) = 1, then c = 1
ÔÓ0 : x=2
            

Thus, f (x) = 1 – x
Thus f (x) is discontinuous at x = 0, 1/2, 2.
Now,
16. Since 1 £ x < 2
{(f (0))2 + (f (1))2 + (f (2))2 + ... + (f (n))2}
fi 0£x–1<1
= 12 + 02 + 12 + 22 + ... + (n – 1)2
fi [x2 – 2x] = Î(x – 1)2 – 1˚
n (n – 1)(2n – 1)
= Î(x – 1)2˚ – 1 = 0 – 1 = – 1. = 1 +​  ______________
 ​    
6
We have
Hence, the result.
ÏÔ 1 - 4 x 2 : 0 £ x <1 19. Given f (x) = x3 – x2 + x + 1
fi f (x) = Ì
2 f ¢ (x) = 3x2 – 2x + 1
ÓÔ[ x - 2 x ] : 1 £ x < 2
Since its D is negative,
4.52  Differential Calculus Booster

so, f ¢ (x) > 0, " x Œ R ​ lim 


  – 
​ f (x) = ​ lim 
  +  ​ f (x) = f (0)
x Æ 0 x Æ 0

( 
Thus, f is strictly increasing in (0, 2)

Now,
Ï f ( x) : 0 £ x £ 1
g(x) = Ì
​ lim 
  – 
x Æ 0
​ (a sinx + b cosx) = ​ lim 
  + 
ex + x 1/x
​​​ ______
​ 
x Æ 0 1 + 2x
  ​  ​​ ​ = b )
Ó3 - x : 1 < x £ 2


ÔÏ x - x + x + 1 : 0 £ x £ 1
= Ì
3 2 b = ​ lim 
  +  (  )
ex + x 1/x
​​​ ______
​ 
x Æ 0 1 + 2x
  
 ​  ​​ ​

​​​( 1 + ​( ​ ______ 
ÔÓ3 - x : 1< x £ 2
 ​ – 1 )​ )​​ ​
x 1/x
e +x
= ​ lim 
   
Clearly, g (x) is continuous in (0, 2) but not differen- x Æ 0 +
1 + 2x
tiable at x = 1
​​​( 1 + ​(_________
 ​ e 1–+1 2x–  x 
​ )​ )​​ ​
x 1/x
20. We have = ​ lim 
     
+
x Æ 0
f (x) = x4 – 8x3 + 22x2 – 24x – 55
f ¢ (x) = 4x3 – 24x2 + 44x – 24 Ê Ê e x -1- x ˆ ˆ 1
lim Á Á ˜˜
= 4(x3 – 6x2 + 11x – 6) + Á Á 1 + 2 x ˜¯ ˜¯ x
x Æ0 Ë Ë
= e
= 4(x – 1)(x – 2)(x – 3)
Ê Ê e x -1- x ˆ ˆ
Clearly, f is increasing in [1, 2] » [3, •) and decreas- lim Á Á ˜˜
+ Á Á x (1 + 2 x ) ˜¯ ˜¯
x Æ0 Ë Ë
ing in (– •, 1] » [2, 3] = e
Now, min{f (t) : x £ t £ x + 1}: – 1 £ x £ 1 ÊÊ e x -1 ˆ ˆ
lim Á Á ˜˜
+ ÁË ÁË 2 x + (1 + 2 x ) ˜¯ ˜¯
x Æ0
Ï f ( x + 1) : - 1 £ x £ 0 = e
= Ì
Ó f (1) : 0 < x £1 = e0 = 1
Ï f ( x + 1) : - 1 £ x £ 0 Again, for x > 0, we have
Ô
Thus, g(x) = Ì f (1) : 0 < x £1
Ô( x - 10) : x > 1
Ó
( 
ex + x 1/x
f (x) = ​​​ ______ 
1 + 2x

 ​  ​​ ​ )
1 Ê ex + x ˆ
Ï x 4 - 4 x 3 + 4 x 2 - 64 : - 1 £ x £ 0 lnÁ ˜
x ÁË 1 + 2 x ˜¯
Ô = e
= Ì- 64 : 0 < x £1 1
Ô( x - 10) ​ __ ​  ln(ex + x)– ln(1 + 2x)
Ó : x >1 = e​ ​x ​
(  )
[ {  (  ) }
1 x + ex
__
​   ​ln​ ​ _____  1 ex + 1
__ 2
​ x ​ ​ ​  ______  ​  ​ – ______
  
​  ​
Now, ​  
lim ​g  (x) = g (0) f ¢(x) = e​  ​x 1 + 2x ​ ​ ​   ​       ​  ​ ​
x Æ 0 ex + x 1 + 2x ​
​ lim 


  + 
​ g (x) = ​  
lim 

 ​ g (x) = g (0)
x Æ 0

– 64 = – 64 = – 64


x Æ 0
​x
1
  – ​ ​ __2  ​ {ln(ex + x) – ln(1 + 2x)}  ​
]
(  )
[ 
1 x + ex x
__
​   ​ln​ ​ _____x  + 2xex + 1 + 2x – 2ex – 2x}
 ​  ​ 1 {e_________________________
1 + e ​ ​ __
So, g (x) is continuous at x = 0 = ​e​x ​ x ​​  x          ​ ​
(e + x)(1 + 2x) ​
Also, g (x) is continuous at x = 1
(  )]
x
1 e +1
Thus, g (x) is continuous in [– 1, •) –  ​ __2  ​  ln​ ​ ​ ______ 

 ​  ​  ​
x ​ 1 + 2x

[ 
Ï4 x - 12 x + 8 x : - 1 £ x £ 0
3 2
Ô
Now, g¢(x) = Ì0 : 0 £ x £1 = e​ ​x
__
(  )
1 x + ex
​   ​ln​ _____
​   ​  ​ 1 (2xe

1 + 2 x ​ ​ __
x
– ex + 1)
1
​ x ​​  _____________
x  ​ – __
ex + x
​  2  ​  ln​ ​ ______ 
  
    
 ​  ​  ​ (  )]
(e + x)(1 + 2x) x 1 + 2x
Ô1 : x >1
Ó
Clearly, g¢(0 +) = g¢ (0 –) = 0 Now, f ¢ (1) = ​e​ (  ) { __​ 1  ​ – ln​( ​ 1_____
1 + e
ln​ ____
​   ​  
 ​
 3 ​ ​
+e
 )​ }​
 ​  
3 3
and g¢ (1+) π g¢ (1–)
Thus, f (x) is not differentiable at x = 1
= ​ ​ _____
1 + e __
3 (  ){ 
1 1+e
 ​​ ​   ​  – ln​ ​ _____
 ​  
3 3
 ​  
 ​  ​ (  ) }
So, f (x) is not differentiable in [– 1, •). p
For x = – __
​   ​ , f (x) = j (x)
21. Since f (x) is continuous at x = 0, so 2

​ lim 
  ​ f (x) = f (0)
x Æ 0
p
(  ) (  )
2
p
f ​ – ​ __ ​   ​ = j ​ __
​   ​   ​ = – a + 0 = – a
2
The Continuity and Differentiability  4.53

e + 1 __
Thus, a = – ​ ​ _____
3
 ​  (  ){  (  ) }
1
3
e+1
 ​​ ​   ​  – ln ​ ​ _____
3
 ​  
 ​  ​
1
4. We have f (x)  ​ _________
   
log|x2 – 4|
 ​

= ​( ​ _____  ​ ​  ln ​  ​ _____


3 ){ ( 3 ) 3}
e+1 e+1 1 f is not defined when
 ​    ​ – __
 ​   ​   ​   ​
(x2 – 4) = 0, |x2 – 4| = 1
Therefore, (x2 – 4) = 0, (x2 – 4) = ±1
e+1
3 (  ) {  (  ) }
a = ​ ​ _____
e+1
 ​ ​ ln​ ​ _____
 ​  
3
 ​  
1
 ​ – __
​    ​  ​, b = 1
3 x2 = 4, x2 = 4 ± 1
__ __
22. If x Œ [– 1, 1], then x = ± 2, ± ​÷5 ​  ,  ± ​÷3 ​
  
Thus m = 6
ÏÔsin x - 1 : - 1 £ x < 0
f (g( )) = Ì 2
ÔÓsin x : 0 £ x £1 Also, = ​  
x Æ 0 ( 
xn – sin(xn)
lim ​ ​ ​ __________
     
x – sinnx
​  ​ )
Ï- sin x : - 1 £ x < 0 Its limit exists only when n = 1
Now, |g(x)| = Ì
Ósin x : 0 £ x £1 Hence, the value of (m + n) is 7.
5. We have
f (|g(x)|) = sin2 x, " x Œ [–1, 1]
f (x) = [3 + 4 sinx], x Œ [p, 2p]
ÏÔ1 - sin x : - 1 £ x < 0 = 3 + [4 sin x]
Also, |f (g(x))| = Ì 2
ÔÓ2sin x : 0 £ x < 1

Now, L.H.L = ​  


lim 

 ​ (sin2 x – sinx + 1) = 1
x Æ 0

and R.H.L = ​  


lim 
+
 ​ (2sin2 x) = 0
x Æ 0

Thus h (x) is not continuous at x = 0


Ïsin (2 x ) - cos x - 1 : - 1 £ x < 0
Again, h¢(x) = Ì
Ó2sin(2 x ) : 0 £ x £1
Thus, the number of points of disc is 8.
Now, L.H.D = ​ lim 
  –  ​ (sin 2x – cosx – 1) = – 2 6. We have
x Æ 0
f (x) = [[x]] – [x – 2]
and R.H.D = ​ lim 
  +  ​ (2sin(2x)) = 0
x Æ 0 = [x] – [x – 2]
Therfore h (x) is not differentiable at x = 0. = [x] – [x] + 2
=2
Integer Type Questions
So f (x) is a constant function.
1. Since f : [1, 7] Æ Q be a continuous function It is continuous everywhere.
where f (1) = 7, so it is a constant function. So, p = the number of discontinuous points = 0

( 
Thus, f (7) = 7.
2. Clearly, f be a constant function, where f (2) = 5 and q = ​ lim 
3x4 – 2
  ​ ​ ​ ___________   
__________   ​  ​
)
( ÷  )
Thus, f (2.5) = 5. x Æ •
÷​ x  8 + 3x   
+ 4 ​
3. Given f (x) = ||x| – 1| 2
3 – __​  4  ​ 
Clearly, f is non differentiable at x = – 1, 0, 1 x
lim ​ ​ ​ ____________
= ​   __________   
   ​  ​
x Æ • 3
Thus, m=3 4
​ 1 + __
​  7  ​  +   __
​  8  ​   ​
1 x x
Also, g (x) = _____
​     ​ 
log|x| =3
So, g (x) is not defined at x = – 1, 0, 1 Hence, the value of (p + q + 2) is 5.
Thus g (x) is discontinuous at x = – 1, 0, 1 7. We have m

( 
So, n=3
Hence, the value of (m + n) is 6. x Æ 0
tan 2x – 2sinx
lim ​ ​ ​ ____________
= ​  
x3
 ​ 
    ​
)
4.54  Differential Calculus Booster

= ​  
lim 

​  b = a + 1 ...(i)
x Æ 0

(  (  )
Also, f (x) is differentiable at x = 1
(2x)3 2(2x)
)
5
_____ ______ x3 __
__ x5
2x + ​   ​   + ​   ​  + ... – 2 ​ x – ​    ​ + ​    ​ – ...  ​
3 15
​ ​ ________________________________________
     
x 3
 ​ 
3! 5!
    ​ So, ​ lim 
x Æ 1 x–1 ( 
f (x) – f (1)
  +​​ ​ _________ ​  

​ = ​  
lim 
x Æ 1– )
f (x) – f (1)
​ ​ ​ _________
x–1
 ​  

​(  )
(  )
8 2
= ​ __
​    ​ + __
3 6
​   ​   ​
​ lim  ( 
(3ax – b + 2) – (3a – b + 2)
  +​​ ​ ________________________
     ​     ​ )
x Æ 1 x–1
= ​( __
3 3) 3
8 1 9
​   ​  + __
​   ​   ​ = __
( 
​   ​  = 3

Also, f (x) = [sin x + cos x], " x Œ (0, 2p)


= ​  
lim 
x Æ 1
(a + bx2) – (3a – b + 2)
​– ​ ​  ____________________
   
x–1
 ​     ​ )
Thus, the number of disc continuous points of f is 5,
p 3p
where x = __
​   ​ , ___
3p 7p
​   ​ , p, ___
​   ​ , ___
​   ​ .
x Æ 1
3a(x – 1)
  +​ ​ ________
​ lim  ​ 
x–1
 ​  
 ​ (  )
2 4 2 4
So, n=5 = ​  
lim 
x Æ 1– ( 
(a + (a + 1)x2) – (3a – a – 1 + 2)
​​ ​ _____________________________
    
x–1
 ​     ​ )
Again, f (x) = |log|x|
Clearly, f is not differentiable at x = – 1, 0, 1 ​ lim 
  –  
x Æ 1
( 
(a + (a + 1)x2) – (2a + 1)
​​ ​ ______________________
   
x–1
 ​     ​ = 3a )
( 
So, p=3
Hence, the value of (m + n – p)
x Æ 1
(a + 1)x2 – a – 1)
  –​ ​ ​ _______________
​ lim 
x –1
 ​ 
     ​ = 3a )
( ( 
= 3 + 5 – 3

8. We have
= 5. ​ lim 
x Æ 1
​ (a + 1)(x2 – 1) )​
  –​ ​ ​ ______________
x–1
 ​ 
     ​ = 3a )
(  )
​ lim 
  –​  
((x + 1)(a + 1)) = 3a
f (h2 + 2h + 2) – f (2) x Æ 1
  ​ ​ ​ __________________
​ lim         ​  ​
h Æ 0 f (h – h2 + 1) – f (1) 2(a + 1) = 3a

( 
2a + 2 = 3a

)
2
f ¢ (h + 2h + 2)(2h + 2) a = 2, b = a + 1 = 2 + 1 = 3
lim ​ ​ ​ ____________________
= ​      
    ​  ​
h Æ 0 f ¢ (h – h2 + 1)(1 – 2 h) Hence, the value of (a + b + 1) is 6.


2f ¢ (2)
= ​ _____ ​ 
2x
10. We have f(x) = sin–1 ​ ​ _____
  ​  
1 + x2

(  )
f ¢ (1) Clearly, f (x) is non-differentiable at x = – 1 and
2×6 x=1
= ​ _____
 ​ 
 = 3 But it is continuous everywhere.
4
9. As we know that, every differentiable function is So, p = 0 and q = 2
continuous, so f (x) is continuous at x = 1 Hence, the value of (p + q + 2) is 4.
11. We have

[  )
Thus, ​  
lim ​ f (x) = f (1) 3
x Æ 1
f (x) = [3x] – {2x}, x Œ ​ 0,  ​ ​__  ​  ​ ​.
​2 ​
​ lim 
  +​ f (x) = ​  
lim 

​ f (x) = f (1) Clearly, f is discontinuous at
x Æ 1 x Æ 1

1 2 __ 1 4
​ lim 
  +​ (3ax – b + 2) = ​  
lim  ​ (a + bx2) = 3a – b + 2 x = __
​   ​ , __
​   ​ , ​   ​ , __
​   ​ , 1
x Æ 1

x Æ 1 3 3 2 3
(3a – b + 2) = (a + b) = 3a – b + 2 Thus, the number of points of discontinuity is 5.
12. We have h (x) = |f (|x|)| = ||x| – 2|
(a + b) = 3a – b + 2
Clearly, h is not differentiable at x = – 2, 0, 2
2a – 2b + 2 = 0 Thus, g (x) is non-differentiable at x = – 2, 0, 2
a – b + 1 = 0 Hence, the number of non-differentiable points is 3.
The Continuity and Differentiability  4.55

Hints and Solutions

Questions asked in Roorkee-Jee Exams Ï2 + ( x + 1) : - 2 £ x £ - 1


and g(x) = Ì
1. Given y = |x – 2| Ó2 – ( x + 1) : - 1 £ x £ 2

Ï3 + x : - 2 £ x £ - 1
= Ì
Ó1 – x : - 1 £ x £ 2

Now, (fo g)(x) = f (g(x))

Ï - g( x ) : - 1 £ g( x ) £ 1
Clearly, y = f (x) is continuous everywhere but not = Ì
Ó g( x ) - 2 : 1 £ g( x ) £ 3
differentiable at x = 2.
2. We have ​ lim 
  ​ f (x) = f (0) Ï- (3 + x ) : - 2 £ x £ - 1, - 1 £ (3 + x ) £ 1
x Æ 0
Ô(3 + x ) - 2 : - 2 £ x £ - 1, 1 £ (3 + x ) £ 3
​ lim 
  + 
​ f (x) = ​  
lim  ​ f (x) = f (0) Ô
= Ì
: - 1 £ x £ 2, - 1 £ (1 – x ) £ 1

Ô( x – 1)
x Æ 0 x Æ 0

1 ÔÓ(1 – x ) - 2
​ lim 
    ​ (x2) = ​  
lim   ​ (x – 1) = ​ __ ​  : - 2 £ x £ - 1, 1 £ (1 - x ) £ 3
x Æ 0 + x Æ 0 – 4
Thus, f (x) is not continuous at x = 0. Ï- (3 + x ) : - 2 £ x £ - 1, - 4 £ x £ - 2
3. We have f (x) = x – |x – x2| Ô (1 + x ) : - 2 £ x £ - 1, - 2 £ x £ 0
Ô
= Ì
ÔÏ x - ( x - x ) : x - x ≥ 0 : - 1 £ x £ 2, 0 £ x £ 2
2 2
= Ì Ô ( x – 1)
2 2 ÔÓ – (1 + x ) : - 2 £ x £ - 1, - 2 £ x £ 0
ÔÓ x + ( x - x ) : x - x < 0
ÏÔ x 2 : x2 - x £ 0 Clearly, it is continuous at x = – 1.
= Ì
2 2
ÔÓ(2 x - x ) : x - x > 0 5. Case-I: When t ≥ 0
ÔÏ
2
x : 0 £ x £1
= Ì Then x = 2t – t = t and y = t 2 + t 2 = 2t 2
2
ÔÓ(2 x - x ) : x Œ (– •, 0) » (1, •)
Thus, y = 2x2

Case-II: When t < 0


Then x = 2t + t = 3t and y = 0
Thus, y = 0, x < 0.
The given function is defined as
ÏÔ2 x 2 : 0 £ x £ 1
f(x) = Ì
ÔÓ0 : -1 £ x < 0

Now, f ¢(0 –) = 0
Clearly f (x) is continuous in [– 1, 1] but not differ-
entiable at x = – 1 and x = 1.
and f ¢(0 +) = ​  
lim  
x Æ 0 +
  ( 
f(x) –f(0)
​ ​ ​ ________
x–0
 ​ 

 ​ )

Ï- 1 - ( x - 1) : - 1 £ x £ 1
4. We have f (x) = Ì
Ó- 1 + ( x - 1) : 1 £ x £ 3
= ​  
lim  
x Æ 0 +
  ( 
2x2 – 0
​ ​ ​  ______ ​ 
x–0

 ​ )

Ï- x
f (x) = Ì
x – 2
: -1£ x £1
: 1£ x £ 3
= ​  
lim  
+
x Æ 0

​ ​(  )
2x
​  x
2
___   ​   ​
Ó
4.56  Differential Calculus Booster

= ​  
lim  
+
 ​ (2x) 9. We have f(0) = 0
x Æ 0
Now, ​ lim 
  +  ​  f (x)

( 
=0 x Æ 0

Thus, f(x) is differentiable at x = 0


Hence, it is also continuous at x = 0.
= ​ lim 
  + 
x Æ 0
x(3e1/x + 4)
​ ​ ​ __________
   ​  
(2 – e1/x)

)
6. We have (fog)(x)
= f(g(x))
= ​ lim 
  + 
x Æ 0 ( x(3 + 4e– 1/x)
​ ​ ​ ___________
  
   ​  ​
(2e– 1/x – 1) )


= f(sin x)

=÷  
________
​ |sin x – 1| ​ 
= ​ lim 
  + 
x Æ 0 ( 0(3 + 4e– •)
​ ​ ​ __________
   ​  
(2e– • – 1)
​= 0 )
Also, (go f)(x)
= g(f(x))
_____
Also, ​ lim 
  – 
x Æ 0   ( 
x(3e1/x + 4)
​​ ​ __________
  
2 – e1/x
  
​  ​ )
( 
  – 1| ​
)
= g(​÷|x  ) 0(3e– • + 4)
________ = ​  
lim   ​ ​ ​ __________
     
​ ​

= (​÷sin|x – 1| ​ 
) x Æ 0 –
2 – e– •
Clearly, (go f)(x) is not differentiable at x = 1. =0
Thus, f(x) is continuous at x = 0
ÏÔ x 3 : x 2 < 1
7. We have f(x) = Ì
2
Now, f  ¢(0 +)
ÔÓ x : x ≥ 1


ÔÏ x 3 : - 1 < x < 1
= Ì
ÔÓ x : x ≥ 1 and x £ – 1
= ​ lim 
  + 
x Æ 0 ( f(x)– f(0)
​ ​ ________
​ 
x – 0)
 ​  
 ​)
= ​ lim 
   
x Æ 0 +
(   x(3e1/x + 4)
​ __________
​ ​ ​ 
x–0
   ​ 
(2 – e1/x)
_____________
–0
 ​ 
    ​ )
= ​ lim 
  + 
x Æ 0 ( (3e1/x + 4)
​ ​ ​ _________
(2 – e1/x) )
 ​  

= ​ lim 
  + 
x Æ 0 ( (3 + 4e–1/x)
​ ​ ​ _________
   ​  
(2 – e–1/x)

)
( 
limit = 1
Function f(x) is continuous at x = 1
but not differentiable at x = 1
= ​ lim 
  + 
x Æ 0
(3 + 4e– •)
​ ​ ​ _________
(2 – e– •)
  )
 ​  ​

3
ÏÔ3 x : -1£ x £1 = __
​   ​ 
8. We have f(x) = Ì 2
ÔÓ 4 - x : 1< x < 4 –
Also, f ¢(0 )
= ​  lim 
x Æ 0 –
  ( 
f(x) – f(0)
​ ​ ​ ________
x–0
 ​  

​ )
= ​ lim 
   
x Æ 0 +
(  x(3e1/x + 4)
​ __________
  
(2 – e1/x)
__________
​ ​ ​  x      
 ​ 
)
​  ​


= ​ lim 
  – 
x Æ 0 ( (3e1/x + 4)
​ ​ ​ _________
(2 – e1/x)
 ​  

)
Thus, f(x) is continuous in [– 1, 4] but not differen-
tiable at x = 1.
= ​ lim 
  – 
x Æ 0 ( (3e– • + 4)
​ ​ ​  _________
(2 – e– •)

 ​ ​ )
The Continuity and Differentiability  4.57


4
= __
​   ​  = 2
​  
lim   (  )
ax
​ ​ ___ ​   ​
= ​​e​x Æ 0 ​​ x ​ = ea

2
Thus, f(x) is not differentiable at x = 0. Also, ​ lim 
  +  ​ f (x)
x Æ 0

( 
10. We have f(0)

= ​ lim 
  ​  f (x)
x Æ 0
 (x + c)1/3 – 1
​ ​ ​ ___________
= ​ lim 
  + 
x Æ 0 (x + 1)1/2 – 1
   ​  ​
)
(  )  (
_____

)
1 2
lim ​ ​ __
= ​   ​   ​ – ______
​  2x    
 ​  ​ ((x + c) – 1)(​÷x  + 1 ​
 + 1)
x Æ 0 x e –1 ​ ​ ​ ________________________________
= ​ lim 
  +      
     ​  ​

( 
2/3
x Æ 0 ((x + 1) – 1) ((x + c) + (x + c)1/3 +  1)
= ​  
e2x – 1 – 2x
lim ​ ​ ​ __________
   ​ 
 ​
)
 (
Its limit exists only when c = 1
x Æ 0 x(e2x – 1)

)
So, R.H.L

( 
4x2 8x3 _____

= ​  
x Æ 0
1 + 2x + ​ ___ ​ + ​ ___ ​ + ... – 1 – 2x
2! 3!
lim ​ ​ ​  __________________________
   
( 
2x
2 e______–1
     ​  ​
)
= ​ lim 
  + 
((x + 1) – 1)(​÷x  + 1 ​
    
   
x Æ 0 ((x + 1) – 1) ((x + 1) 2/3
 + 1)
​ ​ ​ ________________________________
+ (x + 1)1/3 + 1)
 ​  ​
)
( 
2x  ​ ​      
​  ​

( 
2x _____

)
)
(​÷x  + 1 ​
 + 1)
  4x2 ___ 8x3 = ​ lim 
    ​ ​​  ______________________
   
    ​  ​
 ​ ____ ​ 
 + ​   ​ +... x Æ 0 + ((x + 1) 2/3 + (x + 1)1/3 + 1)
2! 3!
lim ​ ​ ​ _____________
= ​     
   ​  ​
x Æ 0
( 
2x  ​ ​ 
2x
2 e______–1
2x
   

​  ​) 2
= ​ __ ​ 
3
=1 2
Thus, ea = __
​    ​ = b, c, = 1
Ï2 + 1 - x 2 : | x| £ 1 3
Ô
11. We have f(x) = Ì
ÔÓ2e(1- x )
2
: | x| > 1 Hence, a = loge ​ __
2
(  ) 2
​   ​   ​, b = __
3
​   ​ , c = 1
3
Ï2 + 1 - x 2 : -1 £ x £ 1
Ô
= Ì 2
ÔÓ2e(1- x ) : x > 1 and x < – 1

13.

Clearly the function f(x) is not continuous at x = 1


and x = – 1
So, f(x) is not differentiable at x = –1 and x = 1.  

(  )
14. We have
Clearly f(x) is discontinuous at x = – 1 and not 1
____
​     ​ 
differentiable at x = – 1 and x = 1. e​ ​x – 1 ​ – 2
​   
lim  ​ ​ _______
​  1
  ​  ​
+ ____
12. Since f(x) is continuous at x = 0, so x Æ 1 ​     ​ 
​e​x – 1 ​ + 2

(  )
​ lim 
  ​  f (x) = f(0)  1
x Æ 0 – ​ ____
   ​ 
1 – 2​e​ x – 1 ​
​ lim 
  + 
​ f (x) = ​  
lim  ​  f (x) = f(0)   +​ ​ ​ _________
= ​ lim  1
   ​  ​

x Æ 0 x Æ 0 x Æ 1 – ​ ____
   ​ 
1 + 2 ​e​ x – 1 ​

( 
​ lim 
  + 
​ f (x) = ​  
lim  ​  f(x) = b ...(i)

x Æ 0 x Æ 0
= ​ lim 
x Æ 1
1 – 2e–•
  +​ ​ ​  ________ 

1 + 2 e–•
 ​  ​ )
Now, ​  
lim  ​ f (x)

( 

x Æ 0
= ​  
lim 
x Æ 0–
 ​ (1 + ax)1/x
1 – 2.0
= ​ ​ ______ 
1 + 2.0 )
 ​  ​ = 1
4.58  Differential Calculus Booster

(  )
1
____
​     ​  Here, 0 < x sin px £ 1
e​ ​x – 1 ​ – 2
lim ​ ​ ​ _______
Also, ​   1

 ​  ​ Thus, f(x) = [x sin p x] = 0
x Æ 1– ____
​     ​ 
​e​x – 1 ​+ 2 Therefore, f(x) is continuous and differentiable at

( 
(– 1, 1).
= ​  
lim 
x Æ 1–
​ 
1e–•  – 2
​​ ________
e–• + 2
  )
 ​  ​ Ï- 1
5. We have, f(x) = Ì
: -2 £ x £ 0
Óx - 1 : 0 < x £ 2
( 
1.0 – 2
= ​ ​ ______ ​  
0+2
​ ) Ï- 1 : - 2 £ | x| £ 0
=–1 Now, f(|x|) = Ì
Ó| x | - 1 : 0 < | x | £ 2
Since R.H.L π L.H.L., so limit does not exist.
Thus, f(x) is not continuous at x = 1. fi f(|x|) = |x| – 1 : 0 < |x| £ 2

Problems asked in Past Iit-Jee Exams Ï- x - 1 : - 2 £ x £ 0


fi f(|x|) = Ì
Ó x -1 : 0 < x £ 2
1. Ans. (a, c) _____
__
​ x  + 1 ​
   ​ – ÷
We have f(x) = x(​÷x  )
  Ï1 : -2 £ x £ 0
Also, |f(x)| = Ì
Clearly, ​  
lim ​  f (x) = 0 = f(0) Ó| x - 1| : 0< x£2
x Æ 0

Thus, f(x) is differentiable at x = 0 Ï1 : -2£ x£0

( 
Ô
)
_____
1 1
__
Also, f ¢(x) = (​÷x  ) + x​ ____
  + 1 ​
  ​  – ​÷x   ​  __
   ​ – _______
​  _____
   
 ​ ​ fi |f(x)| = Ì1 - x : 0 < x £ 1
2​÷x 
   ​ 2​÷x  + 1 ​
  Ôx - 1 : 1 < x £ 2
Clearly, f(x) is not differentiable at x = 0. Ó

2. We have, f(x) = x3 – x2 + x + 1 Therefore, g(x) = f(|x|) + |f(x)|


f ¢(x) = 3x2 – 2x + 1 > 0, " x Œ R Ï- x : -2£ x£0
2
Î a = 3 > 0, D = b – 4ac = 4 – 12 = – 8 < 0˚ Ô
= Ì0 : 0 < x £1
Thus, f(x) is strictly increasing in (0. 2). Ô2 x - 2 : 1 < x £ 2
Ó
Ï f ( x) : 0 £ x £ 1
g(x) = Ì Ï- 1 : - 2 £ x £ 0
Ó3 - x : 1 < x £ 2 Ô
fi g¢(x) = Ì0 : 0 < x £ 1
ÔÏ x 3 - x 2 + x + 1 : 0 £ x £ 1 Ô2 : 1 < x £ 2
g(x) = Ì Ó
ÔÓ3 - x : 1< x £ 2
Clearly, g(x) is continuous but not differentiable at
ÔÏ3 x 2 - 2 x + 1 : 0 £ x £ 1 x = 0 and x = 1.
g¢(x) = Ì
ÔÓ- 1 : 1< x £ 2 6. Ans. (a).
Thus, g(x) is continuous for all x in [0, 2] but not x
We have f(x) = _____
​       ​
differentiable at x = 1. 1 + |x|
3. Ans. (a, d)
We have f(x) = 1 + |sin x|


Clearly, f(x) is differentiable everywhere.
7. Let f(x) = x2, since f(– x) = f(x).
Clearly f(x) is continuous everywhere but not dif- Now, f ¢(x) = 2x
ferentiable at x = np, n ΠI.
Thus, f ¢(0) = 0
4. We have f(x) = [x sin p x]
The Continuity and Differentiability  4.59

8. Let f(x) = x and g(x) = [x]


Clearly f(x) is continuous everywhere and g(x) is
f(x) – f (y)
fi ​________
​ 
(x – y) | 
​ £ (x – y)
 ​  

|
| 
discontinuous function.
Thus, f(x) + g(x) = x + [x] is a discontinuous
function.
9. Let g(x) = ax + b



x Æ y
f (x) – f (y)
​ lim ​ ​_________
​ 

|f ¢(y)| £ 0
(x – y) | ​ £ ​ 
 ​  
   lim ​ (x – y)
x Æ y

Ïax + b x£0 fi |f ¢(y)| = 0


Then f(x) = Ô 1/ x
ÌÊ 1 + x ˆ fi f ¢(y) = 0
ÔÁ ˜ x>0
ÓË 2 + x ¯ Thus, f(x) is a constant function.
Since f(x) is continuous, so it is continuous at Ï| x - 3| :x≥0
x = 0. Ô
11. We have f(x) = Ì x 2 3 x 13
  Ô - + :x<0
Now, ​  
lim 

 ​f(x) = ​  
lim 

 ​(ax + b) = b Ó4 2 4
x Æ 0 x Æ 0

(  ) ​​( __​ 12 ​  )​​ ​ = 0
x + 1 1/x • Ï| x - 3| : x ≥1
Also, ​  
lim   
​f(x) = ​  
lim  ​​​ ​ _____ 
 ​  ​​ ​ =
Ô
= Ì1
x Æ 0 + x Æ 0 + x + 2 2
ÔÓ 4 ( x - 3) + 1 : x < 1
Thus. b = 0.
Now, f(– 1) = – a
x + 1 1/x
We have, f(x) = ​​ ​ _____ 
x+2
 ​  ​​ ​ (  )
= ​e​ (  )
x + 1 1/x
log​​ ____
​   ​  ​​ ​
x + 2 ​


1
= e​ ​x (  )
x + 1
​ __ ​log​ ____
​   ​  ​
x + 2 ​

fi f ¢(x) = e​ ​x
1
(  ) x + 1
​ __ ​log​ ____
​   ​  ​
x + 2 ​ ×

x
1
(  x+1
) (  )(  ) )
x+2 ( 
​ – ​ __2  ​  log​ ​ _____ 
1 x + 2 _______
 ​  ​ + __
​ x ​ ​ ​ _____ 
 ​  ​​ ​ 
x + 1 (x + 2)2
1
   
 ​  ​  ​ Clearly f(x) is continuous at x = 1 and x = 3.
and f(x) is not differentiable at x = 3.

​    ​ )​ ​( – log​( __
Thus, f ¢(1) = ​( __
Ï| x - 3| : x ≥1
2 9)
​    ​ )​ + __
2 2 3 1
​    ​ × __
​   ​   ​ Ô
3 3 Also, f(x) = Ì 1 2
ÔÓ 4 ( x - 3) + 1 : x < 1
​   ​  )​ ​( – log​( __
= ​( __ ​   ​  )​ + __ ​   ​  )​
2 2 1

3 3 6 Ï
Ô- 1 : x <1
= ​( __
​   ​  log ​( __​   ​  )​ + __ ​   ​  )​
2 3 1 Ô
f ¢(x) = Ì1 : 1£ x < 3
3 2 9
Ô1
Since, f ¢(1) = f(– 1), so Ô ( x - 3) : x < 1
Ó2


2
a = – ​ __
3 (  (  ) __​ 19 ​  )​
3
​   ​  log ​ __
​   ​   ​ +
2
Thus, f ¢(1) = – 1
Therefore, f(x) is differentiable at x = 1.
Thus, the required function is 12. We have
Ï p
Ï Ê2 Ô x + a 2 sin x : 0£ x<
Ê 3ˆ 1ˆ Ô
4
Ô- Á log ÁË ˜¯ + ˜ x : x £ 0 p p
Ô Ë 3 2 9¯ f(x) = ÔÌ2 x cot x + b : £x£
f(x) = Ì 4 2
ÔÊ x + 1 ˆ
1/ x Ô
:x>0 Ô p
ÔÁË x + 2 ˜¯ Ôa cos2 x - b sin x : < x£p
Ó Ó 2
p
10. We have |f (x) – f (y)| £ (x – y)2 At x = __ ​   ​ 
4
4.60  Differential Calculus Booster

​   
lim + 
p
​ f(x) = ​   
lim – 
p
__
p
​ f (x) = f ​ __
​   ​   ​
4 (  ) ​ lim 
  + 
​ f (x) = ​  
x Æ 0
lim 

 ​ f (x) = f (0)
x Æ 0

(  ( 
x Æ ___
​   ​  x Æ ​   ​  __

) )
4 4
​ x 
÷   ​  1 – cos 4x
__ ​ lim 
  +  ​ ​ ___________
​  ___  __    ​  ​ = ​  
lim   ​ ​ ​ ________
 ​  
 ​ = a
​   
lim + 
​ (2x cot x + b) = ​   
lim –     sin x)
​ (x + a​÷2 ​  x Æ 0    + ÷
​÷16 ​ ​ x    ​ – 4 x Æ 0 –
x2
p p
x Æ ​ __ ​ 

(  ( 
x Æ ​ ___ ​  ___

) )
4 4 __ __
   ​(÷
​÷x     + ÷
​ 16 ​ ​ x 
   ​ + 4) 2sin22x
​ ​ ​ _______________ ​ ​ ​ _______
(  (  ) )
p p ​ lim 
  +  __        ​  ​ = ​  
lim     ​  
 ​ = a
= ​ 2◊​ __ ​  cot ​ __ 16 + ÷ ​ x 
   ​ – 16 x2

​   ​   ​ + b  ​ x Æ 0 x Æ 0

( 
4 4

p
​ __ ​  + a​÷2 ​
  ◊ sin ​ __
p  __
(  ) (  (  ) )
p  p 
​   ​   ​ = ​ 2◊ ​ __ ​  cot ​ __
​   ​   ​ + b  ​
​ lim 
  + 
x Æ 0
a = 8
___
   + ÷
​ ((​÷16 ​
__
​ x 
  ​  + 4)) = ​  
lim 
x Æ 0 –

8sin22x
​ ​ ​ _______
4x2
 ​  
 ​ = a )
4 4 4 4

( 
p __ p
​ __ ​  + a​÷2 ​
4
1
  ◊  ___
​  __  ​ = ​ 2.​ __ ​  + b  ​
÷   
​ 2 ​ 4 ) 15.

p
a – b = ​ __ ​   ...(i)
4
p
At x = __
​   ​ 
2
​ lim    + ​ f (x) = ​ lim 
p
x Æ ​ ___ ​ 
p
__
x Æ ​   ​ 
p
  – ​ f (x) = f ​ __
​   ​   ​
2 (  )
2 2

​ lim 
  + ​ f (x) = ​ lim 
p
___
x Æ ​   ​ 
p
__
x Æ ​   ​ 
p
  – ​ f (x) = f ​ __
​   ​   ​
2 (  )
2 2
Clearly, the function is not differentiable at x = – 1,
​ lim 
  + ​ (a cos2x – b sinx) = ​ lim 
  – ​ (2x cot x + b) = b 0, 1, 2.
p p
x Æ ​ ___ ​  x Æ ​ __ ​ 
2
2 16. Given f (x + y) = f (x). f (y)
(– a – b) = b Put x = 0 = y, f (0) = 0, then f (0) = 1
a = 0. It is given that f ¢ (0) = 2
p
Thus, b = – ​ __ ​ 
4
Hence, the values of a = 0, b = – ​ __ ​ .
p
​ lim 
h Æ 0
f (0 +  h) – f (0)
  ​ ​ ​ _____________
h–0
  
 ​   
​= 2 (  )
4
13. Ans. (b) f (0) . f (h)  –  f (0)
  ​ ​______________
​ lim  ​         ( 
​  ​ = 2 )
( 
h Æ 0 h
Given
x
f (x) = ​ __
​    ​ – 1  ​ in [0, p] )
2
Ï- 1 : 0 £ x < 2 h Æ 0
f (h) – 1
  ​ ​ ​ _______
​ lim 
h
    
​  ​ = 2 (  ) ...(i)

( 
Thus, [f (x)] = Ì
Ó0 : 2 £ x £ p Now, f ¢ (x) = ​  
h Æ 0
f (x  + h) – f (x)
lim ​ ​ ​ _____________
h
       ​  ​ )
Ïtan (- 1) : 0 £ x < 2
tan ([f (x)]) = Ì
Ótan (0) : 2 £ x £ p = ​  
h Æ 0 ( 
f (x) . f (h)  – f (x)
lim ​ ​ ​  ____________
h
     ​   ​ )
(  )
Clearly tan(f (x)) is not continuous at x = 2 f (x)(f (h – 1)
x
( 
x–2
Now, f (x) = ​ ​ __  ​ – 1  ​ = ​ ​ _____
2 2
 ​  
 ​ ) (  ) lim ​ ​ ​ ___________
= ​  
h Æ 0 h
     ​  

​ ____
1 2
   ​ = ______
​     
f (x) (x – 2)
 ​ = ​  
h Æ 0
f (h) – f (0)
lim ​ f (x) × ​ ​ _________
h
  ( 
 ​  
​ )
1 = 2f (x), from (i)
Thus, ​ ____
   ​ is also not continuous at x = 2.
f (x) f ¢ (x)
Thus, ​ ____ ​ = 2
So, tan[f (x)] & tan​ ​ ____
x = 2.
1
[  ]
   ​  ​ are both discontinuous at
f (x)
f (x)
On integration, we get,

14. It is given that f(x) is continuous at x = 0, log|f (x)| = 2x + c

so, ​  
lim ​ f (x) = f (0) If x = 0, f (0) = 1, then c = 0.
x Æ 0
The Continuity and Differentiability  4.61

Thus, log|f (x)| = 2x. Ï 3p


2x ÔÔ1 : 0< x£
fi f (x) = e 4
(c) Let f (x) = Ì
17. Case-I : When t ≥ 0 Ê 2 ˆ
Ô2sin Á ˜ x : 3p
< x<p
Then x = 2t – t = t and y = t 2 + t 2 = 2t 2 ÔÓ Ë 9¯ 4
Thus, y = 2x2 Now, ​   
lim 
+

​ f (x) = ​   
lim 
+

​ f (x) = 1
3p 3p
x Æ ​ ____
 ​ 
  x Æ ​ ____
 ​ 

Case-II: When t < 0 4 4
3 p
Then x = 2t + t = 3t and y = 0 Thus f(x) is continuous at x = ___ ​   ​ 
4
Thus, y = 0, x < 0. Also, f(x) is continuous at all other points in
The given function is defined as (0, p)
Ï p

ÔÏ2 x 2 : 0 £ x £ 2
f (x) = Ì ÔÔ x + sin x : 0< x£
2
: -1 £ x < 0 (d) Let f (x) = Ì
ÔÓ0 p p
Ô sin (p + x ) : < x < p
ÔÓ 2 2

Now, ​ lim 
  + ​ = ​ lim 
p
___
x Æ ​   ​ 
p
___
x Æ ​   ​ 
p
  – ​ f (x) = f  ​ __
​   ​   ​
2 (  )
2 2

​ lim 
p
___
x Æ ​   ​ 
  + ​ ​ __
  + ​ f (x) = ​ lim 
p
___
x Æ ​   ​ 
p
2 ( 
​   ​  sin(p + x  ​ )
2 2
p
= – __​   ​ 
2
And, ​ lim 
  – ​ f (x) = ​ lim 
  – ​ (x + sin x)
p p
x Æ ​ __ ​  x Æ ​ __ ​ 
2 2
Now, f ¢ (0 –) = 0
p
= ​ __ ( 
​   ​  + 1  ​ )
( 

and f ¢(0 +) = ​  
lim  
x Æ 0 +
f (x) – f (0)
​ ​ ​ _________
x–0
 ​ 
  ​ ) p
Thus, f(x) is not continuous at x = __
2

​   ​ 

( 
2
= ​ lim 
  + 
x Æ 0
2x2 – 0
​ ​ ​  ______ ​ 
x–0

 ​ ) So, f (x) is not continuous in (0, p).

= ​ lim 
  + 
x Æ 0
(  )
2x2
​ ​ ​ ___
x   ​   ​
19. (i) Let f (x) = sin(p [x])
Since [x] provides us integer, so f(x) = 0
It is continuous and differentiable everywhere
= ​ lim 
  +  ​ (2x)
x Æ 0
(ii) Let f (x) = sin (p (x – [x]))
=0 = sin (p{x})
Thus, f (x) is differentiable at x = 0 As we know that, {x} is discontinuous at every inte-
Hence, it is also continuous at x = 0. gral points, so f (x) is disc at all integers
18. Ans. (b, c) Thus f(x) is non-differentiable at x = – 1 and 1.
p
(a) Since tan x is not defined at x = __
​   ​  20. Given f (x) = x |x|
2
So, it is not continuous in (0, p) ÔÏ x
2
: x≥0
= Ì

1
(b) Let g (x) = x sin ​ __ (  )
​ x ​  ​ 2
ÔÓ- x : x < 0
which is continuous in (0, p). Ï2 x : x≥0
So, the integral function of a continuous func- f ¢ (x) = Ì
Ó- 2 x : x < 0
tion is also a continuous function.
x

(  )
1 Ï2 : x≥0
Thus, ​Ú ​    ​t sin ​ __
​   ​   ​dt is continuous in (0, p) f ¢¢ (x) = Ì
0
t Ó- 2 : x < 0
4.62  Differential Calculus Booster

Clearly, the function f (x) is not twice differentiable


at x = 0.
fi (  )
x+y
f ¢ ​ ​ _____
2
 ​ = f ¢(x)
 ​  

Thus, f (x) is twice differentiable at x Œ R – {0} Replacing x by 0 and y by 2x, we get,
21. Ans. (a, b, c)
f ¢ (x) = f ¢ (0) = – 1
We have f(x) = Îtan2 x˚
On integration, we get,
Now, to find ​ lim 
  +  ​ f (x) = ​  
lim  ​ f (x) = f(0)
x Æ 0

x Æ 0 f (x) = – x + c
2 2 When x = 0, c = 1.
​ lim 
  + 
​[tan x] = ​  
lim 

 ​ [tan x] = 0 = f (0)
x Æ 0 x Æ 0
Thus, f (x) = – x + 1
Thus, f (x) is continuous at x = 0.
Therefore, f (2) = – 2 + 1 = – 1.
Also, f ¢ (0) = ​  
x Æ 0
f (x) – f (0)
​ ​ ​ _________
lim  
x–0
 ​ 
 ( 
 ​ ) 24. Now, [x + 1] = 0
0 £ (x + 1) < 1

= ​  
x Æ 0
( 
[tan2 x] – 0
lim ​ ​ ​  _________
x   ​  
​ )

– 1 £ x < 0
x Œ [– 1, 0)
= ​  
x Æ 0 x
0
lim ​ ​ __ (  )
​   ​  ​ Thus, Df = R – [– 1, 0)
25. Here f is a constant function, where f (10) = 10
=0 Thus, f (1.5) = 10
Thus, f (x) is also differentiable at x = 0. 26. (a) ​ lim 
  +  ​ f (x)
x Æ 0
22. Since f is continuous at x = 0, so
= ​ lim 
  +  ​ ​e​ {  1
|x| x ​
1
​ – ​ __   ​ + __ }
​   ​  ​
​ lim 
  ​ f (x) = f (0) x Æ 0
x Æ 0
​ ​e​{​  – ​ x ​ + ​ x ​ }​​ = 1
1
__ 1
__
= ​ lim 
  + 
​ lim 
  + 
​ f(x) = ​  
lim 

 ​ f(x) = f (0) x Æ 0
x Æ 0 x Æ 0
Also, ​  
lim 

 ​
​ lim 
  + 
​ f (x) = ​  
lim  ​ f (x) = b x Æ 0

{ 

x Æ 0 x Æ 0
= ​  
lim 

 ​ ​e​
1
|x| x ​
1
​ – ​ __   ​ + __ }
​   ​  ​

Now, ​  
lim  ​ f (x) x Æ 0

{ 

x Æ 0
tan2x
 ​ _____ 
 ​ = ​  
lim 

 ​ ​e​
1 1
​ – ​ __   ​ + __
|x| x ​ }
​   ​  ​

= ​  
lim –
 ​ ​e​tan3x ​ x Æ 0
x Æ 0
​ ​e​{​  ​ x ​ + ​ x ​ }​​
1
__ 1
__
= ​  
lim  
= e2/3 –
x Æ 0

​ ​e​{​  ​ x ​ }​​ = 0
2
__
Also, ​ lim 
  +  ​ f (x) = ​  
lim 


x Æ 0 x Æ 0
a
​ _____
   ​
= ​ lim 
  +  ​ (1 + |sin x|​)​|sinx| ​ Thus, f (x) is disc at x = 0
x Æ 0
a
(b) As we know that, every disc function is not
____
​     
 ​
= ​ lim 
  +  ​ (1 + sin x​)​sinx ​ differentiable. So, f (x) is not differentiable at
x Æ 0 x = 0.
a
sinx × ____
​     
 ​
= ​ lim 
  +  ​​e​ sinx ​ = ea
x Æ 0
2
__
​   ​ 
Thus, b = e​ ​3 ​ = ea
2
__
2
a = __
​   ​ 
​   ​ , b = e​ ​3 ​
3
27.
x+y
2(  )
23. We have f ​ ​ _____
 ​  
f (x) + f (y)
 ​ = ​ _________
2
 ​   

f ¢ ​( ​ _____
2 ) 2 2
x + y __ 1 1
fi  ​ ◊ ​   ​  = __
 ​   ​   ​  f ¢(x), y as a constant Clearly, f (x) is not continuous at x = 2
and not differentiable at x = 1 and x = 2.
The Continuity and Differentiability  4.63

28. Ans. (a, c, d) fi |p¢(1)| £ 1


fi |a1 + 2a2 + 3a2 ... + nan| £ 1

33.


29. Ans. (c, d)
We have
Clearly the function f(x) is not continuous at x = 1
f (x) = (x2 – 1)|x2 – 3x + 2| + cos(|x|)
and x = – 1
f (x) = (x2 – 1)|x2 – 3x + 2| + cos(x)
So, f(x) is not differentiable at x = – 1 and x = 1.
f (x) = (x2 – 1)|(x – 1)(x – 2)| + cos(x)
34. We are given, for x ≥ 0,
Thus, f(x) is not differentiable at x = 1, 2
x
30. The function f(x) = [x] 2 – [x2], where [,] = G.I.F., is
F(x) = Ú​  ​    ​ f (t) dt
discontinuous at 0
(a) all integers 0
(b) all integer except 0 and 1 F(0) = Ú​  ​    ​ f (t) dt = 0
0
(c) all integers except 0
(d) all integers except 1. As f(x) £ cF(x), for all x ≥ 0, we get,
[IIT-JEE, 1999] f(0) £ F(0)
31. Ans. (c)
f(0) £ 0
Let f (x) = x
Since f(x) ≥ 0, " x ≥ 0, we get, f(0) ≥ 0
Then g(x) = |f (x)| = |x|
f(0) = 0
Clearly f(x) is continuous everywhere, so its modulus
is also continuous everywhere. Since f is continuous in (0, •) and F is differentiable
in (0, •), so F¢ (x) = f(x) " x ≥ 0
Thus, g(x) is continuous, whenever f is continuous.
32. Given Since, f (x) £ cF(x), " x ≥ 0, we have

p(x) = a 0 + a1x + a2 x2 + a3 x3 + ... + an xn F¢ (x) – cF(x) £ 0


Multiplying both sides by e–cx, we get,
fi p¢(x) = 0 + a1 + 2a2 x + ... + nan xn – 1
F(x) e–cx – cF(x) e–cx £ 0
fi p¢(1) = 0 + a1 + 2a2 + ... + nan
d
Now, |p(x)| £  |ex – 1 – 1| ​ ___   ​ (e–cx F(x)) £ 0
dx
fi |p(1)| £ |e1 – 1 – 1| = |e0 – 1| = |1 – 1| = 0 Thus, g(x) = e–cx F(x) is a decreasing function on
fi |p(1)| £ 0 (0, •)

fi p(1) = 0 Therefore g(x) £ g(0), for each x ≥ 0


But g(0) = e–c0 F(0) = 0
As |p(x)| £ |ex – 1 – 1|
Thus, g(x) £ 0, " x ≥ 0
fi |p(1 + h)| £ |eh – 1|
e–cx F(x) £ 0, " x ≥ 0
fi |p(1 + h) – p(1)| £ |eh – 1| ( p(1) = 0)
F(x) £ 0, " x ≥ 0

| 

p(1 + h) – p(1)
fi ​ ​  ____________
h
     ​  
| |  |
eh – 1
​ £ ​ ​ _____
h
    
​  ​ Thus, f(x) £ cF(x) £ 0 " x ≥ 0

|  | |  |
But, it is given that, f(x) ≥ 0 " x ≥ 0
p(1 + h) – p(1) eh – 1
  ​ ​ ​  ____________
fi ​ lim         ​  ​ £ ​  
 lim  ​ ​ ​ _____
   
​  ​ Hence, f (x) = 0, " x ≥ 0
h Æ 0 h h Æ 0 h
4.64  Differential Calculus Booster

(  )
35. We have
1
____
​     ​ 
e​ ​x – 1 ​ – 2
  +​ ​ ​ _______
​ lim  ____1
 ​  

x Æ 1
​e​ ​x – 1   ​ ​ + 2

(  )
1
– ​ ____
   ​  (c)
1 – 2​e​ x – 1 ​
  ​ ​ ​ _________
= ​ lim  1
  ​  ​
x Æ 1+ – ​ ____
   ​ 
1 + 2​e​ x – 1 ​

= ​ lim 
x Æ 1
( 
1 – 2e– •
  +​ ​ ​ ________
1 + 2e– •
  ​ ​ )
Clearly, sin(|x|) + |x| is not differentiable at
( 1 – 2.0
= ​ ​ ______ 
1 + 2.0
 ​  ​ ) x=0

=1

(  )
1
​ ____
   ​ 
e​ ​ x – 1 ​ – 2
Also, ​  
lim 

​ ​ ​ _______
1

 ​  
​ (d)
x Æ 1 ____
​     ​ 
​e​ ​ + 2
x – 1

= ​  
lim 
x Æ 1– ( 
1e– • – 2
​ ​ ​ _______
e– • + 2
  ​ ​ )
( 
1.0 – 2
= ​ ​ ______ ​  
0+2
​ ) Clearly, sin(|x|) – |x| is differentiable at x = 0.
37. Ans. (d)
= – 1
Since R.H.L π L.H.L, so limit does not exist.
Thus, f(x) is not continuous at x = 1.
36. Ans. (d)

(a) X¢ X
O
38. Ans. (a)
Now, f ¢(k–)

Clearly cos(|x|) + |x| is not differentiable at


= ​  
lim  
x Æ k – (  f(x) – f(k)
​ ​ ​  ________
x–k
   )

​  ​

x=0
= ​  
lim  
x Æ k – (  [x] sin (pk) – 0
​ ​ ​ ____________
x–k
  
    )
​  ​

= ​ lim 
   
h Æ 0 – (  [k – h] sin (pk – ph)
​ ​ ​ ________________
k–h–k
      ​​ )
(b) = ​ lim 
  – 
h Æ 0
( [k – 1](– 1) k – 1 sin (p h)
​ ​ ​ __________________
   
–h
     ​  ​)
= ​ lim 
  – 
h Æ 0
( [k – 1]p (– 1) k sin (p h)
​ ​ ​  __________________
ph
       ​  ​ )
= (k – 1)p (– 1) k
Clearly, cos(|x|) – |x| is not differentiable at
x=0 39. Since g(x) is continuous at x = a
The Continuity and Differentiability  4.65

  ​ g(x) = g(a)
so, ​ lim  Ïx + 2 : x < -1
x Æ a
Ô 2
​ lim 
x Æ a
f(x) – f (a)
  ​ ​ __________
​ 
x–a
   
   ( 
​  ​ = g(a) ) Thus, (go f)(x) = Ì x
Ô 2
: -1£ x < 0
Ó(| x - 1| - 1) : x ≥ 0
f ¢ = g(a)
In the neighbourhood of x = 0, (go f)(x) = x2 which
Thus, f(x) is differentiable at x = (a) is also differentiable at x = 0.
Conversely, let f(x) is differentiable at x = a 42. Given f ¢(a–) = 0

i.e.
x Æ a
( 
f(x) – (a)
lim ​ ​ ​ ________
f ¢(a) = ​   )
x – a   

​ ​ ​ lim 
  + 
h Æ 0
f (a – h) – f (a)
​ ​ ____________
​ 
– h
       ( 
​  ​ = 0 )
ÏÊ f ( x ) - f (a ) ˆ Now, f ¢(– a–)
: x πa
( 
Ô
Let g(x) = ÌÁË
Ô
x - a ˜¯
Ó f ¢(a ) : x =a
= ​ lim 
  – 
h Æ 0
f (– a – h) – f (– a)
​ ​ ______________
​ 
– h
       ​  ​ )
lim ​ g(x) = f ¢(a)
Clearly, ​  
x Æ a
g(x) is continuous at x = a
= ​ lim 
  – 
h Æ 0 ( 
– f (a + h) + f (a)
​ ​ ​ _____________
– h
       ​  ​ )
Hence, f(x) is differentiable at x = a, iff g(x) is con-
tinuous at x = a.
= ​ lim 
  – 
h Æ 0 ( 
f (a + h) – f (a)
​ ​ ____________
​ 
h
       ​  ​ )
Ïtan -1 x
Ô
40. We have f(x) =  Ì 1
: | x| £ 1 = ​ lim 
  – 
h Æ 0 ( 
f (2a – (a + h)) – f (a)
​ ​ ​ _________________
h
       ​  ​ )
Ô (| x | - 1) : | x| > 1
Ó2
= ​ lim 
  – 
h Æ 0 ( 
f (a – h) – f (a)
​ ​ ​ ____________
h
       ​  ​ )
Ï1


Ô 2 (- x - 1) : x < - 1
ÔÔ
f (x) =  Ìtan -1 x : -1 £ x £ 1
= ​ lim 
  – 
h Æ 0 ( 
f (a – h) – f (a)
​ ​ ____________
​ 
– h
    ​  ​ )
Ô1 = – f ¢(a–)
Ô ( x - 1) : x > 1
ÔÓ 2 =0

Thus, f(x) is discontinuous at x = 1, x = –1 43. Since f(x) is differentiable at x = 0, so it is continuous


at x = 0 .
Hence, the domain of f ¢(x) is R – {–1, 1}.
41. We have (go f)(x) ​ lim 
  + 
​ f(x) = ​  
lim 

 ​ f (x) = f(0)
x Æ 0 x Æ 0

(  ) (  (  ) )
= g(f(x)) eax/2 – 1 x+c 1
fi ​ lim 
  +  ​ ​ ​ _______
x    ​  ​ = ​  
lim   ​​ b sin–1 ​ _____
​   ​  
 ​  ​ = __
​   ​ 
x Æ 0 x Æ 0 –
2 2
ÏÔ f ( x ) + 1 : f ( x) < 0
= Ì 2
ÔÓ( f ( x ) - 1) + b : f ( x ) > 0 a
fi __
2
c
​   ​  = bsin–1 ​ __ (  )
​    ​  ​ =
2
1
__
​   ​ 
2
Ïx + a + 1 : x<–a fi a = 1
Ô 2
= Ì( x + a - 1) + b : - a £ x < 0
Ô 2
Also, it is differentiable at x = 0, so f ¢ (0 +) = f ¢ (0 –)
Ó(| x - 1| - 1) + b : x ≥ 0
As go f is continuous at x = – a
i.e. (go f)(– a +) = (go f)(– a +) = (go f)(– a)
fi ​ lim 
  + 
x Æ 0
f(x) – f(0)
​ ​ ________
​ 
x–0
 ​ 

  ( 
​ = ​  
lim 
x Æ 0 –
 ​ ​ ​ ________
x–0 )
f(x) – f(0)
 ​  

​ (  )
(  ) (  (  ) )
1+b=1+b=1 1 x+c 1
eax/2 – __
​   ​  b sin–1 ​ ​ _____  ​ – __
 ​   ​    ​
b=0 2
_______ 2
________________ 2
fi ​ lim 
  +  ​ ​ ​  x    ​   ​= ​ lim 
  –  ​ ​ ​  x     ​   ​
x Æ 0 x Æ 0
Also, go f is continuous at x = 0

(  ) (  (  )
(go f)(0 +) = (go f)(0 –) = (go f)(0)
(a – 1)2 + b = b = b
fi ​ lim 
  + 
1
ex/2 – __
​   ​ 
2
​ ​ ​ _______
x+c
b sin–1 ​ ​ _____
2
​ ​ ​ _________________
1
 ​ – __
 ​   ​    ​
2 )
a=1 x Æ 0 x    ​  ​ = ​  
lim 


x Æ 0 x       ​  ​
4.66  Differential Calculus Booster

fi ​ lim 
  + 
1 + ​ __
2
x

x Æ 0
(  1 x 2
​    ​  ​ + __
2 (  ) (  )
​   ​ ​​  __
​ ​ ​ _______________________
2
   
x 
1
​    ​  ​​ ​ + ... – __
​   ​ 
2
  ​  ​ ) fi f ¢¢ (x) – 2 = 0 for atleast 1 real root in x Œ (1, 3)
fi f ¢¢ (x) = 2 has 1 real root in x Œ (1, 3)

(  )
47. It is given that, f (x – y) = f (x)g(y) – g(x)f(y)
1 1
b ____________      ​× __
​  ___________ ​   ​ 
÷  ( 
and g(x – y) = g(x) g(y) + f(x)f(y)

= ​  
lim 
+c 2 2
x_____
​ 1 – ​​ ​    
2
 ​  
​– ​ ​  _________________
   ​ 
)
 ​​ ​  ​
      ​
Now, f(0) = f(x – x)
x Æ 0 1 = f(x) g(x) – g(x)f(x)
1 b =0
fi ​ __ ​  = ________
​  _____    ​ 

÷ 
8 c2 Also, g(x) = g(x – 0)
2​ 1 – ​ __ ​ ​  
4 = g(x)g(0) + f(x)f(0)
1 b
fi ​ __ ​  = _______
​  _____     ​

÷ 
8 = g(x)g(0)
c2
__
​ 1 – ​   ​ ​  
4 g(x) – g(x)g(0) = 0
1 b
fi ​ __ ​  = ______
​  __   2   ​ g(x)(1 – g(0)) = 0
8 ÷    – c
​ 4 ​
Case-I: When g(x) = 0
fi 64b2 = 4 – c2 We can say that, g¢(x) = 0 for all x in R
Hence, the result.
Thus, g¢(0) = 0
44. Ans (a)
Case-II: When g(x) π 0
Then, 1 – g(0) = 0
g(0) = 1
Now, f ¢ (0 +)


h Æ 0 +

 ​ ​ ​  ( 
f(0 + h) – f(0)
lim ____________
= ​ ______

h
       ​  ​ )
= ​ lim 
  + 
h Æ 0
( 
f(0) g(– h) – g(0)f(– h) – 0
​ ​ ​ ______________________
   
h
    ​  ​ )
45. Ans. (b)
Given
= ​ lim 
  + 
h Æ 0
( 
0.g(– h) – 1.f(– h)
​ ​ ​ ______________
h
     ​   ​ )

1
f (1) = f ​ __
2 (  ) (  )
1
​   ​   ​ = f ​ __
3 n Æ •
1
(  )
lim ​ f ​ __
​   ​   ​ = ... ​   ​ n ​  ​ = 0
= ​ lim 
  + 
h Æ 0
( 
– f(– h)
​ ​ ​ ______
h
    
​  ​ )
fi ​ lim 
n Æ •
1
  ​ f ​ __
​ n ​  ​ = 0 (  ) = ​ lim 
  +  ( 
f(– h) – f(0)
​ ​ ​  __________ )
     ​  

h Æ 0 – h
fi f(0) = 0
= f ¢ (0 –)
Since there are infinitely many points in the neigh-
bourhood of x = 0, so Thus f is derivable at x = 0
f(x) = 0 So f is continuous at x = 0

fi f ¢ (x) = 0 Now, g(0) = g(x – x)


= g(x)g(x) + f(x)f(x)
fi f ¢ (0) = 0
= (g(x))2 + (f (x))2
Thus, f(0) = 0 = f ¢(0)
Thus, (g(x))2 + (f(x))2 = 1
46. Ans. (c)
Since f is continuous at x = 0, so g is also continuous
Let g(x) = f (x) – x2
at x = 0.
By mean value theorem, we can say that, g(x) has
We have g¢(0)
three roots at x = 1, 2, 3.
fi g¢(x) has atleast 2 real roots in x Œ (1, 3)
fi g¢¢ (x) has atleast 1 real root in x Œ (1, 3)
= ​  
h Æ 0 (  g(h) – g(0)
lim ​ ​ ​  _________
h
     ​  
​ )
The Continuity and Differentiability  4.67

= ​  
h Æ 0 ( g(h) – 1
lim ​ ​ ________
​ 
h
    ​  ​ ) fi

F¢(x) = 0
F(x) = constant

= ​  
h Æ 0 ( g(h) – 1 g(h)
lim ​ ​ ​ _______
h
   
+1
​ × ​ _______ 
g(h) + 1
 ​  ​ )
50.

Ans.
F(10) = F(5) = 5.

(  )
(g(h))2 – 1 (a) Æ p, q, r
lim ​ ​ __________
= ​   ​      ​  ​ (b) Æ p, s
h Æ 0 h(g(h) + 1)
(c) Æ r, s

= ​  
h Æ 0 ( – (f(h))2 ________
lim ​ ​ ​ _______
h
   
​ × ​ 
1
    ​  ​
(g(h) + 1) )

(d) Æ p, q
(a) Let f(x) = x |x|


h Æ 0 ( 
 f(h) ________
lim ​ ​ ​ ____
= – ​  
h
  ​ × ​ 
f(h)
  
(g(h) + 1)
 ​  ​ ) ÏÔ x 2
= Ì
2
: x≥0
ÔÓ- x : x < 0

= ​–  
 lim 
h Æ 0 ( f(h) – f(0) ________
________
 ​ ​ ​ 
h
    ​ 
× ​ 
f(h)
  
(g(h) + 1)
 ​  ​ ( f(0) = 0) )
ÏÔ2 x
f ¢(x) = Ì 2
: x≥0
ÔÓ- 2 x : x < 0
​ 
0
= f ¢ (0) × ​ _______( 
   
g(0) + 1
 ​  ​ ) f(x) is differentiable everywhere.
=0 So, f(x) is differentiable as well as continuous
48. Ans. (a, b, c) in (– 1, 1) __
(b) Let g(x) = ÷    
​ |x| ​
ÏÔ x : x≥0
g(x) = Ì
ÔÓ - x : x < 0

Ï 1
Ô2 x : x≥0
Ô
g¢(x) = Ì
Ô -1 : x<0
ÔÓ 2 - x

Clearly, f(x) is not differentiable at x = 0.


Clearly f(x) is continuous for all x in R. However, it is continuous at x = 0.
(c) Let h(x) = x + [x]
Now f(x) = 1 for all x > 1
f ¢ (x) = 1 Ïx - 1 : - 1 < x < 0
= Ì
Óx : 0 £ x <1
So, f(x) is continuous for all x in R but not differen-
tiable at x = 1. Clearly, f(x) is not continuous at x = 0
49. Ans. (b) So it is not differentiable at x = 0
Given f ¢¢(x) = – f(x)
d
fi ​ ___   ​ (f ¢(x)) = – f(x)
dx
d
fi ​ ___   ​ (g(x)) = – f(x)
dx (d)
fi g¢(x) = – f(x)
Also, F(x) = ​​ f ​ __ (  (  ) ) (  (  ) )
x 2
​    ​  ​  ​​ ​ + ​​ g​ __
2
x 2
​    ​  ​  ​​ ​
2
fi F¢(x) = 2f ​ __
x
(  ) (  )
​    ​  ​ . f ¢ ​ __
2
x 1
(  ) (  )
​    ​  ​ . ​ __ ​  + 2g​ __
2 2
x
2
x  1
​    ​  ​.g¢​ __
​    ​  ​ . ​ __ ​ 
2 2
Clearly f(x) is differentiable and continuous in

2
x
F¢(x) = f ​ __ (  ) (  ) (  ) (  )
​    ​  ​ . f ¢ ​ __
x
​    ​  ​ + g​ __
2
x
​    ​  ​.g¢​ __
2
x
​    ​  ​
2
(– 1, 1)
51. Ans. (c)

2
x
(  ) (  ) (  ) (  )
2
x x
2
x
F¢(x) = f ​ ​ __  ​  ​ . f ¢ ​ ​ __  ​  ​ – f ​ ​ __  ​  ​ . f ¢ ​ ​ __  ​  ​ = 0
2 Given ​ lim 
  +​  
(g(x))= p = – 1
x Æ 1
4.68  Differential Calculus Booster

(1  + h – 1) n
  ​ ​​  _______________
fi ​ lim     ( 
    ​  ​ = – 1
log(cosm (1  + h –1) ) (  )
f ¢​ __
1
​   ​ ​ = 0
2

(  )
h Æ 0
1

( 
Also, f ¢​ __
​   ​ ​ = 0
fi ​ lim 
hn
  ​ ​ ​ _________
   ​  
h Æ 0 log(cosmh)
​ = – 1
) 4
Thus, f ¢(x) = 0 at two points in [0, 1]

fi ​ lim 
hn
  ​ ​ ​ __________ ( 
   ​  ​ = – 1

h Æ 0 m log(cosh) ) Let (  )
g(x) = f ​ x + __
1
​   ​   ​ sinx
2

nhn – 1
  ​ ​ ​ _______
fi ​ lim      ( 
​  ​ = – 1 ) (  )
g(– x) = f ​ – x + __
1
​   ​   ​ sin(– x)
2

 ( ) (  )
h Æ 0 – m tan h
1
= – f ​ __
​   ​ – x  ​ sinx
hn – 2 2
fi ​ lim 
n _______
  ​ ​ –  ​ __ 
  
​  ​​ ​  (  )   
 ​  ​ = – 1
= – f ​( 1  –  ​( __
m
​   ​ – x )​ )​ sinx
h Æ 0 tanh
____ 1
– m ​     ​ 

(  )  
h 2

  ​ ​ __
fi ​ lim 
h Æ 0
n hn – 2
​ m  ​   ​​ ​ ______
tanh
m ​ ____
h
( )
  
  
 ​  ​ = 1
​ 

= – g(x)
= – f ​( x + __​   ​)​ sinx
1
2

It is possible only when n – 2 = 0 and ​ __


n
​ m  ​   ​ = 1 (  ) Thus, g(x) is an odd function.

(  )
1/2
n = 2 = m 1
​Ú   ​ ​  f ​ x + __
​   ​   ​ sin x  dx = 0
2

( 
52. Statement-I
)
– 1/2
g(x) cosx – g(0)
We have ​   lim   ​ ​ ​ _____________
 ​  
     ​ 1
sinx
Again, ​Ú  ​ ​ f(1 – t)esin(pt) dt
x Æ 0

(  )
1/2
g¢(x) cosx – g(x)sinx
lim ​ ​ ​ _________________
= ​   cosx      ​  ​
x Æ 0 Put 1 – t = u
1/2
=0
= ​Ú ​   ​   ​f (u)esinp (1 – u) du
Also, f(x) = g(x)sinx 0

1/2
f ¢(x) = g¢(x)sinx + g(x)cosx
= ​Ú ​   ​   ​f (u)esin(p – pu) du
f ¢¢(x) = g¢¢(x)sinx + 2g¢(x)cosx – g(x)sinx 0
1/2
f ¢¢(0) = 0 = ​Ú ​   ​   ​f (u)esinp udu
Thus, ​  
lim ​ (g¢(x)cotx – g(0)cosec x) 0
x Æ 0
1/2
0 = f ¢¢(0) = ​Ú ​   ​ ​ f(t)esinp tdt
fi Statement is true 0

Satetement-II 54. We have g(x) = f –1(x)


f ¢(x) = g¢(x)sinx + g(x)cosx fi f(g(x)) = x
f ¢(0) = g(0)
fi f ¢(g(x)). g¢(x) = 1 ...(i)
It is not a correct explanation of statement-I.
53. Ans. (a, b, c, d) Now, f(0) = 1
Given f(x) = f(1 – x) From (i), we get, f ¢(g(1)). g¢(1) = 1
1
f ¢(x) = f ¢(1 – x) × – 1 fi g¢(1) = ​ ______
    ​
f ¢(g(1))
Put x = 1/2

(  ) (  )
1 1

1
f ¢ ​ __
1
​   ​ ​ = – f ¢ ​ 1 – __
​   ​ ​ fi g¢(1) = ​ ____
   ​ = ___
​     ​ = 2
2 2 f ¢(0) 1/2

f ¢​( __
​   ​)​ = – f ¢​( __
[Let g(1) = k
​   ​)​
1 1

2 2 Also, f(g(1)) = 1

2f ¢​( __​   ​)​ = 0
1 f(k) = 1 = f(0)

2 k=0
The Continuity and Differentiability  4.69

Thus,
55. Ans. (b, c)
g(1) = k = 0]
Now, f ¢ (x) = ​  
h Æ 0 (  h )
f (x + h) – f (x)
lim ​ ​ ​ _____________
       ​  ​

(  )
x
________ f (x) + f(h) – f(x)
We have, f(x) = ln x + ​Ú ​    ​ ​÷1  + sin t  
​ dt lim ​ ​ ​ ______________
= ​        ​   ​
0
h Æ 0 h


1
f ¢(x) = __
_______
​ x ​ + ​÷1  + sinx   
​  dx = ​  
h Æ 0 (  )
f (h)
lim ​ ​ ____
​     
h
​  ​

Clearly, f ¢(x) exists for all x > 0


Thus, f ¢(x) is continuous for all x > 0 but not dif-

h Æ 0 (  )
f (h) – f (0)
lim ​ ​ ​  _________
= ​  
h
   ​  

ferentiable at x > 0. = f ¢ (0) = k


More-over, f ¢(x), f(x) > 0 " x Œ (1, •)
On integration, we get, f (x) = kx + c
x
________ If x = 0, f (0) = 0, then c = 0.
ln x + Ú 
​  ​   ​​÷ 1 + sin t  
​ dt
0 Thus, f (x) = kx.
1 _______
> __
​ x ​ + ÷ ​ dx " x Œ (p, •)
​ 1  + sinx   fi f ¢ (x) = k
1
Thus, ​ __x ​ is not bounded.
Ï 2 Êpˆ
Ô x cos Á ˜ : xπ0
58. We have f (x) = Ì Ë x¯
56. Ans. (a, b, c, d)
Ô0 : x=0
We have Ó
Y At x = 0,

y = –x –
L
2
ln
x
f ¢ (0 +) = ​ lim 
  + 
x Æ 0
​  ( 
f (x) – f (0)
​ ​ __________
x–0
   ​  
​ )
( 
y=
(  )
)
p

O X x2|cos​ __
​ x ​  ​| – 0
y= –1 = ​ lim 
    ____________
​ ​ ​    
 ​   ​
co
s y =x x Æ 0 + x–0
x

Y¢ = ​ lim 
  + 
x Æ 0
(  (  (  ) ) )
p
​ ​ x​ cos​ __
​ x ​  ​  ​  ​ = 0
p
Clearly f (x) is continuous at x = __ ​   ​  and f (x) is not
differentiable at x = 0.
2
Also, f ¢ (0 –) = ​  
lim 
x Æ 0 –
  ( 
f (x) – f (0)
​  ​ ​ ________
x–0
 ​ 
  ​ )
Also, f (x) is differentiable at x = 3/2

Now,
x Æ 1
f (x) – f (1)
  +​ ​ ​ _________
f ¢ (1+) = ​ lim 
x–1
 ​  
 ( 
​ ) = ​  
lim  
x Æ 0 –
(  (  (  )) )
p
x2​ cos​ __
​ x ​  ​  ​
__________
​ ​ ​  x     ​   ​

= ​ lim 
x Æ 1+
lnx – 0
x–1 ( 
  ​ ​ ​  ______ ​ 

 ​ ) = ​  
lim 

  (  (  (  ) ) )
p
​ ​ x​ cos​ __
​ x ​  ​  ​  ​ = 0
x Æ 0

= ​ lim 
1
__
(  )
​ x ​
__
  ​ ​ ​   ​   ​ = 1
x Æ 1+ 1
Thus, f(x) is differentiable at x = 0
At x = 2

x Æ 1
 f (x) – f (1)
  –​ ​ ​ _________
Again, f ¢ (1–) = ​ lim 
x–1 ( 
 ​  

​ ) f ¢ (2+) = ​ lim 
  + 
x Æ 2
( 
f (x) – f (2)
​ ​ ​ _________
x–2
 ​ 
  ​ )
x–1–0
(  )
(  |  (  )| )
  ​ ​ ​ ________
= ​ lim   ​ 

 ​ = 1 p
x Æ 1– x–1 x2 ​ cos​ __
​ x ​  ​  ​ – 0
= ​ lim 
  +  ​ ​ ​ ____________   
 ​   ​
Thus, f (x) is differentiable at x = 1. x Æ 2 x–2

(  (  ) )
57. Ans. (b, c) p
–x2cos​ __
​ x ​  ​
Given f (x + y) = f (x) + f (y) = ​ lim 
  +  ​ ​ ​ _________ ​   
  ​
x Æ 2 x–2
Put x = 0 = y, f (0) = 0
4.70  Differential Calculus Booster

(  (  ) (  )
)
p p p
 x2sin ​ __
​ x ​  ​ × – ​ __2  ​  – 2x cos​ __
​ x ​  ​
x
= ​ lim 
    ​ ​ ​ _________________________
     ​     ​
x Æ 2 + 1


p
= 4sin ​ __ (  ) (  )p
​   ​   ​ × ​ – ​ __ ​   ​ = – p
2 4

Also, f ¢ (2–) = ​  


lim 
x Æ 2 – ( 
 f (x) – f (2)
​ ​ ​ _________

x–2
 ​    ​ )
(  (  )
)
p
x2cos​ ​ __ x ​  ​ – 0
= ​  
lim   
​– ​ ​  ____________   
 ​   ​
x Æ 2 x–2

(  )
Clearly, it will intersect at
p
2x cos ​ __(  ) p
​ x ​  ​+ x2 sin​ __ (  ) p
​ x ​  ​ × __
​  2  ​ 
x
= ​  
lim   ​ ​ ​ _______________________
    ​     ​ 1 3 5
x Æ 2 – 1 ​ __ ​  < x < 1 or __
​   ​  < x < 2 or __
​   ​  < x < 3
2 2 2

p
= 4 sin ​ __(  )
​   ​   ​ ×
2
p
__
​   ​  = p
4 Thus, it will intersect at unique point in

( 
Thus, f(x) is differentiable at x = 0 but not differen-
tiable at x = 2.
1
​ n + __
​   ​ , n + 1  ​ or (n, n + 1)
2 )
59. Ans. (d)
61. Ans. (a, c)
We have, f ¢¢(x) – 2f ¢(x) + f (x) ≥ ex
Since f (x) ≥ 1 " x Œ [a, b]
fi f ¢¢(x).ex – f ¢(x).ex – f ¢(x) ◊ e–x + f (x) e–x ≥ 1 For g(x)
L.H.D at x = a is zero.
d d
fi ​ ___   ​ ( f ¢(x).e–x) – ___
​     ​ ( f (x).e–x) ≥ 1 and R.H.D at x = a
dx dx

(  )
x
d
fi ​ ___   ​ ( f ¢(x).e–x – (f (x)e–x)) ≥ 1 ​Ú ​    ​f (t) dt – 0
dx ​ ​ ​  __________
= ​ lim 
  + 
a
x Æ a x – a     ​  ​
2
d
fi ​ ___2  
 ​ (f (x)e–x)) ≥ 1
dx ​ f (x) ≥ 1
= ​ lim 
  + 
x Æ a
2
d
fi ​ ___2  
 ​ (j (x)) ≥ 1, where j (x) = f (x)e–x
dx Thus, g(x) is not differentiable at x = a
Similarly, L.H.D at x = b is greater than 1.
fi j (x) is concave upward.
So, g(x) is not differentiable at x = b.
f (0) = 0 = f (1)

fi j (0) = 0 = j (1)
fi j (x) < 0
fi f (x) < 0
60. Ans. (b, c) 62.
We have f (x) = x sin (px)
f ¢ (x) = sin(p x) + p x cos (p x)
Now, f ¢ (x) = 0 gives
Clearly f(x) is not differentiable at x = – 1, 0, 1.
sin(p x) + p x cos (p x) = 0
Thus, the number of points non-differentiable points
sin(p x) = – p x cos (p x) = 3.
63. Differentiability at x = 0
sin(p x)

(  )
​ _______   ​ 
= – p x f (0) – f (0 – q)
cos (p x) lim ​ ​ ​ ____________
L.H.D = f ¢ (0 –) = ​        ​  ​
q Æ 0 q
tan(p x) = – p x
The Continuity and Differentiability  4.71

= ​  
q Æ 0 ( 0 + g (– q)
lim ​ ​ ​ _________
q
    ​   )
​= 0
since g¢(1) π 0, so f (h (x)) is non differentiable at
x=0
Differentiability of h ( f (x)) at x = 0

q Æ 0 ( 
f (0) – f (0 + q)
lim ​ ​ ​ ____________
RHD = f ¢ (0 +) = ​  
q
     ​  
​ ) ÏÔe| f ( x )| : x π 0
h ( f(x)) = Ì
= ​  
q Æ 0 (  )
g (q)
lim ​ ​ ​ ____
q
  
​  ​ = 0
ÔÓ1 : x=0

LHD = h¢ (f (0 – q))


Thus, f (x) is differentiable at x = 0
Differentiability of h (x) at x = 0
h ¢ (0 +) = 1, h (x) is an even function.
= ​  
q Æ 0 (  h ( f (0)) – h ( f (0 – q))
lim ​ ​ ​ __________________
q
     )
  ​  ​

Hence, non differentiable at x = 0


Differentiability of f (h (x)) at x = 0
f (h (x)) = g (e|x|) " x Œ R
= ​  
q Æ 0 ( 
lim ​ ​ ​ _________
|g (– q)|
 ​ 

g (– q)
1 – e|g (– q)| ______
× ​     
q )
​  ​ = 0

LHD = f ¢ (h (0 –)) = ​  


q Æ 0
​  ( 
f (h(0)) – f (h(0 – q))
lim ​ ​ _________________
q
       ​  ​ ) Also, RHD = h ¢ (f (0 + q))

( 
g (1) – g (e q)
lim ​ ​ ​ __________
= ​  
q Æ 0 q
     ​   )
​ = – g ¢ (1)
= ​  
q Æ 0 ( 
h (f (0 + q)) – h (f (0))
lim ​ ​ ​ _________________
q
       ​  ​ )
q Æ 0 ( 
f (h (0 + q)) – f (h (0))
lim ​ ​ ​ _________________
RHD = f ¢ (h (0 +)) = ​  
q
       ​  ​ ) = ​  
q Æ 0 (  |g(q)|
g (– q)
e|g (q)| – 1 ______
lim ​ ​ ​ ________ × ​     
 ​ 

q )
​  ​ = 0

= ​  
q Æ 0
( 
g (e q) – g (1)
lim ​ ​ ​ __________
q
     ​   )
​ = g¢(1)
Chapter

5 Differentiation

between the derivative (slope of a curve) and the integral


Concept Booster (the area beneath it), which deemed him as the creator of
1.  Introduction calculus. Thereafter, calculus was actively used to solve the
major scientific dilemmas of the time, such as:
In mathematics, differential calculus is a subfield of calculus 1. calculating the slope of the tangent line to a curve at
concerned with the study of the rates at which quantities any point along its length.
change. It is one of the two traditional divisions of calculus,
2. determining the velocity and acceleration of an object
the other being integral calculus.
given a function describing its position, and design-
The primary objects of study in differential calculus are ing such a position function given the object’s veloc-
the derivative of a function, related notions such as the dif- ity or acceleration.
ferential, and their applications. The derivative of a function
3. calculating arc lengths and the volume and surface
at a chosen input value describes the rate of change of the
area of solids.
function near that input value. The process of finding a
derivative is called differentiation. Geometrically, the deriva- 4. calculating the relative and absolute extrema of
tive at a point is the slope of the tangent line to the graph of objects, especially projectiles.
the function at that point, provided that the derivative exists For Newton, the applications for calculus were geometri-
and is defined at that point. For a real-valued function of a cal and related to the physical world-such as describing the
single real variable, the derivative of a function at a point orbit of the planets around the sun.
generally determines the best linear approximation to the For Leibniz, calculus was more about analysis of change
function at that point. in graphs. Leibniz’s work was just as important as Newton’s,
Differential calculus and integral calculus are connected and many of his notations are used today, such as the nota-
by the fundamental theorem of calculus, which states that tions for taking the derivative and the integral.
differentiation is the reverse process to integration.
2. Definitions
Differentiation has applications to nearly all quantitative
disciplines. For example, in physics, the derivative of the The derivative of the function y = f (x) with respect to the
displacement of a moving body with respect to time is the variable x is the function f ¢(x) is given by
velocity of the body, and the derivative of velocity with
respect to time is acceleration. Newton’s second law of
motion states that the derivative of the momentum of a body

dy
___
(  )
D y
​  lim ​ ​ ___
​    ​ =   
dx D x Æ 0 D x
​    ​ ​
equals the force applied to the body. The reaction rate of
a chemical reaction is a derivative. In operations research,
derivatives determine the most efficient ways to transport
materials and design factories.
i.e f ¢(x) =   
D x Æ 0 ( 
f (x  + D x) – f (x)
​  lim ​ ​ ​ ______________
D x
     ​   ​)
Calculus was created by Isaac Newton, a British scientist,
as well as Gottfried Leibniz, a self-taught German math-
ematician, in the 17th century. It has been long disputed
f ¢(x) =  
h Æ 0 ( 
f (x  + h) – f (x)
​ lim ​ ​ ​ _____________
h
       )
​  ​

who should take credit for inventing calculus first, but both
The function f ¢ is called the derivative of f, since it is
independently made discoveries that led to what we know
derived from f by the limiting operation.
now as calculus. Newton discovered the inverse relationship
5.2  Differential Calculus Booster

We have a variety of notations for the derivative of the d (loga x) ______1


function f(x) (viii) ​ ________
    ​ 
= ​     ​ 
dx x loge a
d
___ df
​    ​ (f (x)), ___
​    ​, D (f(x)), f ¢(x), f ¢, y¢, y1 Step-II
dx dx
d (sin x)
The process of finding the derivative of function’s is 2. (i) ​ _______
    ​ = cos x
called the differentiation. dx
Mathematically, derivative is equal to the slope of the d (cos x)
tangent to a curve at a point. (ii) ​ _______
    ​ = – sin x
dx
Y d (tan x)
(iii) ​ _______
    ​ = sec2 x
dx
f ¢(a )
d (cot x)
(iv) ​ _______
    ​ = – cosec2 x
dx
d (sec x)
(v) ​ _______
     
​ = sec x tan x
dx
X
O x=a d (cosec x)
(vi) ​ _________
    ​ = – cosec x cot x
dx

Also, f ¢(x) =  
h Æ 0 ( 
f (x + h) – f (x)
​ lim ​ ​ _____________
​ 
h
       )
​  ​
Step-III
d (sin–1 x) ______
3. (i) ​ ________     ​ 
1
= ​  _____
   
 ​
dx ​÷1  – x2 
 ​
is known as first principle of differentiation or ab-initio
method of differentiation. d (cos–1 x) 1
(ii) ​ ________
    = – ​ ______
​     
_____  ​
dx ​÷1  – x2 
 ​
Rules to find out the differentiation by the first
principle d (tan–1 x) _____1
(iii) ​ ________
    ​ 
= ​    2 
 ​
dx 1+x
1. Let y = f (x)
d (cot–1 x) 1
2. Write f (x + h) – f (x) (iv) ​ ________
    = – ​ ______
​     ​ 
dx 1  + x2
3. Find  
h Æ 0 ( 
f (x  + h) – f (x)
​ lim ​ ​ ​  ____________
h
       ​  ​ )
d (sec–1 x) ________
(v) ​ ________
dx
    ​ 
1
= ​  _____      ​
|x|​÷x  2 – 1 ​ 
dy
4. Which is the value of f ¢(x) or ___
​    ​ d (cosec–1 x)
dx 1
(vi) ​ __________
     ​ = – ​ ________
   
_____  ​
dx |x|​÷x  2 – 1 ​ 
3. Derivative of some standard functions
Step-IV
Step-I
d

d 1
dx x
(  )
4. (i) ​ ___  ​ ​ __
1
​   ​  ​ = – ​ __2  ​ 
x
1. (i) ​ ___   ​ (c) = 0

(  )
dx d 1 – 2
(ii) ​ ___  ​ ​ __
​    ​   ​ = ___
​  3 ​ 
d (x) dx x2 x
(ii) ​ ____   
​ = 0
dx d __ 1
(iii) ​ ___  ​ (​÷x  ____
   ​) = ​  __    ​ 
d (k x) dx 2​÷x    ​
(iii) ​ _____    ​ = k


dx
d (xn)
(iv) ​ _____    ​ = nxn – 1

d 1__
(iv) ___
dx ​÷x  (  )
​     ​ ​ ___
  ​ 
1
​    ​  ​ = – ​ _____
 __ ​ 
2x ​÷x   ​ 
dx
Step-V
d (ex)
(v) ​ _____   ​ = ex 5. (i) The Sum Rule:
dx
d du dv
d (ax) ​ ___  ​  (u + v) = ___ ​    ​ + ___
​    ​
(vi) ​ _____    ​ = ax log a dx dx dx
dx
(ii) The Difference Rule:
d (loge x) __ 1
(vii) ​ ________     ​ 
= ​ x ​ d du dv
dx ​ ___  ​  (u – v) = ___​    ​ – ___
​    ​
dx dx dx
Differentiation  5.3

(iii) The product Rule: (iii) tan–1 (tan x)


Ï p p
d dv du
​ ___  ​( u × v) = u ​ ___  ​ + v ​ ___  ​ Ôx : - <x<
dx dx dx 2 2
Ô
Ô p 3p
= Ìx - p : < x < .
(iv) The quotient Rule: 2 2
Ô
dv du Ô 3p p
u ​ ___  ​ – v ​ ___  ​ Ôx + p : - 2 < x < 2
Ó
d __
___ u
(  )
​     ​ ​ ​ v ​  ​ = ​ 
dx
dx
_________
v2
 ​ 
dx

p
5. (i) sin–1 x + cos–1 x = __ ​   ​ , x Œ [– 1, 1]
2
4. Differentiation of Composite function –1 –1 p
__
(ii) tan  x + cot  x = ​   ​ , x Œ R
2
If y is a differentiable function of u and u is a differentiable p
function of x, then (iii) cosec  x + sec  x = __
–1 –1
​   ​  : x Œ (– •, 1] » [1, •)
2
dy dy ___ du 6. (i) sin–1 x + sin–1 y ______ ______
​ ___  ​ = ___ ​    ​ . ​    ​ = sin–1 (x​÷1  – y2    ​+ y​÷1  – x2) ​  
dx du dx
Thus chain rule can aslo be difined as (ii) cos–1 x + cos–1 y _____ ______
d = cos–1 (xy – ÷ ​ 1  – x2    ​ ​÷1  – y2) ​ 
​ ___   ​ ( f (g (x))) = f (g (x))◊ g¢(x), where –1 –1 
dx (iii) sin  x – sin y _____ ______
y = f (u) and u = g (x) = sin–1 (x​÷1  – y2   ​ – y​÷1  – x2) ​  
(iv) cos–1 x – cos–1 y _____ _____
5. Differentiation by Inverse Trigonometric Function = cos–1 (xy + ÷ ​ 1  – x2   ​ ÷ ​ 1  – y2 
 ​)

( 
We shall use the following standard result to find the deriva-
tive of inverse trigonometric functions.
1. (i) sin–1 (– x) = – sin–1 (x)

x+y
7. (i) tan–1 x + tan–1 y = tan–1 ​​ _____ 
1 – xy

 ​  ​ )


(ii) tan–1 (– x) = – tan–1 (x)
(iii) cosec–1 (– x) = – cosec–1 (x)

–1
8. 2 tan  x
x–y
1 + xy ( 
(ii) tan–1 x – tan–1 y = tan–1 ​ ​ ______   ​  ​ )
(  ) (  ) (  )
2. (i) cos–1 (– x) = p – cos–1 (x) 2x 2x 1 – x2
(ii) cot–1 (– x) = p – cot–1 (x) = tan–1 ​ _____
​   2  ​  ​ = sin–1 ​ _____
​   2   ​  ​ = cos–1 ​ ​ ______2 

 ​  ​
1–x 1 +  x 1+x
(iii) sec–1 (– x) = p – sec–1 (x)
3. (i) sin (sin–1 x) = x : – 1 £ x £ 1 Some important substitutions
(ii) cos–1 (cos x) = x : – 1 £ x £ 1 Expressions Substitutions
(iii) tan–1 (tanx) = x : x Œ R ______
(i) ​÷a  2 – x2 
 ​ x = a sinq, a cosq
(iv) cot–1 (cotx) = x : x Œ R ______
(v) cosec–1 (cosecx) = x : x Œ (– •, 1] » [1, •) (ii) ​÷a  2 + x2 
 ​ x = a tanq, a cotq
______
(vi) sec (sec–1 x) = x : x Œ (– •, 1] » [1, •)
(iii) ​÷x  2 – a2 ​
  x = a secq, a cosecq
4. (i) sin–1 (sin x) _____
(iv) ​÷a  – x ​
  x = a sin2q, a cos2q
_____
Ï p p (v) ​÷a  + x   ​ x = a tan2q, a cot2q
Ôx : - £x£ _____

÷ 
2 2 –x
a_____
Ô (vi) ​ ​    ​ ​ x
  = a cosq, a cos2q
Ô p 3p a+x
= Ìp - x : £x£
Ô 2 2 6. Formula for differentiation of inverse function
Ô 3p p
Ô- p - x : - 2 £ x £ - 2
Ó

(ii) cos–1 (cos x)

Ïx : 0£ x£p
Ô
= Ì2p - x : p £ x £ 3p
Ô- x : -p £ x £ 0
Ó
5.4  Differential Calculus Booster

If g is the inverse of f, then we can write dx


Now, we shall find ___
​    ​
dy
y = f (x) ¤ g (y) = x
dx
___
dy dy ​   ​  f ¢(t)
So, ​ ___  ​ = f ¢(x) and g¢ (y) ​ ___  ​ = 1
dx
___
​    ​ = ___ dt
​    ​ = ____
​     ​
dx dx dy dy g¢(t)
___
​   ​ 
dy dt
fi g¢ (y) ​ ___  ​ = 1
dx
fi g¢ (y) × f  ¢(x) = 1 11. Differentiation of a function w.r.t another function
1
fi g¢ (y) = ____
​     ​  Let u = f (x) and v = g (x),
f ¢(x)
du
​ ___  ​ f ¢(x)
7. Differentiation of Implicit Functions du dx
then ​ ___  ​ = ___
​   ​ = ____
​    ​ 
.
When y is expressed directly in terms of x, then it is known dy dv g¢(x)
___
​    ​
as explicit function. dx
__
So, when y is not expressed directly in terms of x or For example, if u = sin x and v = ÷
​ x 
   ​
conversely, then it is known as implicit function.
For examples, x3 + y3 = xy, sin (xy) = ex + y, log(xy) = du cosx
then ​ ___  ​ = ____
__
​   ​ = 2​÷x 
  ​  cos x
x + y2 etc. dv 1
____
​  __
   ​ 
d (y) ___dy d (y2) dy 2​÷x 
   ​
Here, we shall use ____ ​ = ​    ​, _____
​     
  ​ = 2y ◊ ​ ___  ​,
​     

dx dx dx dx
12. Higher Order Derivatives
d (y3) dy d (y4) dy
​ _____
   ​ = 3y2 ◊ ​ ___  ​, _____​ = 4y3 ◊ ​ ___  ​ and etc.
​     
  Let y = f (x) be a differentiable function of x.
dx dx dx dx
dy
Then, y¢ = ​ ___  ​ = f ¢(x) is the first derivative or first order
8. Logarithmic Differentiation dx
derivative of y w.r.t x.
We shall use the logarithmic differentiation, only when, if
we get, a function which is the product or quotient of a If y¢ is differentiable, then its derivative

(  )
number of functions or a function is of the form (f (x))g (x) d2y
d d dy
where f and g both are differentiable. y¢¢ = ​ ___  ​  (y¢) = ​ ___  ​ ​  ___
​    ​  ​= ​ ___2 ​ is the second order derivative
dx dx dx dx
9. Differentiation of Infinite Series of y w.r.t x

Here, we shall find out the derivative of an infite series., If y¢¢ is differentiable, then its derivative

(  )
If we take one term from an infinite series, it remains d (y¢¢) ___ 2
d3y
d dy
unchanged. y¢¢¢ = ​ _____
  ​ = ​     ​ ​ ___
  ​  2 ​  ​ = ​ ___3 ​ is the third order derivative
_________________________
___________________
____________
dx dx dx dx
Let ​  f (x) + ÷
y=÷ ​  f (x) +   
÷​  f (x)
   + ... to
  • ​ ​ ​ of y w.r.t x.
_______ Thus, if y is a function of x, then its several derivatives
fi ​  f (x) + y  
y=÷ ​
dy d2y ___ d3y dny
are ___
​    ​, ___
​  2 ​ , ​  3 ​ , ..., ___
​  n  ​
fi y2 = (f (x) + y) dx dx dx dx
dy dy
fi 2y ​ ___  ​ = f ¢(x) + ___
​    ​ i.e. y¢, y¢¢, y¢¢¢, ..., yn
dx dx
dy
fi (2y – 1) ​ ___  ​ = f ¢(x) 13. Differentiation of a Determinant
dx
dy f ¢(x)
fi ​ ___  ​ = _______
​    
 ​ f ( x ) g( x ) h( x )
dx (2y – 1)
If F(x) = l ( x ) m( x ) n( x ) , where f (x),
10. Differentiation of Parametric Functions u( x ) v( x ) w( x )

Let a function y of x be represented by the parametric equa- g (x), h (x), l (x), m (x), n (x), u(x), v(x) and w(x) are all
tions x = f (t) and y = g (t) functions of x and differentiable then F ¢(x)
Differentiation  5.5

f ¢( x ) g ¢( x ) h ¢( x ) Some Standard Results:


= l ( x ) m( x ) n( x )
u( x ) v( x ) w( x )

d n
( np
(i) ​ ___n  ​ {sin(ax + b)} = an ◊ sin ​ ax + b + ___
dx
​   ​   ​, n Œ N
2 )
dn
f ( x ) g( x ) h( x ) (ii) ​ ___n  ​  {cos(ax + b)}
+ l ¢( x ) m ¢( x ) n ¢( x ) dx


u( x ) v( x ) w( x )
( 
np
= an ◊ cos ​ ax + b + ___
​   ​   ,​ n Œ N
2 )
f ( x ) g( x ) h( x ) d n
(iii) ​ ___n  ​  {(ax + b)m}
+ l ( x ) m( x ) n( x ) dx
u ¢( x ) v ¢( x ) w ¢( x )
  m!
= _______
​     ​ ◊ an ◊ (ax + b)m – n, m Œ N
or (m – n)!
f ¢( x ) g( x ) h( x ) f ( x ) g ¢( x ) h( x )
dn
F ¢x = l ¢( x ) m( x ) n( x ) + l ( x ) m ¢( x ) n( x ) (iv) ​ ___n  ​  {log(ax + b)}
dx
u ¢ ( x ) v ( x ) w ( x ) u( x ) v ¢ ( x ) w ( x )
(– 1)n – 1 (n – 1)! n
f ( x ) g( x ) h ¢( x ) = ​ _____________
    ​ ◊ a , n Œ N
  
(ax + b)n
+ l ( x ) m( x ) n ¢ ( x )
dn
u( x ) v( x ) w ¢( x ) (v) ​ ___n  ​  {ax} = (logea)n ax, n Œ N
dx

14. Leibnitz Rules for Differentiation dn


(vi) ​ ___n  ​ {emx} = mn ◊ emx, n Œ N.
dx
If u and v are two functions of x, then
d n
(uv)n = un v + C1 un – 1 v1 + C2 un – 2 v2 (vii) ​ ___n  ​ {eax ◊ sin (bx + c)}
dx
+ C3 un – 3 v3 + ... + Cr un – r vr
= rn ◊ eax ◊ sin (bx + c + nj) n Œ N
+ ... + Cn u vn,
dn
where (viii) ​ ___n  ​  {eax ◊ cos (bx + c)}
dx
dn
n
Cr = Cr, (uv)n, = ___
​  n  ​ (uv)
dx = rn ◊ eax ◊ cos (bx + c + nj), n Œ N,

{ 
d n – 1
(un – 1v) = ​ ​ _____
dxn – 1

 ​   } dv
(u)  ​ ___
​    ​ and so on.
dx
where r = ​÷a  2 + b2 
______
b
 ​, j = tan–1 ​ __
​ a ​  ​ (  )
Exercises
7. Find the derivative of y = f (x) = sin(x2), using first
(Problems Based on Fundamentals) principle.

First principle of differentiation ABC of Differentiation
dy
1. Find the derivative of y = ex, using first principle. 8. If y = logxx + 10, find ___ ​    ​
2. Find the derivative of y = f (x) = logex, using first dx
principle. dy
​ 35​ , find ___
9. If y = 5lo​g​3​x – xlog​ ​    ​
3. Find the derivative of y = f (x) = logax, using first dx
principle. 1 __ 2 3 dy
10. If y = ​ __ __ ___
x ​ + ​ x2  ​  + ​ x3  ​,  find ​ dx  ​
4. Find the derivative of y = f (x) = sin x, using first
principles. __ __ dy
2 __ 3  __ ___
5. Find the derivative of y = f (x) = tan–1 x, using first 11. If y = ​÷x 
  ​  + x​÷x 
  ​  + x ​÷x     ​ + x ​÷x   ​,  find ​    ​
dx
principle of differentiation.
6. Find the derivative of y = f (x) = esinx, using first
principle.
( 
12. If y = ​ 1 + tan ​ __
p
(  ) ) ( 
​   ​  – x  ​  ​ ​ 1 + tan ​ x + __
8 ( p
))
dy
​   ​   ​  ​, find ___
8
​   
dx
5.6  Differential Calculus Booster

sec x + tan x – 1 dy __ 1 dy __ 1
 ​ 13. If y = ______________
​     ​, find ___
   ​    ​ at x = 0 34. If y = ​÷x  ___ , prove that 2x ​ ___  ​ = ÷
  ​  + ​  __  ​  ​ x  ___
   ​ – ​  __  ​. 
sec x – tan x + 1 dx ​÷x 
   ​ dx ​÷x 
  ​ 

( 
dy __ p
x4 + x2 + 1
14. If __________
​  2
x –x+1
   ​ 
dy
such that ​ ___  ​ = ax + b, find the, value
dx
35. If y = ex cos x, prove that ___ ​    ​ = ÷
dx
   ex cos ​ x +
​ 2 ​ 
4 )
​ __ ​   ​.
______
dy
of a + b + 10 36. If y = ÷​ x  2 + a2 
 ​, prove that y ​ ___  ​ – x = 0.
dx
( 
dy p
x
2 ) x 2
15. If y = ​​ sin ​ __  ​ + cos ​ __  ​  ​​ ​, find ___
2
​    ​ at x = __
dx
​   ​ 
6 dy
37. If y = ex + e– x, prove that ___
______
​ y  2 – 4 ​ 
​    ​ = ÷ .
__ __
( ÷  ÷  )
x a dy x a dx
16. If y = ​ ​ __
​ a ​ ​  + ​ __
​ x ​ ​   ​, prove that 2xy ​ ___  ​ = __
​   ​ – __
​   ​

Differentiation of composite functions


dx a x dy
( 
38. If xy = 4, prove that x ​ ​ ___  ​ + y2  ​ = 3y.
dx )
dy 39. Prove that
17. If y = log (sin (3x + 5)), find ___ ​    ​
{  (  ) }
dx ______ ______
d x 2 a2 x
______ ​ ___  ​ ​  __  ​ + __
​    ​ ​÷ a – x2  ​   ​  sin–1 ​ __ ​ a  2 – x2 
​ a ​  ​  ​ = ÷  ​
dy dx 2 2
18. If y = log (x + ÷ ), find ___
​ x  2 + 1 ​  ​    ​
dx
Differentiation by Inverse Trigonometric Function
_____ _____ dy
19. If y = log (​÷x  – 1 ​ – ÷ ), find ___
​ x  + 1 ​  ​    ​ 40. Differentiate the following functions w.r.t. x:
dx _________
x
20. If y = (sin x) , find y = (sin x) . x (i) tan–1 (​÷1  + x2 + x  
​)
_________
dy (ii) tan–1 (​÷1  + x2 – x  
​)
21. If y = log (sin x + cos x), find ___ ​    ​ _____
dx
1
22. If f (x) = _____
​     
d((f (f (x))))
 ​, find __________
​       ​ 
{ 
÷​ 1  + x 2 
_____
 ​ – 1
(iii) tan  ​ ​ __________
–1
x    ​  ​ x π 0 }
x–1 dx

x–1
23. If f (x) = ​ _____ 
d(f (f (f (x))))
 ​, find __________
​        ​
(iv) tan  ​ ​  { ÷ 
1 + x2 
 ​ + 1
–1 ​__________

______
x     ​  ​x π 0 }
x+1 dx
(v) cot–1 (​÷1  + x2 ​ 
+ x)

{ 
1
24. Let f be a function for which f ¢(x) = _____ _______ _______

}
​  2    
 ​ ​÷1  + sinx  
​+ ÷ ​ 1  – sinx  ​ 
–1 ____________________
x +1 (vi) tan  ​​  _______         _______ ​  ​, 0 <x<p
​÷1  + sinx     –  sinx ​ 
​ – ​÷1  
If g (x) = f (3x – 1), find g¢(x).
41. Differentiate the following functions w.r.t. x:
25. Let f be a function for which f ¢(x) = x2 + 1
dy
If y = f (sin(x3)), find ___
​    ​.

a+x
( 
(i) tan–1 ​ ​ ______ 
1 – ax
 ​  
​ )
{ 
dx
26. If f (x) = |x – 1| + |x – 3|, find f ¢(2)

a cos x – b sin x
(ii) tan–1 ​ _____________
​ 
a cos x + b sin x
p
2 } p
   ​  ​, – ​ __ ​  < x < __
​   ​ 
2
3
27. If f (x) = |x2 – 1| + |x2 – 4|, find the value of f ¢​ __
​   ​   ​.
2 (  ) ( 
3a2x – x3
(iii) tan–1 ​ ​ ________
a3 – 3ax2 ) 1__ __
 ​  ​, – ​ ___

x 1
  ​ < ​   ​ < ___
    a ​÷3 ​
​÷3 ​
​  __  ​ 
  
28. If f (x2) = x4 + x3 + 1, find f ¢(x4) _____

29. If g is the inverse of f and f ¢(x) = cos 2x, find


( ÷ 
a–x
(iv) tan–1 ​ ​ ​ _____ 
a+x )
 ​ ​  
​, – a < x < a
g¢(x).
30. If f (x) = x + tan x and f is the inverse of g, find
(  ​ 
sin x
(v) tan–1 ​ ________
1 + cos x
   )  ​  ​, – p < x < p.

(vi) tan  ​( _________


b – a tan x )
g¢(x). –1a + b tan x
​     ​  ​
4
x –x +1 2 dy
31. If y = ​ ___________
__      ​ such that ​ ___  ​ = px + q then prove
(vii) tan  ​( ______
b – ax )
2
  x  + 1
x + ​÷3 ​ dx –1a + bx
​   ​   ​
5p
that p – q = tan ​ ___
12 (  )
​   ​   ​
_____
(viii) tan  ​  ______
( ​ 1 +x 6x   ​ )​
–1
2
_____ _____ dy 1
, prove that ​÷x  2  – 1 ​ ___
32. If y = ​÷x  – 1 ​ + ​÷x  + 1 ​  ​    ​ = ​ __ ​  y.

( 
dx 2
x
33. If y = _____
​     
x+2
dy
 ​, prove that x ​ ___  ​ = (1 – y)y.
dx

sin x  +__  cos x
(ix) sin–1 ​ __________
​ 
÷   
​ 2 ​
 ​ 
     )
3p
4
3p
​, – ​ ___ ​ < x < ___
​   ​ 
4
Differentiation  5.7

​ 
1+x(  )
1 – x2
(x) sin–1 ​ ______
2

1 + x2
 ​  ​ + sec–1 ​​ ______2 
1–x (  )
​, x Œ R
 ​  
1
2
dy
​ __ ​  < x <1, find ___
​    ​.
dx
_____ ______
dy
(  ) (  )
x–1
(xi) sin  ​ _____
–1
​ 
x+1
x–1
 ​  ​ + cos–1 ​ _____
​ 
x+1
  ​  ​  ​), find ___
– ​÷x  – x3 
52. If y = sin–1 (x ​÷1  – x ​  ​    ​
dx

(xii) sin  ​( _____________


53. If y = f (x) = x3 + x5 + x7 and g is the inverse of f,
–13sin x + 4cos x
​ 
5
 ​ 
     )​ then find g¢(3)

(xiii) If y = tan  ​( ​ _______


54. If y = f (x) = x5 + 2x3 + 2x and g is the inverse of f,

1 – cos x
–1
sin x
    )
​  
dy
​, find ___
​    ​
dx find g¢ (– 5)
55. If f (x) = x3 + 2x2 + 3x + 4 and g(x) is the inverse
( 
dy
​ 
cos x
(xiv) If y = tan  ​ _______
–1
 ​  ​, find ___
  
1 – sin x
​    ​
dx ) of f (x), find g¢ (4).
1
56. If f and g are inverse of each other and f ¢(x) = ​ _____
   ​
2x
42. If y = sin–1 ​ _____
​   2 
1+x (  ) 1 – x2
 ​  ​ + cos–1 ​ ​ ______2 
1+x

 ​  ​, (  ) find g¢(x).
1 + xn

dy Differentiation of Implicit Functions


4
0 < x < 1, prove that, ___
​    ​ = _____
​      ​. dy
dx 1 + x2 57. Find ​ ___  ​, if 2x2 + 3xy + 3y2 = 1

(  ( 
dx
x
43. If y = sin–1 ​ _______
​  _____
   
÷​ 1  + x  
2
 ​ ) 1
 ​  ​ + cos–1 ​ _______
​  _____
   
 ​  ​,
​÷x  + 1 ​
2

) 58. If e + ey = ex + y, prove that, ___
x dy
​    ​ + ey – x = 0
dx
_____ _____ dy 1
dy
0 < x < •, prove that, ___
2
​    ​ = _____
​      ​ 59. If x ​÷1  + y   = 0, prove that ___
​ + y ​÷1  + x ​  ​    ​ = – ​ _______
    ​
dx 1 + x2 dx (1 + x)2

{ 
______ ______
+ x ​÷1  – y2 ​ 
_____ _____
÷​ 1 _____
+ x 
44. If y = tan–1 ​​  _________________
​ 1  – x  
 ​ + ÷ 2
 ​ dy
    ​  ​, find ___
2

} 60. If y ​÷1  – x2 ​  = 1 prove that

÷ 
  
______ ​    ​. ______
÷​ 1  + x2  ​ 1  – x2 
 ​ – ÷  ​ dx dy 1 – y2
​ ___  ​ = – ​ ​ _____2 
 ​ ​

{ 
_______ _______ dx
}
1–x
​÷1  + sin x  ​+ ÷​ 1  – sin x ​

45. If y = cot–1 ​ ____________________
​  _______         _______  ​  ​,
dy sin2 (a + y)
​÷1  + sin x  
​ – ​÷1  – sin x  
​ 61. If sin y = x sin (a + y), prove that ___
​    ​ = __________
​     ​ 
dx sin a
dy
then prove that ___ ​    ​ is independent of x. 62. If log (x + y) = 2xy, then prove that y¢ (0) = 1.
dx

(  ) (  )
______ ______
2x 1 + x2 +÷
​ 1  – x6 ​ 
63. If ÷ ​ 1  – y6 ​ 
46. If y = tan–1 ​ _____
​   2   ​  ​ + sec–1 ​ ​ ______2 
 ​  ​, x > 0, = a (x3 – y3) prove that

÷ 
1–x 1–x ______
dy dy x2 ______ 1 – y6
then prove that ___ ​    ​ = _____
4
​       ​. ​ ___  ​ = __
​  2 ​  ​ ​   
 ​ ​
dx 1 + x2 dx y 1 – x6

x–1
x+1 ( 
47. If y = sec–1 ​ ​ _____  )1 + x2
 ​  ​ + sin–1 ​ _____
​ 
1 – x2 (  )
 ​  ​, x > 0, 64. If x2 + y2 = t – __
dy
1 1
​   ​  and x4 + y4 = t2 + __
t
​  2  ​  then prove
t
1
dy
prove that ___
​    ​ = 0. that ___
​    ​ = ___
​    ​. 
dx dx x3y

[  { ÷  } ]
_____

( 
dy
1–x
48. If y = sin ​ 2tan  ​ ​ _____
​ 
1+x
 ​  ​, find ___
–1
 ​ ​ 
  ​    ​.
dx
_______
x+y
65. If sec ​ ​ _____  
x – y  ) dy y
​  ​ = a, prove that ___
​    ​ = __
dx x
​   .​

(  )
49. If y = cos–1 (2x) + 2cos–1 (​÷1  – 4x2   ​),
x2 – y2 dy x (1 – tan a)
1 dy 66. If tan–1 ​ ______
​  2  ​  ​ = a prove that ___
  ​    ​ = ___________
​     ​.
0 < x < __ ​   ​ , find ___
​    ​. x +y 2 dx y (1 + tan a)
2 dx

( 
_____ _____
​ 1  + x  
÷
–1 _______________
50. If y = tan  ​ ​  _____
​ 1  + x  
÷
​– ÷
​+ ÷
​ 1  – x ​
  

   ​  ​,
_____
​ 1  – x   ​ ) dy
67. If xy = 1, prove that ___
​    ​ + y2 = 0
dx
dy
find ___ ​    ​. dy
68. If y = x sin y, prove that ___
y
​    ​ = ___________
​       ​.
dx
( 
______

)
dx x (1 – x cos y)
x ÷
​ 3  – 3x2   ​
51. If y = cos–1 x + cos–1 ​ __ ​    ​ + ​ _______ ​    ​
2 2 69. If cos y = x cos (a + y) with cos a π ± 1,
5.8  Differential Calculus Booster
____________________
____________
_____
dy cos2(a + y)  
88. If y = ​÷sin x +÷      
​ sin x ​ ​ ​ + ... to •, prove that
+ ​÷sin x 
    
prove that ___
​    ​ = __________
​      ​ 
dx sin a dy cosx
​ ___  ​ = ______
​    
 ​
Logarithmic Differentiation dx 2y – 1
dy  x+ ex + ... to • dy y
70. Find ___
​    ​, if y = xsin x 89. If y = e​ x +​ 
​ e​ ​, prove that ___

​    ​ = _____
​      ​
dx dx 1 – y
dy dy p
71. Find ___
​    ​, if y = (sin x)cosx 90. If y = (tan​)(tan)
​ ​, prove that ___
(tan) ... to x
​    ​ at x = __
​   ​ .
dx dx 4
dy 1 1 ___ 1
72. Find ​ ___  ​, if y = (tan x)cot x + (cot x)tan x 91. If y = x + ___
​    ​ ___
​    ​ ​    ​ ... to •, then prove that,
dx x + x + x +
dy y dy y
73. If xm yn = (x + y)m + n, prove that, ___ ​    ​ = __
​ x ​ ​ ___  ​ = _____
​       ​.
dx dx 2y – x

( 
​ x ​  ​ ​ ​ + x​ ​(​ 1 + ​ x ​ )​​, find ___
1 x
74. If y = ​​ 1 + __ ) 1
__ dy
​    ​ at x = 1
dx
sinx cosx ____
92. If y = ____
​   ​  ◊ ​ ____ ​  
1+ 1+ 1 + 1 +
sinx cosx
◊ ​   ​  ◊ ​ ____ ​ ... to •,

1
75. If y = sin x ◊ sin2x ◊ sin3x ... sin (2014) x, find ___
dy
​    ​. then prove that, y¢(0) = __ ​   ​ .
2
dx
x x x x dy
dy log x 93. If y = ___
​    ​  ◊ ​ ___  ​  ◊ ​ ___  ​  ◊ ​ ___  ​ ... to •, then find ___
​    ​.
76. If xy = ex – y, prove that, ___ ​    ​ = _________
​    
 ​. 1+ 2+ 1+ 2+ dx
dx (1 + logx)2
dy Differentiation of Parametric Functions
77. If xy = yx, find ___
​    ​. dy
dx
94. Find ___
​    ​, when
dy (logy)2 dx
78. If ey = yx, prove that ___ ​    ​ = _______
​   

. ​
dx logy – 1 x = a (t – sin t), y = a (l – cos t).
1 1
dy
79. If xm y n = 1, prove that, ___
m y
​    ​ = – ​ ___ 95. If x2 – y2 = t – __ ​   ​  and x4 + y4 = t2 + __ ​  2  ​ 
dx n x ​  t t
dy
dy 1 – x then prove that x3y ___ ​    ​ + 1 = 0
80. If ex + y – x = 0, prove that ___​    ​ = ​ _____
x   ​.  dx

(  ) (  )
dx 1 1
96. If x = a ​ t + __
​   ​   ​ and y = a ​ t – __ ​   ​   ​,
dy t t
x2 – 1
81. Find ___
​    ​, if y = xsin x – cos x + ______
​  2  
 ​. dy x
dx x +1 then prove that ___ ​    ​ = __ ​   ​.
dx y

( 
82. Find y¢ (0), if _____

y = (x + 1) (x + 2) (x + 3) ... (x + 2012)


5t + 12​÷1  – t2 ​ 
97. If y = cos  ​ _____________
​  –1
13
 ​      ​ )
83. Find y¢ (0), if
y = (1 + x) (1 + x2) (1 + x4) (1 + x8) ... (1 + x1006)
1 – t2
and x = cos–1 ​ ​ _____2 
1+t
 ​  
​, find (  ) dy
___
​    ​.
dx
Differentiation of infinite series

84. If y =
____________________________

​÷cos x +÷
____________________
      
​ cos x +÷
_____________
​ cos x
      to • ​ ​
+ ...   
5 ( 
3sin t + 4cos t
98. If x = sin–1 ​ ____________
​   ​ 
     ​ )
dy
then find ___
​    ​.
6cos t + 8sin t
and y = sin–1 ​ ____________
​ 
10
 ​ 
     ( 
​, find ) dy
___
​    ​.
dx
dx
dy 1
sin x cos x ____
sin x cos x dy 99. If x = cos t, and y = sin t, prove that ___
​    ​ = ___
​  __  ​ at
85. If y = ____
​   ​ ____  ​   ​ ____
​   ​  ​   ​  ..., find ___
​    ​ dx ​÷3 ​   
1+ 1+ 1+ 1+ dx 2p
_______________
__________
___
x = ​   ​ .
______
86. If y = ​÷x  + ​÷x  + ​÷x    
+   to •, prove that
... ​ ​ ​  3
dy ylogx
dy 1 100. If x = ecos2t and y = esin2t, prove that ___
​    ​ = _____
​   ​ 

​ ___  ​ = ______
​       ​. dx xlog y
dx 2y – 1
101. If x = x = 2cosq – cos(2q) and
dy y2
, prove that ___
​    ​ = ___________
x...to •

87. If y = x​x​ ​       ​. y = 2sinq – sin2q, prove that
dx x(1 – ylog x)
Differentiation  5.9

dy
dx
3q
​ ___  ​ = tan ​ ___
​   ​   ​
2 (  ) 116. If y = e2x, find ​ ___ (  ) (  )
d2y d2x
​  2 ​  ​ ​ ___
dx dy2
​   ​  ​.

(  )
1
102. If x = a ​ t + __
t
1
​   ​   ​ and y = a ​ t – __
t (  )
​   ​   ​, x
__ d2y dy
(  2
)
117. If (a + bx)​e​y ​ = x, then prove that x3 ​ ___2 ​ = ​​ x ​ ___  ​ – y  ​​ ​.
​   ​

dy x dx dx
then prove that ___ ​    ​ = __
​   ​ 1
dx y 118. If x2 + y2 = a2 and k = __ ​ a ​, then prove

1+t
2t
(  )
103. If x = sin–1 ​ _____
​   2 
2t
 ​  ​ and y = tan–1 ​ _____
​   2 
1–t
 ​  ​,
(  ) that k = _________
|y¢¢|
​  ________
  + y¢2)3 
÷​ (1
  
 ​.
 ​
dx
t > 1, then prove that ___ ​    ​ = –1 119. If y = eax sin bx, prove that, y2 – 2ay1 + (a2 + b2) y = 0
dy

( 
_____ d2y
5t + 12​÷1  – t2 
104. If y = cos  ​ _____________
​  –1
13
 ​     
 ​
​ ) 120. If y = 2 sin x + 3 cos x, prove that, ___
​  2 ​ + y = 0
dx

(  )
d2y
1 – t2 dy 121. If y = x + tan x, prove that cos2x ___
​  2 ​ – 2y + 2x = 0
and x = cos  ​ ​ _____2 
 ​  ​, find ___
–1
​    ​. dx
1+t dx
d2y
( 
cos x
122. If y = tan x + sec x, prove that ___
​  2 ​ = _________
3sin t + 4cos t
105. If x = sin–1 ​ ____________
​ 
5
 ​ 
     ​ ) dx
​    
(1 – sin x)2
 ​

123. If y = A cos (log x) + B sin (log x), then prove that


6cos t + 8sin t
and y = sin  ​ ____________
​ 
10
–1
 ​ 
     ( 
​, find ) dy
___
​    ​.
dx d2y dy
x2 ​ ___2 ​ + x ​ ___  ​ + y = 0

( 
_______ ________ dx dx
​ 1  + sin t  
÷
_______    
​ 1  + sin t  
÷
​ 1  – sin t ​ 
 ​+ ÷
106. If x = tan–1 ​ ​ ___________________
   
​– ÷
_______  ​  ​
​ 1  – sin t  
​ ) 124. If y = tan–1 x, prove that (1 + x2) y2 + 2xy1 = 0
125. If y = ex (sin x + cos x), prove that y2 – 2y1 + 2y = 0
______
÷​ 1  + t   ​  
and y = tan–1 ​ ​ __________
t
–1
( 
   ​  
​, find
2
) dy
___
​    ​.
dx
126. If y = sin–1 x, prove that (1 – x2) y2 – xy1 = 0
d2y
Differentiation of a function with respect to another 127. Find ​ ___2 ​ , if
dx
  function
(i) x = at2, y = 2at
107. Differentiate f (x2 + 2012). w.r.t f (x3 + 2013).
(ii) x = a cos3q, y = a sin3q
108. Differentiate xx w.r.t. x log x.

{ 
_____ _____ (iii) x = a cosq, y = b sinq
109. Differentiate tan–1
÷​ 1  + x2  ​ – ​÷1  – x2 
​ ________________
​  _____
÷​ 1  + x  
2
 ​ + ÷
  
 ​
   ​  ​
_____
​ 1  – x  
2
 ​
} d2y
128. If x = a sec q, y = b tan q, prove that ___
dx
b4
​  2 ​ = – ​ ____
a2y3
  ​. 

w.r.t. cos–1 x2. 129. If y = sin–1 x, prove that (1 – x2) y2 – xy1 = 0


2x
110. Differentiate sin–1 ​ _____
​   2 
1+x (  ) 1 – x2
 ​  ​ w.r.t. cos–1 ​ ​ ______2 
1+x
 ​  ​ (  ) d2y
130. Find ___
​  2 ​ , if
dx
(i) x = at2, y = 2at
111. Differentiate f (sin x) w. r.t. f (cos x)
(ii) x = a cos3q, y = a sin3q
Higher Order Derivatives
(iii) x = a cosq, y = b sinq
d2y
112. If y = c1ex + c2e–x, then prove that ___
​  2 ​ – y = 0. d2y b4
dx 131. If x = a secq, y = b tanq prove that ___
​  2 ​ = – ​ ____  ​. 
dx a2y3
d2y
113. If x = at2, y = 2at, find ___
​  2 ​  132. If x = a (q + sinq), y = a (1 + cos q)
dx
d2y a
d2y prove that ___ ​  2 ​ = – ​ __2  ​ 
114. If x = a cosq, y = b sin q, find ___
​  2 ​  dx y
dx

(  ) (  )
133. If x = a(1 – cos q), y = a(q + sinq), prove that
d2y d2y dy –3
115. Prove that ___
​  2 ​ = –  ​ ___
​  2 ​  ​ ​​ ___
​    ​  ​​ ​ d2y 1 p
dx dx dx ​ ___2 ​ = – ​ __ __
a ​ at q = ​ 2 ​ .
dx
5.10  Differential Calculus Booster

134. If y = x log ​ ______( 


x
​     
a + bx )
d2y dy
(  )
 ​  ​, prove that x3 ​ ___2 ​ = ​​ x ​ ___  ​ – y  ​​ ​.
dx dx
2 Then prove that f (x) is a linear polynomial of x.

1 2 x 3x2
2
_____ _____ dy 2 f (x) = x x 2 x 2
135. If ÷​ y  + x ​ 
+÷ ​ y  – x ​  = c, then prove that ___ ​  2 ​ = __ ​  2  ​  147. Let
dx c 0 2 6x
cos4a _____ sin4a _____ 1 dy
136. If _____
​  x    ​ + ​  y   
  ​ = ​       ​, then prove that ___ ​    ​ = tan2a. Then find the value of f ¢(1).
x+y dx
137. If y = x sin x, then prove that, cos x sin x cos x
d2y dy 148. Let f (x) = cos2 x sin 2 x 2 cos2 x
x2 ​ ___2 ​ – 2x ​ ___  ​ + (x2 + 2) y = 0.
dx dx cos3 x sin 3 x 2 cos3 x
138. If f : R Æ R be a function is defined by p
Then find the value of f ¢​ __
​   ​   ​. (  )
(  ) (  ) 2
2 2
dy dy 1
y = f (x) = x , then prove that, ​ ___
2
​  2 ​  ​ × ​ ___
​  2 ​  ​ = – ​ ___ 3 ​ .
dx dx 2x Leibnitz Rules for Differentiation
139. There is a polynomial
P (x) = ax3 + bx2 + cx + d such that 149. If y = ex, find the value of yn(0).
x
P (0) = P (1) = – 2, P¢(0) = – 1, then find the value 150. If y = a , find the value of yn(0).
of a + b + c + d + 10. 1
151. If y = ______
​       ​, find the value of yn(0).
140. If f (x) = x + tan x and g is the inverse of f, then (x + 1)
1 152. If y = sin x, find the value of yn(0).
prove that g¢ (x) = ____________
​       ​.
2
2 + tan (g (x)) 153. If y = xn, find the value of yn(1).
154. If y = A sin x + B cos x, then prove that
141. If g is the inverse of f such that f (x) = ex + x3 – 1,
e+6 (i) y2 + y = 0
then prove that g¢¢(e) = – ​ _______   
 ​
.
(e + 3)3 (ii) yn + 2 + yn = 0.
–1
142. Let f (x) = 1 + x3. If g (x) = f –1(x), then prove that 155. If y = tan  x, then prove that
8 (i) (1 + x2) y2 + 2xy1 = 0
g¢¢¢(2) = __​   ​ .
3 (ii) (1 + x2) yn + 2 + 2(n + 2)xyn + 1 + n (n + 1)yn
143. Suppose f is a differentiable function such that = 0.
2
f (g (x)) = x and f ¢(x) = 1 + ( f (x)) , then prove that 156. If y = sin–1 x, then prove that
1 (i) (1 – x2) y2 – xy1 = 0
g¢(x) = _____
​    2   ​.
1+x (ii) (1 – x2) y – (2n + 1)xy – n2y = 0
n + 2 n + 1 n
Differentiation of a determinant
157. If y = a cos (log x) + b sin(log x), then prove that
x3 sin x cos x x2 ◊ yn + 2 + (2n + 3) x ◊ yn + 1 + (n2 + 1)yn = 0.
144. Let f (x) = 6 -1 0 , where p is a constant
p p2 p3 (Mixed Problems)
3 ______ dy
d
then prove that ___ p
​  3  ​ ( f (x)) at x = 0 is 0. 1. If y = cot–1 (​÷cos 2x 
  ​), then the value of ​ ___  ​ at x = __
  ​   ​ 
dx dx 6
145. Find the coefficient of x in the determinant will be

(1 + x )a1b1 (1 + x )a1b2 (1 + x )a1b3 (  )


2 1/2
(a) ​​ ​ __ ​   ​​ ​
3 3(  )
1 1/2
(b) ​​ ​ __ ​   ​​ ​

(1 + x )a2b1 (1 + x )a2b2 (1 + x )a2b3 (c) (3)1/2 (d) (6)1/2

÷ 
_____
(1 + x )a3b1 (1 + x )a3b2 (1 + x )a3b3 1+x dy
2. If y = ​ ​ _____ 
  ​ ​, then ___
​    ​ =
1–x dx
with the help of determinant.
2
(a) ​ ________________
     ​
x +1 x+2 x+3 (1 + x)1/2  (1 – x)3/2
146. Let f (x) = x + 4 x+5 x+6 1
(b) ​ ________________
     ​
x+7 x+8 x+9 (1 + x) (1 – x)3/2
1/2
Differentiation  5.11

1 p dy
(c) ​ _________________ 10. y = (tan x)(tan x​)​ ​, then at x = __ ___
tan x
       ​ ​   ​ , the value of ​    ​ =
2(1 + x) (1 – x)3/2
1/2 4 dx
(a) 0 (b) 1
2
(d) ​ ________________
     ​ (c) 2 (d) None of these
(1 + x) (1 – x)1/2
3/2
_____
d
_____________ 11. If x = sin–1 (3t – 4t3) and y = cos–1 (​÷1  – t2 
 ​)
3. ​ ___  ​  (​÷sec
  2x + cosec
  
2
x ​) =
dx dy
then ___ ​    ​ =
dx
(a) 4 cosec 2x ◊ cot 2x (b) – 4 cosec 2x ◊ cot 2x
(a) 1/2 (b) 2/5
(c) – 4 cosec x ◊ cot 2x (d) None of these
(c) 3/2 (d) 1/3
d
4. ​ ___  ​  (lo​g​​÷__x   ​ ​ (1/x)) = x + e x + ... • dx
dx ​ e​
12. If y = e​ x + ​ ​ then ___

​    ​ =
dy
1 y
(a) – ​ ____    ​  
__ (b) – 2 (a) ​ _____
    ​
1
(b) ​ _____
   ​
2​÷x     ​ 1–y 1–y
1 y y
(c) – ​ _____
2 __
   ​   (d) 0 (c) ​ _____
    ​ (d) ​ _____
    ​
x  ​÷x     ​ 1+y y–1

(  )
_______
1 – x2

{ 
5. If y = log  x____________
​ ​        
​  } dy
  2 + x2) ​ ​, then the value of ___
​ (a ​    ​ is
dx
13. The derivative of cos–1 ​ ​ ______2 
1+x

 ​  ​ w.r.t.

( 
a

______
(a) ​÷a  2 – x2 
 ​
______
(b) a ​÷a  2 + x2 
______
1 – 3x2
cot–1 ​ ​ ______2 
3x – x
 ​  ​ is )
1 3
 ​ (c) ​ _______
______   
 ​ (d) x ​÷a  2 + x2 
 ​ (a) 1 (b) ​ __ ​ 
2
÷​ a  + x2 
2
 ​
2 1
( 
__ (c) ​ __ ​   (d) ​ __ ​ 
)
​ x 
÷    ​ – x
6. If y = tan–1 ​ ​ ______   
 ​  ​, then y¢(1) is 3 2
1 + x3/2
( 
d 2y
(a) 0
1
(b) ​ __ ​ 
2
x
14. If y = x log ​ ______
​     
a + bx dx
)
 ​  ​, then x3 ​ ___2 ​ =

(c) –1
1
(d) – ​ __ ​ 
4
dy
(a) x ​ ___  ​ – y
dx (  dy
)
2
(b) ​​ x ​ ___  ​ – y  ​​ ​
dx


2x
7. If y = sin–1 ​ _____
​   2 
1+x (  ) 1+x
 ​  ​ + sec–1 ​ ______
​ 
1 – x2
  (  )
 ​  ​, then
2 dy
___
​    ​ =
dx
dy
(c) y ​ ___  ​ – x
dx (  dy
)
2
(d) ​​ y ​ ___  ​ – x  ​​ ​
dx
4 1 15. If x = et sin t, y = et cos t, t is a parameter,
(a) ​ _____
    ​ (b) ​ _____
    ​
1 – x2 1 + x2 d2y
then ___
​  2 ​ at (1, 1) is equal to
4 – 4
(c) ​ _____
    ​ (d) ​ _____  
 ​ dx
1 + x2 1 + x2 (a) – 1/2 (b) – 1/4

( 
__ __

​ a 
÷    ​ – ÷
8. If y = tan–1 ​ ​ ________
1+÷
​ x 
  ​ 
___ 
​ ax 
   
 ​  
​ )
dx
​, then ___
​    ​ =
dy
(c) 0
_______ _______
(d) 1/2
dy
1 1 16. If ÷   – x6) ​ 
​ (1 +÷   – y6) ​ 
​ (1 = a3(x3 – y3), then ___
​    ​ =
(a) ​ __________
    __ ​ (b) ​ _________  __ ​  dx

÷  ÷ 
2(1 + x) ​÷x 
   ​ (1 + x)​÷x   ​  ______ ______
x2 1 – x6 y2 1 – y6

1
(c) – ​ ________
  __  ​ (d) None of these (a) ​ __2 ​  ​ ​ _____6 

 ​ ​ (b) ​ __2 ​ ​  ​ ______6 
 ​ ​ 

(1 + x)​÷x 
   ​ y 1–y x 1–x

÷ 
______
dy 6
9. If yx + xy = ab, then ___ ​    ​ = x2 1 – y
dx (c) ​ __2 ​  ​ ​ ______6 
 ​ ​ (d) None of these
y – 1 x y 1–x
yx + y logy yxy – 1 + yxlogy
(a) – ​ ____________    ​
  _____________
(b) ​    
   17. If x = f (m) cos m – f ¢(m) sin m and
xyx – 1 + xylogx xyx – 1 + xylogx
y = f (m) sin m + f ¢(m) cos m, then

(  ) (  )
yxy – 1 + yx yxy – 1 + yx
 ​ (c) – _________
​  x – 1  ​   (d) ​ _________  ​  dy 2 dx 2
xy + xy xyx – 1 + xy ​​ ___
​    ​  ​​ ​ + ​​ ___
​    ​  ​​ ​ =
dm dm
5.12  Differential Calculus Booster

(a) [f (m) + f ¢¢(m)]2 (b) [f (m) – f¢¢(m)]2 dy


24. If y = (x log x)log log x, then ___
​    ​ =
dx
{f (m)}2
(c) {f (m)}2+ {f ¢¢(m)}2 (d) ​ _______2  ​  1
{f ¢(m)} (a) (x log x)log log x ______
​     ​  (log x + log (log x))
x log x
1 – (logx)2
18. If f (x) = cos–1 ​ __________
​ 
[  ]
   ​  ​, then the value of f ¢(e) =
1 + (logx)2
1 ______
+ (log(log x)) ​ ​ __
1
x ​ + ​ x log x ( 
   ​  
​ )
(a) 1 (b) ​ __
1
e ​

2
(b) (x log x)x log x log (log x) ​ ____
​      
log x
​+
[  1
__
]
​ x ​  ​

[ 
2 2
(c) ​ __
e ​ (d) ​ __2  ​ 
e
log (log x) ____
(c) (x log x)x log x  ​ _________
x    ​ 
1
​ ​      
log x
​ + 1  ​
]
x dy (d) None of these
19. If y = , then ___
​    ​ =
x dx
a+ 25. If f ¢¢(x) = – f (x), where f (x) is a continuous double
x
b+ differentiable function and g (x) = f ¢(x).
x
a+

b
b + ...
b
If (  (  ) ) (  (  ) )
2
x 2
F (x) = ​​ f  ​ __
x 2
​    ​  ​  ​​ ​ + ​​ g ​ __
​    ​  ​  ​​ ​ and
2
(a) ​ ________
    ​ (b) ​ _______
   ​  F (5) = 5, then F (10) is :
a (b + 2y) b + 2y
a (a) 0 (b) 5
(c) ​ ________
    ​ (d) None of these (c) 10 (d) 25
b (b + 2y) ______
÷​ 1  + x 2 ​  
20. The derivative of tan–1 ​ ​ __________
–  1
x    ​   ( 
​ with respect to ) 26. If x2 + y2 = 1, then:
(a) yy¢¢ – 2 (y¢)2 + 1 = 0

( 
_____
2x ​÷1  – x  
tan–1 ​ _________
​ 
1 – 2x
  
2
 ​
 ​  
2
​ at x = 0, is)

(b) yy¢¢ + (y¢)2 + 1 = 0
(c) yy¢¢ + (y¢)2 – 1 = 0
1 1
(a) ​ __ ​   (b) ​ __ ​ 
8 4 (d) yy¢¢ + 2 (y¢)2 + 1 = 0
1
(c) ​ __ ​   (d) 1 x3 sin x cos x
2
__ __
1 1 27. Let f (x) = 6 −1 0 where p is constant.
​ ÷x   ​ ​, then xy2 + __
21. If y = e​ ​÷​ x   ​ ​ + e​ – ​ ​   ​  y1 – __
​    ​ y =
2 4 p p 2
p 3

(a) 0 (b) 1
d3
(c) – 1 (d) 2 Then ___​  3  ​ ( f (x)) at x = 0 is
dx
x3 x2 3x2 (a) p (b) p + p2
3
22. f (x) = 1 − 6 4 , here p is a constant, (c) p + p (d) Independent of p
p p 2 p3 28. There exists a function f (x) satisfying
f (0) = 1, f ¢(0) = – 1, f (x) > 0 for all x and
d3
then ___
​  3  ​  ( f (x)) is (a) f ¢¢(x) < 0 for all x
dx (b) – 1 < f ¢¢(x) < 0 for all x
(a) Proportional to x2 (b) Proportional to x
(c) – 2 £ f ¢¢(x) £ –1 for all x
(c) Proportional to x3 (d) A constant
(d) f ¢¢(x) < –2 for all x
1
23. If x + y = a and k = __
2 2 2
​ a ​ then k is equal to 29. If y2 = P (x), a polynomial of degree 3, then


y¢¢
(a) ​ ______
_____   ​
÷​ 1  + y¢ ​ 
|y¢¢|
(b) ​ _________
________   
  + y¢2)3 
​÷(1
 ​
 ​
dx
d
(  )d2y
2 ​ ___  ​​  y3 ___
​  2 
dx
 ​  ​ =

2y¢¢ y¢¢ (a) P¢¢¢(x) + P¢¢ (x) (b) P¢¢(x). P¢¢¢(x)


(c) ​ ________
______  ​   (d) ​ ___________
    ​
________
÷​ 1  + y¢2 
 ​   + y¢2)3 
2​÷(1  ​ (c) P(x) P¢¢¢(x) (d) A constant
Differentiation  5.13

30. If y is a function of x and log(x + y) = 2xy, then the (a) 0 (b) 2


value of y¢ (0) is equal to (c) 1 (d) – 1
(a) 1 (b) – 1 41. The differential co-efficient of the function
(c) 2 (d) 0 f (x) = |x – 1| + |x – 3| at x = 2 is
31. If f (x) = (logtan x (cot x))(log cot x tan x)–1, then the value (a) 1 (b) 3
of f ¢(1) is (c) 0 (d) not defined


(a) 0
p
__
(c) ​   ​  
(b) 1
(d) p/2.
1
42. Let f (x) = max ​ sin x, __ {  }
​    ​, cos x  ​. Then the value of
2
4
32. Let f (x) = |x| + |x – 2|, then the value of f ¢(1) is (  )
p
__
f ¢​ ​   ​   ​ is
4
(a) 2 (b) 0 (a) 1 (b) 0
(c) 1 (d) not exist (c) –1 (d) not defined
dy
(  )
33. If ex + ey = ex + y, then the value of ​​ ​ ___  ​  ​​ ​ is
dx /(1, 1)
dy
43. If xm ◊ yn = (x + y)m + n, then ___
​    ​ is
dx
(a) 0 (b) 1 1 1 1 __ 1
(a) ​ __ __
x ​ + ​ y ​ (b) ​ __
x ​ – ​ y ​
(c) –1 (d) not defined
y 1 __ 1
( 
x–1
34. If y = f  ​ ​ _____ 
x+1 ) dy
 ​  ​ and f ¢(x) = x2 ​ ___  ​ at x = 0 is
dx
(c) ​ __x ​ (d) ​ __
x ​ – ​ y ​
44. If y is a function of x and log(x + y) = 2xy, then the
(a) 1 (b) 0 value of y¢(0) is
(c) 2 (d) – 2 (a) 1 (b) –1
35. Let f (x) = cos x ◊ cos2x ◊ cos4x ◊ cos8x, (c) 2 (d) 0

(  )
2 2
p 45. Let f (x) = (x – 1)|x – 3x + 2| + cos(|x|). Then the
Then the value of f ¢​ ​ __ ​   ​ is
4 value of f ¢(1) is
(a) 1 (b) 0__ (a) 0 (b) 1
(c) 2 (d) ​÷ 2 ​  (c) –1 (d) not defined
36. If x = secq – tanq and y = secnq – tannq such that 46. Let y = y(x) and it follows the relations x cos y +

(  )
dy 2 y cos x = p, then y¢¢(0) is
(x2 + A) ​​ ​ ___ ​  ​​ ​ = n2 (y2 + B), then the value of (a) 1 (b) –1
dx
a + b + 2 is (c) p (d) – p
(a) 4 (b) 6 47. If xexy = y + sin2x, then the value of y¢(0) is
(c) 8 (d) 10 (a) –1 (b) 2
37. Let f (x) = logx (logex), then f ¢(e) is (c) 0 (d) 1


(a) e
(c) 1/e
(b) – e
(d) 1/e2.
2x
–1 _____
48. Let y = sin  ​ ​   2 
1+x
 ​  ​.
(  )
3 2  Then the value of y¢(1) is
38. Let f (x) = x + x f ¢(1) + x f ¢¢(2) + f ¢¢¢(3) for all x
p p
in R, where f ¢(1) = a, f ¢¢(2) = b, f ¢¢¢(3) = c then the (a) ​ __ ​   (b) ​ __ ​ 
2 4
value of a + b + c + 10 is
p
__
(a) 10 (b) 11 (c) – ​   ​   (d) not defined
2
______
(c) 12 (d) 13 (  )​ = ÷​ x  2 + y2 
y
 ​ __
(  )
–1
1 49. If e​ tan
​ ​ x ​  ​
 ​, then y¢¢(0) is
39. If 5f (x) + 3f  ​ __
​ x ​  ​ = x + 2 and y = xf (x), then the
p p
​ __ ​  – ​ __ ​ 
value of y¢(1) is (a) – 2​e​2 ​ (b) – 2​e​ 2 ​
(a) 14 (b) 7/8 (c) – 2ep (d) – 2e–p.
(c) 1 (d) 10 50. Let g be a differentiable function of x.
40. Let f (x) g(x)

(  ( 
If f (x) = ____ ​  2 ​ 
 , x > 0, such that g (2) = 3, g ¢(2) = – 2,
2cos x ___
÷
– 3sin x
= cos–1 ​ ​ ____________
   
 ​ 
​ 13 ​
     ) 2sin x +___3cos x
​ + sin–1 ​ ​ ____________
   
​÷13 ​
 ​ 
     ​, ) x
then the value of f ¢(2) is
then the value of f ¢(2012) is
5.14  Differential Calculus Booster

(a) – 4/5 (b) 4/5 59. Let y = (1 + tan (x + 1°)) (1 + tan (x + 2°)).
(c) – 5/4 (d) 5/4 (1 + tan (x + 3°)) (1 + tan (42° – x)).
dy (1 + tan (43° – x))(1 + tan(44° – x)).
51. If xy = ex – y, then the value of ___ ​    ​ at (1, 2) is
dx d
(a) – 1 (b) 2 Then the value of ___
​    ​  ((1 + y)1 + y) is
dx
(c) 1 (d) – 2 (a) 1 (b) 2
2x
52. Let f (x) = tan–1 ​ ​ ______
( 
  
1 – x2 )
 ​  ​. If the differential co- (c) – 1 (d) 0

efficients of f (x) at x = – 2, 0, 2 are a, b and c


4
xx – x– x
60. If f (x) = cot–1 ​ ​ _______
2
 ​  (  )
 ​, then f ¢(1) is
respectively, then the value of a + b + c + __ ​   ​  is
5 (a) 1 (b) 2
(a) 1 (d) 2 (c) – 1 (d) 0
(c) 3 (d) 4 cos4a _____
sin4a _____

(  )
1
2 61. If _____
​  x   
​ + ​  y   
  ​ = ​     
 ​, then
1–x x+y
53. Let f (x) = cos–1 ​ ______
​    ​  ​.
1 + x2 (a) y = x tan2a (b) y = x cot2a
If the differential co-efficients of f (x) at x = – 1 and dy dy
1 are p and q respectively, then the value of p + q (c) ​ ___  ​ = tan2a (d) ​ ___  ​ = cot2a
dx dx
+ 10 is _____ ____
(a) 0 (b) 5 ​ x  + y  
62. If ÷ ​ + ​÷y  – x  
​ = c, then
(c) 10 (d) 15 dy 2x d2y __ 2
(a) ​ ___  ​ = ___
​  2 ​   (b) ​ ___2 
 ​ = ​  2  ​ 
54. Let f (x) = min {|x + 1|, |x|, |x – 1|}. dx c dx c
1
Then the value of f ¢​ ​ __ ​   ​ is
2 (  )
d2y
(c) ​ ___2 ​ = __
dx
4
​  2  ​  
c
dy 4x
(d) ​ ___  ​ = ___
dx c2
​   ​ 
(a) 1 (b) –1
x4 – x2 + 1 dy
(c) 0 (d) not defined 63. If y = ​  ___________
__      ​ and ___
​    ​ = cx + d then the value
2
x +÷    x + 1
​ 3 ​  dx
2 x , x <1
55. Let f (x) =  . of (c – d) is
3 − x x ≥1
Then the value of f ¢(1) is
p
(a) cot ​ __(  )
​   ​   ​
8 (  )
5p
(b) cot ​ ___
​   ​  ​
12

(d) tan ​( ___
(a) 2 log 2 (b) – 1
(c) – 2 log 2 (d) not defined
5p
(c) tan ​ ___
12 (  )
​   ​  ​ ​   ​  )​
5p
8
Let x sin3q + y cos3q = sinq cosq and
56.
dy 64. Let f (x) = |x – 1| + |x – 2|. Then
x sinq – y cosq = 0, then the value of ​ ___  ​ at (2, 1)
dx (a) f ¢ (1 ◊ 5) = 0 (b) f ¢ (2) = not defined
is
(c) f ¢ (3) = 2 (d) f ¢ (0) = – 2

(  )
(a) 2 (b) 1 2x
(c) – 2 (d) – 1 65. Let f (x) = sin– 1 ​ _____ ​   2 
 ​  ​. Then
1+x
57. Let y = (sin25° + sin210° + ... + sin290°) x, then the 2
dy (a) f ¢ (2) = – ​ __ ​   (b) f ¢ (0) = 2
5
value of ___ ​    ​ at (2011) is


1
__
(a) 6 ​   ​  
dx
1
(b) 7 ​ __ ​ 

1
(c) f ¢ ​ __
2 (  ) 8
​   ​   ​ = __
​   ​  
5
(d) f ¢ (1) = 0
2 2
1 1
(c) 8 ​ __ ​   (d) 9 ​ __ ​ ...
2 2   (Problems for JEE-Advanced)

(  p
58. Let y = ​ 1 + tan ​ __
8 (  ) ) ( (  ( 
p
​   ​  – x  ​  ​ ​ ​ 1 + tan ​ x + __ )))
​   ​   ​  ​  ​.
8 {  ( 
x+÷ ​ x  2 – a2 ​ 
1. If y = ​​ log ​ ___________
​ 
_______

)}
2
​  ​​ ​ + k log ​( x + ÷  ​ )​
​ x  2 – a2 
______
a     ​  
Then the value of y¢(2012) is
d2y dy
(a) 2012 (b) 2010 then prove that (x2 – a2) ___
​  2 ​ + x ​ ___  ​ = 2a
(c) 0 (d) 1 dx dx
Differentiation  5.15

__y 13. If x < 1, using differentiation, prove that


2. If (a + bx)​e​ ​x ​​ = x, then prove that

(  ) ( 
1 2x 4x3 8x7 1
​ _____  ​ + ______ + _____  ​ + _____  ​ +  ... to • = _____
d2y
x  ◊ ​ ___
dx
dy
dx
2
​  2 ​  ​ = ​​ x ​ ___  ​ – y  ​​ ​
3
)   
1+x 1+x
​   2 ​  ​ 
1+x
 4  ​ 
1 + x8
   ​     
1–x
 ​

[ { ÷  } ]
_____ 14. If a = x sin q + y cosq c and b = x cosq – y sinqi then
2 a–b x
3. y = _______
​  ______
    ​ tan–1 ​ ​ ​ _____
​   ​ ​  ​  tan ​ __  ​  ​, then prove that prove that,
÷​ a  – b  ​ 
2 2 a + b 2 2
d3x d y d2y ____ d3y
dy ​ ____3  ​  ◊ ​ ____2  ​ – ____
​  2  ​  
◊ ​  3  ​ = a2 + b2.
1
(i) ​ ___  ​ = _________
​       ​. dq dq dq dq
dx a + b cos x
15. If x2 + y2 + z2 – 2xyz = 1 then prove that
d2y b sin x
(ii) ​ ___2 ​ = ​ ___________
  
   ​. dy
(a + b cos x)2 dx dz
dx ​ ______   
______  ​ + ______
​  ______
    ​ + ​ ______
  
______  ​ = 0
4. If x = secq – cosq and y = secnq – cosnq ÷​ 1  – x  ​  ​÷1  – y  ​  ​÷1  – z2 ​ 
2 2

then prove that (x2 + 4) ​​ ___


dy 2
(  )
​    ​  ​​ ​ = n2 (y2 + 4).
dx
​ 
ax2
16. If y = _________________
       ​
(x – a)(x – b)(x – c)
5. If y = (logcos x sin x)(logsin x cos x)–1 + sin–1 ​ _____
1+x
2x
​   2 ​   ​then
(  )
bx
+ ​ ____________

c
    ​ + ​ _____
(x – b)(x – c) x – c
   ​ 
+ 1 then prove that,
dy
prove that ​ ___ (  )
p
​    ​  ​ at x = __
​   ​  is 8 ​ _______
4
​  2    ​ 
1
– _____
( 
​     ​  ​
)
( 
dx 4 log e2

6. If y = logn (|cos 4x|) + |sin x|,
p   +  16 1 ___
​ __
dy
y ​ ◊ ​ dx  ​ =
1 a
__
​ x ​ ​ _____
b
 ​ + _____
​ a – x  ​     
c
 ​ + ____
b–x c–x
​     
 ​  ​ )
dy p 17. If f, g, h are differential functions of x and
where x = sec2x, find ​ ___  ​ at = – ​ __ ​ 
dx 4

(  ) ( 
__ __

)
f g h
  
x ​÷2 ​     + x2
+ x​÷2 ​
1___________
7. If y = 2tan–1 ​ ______
​     ​ 
  ​ + log ​ ​  __     ​  ​ D = ( xf )I ( xg )I ( xh )I then prove that
1 – x2     + x2
1 – x ​÷2 ​
__ ( x 2 f )II ( x 2 g )II ( x 2 h )II
dy   
4​÷2 ​
then prove that ___
​    ​ = _____
​      ​
dx 1 + x4 f g h
I
D = f I gI hI
∏ cos ​( __
n

​  r  ​  )​ = __________
x sinx
8. Prove that If ​    
x  ​,
(  )
  ( x 3 f II )I ( x 3 g II )I ( x 3h II )I
r =1 2 n __
2  ◊ sin ​ ​  n  ​   ​
2
cos4a _____
sin4a _____ dy
(  ) (  )
n 1

1 x 1 x
(i) ​S  ​ ​ ​  ​  r  ​  tan ​ ​  r  ​   ​ = ​  n  ​ cot ​ ​  n   ​  ​ – cot x
__ __ __ __ 18. ​ _____
x   
​ + ​  y     ​, prove that ___
​ = ​      ​    ​ = tan2a
x+y dx
r = 1 2 2 2 2

( ÷ 
_________
(ii) ​S  ​ ​ ​ ​( ​ __  ​  sec x  ​( ​ __  ​ )​ )​ = cosec ​x​ ​ – ​ ___    ​ cosec ​
)
n
1 x 1 x2 + x + 1
19. If y = log ​ ​ ​ _________
n n
  r r 2n     ​ ​ 
  ​
r = 1 2 2 2 x2 – x + 1
(  )
___
2
x
​  2n   ​  ​ 1
+ ____
{  ( 
2x +__ 1
   ​ ​  tan–1 ​ ​ ______
​  __
  
2​÷3 ​   
​÷3 ​ )
2x __ –1
 ​ + tan–1 ​ ​ ______
 ​    ​  
  
​÷3 ​
 ​  ​ (  )}
2 2 2 2 2
9. If p = a cos q + b sin q, then prove that
dy 1
d 2p ____ a2b2 then prove that, ___ ​    ​ = __________
​       ​.
p + ____ ​  2   ​ = ​  3 ​ 

. dx x4 + x2 + 1

(  ( 
dq p
10. If z = cos7 x, y = sin x, then prove that,
d3z 105
​  2
1
20. If y = tan–1 ​ _________
x +x+1
    ​  2
1
 ​  ​ + tan–1 ​ __________
)
     ​  ​
x + 3x + 3 )
​ ___3  ​ = ____
( 
​   ​   sin 4 x
dy
2
4
11. If y = p (x) is a polynomial of degree n ≥ 3,
​  2
1
+ tan–1 ​ __________
     ​  ​ + ... to n terms.
x + 5x + 7 )
d d 2y
{  }
then prove that 2 ◊ ​ ___  ​ ​  y3 ◊ ​ ___2 ​   ​ = p(x). p¢¢¢(x).
dx dx
dy
Then prove that, ___ ​ 
1
​    ​ = __________
1
     ​ – _____
dx 1 + (x + n)2 x2 + 1
​      ​

12. If j (x) = f (x) ◊ g(x), where f ¢(x) ◊ g¢(x) = c


j¢¢ f ¢¢ g¢¢ 2c
​  2
1
( 
21. If y = tan–1 ​ ______________
    ​  ​
sin x + sin x + 1 )
then prove that ___
​   ​ = ___
​     ​ + ___
​  g ​ + ___
​    . ​
j f f.g
5.16  Differential Calculus Booster


(  1
+ tan–1 ​ ______________
​  2      ​  ​
sin + 3sin x + 3 ) f (x) = x3 + x2f ¢(1) + x f ¢¢(2) + f ¢¢¢(3)
for all x in R, show that f (2) + f (0) = f (1)


( 
​  2
1
+ tan–1 ​ _______________
    ​  ​
sin x + 5sin x + 7 )
ax + b
2. If y = ______
​  2
x +c
, then show that
 ​ 


dy
+ ... to n-terms. then prove that
cos x cos x
dy
dx ( 
d3y
dx
)
d2y
dx ( 
​ 2x ​ ___  ​ + y  ​ ​ ___3 ​ = 3 ​ x ​ ___2 ​ +
dy d2y
___
)
​    ​  ​ ​ ___2 ​ 
dx dx
​ ___  ​ = ____________
    ​ – _________
(  ( 
​    ​     ​
dx 1 + (sin x  +  n)2 1 + (sinx)2 ​ 
x
3. If y = tan–1 ​ _______
  2 
1.2 + x ) ​ 
x
 ​  ​ + tan–1 ​ _______
   
2.3 + x2
 ​  ​
)
22. For a function y = f (x), then prove that
dy 2 3/2
​​ 1 + ​​ ___ {  (  ) } {  (  ) }
dx 2 3/2
___
​    ​  ​​ ​  ​​ ​ ​​ 1 + ​​ ​ dy  ​  ​​ ​  ​​ ​

x
+ tan–1 ​ _______
​     ​  
3.4 + x2 (  )
​+ ... to n-terms then prove that
dx
​ ____________2
 ​      + ​ ____________  ​      = 0. dy 1
​ ___  ​ = ______
​     ​ 
n+1
– ​ ___________     ​
d
___ y d2x
___ dx 1 + x2 x2 + (n + 1)2
​  2 ​  ​  2 ​ 
dx dy
4. If f (x) = (ax + b)sin x + (cx + d) cos x, find the values
23. If y = cos–1 (8x4 – 8x2 + 1), then prove that of a, b, c, and d such that
dy 4
​ ___  ​ + ______ ​  _____
    ​ = 0. f ¢(x) = x cos x
dx ​÷1  – x2   ​
a b c d 5. If g(x) = (ax2 + bx + c) sin x + (dx2 + ex + f ) cos x
24. If f (x) = ​ _____ + ​ _____
   ​     ​ – ​ _____ – ​ _____
   ​     ​ 
and f (x) is then find the values of a, b, c, d, e and f such that
a+x b+x c+x d+x
g¢(x) = x2sin x.
1 __ 1 1 __ 1
divisible by x2, then find the value of ​ __ a ​ + ​ b ​ – ​ __
c ​ – ​ d ​. ay + b
ax + b
6. If y =  ​ ______ 
 ​ and z = ​ ______ 

 ​, prove that
25. If y = (sin–1 x)2 + (cos–1 x)2, then prove that Ax + B Ay + B


d2y
(1 – x2) ___
​  2 ​ –
dx
dy
x ​ ___  ​ = 4
dx
y¢¢¢ 3 ___
​ ___ ​ – __
y¢ 2 y¢
y¢¢ 2
(  )
z¢¢¢ 3 __
​    ​ ​​ ​   ​   ​ ​ = 0 = ___
​   ​ – __
z¢ (  )
z¢¢ 2
​    ​ ​​ ​   ​   ​ ​
2 z¢

( 
x+b
26. If ​ _____ )
 ​ = a tan–1 (a ln y), a > 0 show that
7. Find the value of

( 
​   ​  
2
​ lim 
1 – cosx cos2x cos3x
  ​ ​ __________________
​   ​ 
      ​
)

d2y
dx
dy
dx (  )
dy 2
y ​ ___2 ​ – (y ln y) ​ ___  ​ = ​​ ___
​    ​  ​​ ​
dx
x Æ 0 x2
8. If y = e​ tan
–1
​  x​ , show that
1 dy dx
27. If y = __​ x ​, then show that _______ ​  ______
    ​ + _______
​  ______
   ​ 
=0 d2y dy
÷​ 1  + y  ​  ÷​ 1  + x4 ​ 
4 (x2 + 1) ___
​  2 ​ + (2x – 1) ___
​    ​ = 0
dx dx
28. If 2x = y1/3 + y–1/3, prove that 9. If y = xn – 1 ln x, then prove that


d2y
(x2 – 1)  ___
​  2 ​ +
dx
dy
x ​ ___  ​ – 9y = 0
dx

d2y
x2 ​ ___ (  ) dy
​  2 ​  ​ + (3 – 2n) x ​ ___  ​ + (n – 1)2 y = 0
dx dx
29. If a curve is represented parametrically by the 10. If y = (C1 + C2x) sin x + (C3 + C4x) cos x,
equations x = f (t), y = g(t), then prove that
d 4y d2y

(  ) (  )
2 3 show that ___
​  4 ​ + 2 ​ ___2 ​ + y = 0
d y d2x g¢(t) dx dx
​ ___
​  2 ​  ​ / ​ ___2 ​ = – ​​ ____
​   ​  ​​ ​
dx dy f ¢(t)
11. Let f (x) = x3 f (1) + x2 f ¢(2) + x f ¢¢(3) + f  ¢¢¢(4)
30. If y = x5 (cos (ln x) + sin (ln x)), prove that Then prove that f (x) is an independent of x.

{ 
_____________

d2y
dx
dy
x2 ​ ___2 ​ – 9x ​ ___  ​ + 26y = 0
dx
7
12. If y = cos–1 ​ ​ __  ​ (1 + cos2x) + ÷
2
  2x – 48cos
​ sin 2
  x ​ sinx  ​ }
for all x in ​ 0, __ (  )
p
​   ​   ​ then prove that
2
(Tougher Problems for JEE-Advanced)
dy sinx
​ ___  ​ = 1 + ______________ ​  _____________
     ​.
1. If f is a function from R to R such that dx   x – 48cos
​÷sin 2
  
2
x ​
Differentiation  5.17

13. If x = tan ​ __
dy
2
y
(  ) ( 
(1 + tan(y/2))2
​    ​  ​ – ​ ​ _____________
tan(y/2)
  
 ​  )
   ​, prove that
12. If f ¢¢(x) = 10, f ¢(1) = 6 and f (1) = 4, then find the
value of f (0) + 2.
13. There is a polynomial P(x) = ax3 + bx2 + cx + d such
2 ​ ___  ​ = – siny (1 + siny + cosy)
dx that P(0) = –2 = P(1), P¢(0) = – 1 and P¢¢(0) = 10,

( ÷  )
_____
cos3x then find the value of (a + b + c + d + 10).
14. If y = cos–1 ​ ​ ​ _____  ​ ​  
  ​, prove that
cos3x

÷ 
____________
dy 6 Comprehensive Link Passages
​    ​ = ​ ____________
___ ​       ​ ​
dx cos2x + cos4x Passage I
_______
If f and g both are differentiable functions and F = f0g is
15. If ​÷x  2 + y2 ​ 
–1
= aeta​n​ ​x, where a > 0, y (0) π 0 then find
the composite function defined by F (x) = f (g(x)) then f is
the value of y¢¢(0). differentiable and F¢ is given by the product

Integer Type Questions F¢(x) = f ¢(g(x)) ◊ g¢(x)


x – x–1
x + x ( 
1. If y = cos–1 ​ ​  ______–1 
  )
 ​  ​, x > 0, find the value of
(i) Let f (x) = sin x, g(x) [x + 1] and g{f (x)} = h(x), where

{.} is the G.I.F, then the value of h¢ ​ __


p
(  )
​   ​   ​ is
2
y¢(0).
(a) non-existent (b) 1
2. If y = y (x) and it follows the relation 4x exy = y +
(c) – 1 (d) None
5 sin2x, find y¢(0).

(  )
(ii) If f (x) = |3 – x| and g(x) = f {f (x)}, then g ¢(2) is
_____ ____ d2y
3. If ​÷x  + y ​ 
+÷​ x  – y  
​= c, find the value of ​ c ​ ___2 ​ + 3  ​.
2
(a) 1 (b) – 1
dx (c) 3 (d) – 3
4. Let f ¢(x) = g(x) and g¢(x) = f (x) for all x
Also, f (3) = 5 and f ¢(3) = 4. Then find the value of ( 
x–1
(iii) If y = f  ​ _____
​ 
x+1
  ) dy
 ​  ​ and f ¢(x) = x2, then ___
​    ​ at x = 0 is
dx
[ f (10)]2 – [g(10)]2. (a) 0 (b) 2
2t 1–t 2 (c) – 2 (d) 1
5. If sin x = ​ _____    ​ and cot y = ​ _____
   
​, find the value of​
1 + t2 2t Passage II

( 
d2y
___
​  2 ​ + 3  ​.
dx ) Let x be the independent variable and y be a dependent
variable defined as y = f (x).
6. If 2x = (y1/3 + y–1/3), find the value of Assuming y1/m + y– 1/m = 2x, where m Œ R – {0}, then

​ ______ ( 
x2 – 1 ___
​  y   
d2y x ___
dx
)
​  ​ ​  2 ​ + __
dy
​ y ​ ​    ​.
dx


(i) The value of y is given by
_____
(a) (x – ​÷x  2 – 1 ​ 
)m (b) (x + ÷
_____
​ x  2 – 1 ​ 
)m
7. Let g be a differentiable function of x such that _____ _____

g(x) (c) (x + ​÷x  2 + 1 ​


)m
  (d) (x – ​÷x  2 + 1 ​
)m

f (x) = ____
​  x   
​.  If g¢(2) = 6, g(2) = 4, find the value of
dy
(ii) ​ ___  ​ is equal to
f ¢(2). dx
8. Suppose that F(x) = f (g(x)) and g(3) = 6 my my
(a) ​ _______
_____    ​ (b) ​ ______
_____    ​
g¢(3) = 4, f ¢(3) = 2, f ¢(6) = 2. Find F ¢(3). ÷​ x  2 + 1 ​  ÷​ x  2 – 1 ​ 
_____ _____
9. If f (x) = x2 + x3 and g is the inverse of f then find ÷​ x  2 –   1 ​  ÷​ x  2 –   1 ​ 
the value of (5g¢(2) + 3). (c) ​ ______
my    
​ (d) – ​ ______
my    

dy
10. If xy = e x – y, then find the value of 2 ​ ___
​    ​  ​ at x = e.
dx (  ) d2y dy
(iii) (x2 – 1) ​ ___2 ​ + x ​ ___  ​ =
dx dx
11. Let f be twice differentiable function such that
(a) my (b) m2/y
f ¢(x) = g(x), f ¢¢(x) = – f (x).
(c) m2y (d) my2
If h(x) = {f (x)2} + {g(x)}2 and h(5) = 7, then find
the value of h(2016).
5.18  Differential Calculus Booster

(  )
Passage III _____ _____
÷     
÷_____
​ 1 + x2 
 ​ + ​ 1 – x2 
–1 ________________
If y = tan ​ ​  _____    ​  ​
 ​
Since the graph of f (x) = |x – a| has a turning point at
(B) ​÷1  + x  
2
 ​ (Q) x y (1 + 2
 ​ – ​÷1  – x2 
x = a, so f (x) = |x – a| is not differentiable at x = a and dy logx)
|x – a| then ___
​    ​ is
f ¢(x) ​ ______ ​ 
, a π 0, for example f (x) = |x| is not differentiable dx
x–a
|x| dy
at x = 0 and f ¢(x) = ___ ​  x ​ , x π 0. y x – y ___
(C) If x = e , then ​    ​ is (R)
4
_____
​       ​
dx 1 + x2
(i) If f (x) = e||x| – 1|, then f ¢ ​ __
__
1
2 (  )
​   ​   ​ is equal to
__ (D)
dy
If xp yq = (x + y) p + q, then ​ ___ ​  logx
dx (S) ​ _________
    ​
(a) ​÷e 
   ​ (b) 1/​÷e     ​
is (l + logx)2
__ __
(c) – ​÷e    ​ (d) – 1/​÷e     ​
dy x
p
(  )
(ii) If f (x) = |x| |tan x|, then f ¢​ – ​ __ ​   ​ is equal to
6
(E) If y = (x x)x, then ___
​    ​ is
dx
(T) – ​ ______
   
_____  ​
÷​ 1  – x4 
 ​

(  ) {  __​ 43 ​  log ​ __p6 ​ }​
__
__
p ÷3 ​   
2​÷3 ​ 2. Match the following columns:
(a) ​​ __ ​​    ​ ​
​   ​   1/​ ____
​  p   
 ​ –
6
Column I Column II
​   ​  )​​ ​ ​{ ____ p ​ }​
__
(b) – ​​( __
__
p  
2​÷3 ​ 4   6
  
1/​÷3 ​
​ – __
​  p    ​   ​  log ​ __ 1 1 sin 2 y
6 3 If x = t – __
​   ​  and y = t + __
​   ​  then
(A) t t (P) (cot x-2
dy/dx is
{  }
__
sin2 x)
(  )
__
p ÷3 ​     __
2​÷3 ​

[  (  ( ÷  ) ) ]
4 6
(c) ​​ __ ​​    ​ ​ ____
​   ​   1/​  ​ + ​    ​ log ​ __
​  p    p ​  ​
_____
6 3 1 – x
If y = ​ cos2 ​ tan–1 ​ ​ ​ _____ 
 ​ ​  ​  ​  ​

(B) 1 + x (Q) x/y
(d) None

(  )
then dy/dx is
2p
(iii) If f (x) = |cos x| + |sin x|, then f ¢​ ___
​   ​   ​ is equal to ____
3
__   ​ = ecos2x × sinx then
If y = ​÷tany 
–÷
1______  
​ 3 ​ (C) (R) 1/2
(a) ​   ​    (b) 0 dy/dx is
2
__
÷    – 1
​ 3 ​ If xy = (x + y) p and dy/dx =
(c) ​ ______ ​  
  (d) None (D)
y/x, then p is
(S) 2
2
p
(iv) If f (x) = |cos x – sin x|, then f ¢ ​ __ (  )
​   ​   ​ is equal to
4
__ __ Questions asked in Roorkee-JEE Exams
  
(a) ​÷2 ​   
(b) – ​÷2 ​
(c) 0 (d) None
1–x
1. Differentiate sin–1 ​ ​ _____ 
1+x (  )  ​  ​ w.r.t. ÷
__
​ x 
   ​

Matrix Match [Roorkee-JEE, 1984]


dy
2. Find ___
​    ​, if (tan–1 x)y + ycot x = 1
Given below are Matching type questions, with two columns dx
(each having some items) each. Each item of column I has [Roorkee-JEE, 1987]

( ÷ 
_____

(  ) )
to be matched with the items of Column II, by encircling 2 a – b x
the correct match(es). 3. If y = ​ _______
______     ​ tan–1 ​ ​ ​ _____    ​ ​ tan ​ __​    ​  ​  ​ then prove
​÷a  – b  
2 2
 ​ a + b 2
Note:  An item of Column I can be matched with more
than one items of Column II. All the items of Column II d2y b sin x
that ___
​  2 ​ = ___________
​    
   ​
have to be matched. dx (a + b cos x)2
1. Match the following columns: [Roorkee-JEE, 1988]
dy
Column I Column II 4. Find ___
​    ​, when
dx

(  )
______ _____
2x
If y = sin–1 ​ ​ _____     ​  ​ + sec–1 ​÷1  – y2 ​ 
+ ​÷1  – t2 ​ 
= a (y – t) and
1 + x 2 y
__

(  )
(A) (P) ​ x ​ ____ _ _____
​ ​ 
+ x2
1______
  ​  
dy
___
​ then ​    ​ is x = sin–1 ​( t ​÷1  
  –  t ​ + ​÷t  2)
  ​ ​ ÷ 1  – t  
 ​  ​
1  –  x2 dx
[Roorkee-JEE, 1990]
Differentiation  5.19

5. If f, g and h are differentiable functions of x and


f g h

d2y
(  dy
)2
x3 ​ ___2 ​ = ​​ x ​ ___  ​ – y  ​​ ​
dx dx
[IIT-JEE, 1983]
D = ( xf )′ ( xg )′ ( xh )′ , prove that 8. Find the derivative w.r.t x of the function
( x 2 f )′′ ( x 2 g )′′ ( x 2 h )′′

f g h
2x
(logsin x cos x) (logcos x sin x)–1 + sin–1 ​ _____
​     
1 + x2 (  )
p
 ​  ​ at x = ​ __ ​ 
4
[IIT-JEE, 1984]
D = f¢ g¢ h¢
9. If f (x) = log x (ln x), then f ¢(x) at x = e is...
( x 2 f ¢¢ )¢ ( x 2 g ¢¢ )¢ ( x 2 h ¢¢ )¢
[IIT-JEE, 1985]

(  )
[Roorkee-JEE, 1991] 1
______ ______ 10. The derivative of sin–1 ​ – ​ ______    ​  ​ w.r.t
2x2 – 1
6. If ÷​ 1  – x6 ​  ÷ 
+ ​ 1 – y6 ​ 
= a3 (x3 – y3) prove that ______
1
​÷ 1 – x2 ​  at x = __

÷ 
______ ​   ​  is...
dy x2 ______ 1 – y6 2
​ ___  ​ = __
​  2 ​  ​ ​   
 ​ ​ [IIT-JEE, 1986]
dx y 1 – x6 11. No questions asked in 1987.
[Roorkee-JEE, 1994] 12. If y2 = P(x), a polynomial of degree 3, then

(  )
7. Find the differential co-efficient of
d d2y
2 ​ ___   ​ ​ y3 ​ ___2 ​  ​ equals
__
–1 __ 2​ (1 – ​
​ ÷x   ​)  ​
lo​g​(1 – ​÷x   ​)  ​ (sin  (1 – ÷
__ ​ x ​
   )) with respect to ​2​ ​ dx dx
[Roorkee-JEE, 1996] (a) P¢¢¢(x) + P¢(x) (b) P¢¢(x) ◊ P¢¢¢(x)
8. Find the differential co-efficient of f (x) = logx (sin(x2)) (c) P(x) ◊ P¢¢¢(x) (d) a constant.
_____
+ (sinx2)lo​g​e​x with respect to ÷
​ x  + 1 ​
  [IIT-JEE, 1988]
[Roorkee-JEE, 1997] 13. If x = secq – cosq and y = secnq – cosnq then prove

Questions asked in previous Years’ IIT-JEE Exams


dy 2
that (x2 + 4) ​​ ___ (  )
​    ​  ​​ ​ = n2(y2 + 4)
dx
[IIT-JEE, 1989]
1. Find the derivative of sin (x2 + 1) w.r.t
14. If f (x) = |x – 2| and g(x) = f (f (x)), then g¢(x) = ... for
x from first principle. [IIT-JEE, 1978]
x>2
2. Find the derivative of [IIT-JEE, 1990]
Ï x -1 15. Let f (x) be a quadratic expression which is +ve for
ÔÔ 2 x 2 - 7 x + 5 : x π 1 all real values of x.
f (x) = Ì at x = 1 If g(x) = f (x) + f ¢(x) + f ¢¢(x), then for any real x,
Ô- 1 : x =1
ÔÓ 3 (a) g(x) < 0 (b) g(x) > 0
(c) g(x) = 0 (d) g(x) ≥ 0
[IIT-JEE, 1979]
5x [IIT-JEE, 1990]
_________
3. Given y = ​  _______    ​  2
+ cos  (2x + 1). dy
___
  – x)2 ​ 
3​÷(1 16. Find ​    ​ at x = – 1, when
dx __
dy ÷   
​ 3 ​
Find ___​    ​. [IIT-JEE, 1980] (sin y)sin(px/2) + ​ ___ ​  sec–1 (2x) + 2x tan (ln(x + 2)) = 0
dx 2
[IIT-JEE, 1991]
dy
​ + (tan x)x, find ___
3
4. Let y = ​ex sin x
​ ​    ​ 17. No questions asked in 1992.
dx
[IIT-JEE, 1981] 18. The derivative of an even function is always an odd
function. Is it true or false? [IIT-JEE, 1993]
5. Let f be a twice differentiable function such that dy
f ¢¢(x) = – f (x), f ¢(x) = g(x) and h(x) = [f (x)]2 + [g(x)]2, 19. If y = (sin x)tan x, then ___ ​    ​ is
dx
find h(10) if h(5) = 11 [IIT-JEE, 1982]
(a) sin x tan x (1 + sec2x logsin x)

( 
2x – 1
6. If y = f ​ ______
​  2
x +1
 ​  
) dy
​ and f ¢(x) = sin(x2) then ___
​    ​ = ...
dx


(b) tan x (sin x)tanx – 1 ◊ cos x 
(c) sin xcosx ◊ sec2x log sin x
[IIT-JEE, 1982]
(d) tan x ◊ (sin x)tanx – 1
7. If (a + bx)ey/x = x, then prove that
[IIT-JEE, 1994]
5.20  Differential Calculus Booster

20. No questions asked in 1995.


dy
21. If xe xy = y + sin2x, then find ___
​    ​ at x = 0
dx
(  ) (  )
d2y dy –2
(c) ​ ___
​  2 ​  ​ ​​ ___
dx dx
​    ​  ​​ ​
d2y dy –3
(d) – ​ ___
​  2 ​  ​ ​​ ___
dx dx (  ) (  )
​    ​  ​​ ​

[IIT-JEE, 2007]
[IIT-JEE, 1996]
31. Let f (x) = 2 + cos x for all real x.
x3 sin x cos x Assertion (A):  For each real t, there exist a point c
22. Let f (x) = 6 -1 0 in [t, t + p] such that f ¢(c) = 0
p p2 p3 Reason (R):  Because f (t) = f (t + 2p) for each real t.
[IIT-JEE, 2007]
where p is a constant.
32. Let f and g be real valued functions defined on
d3 (– 1, 1) such that g¢¢(x) is continuous g(0) π 0,
Then ___​  3  ​ ( f (x)) at x = 0 is
dx g¢(0) = 0, g¢¢(0) π 0 and f (x) = g(x) sin x
(a) p (b) p + p3 Assertion (A): ​  
lim ​ (g (x) cotx – g(0) cosec x) = f ¢¢(0)
x Æ 0
(c) p – p3 (d) independent of p.
Reason (R):  Because f ¢(0) = g(0)
[IIT-JEE, 1997]
[IIT-JEE, 2008]
2
ax bx 33. Let g (x) = log( f (x)), where f (x) is twice differen-
23. If y = _________________
​         ​ + ____________
​       ​
(x – a)(x – b)(x – c) (x – b)(x – c) tiable function on (0, •) such that f (x + 1) = x f (x).
Then N = 1, 2, 3...
c
+ _____
​ x – c ​ + 1, then prove that
(  1
g¢¢ ​ N + __
2 ) (  )
1
​   ​   ​ – g¢¢ ​ __
​   ​   ​ =
2
dy
​ ___  ​ =
dx
y a
__
( 
​ x ​  ​ a_____
b
 ​ + _____
​  – x  ​     
b–x c–x
c
 ​ + ____
​       ​  ​ )
{  1
(a) – 4 ​ 1 + __
9 25
1
​    ​ + ___
1
​    ​ + ... + ________
​       ​  ​
}
(2N – 1)2
[IIT-JEE, 1998]
24. No questions asked in 1999.
25. If x2 + y2 = 1, then

{ 1
(b) 4 ​ 1 + __
​    ​ +
9 25
1
___ ​ 
1
​    ​ + ... + ________
   
(2N – 1)2 }
 ​  ​

{ 
(a) y y¢¢ – 2(y¢)2 + 1 = 0
(b) y y¢¢ + (y¢)2 + 1 = 0

1
(c) – 4 ​ 1 + __
9 25
1
​    ​ + ___ ​ 
1
​    ​ + ... + _________
   
(2N + 1)2
 ​  ​
}
{ 
(c) y y¢¢ – (y¢)2 – 1 = 0
(d) y y¢¢ + 2(y¢)2 + 1 = 0
[IIT-JEE, 2000]

1
(d) 4 ​ 1 + __
​    ​ +
9 25
1
___ ​ 
1
​    ​ + ... + _________
   
(2N + 1)2 }
 ​  ​

x [IIT-JEE, 2008]
26. Let f : (0, •) Æ  and F (x) = Ú​  ​  ​ f (t) dt 34. No questions asked in between 2009-2010
b–x
35. Let f : (0, 1) Æ R be defined by f (x) = ​ ______ 
0

 ​, where
If F(x2) = x2 (1 + x), then f (4) is 1 – bx
(a) 5/4 (b) 7 b is a constant such that 0 < b < 1. Then
(c) 4 (d) 2 (a) f is not invertible on (0, 1)
[IIT-JEE, 2001]
1
(b) f π f  –1 on (0, 1) and f ¢(b) = ____
​     ​ 
27. No questions asked in between 2002-2003. f (0)
1
28. If y is a function of x and log (x + y) = 2xy, then the (c) f = f  –1 on (0, 1) and f ¢(b) = ____
​     ​ 
value of y¢(0) is f (0)
(a) 1 (b) –1 (d) f  –1 is differentiable on (0, 1),
(c) 2 (d) 0 [IIT-JEE, 2011]
[IIT-JEE, 2004]
29. No questions asked in between 2005-2006.   (  ( 
sinq
36. Let f (q) = sin ​ tan–1 ​ _______  ​  
​  _____
​÷cos2q  ​

p
))
4
p
​  ​, where – ​ __ ​  < q < ​ __ ​ .
4
d
d2x Then the value of ______ ​     ​    ( f (q)) is...
30. ​ ___2 ​ is d(tanq)
dy
[IIT-JEE, 2011]
(  )
d2y –1
(a) ​​ ___
​  2 ​  ​​ ​
dx (  ) (  )
d2y –1 dy –1
(b) – ​​ ___
​  2 ​  ​​  ​​​ ___
dx
​    ​  ​​ ​
dx
37. No questions asked in between 2012-2014.
Differentiation  5.21

Answers

Level II INTEGER TYPE QUESTIONS


1. (a) 2. (b) 3. (b) 4. (d) 5. (c)
1. 2 2. 4 3. 5 4. 9 5. 3
6. (d) 7. (c) 8. (c) 9. (a) 10. (c)
6. 9 7. 2 8. 8 9. 4 10. 1
11. (d) 12. (a) 13. (c) 14. (b) 15. (a)
11. 7 12. 5 13. 8.
16. (c) 17. (a) 18. (b) 19. (a) 20. (b)
21. (a) 22. (d) 23. (b) 24. (a) 25. (c) Comprehensive Link Passages
26. (b) 27. (d) 28. (a) 29. (c) 30. (a) Passage I : (i) (b) (ii) (d) (iii) (a)
31. (a) 32. (b) 33. (c) 34. (c) 35. (d) Passage II : (i) (c) (ii) (b) (iii) (c)
36. (d) 37. (c) 38. (b) 39. (c) 40. (b) Passage III : (i) (c) (ii) (b) (iii) (c) (iv) (d)
41. (d) 42. (d) 43. (c) 44. (a) 45. (d)
46. (a) 47. (d) 48. (d) 49. (b) 50. (c) Matrix Match
51. (a) 52. (d) 53. (c) 54. (d) 55. (d) 1. (A)Æ(P, S), (B)Æ(R), (C)Æ(Q, T)
56. (c) 57. (d) 58. (c) 59. (d) 60. (c) 2. (A)Æ(P, S), (B)Æ(Q, T), (C)Æ(P, R)
61. (a, c) 62. (a, b) 63. (a, c) 64. (a, b, c, d)
65. (a, b, c)

Hints and solutions

1. We have f ¢(x) =   ( (  (  ) )
log ​ 1 + __
1 __________
__
​ lim ​ ​ ​ x ​ ◊ ​ ​ 
h
​ __
 ​ 
h
​ x ​  ​
     ​  ​

( 
h Æ 0

)
f (x + h) – f (x) x  ​
​ lim ​  ​ _____________
=  ​         ​  ​
h Æ 0 h = 1/x

( 
e
)
x + h
–e
​ lim ​ ​ ________
x 3. We have f ¢(x)

( 
=  ​     ​  ​
h Æ 0 h
= 
f (x + h) – f (x)
​ lim ​ ​ _____________
​       ​   ​ )
(  )
eh – 1
​ lim ​ ​ e ◊ ​ _____
=  x 
   
​  ​
h Æ 0 h


h Æ 0

= 
h

(  )
e –1
ex ◊ ​ ______
​ lim ​   ​     
h
​  ​

h Æ 0
( 
loga (x + h) – loga x
​ lim ​ ​ ​ _________________
= 
h
     ​   ​ )
(  (  )
h
)
h Æ 0
x +  h
loga ​ ​ _____​  ​
x   
= ex =  __________
​ lim ​ ​ ​       ​   ​

( 
h Æ 0 h

)
2. We have f ¢(x)
( 
loga ​ 1 + __
h
)
(  )
f (x + h) – f (x) ​ x ​  ​
= ​ lim ​ ​ _____________
  ​         ​  ​ =  ___________
​ lim ​ ​ ​      ​   ​
h Æ 0 h h Æ 0 h
__
​ x ​ ◊ x

( 
(  log (x + h) – log x
)
)
​ lim ​ ​ ​ ________________
=  
h Æ 0 h
       ​  ​
( 
loge ​ 1 + __
h
​ x ​  ​)
( 
1 ___________
(  )
__
x+h
log ​ _____
​      )  
​  ​
= 
​ lim ​  ​  x ​  ​ ​ 
h Æ 0 h
__
  
​ x ​ ◊ logea
  ​   ​

(  ( 
__________ h
=  
​ lim ​ ​ ​         ​  ​

)
h
(  )
h Æ 0
h

(  )
loge ​ 1 + __
=  
​ lim ​ ​ ​ 
( 
log ​ 1 + __
__________
h
)
​ x ​  ​
     ​   ​
= 
1
_______
​ lim ​ ​ ​ 
h Æ 0 x ◊ loge a
   ​   ___________
​ ​ 
h
 ​ 
​ x ​  ​
     ​  ​

 ( (  ) )
__
​ x ​
h Æ 0 h
h
log ​ 1 + __ ​ x ​  ​ ​ 
1
= _______    ​ 
=   __________
​ lim ​ ​ ​      ​   ​ x ◊ loge a
h Æ 0 h
__
​ x ​ ◊ x
5.22  Differential Calculus Booster

4. We have f ¢(x) = (esin × cos x)


h Æ 0 ( 
f (x + h) – f (x)
​ lim ​ ​ _____________
=  ​ 
h
       ​  ​ ) 7. We have f ¢(x)

= 
h Æ 0 ( 
sin (x + h) – sin x
​ lim ​ ​ ________________
​ 
h
       ​  ​ ) h Æ 0 ( 
f (x + h)  –  f (x)
​ lim ​ ​ _____________
=   ​ 
h
       ​  ​ )
(  ( 
x+h+x
2cos ​ ​  ________
2
​ lim ​ ​ ___________________________
=  ​        
x+h–x
2) ( 
 ​ sin ​ ​  ________
 ​    ​   ​
  ​  ​
) ) =  
h Æ 0
​  ( 
sin2 (x + h)  –  sin (x)2
​ lim ​ ​ __________________
h
       ​  ​ )
(  ))
h
(  ) ( 
h Æ 0

(  (  ) (  ) )
(x + h)2  +  x2 (x + h)2 – x2
h
2cos ​ x + __
h
​   ​   ​ sin ​ __
​   ​   ​ cos ​ ​ ___________
 ​ 
     ​  sin ​ ​ ___________
 ​ 
    ​
2 2
=  ​ 
2
​ lim ​ ​ _________________
    
2
  ​  ​ ​ lim ​  ​  ​ _______________________________
=            ​  ​

(   ( (  ( 
h Æ 0 h h Æ 0 h

(  ) (  )
) ) (  )
)
h h
2cos ​ x + __ ​   ​   ​ sin ​ __
​   ​   ​ (x + h)2 + x2 (2hx + h2)
​ lim ​ ​ _________________
=  ​ 
2
 ​    
2
   ​ 2cos ​ ___________
​   ​ 
     ​ sin ​ _________
​   ​ 
    

2 2
h ​ lim ​  ​  ​ ______________________________
=             ​  ​
) ( 
h Æ 0 __
​    ​ ◊ 2
2hx + h2 ________
)
h Æ 0 2h
2 ​ ________
​   ​  
 ​ ​ ​    
 ​  ​

 ( (  ( 
= cos x 2 2hx + h2

)
5. We have ¢(x)

( 
f (x + h) – f (x)
​ lim ​ ​ _____________
=  ​ 
h
     ​  ​ ) =  
(x + h)2 + x2
2cos ​ ___________
​ 
2
 ​ 
    
​ lim ​  ​  ​ ______________________________
        
(2hx + h2)
​ sin ​ _________
​ 
2
 ​ 
    
 ​  ​
​ ) (  )
) ( 
h Æ 0

(  )
2hx + h2 ______
)
h Æ 0 2
tan–1 (x + h) – tan–1 x ​ ​ ________
 ​  
 ​ ​ ​     ​   ​
​ lim ​ ​ ​ __________________
=         ​  ​ 2

 ( ( 
h Æ 0 h 2x + h

(  (  )) )
)
x+h–x (x + h)2 + x2
tan–1 ​​  ___________
     ​  ​ cos ​ ​ ___________
 ​      ​
1 + x (x + h)
​ lim ​ ​ ​ _________________
=         ​  ​ =   ​ 
2
​ lim ​ ​ ________________     ​ 
   ​
(  )
h Æ 0 h 1
​ ______
h Æ 0
​       ​  ​

=  ( 
​ 
​ 
1 ( 
+
h
tan–1 ​ ___________
x (x
​ lim ​ ​ _________________
     ​  ​
    
+ h)
  ​  ​
)) = 2x cos(x2).
2x + h

( 
h Æ 0 h
8. We have y = logx x + 10 = 1 + 10 = 11

(  )
)
h
tan  ​ ___________
​  –1
    ​  ​ dy
1 + x (x + h) fi ​ ___  ​ = 0.
​ lim ​ ​​  ________________________
=     
    ​  ​ dx
h Æ 0 ___________ h
​       ​ ◊ (1 + x (x + h))
1 + x (x + h) 9. We have y = 5​ log
​ 3x​ – x​ log
​ 35​

1 fi y = x​ log
​ 3x​ – x​ log
​ 35​ = 0
= ​ ______
    ​
1 + x2 dy
fi ​ ___  ​ = 0
6. We have f ¢(x) dx

( 
f (x + h) – f (x)
​ lim ​ ​ ​ _____________
= 
h Æ 0 h
       ​  ​ ) 1 __
10. We have y = ​ __
2 __ 3
x ​ + ​ x2  ​  + ​ x3  ​ 

( 
dy
)
1 4 9
esin (x + h) – esinx fi ​ ___  ​ = – ​ __2  ​  – __ ​  3  ​  – __
​  4  ​ 
​ lim ​ ​ _____________
=  ​         ​  ​ dx x x x
h Æ 0 h __ __ 2  __ 3 __
11. We have y = ​÷x    ​ + x ​÷x     ​ + x ​÷x     ​ + x ​÷x 
  ​ 

=  
h Æ 0
​  ( 
esin (esin (x + h) – sinx  –  1)
​ lim ​ ​ ___________________
   
h
    ​  ​ )
1
__
​   ​ 
3
__
​   ​ 
5
__
​   ​ 
= x​ ​2 ​ + ​x​2 ​ + ​x​2 ​ + ​x​2 ​
7
__
​   ​ 

( 
dy 3 __ 5 __ __ 7 2 __
)
1
sin sin (x + h) – sinx
 (e
e__________________ – 1) sin (x + h) – sin x) fi ​ ___  ​ = ____ ​   __ ​ + __
​    ​ ​÷ x ​  + __
​   ​  x ​÷x 
   ​ + ​   ​  x  ​÷x 
  ​ 
=  
​ lim ​ ​ ​          ​ × ​ _______________
       ​  ​ dx 2 ​÷x    ​  2 2 2
h Æ 0 (sin (x + h) – sin x) h
12. We have

=   ( 
esin × (sin (x + h) – sin x)
______________________
​ lim ​ ​ ​         ​  ​ )
p
8 ( 
y = ​ 1 + tan ​ ​ __ ​  – x  ​  ​ ​ 1 + tan ​ x + __ ( 
p
​   ​   ​  ​
8 ) ) (  (  ))
h Æ 0 h
Differentiation  5.23
_____ _____
fi y=2 19. Given y = log (​÷x  – 1 ​ + ​÷x  + 1 ​
)

( 
dy
fi ​ ___  ​ = 0
dx
sec x + tan x – 1
dy
fi ___
dx (​÷x  – 1 ​ – ÷
1 _____
​    ​ = _______________
​  _____  
​ x  + 1 ​
)

1
    ​ × ​ _______
​  _____
2​÷x  – 1 ​
  2​÷x
1
 ​ – ​ _______
       
_____  ​  ​
  + 1 ​
  )
( 
13. We have y = ______________
​    
   ​ _____ _____


sec x – tan x + 1
sec x + tan x – (sec x2 – tan x2)
= __________________________
​           ​
dy
fi ___
1 _____
​    ​ = _______________
​  _____  
dx (​÷x  – 1 ​ – ​÷x  + 1 ​
)

​÷x  + 1 ​ – ​÷x  – 1 ​ 
    ​ × ​ ​ ______________
_____     
 ​  ​
2​÷x  2 – 1 ​

)
sec x – tan x + 1 dy 1
fi ​ ___  ​ = – ​ ________
   ​ 
_____
(sec x + tan x)(1 + sec x – tan x) dx 2​÷x  – 2 ​ 
2
= ​ __________________________
         ​
(sec x – tan x + 1)
20. Given y = (sinx)x
= (sec x + tan x) x
fi y = elog(sinx)​​ ​ = ex log(sinx)
dy
Thus, ​ ___  ​ = (sec x tan x + sec2x)
( 
dy
dx cos x
fi ​ ___  ​ = ex log(sinx) ​ log sin x + x ◊ ​ ____ ​  ​ )
(  )
dx sin x
dy
​​ ___
​    ​  ​​ ​ = 0 + 1 = 1 dy
dx x = 0 fi ​ ___  ​ = (sinx)x (log sin x + x ◊ cot x)
dx
x4 + x2 + 1
14. We have y = ​ __________
   ​ 
x2 – x + 1 21. Given y = log (sin x + cos x)

(x2 – x + 1)(x2 + x + 1) dy cos x – sin x


= ​ ____________________
     ​
   fi ​ ___  ​ = ___________
​     ​
dx sin x + cos x
(x2 – x + 1)
dy 1 – tan x
= (x2 + x + 1) fi ​ ___  ​ = ________
​   ​ 
dx 1 + tan x
dy
Thus, ​ ___  ​ = 2x + 1
( 
dy p
dx fi ​ ___  ​ = tan ​ __
dx
​   ​  – x  ​
4 )
dy
Given ​ ___  ​ = ax + b 1
22. We have f (x) = _____
​      ​
x–1
Comparing the co-efficients of x and the constant
term, we get, a = 2 and b = 1
Thus, a + b + 10 = 2 + 1 + 10 = 13.
Now, (  )1
f  ( f (x)) = f  ​ _____
​     
x–1
 ​  ​

1
15. = ________
​       ​
16. 1
_____
​       ​ – 1 
x–1
17. Given y = log(sin(3x + 5))
x–1
dy 1 = ​ ________
    ​
fi ​ ___  ​ = ​ __________
     ​ ◊ cos (3x + 5) ◊ 3 1–x+1
dx sin (3x + 5)
x–1
= _____
​   ​ 
dy 2–x
fi ​ ___  ​ = 3 cot (3x + 5)
dx d((f (f (x))))
______
Thus, ​ __________
     ​ 
18. Given y = log (x + ​÷x  2 + 1 ​ 
) dx

(  ) (  )
d x–1
dy
fi ​ ___  ​ =
1______
___________
​ 
1 _____
× 2x
     ​ × ​ 1 + ​ ________     ​  ​ = ___
​     ​ ​ ​ _____ 
 ​  ​
dx dx 2 – x
​ x  + 1 ​ 
(x + ÷ 2
) 2​÷x  2 + 1 ​

( 

dy (2 – x) ◊ 1 – (x – 1) ◊ (–1)
fi ​ ___  ​ =
dx
​ 
1_____
____________
    ​ × ​ 1 + _______
​ x  + 1 ​ 
(x + ÷ 2
)
x
​  _____    
​÷x  + 1 ​ 
2 )
 ​  ​ = ​ ____________________
   
(2 – x)2
 ​
  

(  )
_____ (2 – x + x –  1) 1
dy 1_____ ÷​ x  2 _____
+ 1 ​ + x = ​ _____________
    ​ = _______
   ​       ​
fi ​ ___  ​ = ___________
​       ​ × ​ ​ __________  ​  
   ​ (2 – x)2
(2 – x)2
dx (x + ÷ ​ x  + 1 ​
2
 ) ÷​ x  2 + 1 ​ 
x–1
dy 23. Given f (x) = ​ _____ 
 ​
1 x+1
fi ​ ___  ​ = ​ _______
_____   ​ 
dx ÷ ​ x  + 1 ​
2

5.24  Differential Calculus Booster

(  ( x–1
f ( f ( f (x))) = f ​  f  ​ ​ _____  ))
 ( )) )
 ​  ​  ​ Thus, f ¢(2) = 0
x+1


( 
x–1
​ _____
​ 
x+1
= f  ​ ​ __________

 ​  ​  –  1
    ​  ​
27. Given f (x) = |x2 – 1| + |x2 – 4|
Ï-( x 2 - 1) - ( x 2 - 4) : x < -2
( 
x–1
​ ​ _____ 
x+1
 ​  ​  +  1 Ô 2
Ô
2
Ô( x - 1) - ( x - 4) : - 2 £ x < -1


x
(  –1–x–1
= f  ​ ____________
​ 
x–1+x+1
   )
   ​  ​
= Ì(1 - x 2 ) + (4 - x 2 )
Ô 2 2
: -1 £ x < 1
Ô( x - 1) + (4 - x ) : 1£ x < 2

– 2
= f  ​ ___ (  )
​   ​  ​
2x
Ô 2 2
Ó( x - 1) + ( x - 4) : x≥2


–1
= f  ​ ___ (  )
​  x ​   ​
(  )
3
f ¢ ​ __

(  )
Thus, ​   ​   ​ = 0
2
1
– ​ __
x ​ – 1
______
= ​ ​    
 ​  ​ 28. Given f (x2) = x4 + x3 + 1
1
– ​ __
x   ​ + 1 fi f ¢(x2) ◊ 2x = 4x3 + 3x2

( 
–1 – x
= ​ ______
​ 
1–x

   ) ( 
x+1
​  ​ = ​ ​ _____ 
x–1
 ​  ​ ) fi
4x3 + 3x2
f ¢(x2) = ​ _________
2x
   ​ 
3
= 2x2 + __
​   ​  x
2

d(f (f (f (x)))) Replacing x by x2, we get,


Thus, ​ __________      ​  3
dx f ¢(x4) = 2x4 + __
​   ​  x2
2

d x+1
= ___ ( 
​    ​  ​  ​ _____ 
dx x – 1 )
 ​  ​ 29. We have g(x) = f  –1(x)
fi f (g (x)) = x
(x – 1) ◊ 1 – (x + 1) ◊ 1
= ​ ___________________
     ​   fi f (g (x)) ◊ g¢(x) = 1
(x – 1)2
1

( 
fi g¢ (x) = _______
​     ​ 

x–1–x–1
= ​ ____________
​    
(x – 1)2
​ 
– 2
​ = _______
 ​     
(x – 1)2 )
 ​
fi ​ 
f ¢(g (x))
1
g¢ (x) = _________
   ​ 
cos (2g(x))
24. Given g(x) = f (3x – 1)
30. We have f (x) = x + tan x
g¢(x) = 3f ¢(3x – 1)
Given f (x) = g–1(x)
1
= 3 × ___________
​       ​ fi g ( f (x)) = x
(3x – 1)2 + 1
3 fi g¢(f (x)) ◊ f ¢(x) = 1.
= ___________
​  2      ​
9x – 6x + 2 1 1
fi g¢(f (x)) = ____
​     ​ = ________
​       ​
3 f ¢(x) 1 + sec2x
25. Given
y = f (sin(x ))
1
dy
fi ​ ___  ​ = f ¢(sin(x3)) ◊ cos(x3) ◊ 3x2 fi g¢( f (x)) = ________
​      ​
dx 2 + tan2x
dy 1
fi ​ ___  ​ = 3x2 ◊ cos(x3) ◊ ((sin(x3))2 + 1) fi g¢(f (x)) = _____________
​       ​
dx 2  + (f (x) – x)2
26. Given f (x) = |x – 1| + |x – 3|
1
Ï-( x - 1) - ( x - 3) : x < 1 fi g¢(x) = ____________
​       ​
Ô 2 + (g(x) – x)2
= Ì( x - 1) - ( x - 3) : 1 £ x < 3
Ô( x - 1) + ( x - 3) : x ≥ 3 x4 – x2 + 1
Ó 31. Given y = ​  ___________
  
___     ​
x2 + ÷    ​ + 1
​ 3x 
Ï-2 x + 2 : x < 1
Ô (x4 + 1) – x2
= Ì2 : 1£ x < 3
= ​ ___________
___  
    ​
Ô2 x - 4 : x ≥ 3 x2 + ​÷3x 
   ​ +  1
Ó
Differentiation  5.25

(x2 + 1)2  –  3x2 dy 2x 2x


= _____________
​  2 __      ​ 2x ​ ___  ​ = ____
​  __  ​ – _____
​  __
  ​ 
x +÷    x + 1
​ 3 ​ dx 2​÷x 
   ​ 2x​÷x 
  ​ 

__ __ dy __ 1__
(x2 + ​÷3 ​
  x  + 1)(x2 – ​÷3 ​
  x  + 1) 2x ​ ___  ​ = ​ x 
÷   ​  – ​ ___  ​ 
= ​ _________________________
2
   
__      ​ dx ​ x 
÷    ​
(x + ÷    x + 1)
​ 3 ​ Hence, the result.
__
2
  x  + 1)
= (x – ​÷3 ​ 35. Given y = ex cosx
dy __ dy
​ ___  ​ = 2x – ÷   
​ 3 ​ ​ ___  ​ = ex cosx – ex sinx
dx dx
dy dy
Given ​ ___  ​ = px + q ​ ___  ​ = ex (cosx – sinx)
dx dx

(  )
Comparing the co-efficients __ of x and constant term, dy __ 1 1
we get, p = 2, q = – ​÷3 ​    ​ ___  ​ = ÷   e  x ​ ___
​ 2 ​ ​  __  ​  cosx – ___
​  __  ​  sinx  ​
dx   
​÷2 ​ ÷   
​ 2 ​
Thus, p – q
( 
dy __ p
)
__
=2+÷   
​ 3 ​ ​ ___  ​ = ÷   e  x cos ​ x + __
​ 2 ​ ​   ​   ​
dx 4
= cot ​ ___
12
p
​    ​  ​ (  ) 36. Given
______
​ x  2 + a2 
y=÷  ​

p
2 12
p
= tan ​ ​ __ ​  – ___(  )
​    ​  ​ dy
dx
1 ______
× 2x
​ ___  ​ = ​ ________    
 ​
2​÷x  2 + a2 
(  )
5p  ​
= tan ​ ___
​   ​   ​
12 dy x
_____ _____ ​ ___  ​ = _______
​  ______
    ​
32. We have y = ​÷x  – 1 ​ ​ x  + 1 ​
 + ÷  
dx ÷ ​ x  + a2 
2
 ​
dy ______
1 1 dy
​ ___  ​ = _______
​  _____
    ​ + _______
​  _____
    ​ ​÷ x2 + a2   ​ ___
​    ​ = x
dx 2​÷x  – 1 ​  2​÷x  + 1 ​  dx
_____ _____
dy ​÷x  + 1 ​ dy
 + ​÷x
​ ___  ​ = _______________
​  _____ _____
  – 1 ​
  
   ​
  y ___
​    ​ – x = 0
dx 2​÷x  – 1 ​ ​
  ÷ x + 1 ​

dx
dy y 37. Given y = ex + e– x
​ ___  ​ = _______
​  _____
     ​
dx 2​÷x  2 – 1 ​  dy
_____ ​ ___  ​ = ex – e– x
 dy __
___ y dx
​÷ x2 – 1 ​ ​ 
   ​ = ​    ​
dx 2 dy _________
Hence, the result. ​ ___  ​ = ÷   x – e– x)2 ​ 
​ (e
dx
33. Given
x+2
x
y = _____
​     
1
 ​ = ​ 1 – ______
​     
(x + 2)
 ​  ​(  ) dy
​ ___  ​ = ÷
dx
_________
  x – e– x)2 ​ 
​ (e –4
dy 1
​ ___  ​ = _______
​       ​ dy _____
​ y  2 – 4 ​ 
dx (x + 2)2
​ ___  ​ = ÷
dy dx
x
x ​ ___  ​ = _______
​       ​
dx (x + 2)2 38. Given
xy = 4
4
dy 1 x y = __ ​ x ​
x ​ ___  ​ = ______
​       ​ ​ ______     ​
dx (x + 2) (x + 2) dy 4
​ ___  ​ = – ​ __2  ​ 
( 
dy
x ​ ___  ​ = ​ 1 – ______
dx
​     
x
(x + 2) (x + 2) ) x
 ​  ​ ______
​       ​ dx
dy
x
4
dy ​ ___  ​ + y2 = – ​ __2  ​  + y2
dx
x ​ ___  ​ = (1 – y) y x
dx dy 4 16 12
__ 1 ​ ___  ​ + y2 = – ​ __2  ​  + ___
​  2 ​ = ___
​  2 ​ 
34. Given y = ​÷x  ___
   ​ + ​  __  ​  dx x x x

( 
​ x 
   ​

)
÷
dy 12 12
dy 1 1 x ​ ___
​    ​ + y2  ​ = x × ___ ​  2 ​ = ​ ___
x ​ 
​ ___  ​ = ​ ____    ​ – ​ _____
__  __ ​  dx x
dx 2​÷x     ​ 2x​÷x     ​
5.26  Differential Calculus Booster

(  ) (  )
( 
(  )
)
dy p q
x ​ ___
​    ​ + y2  ​ = 3 ​ __
4
​ x ​  ​ = 3y 2sin2 ​ __ ​   ​  – __
​   ​   ​
2 2
dx y = tan–1 ​ ​ _____________________         ​  ​
(  ) (  )
______ p q p q
a2
x
39. Let y = __
​    ​ ​÷a  2 – x2 ​ 
2
+ __
​   ​  sin–1 ​ __
2 (  )
x
​ a ​  ​ 2sin ​ __ ​   ​  – __
2 2
​   ​   ​ cos ​ __
​   ​  – __
4 2
​   ​   ​

dy 1 ______
​ ___  ​ = __
dx 2
​   ​ ​ ÷ a2 – x2  ​ + __
x 1 ______
× –2x
​    ​ × ​ ________
2
  ​ 
2​÷a  2 – x2 ​ 
y = tan–1 ​ tan ​ __
p
2 2 (  (  ) )
​   ​  – __
q
​   ​   ​  ​

+ __
a2 1
​   ​  × ________
1
× __
p q
y = ​ ​ __ ​  – __
​   ​   ​ = __(  )
p tan–1 x
​   ​  – ______
​   ​   

÷ 
​  ______
   ​  ​ a ​ 2 2 4 2
2 x 2
​ 1 – __ ​  2  ​ ​  dy 1
a ​ ___  ​ = – ​ ________      ​
______ dx 2 (1 + x2)
1 1 x2 a2 1
= __ – ​ __ ​  × ​ _______  ​ + __
​   ​  × _______
______
​   ​  ÷​ a  – x  ​ 
{ 
2 2
    ​  ______
    ​
}
______
2 2 ​÷a  2 – x2 ​  2 ​÷a  – x2 
2
 ​ (iii) ÷​ 1  + x 2 ​  
y = tan  ​ ​ __________
–1 –  1
x    ​  ​, x π 0
a2 – ______ 2x2 + a2
= ____________
​       ​
2​÷a  2 – x2   ​ Put x = tanq


2(a2 – x2)
= _________
​  ______  ​ 
​  ( 
secq – 1
y = tan–1 ​ ________
tanq
    ​​
   )
2​÷a  2 – x2 
______
​ a  2 – x2 

 ​
( 
1 – cosq
y = tan–1 ​ ​ _______
sinq
   ​​
   )
( 
 ​
Hence, the result.
40. ______
​ 
2sin2 (q/2)
y = tan–1 ​ _______________
     ​  ​
2sin (q/2)cos (q/2) )
(i) y = tan–1 (​÷1  + x2 ​ 
+ x) q
y = tan–1 (tan (q/2)) = __
​   ​ 
Put x = tanq 2
–1
tan  x
y = tan–1 (secq + tanq) y = ​ ______  ​   
2

1 + sinq
y = tan–1 ​ ​ _______    ( 
​   ​ ) dy
​ ___  ​ = ________
1

( 
​     
 ​
cosq dx 2(1 + x2)

) )
1 + cos ​ __
–1 ______________
p
2 ( 
​   ​  – q  ​ ) (iv) { 
_____
÷​ 1  + x2  
–1  ​ + 1
y = tan  ​ ​ __________ ​, x π 0 }
y = tan  ​ ​         ​  ​ x    ​  
p
__
( 

( 
sin ​ ​   ​  – q  ​
2

)
Put x = tanq

y = tan  ​ ​  –1 ____________________
(  )
2cos2 ​ __
p q
​   ​  – __
4 2
      
​   ​   ​
 ​  ​
( 
secq + 1
y = tan–1 ​ ​ _______
tanq
   
​   ​ )
p __
__
(  ) (  )q p __
__
2sin ​ ​   ​  – ​   ​   ​ cos ​ ​   ​  – ​   ​   ​
4 2 4 2
q
( 
1 + cosq
y = tan–1 ​ ​ ________
sinq
     ​ ​
  )
y = tan–1 ​ cot ​ __
p q
(  (  ) )
​   ​  – __
4 2
​   ​   ​  ​ ( 
2 cos2 (q/2)
y = tan–1 ​ _______________
​         ​  ​
2 sin (q/2) cos (q/2) )
y = tan–1 ​ tan ​ __
p
2 (  (  (  ) ) )
​   ​  – ​ __
p q
​   ​  – __
4 2
​   ​   ​  ​  ​ y = tan–1 (2cos (q/2))

y = ​ __
p q
​   ​  + __
4 2 (  )
​   ​   ​ = __
p tan–1 x
​   ​  + ______
4
​   ​ 
2
  y = tan–1 ​ tan ​ __
p q
2 2 (  ( 
​   ​  – __
​   ​   ​  ​ ))
dy
​ ___  ​ = ________
​ 
dx 2(1 + x2)
1
     ​ y = ​ __
p q
​   ​  – __
2 2
​   ​   ​ (  )
(ii) y = tan–1 (​÷1  + x2 
_____
 ​ – x)
y = ​ __
p tan–1 x
​   ​  – ______
2
​   ​  
2 ( 
 ​ )
Put x = tanq dy 1
​ ___  ​ = – ​ ________      ​
y = tan–1 (secq – tanq) dx 2(1 + x2)

( 
______

1 – sinq
y = tan–1 ​ ​ _______
    ​  ​
  ) (v) y = cot–1 (​÷1  + x2 ​  + x)

( 
cosq
Put x = tanq
( 
) ( 
p

1 – cos ​ __
y = tan  ​ ​ _____________
–1
​   ​  –  q  ​
2
     ​  ​
)
secq + 1
y = cot–1 ​ ________
​ 
tanq
    ​​
   )
p
__
( 
sin ​ ​   ​  – q  ​
2 ) ( 
1 + cosq
y = cot–1 ​ ​ ________
sinq
   ​​
   )
Differentiation  5.27

( 
2 cos2 (q/2)
y = cot–1 ​ _______________ ) Put x = a cos q

( ÷ 
​        ​  ​ __________


2 sin (q/2) cos (q/2)
–1
y = cot  (cot(q/2))

a(1 – cosq)
y = tan  ​ ​ __________
–1
​     ​ ​  ​
a(1 + cosq) )
y = __
dy
q tan–1 x
​   ​  = ______
2
​   ​ 
2
  ​  ( 
sin (q/2)
y = tan–1 ​ _______
cos (q/2)
  ) q
 ​  ​ = tan–1 ​ tan ​ __
2 (  (  ) )
​   ​   ​  ​

(  )
1
​ ___  ​ = ________
​     
 ​ q cos–1 x
dx 2(1 + x2) y = ​ __ ​   ​   ​ = ______
​   ​   
2 2

{ 
_______ _______
​ 1  + sinx  
÷
(vi) y = tan–1 ​ ​ ___________________
_______    
​ 1  + sinx  
÷
​+ ÷​ 1  
    }
  –  sinx ​ 
_______  ​ ​, 0 < x < p
​ 1  – sinx  
​– ÷ ​
dy
​ ___  ​ = _______
​  _____
dx 2​÷1  – x2 
1
     ​
 ​

y = tan–1 ​ ________ ( cos (x/2)


​ 
sin (x/2) )  ​  
​ (v) ​  ( 
sinx
y = tan–1 ​ _______ )
 ​  ​, – p < x < p.
  
1 + cosx

(  (  ) ) (  )
x
y = tan–1 ​ cot ​ __ ​    ​  ​  ​ 2 sin(x/2) cos (x/2)
2 y = tan–1 ​________________
​    
  
 ​  ​

(  (  (  ) ) )
p
y = tan–1 ​ tan ​ ​ __ ​  – ​ ​ __  ​  ​  ​  ​
x 2cos2 (x/2)

y = __
p x
​   ​  – __
​    ​
2 2

x
2(  (  ) )
y = tan–1 ​ tan ​ ​ __  ​  ​  ​ = ​ __  ​
x
2
2 2 dy 1
​ ___  ​ = __ ​   ​ 
dy 1 dx 2
​ ___  ​ = – ​ __ ​ 
dx 2 (vi)
a + b tan x
y = tan–1 ​ _________
​  (     ​   )

(  )
41. b – a tan x
(i) Given ( 
y = tan–1 ​ ​ ______ 
a+x
)
1 – ax
  ​  ​
y = tan  ​ ​  a
a
__
​   ​ + tan x
b
–1 _________
   
 ​  ​
y = tan–1 (x) + tan–1 (a) 1 – ​ __ ​ tan x
b

(  )
dy 1 a
​ ___  ​ = _____
​       ​ –1 __
y = tan  ​ ​   ​  ​ + tan–1 (tan x)
dx 1 + x2 b

(ii)
a cosx – b sinx
y = tan–1 ​ _____________
​  {  }
p
    ​  ​ – ​ __ ​  < x < __
p
​   ​  –1 __a
y = tan  ​ ​   ​  ​ + x
b (  )

(  )
b cos x + a sin x 2 2 dy
a
__ ​ ___  ​ = 1
​   ​ – tan x dx
b
y = tan–1 ​ ​ _________
1 + __
a  
​   ​ tanx

 ​  ​
(vii)
a + bx
y = tan–1 ​ ______
​  (   ​   ​ )
(  )
b b – ax

(  )
a a
__
–1 __
y = tan  ​ ​   ​  ​ – tan–1 (tan x) ​   ​ + x
(  )
b
–1 _______ a
b y = tan  ​​  a   ​   ​ = tan–1 ​ __​   ​  ​ + tan–1 (x)

a
y = tan–1 ​ __
​   ​  ​ – x
b (  ) 1 – ​   ​ x__
b
b

dy
​ ___  ​ = – 1
dx
y = tan–1 ​ __
a
(  )
​   ​  ​ + tan–1 (x)
b

(iii)
3a2x – x3
y = tan–1 ​ ​ ________ (   ​  ​, – ___
  )
1 x
​  __  ​ < __
1
​   ​ < ___
    a ​÷3 ​
​  __  ​ 
dy
​ ___  ​ = _____
​     
1
 ​

 ( (  ) ((   )) )
a3 – 3ax2 ÷
​ 3 ​    dx 1 + x2

( 
x x 3

__
y = tan–1 ​ ​ ___________
__
3 ​ ​ a ​  ​ – ​​ ​ a ​  ​​ ​
x  
    ​  ​
(viii) y = tan–1 ​ ​ ______
1 + 6x2
x
     ​  ​
)
1 – 3 ​​ __
( 
2


3x
y = tan–1 ​ ___
​ a ​  ​​ ​

(  )
​ a ​   ​

3x – 2x
y = tan–1 ​ _______
​ 
1 + 6x
    
​  ​)
y = tan–1 (3x) – tan–1 (2x)
dy 1 3
​ ___  ​ = ________
​    2   ​ × __
​ a ​ dy 3 2
​ ___  ​ = _______ – ______
dx 3x
1 + ​​ ___
​ a ​   ​​ ​ (  ) ​     ​ 
dx 1 + 9x2 1 + 4x2
​       ​

(  )
dy 3a sin x +__cos x
​ ___  ​ = _______
​     ​  (ix) Given y = sin–1 ​ ___________
​   ​      ​
dx 9x2 + a2 ÷   
​ 2 ​

(  )
_____
( ÷ 
1 1
(iv)
a–x
y = tan–1 ​ ​ ​ _____ 
a+x
 ​ ​  ​
  ) = sin–1 ​ ___
​  __  ​ sin x + ___
÷   
​ 2 ​
​  __  ​ cos x  ​
÷   
​ 2 ​
5.28  Differential Calculus Booster


p
= sin–1 ​ sin ​ x + __
​   ​   ​  ​
4 (  (  )) = tan–1 ​ cot ​ __ (  (  ) )
x
​    ​  ​  ​
2
Thus,
p
__
y = ​ x + ​   ​   ​
4 (  ) = tan–1 ​ tan ​ __ (  (  ) )
p x
​   ​  – __
2 2
​    ​  ​  ​

(  )
dy p x
fi ​ ___  ​ = 1 fi y = ​ __
​   ​  – __
​    ​  ​
dx 2 2
(x)
1 – x2
y = sin–1 ​ ​ ______2  (  )
1 + x2
 ​  ​ + sec–1 ​ ​ ______2 
 ​  ​, x ŒR (  ) dy
fi ​ ___  ​ = – ​ __ ​ 
dx
1
2

(  )
1+x 1–x


1 – x2
y = sin–1 ​ ______
​ 
1 + x2
 ​ 
  ​ (  )
+ cos
1 – x2
–1 ______
 ​ ​ 
1 + x2
 ​  ​ (  ) 42. Given y = sin–1 ​ _____
​   2 
1+x
2x
(  ) 1 – x2
 ​  ​ + cos–1 ​ ​ _____2 
1+x
 ​  ​

p y = 2 tan–1 x + 2 tan–1 x = 4tan–1 x


y = __
​   ​ 
2
dy 4
dy ​ ___  ​ = _____
​      ​
​ ___  ​ = 0 dx 1 + x2
dx
( ÷  ) ( ÷  )
_____ _____
–1
x_____ x–1 Hence, the result.
–1
 ​ ​  ​ + cos  ​ ​ ​ _____ 
–1

( 
(xi) Given y = sin  ​ ​ ​   
   ​ ​  ​

(  )
x+1 x+1

p
y = __
​   ​ 
2
43. Given
x
y = sin–1 ​ _______
​  _____
   
÷​ 1  + x  
2
 ​ ) 1
 ​  ​ + cos–1 ​ _______
​  _____
   
 ​  ​
​÷x  + 1 ​
2

Put x = tan q

(  ( 
dy
fi ​ ___  ​ = 0
dx Then
tanq
y = sin–1 ​ __________ 
​  _________
) 1
   ​ ​ + cos–1 ​ __________
    ​ ​
​  _________
)
(  ÷​ 1  + tan q ​  ​÷1  + tan2q  
2

(xii) Given
3 sin x + 4 cos x
y = sin–1 ​ _____________
​   ​       ​ )
fi y = sin–1 ​ __
3
5
​   ​  sin x + __ ( 4
​    ​ cos x  ​ )
y = sin–1 ​ ____
​ 
secq(  )
tanq

1
 ​ ​ + cos–1 ​ ____
secq (  )
​      
​​
5 5
fi y = sin–1 (cos a  sin x + sin a  cos x) y = sin–1 (sinq) + cos–1 (cosq)
fi y = sin–1 (sin (x + a)) = (x + a) y = q + q = 2q = 2 tan–1 x
dy
(  )
2
dy
fi ​ ___  ​ = 1, where a = tan–1 ​ __
4
​   ​   ​ ​ ___  ​ = ______
​       ​
dx 3 dx 1 + x2

(  ) { 
_____ _____

}
1 – cos x
(xiii) Given y = tan–1 ​ _______
​      ​  ​ ÷​ 1  + x2  ​ + ÷​ 1  – x2 
 ​

(  )
sin x 44. Given y = tan  ​ ________________
–1
​  _____   
   ​  ​
_____

(  ) ÷​ 1  + x   ​ 1  – x  
 ​ – ÷
2 2
x  ​
2 sin2 ​ ​ __  ​  ​
–1 _____________ 2
= tan  ​ ​  x       x  ​  ​ x2 = cos(2q)
(  ) (  )
Put
2sin ​ __ ​    ​  ​ cos ​ __ ​    ​  ​

( 
(  )
__________ __________

)
2 2
​÷1  + cos(2q) ​
   + ÷ ​ 1  – cos(2q) ​
  

(  ) y = tan  ​ _________________________
–1
x Thus, ​  __________        __________  ​  ​
sin ​ ​ __  ​  ​ ​÷1  + cos(2q) ​
   – ÷ ​ 1  – cos(2q) ​
  
–1 _______ 2

( 
= tan  ​ ​  x    ​   ​
(  )
)
______ ______
cos ​ __ ​    ​  ​
2
  2q  
÷​ 2cos ​+ ÷
y = tan  ​ ________________
–1
​  ______  ______
    
​ 2sin 2
q ​ 
 ​ 
 ​
(  (  ) )  
÷​ 2cos ​– ÷   q  
x x 2 2
= tan  ​ tan ​ __
–1
​    ​  ​  ​ = __ ​    ​ q   ​ 2sin ​
2 2
dy 1
Thus, ​ ___  ​ = __
dx 2
​   ​ 
(  cosq + sinq
y = tan–1 ​ ___________
​       ​ 
cosq – sinq )
 ​

y = tan  ​( ________
(  1 – tanq )
1 +  tanq
(xiv) Given y = tan–1 ​ ________
​ 
cos x
  ​   ​) –1
​   ​ 
 ​

(  )
1 – sin x


2cos2 ​ __
–1 _____________
= tan  ​ ​ 
x
(  )
​    ​  ​
2
(  (  ) ) (  )
p
y = tan–1 ​ tan ​ __
4
p
​   ​  + q  ​  ​ = ​ __
​   ​  + q  ​
4
x       x  ​  ​
2sin ​ __ (  ) (  )
​    ​  ​ cos ​ __ ​    ​  ​
y = (​  __
p p 1
​   ​  + q )​ = __
​   ​  + __

(  )
–1 2
2 2 ​   ​  cos  (x )
4 4 2

= tan–1 ​ _______
​ 
x
__
cos ​ ​    ​  ​
2 (  ) dy 1 _____
× 2x
​ ___  ​ = – ​ _______    
x
 ​ = – ​ ______
    ​
x  ​   ​ _____
__
sin ​ ​    ​  ​
2 (  ) dx 2​÷1  – x  
4
 ​ ​÷1  – x4 
 ​
Differentiation  5.29

{  [  (  ) ]
_______ _______
45. Given
​÷1  + sinx  ​+ ÷
–1 ___________________
y = cot  ​ ​  _______        
​ 1  – sinx  

_______  ​  ​
}
q
y = sin ​ 2 ​ __
​   ​   ​  ​ = sinq

{ 
​÷1  + sinx  
​ – ​÷1  – sinx  ​ 2

}
________ _____

÷(   (  ) (  ) ) ÷(   (  ) (  ) )
_________________ ________________
x
​ ​​ cos ​ __
​    ​  ​ + sin ​
  
x 2
__
​    ​  ​  ​​ ​ ​ + ​ ​​ cos​ __
x
​    ​  ​ – sin ​
  
x 2
__
​    ​  ​  ​​ ​ ​ y = ​÷1  – cos2q   ​ = ​÷1  – x2   ​
2 2
–1 ______________________________________ 2 2

÷(   (  ) (  ) ) ÷(   (  ) (  ) )
= cot  ​ ​  _________________      
      ________________  ​  ​ dy 1 × (– 2x) x
x
__
​ ​​ cos ​ ​    ​  ​ + sin ​
  
x 2
__ x
​    ​  ​  ​​ ​ ​ – ​ ​​ cos ​ __
​    ​  ​ – sin ​
  
x 2
__
​    ​  ​  ​​ ​ ​ ​ ___  ​ = ​ ________
_____  ​   = – ​ ______   
_____
2 2 2 2 dx 2​÷1  – x  
2
 ​ ​÷1  – x2 

{  }
______
​( cos ​( ​ __  ​ )​ + sin(​( __​    ​ )​ )​ + ​( cos​( __
​    ​ )​ – sin ​( __
​    ​ )​ )​
x x x x  ​ 49. Given y = cos–1 (2x) + 2cos–1 (​÷1  – 4x2 
 ​)
2 2 2 2  ​
= cot   ​ ___________________________________
​  –1
    
     ​  ​
​( cos ​( __ ​    ​ )​ )​ – ​( cos ​( __
​    ​ )​ + sin ​( __ ​    ​ )​ – sin ​( __
Put 2x = sin q
​    ​ )​ )​
x x x x ________

{  }
2 2 2 2 Thus, y = cos–1 (sinq) + 2cos–1 ​÷1  – sin2q  

2cos ​( __ ​    ​ )​
x
2 y = cos–1 (sinq) + 2cos–1 (cosq)
________
= cot   ​ ​  –1
x  ​   ​
2sin ​( __
​    ​ )​
2 y = cos–1  ​ cos ​ __
p
(  (  ))
​   ​  – q  ​  ​ + 2cos–1 (cosq)
2

= cot   ​{ cot ​( __
​    ​ )​ }​ = __
x
–1
2
x
​    ​
2
y = ​ __
p
( 
​   ​  – q  ​ + 2q
2 )
dy
​ ___  ​ =
1
__
​   ​  = independent of x

p
2 ( 
y = ​ ​   ​  + q  ​ = __
__ p
2 )
​   ​  + sin–1 (2x)
dx 2 dy 2
​ ___  ​ = _______
​  ______     ​
Hence, the result. dx ​÷1  – 4x2   ​

46. Given
2x
(  )
y = tan–1  ​ _____
​   2 
1–x
1 + x2
 ​  ​ + sec–1  ​ ​ _____2 
1–x (  )
 ​  ​
50. Given ( 
​ 1  + x  
÷
_____
​ – ​÷1  – x  
y = tan   ​ ​ _______________
–1
_____
​ 1  + x  
÷ ​+ ÷
  
​)
   ​  ​
_____
​ 1  – x  

_____

)
(  ) (  )
2x 1 – x2 Put x = cosq
y = tan–1  ​ _____  ​  ​ + cos–1  ​ ​ _____2 

( 
​   2   ​  ​ ________ _______


1–x

y = 2tan–1 x + 2tan–1 x = 4tan–1 x


1+x

​÷1  + cosq  
​  ________
​ – ​÷1  – cosq ​
y = tan–1  ​ ____________________
       
_______


 ​  ​
​ + ​÷1  – cosq  
​÷1  + cosq   ​ )
dy ( 
cos(q/2) – sin(q/2)
y = tan–1  ​ ​ ________________
    ​  ​ )
 ( ((   )))
4
​ ___  ​ = _____
​      cos(q/2) + sin(q/2)
dx 1 + x2
q
1 – tan ​ __
​   ​  ​
 ​ 47. Given
x–1
(  )
y = sec–1  ​ ​ _____ 
x+1
x+1
 ​  ​ + sin–1  ​ ​ _____ 
x–1 (  )
 ​  ​ y = tan–1  ​ ​ _________
2
    ​  ​
q
__
1 + tan ​ ​   ​   ​
2
  ​( ​ _____    ​( ​ _____ 
x – 1) x – 1)
x+1 x+1
(  (  ) ) (  )
y = cos–1  ​  ​ + sin –1
 ​  ​ p q p q
y = tan–1  ​ tan ​ __
​   ​  – __
​   ​   ​  ​ = ​ __
​   ​  – __
​   ​   ​
p 4 2 4 2
y = __ ​   ​ 
y = (​  __
2 2) 2
–1
2 p q p cos  x
​   ​  – __
​   ​   ​ = __
​   ​  – ______
​   ​  
dy 2
​ ___  ​ = 0
dx dy 1

[  { ÷  } ]
_____ ​ ___  ​ = ​ _______
   
_____  ​
1–x dx 2​÷1  – x2 
48. Given y = sin ​ 2tan  ​ ​ ​ _____ 
–1
 ​ ​  ​  ​
   ​
1+x

( 
______
Put x = cosq
51. –1 x ​÷3  – 3x2 
y = cos  x + cos   ​ __
​    ​ + ​ _______
–1  ​
 ​   ​ )
[  { ÷ 
________ 2 2

1 – cosq
y = sin ​ 2 tan   ​ ​ ​ ________ 
–1
1 + cosq

 ​  ​  ​ }]
1
y = cos–1 x + cos–1  ​ __ ( 
÷
​   ​  ◊ x + ___
  
​ 3 ​
​   ​ ​÷1  – x2 
__
 ​  ​
_____
)
[  { ÷ 
2 2

}]
_________
2

(  )
2sin  (q/2)
y = sin ​ 2tan–1  ​ ​ _________
​       ​ ​  
​  ​
1
y = cos–1 x + cos–1  ​ __
​   ​   ​ – cos–1 (x)
2cos2 (q/2) 2

[  (  (  ) ) ]
q
y = sin ​ 2tan–1  ​ tan ​ __
​   ​   ​  ​  ​
2

1
y = cos–1  ​ __ (  ) p
​   ​   ​ = __
2
​   ​ 
3
5.30  Differential Calculus Booster
dy 57. We have 2x2 + 3xy + 3y2 = 1
​ ___  ​ = 0
dx dy dy
_____ _____
fi 4x + 3x ​ ___  ​ + 3y + 6y2 ​ ___  ​ = 0
​ – ​÷x  – x3 
52. We have y = sin–1 (x ​÷1  – x    ​) dx dx
_____ _____
__ dy
y = sin–1 (x ​÷1  – x    ÷ 1 – x   (3x + 6y2)  ​ ___  ​ = – (4x + 3y)
2
fi ​ – ​÷x 
  ​ ​  ​) fi
dx

( 
__
y = sin–1 x – sin–1 (​÷x 
)
fi    ​)
dy 4x + 3y
__ fi ​ ___  ​ = –  ​ ​ _______   ​  ​
dy 1 ​÷x 
   ​ dx 3x + 6y2
fi ​ ___  ​ = ______
​  _____
    ​ – _______
​  _____   ​
dx ÷ ​ 1  – x  
2
 ​ 2​÷1  – x   ​ 58. We have, ex + ey = ex + y
53. Given
dy
y = f (x) = x3 + x5 + x7 dy
fi ex + ey ​ ___  ​ = ex + y  ​ 1 + ___
dx ( 
dy
) dy
​    ​  ​ = ex + y + ex + y ​ ___  ​
dx dx
fi ​ ___  ​ = 3x2 + 5x4 + 7x6 dy dy
dx fi ex + ey ​ ___  ​ = ex + y + ex + y ​ ___  ​
dx 1 dx dx
Now, g¢(y) = ___
​    ​ = _____________
​       ​ dy
dy 3x2 + 5x4 + 7x6
fi (ey – ex + y) ​ ___  ​ = (ex + y – ex)
dx
When y = 3, 3 = x3 + x5 + x7

Thus, x=1
dy
fi ___
dx y
e –e ( 
ex + y – ex
​    ​ = ​ ​ ________  
x + y ) ( 
ex + ey – ex
 ​  ​ = ​ ​  _________
ey – ex – ey
  )
 ​  ​

1 1 dy ey
Therefore, g¢(3) = _________
​       ​ = ___
​    ​  fi ___
​    ​ = __
​  x  ​ = – ey – x
3 + 5 + 7 15 dx e
_____ _____
54. Given y = x5 + 2x3 + 2x 59. We have x ​÷1  + y  
​ + y ​÷1  + x  
​= 0
_____ _____
dy fi x ​÷1  + y  
​ = – y ​÷1  + x  

​ ___  ​ = 5x4 + 6x2 + 2
dx
fi x2 (1 + y) = y2 (1 + x)
dx 1
Now, g¢(y) = ___
​    ​ = ​ ____________
     ​ fi x2 + x2y = y2 + xy2
dy 5x4 + 6x2 + 2
fi x2 – y2 = xy2 – x2y
when y = – 5, then x = – 1
fi (x + y)(x – y) = – xy(x – y)
1 1
Therefore, g¢(– 5) = _________
​       ​ = ___
​    ​  fi (x + y) = – xy
5 + 6 + 2 13
fi y + xy = – x
55. Given y = f (x) = x3 + 2x2 + 3x + 4 fi y (1 + x) = – x
dy
fi ​ ___  ​ = 3x2 + 4x + 3
fi y = – ​ _____
1+x
x
  
1
 ​ = –  ​ 1 – _____
​     
1+x
 ​  ​ (  )
dx

Now,
dx
g¢(y) = ___
1
​    ​ = ___________
​       ​
fi y = ​ – 1 – ​ _____
1+x
1
  ( 
 ​  ​ )
dy 3x2 + 4x + 3 dy 1
fi ​ ___  ​ = – ​ _______
     ​
dx (1 + x)2
When y = 4, x3 + 2x2 + 3x + 4 = 4 _____ _____
Thus, x=0  ​ + x ​÷1  – y2 
60. We have y ​÷1  – x2   ​ = 1 ...(i)
1 1 Put x = sinq and y = sinj
Therefore, g¢(4) = _________
​       ​ = __
​   ​ 
0+0+3 3 (i) reduces to sinj  cosq + sinq cosj = 1
56. Given f and g are inverse of each other fi sin (q + j) = 1
Thus, f (g(x)) = g( f (x)) = x fi q + j = sin–1 (1)
fi f ¢(g(x)) ◊ g¢(x) = 1 fi sin–1x + sin–1y = sin–1(1)
1 1  dy

1
g¢(x) = _______
​     ​  fi ______
​  _____      ​ + _______
​  _____     ___
​  ​ = 0
f ¢(g(x)) ÷​ 1  – x2  ​ ÷ ​ 1  – y2    ​dx
1 dy 1
fi ​ 
1
g¢(x) = _________
     ​ fi ______
​  _____      ​ ___
​    ​ = – ​______
  _____      ​
1 + (g(x))n ​ 1  – y  
2 dx
 ​ ÷ ​÷1  – x2   ​
Differentiation  5.31

(  )
_____
dy ​÷1  – y2  
 ​ fi
q–j
cos​ ​ _____
 ​  
 ​ = a
fi ​ ___  ​ = – ​ _______
_____    ​ 2
dx ​÷1  – x2  ​

(  )
÷ 
_____ q–j
dy 1 – y2 fi ​ ​ _____  ​ = cot–1 (a)
 ​  
fi ​ ___  ​ = –  ​ ​ _____2 

 ​ ​ 2
dx 1–x
fi (q – j) = 2cot–1 (a)
Hence, the result.
61. Given sin y = x sin (a + y) fi sin–1 (x3) – sin–1 (y3) = 2 cot–1 (a)

sin y Differentiating w.r.t x, we get,


fi x = ________
​     ​
sin(a + y) 3x2 3y2 ___ dy
fi ​ ______
_____  ​ – ______
   ​  _____
   ​ ​    ​ = 0
Differentiate w.r.t y, we get,  ​ ​÷1  – y6 
÷​ 1  – x6   ​dx

dx sin(a + y) ◊ cosy – sin y ◊ cos(a + y) 3y2 ___ dy 3x2


fi ​ ___  ​ = ​ _____________________________
         ​ fi ​ ______
_____   ​ ​    ​ = ______
​  _____  
 ​
dy {sin(a + y)}2 ÷​ 1  – y6 
 ​dx ​÷1  – x6   ​
______
sin(a + y – y)
dx
fi ​ ___  ​ = ____________
​    
   ​ dy x2​÷1  – y6 ​ 
dy {sin(a + y)}2 fi ​ ___  ​ = _________
​  _____  ​ 
dx y2÷​ 1  – x6 
 ​
dx sina
fi ​ ___  ​ = _________
​     ​ 1 1
dy sin2 (a + y) 64. Given, x2 + y2 = t – __
​   ​  and x4 + y4 = t2 + __
​  2  ​ 
t t
dx 1 sin2(a + y)
fi ​ ___  ​ = ​ ___    ​= ​ _________   
​.  Now,
1
x2 + y2 = t – __
​   ​ 
dy dx
___ sin a t
​    ​
dy
62. We have log (x + y) = 2xy

1 2
(x2 + y2)2 = ​​ t – __ (  )
​   ​   ​​ ​
t

(  ) ( 
1
(x2 + y2)2 = t2 + ​ __2  ​  – 2
1
fi ​ ______
   
(x + y)
dy
 ​  ​ 1 + ___
dy
​    ​  ​ = 2  ​ x  ​ ___  ​ + y ◊ 1  ​
dx dx ) ...(i)

t
1
fi x4 + y4 + 2x2y2 = t2 + __
​  2  ​  – 2
when x = 0, then y = 1 t
Put x = 0 and y = 1 in (i), we get, 1 1
fi t2 + __
​  2  ​  + 2x2y2 = t2 + __
​  2  ​  – 2

(  ) ( 
t t
1
fi ​ ______
   
(0 + 1)
dy
 ​ ​ 1 + ___
dy
​    ​  ​ = 2 ​ 0 ◊ ​ ___  ​ + 1.1  ​
dx dx ) fi 2x2y2 = – 2

dy
fi ​ 1 + ___ ( 
​    ​  ​ = 2
dx ) fi


x2y2 = – 1

y2 = ___
– 1
​  2 ​ 
dy x
fi ​ ___  ​ = 1. dy
dx 2
fi 2y ​ ___  ​ = __​    ​ 
_____ _____ dx x3
63. Given ​÷ 1 – x6  ​ 1  – y6 
 ​ + ÷  ​ = a(x3 – y3)
dy 1
fi ​ ___  ​ = ___
​     

Put x3 = sinq, y3 = sinj dx x3y
The given equation reduces to
cosq + cosj = a(sinq – sinj) 65. Given ( 
x+y
sec ​ ​ _____
x – y  )
 ​  ​ = a

fi (  )
q+j
2cos ​ ​ _____
2 (  )
q–j
 ​ cos ​ ​ _____
 ​  
2
 ​  
 ​
(  )
x+y
fi ​ ​ _____
x–y  ​ 
  ​ =
sec–1 (a)
_______
​ 
1
 ​   

( q+j
= 2a cos ​ ​ _____
Z 

2 ) (  )
q–j
 ​ sin ​ ​ _____
 ​  
2
 ​  
 ​
 ​ xx ++ yy +– xx  
fi ​(____________     )​ ​ = ​ __________
–y
+y
sec  (a) + 1
    ​
–1

–1

(  ) (  )
sec  (a) – 1
q–j q–j
fi cos ​ ​ _____  ​ = a sin ​ ​ _____
 ​    ​  
 ​
2 2
5.32  Differential Calculus Booster

sec–1 (a) + 1 Hence, the result.


x
fi ​ __y ​ = ___________
​  –1   
   ​ 68. Given y = x sin y
sec  (a) – 1
dy dy
y sec–1 (a) – 1 fi ​ ___  ​ = sin y + x cos y ​ ___  ​
fi ​ __x ​ = ​ __________
    ​ dx dx
sec–1 (a) + 1
dy
fi ​ ___  ​ (1 – x cos y) = sin y

(  ) dx
–1
sec  (a) – 1
fi y = ​ ​ __________
    ​  ​ x
sec–1 (a) + 1 dy sin y
fi ​ ___  ​ = ​ __________
     ​
dx (1 – x cos y)
dy sec–1 (a) – 1
fi ​ ___  ​ = ​ __________
    ​ dy y
dx sec–1 (a) + 1 fi ​ ___  ​ = ___________
​       ​
dx x (1 – x cos y)
dy y
fi ​ ___  ​ = __
​ x ​ Hence, the result.
dx
69. Given cos y = x cos (a + y)
Hence, the result.

(  )
cos y
2 2 fi x = _________
​     ​
 x – y cos(a + y)
66. Given tan–1 ​ ​ ______   ​  ​ = a
x2 + y2
dx – cos(a + y)siny + cosy sin(a + y)
fi ​ ___  ​ = ​ ____________________________

(  )
        
 ​
x –y 2 2 dy cos2 (a + y)
tana
fi ​ ​ ______   ​  ​ = ____
​   ​ 

x2 + y2 1 dx sin(a + y – y)
fi ​ ___  ​ = ​ ____________
  
   ​

( 
dy cos2 (a + y)
x2 – y2 + x2 + y2
fi ​ ​  _____________
2 2
x –y –x –y
  
2 2 ) tan a + 1
 ​  ​ = ​ ________ 
tan a – 1
 ​
dx sin(a)
fi ​ ___  ​ = __________
​ 
dy cos2 (a + y)
  
  ​

x2 tan a + 1
fi – ​ __2 ​  = ​ ________  ​ 2
tan a – 1 dx cos  (a + y)
y fi ​ ___  ​ = ​ __________
   ​ 
dy sin(a)
y2 1 – tan a
fi ​ __2 ​  = ​ ________  ​ Hence, the result.
x tan a + 1
70. Given y = xsin x
fi ( 
1 – tan a 2
y = ​ ​ ________ 
2
tan a + 1
 ​  ​ x ) fi log(y) = log ​( xsin x )​


dy
dx ( 
1 – tan a
2y ​ ___  ​ = 2 ​ ​ ________ 
tan a + 1
 ​  ​ x ) fi log (y) = sinx log x

__1 dy ____ sinx


fi ​ y ​ ___
​    ​ = ​  x   
​ + cos x log x
( 
dx
dy x 1________
fi ​ ___  ​ = __
​   ​ ​ ​ 
dx y 1 + tan a
– tan a
  ​  ​ )

dy
___ (  sinx
​    ​ = y ​ ____
dx
​  x    ​ + cos x log x  ​)
Hence, the result.

67. Given xy = 1

dy
___
dx
(  sinx
​    ​ = xsinx ​ ____
​  x    )
​  +  cos x log x  ​


1
y = __
​ x ​ 71. Given y = (sin x)cos x
cos x
dy 1 fi y = elog (sin x)​​ ​
fi ​ ___  ​ = – ​ __2  ​ 
dx x fi y = ecosx log (sin x)

( 
dy dy
fi ​ ___  ​ = – y2
dx
cos x
)
fi ​ ___  ​ = ecos xlog (sin x) ​ cos x ◊ ​ ____ ​ – sin x log sin x  ​
dx sin x
dy dy
fi ​ ___  ​ + y2 = 0 fi ​ ___  ​ = (sin x)cos x (cos x ◊ cot x – sinx log sin x)
dx dx
Differentiation  5.33

72. Given y = (tan x)cot x + (cot x)tan x

fi y = e  log(tan x)​cot x
​ ​+ e  log(cot x)​tan x
​ ​
dy
fi ​​ ___ (  ) (  1
)
​    ​  ​​ ​ = 2 ​ log 2 – ​ __  ​  ​ + 2 = log 4 + 1
dx x = 1 2
75. Given y = sinx ◊ sin2x ◊ sin3x ... sin (2014) x
 cot x log(tan x)  tan x log(cot x)
fi y = e +e
log y = log (sinx ◊ sin2x ... sin (2014) x)
dy sec2x
fi ​ ___  ​= e cot x log(tan x) ​ cotx ◊ ​ _____ ​ 
dx tanx ( 
– cosec2 x ◊ log(tan x)  ​
1 ___
dx
)
logy = log sinx + log sin x2x + ... + log sin(2014)x

​ __
+e  tan x log(cot x)
 ​ tanx ◊ ​  ( 
– cosec2x
________
cotx
​ 
    2
+ sec x ◊ log(cot x)  ​ )
y ​ ​ dy  ​ = cot x + 2 cot 2x + ... + 2014 cot (2014) x

dy
dy
fi ___
​    ​ = (tanx)cotx (cosec2x – cosec2x ◊ log(tanx)) ​ ___  ​ = y (cotx + 2cot2x + ... + 2014 cot (2014)x)
dx dx

+ (cot x) tan x (– sec2x + sec2x ◊ log(cot x)) 76. Given x y = ex – y



73. Given xm xn = (x + y)m + n fi y logx = (x – y)

fi log (xm xn) = log((x + y)m + n) fi y (1 + log x) = x


x
fi m logx + n log y = (m + n)log (x + y) fi y = _________
​       ​
(1 + logx)
  

m __
fi ​ __
n ___ dy
x ​  + ​ y ​ ​ dx  ​ =
(m + n)
​ _______ ​ ​
(x + y)
dy
 1 + ___ ( 
​    ​  ​
dx ) dy (1
___
fi ​    ​ = ​ 
1
+ logx) – x ◊ ​ __
_____________
  
    ​
x ​

(  ( 
dy dx 2
n
fi ​ __
​ y ​ –
m + n ___
​ _____  ) m+n
 ​  ​ ​    ​ = ​ ​ _____ 
x + y dx x+y
 ​ –
m
__
)
​ x ​   ​
(1 + logx)

dy (1 + logx) – 1

( 
fi ​ ___  ​ = ​ ___________
  
   ​
nx + ny – my – ny ___
fi ​ _________________
​   ​ 
  
y(x + y)
dy
   ​ ​    ​
dx ) dx (1 + logx)2

logx
( 
dy
fi ​ ___  ​ = _________

mx + nx – mx – my
= ​ ​ _________________
x(x + y)
 ​ 
     ​ )
​ 
dx (1 + logx)2
    ​

Hence, the result.


nx – my ___
fi ​ ​ _______ 
dy
(  nx – my
 ​  ​ ​    ​ = ​ ​ _______ 
y(x + y) dx x(x + y)
 ​  ​ ) (  ) 77. Given xy = yx

(  )
fi y log x = x log y
1 dy
fi ​ __
dx
1
​ y ​  ​ ​ ___  ​ = ​ __
​ x ​  ​ (  ) y dy x dy
fi ​ __x ​ + log x ​ ___  ​ = log y + __
​ y ​ ​ ___  ​
dy y dx dx
fi ​ ___  ​ = __
​   ​
dx x x dy
(  (  )
y
fi ​ log x – __ )
​ y ​  ​ ​ ___  ​ = ​ log y – __
​ x ​  ​
1 x
74. Given y = ​​ 1 + __ ( 
​ x ​  ​​ ​ + x​ ​(​ 1 + ​ x ​ )​​
)1
__ dx

(  y
)
( 
​ log y – __
)
dy ​ x ​  ​ y x log y – y
(  )​ + ​e​(​ 1 + ​ __1x ​ )​logx​
1
x log​ 1 + ​ __ fi ​ ___  ​ = ​ _________
x    ​ = __
​ x ​ ​ ​ ________   ​  ​
( 
fi y = ​e​ x ​  ​
dx ​ logx – __ )
​ y ​  ​ y log x – x

​ ( 1 + ​ x ​ )​​ ​ log​ x + __
(  (  ) )
dy 1
__ 1 1 1
fi ​ ___  ​ = e​ x log​ ​ x ​  ​ + x ​ _______
     ​ ◊ – ​ __2  ​   ​ 78. Given ey = yx,
dx
​ 1 + __
1
​ x ​  ​ x (  ) fi log (e y) = log(yx)
(  )
( (  ) )
1
__ 1 __ 1 __ 1
  + e​ ​​ 1 + ​ x ​  ​log x​ ​ ​ 1 + ​ __
x ​  ​ ​ x ​ – ​ x2  ​  logx  ​ fi y log(e) = x log (y)
fi y = x log(y)
dy
fi ___
dx
1 x
​    ​ = ​​ 1 + __ (  1
) (  ( 
​ x ​  ​​ ​ ​ log​ x + __
1
​ x ​  ​ – ______
​     
(1 + x)
 ​  ​ ) ) dy x dx
fi ​ ___  ​ = log(y) + __
​ y ​ ​ ___  ​
dx dy

( ( 
+ x​ ​(​ 1 + ​ x ​ )​​ ​ ​ 1 + __ ) ) dy
(  )
1
__ 1 1 __ 1 x
  ​ x ​  ​ ​ __
x ​ – ​ x2  ​  logx  ​ fi ​ ___  ​ ​ 1 – __
​ y ​  ​ = log (y)
dx
5.34  Differential Calculus Booster

dy
dx
1
fi ​ ___  ​ ​ 1  – ​ _____( 
   ​  
log(y)
​ = log (y) ) ​ 
1+x
8x7
+ _____  8 
1006x1005
 ​ ... + ________
​ 
1 + x1006
  ​

(  )
dy dy 1006x1005
fi ​ ___  ​ (log(y) – 1) = {log (y)}2 1
​ ___  ​ = y ​ _____
​     
2x
 ​ + _____
​   2  ​ + ... + ________
​   ​  

dx dx 1+x 1+x 1 + x1006
dy {log(y)}2
fi ​ ___  ​ =
(  )
__________
​      ​ dy
dx (log(y) – 1) ​​ ___
​    ​  ​​ ​ = 1(1 + 1 + 1 + ... + 1) (1006-times) = 1006.
dx x = 0
79. Given xm yn = 1
84. We have
_________________________
___________________
fi log(xm yn) = log(1) = 0 ____________
  + ÷​ cosx
y = ​÷cosx   +   
​÷cosx
      to • ​ ​ ​
+ ...   
fi m log x + n log y = 0
dy f ¢(x)
fi ​ ___  ​ = ​ _______   
 ​
m __ n ___ dy dx (2y – 1)
fi ​ __
x ​  + ​ y ​ ​ dx  ​ = 0
dy – sinx
dy my fi ​ ___  ​ = ______
​    
 ​
fi ​ ___  ​ = – ​ ___ dx 2y – 1
dx nx ​ 
sinx cosx ____ sinx ____
cosx
Hence, the result. 85. We have y = ____
​   ​ ​  ____ ​ ​    ​ ​    ​  
...
1+ 1+ 1+ 1+
80. Given e x + y = x
sinx sin x
fi y = ____________
​       ​ = ​ ________
    ​
fi (x + y) = logx cosx
_________ cosx
1 + ​      ​ 1 + _____
​    
 ​
sinx 1+y
dy 1 1 + ______
​    
 ​
fi 1 + ___
​    ​ = __
​   ​ 1  + ...
dx x
(1 + y) cos x
dy 1 1–x fi y = ___________
​      ​
fi ​ ___  ​ = __
​   ​ – 1 = ​ _____
x   ​  1 + y + cos x
dx x
fi y + y2 + y cos x = sin x + y sin x
82. Given y = (x + 1)(x + 2)(x + 3) ... (x + 2012)
dy dy dy
logy = log(x + 1) + log(x + 2) + ... + fi ​ ___  ​ + 2y ​ ___  ​ + cosx ​ ___  ​ – ysinx
log(x + 2012) dx dx dx
Differentiating w.r.t x, we get, dy
= cosx + sinx ​ ___ ​ 
dx
dy ______
1 ___ 1 1 1
​ __  ​ + ______
y ​ ​ dx  ​ = ​ (x + 1)  ​       ​ + ... + _________
​       ​ dy
(x + 2) (x + 2012) fi (1 + 2y + cos x – sin x) ​ ___  ​ = cosx + y sinx
dx
dy
dx ( 1
​ ___  ​ = y ​ ______
​     
1
 ​ + ______
​     
(x + 1) (x + 2)
​ 
1
 ​ + ... + _________
   
(x + 2012)
 ​  ​ ) dy cosx + ysinx
fi ​ ___  ​ = ​ __________________
dx
       ​
(1 + 2y + cosx – sinx)
Put x = 0, we get, __________________
_____________
_________
86. Given y = ​÷x  + ÷
​ x  + ÷
​ x  +   to • ​ ​ ​
...    
dy
dx ( 
1 1
​ ___  ​ = (2012)! ​ 1 + __
​   ​  + __
2 3
1
​    ​ + ... + _____
​     ​  ​
2012 ) fi
_____
y = ​÷x  + y  

fi y2 = x + y
83. Given y = (1 + x)(1 + x2)(1 + x4)(1 + x8) ...
dy dy
(1 + x1006) fi 2y ​ ___  ​ = 1 + ___
​    ​
dx dx
logy = log(1 + x) + log (1 + x2) + log (1 + x4) +
dy
fi (2y – 1) ​ ___  ​ = 1
dx
log (1 + x8) + ... + log (1 + x1006)
dy 1
Differentiating w.r.t x, we get, fi ​ ___  ​ = _______
​       ​
dx (2y – 1)
1 ___
dx ______ 1 2x 4x3
​ __  ​ + _____
y ​ ​ dy  ​ = ​ (1 + x)  ​   2  ​ + _____
​    
 ​ Hence, the result.
1+x 1 + x4
Differentiation  5.35

87. Given y = x​x ... to •​ 1 1 ___


91. Given y = x + ___
​    ​ ___
1
​    ​ ​    ​ .... to •
y x + x + x +
fi y=x
1
fi log(y) = y log x fi y = x + __
​ y ​
dy
1 ___ y dy
fi ​ __ __
​ x ​ + logx ​ ___  ​ fi y2 = xy + 1
y ​ ​ dx  ​ = dx

( 
Differentiating w.r.t x, we get,
1
fi ​ __
dy y
​ y ​ – log(x)  ​ ​ ___  ​ = __
dx
​ x ​ ) dy dy
dy y2 fi 2y ​ ___  ​ = y + x ​ ___  ​
dx dx
fi (1 – y log(x)) ___ ​    ​ = __ ​ x ​ 
dx dy
dy y2 fi (2y – x) ​ ___  ​ = y
dx
fi ​ ___  ​ = ____________
​       ​
dx x(1 – ylog(x)) dy y
fi ​ ___  ​ = _______
​       ​
Hence, the result. dx (2y – x)
______________________
________________
_________
88. Given y=÷   + ÷​ sinx
​ sinx        
+ ​÷sinx ... to • ​ ​ ​
     Hence, the result.
_______
  + y   sinx cosx ____ sin x ____
cos x
fi y=÷ ​ sinx ​ 92. Given y = ____
​   ​  ◊ ​ ____ ​  
◊ ​   ​  
◊ ​   ​  ... to •
2
1 + 1 + 1 + 1+
fi y = sinx + y
sin x
dy dy fi y = ________
​  cos x     ​
fi 2y ​ ___  ​ = cos x + ___
​    ​ 1 + ____
​   ​

dx dx 1+y
dy
fi (2y – 1) ​ ___  ​ = cos x (1 + y) sin x
dx fi y = ​ ___________
    
(1 + y) + cos x
dy cos x
fi ​ ___  ​ = _______
​    
 ​
dx (2y – 1)  ​ fi y {(1 + y) + cos x} = (1 + y)sin x
Hence, the result. fi y¢{(1 + y) + cos x} + y(y¢ – sin x)
x + to •

89. Given y = ​e​ x + ​ex + e 

​ = (1 + y) cos x + y¢ sin x
x + y
fi y=e when x = 0 and y = 0, then
dy
dx
dy
fi ​ ___  ​ = e x + y ​ 1 + ___
​    ​  ​
dx (  ) fi
fi 2y¢ = 1
y¢(1 + 1) + 0 = 1 + 0

dy
fi ​ ___  ​ = y ​ 1 + ___
dx
dy
( 
​    ​  ​
dx ) fi y¢ = __
1
​   ​ 
2
dy dy 1
fi (1 – y) ​ ___  ​ = y fi ​ ___  ​ = __
​   ​ 
dy dx 2
dy y x x x x
fi ​ ___  ​ = ​ _____
    ​ ​    ​  ◊ ​ ___  ​ ◊ ​ ___  ​  ◊ ​ ___  ​ ............... to •
93. Given y = ___
dx 1 – y 1+ 2+ 1+ 2+
x x(2 + y)
y = _______  ​ = ​ _______ 
(tan x) ... to •
90. Given y = (tan ​x)​(tan x) ​ fi ​   x     ​
1 + ​      ____  ​ 2 + y + x
fi y = (tan x) y 2+y
fi log (y) = y log (tan x) fi y(2 + y + x) = x(2 + y)

(  ) fi 2y + y2 + xy = 2x + xy
y
1 ___dy dy sec2x
fi ​ __ ​ ​ 
y dx ​

  = ___
log (tanx) ​   
 ​ + ​​ _____
​   ​ 
  ​​ ​
dx tanx fi 2y + y2 = 2x
p
when x = __
​   ​ , y = 1, then fi
dy dy
2​ ___  ​ + 2y ​ ___  ​ = 2
4 dx dx
dy dy
fi ​ ___  ​ = 0 ◊ ​ ___  ​ + 2 dy dy
dx dx fi ​ ___  ​ + y ​ ___  ​ = 1
dx dx
dy dy
fi ​ ___  ​ = 2 fi (1 + y) ​ ___  ​ = 1
dx dx
5.36  Differential Calculus Booster

dy dy 1

1
fi ​ ___  ​ = ______
​       ​ fi ​ ___ ​  = ​ _______    
_____  ​
dx (1 + y) dt ​÷1  – t 2  

(  )
 ​
1 – t2
94. We have x = a (t – sin t), y = a (1 – cos t) Also, x = cos–1 ​ ​ _____2 
 ​  ​
1+t
dx dy
fi ​ ___  ​ = a(1 – cos t) and ​ ___ ​  = a sin t fi x = 2 tan–1 t
dy dt
  

dx 2
dy
​ ___ ​  fi ​ ___ ​  = _____
​      ​
dy dt a (1 – cos t) dt 1 + t 2
Thus, ​ ___  ​ = ​ ___  ​ = ​ __________   
   ​
dx ___ dx a sin t dy ______ 1
​   ​  ​  _____      ​
dt ​ ___ ​ 
dy dt ÷ 1  – t 2  
​______ 1+ t 2
 ​ _______
2 sin2 ​ __ (  )
t
​    ​ ​
fi ​ ___  ​ = ​ ___  ​ = ​    ​= ​  _____   

(  )
dx dx 2
___ ______  ​ 2​÷1  – t  
2
dy 2 1 ​   ​  ​    2 
fi ​ ___  ​ = ​ _____________       ​ = tan ​ __
​   ​ ​
(  ) (  )
t t__ dt 1+t
dx 2 sin​ __ ​    ​ ​ cos ​ ​    ​ ​ 2

95. We have
2 2
 ​
( 
3 sin t + 4 cos t
 ​ 98. Given x = sin–1 ​ ​ ____________
5
 ​ 
     ​ )
( 
1 1
fi x2 – y2 = t – __ ​   ​  and x4 + y4 = t 2 +
t
__
​  2  ​ 
t
fi x = sin–1 ​ __
3 4
​    ​ sin t + __
5
​    ​ cos t  ​
5 )
fi 2 2 2 1
(x – y ) = ​​ t – __ (  )
​   ​   ​​ ​
t
2
fi –1 
x = sin (sin (t + a)) = t + a
dx
1 fi ​ ___ ​  = 1
fi x4 + y4 – 2x2 y2 = t 2 + __ dt
( 
​  2  ​  – 2


t
x4 + y4 – 2x2 y2 = x4 + y4 – 2
6 cos t + 8 sin t
Also, y = sin–1 ​ ​ ____________
10
 ​       ​)
fi – 2x2 y2 = – 2
fi y = sin–1 ​ ___
10
6
( 
​    ​  cos t + ___
8
​    ​  sin t  ​
10 )
fi x2 y2 = 1 fi y = sin–1 ​ __
3
5 ( 
​    ​ cos t + __
4
​   ​  sin t  ​
5 )
1
y2 = __
–1 
fi ​  2  ​  fi y = sin (sin (t + b))
x fi y = (t + b)
dy 2
fi 2y ​ ___  ​ = – __ ​  3  ​  fi ​ ___ ​  = 1
dy
dx x dt
dy 1 dy
fi y ​ ___  ​ = – __ ​  3  ​  ​ ___ ​ 
dx x dy dt 1
Thus, ​ ___  ​ = ​ ___  ​ = ​ __ ​  = 1.
dy dx ___ dx 1
fi x3 y ​ ___  ​ + 1 = 0 ​   ​ 
dx dt
dy dy/dt cos t
99. ​ ___  ​ = _____ ​ = ​ _____  
1
96. Given x = a ​ t + __ (  ) 1
​   ​   ​ and y = a ​ t – __
t
​   ​   ​
t (  ) dx dx/dt
​ 
– sin t
 ​

dy

(  ) (  )
dx 1 dy 1 ​ ___  ​ = – cot(t)
fi ​ ___  ​ = a ​ 1 – __
​  2  ​   ​ & ​ ___ ​  = a ​ 1 + __
​  2  ​   ​ dx

(  )
dy dt
(  )
t dy 2p
​​ ​ ___  ​  ​​ ___​ = – cot ​ ​ ___ ​   ​
dy 1
__
(  ) (  )
​ ___ ​  a ​ 1 + ​  2  ​   ​ a​ t + __ 1
​   ​   ​
dx x = ​ 2p ​ 
3
3
dx
dy
dt
dx
dt
t
fi ​ ___  ​ = ​ ___  ​ = ​ ________ 
1
​ ___ ​   a ​ 1 – __
​  2  ​   ​ a ​ t – __
t
 ​ = ​ _______ 
(  ) (  ) 1
​   ​   ​
t
x
 ​ = __
​ y ​ dy
(  )
​​ ​ ___  ​  ​​ ___
dx x = ​ 2p ​ 
3
p
​ = cot ​ __ (  ) 1
​   ​   ​ = ___
3
​  __  ​ 
÷   
​ 3 ​
t
Hence, the result.
( 
_____
5t + 12​÷1  – t 2 
97. Given y = cos ​ ​  ____________
13
–1 
 ​ 
    
 ​
​ ) dy dy/dt
100. ​ ___  ​ = _____
dx dx/dt
​ 
esin2t × 2 cos (2t)
 ​ = ​  _______________
  
    ​
ecos2t × – 2sin (2t)
( 
______

13
5
y = cos–1 ​ ___
12
​   ​ ​ ÷1  – y2 
​    ​ t  + ___
13
 ​ ​ ) dy y × log(x)
​ ___  ​ = – ​ _________ 
dx x × log (y)

 ​

y = cos ​( ___
​    ​  )​ – cos
5 –1  –1  dy dy/dq
fi t 101. ​ ___  ​ = ​ _____   ​

13 dx dx/d q
Differentiation  5.37


2 sin (2 q) – 2 sin(q)
=​ _________________
  
   ​
2 cos (q) – 2 cos (2q)

3
y = sin–1 (sin (a + sin t)), a = tan–1 ​ __
​   ​   ​
4 (  )
​( __
​   ​  )​
3
sin (2q) – sin(q) and x = sin–1 (sin (b + sin t)), b = tan–1
=​  _______________
  
    ​ 4
cos (q) – cos (2 q)
fi y = a + sin (t), x = b + sin (t)

= ​ 
2 cos ​ ___
3q
2
_______________
(  ) (  )
​   ​   ​ sin​ __
     
q
​   ​ ​
2
 ​
dy dy/dt
​ ___  ​ = _____
​   ​ =
cos t
​ ____ 
 ​ = 1
(  ) (  ) q dx dx/dt cos t
2 sin ​ ​   ​   ​ sin ​ __
___
3q
​   ​ ​
2 2 Hence, the result.

(  )  
_______ _______
3q
cos​ ___
​   ​   ​ (​÷1  + sin t  
106. Given x = tan ​  ____________________
–1  ​ 1  – sin t  
 ​+ ÷  ​
= ​  _______ 2
 ​  ( )
= cot ​ ___
3q
​   ​   ​
_______    
​ 1  + sin t  
÷
   
 ​– ÷
_______  ​
​ 1  – sin t  
​)
3q
sin ​ ___ (  ) 2 ______

( 
​   ​   ​
2
102. Given x = a ​ t + __ (  )
1
​   ​   ​ and y = a ​ t – __ (  ) 1
​   ​   ​
÷​ 1  +  t2  
and y = tan  ​ ​ _________
–1
t
– ​ 1
 ​  ​. )
(  (  ) )
t t
t
dy dy/dt fi x = tan–1 ​ cot ​ __ ​    ​   ​  ​
fi ​ ___  ​ = ​ _____   ​ 2

(  (  ) )
dx dx/dt q

(  ) (  )
and y = tan ​ tan​ __
–1 
​   ​   ​  ​, t = tan (q)
1 2
a ​ 1 + __ ​  2  ​   ​ a​ t + __ 1
​   ​   ​
dy t t –1 
___ ________ _______ q tan (t)
(  ) (  )
fi ​    ​ = ​  t
dx 1

 ​ = ​ 
1
  ​ fi x = __
​    ​ , y = __
​   ​  = ​ _______
 ​  
__
a​ 1 – ​  2  ​   ​ a​ t – ​   ​   ​ __ 2 2 2
t t
1
________
​       ​

fi ​    ​ = ​ 
dy a​
___
t + __
_______
(  )
1
​   ​   ​
t x
 ​ = __
  ​ y ​
dy _____
___ dy/dt
fi ​    ​ = ​ 
dx dx/dt
2 (1
 ​ = ​ 
1
+ t 2) ______
________
 ​ 
1
 = ​  2     ​

(  )
dx 1 __
​   ​  (t + 1)
a​ t – ​   ​   ​__ 2
t

103. Given x = sin–1 ​ ​ _____


2t
1 +t (  )
 2   ​  ​ and y = tan–1 ​ ​ _____
(  ) 2t
  
1 – t2
 ​  ​
107. Let u = f (x2 + 2012) and v = f (x3 + 2013)

du d(f (x2 + 2012))
Thus, ​ ___  ​ = ​ _____________
  
    ​
fi x = p – 2 tan–1 (t), y = 2 tan–1 (t) dv d (f (x3 + 2013))
1
– ​ ______
 2  
 ​ du   f ¢(x2 + 2012) ◊ 2x
dy dy/dt (1 + t) fi ​ ___  ​ = _______________
​  3     ​
fi ​ ___  ​ = _____
​    ​ ​ ________  ​
  = –1 dv f ¢(x + 2013) ◊ 3x2
dx dx/dt 1
______
​    2    ​ 2
(1 + t ) du 2 f ¢(x + 2012)
fi ​ ___  ​ = ___
​    ​ ​ ____________
    ​.
Hence, the result. dv 3x f ¢(x2 + 2013))

( 
_____
5t + 12​÷1  – t 2 
104. Given y = cos ​ _____________
​ 
13
 ​  –1 
    
 ​
​ ) 108. Let u = xx and v = x log x
du d(x x)
Thus, ​ ___  ​ = ​ ________

(  )
  ​ 
1 – t2 dy dv d(x log x)
and x = cos–1 ​ ​ _____2 
 ​  ​, then find ​ ___  ​
1+t dx
xx (1 + log x)
= ​ ____________  ​ = xx
(  )
  
  
5 (1 + log x)
fi y = cos–1 (t) – cos–1 ​ ___
​    ​  ​, x = 2 tan–1 (t)

{ 
13 ______ ______
dy
dx
dy/dt
fi ​ ___  ​ = ​ _____ 
dx/dt
 ​
÷​ 1  + x2  
109. Let u = tan  ​ ________________
–1
​  ______
÷​ 1  + x  
2
 ​+ ÷
​ 1  – x2 
– ​ ÷
  
______
​ 1  – x  
2
 ​
 ​
}
    ​  ​, v = cos–1 (x2)

( 
1


 –  ​ _______
_________
= ​ 
   
_____
​  1 – t2 
÷  ​
 ​
 ​


+ t2
1 _____
= – ​  _______    ​.

cos q – sin q
u = tan–1 ​ ___________
​       ​ 
cos q + sin q )
  ​, x2 = cos (2 q)
2
_____ 2​÷ 1 – t2 
(  ( 
 ​
))
​    2 
 ​ p
1+t fi u = tan–1 ​  tan ​ __ ​   ​  – q  ​  ​
4
105. Given x = sin–1 ​ ​ ____________
5 ( 
3 sin t + 4 cos t
 ​       ​ ) fi
p
u = ​ __(  )
​   ​  – q  ​ = __
4
p 1
​   ​  – __
4 2
​   ​  cos–1 (x2)

and y = sin–1 ​ ​ ____________


10( 
6 cos t + 8 sin t
 ​ 
     ​, ) fi
p 1
u = __
​   ​  – __
4 2
​   ​  cos–1 (x2); v = cos–1 (x2)
5.38  Differential Calculus Booster

du du/dx
fi ​ ___  ​ = ______
dv dv/dx
​ 
1
 ​ = – __
​   ​ 
2
d 2y
fi ​ ___2  ​ = ___
dx
b
​     ​ ​ – ​ __
dq a
dq
 ​ cot q  ​ ◊ ​ ___  ​
dx
(  )
2x
110. Let u = sin–1 ​ ​ _____
1 +x
 2 
(  ) 1 – x2
 ​  ​ and v = cos–1 ​ ​ _____2 
1+x
 ​  ​ (  ) d 2y
fi ​ ___2  ​ =
dx
( b
)– 1
​ a ​ cosec2 q  ​ ◊ ​_____
​ __  
a sinq
   

fi u = 2 tan–1 (x) and v = 2 tan–1 (x)


2/(1 + x2)

( a
b
= ​ –  ​ __2  ​  cosec3q  ​
)
(  )
du du/dx  _________
fi ​ ___  ​ = ______
​   ​ = ​    ​= 1
dv dv/dx 2/(1 + x2) b __ b3
= ​ – ​ __ ◊ ​

a y3 ​   ​
 ​

111. Let u =  (sin x), v = f (cos x)
b4
du du/dx
fi ​ ___  ​ = ______ ​   ​  = – ​ ___3  ​ 
dv dv/dx ay

(  ) (  )
du du/dx   f ¢(sin x) cos x d2 x ___
fi ​ ___  ​ = ​ _____   ​ = – ​ ___________    ​
   115. ​ ___2 
d dx d dx dx
​     ​ ​ ​ ___  ​  ​ ◊ ​ ___  ​
 ​ = ​     ​ ​ ​ ___  ​  ​  = ___
dv du/dx f ¢(cos x) sin x

 ( )
dy dy dy dx dy dy
112. We have y = c1ex + c2 e–x
d 1 ___ dx
dy = ​ ___   ​​ ___
​    ​   ​ ◊ ​    ​
fi ​ ___  ​ = c1ex – c2 e–x dx ___ dy dy
dx ​    ​
dx
d2 y d 2y 1
fi ​ ___2  ​ = c1 ex + c2 e–x = y = _____
–1 ___
​   2  ​  ◊ ​  2  ​ ◊ ​___
    ​ 
dx

fi ​ ___2 
d2 y
(  )
dy   dx ___
___
​​ ​    ​  ​​ ​
dx
dy
​    ​
dx

(  )
 ​ – y = 0.
dx
113. We have x = at2, y = 2 at
(  )
dy – 3 d2y
= – ​​ ​ ___  ​  ​​ ​ ◊ ​ ​ ___2  ​  ​
dx dx
dx dy 116. Given y = e2x
fi ​ ___ ​  = 2 at & ​ ___ ​  = 2a
dt dt dy
fi ​ ___  ​ = 2e2x
dy dx
dy ​ ___ ​ 
dt 2a 1
Now, ​ ___  ​ = ​ ___  ​ = ​ ___   ​ = __ d 2y
dx ___ dx 2at
​   ​ 
t fi ​ ___2  ​ = 4 e2x
​   ​  dx

(  ) (  )
dt
d2 x ___ d dx dx d dx dx
d 2y d dy
fi ​ ___2  ​ = ___
​     ​ ​ ​ ___  ​  ​ (  ) fi ​ ___
2
 ​ = ​     ​ ​ ​ ___  ​  ​ ◊ ​ ___  ​ = ___

dy dy dy
​     ​ ​ ​ ___  ​  ​ ◊ ​ ___  ​
dx dy dy

 ( )
dy
dx dx dx

(  )
d 1 ___ dx
d dy dt = ___
​     ​  ​ ___
​    ​   ​ ◊ ​    ​
fi = __
​      ​​ ​ ___  ​  ​ ◊ ​ ___   ​ dx ___ dy dy
dt dx dx ​    ​
dx

d 1 ___
= __
dt t dx (  )
​     ​ ​ __
dt
​   ​   ​ ◊ ​     ​

– 1   ___
= _____
​   2 ​ 
d 2y 1
◊ ​  2  ​ ◊___
​    ​ 

1 1
= – ​ __2  ​  ◊ ​___
t 2at
      ​ (  )
dy dx ___
___
​​ ​    ​  ​​ ​
dx
dy
​    ​
dx


1
= – ​ ____
2at3
   ​ (  ) ( 
dy –3 d2y
= – ​​ ​ ___  ​  ​​  ​ ◊ ​ ​ ___2  ​  ​
dx dx )
114. We have x = a cos q, y = b sinq
(  ) (  )
2
d y d2 x
dy Thus, ​ ​ ___2  ​  ​ ​ ​ ___2 
 ​  ​.
dx
fi ​ ___ ​ = – a sinq & ​ ___ ​ = b cosq dx dy
dq dq
dy
​ ___ ​
d 2y
= ​ ___2  ​ ◊ –
dx
(  )dy – 3 d2y
​​ ​ ___  ​  ​​ ​​ ___  ​
dx dx2
fi ​ ___ ​ = ​ ___  ​ = ​ _______ 
dx dq b cosq b
​  = – __
   ​ a ​ cot q

(  ) (  )
dq ___ dx – a sinq d2y 2 dy –3
​   ​
dq = ​​ ​ ___2  ​  ​​ ​ ​​ ​ ___  ​  ​​ ​

(  )
dx dx
(  )
2
d y d dy d b
fi ​ ___2  ​ = ___
​     ​ ​ ​ ___  ​  ​ = ___
​     ​ ​ – __
​ a ​ cotq  ​
dx dx dx dx = (4e2x)2 (2e2x) –3
Differentiation  5.39

16e4x |y¢¢|
= _____
​  6x ​  fi k = ​ _________
    ​
8e (1 + y¢ 2)3/2
= ___
2
​  2x   ​
e
2

[  x x2
x + yy¢ = 0 fi y¢ = – ​ __y ​ fi y2 = ​ __2  ​ ​
y ​
= __
​ y ​. x2 x + y
2 2
a2 

fi 1 + y¢2 = 1 + ​ __2  ​ = ​ ______2
 ​  = ​ ___2 ​ 

__y y y y
117. We have (a + bx)​e​ ​x ​​ = x

]
2
a a
__y x fi y2 = ​ ______  ​ fi |y| =  ​ ​_______
      
______  ​  ​
fi ​e​​ x ​ ​= ______
​       ​ 1 + y¢2 ​​÷1  + y¢2   ​
a + bx

( 
y x
fi ​ __x ​ = log ​ ______
​     
a + bx
 ​  ​ ) 119. We have y = eax sin bx
dy
y
fi ​ __x ​ = log x – log (a + bx) fi ​ ___  ​ = a eax sin bx + eax ◊ b cos bx
dx
dy d 2y
x ◊ ​ ___  ​ – y fi ​ ___2  ​ = a 2eax sin bx + abeax cos bx
dx 1 b a + bx – bx
fi ​  ________  ​   = __
​ x ​ – ______
​       ​ = ​ __________
   ​  dx
x 2 a + bx x(a + bx) + abeax cos bx – b2eax sin bx
dy ax2 ax y2 – 2ay1 + (a2 + b2) y
fi x ​ ___  ​ – y = ​ ________     ​ = ​ ______
    ​
dx x(a + bx) a + bx = a2eax sin bx + abeax cos bx
2
dy dy dy (a + bx)◊a – ax ◊ b + abeax cos bx – b2eax sin bx
fi x ​ ___2  ​ + ​ ___  ​ ◊ 1 – ​ ___  ​ = ​ ______________
    
 ​
dx dx dx (a + bx)2 – 2a(aeax sin bx + beax cos bx)
2
dy a2 + (a2 + b2)eax sin bx
fi x ​ ___2  ​ = ​ ________
    ​
dx (a + bx)2 = eax (a2 sin bx + 2 ab cos bx – b2 sin bx

( 
2
dy a2 x 2 – 2a2 sin bx – 2ab cos bx + (a2 + b2) sin bx)
fi x3​ ___  ​ = ​ ________
dx 2
(a + bx)
 2 ​ 
a
ax 2
= ​​ ​ ______
  ​   
+ bx
​​ ​ ) = 0.

( 
Hence, the result.

d 2y dy 2
x3 ​ ___  ​ = ​​ x ​ ___  ​ – y  ​​ ​.
dx2 dx ) 120. Given y = 2 sin x + 3 cos x
dy
Hence, the result. fi ​ ___  ​ = 2 cos x – 3 sin x
dx
118. We have x2 + y2 = a2
d 2y
d 2y
(  ) fi ​ ___2  ​ = – 2 sin x – 3 cos x
2 4
b b b 3 a____ b


​ ___  ​ = – __ ​  2  ​  cot3 q = – __ ​  2  ​​​  __
​ y ​  ​​ ​ = ​  2 3  ​ dx
2
dx a a a y
d 2y

dy
2x + 2y ​ ___  ​ = 0 fi ​ ___2  ​ = – 2 (sin x + 3 cos x) = – y
dx dx
dy d 2y
fi x + y ​ ___  ​ = 0 fi ​ ___2  ​ + y = 0
dx dx
fi (  )
dy 2 d 2y
1 + ​​ ​ ___  ​  ​​ ​ + y ​ ___2  ​ = 0
dx dx
Hence, the result.
121. Given y = x + tan x


d 2y dy 2
– y ​ ___2  ​ = 1 + ​​ ​ ___  ​  ​​ ​ (  ) dy
fi ​ ___  ​ = 1 + sec2 x
dx

|  |
dx

(  ) (  )
|  |
dy 2 dy 2 d 2y
2 1 + ​​ ​ ___  ​  ​​ ​ 1 + ​​ ​ ___  ​  ​​ ​ fi ​ ___2  ​ = 2 sec2 x tan x
d y dx dx dx
fi ​ ​ ___2  ​  ​ = ​ ​  ________
– y    ​ = ​  ________
​    ​   
dx |y| d 2y
fi cos2 x ​ ____2 ​ = 2 tan x
+ y¢2)3/2
(1 + y¢2) (1_________ dx
fi |y¢¢| = ​ _______
 ​ 
 = ​  a   
​ 
|y| d 2y
Now, cos2 x ​ ____2 ​ – 2y + 2x
1 _________|y¢¢| dx
fi ​ __
a ​ = ​ (1 + y¢   
2 3/2
 ​
) = 2 tan x – 2 (x + tan x) + 2x
5.40  Differential Calculus Booster

= 2 tan x – 2x – 2 tan x + 2x = 2 ex (cos x – sin x) – 4ex cos x + 2ex (sin x + cos x)


= 0. = 2 ex (cos x – sin x – 2  cos x + sin x + cos x)
122. Given y = tan x + sec x = 2ex (0)
dy = 0.
fi ​ ___  ​ = sec2 x + sec x tan x
dx Hence, the result.
dy 1 + sin x Given y = sin–1 (x)
126.
fi ​ ___  ​ = ​ _______    ​  dy
dx cos2 x 1
fi ​ ___  ​ = ______
​  _____
   
dy 1 + sin x 1 dx ​÷1  – x2 
fi ​ ___  ​ = ​ ________  ​ = ​ _______
        ​
dx 2 
1 – sin x 1 – sin x d 2y (2x)
 ​ fi ​ ___2  ​ = ​ _____________
    
_____
dy – cos x cos x
fi ​ ___  ​ = ​ _________     ​= ​ _________
   ​  ​ dx 2(1 – x2)​÷1  – x2 
dx (1 – sin x)2 (1 – sin x)2
d 2y _______ x dy
Hence, the result.  ​ fi (1 – x2) ​ ___2 
 ​ = ​  ______
     ​= x ​ ___  ​
123. Given y = A cos (log x) + B sin (log x)  ​ dx ÷​ 1  – x  
2
 ​ dx
d 2y dy
dy A sin (log x) __________
fi ​ ___  ​ = –  ​ __________
b cos (log x) fi (1 – x2) ​ ___2 
 ​ – x ​ ___  ​ = 0
dx x     ​  + ​  x     ​  dx dx
dy Hence, the result.
fi x ​ ___  ​ = – A sin (log x) + B cos (log x) 127.
dx dy dy/dt ___ 2a 1
(i) ​ ___  ​ = _____
​   ​ = ​     ​ = __
​   ​ 
d 2y dy A cos (log x) __________
B sin (log x) dx dx/dt 2at t
fi x ​ ___2  ​+ ​ ___  ​ = – ​ __________
x     ​ 
– ​  x       ​

(  ) (  )
dx dx d 2y d dy d 1
2 fi ​ ___2  ​ = ___
​     ​​ ​ ___  ​  ​ = ___
​     ​ ​ __
​   ​   ​
dy dy dx dx dx dx t
fi x2​ ___  ​ + x ​ ___  ​ = – (A cos(log x) + B sin(log x)
dx2 dx
d 2y dy
d 2y
dx
d 1 ___
fi ​ ___2  ​ = __
​     ​​ __
dt t dx (  )
dt
​   ​   ​ ​    
fi x2 ​ ___  ​ + x ​ ___  ​ = – y
dx 2 dx
d 2y 1 1 1
dy 2
dy  ​ fi ​ ___2  ​ = – ​ __2  ​  × ___
​      ​ = – ____
​   3 
 ​
fi x2 ​ ___  ​ + x ​ ___  ​ + y = 0 dx t 2at 2at
dx2 dx
dy dy/dq 3 a sin2q cosq
Hence, the result. (ii) ​ ___  ​ = ______
​   ​ = – ​ ___________
         ​
dx dx/dq 3 a cos2q sin q
124. Given y = tan–1x
dy dy
fi ​ ___  ​ = _____
1
​       ​ fi ​ ___  ​ = – tanq
dx 1 + x2 dx


dy
(1 + x ) ​ ___  ​ = 1
dx
2
d 2y
dx
d dy
fi ​ ___2  ​ = ___
dx dx (  ) d
​     ​​ ​ ___  ​  ​ = ___
​     ​ (– tanq)
dx

d 2y dy d 2y d dq
fi (1 + x2) ​ ___2  ​ + 2x ​ ___  ​ = 0 fi ​ ___2  ​ = ___
​     ​(– tanq)​ ___  ​
dx dx dq dx
dx
fi (1 + x2) y2 + 2xy1 = 0 d 2y 1
fi ​ ___2  ​ = sec2q × ___________
​      ​
dx 3 a cos2q sinq
125. Given y = ex (sin x + cos x),
dy
fi ​ ___  ​ = ex (sin x + cos x) + ex (cos x – sin x)
dx
dy

a3
= ​ _________
3x (axy)
a8/3
= _______
 1/3 ​  ​  4     ​
  =
1 ___
__
​ 
3(x y)1/3 3 x4 y

 ​ ​​
(  )
​ 
a8 1/3
   
​  ​​ ​

fi ​ ___  ​ = 2 ex cos x 128. Given x = a secq, y = b tanq


dx
d 2y d 2y b 1
fi ​ ___2  ​ = 2 ex (cos x – sin x) fi ​ ___2  ​ = – ​ __ ​ cosec q cot q × _________
​      ​
dx dx a asec q tan q

(  )
d 2y dy d 2y b4
Now, ​ ___2  ​ – 2 ​ ___  ​ + 2y b b b 3
fi ​ ___2  ​= – ​ __2  ​  cot3 q = – ​ __2  ​​​  __
​ y ​  ​​ ​ = –  ​ ____  ​ 
dx dx dx a a a2 y3
Differentiation  5.41

( 
129. Given y = sin–1 x
dy 1
134. Given y = x log ​ ______
x
​     
a + bx
 ​  ​ )
fi ​ ___  ​ = ______
​  _____
   
( 
y
dx ÷ ​ 1  – x2  fi ​ __x ​ = log ​ ______
x
​     
a + bx
 ​  ​ )
d 2y (– 2x) y
 ​ fi ​ ___2  ​ = – ​ _____________ 
  
_____ fi ​ __x ​ = log x – log (a + bx)
 ​ dx 2(1 – x2)​÷1  – x2 
dy
d 2y x dy x ◊ ​ ___  ​ – y
 ​ fi (1 – x2)​ ___2  ​ = _______
​  ______      ​= x ​ ___  ​ dx 1 b a + bx – bx
fi ​  ________  ​   = __
​ x ​ – ​ ______      ​ = ​ ___________
  
   ​
 ​ dx ÷​ 1  – x2    ​ dx x 2 a + bx x(a + bx)
2
dy dy dy ax2
fi (1 – x2)​ ___2  ​ – x ​ ___  ​ = 0 fi x ​ ___  ​ – y = ​ ________   
ax
 ​ = ​ ______   ​
dx dx dx x (a + bx) a + bx
fi (1 – x2) y2 – xy1 = 0 d 2y dy dy (a + bx)a ◊ – ax ◊ b
Hence, the result. fi x ​ ___2  ​ + ​ ___  ​ ◊ 1 – ​ ___  ​ = ​  _______________
   ​
  
dx dx dx (a + bx)2
130. **
d 2y a2
131. ** fi x ​ ___2  ​ = ​ ________
    ​
132. Given x = a (q + sin q), y = a (1 + cos q) dx (a + bx)2

( 
dy d 2y a2 x 2
– a sin q
fi ​ ___  ​ = ​ __________
dx a (1 + cos q)

  
sin q
 ​ = – ​ ________
1 + cos q
   ​ fi x 3​ ___  ​ = ​ ________
dx 2
(a + bx)
 2 
a
ax 2
 ​ = ​​ ​ ______
+
  
bx
 ​  ​​ ​ )
2sin​ __
q
​   ​   ​ cos ​ __ (  ) (  )
q
​   ​   ​ d 2y
( dy 2
x3​ ___  ​ = ​​ x  ​ ___  ​ – y  ​​ ​. )
(  )
dy 2 2 q fi
fi ​ ___  ​ = –  ​ _____________     ​ = – tan​ __
   ​   ​   ​ dx2 dx
dx
2cos2​ __
q
​   ​   ​
2 (  ) 2 _____
135. Given ​÷x  + y   ​ y  – x  
 ​+ ÷  ​= c
_____

(  ) (  ) (  )
2
d y 1 q 1 1 dy 1 dy
fi ​ ___2  ​ = – ​ __ ​  sec2 ​ __ ​   ​   ​ × __________
​       ​ fi ​ _______   ​ ​ 
_____ 1+ ​ ___  ​  ​ + _______
​  _____
    ​​ ​ ___  ​ – 1  ​  =  0
dx 2 2 a(1 + cos q) 2​÷x  + y ​  dx 2​÷y  – x  ​ dx
_____ _____ _____ _____
d 2y 1 __________1 dy (​÷x  + y   ​)(​÷x  + y  
 ​– ​÷y  – x    ​+ ​÷y  – x  
 ​)
fi ​ ___2  ​= – ​ __
a  ​ ​       ​ fi ​ ___  ​ = ​ _______________________________
    
_____    
_____  ​
dx (1 + cos q )2 dx c(​÷x  + y    ​+ ​÷y  – x ​) 
d 2y 1 __ a2 a dy (x + y – y + x)
fi ​ ___2  ​= – ​ __ a  ​ × ​  2 ​  = – ​ __2  ​  fi ​ ___  ​ = ​ _____________ ​
    
dx y y dx c2
Hence, the result. dy 2x
fi ​ ___  ​ = ​ ___2 ​ 
133. Given x = a (1 – cos q), y = a (q + sinq), dx c
2
d y 2
dy a(1 + cosq)
​ ___  ​ = ​  __________
   
   ​ fi ​ ___2  ​ = __​  2  ​ 
dx a sin q dx c

(  )
cos4 a _____
sin4 a _____ 1
2 cos ​ __
q 2 
​   ​   ​ 136. We have ​ _____
x   

​ + ​  y   
​ = ​     
 ​

x+y

 ( ) (  ) ( )
dy 2 q
​ ___  ​ = – ______________    ​ = cot ​ __ ​   ​   ​
(  )
​    
dx q
2 sin ​ __
​   ​   ​ cos ​ __
2
q
​   ​   ​
2
2 x+y
fi ​ ​ _____ (  )
x    ​  ​ cos4 a + ​ ​ _____
x+y
y    ​  ​ sin4a = 1

dy
​     ​ ​( cot ​( __
2
d
​ ___  ​ = ___ ​   ​  )​ )​ × ​ ___  ​
q dq fi ​ 1 + __ (  )
y
(  )
​ x ​  ​ cos4 a + ​ 1 + __
x
​ y ​  ​ sin4 a = 1
dq 2 dx y
(  ) (  )
2
dx x
fi ​ __
​ x ​  ​ cos4a + ​ __ ​ y ​  ​ sin4a + (sin4a + cos4 a) = 1

​ ___  ​ = – ​ __ ​  cosec ​( __


2
​   ​  )​ × _____
dy q
dx
1
2 2
2 
2
1
​      
a sinq
​ y
fi ​ __ (  ) (  )
​ x ​  ​ cos4a + ​ __
x
​ y ​  ​ sin4 a + 1 – 2 sin2 a cos2 a = 1

cosec ​( __ (  ) (  )
y
​   ​  )​
q x
2
2  fi ​ ​ __x ​  ​ cos4a + ​ ​ __y ​  ​ sin4 a – 2  sin2 a cos2 a = 0
dy
___ 1 _________
___ 2
​    ​ = – ​    ​   ​      ​
  __
( ÷  ) ( ÷  )
__
dx2 2 a sinq y 2
x 2  2
fi ​​ ​ __
​ x ​ ​ cos

2 
a  ​​ ​ + ​​ ​ __
​ y ​ ​ sin
  a  ​​ ​

(  )
2

– 2​( ​ __
÷​  y ​ ​ sin a )​ ​( ​÷__​  x ​ ​ cos a )​ = 0
dy 1 2 1 __ __
​​ ​ ___2  ​  ​​q = ​ __p ​ ​ = – ___
​     ​ × __
​   ​  = –  ​ __
a ​
x

2  y 2 
dx 2 2a 1  
5.42  Differential Calculus Booster

( ÷ 
__

÷  )
__
x y 2
139. Given P (x) = ax3 + bx2 + cx + d
fi ​​ ​ __
​ y ​ ​  sin2 a – ​ __
​ x ​ ​ cos

2 
a  ​​ ​ = 0
P ¢(x) = 2 ax2 + 2bx + c
fi ​( ​ __
__

÷​  y ​ ​ sin a – ​÷​  x ​ ​ cos a )​ = 0


__
x
  __y 2 

2  P ¢¢(x) = 4ax + 2b
__ P ¢¢(0) = 10 fi 2b = 10 fi b = 5

÷  ÷ 
__
x 2 y
fi ​ ​ __y ​ ​ sin
  a = ​ __
​ x ​ ​ cos

2 
a P(0) = – 2 fi d = – 2
P(1) = – 2 fi a + b + c + d = – 2
fi y = (x) tan2 a
  a + 5 + c – 2 = – 2
dy
fi ​ ___  ​ = tan2 a   a + c = – 5 ...(i)
dx
P ¢(0) = – 1 fi c – 1
137. Given y = x sin x
From (i), we get, a = – 4
dy
fi ​ ___  ​ = x cos x + sin x Hence, the value of (a + b + c + d + 10)
dx
= – 4 + 5 –1 –2 + 10
d 2y = 8
fi ​ ___2  ​ = cos x – x sin x + cos x
dx 140. Given f (x) = x + tan x
d 2y Since f is the inverse of g, so (f 0 g) (x) = x
fi ​ ___2  ​ = 2 cos x – x sin x
dx f (g (x)) = x
dy 2
dy f ¢(g (x)) g¢ (x) = 1
Now, x2​ ___  ​ – 2x ​ ___  ​ + (x2 + 2) y
dx2 dx 1
g¢(x) = _______
​     ​ 
2
x (2 cos x – x sin x) – 2x (x cos x + sin x) f ¢(g (x))
1 1
+ (x2 + 2) x sin x g¢(x) = ____________
​        ​ = _______________
​       ​
1 + sec (g(x)) 1 + 1 + tan2 (g(x))
2 
= x2 (2 cos x – x sin x + x sin x – 2 cos x)
1
+ x (2 sin x – 2 sin x) g¢(x) = ____________
​       ​
2 + tan2 (g (x))
= x2 (0) + x(0) = 0
Hence, the result. 141. Given f (x) = ex + x3 – 1
138. Given y = f (x) = x2 f ¢(x) = ex + 3x2
dy f ¢¢(x) = ex + 6x
fi ​ ___  ​ = 2x
dx when y = f (x) = e, then x = 1
2
dy f ¢¢(x)
fi ​ ___2  ​ = 2 g¢¢(x) = – ​ _______    ​
dx {f ¢(x)}3
Also, x2 = y f ¢¢(e) e+6
g¢¢(e) = – ​ _______    ​ = – ​ _______
    ​

dx
2x ​ ___  ​ = 1 {f ¢(e)}3 (e + 3)3
dy
142. Given f (x) = 1 + x3

d x
2x ​ ___2 
dy
2
(  )
dy
dx
2
 ​ + 2 ​​ ​ ___  ​  ​​ ​ = 0 fi

f ¢ (x) = 3x2
f ¢¢(x) = 6x

(  )
2
d x dx 2
fi f ¢¢¢(x) = 6
fi x ​ ___2 
 ​ + ​​ ​ ___  ​  ​​ ​ = 0
dy dy
when y = 2, then x = 1
2

d x ___
x ​ ___2 
1 f ¢¢(x)
 ​ + ​   2 ​ = 0 Now, g¢¢(y) = – ​ _______    ​
dy 4x {f ¢(x)}2
d2 x 1
fi ​ ___2  ​ = – ​ ___ 3 ​  f ¢(x) f ¢¢¢(x) – 3{f ¢¢(x)}2
dy 4x fi g¢¢¢(y) = – ​  ___________________
     ​  
{f ¢(x)}3
d 2y
dx (  ) (  )d2 x
Now, ​ ​ ___2  ​  ​ × ​ ​ ___2 
dy 4x
1
2x
1
 ​  ​ = 2 × – ​ ___ 3 ​ = – ​ ___ 3 
 ​

f ¢(1) f ¢¢¢(1) – 3{ f ¢¢(1)}2
g¢¢¢(2) = – ​ ____________________
   
{f ¢(1)}3
 ​   
Differentiation  5.43

3 × 6 – 3(6)2
fi g¢¢¢(2) = – ​ ____________
 ​
    
(3)3 1 1 1

72 8 + 1 1 1
fi g¢¢¢(2) = ___
​   ​ = __
​   ​   a3b1 a3b2 a3b3
27 3
143. Given f (g (x)) = x =0+0+0
fi f ¢(g (x)) g¢(x) = 1 = 0.
1 x +1 x + 2 x + 3
fi g¢(x) = ______
​       ​
f ¢(g(x)) 146. We have f(x) = x + 4 x + 5 x + 6
1 x+7 x+8 x+9
fi g¢(x) = ___________
​       ​
1 + (f (g (x)))2
1 1 1
1
fi g¢(x) = _____
​    2 
 ​ fi f ¢(x) = x + 4 x+5 x+6
1+x
x+7 x+8 x+9
Hence, the result.
x3 sin x cos x x +1 x + 2 x + 3
144 We have f (x) = 6 -1 0 + 1 1 1
p p 2
p 3 x+7 x+8 x+9

3x3 cos x - sin x x +1 x + 2 x + 3


fi f ¢(x) = 6 -1 0 + x+4 x+5 x+6
p p2 p3 1 1 1
fi f ¢¢(x) = 0
6 x - sin x - cos x On integration, we get,
fi f ¢¢(x) = 6 -1 0 fi f (x) = c1
p p2 p3 Again, integrating, we get,
fi f (x) = c1x + c2
6 - cos x sin x
Hence, the result.
fi f ¢¢¢(x) = 6 -1 0
2 3
1 2 x 3x2
p p p
147. We have x x2 x3
6 -1 0 0 2 6x
fi f ¢¢¢(x) = 6 -1 0 =0
2 3 0 2 6x 1 2 x 3x2
p p p
fi f ¢(x) = x x2 x3 + 1 2 x 3x2
3
d (0)
Thus, ​ _____ ​ = 0 0 2 6x 0 2 6x
dx3
1 2 x 3x2
(1 + x )a1b1 (1 + x )a1b2 (1 + x )a1b3
+ x x2 x3
145. Let f (x) = (1 + x )a2b1 (1 + x )a2b2 (1 + x )a2b3
0 0 6
(1 + x )a3b1 (1 + x )a3b2 (1 + x )a3b3
fi f ¢(x) = 0 + 0 + 6 (x2 – 2x2) = – 6x2
Differentiating both sides w.r.t x and then putting fi f ¢(1) = – 6
x = 0, we get, the co-efficient of x 148. We have
a1b1 a1b2 a1b3 1 1 1 cos x sin x cos x
1 1 1 + a2 b1 a2 b2 a2 b3 f(x) = cos2 x sin 2 x 2 cos2 x
1 1 1 1 1 1 cos3 x sin 3 x 2 cos3 x

5.44  Differential Calculus Booster

- sin x cos x - sin x 6 (–1)3 3!


fi y3 = – ​ _______  ​ = ​ _________
  4    
 ​
fi f ¢(x) = cos2 x sin 2 x 2 cos2 x (x + 1) (x + 1)3 + 1
cos3 x sin 3 x 2 cos3 x (– 1) 4 4!
fi y4 = ​ _________     ​
(x + 1) 4 + 1
cos x sin x cos x
+ - 2sin 2 x 2 cos2 x - 4sin 2 x ... ... ...
cos3 x sin 3 x 2 cos3 x ... ... ...
(– 1​)​n​(n)!
cos x sin x cos x fi yn = ​ _________  

 ​
(x + 1​)n + 1​ ​
+ cos2 x sin 2 x 2 cos2 x
(– 1​)​n​(n)!
- 3sin 3 x 3cos3 x - 6sin 3 x Thus, yn (0) = ​ _________  ​ = (–1​)​n​(n)!


(0 + 1​)​n + 1​
-1 0 -1 0 1 0
(  )p 152. We have y = sin x
__
(  )
fi f ¢​ ​   ​   ​ = - 1 0 - 2 + 0 - 2 0 p
2 fi y1 = cos x = sin ​ __
​   ​  + x  ​
0 -2 0 0 -1 0 2

0 1 0
fi (  p
)
y2 = – sin x = sin ​ 2 ◊ ​ __ ​  + x  ​
2
+ -1 0 - 2
3 0 6
fi (  )
y3 = – cos x = sin ​ 3 ◊ ​__
p
   ​  + x  ​
2
... ... ...
fi (  )
p
f ¢​ __
​   ​   ​ = (2 – 1) – 1 (6 – 6) = 1
2


...
...
...
...
...
...
149. We have y = xn
fi y1 = e x

n p
yn = sin ​ ___ ( 
​   ​ + x  ​
2 )


y2 =
y3 =
ex
ex
n p
Thus, yn (0) = sin ​ ___
2 (  ) n p
​   ​ + 0  ​ = sin ​ ___ (  )
​   ​  ​
2
... ... Ï1 : n = (4 k + 1), k Œ1
Ô
... ... = Ì- 1 : n = (4 k - 1), k Œ1
fi yn = e x Ô0 : n = 2 k , k Œ1
Ó
Thus, yn (0) = e0 = 1 153. We have y = xn
150. We have y = ax
y1 = n xn–1
fi y1 = ax log a y2 = n (n – 1) x n–2
fi y2 = ax (log a)2 y3 = n (n – 1) (n – 2)x n–3
fi y3 = ax (log a)3 ... ... ...
... ... ...
... ... ...
yk = n (n – 1) (n – 2)...(n – (k – 1))x n–k+1
... ... ...
Thus, yk (1) = n (n – 1) (n – 2)...(n – (k – 1))
fi yn = ax (log a) n
= (n – (k – 1))!
Thus, yn (0) = a 0 (log a) n = (log a) n 154. We have y = A sin x + B cos x
1
151. We have y = ______
​       ​ fi y1 = A cos x – B sin x
(x + 1)
fi y2 = – A sin x – B cos x = – y
1 fi
fi y1 = – ​ _______
    ​ y2 + y = 0
(x + 1)2 By Leibnitz theorem, we get,
2 fi
fi y2 = _______
​       ​ yn+2 + (0)yn+1 + (0) yn + yn = 0
(x + 1)3 fi yn+2 + yn = 0
Differentiation  5.45

155. We have y = tan–1 x By Leibnitz theorem, we get,


1
fi y1 = _______
​      ​ (1 – x2) yn+2 –  2 (n + 1) x yn+1 – n(n – 1) yn
(1 + x2)
– x yn+1 – n yn = 0
fi (1 + x2)y1 = 1
fi (1 – x2)yn+2 – (2n + 1) xyn+1 – n2yn = 0
fi (1 + x2) y2 + 2x y1 = 0
By Leibnitz theorem, we get, 157. We have y = a cos (log x) + b sin (log x)
2n (n – 1)
(1 + x2) yn+2 + (2 (n + 1)x) yn+1 + ​ ​ ________
2
 ​   ​yn (  ) fi
1
y1 = – a sin (log x)​ __ __1
x ​ + b cos (log x) ◊ ​ x ​
+ 2x yn+1 + (2n)yn = 0 fi x y1 = – a sin (log x) + b cos (log x)

fi(1 + x2) yn+2 + (2 (n + 2)x) yn+1 + n (n + 1) yn = 0 1 1


fi xy2 + y1 = – a cos(log x) ◊ ​__
 x ​ – b sin (log x) ◊ ​ __
x ​
2 
156 We have y = sin–1 x fi x y2 + x y1 = – y
1 fi x2 y2 + x y1 + y = 0
fi y1 = ______
​  _____   
÷​ 1  – x2  By Leibnitz theorem, we get,
– 2x ______ _______xy1
 ​ fi y2 = – ​ ______________
      ​= ​   2  
 ​ x2 yn+2 + 2 (n + 1) x yn+1 + n (n – 1) yn
 ​ 2(1 – x2) ​÷1  – x2  
 ​ (1 – x )
+ x yn+1 + nyn + yn = 0
fi (1 – x2) y2 = x y1 fi x  ◊ yn+2 + (2n + 3)x ◊ yn+1 + (n2 + 1) yn = 0
2

fi (1 – x2) y2 – xy1 = 0

Hints and Solutions


Level III 2a2 1
= ​ ________
______    ​× __
​ a ​
______ ÷​ x  – a  
2 2
 ​
{  (  )}
2
x+÷ ​ x  2 – a2 ​ 
1. We have y = ​​ log ​ ​ ___________
a     ​  ​  ​​ ​ d 2y dy
(x2 – a2) ​ ___2  ​ + x ​ ___  ​ = 2a
______ dx dx
​ x  2 – a2  
+ k log (x + ÷  ​) Hence, the result.
______ y/x

dy x+÷
fi ​ ___  ​ = 2 log ​ ​ ____________
dx
( 
​ x  2 – a2 
a      
 ​
​  ​ ) 2. We have (a + bx​)e​ ​ = x
__y
fi ​e​​ x ​​ = ______
x
​       ​

(  ) ( 
a + bx
​ 
a
× ​ ___________  
______     ​ ​ × ​ 1 + ________
x + ​÷x  2 – a2 

 ​ )x
​  ______   
​÷x  2 – a2 
 ​  ​
 ​
y
fi ​ __x ​ = log ​ ______ ( 
x
​     
a + bx )
 ​  ​


(x + ÷
k 
+ ____________
​  ______
​ x  – a  
2 2
 ​)( 
     ​ × ​ 1 + _______
)
2
x
​  ______   ​  
​÷x  – a  ​  2

y
fi ​ __x ​ = log x – log (a + bx)
dy
______ x ◊ ​ ___  ​– y
(  x+÷ ​ x  – a  
2
= 2 log ​ ​  ___________ )
a     ​  
2
 ​ a
​ × ________
​  ______
​÷x  – a  
2
   
2
 ​+ _______
​  ______
 ​ ​÷x  – a  2
k
    fi ​ 
2
dx
________
x 2
 ​ 
1
 = __
b
​ x ​ – ______
​     
a + bx
a + bx – bx
 ​ = __________
​     ​
x(a + bx)

______ dy ax2
( 
ax
 ​ fi ​( ​÷x  2 – a2 
 ​
______
dy x + ​÷x  2 – a2  
 ​ )​ ​ ___  ​ = 2 a log (​ ___________
dx
​  a
 ​
     ​   ​+ k ) fi x ​ ___  ​ – y = ​ ________
dx
   ​ = ​ ______
x (a + bx) a + bx
    ​

d 2y dy dy (a + bx) ◊ a – ax ◊ b
______
d 2y dy fi x ​ ___2  ​ + ​ ___  ​ ◊1 – ​ ___  ​ =  ​ _______________
    
 ​
fi ​( ​÷x  2 – a2   ​ )​ ​ ___2  ​ + _______
x
​  ______
     ​​ ___  ​ dx dx dx (a + bx)2
dx ÷​ x  2 – a2    ​dx

( 
2 d 2y a2

2a
= ​ ____________
______  
(​  x + ÷​ x  – a  
2
    ​ × ​ 1 + _______
2
 ​ )​
​  ______
x
÷​ x  – a2 ​ 
2 )
   ​  
​ fi x ​ ___2  ​ = ​ ________
dx
   
(a + bx)2
 ​
5.46  Differential Calculus Booster

( 
d 2y a2 x 2
fi x3​ ___  ​ = ​ ________
dx 2
(a + bx) a
ax 2
 ​= ​​ ​ ______
 2 
+
   ​  
bx
​​ ​ )
1 × – b sin x
= – ​ ___________
  
  
(a + b cos x)2


d 2y
( 
dy 2
)
x3 ​ ___  ​ = ​​ x ​ ___  ​ – y  ​​ ​.
dx2 dx
 ​
b sin x
= ​ ___________
  

(a + b cos x)2

Hence, the result.  ​


4. We have x = secq – cosq

[ { ÷  } ]
______
dx
2 a – b x fi  ​ ___ ​ = secq tan q + sinq = tanq (secq + cosq)
3. If y = ________
​  ______     ​tan–1 ​ ​ ​ ​  _____ 

  ​ ​  ​ tan ​ __  ​  ​, then prove dq
÷​ a  – b  
2 2
  ​ a + b 2
Also, y = secn q – cosn q
that
dy
dy 1 fi ​ ___ ​ = n secnq tanq + ncosn–1q sinq
(i) ​ ___  ​ = _________
​       ​ dq
dx a + b cos x
dy
fi ​ ___ ​ = n tanq  (secn q + cosn q)
d 2y b sin x dq
(ii) ​ ___2  ​ = ​ ___________
  
  ​
dx (a + b cos x)2 dy dy/dq
Now, ​ ___  ​ = ______
​   ​ 

[ { ÷  } ]
______ dx dx/dq
2 a–b x
3.(i) Given y = _______
​  ______    ​ tan–1 ​ ​ ​ ​ _____ 
  ​ ​  ​ tan ​ __  ​  ​ n tanq (secn q + cosn q)
÷​ a  – b  ​ 
2 2 a + b 2 = __________________
​        
tanq (secq + cos q)
dy 2 1 n (secn q + cosn q)
​ ___  ​ = ________
​  _______
    ​× ________________
​       ​ = ​ ______________
(  ) (  )
dx ÷ –b  ​   
    ​
​ a  – b  
2 2
 ​ 1 + ​ ​ a_____ x
 ​  ​ tan2 ​ __
  ​    ​  ​  (secq + cos q)
a+b 2

​   ​  ​ ​  _____ 
_____
 ​ ​ sec ​( __
dy 2
fi ​​ ​ ___  ​  ​​ ​ = (  ) n2 (secn q + cosn q)2
________________
​    
   ​
2 ÷a + b
​    ​ )​
1 a–b x dx  (secq + cos q)2
× __  
2 
2


1
= ​ ________
   
(a + b)
 ​× ​ ____________________
  
     ​ dx (  )
dy 2 n2{(secn q – cosn q)2 + 4}
fi ​​ ​ ___  ​  ​​ ​ = _____________________
​     
     ​
{(sec q – cos q)2 + 4}
(  )
_______
2  x
÷​ a  – b  
2 2
 ​ (a + b) + (a – b) tan ​ __
​    ​  ​
2 dy 2   n2 (y2 + 4)
(  )
fi ​​ ​ ___  ​  ​​ ​ =​  __________
   ​
 .

÷ 
_____ dx (x2 + 4)

a – b 2  __
× ​ ​ _____ 
x
(  )
 ​ ​ sec ​ ​    ​  ​

2 
a+b 2

dy 2
(  )
(x2 + 4)​​ ​ ___  ​  ​​ ​ = n2 (y2 + 4)
dx
sec (x/2)
= ____________________
​     
   x  ​
Hence, the result.
(a + b) + (a – b) tan2 ​ __
​    ​  ​ (  ) 5. We have

(  )
2
2x
y = (logcosx sinx)(logsinx cos x) –1 + sin–1 ​ ​ _____
1 + tan ​( __
   ​  

​    ​ )​
x2 
1 + x2
___________________________ 2
= ​     
      ​
a ​( 1 + tan ​( __
​    ​ )​ )​+ b​( 1 – tan ​( __
log (sin x) _________
log (sin x)
​    ​ )​ )​ = _________
x 2  x 2  ​   
 ​ 
× ​    + 2 tan–1 x
 ​ 
2 2 log (cos x) log (cos x)

{log (sin x)​}2​ ​

( 
1
= _________________ = ___________
​     ​ + 2 tan–1 x
  

)
​       ​

a + b ​ ​ __________
  
x
1 – tan2 ​ __
​    ​  ​
2 (  ) {log (cos x)​}2​ ​

x  ​  ​
(  )
{log(cos x)​}2​ ​ 2{log(sin x)} cot x
1 + tan2 ​ __​    ​  ​ ​     
2
    ​
2 + {log (sin x)​}​ ​ 2{log (cot x)} tan x ​ _____
dy ​ ____________________________ 2
___
fi ​    ​ = ​          
 ​ + ​    2   ​
1 dx {log (cos x)​}4​ ​ 1+x
= _________
​       ​
a + b cos x dy
(  )
fi ​​ ​ ___  ​  ​​x = ​ __p ​ 
dx 4
(  )
2

{  (  ) } {  (  ) }
dy d dy
(ii) ​ ___2  ​ = ___
​     ​ ​ ​ ___  ​  ​ 1 3 1 3
dx dx dx 2​​ log​ ___
​  __  ​  ​  ​​ ​+ 2​​ log​ ___ ​  __  ​  ​  ​​ ​
÷   
​ 2 ​ ÷   
​ 2 ​ 2
​ = ________________________    + ​ ________
{  (  ) }
​       ​   2   ​

d
= ___ (  1
​      ​​ _________
​ 
dx a + b cos x
    ​  ​ ) ​​ log​ ___
1 4
​  __  ​  ​  ​​ ​
÷   
​ 2 ​
(  )
p
1 + ​​ __
​   ​   ​​ ​
4
Differentiation  5.47

4 ​​ log ​ ___ { 


1 3
(  ) }
​  __  ​  ​  ​​ ​
x
log ​ cos  ​ __ (  (  ) )
​    ​  ​  ​ + log  ​ cos  ​ __
2
x
(  (  ) )
​  2  ​   ​  ​ + log  ​ cos  ​ __
x
​  3  ​   ​  ​
(  (  ) )
( 
  
​÷2 ​ 32 2 2
= ____________  ​ + _______
{  (  ) }
​       ​  2      ​
(  ) )
1 4
___ p + 16 x sin x
​​ log ​ ​  __  ​  ​  ​​ ​ + ... + log cos ​ __
​  n   ​  ​ = log ​ ​ __________
  x  ​  ​
(  )

÷   
​ 2 ​ 2 2  ◊ sin ​ __
n
​  n   ​  ​
4 32 2
= _______ + _______
{ ( 
​     ​  ​  2      ​
1
​ ​ ​ ___
__  ​  ​  ​ p
)}+ 16 Differentiating w.r.t x, we get,
÷   
​ 2 ​

= _________
​ 
4
   
32
 ​ + _______
​  2      ​
1
2 2
x
– ​ __ ​  tan​ __ (  )
2
1
​    ​  ​ – ​ __2  ​  tan ​ __
2
x
(  ) 2
1
​  2  ​   ​ – ​ __3  ​   tan ​ __
x
(  ) 1
(  )
​  3  ​   ​ – ... – ​ __n   ​ tan​ __
2 2
x
​  n   ​  ​
2
– __
1
​   ​  log (2) p + 16
2
= cot x – __
1
​  n   ​ cot ​ __
2
x
(  )
​  n   ​  ​
2
32
= _______
​  2    
p + 16
8
 ​ – ​ ______
   ​ 
log (2)
1
2 2
x
(  )
​ __ ​  tan​ ​ __  ​  ​ + __
1
​  2  ​  tan ​ __
2
x
(  )
​  2  ​   ​ + __
2
1
​  3  ​   tan ​ __
2
x
(  )
​  3  ​   ​
2

6. If y = logn (|cos 4 x|) + |sin x|, + ... + __


2
1
(  )
​  n   ​ tan ​ __
2
x 1
​  n   ​  ​ = __(  )
​   ​  cot ​ __
2
x
​  n   ​  ​ – cot x
2
dy p
​​     ​  tan​( __ ​     ​cot ​( __
n
where u = sec 2x, then find ​ ___  ​ at x = __ ​   ​  ​S ​ ​ __
1
​     ​ )​ = __
x 1
​     ​ )​ – cot (x)
x  
dx 4 2
  r
2 r
2 n
2 n
r = 1
7. We have

(  ) ( 
__ __
  
x​÷2 ​
y = 2 tan–1 ​ ​ _____2 
1–x
    + x2
1 + x​÷2 ​
​ + log ​ ​ ___________
 ​   __   
1 – x​÷2 ​
    ​  ​
    + x2 ) (ii) Differentiating w.r.t x, we get,

= 2 tan ​ _____
​  –1   
(  )
x​÷2 ​
 2
 ​  
__
__
    + x 2)
​ + log (1 + x​÷2 ​
1
2 2
x
​ __2  ​  sec2 ​ __
​    ​  ​ + __
2
1
(  )
​  4  ​  sec2 ​ __
2
x
​  2  ​   ​ + __
2
1
(  )
​  6  ​  sec2 ​ __
2
x
​  3  ​   ​
(  )
1– x
(  ) (  )
__
    + 2) 1 x 1 x
– log (1 – x​÷2 ​ + ... + ___ ​  2n    ​sec2 ​ __
​  n   ​  ​ = cosec2 x – ___​  2n    ​cosec2 ​ ◊__ ​  n   ​  ​
2 2 2 2

(  )
__
dy
(  ) (  )
2 d x​÷2 ​    n

( 
     ​ ◊ ​___
​ ___  ​ = ​ ____________      ​ ​ _____
​    ​  
​ 1   2  __
​S  ​ ​ ​ ​ ___
x
  ​ sec ​ ​  r   ​  ​ = cosec2 x – ___
1
​  2n    ​cosec2 ​ __
x

)
dx dx ​  n   ​  ​
2x
_______
2
1 – x2 r = 1 2
2r 2 2 2
1 + ​ ​     ​  ​
(1 – x2​)2​ ​
9. We have p2 = a2 cos2q + b2 sin2 q
__ __
2x + ÷   
​ 2 ​   
2x – ​÷2 ​ fi 2p2 = a2 (2 cos2 q) + b2 (2 sin2 q)
+​ ___________
    __ ​ – ​ __________
  
  __ ​
1 + x2 + x​÷2 ​
    1 + x2 – x​÷2 ​   
fi 2p2 = a2 (1 +  cos 2 q) + b2 (1 – cos 2 q)
__ __ __
2(1 – x2​)2​ ​ ÷    (1 + x2)
​_________
2 ​ 2(– 2​÷2 ​   x  2) + 2​÷2 ​
  (  1 + x2) fi 2p2 = (a2  + b2 ) + (a2  – b2 ) cos2 q
= ​ ________ ​  
 ◊ ​
   ​ 
  + ______________________
​        
 ​
1 + x4  (1 – x2​)2​ ​ (1 + x2​)2​ ​ – 2x2 Differentiate w.r.t q, we get,
__ __
  (  1 + x )
2​÷2 ​ 2
  (  1 – x )
2​÷2 ​ 2 dp
= ​ __________     ​  + __________
​     ​  fi 4p ​ ___  ​ = 2 (a2 – b2) sin2 q
(1 + x ) 4
(1 + x ) 4 dq
__ __ dp
   (1 + x2) + 2​÷2 ​
2​÷2 ​    (1 – x2) fi 2p ​ ___  ​ = (a2 – b2) sin2 q
= ​ ______________________      ​    dq
(1 + x4) Again, differentiating w.r.t q, we get,
__

(  )
2 2
   (1 + x + 1 – x )
2​÷2 ​ d2 p
= ​ __________________ 4
  
 ​    fi p ​ ____2   
dp 2
​ + ​​ ​ ___  ​ ​ ​ =  (a2 – b2) cos2 q
(1 + x ) dq dq
__

(  )
  
4​÷2 ​
= _______
​      ​ d2 p dp 2
(1 + x4) fi p3​ ____  ​ + p2 ​​ ​ ___  ​ ​ ​ = p2 (a2 – b2) cos2q
dq 2 dq
x
(  ) sinx
(  )
n
8. We have ’ cos ​ ___ ​  r   ​  ​ = ​ __________   
x  ​, d2 p
(  )
  dp 2
r =1 2 n __
2  ◊ sin ​ ​  n   ​  ​ fi p 3 ____
​    ​ = p2 (a2 – b2) cos2q – p2​​ ​ ___  ​ ​ ​
2 dq 2 dq

​    ​ )​ ◊ cos ​  __
cos ​( __
2
x
( ​ 2x  ​  )​ ◊ cos ​( __​ 2x  ​  )​... cos​( __​ 2x   ​ )​ = ​ __________
2 3 n
sin x
  
2  ◊ sin ​( __
n
 x  ​,
​     ​ )​
2 n
d2 p
dq 2
dp 2
fi p4 + p3 ​ ____  ​ = p4 + p2 (a2 – b2) cos 2q –  p2​​ ​ ___  ​ ​ ​
dq (  )
Taking logarithm of both the sides, we get, = p2 (p2 + (a2 – b2)) cos 2q – (b2 – a2) sin2q cos2q
5.48  Differential Calculus Booster

= (a2 cos2q + b2sin2q) (b2 cos2q + a2 sin2q)


2 2 2 2

d 2y
dx2
dy 2
dx (  )
2y3 ​ ___  ​ + 2y2 ​​ ​ ___  ​  ​​ ​ = y2p¢¢(x)
– (b – a ) sin q  cos q
2  2 4
= a b (cos q + sin q + 2 sin q cos q) 4 2 2 fi
d 2y
dx2
dy 2
dx (  )
2y3 ​ ___  ​ = y2  p¢¢(x) – 2y2 ​​ ​ ___  ​  ​​ ​

= a2 b2 (sin2 q + cos2 q)2


d 2y 1
= a2 b2 fi 2y3​ ___  ​ = y2p¢¢(x) – __
​    ​ ( p¢(x)​)2​ ​
dx 2 2
d2 p a2 p2
Thus, p + ​ ____2  ​ = ​ ____ ​ 
  Again, differentiate w.r.t x, we get,

(  )
dq p3
d d 2y
10. We have z = cos7 x fi 2 ​ ___   ​ ​ y3​ ___  ​  ​
dx dx2
dz
fi ​ ___ ​ = – 7 cos6 x sin x
dx
Also, y = sin x

dy
(  )
= 2y ​ ​ ___  ​  ​ p ¢¢(x) + y2p¢¢¢(x) – p¢(x) p¢¢(x)
dx
dy = p¢¢(x) p¢¢(x) + y2p¢¢¢(x) – p¢(x) p¢¢(x)
fi ​ ___  ​ = cos x
dx
= y2p¢¢¢(x)
dz 7 cos6 x sin x
Now, ​ ___ ​ = – ​ __________
   
cos x   ​ = – 7 cos5 x sin x
dy = p(x) p¢¢¢(x)

(  )
2
dz d dz 12. We have j (x) = f (x) g (x)
fi ​ ___2  ​ = ___
​     ​ ​ ​ ___ ​  ​
dy dy dy Differentiate w.r.t x, we get,
= ___
d dz dx
dx dy dy (  )
​     ​ ​ ​ ___ ​  ​ ​ ___  ​ fi j ¢(x) = f(x) g¢ (x) + f ¢(x) g (x)
fi j¢ = f g¢ + f ¢g
d 1
​     ​ {(– 7 cos5 x sin x)} ◊ ​____
= ___  cos x
    

dx fi j¢¢ = f ¢g¢ + f g¢¢ + f ¢¢g + f ¢g¢
1
= (35 cos4 x sin2 x – 7 cos6 x)​ ____ cos x     
​ fi j¢¢ = f ¢¢g + 2 f ¢g¢ + g¢¢f
= (35 cos3 x sin2 x – 7 cos5 x) j¢¢ f ¢¢g + 2 f ¢g¢ + g¢¢f
fi ​ ___ ​ = _______________
​      ​ 

(  ) (  )
j fg
d3y d d z dz
2
fi ​ ___3 ​ = ​ ___   ​ ​ ​ ___2  ​  ​ ​ ​ ___ ​  ​ f ¢¢ 2 f ¢g¢ g¢¢
dx dy dy
dz = ​ ___   ​  + ​ ____   
​ + ​ ___
g ​ 
f f g
d 1
​     ​ {(35 cos3 x sin2 x – 7 cos5 x)} ◊ ​____
= ___  cos x
    
​ f ¢¢ 2c g¢¢
dx = ​ ___   ​  + ​ ___  ​ + ​ ___
g ​ 
f f g
= – 105 cos x + sin3 x + 105 cos3 x ◊ sin x
j¢¢ f ¢¢ 2 c g¢¢
105 Thus, ___
​   ​ = ​ ___   ​  + ___
​   ​ + ​ ___
g ​ 
= ____
​   ​ [4 cos

  3
 x sin x – 4 cos x sin3 x] j f f g
4
13. We have, (1 – x) (1 + x) = (1 – x2)
105
= ____
​   ​ (4 cos x sin x)[cos

  2 
x – sin2 x]
4 (1 – x) (1 + x) (1 + x2) = 1 – x4
105 (1 – x) (1 + x) (1 + x2) (1 + x4) = 1 – x8
= ____
​   ​ (2 sin 2 x cos 2 x)


4
... ... ... ... ...
105
= ____
​   ​ (sin 4 x)


4 ... ... ... ... ...
11. Given y2 = p (x) ... ... ... ... ...
Differentiate w.r.t x, we get, (1 – x) (1 + x) (1 + x )...(1 + x2n–1) = 1 – x2n
2

dy
fi 2y ​ ___  ​ = p¢(x) when n Æ •, x < 1, then x2n Æ 0
dx
Again, differentiate w.r.t x, we get, Thus, (1 – x) (1 + x) (1 + x2)...(1 + x2n–1) = 1

(  )
d 2y dy 2 log{(1 – x) (1 + x) (1 + x2)...(1 + x2n–1)} = log (1) = 0
fi 2y ​ ___2  ​ + 2 ​​ ​ ___  ​  ​​ ​ = p¢¢(x)
dx dx log{(1 – x) (1 + x) (1 + x2)...(1 + x2n–1)} = 0
Differentiation  5.49

log (1 – x)  + log (1 + x) + log (1 + x 2) dx dy


fi _____________
​       ​ + ____________
​       ​
(y – zx) (z – xy) (x – yz)(z – xy)
+ log (1 + x4) + log (1 + x8) + ... = 0
dz
Differentiate w.r.t x, we get, + _____________
​        ​ = 0 ...(i)
(y – zx) (x – yz)
1 1 2x 4x3 8x 7
– _____
​      ​ + _____ ​ + ​ _____
​        2  ​ + ​ _____  ​
  + _____
​    ​ 
+ ... = 0 Now, (y – zx)2
1–x 1+x 1+x 1 + x4 1 + x8
1 2x 4x3 8x 7 1 = y2 + z2 x2 – 2xyz
fi _____
​      ​ + _____
​   2  ​ + ​ _____   ​ + ​ _____  ​ + ... = _____
   ​      ​
1+x 1+x 1+x 4
1 + x8 1–x = z2 x2  + (y2 – 2xyz)
Hence, the result. = z2 x 2 + 1 – x 2 – z2
14. Given x sin q + y cos q = a
= 1 – x 2 – z2 + z2 x 2
x cos q – y sin q = b
= (1 – x2) (1 – z2)
On solving, we get,
x = a sin q + b cos q Similarly, (z – xy)2 = (1 – x2) (1 – y2),
y = a cos q – b sin q (x – zy)2 = (1 – z2) (1 – y2)
Now, put a = r sin j, b = r cos j From (i), we get,
a
Therefore, r 2 = a2 + b2, j = tan–1 ​ __
​   ​  ​
b (  ) dx
​ _______   
______
dy
 ​+ ​ _______
  
______
dz
 ​+ ​ _______
  
______  ​= 0
÷​ 1  – x  
2
 ​ ​÷1  – y  
2
 ​ ​÷1  – z2   ​
Then, x = r cos (q – j), y = r sin (j – q)
ax2
d3y 3p
Hence, ​ ___3  ​ = r cos ​ ___
dx
( 
​   ​ + q – j  ​
2 ) 16. We have y = _________________
​         ​
(x – a) (x – b) (x – c)

= r cos ​ p + __ ( p
​   ​  + q – j  ​
2 ) ​ 
bx
+ ___________

c
    ​ + ____
​     
(x – b) (x – c) x – c
 ​ + 1.


p
__
2 ( 
= – cos ​ ​   ​  + q – j  ​ ) ax2
= ​ _________________
bx
       ​ + ___________
​   
x
    ​ + ______
​     
(x – a) (x – b) (x – c) (x – b)(x – c) (x – c)
 ​
= r sin (q – j)
ax2  bx + x2 – bx 
= ​ _________________
       ​ + ____________
( 
2 ​     ​
  
p
dz
​ ____2  ​ = r cos ​ 2__
dq
​   ​  + q – j  ​
2 ) (x – a) (x – b) (x – c) (x – b)(x – c)
ax2 x2
= – r cos (q – j) = ​ _________________
       ​ + ​ ___________
     ​
(x – a) (x – b) (x – c) (x – b)(x – c)
d3y
dq
3p
​ ____3  ​ = r sin ​ ___
2 (  )
​   ​ + q – j  ​ = r cos (j – q)
​ 
ax2 + x3 – ax2
= _________________
      ​
(x – a) (x – b) (x – c)

( 
d 2y
dq
p
​ ____2  ​ = r sin ​ 2 ◊ ​__
2 )
   ​  + q – j  ​ = – r sin (j – q) x3
= ​ _________________
       ​
(x – a) (x – b) (x – c)
d 2y d2y ____ d3y

( 
d3x ____
Now, ​ ____3  ​  
◊ ​  2  ​ – ​ ____2  ​  
d q  d q 
2 
d q  d q 
◊ ​  3  ​  x3
fi log (y) = log ​ ​ ________________
     ​   ​
(x – a) (x – b) (x – c) )
= r [sin (q – j)sin (q – j) + cos (q – j) cos (q – j)] fi log (y) = 3 log x – log ((x – a) (x – b) (x – c))
2  2 2
= r [sin  (q – j) + cos  (q – j)
fi log (y) = 3 log x – log (x – a) – log (x – b) 
= r 2 = (a2 + b2) – log (x – c)
dy
1 ___ __ 3 ______ 1 1 1
15. We have x2 + y2 + z2 – 2xyz = 1 fi ​ __  ​ – ______
y ​ ​ dx  ​ = ​ x ​ – ​ (x – a)  ​       ​ – ______
​       ​
(x – b) (x – c)
fi 2xdx + 2ydy + 2zdz – 2yzdx – 2xzdy
– 2xydz = 0
fi ​ __
dy
1 ___ __
( 
1 ______ 1
y ​ ​ dx  ​ = ​ ​ x ​– ​ (x – a)  ) (  1
 ​  ​ + ​ __
1
​ x ​  – ______
​     
(x – b) )
 ​  ​

fi xdx + ydy + zdz – yzdx – xzdy – xydz = 0


fi (x – yz) dx + (y – zx) dy + (z – xy) dz = 0 ( 
1 ______
+ ​ ​ __
1
x ​ – ​ (x – c)  )
 ​  ​
5.50  Differential Calculus Booster

(  ) (  (  ) (  )
y
)
dy x
1 ___
fi ​ __
x–a–x
________ x–b–x
​ + ​ ________ fi ​ ​ __x ​  ​ cos4 a + ​ __ ​ y ​  ​ sin4 a + (sin4 a + cos4 a)= 1
y ​ ​ dx  ​ = ​ ​ x(x – a) ​  
  ​ 
x(x – b)
 ​  
  ​

(  (  ) (  )
y x
x – c– x
+ ​ ________
​ 
x(x – c)
​ fi ​ __
 ​  
  )
​ x ​  ​cos4 a + ​ __ ​ y ​  ​ sin4 a + 1 – 2sin2 a cos2a = 1

(  ) (  ) (  (  ) (  )
dy y x
fi ​ __
​ x ​  ​cos4 a + ​ __
1
fi ​ __ ​

y dx
___  ​ = ​ _______
​  ​ 
x(x – a)
– a
   
 ​  ​ + ​ ​ 
– b
_______
x (x – b)
   
 ​  ​ + ​ ​ 
– c
_______
x(x – c)
   
 ​  ​ ) __
​ y ​  ​ sin4 a – 2sin2 a cos2a = 0

( ÷  ) ( ÷ 
__

(  )
y 2
x 2

)
1 dy 1 a b c fi ​​ ​ __
​   ​ ​
 cos2 
a  ​​ ​ + ​​ ​ __
​ y ​ ​  sin2 a  ​​ ​
fi ​ __ ___  ​ = __ ______ ______ ______  
 ​
​ 
  ​  ​ ​
  ​      ​ + ​      ​ + ​     
 ​  ​ x
y dx x (a – x) (b – x) (c – x) __

( ÷  ) ( ÷ 
__

f g h

x 2  y
–2​ ​ ​ y ​ ​  sin a  ​ ​ ​ __
__ ​ x ​ ​  cos2 a  ​ = 0 )
( ÷ 
__

÷  )
__
17. We have D = ( xf )¢ ( xg )¢ ( xh )¢ x 2  y 2
fi ​​ ​ ​ __y ​ ​ sin
  a – ​ __
​ x ​ ​ cos

2 
a  ​​ ​ = 0
( x 2 f )¢¢ ( x 2 g )¢¢ ( x 2 h )¢¢
fi ​( ​ __
__

÷​  y ​ ​ sin a – ​÷​  x ​ ​ cos a )​ = 0


__
x
 __y 2 

2 
f g h
__

÷  ÷ 
= xf ¢ + f xg ¢ + g xh ¢ + h __
x y
fi ​ __
​ y ​ ​  sin2 a = ​ __
​ x ​ ​ cos

2 
a
( x 2 f ¢¢ + 4 xf ¢ ) ( x 2 g ¢¢ + 4 xg ¢ ) ( x 2 h ¢¢ + 4 xh ¢ )
fi y = (x) tan2 a
f g h
dy
= xf ¢ xg ¢ xh ¢ fi ​ ___  ​ = tan2 a
dx

( ÷ 
( x 2 f ¢¢ + 4 xf ¢ ) ( x 2 g ¢¢ + 4 xg ¢ ) ( x 2 h ¢¢ + 4 xh ¢ ) _________

(R2 Æ R2 – R1)
x2 + x + 1
19. If y = log ​ ​ ​ _________
   ​ ​  
  
x2 – x + 1
​ )
{  (  ( 
f g h
= xf ¢
2
xg ¢
2
xh ¢ (R3 Æ R3 – R1)
2
1
+ ____
  
2​÷3 ​
2x +__ 1
   ​ ​  tan–1 ​ ​ ______
​  __
  
​÷3 ​ )
2x –__ 1
 ​ + tan–1 ​ ​ ______
 ​    ​  
  
​÷3 ​
 ​  ​, )}
x f ¢¢ x g ¢¢ x h ¢¢
dy 1
then prove that, ​ ___  ​ = _________
​      ​.
f g h dx x4 + x2 + 1

( 
= x3 f ¢ g¢ h¢
f ¢¢ g ¢¢ h ¢¢
1
20. We have t1 = tan–1 ​ ​ ________
   
x2 + x + 1
 ​  ​
)
f g h
3
f
2
g
Thus, D¢ = 3x f ¢ g ¢ h ¢ + x f ¢ g ¢ h ¢
h
( 
(x + 1) – x
= tan–1 ​ ​ __________ )
   ​  ​ = tan–1 (x + 1) – tan–1 (x)
  
1 + x (x + 1)
f ¢¢ g ¢¢ h ¢¢

f
f ¢¢¢ g ¢¢¢ h ¢¢¢

g h f g h

( 
​  2
1
t 2 = tan–1  ​ _________
   
x + 3x + 3
 ​  ​
)
= f¢ g¢
3 x 2 f ¢¢ 3 x 2 g ¢¢ 3 x 2 h ¢¢
h¢ + x3 f¢
x 3 f ¢¢¢

x 3 g ¢¢¢

x 3h ¢¢¢
( 
(x + 2) – (x + 1)
= tan–1 ​ ​  ______________
  
   ​  ​
1 + (x + 2) (x + 1 )
= tan–1 (x + 2) – tan–1 (x + 1)

( 
f g h
=
3

3

3
h¢ ​  2
1
t 3 = tan–1 ​ _________
     ​  ​
x + 5x + 7 )
( x f ¢¢¢ ) ( x g ¢¢¢ ) ( x h ¢¢¢ )

Hence, the result.


( 
(x + 3) – (x + 2)
= tan–1 ​ ​  _______________
  
    ​  ​
1 + (x + 3) (x + 2) )
cos4 a _____
sin4 a _____ 1
18. We have ​ _____
x   
​ + ​  y   
​ = ​     
 ​ = tan–1 (x + 3) – tan–1 (x + 2)
x+y
.... ..... ..... ..... ....
x+y
fi ​ _____
​  x    (  x+y
​  ​ cos4 a + ​ _____
​  y    )​  ​ sin4 a = 1 (  )

....
....
.....
.....
.....
.....
.....
.....
....
....
y
fi ​ 1 + __ (  x
​ x ​  ​cos4 a + ​ 1 + __ )
​ y ​  ​ sin4 a = 1 (  ) –1 –1
tn = tan (x + n) tan (x + (n –))
Differentiation  5.51

Thus, Sn = t1 + t 2 + t 3 + ... + tn = (8cos4q – 8cos2q + 2) –1


y = tan –1
(x + n) – tan –1
(x) = (8cos4q – 8cos2q + 1)
dy = (8x4 – 8x2 + 1), x = cosq
1 1
​ ___  ​ = __________
​       ​ – _____
​      ​
dx 1 + (x + n)2 1 + x2 24. As we know that, if f(x) is divisible by x and f ¢(x) is
21. We have also divisible by x, then f(x) is also divisible by x2.
y = tan–1 (sin x + 1) – tan–1 (sin x) a b c d
Given f(x) = _____​       ​ + _____
​      ​ – _____
​      ​ – _____
​       ​
a+x b+x c+x d+x
+ tan–1 (sin x + 2) – tan–1 (sin x + 1)
a b c d
+ tan–1 (sin x + 3) – tan–1 (sin x + 2) f ¢(x) = – ​ _______
  2   ​ – ​ _______
   
 ​ + _______
​    2   ​ + _______
​      ​
(a + x) (b + x)2 (c + x) (d + x)2
+ .... + tan–1 (sin x + n) – tan–1 (sin x + (n – 1))
= – tan–1 (sin x + n) – tan–1 (sin x) Since f ¢(x) is divisible by x, then f ¢(x) = 0
Differentiating w.r.t x, we get, a b c d
– _______
​       ​ – _______
​      ​ + _______
​       ​ + _______
​      ​ = 0
dy (a + x)2 (b + x)2 (c + x)2 (d + x)2
cos x cos x
​ ___  ​ = ​ ____________
     ​ – ​ _________
   ​
dx 1 + (sin x + n)2 1 + (sin x)2 Put x = 0, we get,
1 1 1 __ 1
22. Given y = f (x) – __
​ a ​ – __
​   ​ + __
​   ​ + ​   ​ = 0
b c d
Differentiating w.r.t x, we get, 1 __ 1 __ 1 __ 1
​ __ a ​ + ​ b ​ – ​ c ​ – ​ d ​ = 0
dy d 2y
​ ___  ​ = f ¢(x), ​ ___2  ​ = f ¢¢(x)
dx dx 25. We have y = (sin–1x)2 + (cos–1x)2
dx 1 d___ 2
x f ¢¢(x)  ___
dx f ¢¢(x)  dy 2 sin–1x 2 cos –1
x
​ ___  ​ = ____
​     
 ​, ​  2   ​ = – ​ _______   ​    ​ = – ​ _______
 ​     ​ ​ ___  ​ = ​ _______
_____   ​ – ​ _______
_____  ​  
dy f ¢(x) dy {f ¢ (x)}2 dy
{f ¢(x)}3 dx ​÷1  – x2   ​ ÷ ​ 1  –x  
2
 ​

{  (  ) } {  (  ) }
dy 2 3/2 dx 2 3/2 dy 2(sin–1x – cos–1x)
___
​​ 1 + ​​ ​ ___  ​  ​​ ​  ​​ ​ ​​ 1 + ​​ ​ dy  ​  ​​ ​  ​​ ​ ​ ___  ​ = ​  _______________
_____   ​   
dx dx ÷​ 1  – x2 
 ​
Now, ​  ____________  ​      + ​  ____________  ​     

( 
2 2
d y d___ x

)
___ ​  2   ​ _____
​  2  ​ 2 (– 2x)
dx dy  ​​ ______
 2​÷1  – x2  ​  _____     ​  ​ – 2(sin–1x – cos–1x) ​ _______
_____   
 ​
2
÷​ 1  – x  
2
d y __________________________________________
 ​ 2​÷1  – x2 
 ​

{  {  } }
___
​  2  ​ =​         ​    
1 2 3/2
​​ 1 + ​​ ____
​     ​  ​​ ​  ​​ ​ dx (1 – x2)
{1 + {f ¢(x)} } 2 3/2
f ¢(x)
= ​  _____________
 ​
     + ​ _______________      ​  d2y x
f ¢¢(x) f ¢¢(x)
_______ (1 – x2)​ ____2 ​ = 4 + 2(sin–1x – cos–1x) ______
​  _____
    ​
–  ​     
 ​ dx ÷​ 1  – x2 
 ​
{f ¢(x)}3
d 2y dy
{1 + {f ¢(x)}2}3/2 {1 + {f ¢(x)}2}3/2 (1 – x2) ​ ___2  ​ = 4 + x ​ ___  ​
= ​  _____________
 ​ – ​  ___________________
      ​
     dx dx
f ¢¢(x) f ¢¢(x)
=0 d 2y dy
(1 – x2) ​ ___2  ​ – x ​ ___  ​ = 4
23. We have y = cos–1(8x4 – 8x2 + 1) dx dx
Put x = cos q Hence, the result.
y = cos –1
(cos4q) x+ b
26. We have ​ _____ (  )
 ​ = a tan–1(a ln y), a > 0
​   ​  
2
(  )
–1
y = 4q = 4cos x x+b
a ln y = tan ​ ​ _____   
​  ​
dy 4 2a
​ ___  ​ = – ______
​  _____
     ​ Differentiating w.r.t x, we get,
dx ​÷1  – x2   ​
dy 4
​ ___  ​ + ​ ______
_____
dx ​÷1  – x2 
     ​ = 0
 ​
a ___
​ __
dy
(  ) 1
___
(  ) 2 x____+b
y ​ ​ ​ dx  ​  ​ = ​ 2a   ​ sec  ​ ​  2a   

​  ​

Note: 1. cos(4q) = 2(cos(2q))2 – 1


= 2(2cos2q – 1)2 – 1
2a ___
​ ___
y   
dy
dx
2
(  )
​ ​  ​    ​  ​ = 1 + tan  ​( ​ _____
x+b
2a )
2
   
​  ​

= 2(4cos4q – 4cos2q + 1) – 1
2a2 ___
​ ___
dy
(  )
​ ​  ​    ​  ​ = 1 + a2 (ln y)2
y    dx
5.52  Differential Calculus Booster

2 ___
​ __
dy
(  )
y ​ ​ ​ dx  ​  ​ =
__
a
1
​  2  ​  + (ln y)2
dy 3y
fi ​ ___  ​ = ______
dx ÷
​  _____
    ​
​ x  2 – 1 ​

(  )
Again differentiating both sides w.r.t x, we get, dy 2 9y2
fi ​​ ​ ___  ​  ​​ ​ = ​ _______

(  ) (  )
   ​

(  )
d 2y dy 2 2(ln y) ___dy dx (x2 – 1)
2 ___ 2 ___
​ __ ​ ​ ​    ​  ​ – __
​    ​ ​​
  ​    ​  ​​ ​ = ​ ______
    
​ ​ ​    ​  ​
(  )
y dx2 y 2 dx y dx dy 2
fi (x2 – 1) ​​ ​ ___  ​  ​​ ​ = 9y2

(  )
dx
yd2y dy dy 2
​ ____  ​
  – ___
(y ln y) ​  ​

  = ​​ ___
​    ​  ​​ ​ Again differentiating w.r.t x, we get,
dx2 dx dx
Hence, the result.
27. We have y = __
1
​ x ​

dy d2y
dx dx dx (  )
dy 2 dy
2(x2 – 1)(​ ___  ​) ​ ___2  ​ + 2x ​​ ​ ___  ​  ​​ ​ = 18y ​ ___  ​
dx
dy
​ ___  ​ = – ​ __2  ​  
dx
_____
1
x
...(i) fi (  )
dy d2y
(  )
dy 2 dy
(x2 – 1) ​ ​ ___  ​  ​ ​ ___2  ​ + x ​​ ​ ___  ​  ​​ ​ = 9y ​ ___  ​
dx dx dx dx
Now, ​÷  1 + y4 
 ​ d 2y dy
_______ fi (x2 – 1) ​ ___2  ​ + x ​ ___  ​ = 9y

÷  (  )1 4 dx dx
= ​ 1 + ​​ __
​ x ​  ​​ 
​ ​
Hence, the result.

÷ 
______
dx 1

x4 + 1
= ​ ​ _____ ​ ​
  
  29. As we know that ​ ___  ​ = ______
​       ​
dy (dy/dx)
x4

÷ 
d2 x
(  )
______ d 1
x4 + 1 fi ​ ___2  ​ = ___
​     ​ ​ ______
​       ​  ​
= ​ ​ _____ ​ ​
  
  dy dy (dy/dx)

( 
x2
​÷1  + y4 
​ _______
_____ 
 ​ __ 1
 ​ = ​  2  ​  
_____

...(ii)
d2 x
fi ​ ___2 
dy
 ​ =
d
___ 1
​     ​ ​ ______
​     
dx (dy/dx) dy )dx
 ​  ​ ​ ___  ​

÷​ 1  + x  
4
 ​ x
d2 x 1 d2y dx
fi ​ ___2   ​ = – ​ _____  2 ​  × ​ ___2  ​ × ​ ___  ​
From (i) and (ii), we get,

dy ​÷1  + y4 
_____
 ​
dy dy
​​ ​ ___  ​  ​​ ​
dx (  )
dx dy

​ ___  ​ = – ​ _______
_____   ​ d 2y
___
dx ÷​ 1  + x4   ​ 2
​    ​
d___ x _____dx2
fi ​  2   ​ = – ​    ​ 
dy
​ _______  
_____
dx
 ​ = – ​ _______
_____   ​ dy dy 3
​​ ​ ___  ​  ​​ ​ (  )
(  )
÷​ 1  + y  
4
 ​ ÷​ 1  + x4 
 ​
2
dx
dy dy 3
​ ___2  ​
(  )
​ ___ ​ 
(  )
dy dx
​ _______
  
_____  ​ + ​ _______
  
_____  ​ = 0 dx dy 3
dt g¢(t) 3
​÷1  + y  
4
 ​ ​÷1  + x4   ​ fi ​ ___  
 ​ = – ​​ ___  ​  ​​ ​ = – ​​ ​ ___
​   
 ​  ​​ ​ = – ​​ ____
​   ​  ​ ​
2
d___x dx dx
___ f ¢(t)
​  2   ​ ​   ​ 
Hence, the result. dy dt
28. We have 2x = y1/3 + y–1/3
Hence, the result.
1/3 1/3 2
fi 2xy = (y ) + 1 30. We have
y = x5 (cos(ln x) + sin(ln x))
fi (y ) – 2xy1/3 + 1 = 0
1/3 2
______ dy
   ​
2x ± ​÷4x 2
– 4 ​
  fi ​ ___  ​ = 5x4 (cos(ln x) + sin(ln x))
fi (y ) = ​  ___________
1/3
     dx
2

_____
(y1/3) = (​  x ± ​÷x  2 – 1 ​ 
  )​
sin(ln x) cos(ln x)
+ x5 ​ – ​ _______
x      ( 
​ + ​ _______
x    ​  ​ )
_____
​ )​
dy
fi ​ __ ​  log(y) = log ​( x ± ​÷x  2 – 1  
1
fi x ​ ___  ​ = 5y + x5(cos(ln x) – sin(ln x))
3 dx
Differentiating w.r.t x, we get, d2y dy dy
fi x ​ ___2  ​ + ​ ___  ​ = 5 ​ ___  ​ + 5x4 (cos(ln x) – sin(ln x))

(  )
1 dy 1 1 _____
× 2x dx dx dx
fi ​ ___  ​ ​ ___  ​ = ___________
​       ​ × ​ 1 ± ​ _______   
 ​  ​
( 
_____
3y dx (​  x ± ÷   )​
​ x  2 – 1 ​  2​÷x  2 – 1 ​

sin(ln x) cos(ln x)
+ x5 ​ – ​ _______
x    ​ – ​ _______
  x    ​  ​ )
Differentiation  5.53


d 2y dy
(  dy
)
x2​ ___  ​ – 4x ​ ___  ​ = 5 ​ x ​ ___  ​ – 5y  ​ – y
dx2 dx dx
dy
( 
d3y
) d2y dy d2y
( 
fi ​ 2x ​ ___  ​ + y  ​ ​ ___3  ​ = 3 ​ x ​ ___2  ​ + ​ ___  ​  ​ ​ ___2  ​
dx dx dx dx dx )
d 2y dy Hence, the result.
x2 ​ ___  ​ – 9x ​ ___  ​ = – 26y
(  ( 

dx 2 dx 3. Given y = tan–1 ​ _______
​ 
x
   
1.2 + x2 ) ​ 
x
 ​  ​ + tan–1 ​ _______
   
2.3 + x2
 ​  ​
)
( 
2
dy dy
fi x2​ ___  ​ – 9x ​ ___  ​ + 26y = 0
dx 2 dx
x
+ tan–1 ​ _______
​     
3.4 + x2 )
 ​  ​ + ...to n-terms

LEVEL IV ( 
x
Let tn = tan–1 ​ ​ ___________     ​  ​
)

( 
n(n + 1) + x2

)
1. Given f (x) = x3 + x2 f ¢(1) + xf ¢¢(2) + f ¢¢¢(3) x
​ _______
     ​
Let a = f ¢(1), b = f ¢¢(2), c = f ¢¢¢(3) n(n
–1 __________ + 1)
= tan  ​ ​      ​  ​
Then f(x) = x3 + ax2 + bx + c x2
_______
1 + ​      ​

( 
2 n(n + 1)
fi f ¢(x) = 3x + 2ax + b

)
x
__ x
fi f ¢¢(x) = 6x + 2a ​ n ​ – ______
​       ​
(n + 1)
= tan–1 ​ ​  ____________
x   
  
x  ​  ​
fi f ¢¢¢(x) = 6 1 + __ ​ n ​ ◊ ​______
       ​
(n + 1)


f ¢¢¢(3) = fi c = 6

f ¢¢(2) = 12 + 2a fi b = 12 + 2a
(  )
x
= tan–1 ​ __ ( 
​  x   
​ n ​  ​ – tan–1​ _____
n+1
 ​  ​ )
Thus, y = Sn = t1 + t 2 + t 3 + ... + tn
fi f ¢(1) = 3 + 2a + b fi a = 3 + 2a + b
On solving, we get, a = – 5, b = 2
x
= tan–1x – tan–1 ​ _____
​     
n+1
 ​  ​ (  )
3 2
Therefore, f(x) = x – 5x + 2x + 6
Now, f(2) + f(0)
dy 1
​ ___  ​ = _____
​      ​ 
dx 1 + x2 1 + ​​ _____
1
 ​ – ___________

1
_____
x   2 ​ ​ ​ n + 1 
​      ( 
 ​  ​​ ​
 ​  ​
)
(  )
= 8 – 20 + 4 + 6 + 6 n+1
dy 1 n+1
= 24 – 20 ​ ___  ​ = _____
​      ​ – ​ ___________
  
   ​
dx 1 + x2 (n + 1)2 + x2
=4
=1–5+2+6 4. Given f (x) = (ax + b)sin  + (cx + d)cos x
= f(1)
fi f ¢(x) = a sin x + (ax + b)cos x
ax + b
2. Given y = ​ ______  ​ + c cos x – (cx + d)sin x
x2 + c
fi x cos x = a sin x + (ax + b)cos x
Differentiating both sides w.r.t x, we get,
+ c cos x – (cx + d)sin x
(x2 + c)y¢ + 2 xy = a
= (a – cx – d)sin x + (ax + b + c)cos x
Again differentiating w.r.t x, we get,
Comparing the co-efficients of cosx and sin x we
fi (x2 + c)y¢¢ + 2 xy¢ + 2y + 2xy¢ = 0 get,
fi (x2 + c)y¢¢ + 4xy¢ + 2 = 0 (a – cx – d) = 0, (ax + b + c) = x

fi (x2 + c)y¢¢ = – 4xy¢ – 2y fi a – d = 0, c = 0; a = 1, b + c = 0

fi – (x2 + c) = (4xy¢ + 2y)/y¢¢ fi a = d = 1, b = 0 = c


Again, differentiating, w.r.t x, we get, 5. Given g(x) = (ax2 + bx + c)sin x + (dx2 + ex + f)
cos x
y¢¢{(4xy¢¢ + 4y¢) + 2y¢} – (4xy¢ + 2y)y¢¢¢
fi – 2x = _________________________________
​            ​ fi g¢(x) = (2ax + b)sin x + (ax2 + bx + c)cos x
y¢¢
fi – x(y¢¢)2 = 2x(y¢¢)2 + 3y¢y¢¢ – (2xy¢ + y)y¢¢¢ + (2dx + e)cos x – (dx2 + ex + f)sin x

fi (2xy¢ + y)y¢¢¢ = 3(xy¢¢ + y¢)y¢¢ = (2ax + b – dx2 – ex – f)sin x


5.54  Differential Calculus Booster

(  )
2
+ ( + bx + c + 2 + e)cos x ​ 0 + sin x cos 2x cos 3x   
+ 2cos x sin 2x
   
  cos 3x ​
​ + 3cos x cos2 x sin 3x
Comparing the co-efficients of sin x and cos x, we            ​​  ​
  ​ ​  ​  ______________________________________
​ lim 
x Æ 0 2x
get, (2ax + b – dx2 – ex – f) = x2
and (ax2 + bx + c + 2dx + e) = 0 = ​  
lim 
x Æ0 2
( 
1 sin x cos 2x cos 3x
​  ​  __ ​  ​ ​ _______________
x      
2cos x sin 2x cos 3x
​ ​​ + ​ ________________
x     ​ 
Ïd = - 1, b = f , 2a = e
fi Ì
Óc = 0 = e, a = 0, b = - 2 d
​ + _________________

3cos x cos 2x sin 3x
​ 
2x
     )
  ​  ​
1
Ïa = 0, b = 2, c = 0 = ​ __ ​  (1 + 22 + 33)
fi Ì 2
Ód = - 1, e = 0, f = 2 1
= __
​   ​  (14) = 7
ax + b 2
6. Given y = ​ ______ 

 ​
Ax + B 8. Given y = e​ tan
​ x​
–1

(Ax + B)y = ax + b dy 1
​ ___  ​ = etan–1x × _______
​     
 ​
Differentiating w.r.t x, we get, dx (1 + x2)
(Ax + B)y¢ + Ay = a ...(i) dy
(x2 + 1) ​ ___  ​ = etan–1x
(Ax + B)y¢¢ + 2Ay¢ = 0 ...(ii) dx
(Ax + B)y¢¢¢ + 3Ay¢¢ = 0 ...(iii) d 2y dy 1
(x2 + 1) ​ ___2  ​ + 2x ​ ___  ​ = etan–1x × _____
​  2    
 ​
Dividing (iii) by (ii), we get, dx dx x +1
y¢¢¢ 3y¢¢
​ ___ ​ = ​ ____ ​  d 2y dy dy
y¢¢ 2y¢ (x2 + 1) ​ ___2  ​ + 2x ​ ___  ​ = ​ ___  ​
dx dx dx
y¢¢¢ 3y¢¢
​ ____   ​ = ​ _____   ​ d 2y  dy
y¢¢y¢ 2(y¢)2 (x2 + 1) ​ ___2  ​ + (2x – 1)​ ___ ​ = 0
dx dx
y¢¢¢ 3(y¢¢)2 Hence, the result.
​ ___ ​ = ​ ______2 
 ​
y¢ 2(y¢) 9. Given y = xn–1ln x ...(i)

(  )
y¢¢¢ 3 y¢¢ 2 dy 1
​ ___ ​ – ​ __ ​ ​​  ​ ___ ​   ​ ​ = 0 ​ ___  ​ xn–1◊ ​__
 x ​ + (n – 1)xn–2ln x
dx

y¢ 2 y¢
dy
Hence, the result ​ ___  ​ = xn–2 + (n – 1)xn–2  ln  x

( 
dx
ay + b
Also, z = ​ ______ 
ax + b
a ​ ​ ______ 
Ax + B
 ​ = ​ _____________


 ​  ​+ b
    ​
) d 2y 1
​ ___2  ​ = (n – 2)xn–3 + (n – 1)xn–2 ◊ ​__
 x ​
Ay + B ax + b
A​ ​ ______ 
Ax + B

 ​  ​+ B (  )
dx
+ (n – 1)(n – 2)xn–3ln x
(a2 + Ab)x + (ab + Bb) cx + b d 2y
= ____________________
​     
     ​ = ​ ______  
 ​ ​ ___  ​ = (n – 2)xn–3 + (n – 1)xn–3
(aA + AB) + (Ab + B )2 Cx +D dx2
ax + b + (n – 1)(n – 2)xn–3ln x
which is same as of the form y = ​ ______    ​
Ax + B
So, we can easily prove that x 2 d 2y
​ _____  = (n – 2)xn–1 + (n – 1)xn–1
 ​ 
z¢¢¢
​ ___ ​  –

__
(  )
3 z¢¢ 2
​   ​ ​​  ​ __ ​   ​ ​ = 0
2 z¢
dy
dx2
+ (n – 1)(n – 2)xn–1ln x  ...(ii)

(  )
(3 – 2n) x ​ ___  ​ = (3 – 2n)xn–1 + (3 – 2n)(n – 1)
(  )
y¢¢¢ 3 y¢¢ 2
z¢¢¢ 3 z¢¢ 2
dx
Thus, ​ ___ ​ – __
​   ​ ​​  ​ ___ ​   ​ ​ = 0 = ​ ___ ​  – __
​    ​ ​​ ​ ___ ​   ​ ​
y¢ 2 y¢ z¢ 2 z¢
xn–1ln x ...(iii)
Hence, the result.
From (i), (ii) and (iii), we get,
7. We have ​  
x Æ 0 ( 
1 – cos x cos 2 x cos 3 x
lim ​ ​ ​ __________________
   
x2
 ​     ​
) x 2 d 2y
​ _____
dy
 + (3 – 2n) x ​ ___  ​ + (n – 1)2y
 ​ 
dx 2 dx
Applying L’Hospital Rule, we get,
Differentiation  5.55

= {(n – 2 + n – 1) + (n2 – 3n + 2)ln x}xn–1 fi f ¢(2) = b = 12a + 4b + c


+ {(3 – 2n) + (3 – 2n)(n – 1)ln x}x n–1 fi b = 12a + 4b + 18a + 2b
+ {(n – 1)2ln x}xn–1 fi b = 30a + 6b
fi – 5b = 30a
= {(2n – 3) + (3 – 2n)}xn–1
fi b = – 6a, c = 18a + 2b = 18a – 12a = 6a
+ {(n2 – 3n + 2) + (n2 – 2n + 1)
Now, f(1) = a + b + c + d
+ (5n – 2n2 – 3)}xn–1ln x = a – 6a + 6a + 6a = 7a
= (0)xn–1 + (0)xn–1 ln x fi f (1) = 7a
= 0. fi a = 7a
10. Given = (C1 + C2 x)sin x + (C3 + C4x)cos x ...(i) fi a=0
dy Thus, f (x) = ax3 + bx2 + cx + d
​ ___  ​ = (C1 + C2 x)cos x + C2 sin x
dx = ax3 – 6a x2 + 6a x + 6a
– (C3 + C4x)sin x + C4cos x =0
Thus, f(x) is an independent of x.

{ 
= (C1 + C2 x + C4)cos x + (C2 – C3 – C4x)sin x
_____________
d 2y
​ ___2  ​ = C2cos x – (C1 + C2 x + C4)sin x
​ 7  ​ (1 + cos2x) + ÷
12. y = cos–1​ __
2
  2 x – 48cos
​ sin 2
  x ​ sin x  ​ }
dx
+ (C2 – C3 – C4x)cos x – C4sin x ( 
= cos–1​ 7cos2 x + ​÷sin 2
_____________
  x ​ sin x )​
  2 x – 48cos
__________
= (2C2 – C3 – C4x)cos x – (C1 + C2 x + 2C4)sin x
​ 1  – 49cos
= cos–1(7cos2 x + ÷ 2
  x ​ sin x)
...(ii) __________
d3y
___
​  3  ​ = – (2C2 – C3 – C4x)sin x – C4cos x ​ 1  – 49cos
= cos–1(7cos x. cos x + ÷ 2
  x ​ sin x)
dx
– (C1 + C2 x + 2C4)cos x – C2sin x = cos–1(cos x) – cos–1(7cos x)
= x – cos–1(7cos x)
= – (3C2 – C3 – C4x)sin x – (C1 + C2 x + 3C4)cos x
d 4y dy sin x
​ ____4 ​ = – (3C2 – C3 – C4x)cos x + C4sin x ​ ___  ​ = 1 + ​ ___________

__________    ​
dx ÷​ 1  – 49cos 2
  x ​
dx
+ (C1 + C2 x + 3C4)sin x – C2cos x dy sin x
fi ​ ___  ​ = 1 + ​ ______________
     ​
_____________
= – (4C2 – C3 – C4x)cos x + (C1 + C2 x + 4C4)sin x dx   x – 48cos
÷​ sin 2 2
  x ​
...(iii)
d 4
y d 2
y Hence, the result.
Now, ____
​  4 ​ + 2 ​ ___2  ​ + y
dx dx
= {– (4C2 – C3 – C4x) + 2(2C2 – C3 – C4x)
y
13. Given x = tan ​ __ (  ) (  (1 + tan(y/2))2
​    ​  ​ – ln ​ ​  ____________
2
  
  
tan(y/2)
 ​  ​ )
​    ​ )​ – 2ln ​( 1 + tan​( __
= tan ​( __ ​    ​ )​ )​ – ln ​( tan​( __
​    ​ )​ )​
+ (C3 + C4x)}cos x + {( 1 + C2 x + 4C4) y y y
– 2(C1 + C2 x + 2C4) + (C1 + C2 x)}sin x 2 2 2
= (0)cos x + (0)sin x
= 0.
11. Given f(x) = x3f(1) + x2f ¢(2) + xf ¢¢(3) + f ¢¢¢(4)
dx 1 2 __
fi ​ ___  ​ = __
dy 2 2
y
(  )
sec2 (y/2)
​   ​  sec ​ ​    ​  ​ – ​ ___________
  
y
​ 1 + tan​ __

( 
 ​
​    ​  ​  ​
2 (  ) )
Let f(1) = a, f ¢(2) = b, f ¢¢(3) = c, f ¢¢(4) = d

1
– ________
​     ​  
1
 ◊ ​__ (  )
y
    ​ sec2 ​ __
​    ​  ​
Then f(x) = ax3 + bx2 + cx + d
fi f ¢(x) = 3ax2 + 2bx + c
(  (  ) )
y
​ tan ​ __
​    ​  ​  ​
2
2 2

fi f ¢¢(x) = 6ax + 2b
dx
dy 2
y
fi 2 ​ ___  ​ = sec2 ​ __ (  ) (  ​ 
2
​    ​  ​ ​ 1 – __________ ​ 
1
     ​ – _______
1 + tan(y/2) tan(y/2)
    ​  ​ )
fi f ¢¢¢(x) = 6a


f ¢¢¢(4) = d = 6a
f ¢¢(3) = c =18a + 2b

dx
fi 2 ​ ___  ​ = sec2  ​ __
dy 2 (  ) ( 
y 1 – tan(y/2)
​    ​  ​ ​ ​  __________
1
    ​ – _______
​     
1 + tan(y/2) tan(y/2)
 ​  ​ )
5.56  Differential Calculus Booster

(  ) (  ÷ 
__________________


dx
dy 2
y
2 ​ ___  ​ = sec2 ​ __
– tan2 (y/2) – 1
​    ​  ​ ​ ​  __________________
   )
    ​  ​
(1 + tan(y/2))tan(y/2)

3
= ​  ​ __________________
  
        ​ ​
cos x(4cos3x – 3cos x)

(  ÷ 
___________


dx sec4 (y/2)
2​ ___  ​ = – ​ ​ __________________
dy
   
  
{1 + tan(y/2)}tan(y/2)
 ​  ​ )
3
= ​  ​ ___________
   
    ​ ​
cos x cos(3x)

Hence, the result.

(  )
{1 + tan(y/2)}tan(y/2) _______
1 dy
fi ​ __ ​ ​  ___  ​ = – ​ ​  __________________ ​ x  2 + y2 ​ 
15. Given ÷
–1
     ​ 
   ​ = a ​etan
​ x​
2 dx sec4 (y/2)

( 
–1

)
dy 4{1 + tan(y/2)}tan(y/2) fi (x2 + y2) = a2​e2tan
​ x

fi 2​ ___  ​ = – ​ ​  ___________________
     ​ 
   ​
dx sec4 (y/2) dy 2
2x + 2y ​ ___  ​ = a2​e2tan ​ × _____
–1
x
fi ​ ​    2 
 ​
dx 1+x
dy 2tan(y/2)
fi 2​ ___  ​ = – ​ ____________
       ​ dy a2​e2tan ​
–1
x

dx (1 + tan2 (y/2)) fi x + y ​ ___  ​ = ​ _______   ​

( 
dx (1 + x )2

2tan(y/2)
​ 1  + ​ ___________
   
1 + tan2 (y/2)
 ​ +
1 – tan2 (y/2)
​ __________
    ​  ​ fi
1 + tan2 (y/2) ) dy
x(1 + x2) + y(1 + x2) ​ ___  ​ = (x2 + y2)
dx
when x = 0, then y2 = a2
dy
fi 2 ​ ___  ​ = – sin y(1 + sin y + cos y) dy
when x = 0, y2 = a2, then ​ ___  ​ = a
dx dx
Hence, the result. Again differentiating both sides w.r.t x, we get

( ÷  )
_____
cos3x
(  )
14. –1 _____
Given y = cos  ​ ​ ​  3  ​ ​    ​ dy 2 dy
cos x (1 + x2) + 2x2 + (1 + x2) ​​ ​ ___  ​  ​​ ​ + 2xy ​ ___  ​

÷ 
_____ dx dx
cos3x
fi cos(y) = ​ _____ ​  3  ​ ​ 
  dy 2
dy
cos x fi y(1 + x2) ​ ___2  ​ = 2x + 2y ​ ___  ​
dx dx
cos3x 4cos3x – 3cos x
fi cos2 (y) = _____ ​  3  ​ = ​ _____________

    
​ dy
cos x cos3x Put x = 0, y = a, ​ ___  ​ = a, then
dx
fi 2
cos (y) = 4 – 3sec x 2 d 2y
fi 1 + a2 + 0 + a(1 + 0) ​ ___2  ​ = 0 + 2a2
fi cos2 (y) = 4 – 3(1 + tan2 x) dx
2 2 2 d 2y
___ 2
fi cos (y) = 1 – 3tan x fi 1 + a + a ​  2  ​ = 2a
2 2 dx
fi 1 – sin y = 1 – 3tan x
(  )
2
dy 2
a –1
fi sin2y = 3tan2 x fi ​ ___2  ​ = ​ ​ _____ a    ​  ​
__ dx
fi sin y = ÷    tan x
​ 3 ​ 
Differentiating both sides w.r.t x, we get, Hence, the value of y¢¢(0) is ​ ​ _____
a2 – 1
a    (  1
​  ​ = ​ a – __ ) (  )
​ a ​  ​
dy __
fi cos y ​ ___  ​ = ÷    sec2 x
​ 3 ​  Integer Type Questions
dx
__ __
dy ÷    sec2 x _________

( 
​ 3 ​    
​÷3 ​
fi ​ ___  ​ = ​ _______
)
cos y    
​ = ​       ​ x – x–1
dx cos y cos2 x 1. We have y = cos–1 ​ ​ ______–1 
 ​  ​, x > 0

÷ 
__________ x+x
dy 3
fi ​ ___  ​ = ​ __________
​  2      ​ ​

(  ( 
dx cos y cos4x
) )
÷ 
x2 – 1 – x2
1_____
______________
3 fi y = cos–1 ​ ​ _____   ​  ​ = cos –1
 ​ –  ​    ​  ​
= ​ ______________
​       ​ ​ x2 + 1 x2 + 1
(1 – 3tan2 x)cos4x


÷ 
__________________
​  2   
3
= ​ __________________
    ​ ​
cos x(cos2 x – 3 sin2 x)
fi ( 
1 – x2
y = p – cos–1 ​ ​ _____
x2 + 1

 ​  ​ )
÷ 
_______________
3 ÏÔ2 + tan -1 x : x < 0
= ​ _______________
​  2      ​ ​ fi y= Ì -1
cos x (4cos2 x – 3) ÔÓ2 - tan x : x > 0
Differentiation  5.57

Ï 2
Ô1 + x 2 : x<0 fi
1+t (  )
2t
x = sin–1 ​ ​ _____ 2 
2t
 ​  ​, y = tan–1 ​ ​ _____
1 – t2
  
(  )
 ​  ​

dy Ô
Ô
fi ​ ___  ​ = Ìnon existent : x = 0 fi x = 2tan–1t, y = 2tan–1t
dx Ô 2 dx
Ô- : x>0 fi ​ ___  ​ = 1
ÔÓ 1 + x
2 dy

dy
(  )
Thus, ​​ ​ ___  ​  ​​ ​ = 2
dx x = 0
d2 x
fi ​ ___2 
dy
 ​ + 3 = 0 + 3 = 3

2. Given 4x exy = y + 5sin2 x 6. Given 2x = (y1/3 + y–1/3)

(  ( 
fi (y1/3)2 – 2xy1/3 + 1 = 0
fi 4 ​ e xy xy dy
dx ))
+ xe  ​ y + x ​ ___  ​  ​  ​ =
dy
​ ___  ​ + 5 sin2x
dx    ​
2x ± ​÷4x 2
– 4 ​

______

fi (y ) = ​  ___________
1/3
    
when x = 0, then y = 0 2
_____
Now, put x = 0 and y = 0 in (i), we get, fi (y1/3) = x ± ​÷x  2 – 1 ​

(  ( 
_____

dy
))
4 ​ 1 + 0 ​ 0 + 0 ◊ ​ ___  ​  ​  ​ =
dx
dy
​ ___  ​ + 0
dx
fi ln(y) = 3 ln(x ± ​÷x  2 – 1 ​
Differentiating w.r.t x, we get,
)

( 
dy
fi ​ ___  ​ = 4

3. Given ÷
dx
_____
​ x  + y  
____
​ x  – y  
​+ ÷ ​= c
(dy/dx) ___________
​ ______
y   ​ = ​ 
3_____
(x ± ​÷x  – 1 ​
2
)

2x
     ​​ 1 ± ​ _______
_____    ​  ​
2​÷x  2 – 1 ​

)
(  ) (  ) (  )
_____
1 dy 1 dy
fi ​ _______   
_____  ​ ​ 1 + ​ ___  ​  ​ + _______
​  ____    ​ ​ ​ ___  ​ – 1  ​ = 0 (dy/dx) ___________ ± ​÷x  2 – 1 ​
3_____ x _________  
2​÷x  + y  
​ dx 2​÷y  – x  
​ dx fi ​ ______y     
​ = ​       ​​ ​  _____    ​ 
  ​
_____ ____ _____ ____ (x ± ÷ ​ x  – 1 ​
2
)
  ​÷x  – 1 ​
2

dy (​÷x  + y   ​)(​÷x  + y  
​ – ​÷y  – x   ​ + ​÷y  – x  
​) (dy/dx) ______ 3
fi ​ ___  ​ = _______________________________
​      
_____    
____  ​ fi ​ ______
dx c(​÷x  + y  ​+ ÷ ​ y  – x  
​) y    ​ = ​  _____      ​
÷​ x  – 1 ​ 
2

dy (x + y – y + x) 3y
fi ​ ___  ​ = ​  ____________
 ​
     dy
fi ​ ___  ​ = ______
​  _____
    ​
dx c2 dx ​÷x  2 – 1 ​  
dy 2x
fi ​ ___  ​ =
dx
d 2y 2
​ ___2 ​ 
c fi
dx(  )
dy 2
(x2 – 1) ​​ ​ ___  ​  ​​ ​ = 9y2

fi ​ ___2  ​ = __
​  2  ​ 
dx c
fi (  )
dy 2 dy d2y dy
2x ​​ ​ ___  ​  ​​ ​ + 2(x2 – 1) ​ ___  ​ ​ ___2  ​ = 18y ​ ___  ​

(  )
dx dx dx dx
d 2y
fi ​ c2 ​ ___  ​ + 3  ​ = 2 + 3 = 5


dx2
4. Let h(x) = [f(x)] 2 – [g(x)] 2
fi (  )
dy
dx
d 2y
x ​ ​ ___  ​  ​ + (x2 – 1) ​ ___2  ​ = 9y
dx



h¢(x) = 2 f(x)f ¢(x) – 2g(x)g¢(x)
h¢(x) = 2 f(x)g(x) – 2g(x)f(x) = 0
fi ​ ​ _____ y (  )
x2 – 1 ___


​ 
d2y __x ___
​ ​ 
dx 2
dy
(  )
  ​ + ​ y  ​​ ​    ​  ​ = 9
dx
g(x)
h(x) = c 7. Given f (x) = ​ ____
x   
​ 
h(10) = [f (10)] 2 – [g(10)] 2 x g¢(x) – g(x)
f ¢(x) = ​  ___________
 ​
    
= [f (3)] 2 – [g(3)] 2 x2
= [f (3)] 2 – [f ¢(3)] 2 2 g¢(2) – g(2)
f ¢(2) = ​  ___________
 ​
    
= 25 – 16 = 9 4
2 ×  6 – 4 __ 8

2t
5. Given sin x = ​ _____
1 – t 2
 ​ and cot y = ​ _____
 2    

​ f ¢(2) = ​ _________
 ​   
= ​   ​  = 2
2t 4 4
1+t

(  ) (  )
8. Given F(x) = f (g(x))
2t 1 – t 2
fi x = sin–1 ​ ​ _____  ​  ​, y = cot–1 ​ ​ _____
 2    

​  ​
1+t 2t fi F ¢(x) = f ¢(g(x))g¢(x)
5.58  Differential Calculus Booster

fi F ¢(3) = f ¢(g(3))g¢(3) h¢(x) = 2f (x)g(x) – 2g(x)f (x) = 0


= f ¢(6)g¢(3) = 2 × 4 = 8 h¢(x) = 0
2 3
9. Given f (x) = x + x h(x) = c
As we know that, if g is the inverse of f, then h(5) = 7
y = f (x) ¤ x = g(y)
h(x) = 7
dy
Now, ​ ___  ​ = 2x + 3 x2 h(2016) = 7.
dx
dx 1 12. Clearly, f (x) = a x2 + bx + c
fi g¢(y) = ​ ___  ​ = _______
​      ​
dy 2x + 3x2
f ¢(x) = 2a x + b
when y = 2, then x = 1

(  )
f ¢¢(x) = 2a.
dx 1 1
Now, g¢(2) = ​​ ​ ___  ​  ​​ ​x = 1 _____
  ​ = ​    
 ​ = __
​    Thus, 2a = 10 ¤ a = 5
dy ​y = 2    ​​ 2 + 3 5
f ¢(1) = 6 fi 2a + b = 6
 ​ Hence, the value of (5g¢(2) + 3)
b = 6 – 2a = 6 – 10 = – 4
=1+3
f (1) = 4 fi a + b + c = 4
= 4.
c = 4 – a – b = 4 – 5 + 4 = 3
10. Given x y = ex – y
Hence, the value of f (0) + 2 = 3 + 2 = 5
fi log(x y) = (x – y) log(e)
13. Given P (x) = ax3 + bx2 + cx + d
fi y log(x) = (x – y)
P ¢(x) = 2ax2 + 2bx + c
y dy dy
fi ​ __x ​ + log(x) ​ ___  ​ = 1 – ​ ___  ​ ...(i) P ¢¢(x) = 4ax + 2b
dx dx
P ¢¢(0) = 10 fi 2b = 10 fi b = 5
when x = e, then y = 0
P(0) = – 2 fi d = – 2
Put x = e and y = 0 in (i), we get,
P(1) = – 2 fi a + b + c + d = – 2
dy dy
0 + 1 ◊ ​ ___  ​ = 1 – ​ ___  ​ a + 5 + c – 2 = – 2
dx dx
dy
___ a + c = – 5 ...(i)
2 ​    ​ = 1
dx P ¢(0) = – 1 fi c = – 1
11. Given h(x) = {f (x)}2 + {g(x)}2 From (i), we get, a = – 4
h¢(x) = 2f (x)f ¢(x) + 2g(x)g¢(x) Hence, the value of (a + b + c + d + 10)
h¢(x) = 2f (x)g(x) + 2g(x)g¢(x) = – 4 + 5 – 1 – 2 + 10
=8
h¢(x) = 2f (x)g(x) + 2g(x)f ¢¢(x)

Hints and Solutions


Questions asked in Roorkee - JEE Exams – 2 1
= ____
​  ___  ​ × ______
​       ​
÷   ​  (1 + x)
​ 4x 

( 
1–x
1. Let sin–1 ​ ​ _____ 
1+x )
 ​  ​ and v = ÷
__
​ x    ​ 
– 1
= ________
 __ 
(1 + x)​÷x 
 ​
   ​

 ​ Then dv 1
Also, ​ ___  ​ = ____
​  __
  
du 1 (1 + x).(– 1) – (1 – x).1 dx 2​÷x   
​ ___  ​ = ____________
     ​ × ​  ___________________
​  __________      ​
  

÷  (  )
dx 1–x 2 (1 + x)2 du – 1
________
​ 1 – ​​ ​ _____
     ​  ​​ ​ ​ ​ ___  ​ ​   __   ​
1+x du ___
___ dx (1 + x)​÷x 
________    ​ 2
 ​ Thus, ​    ​ = ​    ​ = ​   ​    = – ​ ______
    ​
(1 + x) – 2  ​ dv dv
___ 1
____ (1 + x)
= ​ ________________
       ​ × _______
_______________ ​   2 
 ​ ​    ​ ​  __
   ​ 
dx 2​÷x 
   ​
  + x) – (1  
​÷(1 2 2
– x)  ​ (1 + )
2. We have (tan–1x)y + ycot x = 1
Differentiation  5.59

fi ey log tan–1x + ecot x log y = 1 dy 1
 ​ fi ​ ___  ​ = _________
​       ​

( 
dx a + b cos x

dy
dx
1
fi ey log tan–1x ​ log tan–1x . ​ ___  ​ + y . ​ _____
  
tan–1x 1 + x2
1
 ​ . ​ _____
   ​  ​ ) d 2y – b sin x
fi ​ ___2  ​ = – ​ ___________
  

b sin x
 ​ = ​ ___________
  
   ​
2
(a + b cos x)2
( 
dx (a + b cos x)
1 ___
+ ecot x log y ​ – cosec2 x . log y + cot x . ​ __
dy
y ​ . ​ dx  ​  ​ = 0 ) Hence, the result.

(  )
dy _____ _____
1 _____ 1
fi (tan–1x) y ​ log tan–1x ◊ ​ ___  ​ + y◊​_____
  –1
    ​ ◊​    2 
 ​  ​ ​ 1  – y2 
4. We have ÷ ​ 1  – t 2 
 ​ + ÷  ​ = a(y – t)
dx tan x 1 + x
put y = sin q and t = sin j
(  1 dy
+ ycot x ​ – cosec2 x ◊ log y + cot x ◊ ​__
 y ​ ◊ ​ ___  ​  ​ = 0
dx ) cos q + cos j = a(sin q – sin j)


dy
fi (log(tan–1x)tan–1x) y + cot x ◊ ycotx – 1) ​ ___  ​
dx
q + j 
2cos ​ ​ _____
 2 (  ) (  )
q – j 
 ​ cos ​ ​ _____
 ​  
 2
 ​  
 ​

(  1+x
y
= ​ ycot x cosec2 x.log y – (tan–1x)y – 1 ​ _____
​    2 
 ​  ​  ​
(  )) q + j 
= a ◊ 2 cos ​ ​ _____
 2 (  ) (  ) ( 
q – j 
 ​  cos ​ ​ _____
 ​  
 2
q + j 
 ​ cot ​ ​ _____
 ​  
 2
1
 ​ = __
 ​   )
​ a ​

dy ((   
​ ___  ​ = ​ ​ _________________________________
    
    
y
​ ycot xcosec2 x.log y – (tan–1x) y – 1​ _____
​     
1 + x 2
 ​  ​  ​
 ​  ​
(  )) ) q – j 
​ ​ _____
 2
 ​  
1
 ​ = cot–1 ​ __(  ) (  )
​ a ​  ​

dx log(tan–1x)(tan–1x) y + cot x.ycot x – 1 1
q – j = 2cot–1 ​ __
​ a ​  ​ (  )
3. We have
1
sin–1y – sin–1t = 2cot–1 ​ __
​ a ​  ​ (  )
dy
( ÷ 
_____ 1 1
​ ______  ​ ​ ___ ​  – _______
(  ) )
2 a–b x _____    ​  _____
     ​= 0
y = _______
​  ______     ​tan–1 ​ ​ ​ _____ 
  ​  ​tan ​ __
​    ​  ​  ​ ​÷1  – y  
2 dt
 ​ ​÷1  – t 2 
 ​
÷​ a  – b  
2 2
 ​ a + b 2

÷ 
______ _____
dy
fi ​ ___  ​ = _______
​  ______
2
     ​​ ________________
1
     ​
dy
​ ___ ​  =
÷​ 1 _____
– y2 ​  1_____
​ _______   ​= ​ ​ 
– y2

( 

dx ÷ ​ a  – b  
2 2
 ​1 + ​ ​  a
a

+
b
_____
b

x
 ​  ​ tan2 ​ __ ) (  )
​    ​  ​
2
dt ÷​ 1  – t2   ​ 1 – t2

(  _ _____

÷ 
_____ ____
2)

a – b __
◊ ​ ​ _____ 
a+b 2
1
 ​ ​ ◊ ​    ​ sec2 ​ __

x
​    ​  ​
2 (  )   ​ ​ ÷ 1 – t  
 ​ ​ Also, x = sin–1​ t​÷1  – t  ​ + ​÷t   ​  ​
_
= sin–1t – sin–1​÷t 
  ​ 
dy 2 (a + b)
fi ​ ___  ​ = _______
​  ______      ​.​ ____________________
  
   
(  )
dx 1 1 1
dx ÷ ​ a  2 – b2 
2 x
 ​ (a + b) + (a – b)tan  ​ __
​    ​  ​ ​ ___ ​  = ______
​  _____
     ​ – _____
​  ____   ​ ◊ ​___
   _  
2 dt ÷ ​ 1  – t  
2
 ​ ÷
​  
1 –  ​
t  2​
÷t   

÷ 
_____

(  )
a – b __ 1 x _ ____
 ​ ◊ ​ ​ _____   ​ ​ ◊ ​   ​  sec2 ​ __​    ​  ​ 2​÷t    – t  
  ​  – ​÷1 ​
a+b 2 2  ​ = ​  __________
_____  
  _
 ​ 2​÷1  – t 2  ​ ​÷ t  ​

÷  ( ÷  )
_________________________ ______
dy 1 a–b
fi ​ ___  ​ = ​ _______  ​ × ​ _____
​  2   2         ​ × (a + b)2 ​ 1 – y2
dx (a – b ) a + b ​ ​ ​ _____2    
  ​ ​  ​ 
dy 1–t

( 
___ ____________
(  (  ) )
 ​ Thus, ​    ​ = ​        ​
____

)
x _
​ 1 + tan2 ​ __ ​    ​  ​  ​ dx ​ 1  – t  ​
   ​ – ÷
2​__________
÷t 
2  ​ ​  _____     _ ​  ​
​  ____________________________
   
    
(  (  ) ) ( 
2​÷1  – t 2 
(  ) )
x x  ​ ​÷ t  ​
a ​ 1 + tan2 ​ __
​    ​  ​  ​+ b ​ 1 – tan2 ​ __
​    ​  ​  ​

( 
2 2

)
_______
dy   – y2)t  
2​÷(1 ​
​ ___  ​ = ​ ​  ____________
_       ​  ​
____

( 
dy 1 dx (​  2​÷t   ​  – ​÷1  – t  ​ )​
 ​ fi ​ ___  ​ = ​ ________________

)
   
dx
__________
a + b ​ ​    
2
x
1 – tan2 ​ __
​    ​  ​ (  ) f g h
x  ​  ​
1 + tan2 ​ __​    ​  ​
2 (  ) 5. We have D = ( xf )′ ( xg )′ ( xh )′
( x f )′′ ( x g )′′ ( x 2 h )′′
2 2
5.60  Differential Calculus Booster

3x2 3y2 ___ dy


f g h ​ ______
_____     ​ – ______
​  _____
   ​ ​    ​ = 0
= xf ¢ + f xg ¢ + g xh ¢ + h ÷​ 1  – x  
6
 ​ ​÷1  – y  6 dx
 ​
( x 2 f ¢¢ + 4 xf ¢ ) ( x 2 g ¢¢ + 4 xg ¢ ) ( x 2 h ¢¢ + 4 xh ¢ ) 3y2 ___ dy 3x2
​ _______
_____    ​​    ​ = ​ ______
_____  
​÷1  – y6 
 ​ dx ​÷1  – x6 
f g h

÷ 
_____
= xf ¢ xg ¢ xh ¢ dy x2 1 – y6
 ​ ​ ___  ​ = ​ __2  ​ ​ ​ _____ 

( x 2 f ¢¢ + 4 xf ¢ ) ( x 2 g ¢¢ + 4 xg ¢ ) ( x 2 h ¢¢ + 4 xh ¢ )  ​ dx y 1 – x6
__
 ​ ​ ​ ÷__x    ​)​(sin–1(1 – ​÷x 
7. Let u = lo​g(1 – ​   ​)  )
(R2 Æ R2 – R1) __

f g h and v = 2​ 2(1 – ​
​ ÷x   ​)  ​
= xf ¢ xg ¢ xh ¢ (R3 Æ R3 – R1) u = logt (sin–1t) and v = 22t
__
x 2 f ¢¢ x 2 g ¢¢ x 2 h ¢¢ where t = (1 – ​÷x    ​) 
1 1 1
f g h log t ◊ ​_____
  –1 ◊ ​______
   ​     _____      ​– __
​   ​  log(sin–1t)
t
du sin t ÷ ​ 1  – t  
2
 ​
= x   f ¢3g ¢ h¢ ​ ___ ​ = ​ ____________________________
      ​   
dt (log t)2
f ¢¢ g ¢¢ h ¢¢ _____
du t log t – log(sin t)​÷1  _____
–1
Thus, D¢ – t 2 ​  
sin–1t
​ ___ ​ = _________________________
​           ​,
dt t sin–1t(log t)2.​÷1  – t 2 ​ 
f g h f g h
3
= 3x ¢ g ¢ h ¢ + x f ¢ g ¢ h ¢
2 f
dv dt 1
​ ___ ​  = 22t + 1log2 and ​ ___   ​ = – ​ ____
   ​ 
__
f ¢¢ g ¢¢ h ¢¢ f ¢¢¢ g ¢¢¢ h ¢¢¢ dt dx 2​÷x   ​ 
du ___
___  dv
du ___
Thus, ​    ​ = ​   ​  /​   
f g h f g h dv dt dt
_____
= f¢ g¢ h¢ + f¢ g¢ h¢   t log t – log(sin–1t)​÷1  – t 2   ​sin–1t ____ 1
 ​ = __________________________
​     
   
_____  ​◊ – ​  __
   ​ 
3 x 2 f ¢¢ 3 x 2 g ¢¢ 3 x 2 h ¢¢ x 3 f ¢¢¢ x 3 g ¢¢¢ x 3h ¢¢¢ t sin t(log t) ◊ ​÷1  – t  
–1 2  2 2t + 1
 ​ ◊ 2 log 2 2​    ​
÷x 

__
f g h where t = (1 – ​÷x 
  ​) 

= f¢ g¢ h¢ 8. Given f (x) = logx (sin(x2)) + (sin x2​)lo​


​ g​e​x
2 2 2 _____
( x f ¢¢¢ ) ( x g ¢¢¢ ) ( x h ¢¢¢ ) ​ x  + 1 ​
​ and let g(x) = ÷  

Hence, the result. f ¢(x)


To find ____
​    
_____ _____ g¢(x)
6. Given ÷ + ​÷1  – y6 ​ 
​ 1  – x6 ​  = a3 (x3 – y3)
 ​ Now, f (x) = log x (sin(x2)) + (sin x2​)lo​
​ g ​e​  x​
Put x3 = sin q, y3 = sin j log(sin(x2))
= ​ __________ + e log x log(sin x​​ )​
2
   ​ 
cos q + cos j = a3 (sin q – sin j) log x log(sin(x2))
log(x) ◊ cot(x2) ◊ 2x – ​ __________
q + j 
2cos ​ ​ _____
 2 ( 
 ​   ) (  )
q – j 
 ​ cos ​ ​ _____
 2
 ​  
 ​ fi f ¢(x) = ​ __________________________
   
(log x)2
 ​
x     ​ 
  

(  ) (  )
q + j 
= a3 . 2cos ​ ​ _____
 2
q – j 
 ​ sin ​ ​ _____
 ​  
 2
 ​  
 ​ ( 
log(sin(x2))
+ (sin(x2)) log x ◊ ​ ​ __________ + 2x log x.cot(x2)  ​
x     ​  )
(  )
q – j 
cot ​ ​ _____
 2
 ​ = a3
 ​   Also, g¢(x) = _______
f ¢(x)
1
​  _____
   
 ​
2​÷x  + 1 ​

(  )
q – j  Thus, ____
​   

[ 
​ ​ _____  ​ = cot–1(a3)
 ​   g¢(x)
 2 log(sin(x2))
log(x) . cot(x 2
).2x – __________
​ 
q – j = 2 cot–1(a3) 1 x     ​ 
 ​ = _______
​  _____  ​ ​ ​  __________________________
         ​   
sin–1(x3) – sin–1(y3) = 2 cot–1(a3) 2​÷x  + 1 ​
  (log x)2
Differentiation  5.61

( 
log(sin(x ))
+ (sin(x2)) log x. ​ ​ __________
2
+ 2x log x.cot(x2)  ​  ​
x     ​  ) ] 5.



We have, h(x) = [f (x)] 2 + [g(x)] 2



h¢(x) = 2f (x)f ¢(x) + 2g(x)g¢(x)
h¢(x) = 2f (x)g(x) + 2g(x)g¢(x)
h¢(x) = – 2f ¢¢(x)g(x) + 2g(x)f ¢¢(x) = 0
Questions asked in Past IIT-JEE Exams Thus, h(x) is a constant function.
But h(5) = 11

2. f ¢(1) = ​  
lim  
x Æ 1 ( 
f(x) – f(1)
​ ​ ​ ________
x–1
 ​  
 ​ )



h(x) = 11
h(10) = 11

= ​  
lim   (  x–1
​ __________
2x – 7x + 5 3
2
​ ​ ​  ______________
 ​ 
    
1
    ​ + __
  ​   ​ 
​ )
2x – 1
( 
6. Given y = f  ​ ​ ______
x2 + 1
  ​  ​
)
) ( 
x Æ 1 x–1

(  (  )
2
dy 2x – 1 (x___________________
+ 1).2 – (2x – 1).2x
= ​  
lim  
x Æ 1 
  3x – 3 + 2x2 – 7x + 5
​ ​ ​  __________________
    ​  ​
3(x – 1)(2x2 – 7x – 5) ) ​ ___  ​ = f ¢ ​ ​ ______
dx 2
x +1
  ​  ​ ​ ​     
(x2 + 1)2
 ​ 
   ​

= ​  
 x Æ 1 3
2
lim ​ ​ __
(  x2 – 2x + 1
​   ​  × ​  _________________
       ​  ​
(x – 1)(2x2 – 7x + 5) ) dy
dx ( 
x2 + 1

) ( 
 ​  ​ ​ ​ 
2
+ 2 – 4x2 + 2x 
2x – 1 2x_______________
​ ___  ​ = f ¢ ​ ​ ______   
(x2 + 1)2
 ​ 
   ​
)
= ​  
 x Æ 1 3
2
lim ​ ​ __
(  (x – 1)2
​   ​  × ​ _________________
       ​  ​
(x – 1)(2x2 – 7x + 5) ) dy
dx (  ) ( 
2x – 1 2__________
​ ___  ​ = f ¢ ​ ​ ______
2
x +1
  ​  ​  ​ ​ 
+ 2x + 2x2
  
(x2 + 1)2
 ​  

)
 x Æ 1  3
2
lim ​ ​ __
= ​   (  (x – 1)
​   ​  × ​ ____________
  
   ​  ​
(2x – 5)(x – 1) ) dy
dx ( (  ) ) ( 
2x – 1 2 2__________
​ ___  ​ = sin ​ ​​ ​ ______
2
  ​  ​​ ​  ​  ​ ​ 
+ 2x + 2x2
  
(x2 + 1)2
 ​  

)
( 
x +1
= ​  
lim  
x Æ 1 3
2
​ ​ __ ​ 
1
​    ​ × _______
   
(2x – 5)
 ​  ​ ) 7. We have (a + bx)e y/x = x
2
= – __
( 
__y
​   ​ 
9
5x
x
fi ​e​​ x ​​ = ​ ______
​     
a + bx
 ​  ​ )
3. Given y = ________  ​ + cos2 (2x + 1).
( 
​  _______
    y
3
  – x)  
​÷(1 2
 ​
x
fi ​ __x ​ = log  ​ ​ ______
a + bx
     ​  ​ )
2
__ 2 1
__ y
(1– x​)​3 ​ . 5 – __ fi ​ __x ​ = log x – log(a + bx)
​   ​  ​   ​ 
dy ​   ​  (1 – x​)​3 ​
3
​ ___  ​ = ​  __________________   
 ​    – 2sin(4x + 2)
dx 4
__ dy
x . ​ ___  ​ – y
​    ​
(1 – x​)​3 ​
dx 1 b
fi ​ ________  ​   = __
​ x ​ – ______
​       ​
dy 15(1 – x) – 2 x 2 a + bx
​ ___  ​ = ​  ___________
  
    ​– 2sin(4x + 2)
dx 5
__
​    ​ dy
3(1 – x​)​ ​
3 x . ​ ___  ​ – y
dx a + bx – bx
fi ​  ________  ​   = ​ __________    ​ 
dy (13 – 15x) x2 x(a + bx)
​ ___  ​ = ​ _________   ​– 2sin(4x + 2))
dx 5
__
​   ​  dy ax
3(1 – x​)​3 ​ fi x . ​ ___  ​ – y = ​ _______
    ​
3 dx (a + bx)
4. We have y = e​ x sin x
​ ​ + (tan x) x
d 2y dy dy (a + bx)a – ax.b
x . ​ ___2  ​ + ​ ___  ​ – ​ ___  ​ = ​ _____________
2
= e​ log(x sin(x
​ ))
​ + e x log(tan x) fi   
    ​
dx dx dx (a + bx)2
dy
​ ___  ​ = e​ log(x sin(x
dx

2
​ ​ ​ 
( 
)) _______1
 2  
x sin(x )
 ​.(cos(x2) + 2x2 cos(x2))  ​
) fi
d 2y a2
x . ​ ___2  ​ = ​ ________
    ​
( 
+ e x log(tan x) ​ log(tan x) + ____
x
​ . sec2 x  ​
​      
tan x )
dx (a + bx)2

dy 1
​ ___  ​ = (x sin(x2)) ​ ​ __
dx
(  2 2
x ​ . (cot(x ) + 2x  cos(x ))  ​
2
)
d 2y
dx
a2 x 2
fi x3. ​ ___2  ​ = ​ ________
(a + bx)
ax  2
 ​ = ​​ ​ ______
 2    
 a + bx (  )
dy
(  2
 ​  ​​ ​ = ​​ x ​ ___  ​ – y  ​​ ​
dx )
(  x
+ ​ tan x) x (log(tan x) + ____
tan x
​ . sec2 x  ​
​       ) Hence, the result.
5.62  Differential Calculus Booster

( 
( 
_____
8. Let
1
10. Let u = sec–1 ​ ______  ​  ​ and v = ​÷1  – x2 
​  2      ​
)
2x   
y = (logsin  x cos x)(logcos x sin x) –1 + sin–1​ ​ _____
1 + x2
 ​  ​
) 2x – 1


log(cos x) ________
y = ________
​ 
log(sin x)
 ​ 
log(cos  )
× ​ 
log(sin x)
+ 2tan–1x
 ​ 
1
Now, u = sec–1 ​ ______
​  2    ​  
2x – 1

(  )
{log(cos x)}2 Put x = cosq
fi y = ___________
​      ​ + 2tan–1x
(  )
2
{log(sin x)} 1
So, u = sec–1 ​ _____
​     ​ 

cos2q
2
{log(sin x)} (– 2log(cos x).tan x)
  fi u = sec–1(sec2q) = 2q = 2cos–1(x)
​         ​
dy
___ ​ – {log(cos x)}2 (2log(sin x).cot x) ​ _____
___________________________ 2 du 2
​    ​ = ​ 
dx
         ​ + ​    2   ​ fi ​ ___  ​ = – ______
​  _____   
 ​
{log(sin x)}4 1+x dx ÷​ 1  – x2 
 ​
_____

( (  ) )
dy Also, v = ​÷1  – x2 
 ​
fi ​ ​​ ​ ___  ​  ​​x = ​ __p ​ ​  ​ dv 1 ×_____ – 2x x
dx 4 fi ​ ___  ​ = ​ _______   ​ = – ______
  ​  _____
   
dx 2​÷1  – x  
2
 ​ ​÷1  – x2 

{  (  ) } (  (  ) )
1 2
​​ log​ ___
​  __  ​  ​  ​​ ​​ – 2log ​ ___
  
​÷2 ​ ÷

1
​  __  ​  ​  ​
  
2 ​
– ______
​  _____
2
   
​ 1  – x2 
÷
 ​

{  (  ) } (  (  ) )  
 ​         ​ du
___ _________  ​ 2
1 2 1  ​ Thus, ​    ​ = ​  x      ​ = __
​ x 
– ​​ log​ ___​  __  ​  ​  ​​ ​​ 2log​ ___ ​  __  ​  ​  ​ dv – ​ ______      ​

( )
 ​ _____
÷   
​ 2 ​ ÷   
​ 2 ​ 1 ÷​ 1  – x2 
=​ ​______________________   ​ + 2 ​ ______  ​

{  (  ) }
    4
 ​ ​    2   ​  ​
1 p 
​​ log ​ ___ 1 + ___
(​​  ​ ___dudx  ​ )​​
​  __  ​  ​  ​​ ​ ​   ​  2
÷   
​ 2 ​ 16  ​ Now, __
x = ​ __ ​ ​ = ​    ​ = 4
1
2 1
__
​   ​ 

(  )  
2
1
– 4log ​ ___
​  __  ​  ​ 11. No questions asked in 1987.

(  (  ) ) ( )
÷
​   
2 ​ 32
= ___________
​    2 ​ + ​ _______
   ​  2    
 ​  ​ 12. Given y2 = P(x)
1 p  + 16
​​ log ​ ____
​  __   ​  ​  ​​ ​ dy
÷    )
​ 2 ​ 2y ​ ___  ​ = P ¢(x)

(  )
dx
4 32
= – ​ ______________
__  ​ + ​ _______
(  )
    ​  2     ​  ​ d 2y dy 2
   ))
(log1 – log(​÷2 ​ p + 16 fi 2y ​ ___2  ​ + 2 ​​ ​ ___  ​  ​​ ​ = P ¢¢(x)
dx dx

(  ) (  )
2
8 32
= ______ + ​ _______
​     ​  ​  2     ​  ​ d 2y dy
(log2) p  + 16 fi 2y3 ​ ___  ​ + 2y ​​ ​ ___  ​  ​​ ​ = y2P ¢¢(x)
2 
2
dx dx
9. We have f (x) = logx(ln x)

log(log x)

d2y
2y3___
dx2
dy 2
​    ​ = y2P ¢¢(x) – 2y2  ​​ ___
​    ​ ​​ ​
dx (  )
= ________
(  (  ) )
​     ​ 
log x d 2y 1 dy 2
fi 2y3​ ___  ​ = y2P ¢¢(x) – ​ __  ​  ​​ 2y​ ​ ___  ​  ​  ​​ ​
dx2 2 dx
1 __ 1 1
log x . ​ ____ . ​ x ​ – log(log x) . ​ __
   ​   x ​ dy2
1
fi f ¢(x) = ​ 
log x
_______________________     ​    fi 2y3 ​ ___  ​= y2P ¢¢(x) – ​ __  ​ (P ¢(x))2
dx 2 2
(log x)2


1
__
​ x ​ (1 – log(log x))
f ¢(x) = ​ ______________   
 ​
  

dx
d
(  ) d2y
2 ​ ___   ​ ​ y3​ ____ ​  ​
dx2
(log x)2 dy 1
= 2y ​ ___  ​ P ¢¢(x) + y2P ¢¢¢(x) – ​ __ ​  .2 . P ¢(x).P ¢¢(x)
dx 2
(1 – log(log x))
fi f ¢(x) = ​ _____________
  
  
 ​ dy
x(log x)2 = 2y ​ ___  ​ P ¢¢(x) + y2P ¢¢¢(x) – P ¢(x) . P ¢¢(x)
dx
(1 – 1) dy
fi (f ¢(x)) x = e = ​ ______
 ​ 
 = 0 = 2y ​ ___  ​ P ¢¢(x) + y2P ¢¢¢(x) – P ¢(x) . P ¢¢(x)
e2 dx
Differentiation  5.63

= P ¢(x)P ¢¢(x) + y2P ¢¢¢(x) – P ¢(x) . P ¢¢(x) = 4a2 + 4ab + b2 – 8a2 – 4ab – 4ac
= y2P ¢¢¢(x) = – 4a2 – 4ac + b2
= P(x) . P ¢¢(x) = – 4a2 + (b2 – 4ac)
13. We have x = secq – cosq
< 0 for all x in R.
and y = secnq – cosnq
Thus, g(x) > 0, " x Œ R
dx
fi ​ ___ ​ = secq tanq + sinq dy
dq 16. Find ​ ___  ​ at x = – 1, when
dx
dx
fi ​ ___ ​ = tanq (secq + cosq) Given
dq __
÷   
​ 3 ​
Also, (sin y) sin(p x/2)
+ 2 tan(ln(x + 2)) + ​ ___ ​  sec–1(2x) = 0
x
2
dy
fi ​ ___ ​ = n secn – 1q.secq.tanq + n cosn – 1q.sinq
dq {  (  )
(sin y) sin(p x/2) ​ sin ​ ___
p x dy
​   ​  ​ cot y ​ ___  ​
2 dx
dy
fi ​ ___ ​ = n secnq.tanq + n cosnq.tanq
dq
p 
+ __ (  )
p x
​   ​  cos ​ ___
2 }
​   ​  ​ log(sin y)  ​ + 2x log 2tan log(x + 2)
2
__
dy x
sec2 (ln(x + 2))
2______________
fi ​ ___ ​ = n tanq (sec nq + cos nq .) + ​    
 ​
  
​÷3 ​
   + ​   ​ × ​ __________
___ 2x
______      ​= 0
dq (x + 2) 2
n n 2x​ ÷  
4x 2
– 1 ​

dy dy dx n ◊ tanq (sec  q + cos  q .)
fi ​ ___  ​ = ​ ___ ​ ∏ ​ ___ ​ = ​ ____________________
        ​ Put x = – 1, we get,
dx dq dq tanq (secq + cosq )

( 
__

(  ) n2 (sec nq + cos nq)2
dy 2 ________________
___
fi ​​ ​    ​  ​​ ​ = ​    
    ​
dy
(sin y)  ​ – cot y ​    ​  ​ + ​   ​ × ​ ___
–1 ___
dx )
___
2
  
​÷3 ​
÷

1__ __

3 ​

1
    ​+ ​   ​  = 0
2

(  )
dx (secq + cosq )2 dy
cot  ___
​ – ​ ____   ​ ​    ​  ​ + 1 = 0
sin y dx

dx(  )
dy 2 n2 (sec nq – cos nq)2 + 4
fi ​​ ​ ___  ​  ​​ ​ = ___________________
​         ​
(secq – cosq )2 + 4
cos y ___
​ _____  
2 dx
dy
 ​ ​    ​ = 1
sin y

(  )
2 2
dy 2 n (y + 4) dy sin2y
​​ ​ ___  ​  ​​ ​ = ​ ________  ​

 . ​  ___  ​ = ​ _____ ​ = sec y tan y
  
dx (x2 + 4) dx cos y
17. No questions asked in 1992.
14. We have f (x) = |x – 2| 18. Yes, it is true, the derivative of every odd function
is an even function.
Ï( x - 2) : x≥2
= Ì 19. Ans. (a)
Ó- ( x - 2) : x < 2 Given y = (sin x) tan x
Now, g(x) = f (f (x)) y = etan x log sin x
dy
= f (x – 2) ​ ___  ​ = e tan x log sin x (tan x.cot x + sec2 x log sin x)
dx
= (x – 2) – 2
dy
= x – 4, x ≥ 4 ​ ___  ​ = (sin x) tan x (1 + sec2 x log sin x)
dx
Thus, g¢(x) = 1, x > 20 20. No questions asked in 1995.
15. Let f (x) = ax2 + bx + c, a > 0 (since f (x) is + ve for 21. We have xexy = y + sin2 x

( 
all real values of (x)
f ¢(x) = 2ax + b
dy
exy.1 + xexy ​ x ​ ___  ​ + y.1  ​ =
dx ) dy
​ ___  ​ + sin 2x
dx
f ¢¢(x) 2a
Now, g(x) = f (x) + f ¢(x) + f ¢¢(x) fi (  dy
)
exy + ​ x2exy ​ ___  ​ + y.1  ​ =
dx
dy
​ ___  ​ + sin 2x
dx
= (ax2 + bx + c) + (2ax + b) + 2a
when x = 0, then y = 0
= a x2 + (2 + b)x + (2a + b + c) Thus,
dy
2
Now, D = (2a + b) – 4a(2a + b + c) fi 1 + (0 + 0) = ​ ___  ​ + 0
dx
5.64  Differential Calculus Booster

dy x3
fi ​ ___  ​ = 1 fi y = ​ ________________
     ​
dx (x – a)(x – b)(x – c)
22. Ans. (d)
We have
x3 sin x cos x
x3
  ( 
fi log y = log ​ ​ ________________
    ​  ​
(x – a)(x – b)(x – c) )
f (x) = 6 -1 0 fi log y = log(x3) – log((x – a)(x – b)(x – c))
fi log y = 3log x – log((x – a)(x – b)(x – c))
0 p2 p3
fi log y = 3log x – log(x – a) – log(x – b)
3x3 cos x - sin x – log(x – c)
d dy
​ ___   ​ ( f (x)) = 6 -1 0 1
fi __
3
​ y ​ ​ ___  ​ = __
1
​   ​ – ______
​     
1
 ​ – ______
​     
1
 ​ – ______
​       ​
dx dx x (x – a) (x – b) (x – c)
0 p2 p3

d2
6 x - sin x - cos x
1 dy
fi __
​ y ​ ​ ___  ​ =
dx ( 1 1
​ __
​ x ​ ​ ______
   
(x – a) ) ( 
 ​  ​ +
1 1
​ __
​ x ​ ​ ______
   
(x – b) ) ( 
 ​  ​ +
1 1
​ __
​ x ​ ​ ______
   
(x – c) )
 ​  ​
___

) ( 
​  2   ​ (f (x)) = 6 -1 0
dx
p p 2
p3
1 dy
fi __
dx
x–a–x
( 
​ y ​ ​ ___  ​ = ​ ​  _______ 
x(x – a)
x–b–x
 ​  ​ + ​ ​  _______ ​ 
(x – b)

  ​ )
+ (​  ​  _______ ​ 
(x – c) )
x–c–x
6 - cos x - sin x   ​
d3
___
​  3   ​(f (x)) = 6 -1 0
​   ​ ​  ​( ​     
 ​ )​ + ​( _____
dx
dx x ( a – x
 ​ )​ )​
b – x)
dy y _____
 ​  ​ + ​( ____
2
p3 a b c
p p fi ​ ___  ​ = __ ​      ​ c –  x 

dx
d3
( 
Now, ​​ ​ ___3   )
 ​ ( f (x))  ​​ ​
x = 0

24.
Hence, the result.
No questions asked in 1999.
6 -1 0 25. Ans. (b)
= 6 -1 0 We have x2 + y2 = 1
fi 2x + 2y y¢ = 0
p p2 p3
fi x + y y¢ = 0
=0 fi 1 + (y¢)2 + y.y¢¢ = 0
Which is independent of p. 26. Ans. (c)
x
23. We have
ax2 bx We have F(x) = Ú​  ​    ​ f (t)dt
y = ​ ________________
       ​ + ___________
​      0
(x – a)(x – b)(x – c) (x – b)(x – c)
fi F ¢(x) = f (x)
c
 ​ + ____
​ x –  c 
 ​ + 1 Given F(x2) = x2 + x3
ax2 bx fi F ¢(x2).2x = 2x + 3x2
fi y = ​ ________________
       ​ + ___________
​       ​
(x – a)(x – b)(x – c) (x – b)(x – c)
2x + 3x2 3
fi F ¢(x2) = ​ _______
   ​ = 1 + __
  ​   ​  x
2x 2
c + x – c 
+ ​ ________  

x – c    3
fi f (x2) = 1 + __
​   ​  x
ax 2
bx x  2
fi y = ​ ________________
       ​ + ___________
​       ​ + ____
​      Put x = 2
(x – a)(x – b)(x – c) (x – b)(x – c) x – c
3
fi f (4) = 1 + ​ __ ​  ◊ 2 = 1 + 3 = 4
ax2 bx + x2 – bx 2
 ​ fi y = ​ ________________
       ​ + ___________
​      ​
(x – a)(x – b)(x – c) (x – b)(x – c) 27. No questions asked in between 2002 -2003.
28. Ans. (a)
ax2 x2
fi y = ​ ________________
       ​ + ​ ___________
    We have log(x + y) = 2xy
(x – a)(x – b)(x – c) (x – b)(x – c)

 ​
ax2 + x3 – ax2
fi y = ​ ________________
       ​
(x – a)(x – b)(x – c)
1
fi ​ ______
   
(x + y)
dy
dx (  dy
dx ) ( 
 ​ ​ 1 + ​ ___  ​  ​ = 2  ​ x ◊ ​ ___  ​ + y ◊ 1  ​ )
Differentiation  5.65

1 1 dy dy 32. We have f (x) = g(x)sin x


fi ​ ______  ​ + ______
    ​       ​ ◊ ​ ___  ​ = 2x ◊ ​ ___  ​ + 2y
(x + y) (x + y) dx dx fi f ¢(x) = g(x)cos x + g¢(x)sin x
1
fi ​ ______
​     
(x + y) (  dy 1
)
 ​ – 2x  ​ ◊ ​ ___  ​ = ​ 2y – ______
dx
​     
(x + y)
 ​  ​ (  ) fi

f ¢¢(x) = g¢(x)cos x – g(x)sin x
+ g¢(x)cos x + g¢¢(x)sin x
1
​ 2y – ______
​      ( 
 ​ ​ )   Now, f ¢(0) = g(0) + g¢(0) ◊ 0 = g(0)
dy
dx 1
(x
fi ​ ___  ​ = ​ ____________
​ ______
​     
+ y)

( 
    ​ =
 ​– 2x  ​ )
( 2y(x + y) – 1
​ ​ ____________
   )
    ​  ​
1 – 2x(x + y) So Statement-II is true and f ¢¢(0) = 2g¢(0)
(x + y) Now, ​  
lim  
​ (g(x)cot x – g(0)cosec x)
x Æ 0
when x = 0, y = 1.
dy
(  )
fi ​​ ​ ___  ​  ​​
dx x = 1, y = 1
​=
(2 – 1)
​ ______ 
(1 – 0)
 ​ = 1

 x Æ 0 ( 
g(x)cos x – g(0)
lim ​ ​ ​ _____________
= ​  
sin x
      
​  ​ )
29. No questions asked in between 2005-2006.
30. Ans. (d)
= ​  
lim  
x Æ 0
( 
g¢(x)cos x – g(x)sin x
​ ​ ​ _________________
  ​  ​
cos x     )
d2 x
We have ​ ___2 
dy ( 
g¢(0)cos 0 – g(0)sin 0
= ​ ​ _________________
 ​  
     ​ )
(  )
cos 0
d dx
 ​ = ___
​     ​  ​ ​ ___  ​  ​ = g¢(0)
dy dy

(  )
=0
d dx dx
= ​ ___   ​ ​ ​ ___  ​  ​ . ​ ___  ​ = f ¢¢(0)
dx dy dy


d 1 ___
= ___  ( )
​     ​ ​ ___
dx ___
dx
​    ​   ​ . ​    ​
dy dy
​    ​
Thus, statement-I is also true.
But statement-II is not a correct explanation for
statement-I
dx 33. Ans. (a)
1 ___ d2y dx We have g(x) = log(f ( ))
= – ​ _____  2 ​   . ​  2  ​ . ​ ___  ​
(  )
dy dx dy Now, g(x + 1) – g(x)
​​ ​ ___  ​  ​​ ​ = log(f (x + 1)) – log(f (x))
dx

= – _____
1 ____
​   2 ​  
 d2y 1
. ​  2 ​  . ​ ____
   ​  (  )
f (x +1)
= log ​ ​ ______ ​  
 ​

(  ) (  )
f (x)
dy
___ dx ___ dy

(  )
​​ ​    ​  ​​ ​ ​ ​    ​  ​
dx dx x f (x)
2 = log ​ _____
​   ​  ​
1 ___ d y f (x)
= – ​ _____  3 ​  . ​  2  ​
(  )
dy
___
​​ ​    ​  ​​ ​
dx
dx
1
g¢(x +1) – g¢(x) = __
= log x
​ x ​

(  )
2
dy – 3 d y 1
= – ​​ ​ ___  ​  ​​  ​. ​ ___2  ​ g¢¢(x + 1) – g¢¢(x) = – __ ​  2  ​ 
dx dx x
1
31. We have f (x) = 2 + cos x, " x Œ R Put x = k – __ ​    ​
2
f ¢(x) = – sin x, " x Œ R
clearly, f(x) is continuous and differentiable so, there (  )
1
g¢¢ ​ k + __
1
(  ) 4
​   ​   ​ – g¢¢ ​ k – ​ __ ​   ​ = – ________
2 2
​     
(2k – 1)2
 ​

(  ) (  )
exist a point c Œ (t, t + p) such that f ¢(c) = 0 1 1
Thus, statement-I is true, Now, g¢¢ ​ N + ​ __ ​   ​ – g¢¢​ ​ __ ​   ​
2 2

= ​S ​ ​   ​​( g ¢¢ ​( k + __


​   ​  )​ – g¢¢ ​( k – __
​   ​  )​ )​
Also, f (t + 2p) = 2 + cos(t + 2p) N
1 1

= 2 + cos t k = 1 2 2

(  )
= f (t) N
4
​   ​ ​   ​​ – ________
=S ​       ​  ​
Thus, statement-II is also true k = 1 (2k – 1)2
But statement-II is not a correct explanation for
statement-I

( 
1
= – 4 ​ 1 + __
9 25
1
​   ​  + ___
1
​    ​ + ... + ________
​     
(2N – 1)2
 ​  ​
)
5.66  Differential Calculus Booster

34. No questions asked in between 2009-2010.


35. (a)

(  (  sinq 
= sin ​ sin–1 ​ ​ _____________

____________
))
    ​ ​  ​
  q + cos2 q 
 ​÷sin 2
   ​

(  ( ÷  ))
b–x
Given f (x) = ​ ______   
 ​ sinq 
1 – bx = sin ​ sin–1 ​ ​ ___________________
       
__________________  ​ ​  ​
b–x b–y  ​ sin q + cos2q –  
2
sin2q  ​
Let y = ​ ______     ​ fi x = ​ ______   

 ​
1 – bx

0 < x < 1 fi 0 <​ ______ 


b–y
1 – by


 ​ < 1
(  (  ) ) sinq 
= sin ​ sin–1 ​ ​ _____ 
 cosq
  
​ ​   ​

1 – by = sin(sin–1(tanq ))
b–y 1
fi ​ ______    ​ > 0 fi y > b, y < ​ __ ​ = tanq
1 – by b
d
b–y 1 _______
​       ​(f (q ))
fi ​ ______  ​ – 1 < 0 fi – 1 < y < __
   ​   ​ d (tanq )
1 – by b
d
= _______
​      ​ (tanq )
fi –1<y<b d(tanq )

(  (  ))
= 1.
sinq 
36. We have f (q) = sin ​ tan  ​ ​ _______
–1
  ​ 
_____ ​  ​ 37. No questions asked in between 2012-2014.

 ​÷cos2q ​  
Chapter
Rolle’s theorem and
6 Lagrange’s mean
value theorem

Here, f (x) is not continuous at x = a


Concept Booster
So, Rolle’s theorem is not applicable here.
(ii)
1.  Rolle’s Theorem Y

1.1  Introduction
In calculus, Rolle’s theorem essentially states that any real-
valued differentiable function that attains equal values at
two distinct points must have a stationary point somewhere
between them—that is, a point where the first derivative
(the slope of the tangent line to the graph of the function) O x=a x=b x=c
X
is zero.

1.2  Rolle’s Theorem Statement Here, f (x) is not continuous at x = b
So, Rolle’s theorem is not applicable here.
Let f (x) be a real function and a, b Œ R.
(iii)
If
(i) f (x) is continuous in [a, b]
(ii) f (x) is differentiable in (a, b)
(iii) f (a) = f (b)
Then there is a number c in (a, b) such that f ¢(c) = 0.


Here, f (x) is not differentiable at x = c
So, Rolle’s theorem is not applicable here.
(iii)

1.3  Applicability of Rolle’s theorem


(i)

Here, f (x) is not continuous at x = b


6.2  Differential Calculus Booster

So, Rolle’s theorem is not applicable here. 2. Lagranges Mean Value Theorem
(iv)
2.1  Introduction
In mathematics, the mean value theorem states, roughly: that
given a planar arc between two endpoints, there is at least
one point at which the tangent to the arc is parallel to the
secant through its endpoints.
The theorem is used to prove global statements about a
function on an interval starting from local hypotheses about
derivatives at points of the interval.

2.2  Statement
Let  f (x) be a real function and a, b Œ R. If
Here, f (a) is not equal to f (b).
(i) f (x) is continuous in [a, b]
So, Rolle’s thoerem is not applicable here.
(ii) f (x) is differentiable in (a, b)
Here, f (a) is not equal to f (b).
Then there is a number c in (a, b) such that
So, Rolle’s thoerem is not applicable here.

1.4  Algebraical Significance of Rolle’s Theorem f (b) – f (a)


f ¢(c) = __________
​    
    ​.
b–a
Between any two roots of a polynomial, there is atleast one
root of its derivative.

1.5  Applications of Rolle’s Theorem


(i) If y = f (x) satisfies the Rolle’s theorem in [a, b], then
f ¢(x) = 0 for some x in (a, b).
(ii) Let x = a and x = b be the roots of f (x) = 0 and
y = f (x) satisfies the condition of Rolle’s Theorem on
[a, b]
Here, f (a) = f (b) = 0. Hence we can say that between
two roots of f (x) = 0 at least one root of f ¢(x) = 0
would lie.
(iii) If y = f (x) be a polynomial functions of degree n and 3.  Constant Function Theorem
f (x) = 0 has real roots, then all the roots of f ¢(x) = 0
If f is a continuous function in [a, b] and is differentiable
are also real.
in (a, b), with f ¢(x) = 0 for all x in (a, b), then the function
For if f(x) is of degree n, f ¢(x) is of degree (n – 1) f (x) is constant on [a, b].
and a root of f ¢(x) = 0 exists in each of the (n – 1)
intervals between the n roots of f (x) = 0. 4.  Constant Difference Theorem
(iv) If all the roots of f(x) = 0 are real, so also are those Let f and g be two continuous functions on [a, b] and are
of f ¢(x) = 0, f ¢¢(x) = 0, f ¢¢¢(x) = 0, ..., and the roots differentiable on (a, b) such that f ¢(x) = g¢(x) for all x in
of any one of those equations separate those of the (a, b). Then there exists a constant C such that f (x) = g (x)
preceeding equation. This follows from (iii). + C for all x in [a, b].
(v) Not more than two roots of f(x) = 0 can
(a) lie between two consecutive roots of f ¢(x) = 0 5. Applications of L.M.V. Theorem
or (i) If f is continuous on [a, b] and m £ f ¢(x) £ M for all x
(b) be less than the least of these or in (a, b), then m (b – a) £ f (b) – f (a) £ M (b – a).
(c) be greater than the greatest of these. (ii) If f is continuous on [a, b] and ​| f ¢(x) |​ £ M for all x
(vi) If f ¢(x) = 0 has n real roots, then f(x) = 0 can not in (a, b), then |​  f (b)  –  f (a) |​ £ M |​ b – a |​
have more than (n + 1) real roots. (iii) If f is continuous on [a, b] and |f ¢(x)| ≥ m for all x
(vii) If f n (x) is the nth derivative of f (x) and the equation in (a, b), then |​ f (b) – f(a) |​ ≥ m |​ b – a |​
f n (x) = 0 has some imaginary roots, then f (x) = 0 (iv) Rolle’s Theorem is a particular case of L.M.V.
has at least as many imaginary roots. Theorem.
Rolle’s theorem and Lagrange’s mean value theorem  6.3

6.  Cauchy’s Mean Value Theorem


Let f and g be two real functions defined on [a, b] such
that
(i) f and g both are continuous on [a, b]
(ii) f and g both are differentiable on (a, b) and
(iii) g¢(x) does not vanish at any point of (a, b)
then there exists a real number c in (a, b) such

f (b) – f (a) f ¢(c)


that ​ __________
  
    ​ = ____
​   
 ​ 
.
g (b) – g (a) g¢(c)
Example 2.  Let f (x) = 2x3 – 9x2 + 12x + 10
Then f ¢(x) = 6x2 – 18x + 12
7. Generalised Mean Value Theorem
Thus f ¢(x) = 0 has two real roots 1 and 2.
If f, g and h are continuous on [a, b] and differentiable on Now f (1) ◊ f (2) > 0.
(a, b), then there exist a number c in (a, b) such that Thus, f (x) has one real root.
The equation f ¢(x) = 0 has two distinct roots say
(iii)
f ¢ (c ) g ¢ (c ) h ¢ (c ) x1 and x2.
f ( a ) g ( a ) h ( a ) = 0. If f (x1) f (x2) < 0, then the eqution f(x) = 0 has three
f (b ) g (b ) h (b ) distinct real roots.

Y
8. Nature of the cubic polynomial f (x 1)
3 2
Let f (x) = ax + bx + cx + d
Then f ¢(x) = 3 ax2 + 2 bx + c X2
X¢ X
(i) when f ¢(x) = 0 has no real root. O
X1
Then f (x) always increases as x increases and the
f (x 1)
equation f (x) = 0 has one real root.

Example 3.  Let f (x) = x3 – 3x + 1


Then f ¢(x) = 3x2 – 3
Now ¢(x) = 0 has two real roots 1 and –1
Now, f (–1) f (1) = (– 1 + 3 + 1) (1 – 3 + 1)
= – 4 < 0
Thus f (x) = 0 has three real roots.
(iv) The eqution f ¢(x) = 0 has two distinct roots, say
x1 and x2
If f(x2) = 0, then f (x) has two real root.

Example 1.  Let f (x) = x3 – x2 + 4x + 10


Then f ¢(x) = 3x2 – 2x + 4
Therefore, f ¢(x) > 0 since D = 4 – 48 < 0
Hence, f is monotonic and it has one real root.
(ii) When f ¢(x) = 0 has two distinct roots say x1 and x2
(x1 < x2)
If f (x1)  ◊  f (x2) > 0, then the equation
f (x) = 0 has one real root.
6.4  Differential Calculus Booster

(v) The equation f  ¢(x) = 0 has two equal roots, say a. Example 4.  Let f (x) = (x – 3)3
If f (a) = 0, then the equation f (x) = 0 has one rea- Then f ¢(x) = 3 (x – 3)2
peated (treble) root. Thus f ¢(x) = 0 has one repeated root x = 3
Also f (3) = 0.
Thus, f (x) = 0 has one repeated root.

Exercises

12. At what points on the curve


(Problems Based on Fundamentals) y = 12 (x + 1) (x – 2) on [–1, 2], is the tangent
1. Verify Rolle’s theorem for the function parallel to x–axis?.
f (x) = x3 – 3x2 + 2x + 5 on [0, 2].. Algebraic meaning of Rolle’s Theorem
2. Verify Rolle’s theorem for the function 13. Let f (x) = x3 – x2 – x + 1. Prove that there is a root
f (x) = (x – a) m (x – b) n on [a, b], m, n Œ I + of its derivative on (– 1, 1).
14. Prove that the equation x cos x = sin x has a root
3. Verify Rolle’s theorem for the function
between p and 2p.

{ 
x2 + ab
f (x) = log ​ ​ _______ 
x(a + b)
  }
 ​  ​ on [a, b], where 0 < a < b. 15. Prove that the equation x3 + x – 1 = 0 has exactly
one real root.
4. Verify Rolle’s theorem for the function
[  ] p
16. Let f (x) = sin5 x + cos5 x – 1 on, ​ 0, __
​   ​   ​
2
p
f (x) = sin4 x + cos4 x on ​ 0, __ [  ]
​   ​   ​
2
Then prove that the equation f (x) = 0 has two roots

5. Verify Rolle’s theorem for the function [  ]


in ​ 0, __
p
​   ​   ​
2
f (x) = 2 sin x + sin 2 x on [0, p] 17. If 2a + 3b + 6c = 0, then prove that the equation
6. Verify Rolle’s theorem for the function ax2 + bx + c = 0 has at least one root in (0, 1).
p
f (x) = sin x + cos x – 1 on ​ 0, __ [  ]
​   ​   ​
2
18. If a + b + c = 0, then show that the equation
3 ax2 + 2 bx + c = 0 has atleast one root in (0, 1).
7. Verify Rolle’s theorem for the function 19. If 3a + 4b + 6c + 12d = 0, then prove that the

[  ]
equation ax3 + bx2 + cx + d, where a, b, c, d ΠR
p 5p
f (x) = ex (sin x – cos x) on ​ __
​   ​ , ___
​   ​   ​ has atleast one root in (0, 1).
4 4
20. If f(x) = x 2 (1 – x)3, then prove that the equation
8. Suppose that f (x) = x1/2 – x 3/2 on [0, 1]. Find a f ¢(x) = 0 has atleast one root in (0, 1).
number c that satisfies the conditions of Rolle’s
21. If f (x) = (x – 1) (x – 2) (x – 3) (x – 4), then find the
theorem.
number of real roots of f ¢(x) = 0
9. If the value of c prescribed in the Rolle’s theorem
and indicate the intervals in which they would lie.
for the function f (x) = 2x (x – 3) n, n Œ N on [0, 3]
is 3/4, then find the value of n. 22. Find the number of real roots of
10. If the function f (x) = x3 – 6x2 + ax + b is defined (i) x3 – 6x2 + 15x + 3 = 0
on [1, 3] satisfies the hypothesis of Rolle’s theorem, (ii) 4x2 – 21x2 + 18x + 20 = 0
then find the values of a and b. (iii) 3x4 – 8x3 – 6x2 + 24x + 1 = 0
11. If the function f (x) = x3 – px2 + qx is defined on
(iv) x4 – 4x – 2 = 0
[1, 3] satisfies the hypothesis of Rolle’s theorem such
that c = 5/4, then find the values of (p + 4q + 17).
Rolle’s theorem and Lagrange’s mean value theorem  6.5

23. Discuss the applicability of Rolle’s theorem for (ii) f (x) = sin |x| on [– 2p, 2p]
each of the following functions on the indicated (iii) f (x) = |sin x| on [0, 2p]
intervals: (iv) f (x) = log |x| on [0, 5]
(i) f (x) = |x – 2| in [0, 3]. (v) f (x) = |log |x|| on [–1, 1]
(ii) f (x) = 3 + (x – 2)2/3 in [1, 3] 33. Using L.M.V. theorem, prove that


1
(iii) f(x) = sin ​ __(  )
​ x ​  ​ in [–1, 1]
b–a
​ _____
b
b
​ < log ​ __
    (  ) b–a
​ a ​  ​ < ​ _____ ​, 
a   
Ï- 4 x + 5, 0 £ x £ 1 where 0 < a < b.
(iv) f (x) = Ì .
Ó2 x - 3, 1 < x £ 2 34. Using L.M.V. theorem, prove that (b – a) sec2 a <
p
(v) f  (x) = [x] in [– 1, 1], where [,] = G.I.F. tan b – tan a < (b – a) sec2 b, where 0 < a < b < ​ __ ​ 
2
(vi)  (x) = sin |x| in [– 2, 2] 35. Using L.M.V. theorem, prove that
(vii) f (x) = e–|x| in [– 3, 3] b–a b–a
​ ______2   ​ < tan–1 b – tan–1 a < ​ ______2 

| 
   ​, 0 < a < b


1
|
(viii) f (x) = ​ e–|x| – ​ __ ​   ​ in [– 2, 2]
2
1+b 1+a
36. Using L.M.V. theorem, prove that |sin x – sin y| £
|  |
1
(ix) f (x) = ​ __
​ x ​ – 1  ​ in [–1, 1] |x – y|
37. Using L.M.V. theorem, prove that |tan–1 x| £ |x| for
(x) f (x) = sin x + |sin x| in [– p, p] all x in R.
Lagranges Mean Value Theorem 38. Using L.M.V. theorem, prove that |tan x – tan y| ≥
24. Verify Lagranges Mean Value Theorem for the ( p p
|x – y| for all x, y in ​ –  ​ __ ​ , __
2 2 )
​   ​   ​
function f (x) = x3 – x2 – x + 1 on [0, 2].
39. Let f be a differentiable function for all x in R. If
25. Find the value of c for which the function
f (1) = – 2 and f ¢(x) ≥ 2 for all x in [1, 6] such that
f (x) = (x – 1) (x – 2) (x – 3) on [0, 4] is applicable
on L.M.V. theorem the least value of f (6) is m, then find the value of
2(m + 2)3 + 17
26. Find the point on the curve y = 2x2 – 5x + 3 where
the tangent is parallel to the chord joining the points 40. If f is continuous on [0, 2] and differentiable on (0, 2)
A(1, 0) and B(2, 1). such that f (0) = 2, f (2) = 8 and f ¢(x) £ 3 for all x in
(0, 2), then find the value of f (1).
27. Find a point on the parabola y = (x – 4)2, where the
tangent is parallel to the chord joining (4, 0) and
(5, 1). (Mixed Problems)
28. Suppose f (x) is twice differentiable function such that
f (1) = 1, f (2) = 4, f (3) = 9, then prove that there exist at Choose the most appropriate one (s).
least one root in (1, 3) such that f ¢¢(x) = 2, x Œ R 1. The value of c in L.M.V. theorem for the function
29. If f (x) satisfies the property of L.M.V. theorem in f (x) = x2 in the interval [–1, 1] is
1 1
[0, 2] such that f (0) = 0 and f¢(x) £ __ ​   ​  for all x in (a) 0 (b) ​ __ ​ 
2 2
[0, 2], prove that f (x) £ 1. 1
__
(c) – ​   ​   (d) non existent
30. Let f (x) and g(x) be differentiable functions for x Œ [0, 2
1] such that f (0) = 2, g(0) = 0, (1) = 6 Let there 2. The value of c of the L.M.V. theorem for which the
______
exist a real number c in (0, 1) such that f ¢(c) = 2g¢(c),   – x2 
function ​÷25  ​ in [ 0, 5], is
then find the value of g(1).
(a) 5 (b) 3
(  )
p
31. If a, b, c Œ ​ 0, ​ __  ​  ​ and a < c < b, then prove
2


(c) 4 (d) None
3. The value of c, of the L.M.V. theorem for which the

[  ]
cos a – cos b
___________
that ​     ​ = – tan c 1
sin a – sin b function f (x) = x (x – 1) (x – 2) in ​ 0, __
​   ​   ​ is
2
32. Discuss the applicability of L.M.V. theorem for each 1
of the following functions to the indicated intervals (a) ​ __ ​  (b) 0.3
2
1
(i) f (x) = __
​ x ​ on [–1, 1] (c) 0.24 (d) None
6.6  Differential Calculus Booster

4. The equation 3x2 + 4ax + b = 0 has atleast on root 13. Let f be differentiable for all x. If f (1) = – 2 and
in (0, 1) if f ¢(x) ≥ 2 for all x Œ [1, 6] then
(a) 4a + b + 3 = 0 (b) 2a + b + 1 = 0 (a) f (6) < 8 (b) f (6) > 8
4
__ (c) f (6) > 5 (d) None
(c) b = 0, a = – ​   ​  (d) none
3 14. If a, b, c be non-zero real numbers such that
5. If Rolle’s theorem holds for the function 1

​  __  ​  ​Ú ​  ​ (1 + cos x) (ax + bx + c) dx


1 8  2
f (x) = x + bx + ax + 5 on [1, 3] with c = 2 + ___
3 2
   0
÷
​ 3 ​
then (a, b) is/are 2
(a) (3, – 5) (b) (6, – 11) = ​Ú  ​   ​ (1 + cos8 x) (ax2 + bx + c) dx
0
(c) (11, – 6) (d) (11, 6)
then the equation ax2 + bx + c = 0 will have a root
6. The point on the curve y = (cos x – 1) in [0, 2p]
between
where the tangent is parallel to x–axis is
(a) (1, 3) (b) (1, 2)
(a) (0, 0) (b) (p, – 2)
(c) (2, 3) (d) (3, 1)
(c) (2p, 0) (d) (p/2, 2p) 15. The value of |cos a – cos b| is
7. Find out the function in which Rolle’s theorem is (a) < |a – b| (b) > |a – b|
satisfied.
(c) 0 (d) |a + b|.
(a) f (x) = x2 in [1, 2]
16. The length of the longest interval in which Rolle’s
(b) f (x) = x2/3 in [–1, 1] theorem can be applied for the function f(x) = |x2 – a2|
(c) f (x) = |x| in [–1, 1] is (a > 0)
_____
(a) 2a (b) 4a2
(d) f (x) = ÷   – x2 
​ 4    ​ in [– 2, 2]
__
(c) a ​÷2 ​    (d) a
2 
8. Let f (x) = sin x – sin 2 x in [0, p], then a number c 17. The number of roots of the equation sin  + 2 sin 2x
in the Rolle’s theorem is + 3 sin 3x = 0 is the interval [0, p] is
(a) 32°32' (b) 126°23' (a) 1 (b) 2
(c) 48°48' (d) 148°32' (c) 3 (d) More than 3
sinx
____
9. Let f (x) = ​  x     ​ in [0, p], then a number c is the 1
e 18. If ​Ú ​  ​ ex (x – a) dx = 0, then
Rolle’s theorem is 0

p p (a) 1 < a < 2 (b) a < 2


(a) ​ __ ​   (b) ​ __ ​ 
4 3 (c) 0 < a < 1 (d) a = 0
p
__
(c) ​   ​  
6
3p
___
(d) ​   ​ 
4 19. f (x) = x a sin ​ __ (  )1
​ x ​  ​, x π 0, f (0) = 0 satisfies conditions

10. Let f (x) = sin x + cos x + c is [0, p], then a real


number c in the Rolle’s theorem is
of Rolle’s theorem on ​ – ​ __ [  ]
1 __ 1
p  ​, ​ p  ​ ​ for a equals
(a) –1 (b) 0
p p
(a) ​ __ ​   (b) ​ __ ​  (c) 7/2 (d) 5/3
3 4
p 2p 20. If f (x) = ax + b, x Œ [–2, 2], then the point c Œ (– 2, 2)
(c) ​ __ ​   (d) ​ ___ ​  where
6 5
11. Let f (x) = (x – 1) (x – 2)2 in [1, 2], then a real number f ( 2) – f (–2)
c is the Rolle’s throrem is f (c) =   ____________
​   ​    
4
4 3
(a) ​ __ ​   (b) ​ __ ​  (a) does not exist
3 4
(b) can be any c Œ(–2, 2)
5 4
(c) ​ __ ​   (d) ​ __  ​ (c) can be only 1
4 5
12. If 2a + 3b + 6c = 0, then atleast one root of the (d) can be only –1.
equation ax2 + bx + c = 0 lies in the interval 21. Mean value theorem is applicable to f (x) = x + 2x
(a) (0, 1) (b) (1, 2) – p | – 2 |cos x| equation in the interval
(c) (2, 3) (d) None (a) [–2, 2] (b) [0, p]
Rolle’s theorem and Lagrange’s mean value theorem  6.7

[ 
p
(c) ​ – ​ __ ​ ,
2
p
]
​ __ ​   ​
2 [ 
p
(d) ​ – __
​   ​ ,
3
2p
]
​ ___ ​   ​.
3
6. Let f be a continuous in [a, b] and differentiable in
(a, b). If f (a) = a & f (b) = b, then show that there
1
[  1
] [  ]
22. For the function f (x) = ​ x + ​ __ ​   ​ + ​ x – ​ __ ​   ​ – [2x],
2 2

exists distinct c1, c2 Π(a, b) such that

f ¢(c1) + ¢f (c2) = 2
Rolle’s theorem is applicable on the interval

[  ] [  ]
1 1 7. Let f (x) be a twice differentiable function such that
(a) ​ 0, __
​   ​   ​ (b) ​ __
​   ​ , 1  ​ f ¢¢ (x) < 0 in [0, 2]. Then prove that
4 4

(c) ​[ 0, __
​    ​ ]​
1 (i) f (0) + f (2) = 2f (c), c Œ (0, 2)
(d) [0, 1]
2
(ii) f (0) + f  (2) < f (1)
23. Rolle’s theorem is applicable for the function
f(x) = (x – 1) |x| + |x – 1| in the interval 8. Let f (x) is continuous in [0, 2] and differentiable in

(a) [0, 1] [  ]
1 3
(b) ​ __
​   ​ , __
4 4
​    ​  ​
(0, 2). If f (0) = 2, f (2) = 8 and f ¢(x) £ 3 for all x in
(0, 2), find the value of f (1).

[  ] (d) ​[ __ [  )
5 7]
1 1 1 6 p
(c) ​ – ​ __ ​ , __
​    ​  ​ ​   ​ , __
​    ​  ​ ​ ​ __ ​   ​ ​, prove that
9. If a, b Œ ​  0,
2 2 ​ 2 ​
|cos a – cos b| £ |a – b|
p
24. If a, b, c Œ ​ 0, __ (  ) cos a – cos b
​   ​   ​ and a < c < b, then ​ ___________
2
   ​
sin a – sin b 10. Find the value of k for which the equation
may equal to which of the following x3 – 3x + k = 0 has two distinct roots in (0, 1).
(a) tan c (b) cot c 11. Find the number of distinct real roots of
(c) – tan c (d) – cot c x4 – 4x3 + 12x2 + x – 1 = 0.
12. Find the number of real roots of
  (Problems for JEE-Advanced)
x3 – 6x2 + 15x + 3 = 0
1. Let f : [0, 4] Æ R be a differentiable function for some 13. Find the number of real roots of
a, b Œ(0, 4), show that
4x3 – 21x2 + 18x + 20 = 0.
2 2
f  (4) – f  (0) = 8 f (a) f ¢(b) 14 Find the number of real roots of
2. Let f : [2, 7] Æ [0, •) be a continuous and differen- 3x4 – 8x3 – 6x2 + 24x + 1 = 0
tiable function, show that
15. Find the number of real roots of
{(f (7))2 + (f (7))2 + f (2) ◊ f (7)} x4 – 4x – 2 = 0
(f (7) – f (2)) ​ __________________________
   
 ​   
3 16. Find the value of k for which the equation
2
= 5 f  (c) ◊ f ¢(c), for all c Œ (2, 7) x4 – 14x2 + 24x – k = 0 has four real and unequal
roots.
3. If f and g are differentiable on [0, 1],
f (0) = 2, g (0) = 0, f (1) = 6 and g (1) = 2, " c Œ (0, 1), 17. Find the value of k for which the equation
then show that x3 – 3x + k = 0 has two distinct roots in (0, 1).
f ¢(c) = 2g¢(c) 18. Let x3 – 2kx2 – 4kx + k2 = 0. If one root is less
than 1 and the other is in (1, 4) and the third root
4. Let f be differentiable for all x. If f(1) = – 2 and is greater than 4 such that
f ¢(x) ≥ 2 for all x Œ [1, 6] then show that __ __
k Œ (m + ​÷n     ​, n (m + ​÷n 
   ​)), then find the value of

f (6) ≥ 8 2 (m + n – 1)3 + 17.


19. Find the value of ‘a’ for which the equation
5. If f (x) and g (x) are continuous in [a, b] and differ-
x3 – 3x + a = 0 would have two distinct roots in
entiable in (a, b), show that there exists some
(– 1, 1).
c Π(a, b)
20. Find the value of k such that Rolle’s theorem is
f ( a ) f (b ) f ( a ) f ¢ (c ) applicable to the function
such that = (b - a )
g ( a ) g (b ) g ( a ) g ¢ (c ) f (x) = x3 – 2x2 – 4x + 7 on the interval [0, k]
6.8  Differential Calculus Booster

Comprehensive Link Passages


(Tougher Problems for JEE-Advanced)
Passage I
1. If the equation x3 – 3x + 1 = 0 has three real roots Let f (x) be a real valued function and a, b Œ R. If f is con-
a, b, g such that a < b < g, then find the value of tinuous in [a, b] and differentiable in (a, b), then there exist
{a} + {b} + {g}, where {,} = F.P.F atleast one real number c in (a, b) such that
2. Find the smallest natural number c for which the
f (b) – f (a)
equation ex = c x2 has exactly three real and distinct f ¢(c) = __________
​       ​
solutions. b–a
On the basis of the above statement, answers the follow-
3. Let f(x) = x4 + 8x3 + 18x2 + 8x + b
ing questions.
If the equation f (x) = 0 has four real and distinct
1. The point on the curve y = x3 such that the tangent
roots, then find b.
at that point is parallel to the chord joining the point
4. Let f : [0, 4] Æ R be a differentiable function A(1, 1) and B(4, 64) is
(i) for some a, b Π(0, 4), show that, __ __
(a) (7, ÷   ) 
​ 7 ​   ) 
(b) (5, 5​÷5 ​
__
f  2 (4) – f 2 (0) = 8f (a) f (b) (c) (7, 7 ÷   ) 
​ 7 ​ (d) None
(ii) Show that x
2. If log (1 + x) < x < e for all x > 0, then which one
4 is most appropriate?
​Ú ​    ​f (t) dt = 2 {a f (a 2) + b f (b2)}
0 (  )
ex – 1
(a) 0 < log ​ ​ _____
x   
​  ​ < x

(b) x < log ​( ​ _____ ​  )​ < e


for some 0 < a, b < 2 e –1 x
x
x   
5. Prove that 2a 02 < 15a then all roots of x5 – a 0 x4 +
x + 1 _________1 1
3ax3 + bx2 + cx + = 0 can not be real. It is given (c) ​ _____
x   ​ < ​       ​ < __
​   ​
log (1 + x) x
that a 0, a, b, c, d Œ R.
(d) All of the above.
6. For all x Œ [0, 1], f ¢¢(x) exists and such that
|f ¢¢(x)| < 1. If f (0) = f (1), then show that |f ¢(x)| < 1 3. If f and g are differentiable on [0, 1], f (0) = 2,
for all x in [0, 1]. g(0) = 0, f (1) = 6, g(1) = 2 for all c in (0, 1), then
7. Using L.M.V. theorem, prove that (a) f ¢(c) = g¢(c)
x (b) f ¢( ) = 2g¢(c)
_____
​      ​ < log(1 + x) < x, " x > 0
1+x (c) f ¢(c) + g¢(c) = 0
(d) 2f ¢(c) – g¢(c) = 0.72.
Integer Type Questions
Passage II
1. Let f (x) = x1/2 – x3/2 on [0, 1] such that a number c
Between any two roots of a polynomial f (x), there is always
satisfies the hypothesis of Rolle’s theorem, then the
a root of its derivative.
value of 3c + 4 is
On the basis of the above information answer the
2. If the equation ax2 + bx + c = 0 has at-leas one root
following questions.
in (0, 1) such that 2a + 3b + 6c = m where m ΠW,
then find the value of (m + 3) 1. If a, b, c in R such that the equation 3 ax2 + 2 bx +
c = 0 has atleast one root in (0, 1), then a + b + c
3. If m is the number of real roots of x4 – 4x – 2 = 0
is
and n is the number of real solutions of 2x = x2 + 1,
then find the value of (m + n). (a) 1 (b) 0
4. If p is the number of real roots of x4 – 14x2 + 24x (c) 2 (d) –1
+ k = 0 and q is the number of non differentiable 2. If x4 – 14x2 + 24x – k = 0 has four real and unequal
points of f (x) = |x| + |x 2 – 1|, then the value of roots, then the value of k lies in
(p + q + 2) is (a) (8, 11) (b) (2, 5)
5. Let f be differentiable for all x. If f (1) = – 2 and (c) (0, 2) (d) (1, 3)
f ¢(x) ≥ 2 for all x in [1, 4], then the least value of 3. The number of real roots of 4x3 – 21x2 + 18x + 20
f (4). = 0 is
6. Let f (x) = 2x (x – 3) n, n Œ N on [0, 3]. If f (x) satisfies (a) 1 (b) 2
the Rolle’s theorem for which c is 3/4, then find the
(c) 3 (d) 4
value of n.
Rolle’s theorem and Lagrange’s mean value theorem  6.9

4. The number of real roots x4 – 4x – 1 = 0 is


p
__
(a) 1 (b) 2 (D) f (x) = sin2 x, [0, p] (S) ​   ​ 
4
(c) 3 (d) 4
5. The number of real roots of ex = x + 1 is 2. Match the following columns:
(a) 1 (b) 0 If the function f (x) = x3 – px2 + qx is defined on [1, 3]
satisfies the Rolle’s theorem with c = 5/2, then
(c) 2 (d) None
6. The number of real roots of 2x = x2 + 1 is Column I Column II
(a) 1 (b) 2 (A) 4p – q (P) 3
(c) 0 (d) None
Passage III (B) 4p – 2q (Q) 13
Let f (x) = x3 – 3x + k
1. The number of real roots of f (x) = 0 is (C) p (R) 23/4
(a) 2 (b) 1
(c) 3 (d) 0
(D) q (S) 10
2. If f (x) = 0 has three distinct real root then k lies
in
3. Match the following columns:
(a) (–1, 1) (b) (–2, 2)
Ï
(c) (1, 2) (d) (2, 3) Ôax 2 + b : | x | < 1
3. The number of real roots of f (x) = f (x + 1) is ÔÔ
Let f (x) = Ì1 : | x| = 1
(a) 1 (b) 2
Ôc
(c) 3 (d) 0 Ô : | x| > 1
ÔÓ | x |
Passage IV
If Rolle’s theorem is applicable for the function f in
Let f (x) = x4 – 14x2 + 24x – k [– 3, 3], then
1. The number of real roots of f (x) = 0 is
(a) 3 (b) 2 Column I Column II
(c) 4 (d) 1 (A) |a| + |b| + |c| (P) 8
2. If f (x) has four real and unequal roots, then k lies
in (B) 2a + 3b + 4c (Q) 3
(a) (5, 8) (b) (8, 11)
(c) (11, 13) (d) (13, 19)
3. The number of imaginary roots of f (x) = 0 is (C) a+b (R) 1
(a) 2 (b) 1
(c) 0 (d) 3 (D) c (S) 2

Matrix Match
(For JEE-Advanced Exam Only) 4. Match the following columns:
Let f (x) = x3 + bx2 + ax, " x Œ [1, 3] If f (x) satisfies
1. Match the following columns: Rolle’s theorem such that
If each of the follwing functions are satisfied by
1
Rolle’s theorem, then the value of c is c = 2 + ___
​  __  ​,  then
÷   
​ 3 ​
Column I Column II
Column I Column II
(A) x
f (x) = e  sin x, [0, p] (P) p
__
​   ​  (A) the value of a is (P) 17
3
(B) f (x) = x + sin x, [0, p] (Q) p (B) the value of b is (Q) 5

) [  ]
the value of a + b is
(  p p p (C) (R) – 6
(C) f (x) = cos ​ 2x – __
​   ​   ,​ ​ 0, __
​   ​   ​ (R) __
​   ​ 
2 2 2 (D) the value of a – b is (S) 11
6.10  Differential Calculus Booster

5. Match the following columns: ÏÊ 1 ˆ 1


ÔÁË 2 - x ˜¯ : x < 2
Column I Column II Ô
n
(a) f (x) = Ì 2
(A) Let f (x) = 2x (x – 3) on [0, 3] (P) 3 ÔÊ 1 - x ˆ : x ≥ 1
If f (x) satisfies Rolle’s theorem on ÔÁË 2 ˜¯ 2
Ó
[0, 3] for which c = 3/4, then the
value of n is Ï sin x
3/2 3/2 Ô : xπ0
If f (x) = x – x on [0, 1] (b) f (x) = Ì x
(B) satisfies Rolle’s theorem, then the (Q) 1/3 ÔÓ1 : x=0
value of ‘c’ is
(c) f (x) = x |x|
If f (x) = x4 – 2x2 + 1 on [– 2, 2]
(C) satisfies Rolle’s theorem, then the (R) – 2 (d) f (x) = |x|
value of ‘c’ is
[IIT-JEE, 2003]
If f (x) = x a log and f (0) = 0, then
a 
the value of a for which Rolle’s 3. If f (x) = x log x and f (0) = 0, then the value of a for
(D) (S) – 1 which Rolle’s theorem can be applied to [0, 1] is
theorem can be applied in [0, 1]
is (a) – 2 (b) – 1
(c) 0 (d) 1/2.
6. Match the following columns:
[IIT-JEE, 2004]
Column I Column II 4. Using Rolle’s theorem, prove that there is at least one
(A) The number of real roots of
4x3 – 21x2 + 18x + 20 = 0 is
(P) 4
(  1
____
​     ​ 
)
root in ​ 4​5​100 ​, 46  ​ of the polynomial
101
P (x) = 51x – 2323x100 – 45x + 1035.
The number of real roots of
(B) 4 (Q) 3 [IIT-JEE, 2004]
x – 4x – 1 = 0 is
5. If f (x) is a twice differentiable such that
The number of real roots of
f  (a) = 0, f (b) = 2, f (c) = – 1,
(C) 3x4 – 8x3 – 6x2 + 24x + 1 = 0 (R) 2
is f (d) = 2, f(e) = 0 where a < b < c < d < e
The number of real roots of then find the minimum number of zeroes
(D) (S) 1
x4 – 4x3 + 12x2 + x – 1 = 0 is of g (x) = (f ¢(x))2 + g¢(x) f ¢¢(x) in the interval [a.e]

[IIT-JEE, 2006]
Qestions asked in past IIT-JEE Exams 6. The number of distinct real roots of

1. If a + b + c = 0, then the equation x4 – 4x3 + 12x2 + x – 1 = 0 is ...

3 ax2 + 2 bx + c = 0 has [IIT-JEE, 2011]


x

7. If f (x) = Ú​  ​  ​ ​et​ ​(t – 2) (t – 3) for all x > 0 then


2
(a) atleast one root in [0, 1]
0
(b) one root in [2, 3] and the other in [– 2, – 1]
(a) f has a local maximum at x = 2
(c) imaginary roots.
(b) f is decreasing on (2, 3)
(d) None [IIT-JEE, 1983]
(c) there exist some c Œ (0, •) such that f ¢¢(c) = 0
2. In [0, 1] Lagranges Mean Value Theorem is not (d) f has a local minimum at x = 3.
applicable to [IIT-JEE, 2012]

Answers
Level-II 11. (a) 12. (a) 13. (b) 14. (b) 15. (a)
1. (d) 2. (d) 3. (c) 4. (b) 5. (d) 16. (a) 17. (d) 18. (c) 19. (d) 20. (b)
6. (a, b, c) 7. (d) 8. (a, b) 9. (a) 10. (b) 21. (b, c, d) 22. (a) 23. (a, b, d) 24. (c)
Rolle’s theorem and Lagrange’s mean value theorem  6.11

 Integer type questions Matrix Match


1. (5) 2. (3) 3. (4) 4. (9) 5. (4) 1. (A)Æ(R), (B)Æ(S), (C)Æ(P), (D)Æ(Q)
6. (3)
2. (A)Æ(Q), (B)Æ(P), (C)Æ(R), (D)Æ(S)
Comprehensive Link Passages 3. (A)Æ(Q), (B)Æ(P), (C)Æ(R), (D)Æ(R)
Passage I : 1. (c) 2. (a) 3. (b) 4. (A)Æ(S), (B)Æ(R), (C)Æ(Q), (D)Æ(P)
Passage II : 1. (b) 2. (a) 3. (c) 5. (A)Æ(P), (B)Æ(Q), (C)Æ(S), (D)Æ(S)
4. (b) 5. (a) 6. (b) 6. (A)Æ(Q), (B)Æ(R), (C)Æ(R), (D)Æ(R)
Passage III : 1. (c) 2. (a) 3. (b)
Passage IV : 1. (c) 2. (b) 3. (c)

Hints and solutions


fi (m(c – b) + n (c – a)) = 0
(Problems based on Fundaments) mb + na
fi c = ​ _______  
​ Œ (a, b)

m+n
1. Given f(x) = x3 – 3x2 + 2x + 5 Hence, Rolle’s theorem is verified.
As we know that, every polynomial function is
continuous as well as differentiable.
So, f (x) is continuous and differentiable on the
3. Given { 
x2 + ab
f (x) = log ​ ​ _______ 
x(a + b)

 ​  ​ }
As we know that every logarithmic function is
indicated interval. continuous and differentiable in Positive real
Also, f (0) = 5 and f (2) = 8 – 12 + 4 + 5 = 5 numbers
i.e. f (0) = 5 = f (2) So it is continuous in [a, b] and differentiable in
Thus, all the conditions of Rolle’s theorem are (a, b)
satisfied.
Now, we have to show that there exist a point c in
(0, 2) such that f ¢(c) = 0
a2 + ab
( 
Also, f(a) = log ​ ​ _______ 
a(a + b)
  )
 ​  ​ = log 1 = 0

We have f ¢(c) = 3c2 – 6c + 2 = 0 = 0 gives


We have f ¢( ) = 3c2 – 6c + 2 = 0 gives
and (  b2 + ab
f (b) = log ​ ​ _______ 
 b(a + b)
  )
 ​  ​ = log 1 = 0

_______ __
6 ± ​÷36  – 24 ​ ± 2​÷3 ​
  6_______   
Thus, all the conditions of Rolle’s theorem are
1
fi c = ​ ____________  ​
     = ​   ​   = 1 ± ___
​  __  ​  satisfied.
6 6 ÷   
​ 3 ​
1 Now we have to show that there exist a point c in
fi c = 1 ± ___
​  __  ​ Œ (0, 2)
÷   
​ 3 ​ (a, b) such that f ¢(c) = 0

(  )
Hence, Rolle’s theorem is verified.
x2 + ab
m  n We have f(x) = log ​ ​  _______ 
  ​  ​
2. Given f (x) = (x – a) (x – b) x(a + b)
As we know that every polynomial function is 2
= log (x + ab) – log (x (a + b))
continuous and differentiable everywhere.
2x 1
So, f (x) is continuous and differentiable on the given fi f ¢(x) = ______  ​ – __
​  2     ​ x ​
indicated interval. x + ab
Also, f (a) = 0 = f (b). 2 2 – x2 – ab x2 – ab
fi f ¢(x) = ​ ____________
  
    ​ = ​  _________   ​
Thus, all the conditions of Rolle’s theorem are x(x2 + ab) x (x2 + ab)
satisfied.
Now, f ¢(c) = 0 gives c2 – ab = 0
Now, we have to show that there exist a point c in ___
(a, b) such that f ¢(c) = 0 fi c=÷    ​ Œ (a, b)
​ ab 
So, Hence, Rolle’s theorem is verified.
f ¢(x) = m (x – a) m–1 (x – b) n + n (x – a) m (x – b) n–1
4. Given f (x) = sin4 x + cos4 x
m–1 n–1
f ¢(x) = (x – a)  (x – b) (m(x – b) + n(x – a)) As we know that every sine and co-sine functions is
Now, f ¢(c) gives c = a, c = b and continuous and differentiable everywhere.
6.12  Differential Calculus Booster

[  ]
So, it is continuous on ​ 0, __
p
​   ​   ​ and
2
p
is differentiable. So, f ( ) is continuous in ​ 0, ​ __ ​   ​
2 [  ]
(  ) p
differentiable on ​ 0, ​ __ ​   ​
2
p
__
and differentiable in ​ 0, ​   ​   ​
2 (  )
Also, (  ) p
f (0) = 1 = f ​ __
​   ​   ​
f (0) = 0 + 1 – 1 = 0

(  )
2 p
f  ​ __
​   ​   ​ = 1 + 0 – 1 = 0
Thus, all the conditions of Rolle’s theorem are 2

(  )
satisfied. p
So, f (0) = 0 = f  ​ __ ​   ​   ​
Now, we have to show that there exist a point c in​ 2

(  )
p
0, __
​   ​   ​ such that f ¢(c) = 0
2
Thus, all the conditions of Rolle’s theorem are
satisfied.
Therefore, Now, we have to show that, there exists a point


f ¢(x) = 4 sin3 x cos x – 4 cos3 x sinx
= 4 sin x cos x (sin2 x – cos2 x)
c Œ ​ 0, __
p
(  )
​   ​   ​ such that f ¢(c) = 0.
2
fi cos c – sin c = 0
= – 2. (2 sin x cos x) (cos2 x – sin2 x)
fi cos c = sin c
= – 2 . sin 2x ◊ cos 2 x
fi tan c = 1
= – sin 4x
Now, f ¢(c) = 0 gives sin (4c) = 0 fi
p
4 (  )
p
c = ​ __ ​  Œ ​ 0, __
​   ​   ​
2
fi sin (4c) = sin (p) Thus, the Rolle’s theorem is verified.

(  )
p p 7. As we know that, every exponential, sine and cosine
fi c = __
​   ​  Œ ​ 0, __
​   ​   ​. functions are differentiable everywhere.
4 2
5. Clearly, f (x) is continuous in [0, p] and differentiable
in (0, p)
p 5p
So, it is continuous in ​ __
​   ​ , ___
4 4 [  ]
​   ​   ​

and differentiable in (​  __


4 4 )
f (0) = 0, f (p) = 0 p 5p
​   ​ , ___
​   ​   ​
fi f (0) = 0 = f (p)
​   ​  )​ = e​ ​ ​ ​( ___
f  ​( __
​ 2 ​ ​ 2 ​)
p
p __
1
​   ​  1
Thus, all the conditions of Rolle’s theorem are Now, ​    ​ – ___
4 ​    ​  ​ = 0
__ __
satisfied. 4 ÷    ÷   

​   ​  )​ = ​e​ ​ ​( – ___


f  ​( ___
Now, we have to show that, there exists a point
​    ​  )​ = 0
5p
5p ​ ___ ​ 
1 1
c Œ (0, p) such that f ¢(c) = 0. and 4​    ​ + ___ __ __
4 ​ 2 ​ ​ 2 ​ ÷    ÷   

​   ​  )​ = 0 = f ​( ___
Now, f ¢(c) = 0 gives
f  ​( __ ​   ​  )​
p 5p
So,
fi 2 cos c + 2 cos 2c = 0 4 4

fi cos c + cos 2c = 0 Thus, all the conditions of Rolle’s theorem are


satisfied.
fi cos c + 2 cos2 c – 1 = 0
Now, we have to show that, there exists a point
fi 2 cos2 c + cos c – 1 = 0
fi 2 cos2 c + 2 cos c – 1 = 0
( 
p 5p
c Œ ​ __
​   ​ , ___
4 4 )
​   ​   ​ such that f ¢(c) = 0.

fi 2 cos c (cos c + 1) – (cos c + 1) = 0 fi e (sin c – cos c) + ec (cos c + sin c) = 0


c 

fi (2 cos c – 1) (cos c + 1) = 0 fi (sin c – cos c) + (cos c + sin c) = 0


1
fi cos c = ​ __ ​ , – 1 fi 2 sin c = 0
2
p fi sin c = 0
fi c = __
​   ​ , p

p
3

p
c = p Œ ​ __ ( 
​   ​ ,
4
5p
___
)
​   ​   ​
4
fi c = ​ __ ​  Œ [0, p]
3 Thus, all the conditions of Rolle’s theorem are
Thus, the Rolle’s theorem is verified. verified.
6. As we know that, every sine and cosine function 8. Given f (x) = x1/2 – x3/2 on [0, 1]
Rolle’s theorem and Lagrange’s mean value theorem  6.13

Since f (x) satisfies all the conditions of Rolle’s 68


= ___
​   ​ + 17 = 34
theorem, so there exists a point c Π(0, 1) such 4
that 12. Given curve is y = 12(x + 1) (x – 2)
f ¢(c) = 0 Since the tangent is parallel to x–axis, so
1 –1/2 3 1/2 dy
fi ​ __ ​​  c​ ​ – __ ​   ​​  c​ ​ = 0 so, ​ ___  ​ = 0
2 2 dx
1 fi
fi ​ ___
1/2
   ​ – 3​c1/2
​ ​= 0 12{(x + 1) + (x – 2)} = 0
​c​ ​
fi 2x – 1 = 0
fi 1 – 3c = 0
1
1 fi x = __
​   ​ 
fi c = __​   ​  Œ (0, 1) 2

9. Given
3
f (x) = 2x (x – 3) n
when x = 1/2, then y = 12 ​ 1 + __ ( 
1 1
) (  )
​   ​   ​ ​ __
2 2
​   ​  – 2  ​

fi f ¢(x) = 2 (x – 3) n + 2nx (x – 3) n–1 fi


3
y = 12 × __
​   ​  × – __
3
​   ​  = – 27
2 2

( 
Now, f ¢(c) = 0 gives
fi 2(c – 3) n + 2nc (c – 3) n–1 = 0
1
__
Hence, the point is ​ ​   ​ , – 27  ​.
2 )
fi 2(c – 3) n = –2 nc (c – 3) n–1 13. Given f (x) = x3 – x2 – x + 1
fi (c – 3) n = –nc (c – 3) n–1 fi f ¢(x) = 3x2 – 2x – 1
fi (c – 3) = –nc
f (x) = 0 gives 3x2 – 2x – 1 = 0
fi c (1 + n) = 3
3 3 fi 3x2 – 3x + x – 1 = 0
fi (1 + n) = __​ c ​ = ___
​     ​ = 4
3/4 fi 3x (x – 1) + 1 (x – 1) = 0
fi n=3
Hence, the value of n is 3. fi (3x + 1) (x – 1) = 0
10. Given f (x) = x3 – 6x2 + ax + b 1
fi x = 1, – ​ __ ​ 
3
f (1) = f (3)
1
__
fi 1 – 6 + a + b = 27 – 54 + 3a + b fi x = – ​   ​  Œ (–1, 1)
3
fi a + b – 5 = 3a + b – 27
Thus, the derivative of the function has a root in (–1, 1).
fi 2a = 22
fi a = 11, b Œ R 14. Here, we shall show that the equation x cos x = sin x
has a root in p and 2p.
11. Given f (x) = x3 – px2 + qx
sin x
Let f (x) = ​ ____ ​ 
x   
f (1) = f (3)
fi 1 – p + q = 27 – 9p + q Clearly f  (x) = 0, at x = p, 2p
x cos x – sin x
fi 1 – p = 27 – 9p Now, f ¢(x) = ____________
​   ​
    
x2
fi 8p = 26
So, by Rolle’s theorem, f ¢(x) must vanish for some
13
fi p = ___
​   ​  value of x in(p, 2p).
4
Hence, the equation x cos x – sin x = 0 has a root in
Also, f ¢(c) = 0 (p, 2p).
fi 3c2 – 2 pc + q = 0. 15. Let f (x) = x3 + x – 1

fi (  )
5 2
3​​ __ (  )
​   ​   ​​ ​ – 2p ​ __
4
5
​   ​   ​ + q = 0
4
Now,

f (0) = 0 + 0 – 1 = –1 < 0
f (1) = 1 + 1 – 1 = 1 > 0
fi q = 2 p ​ __(  ) (  )
5
​   ​   ​ – 3 ​​ __
4
5 2
​   ​   ​​ ​
4
So, f(x) has a root in between 0 and 1.
16. Given f (x) = sin5 x + cos5 x – 1
fi (  )
q = 2 ​ ___
13 5
​   ​  × ​ __ ​   ​ – ___
4 4 16
75 130 – 75 ___
​   ​ = ​ ________
16
 ​  
55
= ​   ​ 
16 f ¢(x) = 5 sin4 xcos x – 5 cos4x sin x

Hence, the value of (p + 4q + 17) = 5sin x cos x (sin3 x – cos3 x)


13 55 Now, f ¢(x) = 0 gives
= ___
​   ​ + ___
​   ​ + 17
4 4 fi sin x cos x (sin3 x – cos3 x) = 0
6.14  Differential Calculus Booster

fi tan3 x = 1
Hence, f ¢(x) = 0 has three real roots.
p
x = ​ __ ​ 
___
fi 15 ± ​÷15 ​
   
4 Now, f ¢¢(x) = 0 gives x = ​ ________
 ​  
6
Since f ¢(x) has a real root, so f (x) must have two

[  ]
p
roots in ​ 0, __
​   ​   ​
2
ax3 bx2
17. Let f (x) = ​ ___ ​  + ​ ___ ​  + cx
3 2
Now f (0) = 0
a b 2a + 3b + 6c __ 0
and f (1) = ​ __ ​  + ​ __ ​  + c = ​ ____________
 ​
     = ​    ​ = 0 Thus, f ¢(x) = 0 has three real roots
3 2 6 6

(  ) ( 
___ ___ ___
2
Thus, ax + bx + c = 0 has at-least one root in (0, 1).
3 2
6
    15
15 – ​÷15 ​
​ – • ​ ________
 ​  
6
    15
– ​÷15 ​
 ​, ​ ​ ________
 ​ 
   
+ ​÷15 ​
, ​ ________
6
 ​   ​ )
18. Let f (x) = ax + bx + cx
( 
___

Now, f (0) = 0 6
   
15 + ​÷15 ​
and ​ ​ ________
 ​ , •  ​ )
and f (1) = a + b + c = 0.
22.
Thus, by Rolle’s Theorem, there is atleast one root
in its derivative. (i) Let f (x) = x3 – 6x2 + 15x + 3
ax4 bx3 cx2 f ¢(x) = 3x2 – 12x + 15
19. Let f (x) = ​ ___ ​  + ​ ___ ​  + ​ ___ ​  + dx
4 3 2
= 3 (x2 – 4x + 5)
Now, f (0) = 0
Now, f ¢ (x) = 0 has no real roots
a b __ c
and f (1) = ​ __ ​  + __
​   ​  + ​    ​ + d So, f (x) = 0 has only one real root.
4 3 2
(ii) Let f (x) = 4x3 – 21x2 + 18x + 20
3a + 4b + 6c + 12d fi f ¢(x) = 12x2 – 42x + 18
= ​ _________________
 ​      = 0
12
fi f ¢(x) = 6 (2x2 – 7x + 3)
So, by Rolle’s theorem, between any two roots fi f ¢(x) = 6 (2x – 1) (x – 3)
of a polynomial, there is atleast one root of its
derivative. Clearly, f ¢(x) = 0 has two real roots.
Hence, the result. Thus, f (x) = 0 has three real roots.
2 3
20. Given f (x) = x (1 – x) (iii) Let f (x) = 3x4 – 8x3 – 6x2 + 24x + 1
Clearly, f (0) = 0 = f (1) f ¢ (x) = 12x3 – 24x2 – 12x + 24
Thus, f (x) has two roots. = 12(x3 – 2x2 – x + 2)
So, by Rolle’s theorem, between any two roots
of a polynomial, there is atleast one root of its = 12{x2 (x – 2) –1 (x – 2)}
derivative. = 12 (x – 2) (x2 – 2)
Hence, the result.
Clearly, f ¢(x) = 0 has three real roots.
21. Given f (x) = (x – 1) (x – 2) (x – 3) (x – 4)
So, f (x) = 0 has 4 real roots.
f ¢(x) = (x – 1) (x – 2) (x – 3) + (x – 1)
(x – 3) (x – 4) + (x – 1) (x – 2) (x – 4) (iv) Let f (x) = x4 – 4x – 2
+ (x – 2) (x – 3) (x – 4) f ¢(x) = 4x3 – 4
3 2 3 2
= (x – 6x + 11x – 6) + (x – 8x + 19x – 12) = 4 (x3 – 1)
3 2 3 2
+ (x – 7x + 14x – 8) + (x – 9x + 26x – 24) = 4 (x – 1) (x2 + x + 1)
3 2
= 4x – 30x + 70x – 50 So, it has only one real root
2
f ¢¢(x) = 12x – 60x + 70 Thus, f (x) = 0 has 2 real roots.
Clearly, its D = 3600 – 3360 = 240 > 0 23.
So, f ¢¢(x) has two real roots. (i) f (x) = |x – 2| in [0, 3].
Rolle’s theorem and Lagrange’s mean value theorem  6.15

Clearly, f (x) is not differentiable at x = 2 fi 3c2 – 2c – 2 = 0


So Rolle’s theorem is not applicable. ______ __
2±÷ 2 ± 2​÷7 ​
​ 4  + 24 ​  _______   
2/3 fi c = ​ ___________
 ​     = ​   ​   
(ii) f (x) = 3 + (x – 2) in [1, 3] 6 6
__
Clearly, f (x) is not differentiable at x = 2 So 1±÷ ​ 7 ​
  
fi c = ​ ______
 ​   
Rolle’s theorem is not applicable. 3
__

1
(iii) f (x) = sin ​ ​ __ (  )
x ​  ​ in [– 1, 1] fi
1+÷
c = ​ _______
3
  
​ 7 ​
 ​   Œ (0, 2)
Clearly, f (x) is not continuous at x = 0 So Rolle’s Hence, L.M.V. theorem is verified.
theorem is not applicable. 25. Given f (x) = (x – 1) (x – 2) (x – 3) (x – 4)
Ï- 4 x + 5, 0 £ x £ 1 As we know that every polynomial function is
(iv) f (x) = Ì .
Ó2 x - 3, 1 < x £ 2 continuous and differentiable everywhere.
So, it is continuous in [0, 4] and differentiable in
Clearly, f (x) is not continuous at x = 1 So Rolle’s
(0, 4)
theorem is not applicable.
Thus, all the conditions of L.M.V. theorem are
(v) f (x) = [x] in [– 1, 1], where [,] = G.I.F.
satisfied.
Clearly, f (x) is not continuous at x = –1, 0, 1
So Rolle’s theorem is not applicable. Now we have to show that there exists a point
c Π(0, 4) such that
(vi) f (x) = sin |x| in [– 2, 2]
f (4) – f (0) 24 –0
Clearly, f (x) is not differentiable at x = 0 f ¢(c) = __________
​     = ​ ______
 ​   ​  = 6
So Rolle’s theorem is not applicable. 4–0 4
(vii) f (x) = e–|x| in [– 3, 3] fi 4c3 – 30c2 + 70c – 50 = – 6
Clearly, f (x) is not differentiable at x = 0
fi 4c3 – 30c2 + 70c – 44 = 0
So Rolle’s theorem is not applicable.

| 
fi 2 (c – 1) (2c2 – 13c + 11) = 0

1
2 |
(viii) f (x) = ​ e–|x| – ​ __ ​   ​ in [– 2, 2]
fi 2 (c – 1)2 (2c – 11) = 0
Clearly, f (x) is not differentiable at x = 0 11
fi c = 1, ___
​   ​ 
So Rolle’s theorem is not applicable. 2


1
|  |
(ix) f (x) = ​ __
​ x ​ – 1  ​ in [– 1, 1]
fi c = 1 Π(0, 4)
Hence, Lagranges Mean Value Theorem is verified.
Clearly, f (x) is not continuous at x = 0
26. Given curve is y = 2x2 – 5x + 3
So Rolle’s theorem is not applicable.
The arc AB is continuous in [1, 2] and differentiable
(x) f (x) = sin x + |sin x| in [–p, p] in (1, 2)
Clearly, f (x) is not differentiable at x = 0 So, by L.M.V. Theorem,
So Rolle’s theorem is not applicable. f (2)f (1)
f ¢(x) = _______
​   ​ 

= f (2) – f (1)
Lagranges Mean Value Theorem 2–1
24. Given f (x) = x3 – x2 – x + 1 fi 4x – 5 = 1 – 0
As we know that, every polynomial function is con- fi 4x = 6
tinuous and differentiable everywhere. 3
So, it is continuous in [0, 2] and differentiable in (0, 2) fi x = __
​   ​ 
2
Thus, all the conditions of L.M.V. theorem are 3
satisfied when x = __
​   ​ , then y = 0
2
Now, we have to show that, there exists a point c in
f (2) – f (0)
(0, 2) such that f ¢(c) = __________
​     ​ 
Thus, the point is ​ __
3
2 (  )
​   ​ , 0  ​

2 – 0) 27. Since the tangent is parallel to the chord joining


f (2) – f (0) 3_____
–1 (4, 0) and (5, 1), so we get,
fi f ¢(c) = __________
​     ​ 
= ​   ​ 
 = 1
2–0 2 1–0
2 (x – 4) = ​ _____ 
 ​ = 1
fi 3c2 – 2c – 1 = 1 5–4
6.16  Differential Calculus Booster
1
(x – 4) = ​ __ ​  Dividing (i) and (ii), we get,
2
cos b – cos a sin c
1 9 ​ ___________    ​ = – ​ ____
   cos c    
​ = – tan c
x = 4 + __
​   ​  = __
​   ​  sin b – sin a
2 2
32.
(  )
1
when
9
x = __
9
​   ​ , then y = ​​ __
2 1
​   ​  – 4  ​​ ​ = __
​   ​  (i) f (x) = __
​ x ​ on [–1, 1]
2 2 4
Clearly, f (x) is not continuous at x = 0
9 1
Hence, the point is ​ __
​   ​ , __
2 4 (  )
​   ​   ​ So, L.M.V. theorems is not applicable.
(ii) f (x) = sin |x| on [–2p, 2p]
28. Let g (x) = f (x) – x2
Clearly, f (x) is not differentiable at x = 0
fi g (x) has atleast 3 real roots which are
So, L.M.V theorems is not applicable.
x = 1, 2 and 3
(iii) f (x) = |sin x| on [0, 2p]
fi g¢(x) has at least 2 real roots in x Œ (1, 3)
Clearly, f (x) is not continuous at x = p
fi g¢¢(x) has at least 1 real root in x Œ (1, 3)
So, L.M.V. theorems is not applicable.
fi f ¢¢(x) = 2 for at least one root in x Œ (1, 3) (iv) f (x) = log |x| on [0, 5]
Hence, the result.
Clearly, f (x) is not continuous at x = 0
29. Applying L.M.V. theorem, we have,
So, L.M.V. theorems is not applicable.
f (x) – f (0) ____
f (x)
f ¢(x) = _________
​   ​ 

= ​   ​   (v) f (x) = |log | x|| on [–1, 1]
2–0 2
Clearly, f(x) is not continuous at x = –1, 0, 1
f (x) 1 So, L.M.V. theorems is not applicable.
fi ​ ____  = f ¢(x) £ __
 ​  ​   ​ 
2 2 33. Let f (x) = log x, 0 < a < b
fi f (x) £ 1 Clearly, f (x) is continuous and differentiable in (a, b),
where 0 < a < b
Hence, the result. 1
f ¢(x) = __
​ x ​
30. For the function f (x), applying L.M.V. Theorem,
1
f(1) – f (0) f ¢(c) = __
​ c ​, a < c < b
we get, f ¢(c) = _________
​     ​ = f (1) – f (0) ...(i)
1–0
By L.M.V, theorem,
Also, for the function g (x), we have,
f (b) – f (a)
g (1) – g(0) f ¢(c) = __________
​    
    ​
g¢(c) = ​ __________
  
 ​ = g(1) – g (0) ...(ii) b–a
1–0
Dividing (i) and (ii), we get,

(  )
b
log​ __
​ a ​  ​
______
= ​    
  ​
f ¢(c) __________
f (1) – f (0) –a
​ ____   ​ = ​       ​
g¢(c) g(1) – g (0) Now,
a<c<b
1 1 1

6–2
2 = ​ _______
  
4
 ​ = ____
​     ​  fi ​ __ ​ < __
​ c ​ < __
​ a ​
g(1) – 0 g(1) b

fi g (1) = 2
fi ​ __ ​ < ______
​ 
b
__
1 log ​ ​ a ​  ​ __
  

(  )
1
​ < ​ a ​
31. For the function f (x) = cos x, b b–a
f (b) – f (a)
f ¢(c) = ​ _________
b–a
    
​ b–a
fi ​ _____
b
b
​ < log ​ __
   
b–a
(  )
​ a ​  ​ < ​ _____​ 
a   
cos b – cos a
fi – sin c = ___________
​     
   ​ ...(i)
b–a Hence, the result.
p
For the function f (x) = sin x, 34. Let f (x) = tan x in (a, b), where 0 < a < b < __
​   ​ 
2
f (b) – f (a)
f ¢(c) = __________
​    
    ​ Clearly, f (x) is continuous in [a, b] and differentiable
b–a in (a, b)
sin b – sin a f ¢(x) = sec2 x
fi f ¢(c) = ___________
​     
   ​ ...(ii) Now,
b–a
Rolle’s theorem and Lagrange’s mean value theorem  6.17

fi f¢(c) = sec2 c fi f ¢(c) = cos c


By L.M.V Theorem, f (y) – f (x)
fi ​ _________
y – x     ​ = cos c
f(b) – f(a)
f ¢(c) = _________
​      

b–a f (x) – f (y)
tan b – tan a fi ​ _________
x – y  
    ​ = cos c
f ¢(c) = ___________
​     
   ​

| 
b–a
Here, c Œ (a, b)
f (x) – f(y)
fi ​ _________
​  x – y     
​  ​ = |cos c|
|
| 
fi a<c<b
fi sec2 a < sec2 c < sec2 b
f(b) – f (a)
sin x – sin y
fi ​__________
​  x – y  
    ​  ​ £ 1
|
fi sec2 a < _________
​     < sec2 b
  ​  fi |sin x – sin y| £ |x – y|
b–a Hence, the result.
tan b – tan a 37. Let f (x) = tan–1 x – x in [0, x]
fi sec2 a < ___________
​     
   ​ < sec2 b
b–a
Clearly, f (x) is continuous and differentiable. So
fi (b – a) sec2 a < (tan b – tan a) < (b – a)sec2 b L.M.V. theorem is applicable.
Hence, the result. 1 x2
f ¢(x) = _____  ​ – 1 = –  ​ _____
​    2     ​
35. Let f (x) = tan–1 x in (a, b), 0 < a < b 1+x 1 + x2
Clearly f(x) is continuous and differentiable in (a, b) c2
fi f ¢(c) = – ​ _____  
 ​
1 1 + c2
f ¢(x) = _____
​       ​
1 + x2 f(x) – f (0) c2
fi ​ _________
  
 ​  = – ​ _____  
 ​
1 x–0 1 + c2
fi f ¢(c) = _____
​    2 
 ​
1+c f (x) c2
fi ​ ____
x   ​ = – ​ _____  
 ​
Now, a<c<b 1 + c2
fi a2 < c2 < b2 f (x) c2
fi ​ ____
x   

​ = _____
– ​    
 ​ < 0
fi 1+ 2
< 1 + c2 < 1 + b2 1 + c2
tan–1 x – x
1
fi ​ ______
1
 ​ < _____
  2 
1
 ​ < ​ ______
​    2      ​ fi ​ _________
    £0
​ 
1+b 1+c 1+ 2
fi tan–1 x – x £ 0
1 1
fi ​ ______  ​ < f ¢(c) < ______
  2  ​    2  ​ fi tan–1 x £ x
1+b 1+a
fi | tan–1 x | £ |x|
1 f (b) – f (a) ______
1
fi ​ ______  ​ < __________
  2  ​     ​ 
< ​    2 
 ​ Hence, the result.
1+b b – a 1+a
38. Let f (x) = tan x
b–a b–a Clearly, f (x) is continuous and differentiable for all
fi ​ ______2 
 ​ < f (b) – f(a) < ​ ______2 
   
 ​
1+b 1+a
( 
p p
x, y in ​ – __
​   ​ , __
2 2 )
​   ​   ​
b–a b–a
fi ​ ______2 
 ​ < tan–1 b – tan–1 a < ​ ______2 
   
 ​ So, L.M.V. theorems is applicable.
1+b 1+a
fi f ¢(x) = sec2 x
Hence, the result.
fi f ¢(c) = sec2 c
36. Let f (x) = sin x
f (x) – f (y)
Clearly, f (x) is continuous and differentiable So fi ​ _________
x – y  
    ​ = sec2 c
L.M.V. theorem is applicable.
tan x – tan y
f ¢(x) = cos x fi ​ ___________
   
x – y   ​ = sec2 c
6.18  Differential Calculus Booster

| 
tan x – tan y
fi ​ ​  __________
   
x – y   ​  ​ = |sec2 c|
| 1. As we know that every differentiable function

|  |
tan x – tan y is continuous, so it is continuous in [0, 4] and
fi ​ ​  __________
   
x – y   ​  ​ ≥ 1 differentiable in (0, 4)
Thus, by L.M.V. theorem, we have
fi |tan x – tan y| ≥ |x – y|
f (4) – f (0) __________
f (4) – f (0)
Hence, the result. f ¢(a) = __________
​     ​ 
= ​   ​
     ...(i)
4–0 4
39. Given that f is differentiable in (1, 6) So by L.M.V
Theorems Also, by intermediate value theorem, there exists a
point b in (0, 4) such that
f(6) – f(1)
​ _________ ​ = f ¢(x)
  f (4)  +  f (0)
6–1 f (b) = __________
​   ​
     ...(ii)
2
f (6) – f (1) From (i) and (ii), we get,
fi ​ __________
 ​
     ≥2
5 ( f (4) – f (0)) (f(4)  + f(0))
fi f ¢(a) f (b) = ​ _____________________
   
 ​   
fi f (6) – f (1) ≥ 10 8

fi f (6) ≥ f (1) + 10 fi 8f ¢(a) f (b) = ( f 2 (4) – f 2 (0))

fi f (6) ≥ – 2 + 10 = 8 Hence, the result.


2. We have to show that
Thus, m=8
Now, the value of 2 (m + 2)3 + 12 {(f(7))2 + (f(2))2 + f(2) . f(7)}
(f (7) – f (2))​ _________________________
   
 ​   
3
= 2 (8 + 2)3 + 17
= 5f 2 (c) ◊ f (c) 
= 2017.
f 3 (7) – f 3 (2)
40. When f is continuous in [0, 1] and differentiable in i.e. ​ ___________   
 ​  = 3f 2 (c) ◊ f ¢(c)
(0, 1) 7–2
So by L.M.V theorem, we get, Let g (x) = ( f (x))3
f (1) – f (0)
f ¢(x) = __________
​     ​  Clearly, g (x) is continuous in [2, 7] and differentiable
1–0 in (2, 7)
fi f (1) – f(0) = f ¢(x) So, by L.M.V. theorem, there exists a point c in (2, 7)
such that
fi f (1) – 2 £ 3 g (7) – g (2)
g¢(c) = ___________
​    
 ​ 
f (1) £ 5 ...(i) 7–2
When f is continuous in [1, 2] and differentiable in f 3 (7) – f 3 (2)
(1, 2). fi 3f 2 (c)f ¢(c) = ___________
​    
 ​ 
7–2
So, by L.M.V. theorem, we get,
f 3 (7) – f 3 (2)
f (2) – f (1) fi ​ ___________  ​     = 3f 2 (c) f ¢(c)
f ¢(x) = __________
​     ​  5
2–1
f 3 (7) – f 3 (2)
fi f (2) – f (1) = f ¢(x) fi ​ ___________  ​     = 5f 2 (c) f ¢(c)
3
fi f (2) – f(1) £ 3 Hence, the result.
3. As we know that, every differentiable function is also
fi 8 – f (1) £ 3 continuous. So f and g are continuous in [0, 1]
Thus, by L.M.V. theorem, there exist a point c in
fi f (1) ≥ 5 ...(ii)
f ¢(c) f (1) – f (0)
From (i) and (ii), we get, (0, 1) such that ​ ____   ​ = ​ __________
     ​
g¢(c) g(1) – g (0)
f (1) = 5
Rolle’s theorem and Lagrange’s mean value theorem  6.19
f ¢(c) 6–2 4 Theorem, f ¢(c1) = f (1) – f (0)
fi ​ ____   ​ = _____
​    ​ = __
​   ​  = 2
g¢(c) 2–0 2
and f ¢ (c2) = f (2) – f (1).
fi f ¢(c) = 2g¢(c) Thus, f ¢(c2) – f ¢(c1) = f (0) + f (2) – 2f (1)

4. As we know that every differentiable function is fi (c2 – c1) f ¢¢¢ (c) = f(0) + f(2) – 2f (1), c1 < c < c2
continuous. So it is continuous in [1, 6]
fi f (0) + f (2) – 2f (1) = (c2 – c1)  f ¢¢¢(c)
Thus, by L.M.V. theorem, there exists a point
c Œ (1, 6) such that fi f (0) + f (2) – 2f (1) < 0
f (6) – f (1) _______
f(6) + 2
f ¢(c) = __________
​     ​ = ​   ​   fi f (0) + f  (2) < 2f (1)
6–1 5
Hence, the result.
f(6) + 2
fi ​ _______
 ​  = f ¢(c) ≥ 2 8. For some c1 Œ (0, 1) and c2 Œ (1, 2), by L.M.V.
5
fi f(6) ≥ 10 – 2 = 8. f (1) – f (0)
theorem, f ¢ (c1) = __________
​     ​ 
1–0
f (a) f ( x)
5. Let h(x) = f (2) – f (1)
g( a ) g( x ) and f ¢(c2) = ​ _________ ​ 

2–1
Since f (x) and g(x) are continuous in [a, b] and dif-
ferentiable in (a, b), so h(x) is continuous in [a, b] f (1) – f (0)
Now, f ¢(c1) = __________
​     ​ = f (1) – 2
and differentiable in (a, b). 1–0
Thus, by L.M.V. Theorem, there some c in (a, b) fi f(1) – 2 = f ¢(c1) £ 3
h(b) – h(a)
such that h¢(c) = __________
​    
    ​ fi f(1) £ 5
b–a
f (2) – f (1)
f ( a ) f ¢ ( x ) h (b ) - h ( a ) Also, f (c2) = __________
​    
 ​  = 8 – f (1)
fi = 2–1
g( a ) g ¢( x ) b-a
fi 8 – f (1) = f ¢(c2) £ 3
f ( a ) f ¢ (c )
fi h(b) - h(a ) = (b - a ) fi f (1) ≥ 5
g ( a ) g ¢ (c )
From (i) and (ii), we get, f (1) = 5
f ( a ) f (b ) f (a ) f (a ) f ( a ) f ¢ (c ) 9. Let f (x) = sin x
fi - = (b - a )

[  )
g ( a ) g (b ) g ( a ) g ( a ) g ( a ) g ¢ (c ) p
​ __
Clearly f (x) is continuous in ​  0, ​   ​   ​ ​and differentiable
​ 2 ​
f ( a ) f (b ) f ( a ) f ¢ (c )

g ( a ) g (b )
= (b - a )
g ( a ) g ¢ (c ) (  )
p
in ​ 0, __
​   ​   ​
2
Hence, the result. Thus, by L.M.V. theorem, there exists a point c such
6. We have by L.M.V. theorem, f(b) – f(a)
that f ¢(c) = _________
​      

f(b) – f(a) b–a
f ¢(c1) = _________
​     ​, " c1  Œ (a, b)

b–a cos b – cos a
fi ​ ___________
  ​ 
   = cos c
b–a b–a
fi f ¢(c1) = ​ _____ 
​  = 1

| 
b–a
Similarly, f ¢(c2) = 1
cos b – cos a
fi ​ ​ ___________
b–a
   
   ​  ​ = |cos c|
|
| 
Thus, f ¢(c1) + f ¢(c2) = 1 + 1 = 2.

7. By Intermediate Value Theorem,


cos a – cos b
fi ​___________
​ 
a–b
   
   ​  ​ = |cos c|
|

f(0) + f(2)
f (c) = _________
​ 
2
 ​  , 0 < c < 2
cos a – cos b
fi ​___________
​ 
a–b
   
  
| 
​  ​ = |cos c| £ 1
|
fi |cos a – cos b| £ |a – b|
For some c1 Œ (0, 1) & c2 Œ (1, 2), by L.M.V.
Hence, the result.
6.20  Differential Calculus Booster

10. Let f (x) = x3 – 3x + k Then f ¢(x) = 4 x3 – 4 = 4 (x3 – 1)


= 4 (x – 1) (x – w) (x – w2)
Now, f (x) = 3x2 – 3 = 3 (x2 – 1)
Thus, f ¢(x) = 0 has one real root and 2 imaginary
= 3 (x – 1) (x + 1)
roots.
Thus f ¢(x) = 0 has two distinct roots. So, f (x) = 0 has 2 real and 2 imaginary roots.
So, f (–1)  f (1) < 0 16. Let f(x) = x4 – 14x2 + 24x – k
fi (1 – 3 + k) (–1 + 3 + k) < 0
Then f ¢(x) = 4x3 – 28x + 24
fi (k – 2) (k + 2) < 0
= 4 (x3 – 7x + 6)
fi – 2 < k < 2
= 4 (x – 1) (x – 2) (x + 3)
11. Let f (x) = x4 – 4x3 + 12x2 + x – 1
So, f(x) = 0 has four unequal roots.
fi f ¢(x) = 4x3 – 12x2 + 24x + 1
Now, f (– 3) = –117 – k,
fi f ¢(x) = 12x2 – 24x + 24
f  (1) = 11 – k and f (2) = 8 – k
fi f ¢¢(x) = 12 (x2 – 2x + 2) By the sign scheme,
fi f ¢¢(x) = 12 (x – 1)2 + 1) > 0, " x Œ R f (1) < 0, f (2) > 0 & f (– 3) < 0
So f ¢(x) = 0 has only one real root fi (8 – k) < 0, 11 – k > 0 & – 117 – k < 0
and f (x) = 0 has two real roots. fi k > 8, k < 11 and k > – 117
Further more, f (–1) = 15 > 0, f (0) = –1 < 0
fi 8 < k < 11
and f (1) = 9 > 0. Therfore, 8 < k < 11
Thus, f (x) = 0 has one real root in (–1, 0) and another
17. Let f (x) = x3 – 3x + k
real root in (0, 1).
12. Let f (x) = x3 – 6x2 + 15x + 3 fi f ¢(x) = 3x2 – 3

Now, f ¢(x) = 3x2 – 12x + 5 Now, 3 (x2 – 1) = 0 gives x = ± 1


= 3 (x2 – 4x + 3) Since it has two distinct roots in (0, 1)
= 3 (x – 1) (x – 3) so, f (0) f (1) < 0
Thus, f ¢(x) has 2 real roots. fi k (k – 2) < 0
So, f (x) has 3 real roots. fi 0<k<2
13. Let f (x) = 4x3 – 21x2 + 18x + 20
Hence, the value of k is (0, 2).
Then f ¢(x) = 12x2 – 42x + 18
18. Let f (x) = x3 – 2k x2 – 4k x + k2
2
= 6 (2x – 7x + 3)
where f (0) = k2 ≥ 0
= 6 (2x – 1) (x – 3)
Thus, f ¢(x) = 0 has 2 real roots
So, f (x) = 0 has 3 real roots.
14. Let  f (x) = 3x4 – 8x3 – 6x2 + 24x + 1
Now, f ¢(x) = 12x3 – 24x2 – 12x + 24
= 12 (x3 – 2x2 – x + 2) Clearly, f (1) > 0
= 12 (x2 (x – 2) – (x – 2)) fi k2 – 6k + 1 > 0
__
= 12 (x – 2) (x2 – 1) fi (k – 3)2 – (2​÷2 ​
  )  2 > 0
= 12 (x – 2) (x + 1) (x – 1) __
fi (k – 3)2 > (2​÷2 ​
  )  2
Thus, f ¢(x) = 0 has 3 real roots. __
So, f(x) = 0 has 4 real roots. fi   ) 
|(k – 3)| > (2​÷2 ​
__ __
15. Let f (x) = x4 – 4x – 2 fi   )  , (k – 3) < –2​÷2 ​
(k – 3) > (2​÷2 ​   
Rolle’s theorem and Lagrange’s mean value theorem  6.21
__ __
fi   )  , k < 3 – 2​÷2 ​
k > (3 + 2​÷2 ​    ...(i)
Also, f (4) < 0 1. Given f (x) = x3 – 3x + 1
64 – 32k – 16k + k2 > 0 f ¢(x) = 3x2 – 3 = 3 (x2 – 1)
fi k2 – 48k + 64 > 0 = 3 (x + 1) (x – 1)
2
fi (k – 24) – 512 > 0 Clearly, f (x) incresing in (– •, – 1) » (1, •) and
__ decreasing in (– 1, 1)
fi (k – 24)2 – (16​÷2 ​
  )  2 > 0
__ Now, f (– 2) = – 8 + 6 + 1 = – 1
fi (k – 24)2 > (16​÷2 ​
  )  2 f (–1) = –1 + 3 + 1 = 3
__
fi   ) 
|(k – 24)| > (16​÷2 ​ f (0) = 1
__ __ f (1) = 1 – 3 + 1 = –1
fi   )  , (k – 24) < – 16​÷2 ​
(k – 24) > (16​÷2 ​   
f (2) = 8 – 6 + 1 = 3
__ __
fi   )  , k < 24 – 16​÷2 ​
k > (24 + 16​÷2 ​     ...(ii) Now, f (– 2)  f (–1) < 0
From (i) and (ii), we get, Thus, there is a root lies in (–2, –1)
__ __
    < k < 24 + 16​÷2 ​
3 + 2​÷2 ​    So, [a] = – 2
__ __ Also, f (0) f (1) < 0
fi     < k < 8(3 + ÷
3 + ​÷8 ​   ) 
​ 8 ​
Thus, there is a root lies in (0, 1)
Thus m = 3, n = 8 So, [b ] = 0
Hence, the value of 2 (m + n – 1)3 + 17 Again, f (1) f (2) < 0
= 2 (3 + 8 – 1)3 + 17 Thus, there is a root lies in (1, 2)
= 2017. So, [g ] = 1
19. Let f (x) = x3 – 3x + a Since a, b, g are the roots of x3 – 3x + 1 = 0
fi f ¢(x) = 3x2 – 3 so, a+b+g=0
Now, f ¢(x) = 0 gives x = ± 1 Now, {a} + {b} + {g}
Since it has two distinct roots in (–1, 1), so
= a – [a] + b – [b ] + g – [g ]
f (– 1) f (1) < 0
= (a + b + g) – ([a] + [b] + [g ])
fi (– 1 + 3 + a) (1 – 3 + a) < 0
= 0 –(–2 + 0 + 1)
fi (a + 2) (a – 2) < 0 = 1.
fi – 2 < a < 2
ex
Hence, the value of a is (– 2, 2). 2. Let f (x) = ​ __2  ​
x
20. Given f (x) = x3 – 2x2 – 4x + 7
x2 ◊ ex – 2x ◊ ex
fi 2
f ¢(x) = 3x – 4x – 4 f ¢(x) = ____________
​   ​
    
x4
Since f (x) is applicable of Rolle’s theorem, so – 2)ex
xex – 2ex (x
f ¢(k) = 0 f ¢(x) = ________
​   ​ 
 = ________
​   ​  
x3 x3
fi 3k2 – 4k – 4 = 0
fi 3k2 – 6k + 2k – 4 = 0
fi 3k (k – 2) + 2 (k – 2) = 0
fi (3k + 2) (k – 2) = 0
2 Clearly, it has three real and distinct roots.
fi k = 2, – ​ __ ​  At the neighbourhood of x = 2, f(x) will provide us
3
Hence, the value of k is 2. the minimum value
Hence, the minimum value of c is 2.
6.22  Differential Calculus Booster

3. We have f (x) = x4 + 8x3 + 18x2 + 8x + b where g Œ (0, 2)


For 0 < a < g < b < 2, using intermediate
= {(x2 + 4x)2 + 2(x2 + 4x) + 1} + b – 1
mean value theorem
2
= (x2 + 4x + 1)2 + (b – 1)
​Ú ​  ​ 2zf (z2) dz – 2(2g f (g 2))
0
Now, f (x) = 0 gives

fi (x2 + 4x + 1)2 + (b – 1) = 0 ( 
2 gaf (a2) + 2 bb (b2)
= 2​ __________________
​ 
2
 ​ 
      ​ )
fi (x2 + 4x + 1)2 = 1 – b 4
fi ​Ú ​  ​ f (t) dt = 2[af  (a2) + bf (b2)], for 0 < a, b < 2
Let g (x) = (x2 + 4x + 1)2 0

g¢(x) = 2(x2 + 4x + 1) (2x + 4) Hence, the result.

= 4 (x2 + 4x + 1) (x + 2) 5. Let f (x) = x5 – a 0 x4 + 3 ax3 + bx2 + cx + d


__
g¢(x) = 0 gives x = – 2 ± 2​÷3 ​
  ,  2 f ¢(x) = 5x4 – 4a 0 x3 + 9 ax2 + 2 bx + c

Clearly, the range of g(x) is (0, 9) f ¢¢(x) = 20 x3 – 12a 0 x2 + 18 ax + 2b

f ¢¢¢(x) = 60x2 – 24a 0 x + 18a

= 6 (10x2 – 4a 0 x + 3a)

Now, its discriminant is 16​a2​o​​  – 120 a


For four distict solutions, (1 Рb) Π(0, 9)
fi 0 < 1 – b < 9 = 8(2​a2​o​​  – 15a) < 0
fi – 1 < – b < 8 Thus, f ¢¢¢(x) has no real roots
fi – 8 < b < 1 Therefore, all the roots of f (x) = 0 can not be real.
fi b Œ (– 8, 1) 6. Since f ¢¢(x) exists for all x Œ [0, 1]
4. So, f (x), f ¢(x) are differentiable and continuous in
(i) By Lagranges Mean Value theorem [0, 1]
f (4) – f (0) Now, f(x) is continuous in [0, 1] and differentiable
​ __________
   ​ = f ¢(a), a Œ (0, 4) ...(i) in (0, 1) and f (0) = f (1)
4–0
Thus, by Rolle’s theorem there exists atleast one c
Also, from intermediate value theorem in (0, 1) such that f ¢(c) = 0.
f (4) + f (0) Now, let x Œ [0, 1]
​ __________
 ​
     = f (b), b Œ (0, 4) ...(ii)
2 Case-I: when x < c
From (i) and (ii), we get, since f is differentiable in [0, 1]

( 
​ 
4
 ​ 
     ) ( 
f (4) – f (0) __________
fi ​ __________
f (4) + f (0)
​ ​ ​ 
2
 ​ 
     )
​ = f ¢(a)f(b)


So, f ¢(x) will be continuous and differentiable in (0, 1)
By L.M.V. theorem
for a, b Œ (0, 4) f ¢(x) – f ¢(c)
2 2
​ __________
x – c      ​ = f ¢¢(a), " a Œ (c, x)
fi (f (4)) – (f (0)) = 8f ¢(a) f (b), for a, b Œ (0, 4)
Hence, the result. f¢(x) – 0
2
​ ________
x – c   ​ = f ¢¢(a)

(ii) Replacing t by z , we get,
4 2 f ¢(x) = (x – c) f ¢¢(a)
​Ú ​  ​ f (t) dt = ​Ú ​  ​ 2zf  (z2) dz
0 0 |f (x)| = |(x – c)| f ¢¢(a) ...(i)
From Lagranges Mean Value theorem As | f¢¢(x)| £ 1 " x Œ [0, 1]
2 0
|f ¢¢(a)| £ 1, " x Œ [0, 1]
Ú​  ​  ​  2zf (z2) dz – ​Ú ​  ​ ​ 2zf (z2) dz
​ ______________________
0 0
     ​    = 2g f (g 2) |f ¢(x)| < 1, " x Œ [0, 1]
2–0
Rolle’s theorem and Lagrange’s mean value theorem  6.23

Case-II: When x > c Questions asked in IIT-JEE Exams


By L.M.V. theorem,
1. Ans. (a)
f ¢(c) – f ¢(x) Let f (x) = ax3 + bx2 + cx
​ __________
c – x  
    ​ = f ¢¢(a)
f (0) = a ◊ 0 + b ◊ 0 + c ◊ 0 = 0
f ¢(x) – f ¢(c) Also, f (1) = a + b + c = 0
​ __________
x – c      ​ = f ¢¢(a)
Thus, from Rolle’s theorem, there exists atleast a root
f ¢(x) = (x – c) f ¢¢(a) of its derivative.
So, f ¢(x) has a root in (0, 1).
| f ¢(x)| = |(x – c)|  f ¢¢(a) 2. Ans. (a)
|f ¢(x)| < 1 for all x in [0, 1] ÏÊ 1 ˆ 1
ÔÁË 2 - x ˜¯ : x < 2
Case-III: When x = c Ô
(a) We have f (x) = Ì 2
f ¢(x) = f ¢(c) = 0 ÔÊ 1 - x ˆ : x ≥ 1
ÔÁË 2 ˜¯ 2
| f ¢(x)| = 0 < 1 Ó
| f ¢(x)| < 1, for all x Œ [0, 1] Ï 1
From all the cases, we get, ÔÔ-1 : x<
2
f ¢ (x) = Ì
|f ¢(x)| < 1, for all x Œ [0, 1] Ô-2 ÊÁ 1 - x ˆ˜ : x ≥ 1
ÔÓ Ë 2 ¯ 2
Hence, the result.
1
7. Let f (x) = log (1 + x) in [0, x] Clearly f (x) is not differentiable at x = __
​    ​
2
Thus, Lagranges Mean Value Theorem is not
Since f (x) satisfies the condition of L.M.V. theorem
applicable.
in [0, x], there exists q (0 < q < 1) such that
Ï sin x
f (x) – f (0) Ô : xπ0
​ __________
  
 ​  = f ¢(q x) (b) We have f (x) = Ì x
x–0 ÔÓ1 : x=0
log (1 +  x) ______
1 Clearly f (x) is continuous in [0, 1] and differ-
fi ​ _________
x   
​ 
= ​      ​
1 + qx entiable in (0, 1)
Now, 0 < q < 1, x > 0 fi q x < x Thus, Lagranges Mean Value Theorem is applicable.
(c) We have f (x) = x|x| = x2 in [0, 1]
fi 1 + q x < 1 + x
As we know that every polynomial function is
1 1
fi ​ _______ > _____
   ​  ​       ​ continuous and differentiable everywhere.
1 + q x 1 + x
So it is continuous in [0, 1] and differentiable
x x
fi ​ _____ ​ > _____
     ​       ​ ...(i) in (0, 1).
1 +q x 1 + x
Thus, Lagranges Mean Value Theorem is
Again 0 < q < 1, x > 0 applicable.
(d) Also, f (x) = |x| = x
fi q x > 0
Clearly it is continuous in [0, 1] and differentiable
fi 1 + q x > 1 in (0, 1).
1 Thus, Lagranges Mean Value Theorem is
fi ​ ______
    ​ < 1
1 + q x applicable.
x 3. Ans. (d)
fi ​ ______
    ​ < x .....(ii)
1 + q x Given f (x) = x a  log x
From (i) and (ii), we get, Since it is applicable for Rolle’s theorem in [0, 1],
x so it must be continuous in [0, 1]
​ _____    ​ < log (1 + x) < x
1+x Now, we should check it at the right end of x = 0
Hence, the result. i.e. ​ lim 
   + 
​ f (x) = f (0) = 0
xÆ0
6.24  Differential Calculus Booster

Hence, the result.


Now, ​ lim 
   + 
​ f (x) = ​ lim 
   +  ​ (x a log x)
xÆ0 xÆ0
5. We have g (x) = (f ¢(x))2 + g¢(x)f ¢¢(x)

= ​ lim 
    
x Æ 0+ (  )
log x
​ ​ ____
​  –a ​  ​
x

dx
d
g (x) = ___
​    ​  ( f(x) ◊ f ¢(x))

(  )
By the Rolle’s theorem, between any two roots of a
1
__ polynomial, there is a root of its derivative.
​ x ​
= ​ lim 
     _______
​ ​ ​     
 ​  ​
x Æ 0 + – ax–a–1 Now, f(x) ◊ f ¢(x) = 0

(  )
1 Either f(x) = 0 or f ¢(x) = 0
= ​ lim 
   +  ​​ _____  ​  ​
​   –a 
xÆ0 –ax Thus, f (x) has four zeroes at x = a, between b and
c, between c and d and at e
= ​ lim 
   + 
xÆ0 –a(  )
xa
​ ​ ​ ___   ​  ​ So, f ¢(x) has atleast 3 zeroes.
Thus, f (x) f ¢ (x) has atleast 7 zeroes.
It will be zero only when a > 0 Therefore, g(x) has atleast 6 zeroes.

4. Let f (x) = Ú P (x) dx 6. We have x4 – 4x3 + 12x2 + x – 1 = 0


f (x) = Ú (51 x101 – 2323 x100 – 45x + 1035) dx Now, f (– 1) = 1 + 4 + 12 – 1 – 1 = 15 > 0

51 102 _____
2323 45 f (0) = – 1 < 0
f (x) = ____ x – ​   ​ x101 – ___
​    ​   ​   ​ x2 + 1035x
102 101 2 and f (1) = 1 – 4 + 12 + 1 – 1 = 9 > 0
1 45 So f (x) = 0 has a root in (–1, 0) and a root in
f(x) = __
​   ​  x102 – 23x101 – ___
​   ​ x2 + 1035x
2 2 (0, 1).
Clearly f (x) is continuous and differentiable in Thus, f (x) = 0 has atleast two distinct real roots.

( 
​ ​45​100 ​, 46  ​
1
​ ____
   ​ 
) Also, f ¢(x) = 4x3 – 12x2 + 24x + 1

f ¢¢(x) = 12x2 – 24x + 24


Also, (  1
____
​     ​ 
f  ​ ​45​100 ​  ​ ) and
= 12 (x2 – 2x + 1) + 1

1
= __ (  102
____
​   ​  ) ( 
101
____
)
45
​ ​  ​ – ___
​   
​   ​ ​  4​5 ​101 ​  ​ – 23​ 4​5100
 ​
2
____
​     ​ 
​   ​ (45​)​100 ​
= 12(x Р1)2 + 1 > 0, " x ΠR
2 2
1
____
  + 1035 (45​)​100 ​
​     ​  fi f ¢(x) increases on R.
2
____ 1
____
fi y = f ¢(x) intersects the x-axis exactly once.
1
= __
​    ​  ​     ​ 
​   ​  ◊ 45 ◊ 4​5​101 ​ – 23 ◊ 45 ◊ 4​5​100 ​
2 fi y = f (x) has exactly three real roots.
2 1
1 ____ ____ 7. Ans. (a, b, c, d)
  – ​ __  ​ 45 ◊ 4​5​100 ​ + 23 ◊ 4​5​100 ​
​     ​  ​     ​ 
2 x

We have f (x) = Ú​  ​    ​​e​t ​ (t – 2) (t – 3) dt


2
=0
0
Also, f (46)
f ¢(x) = ex2 (x – 2) (x – 3)
1 45
= __
​   ​  (46)102 – 23 (46)101 – ___
​   ​  (46)2 + 1035 (46)
2 2
1
= __
​   ​  (23 . 2)102 – 23 (23 . 2)101
2
45
  – ___ ​   ​  ◊ 46 ◊ 46 + 1035 (46)
2 Clearly, has maximum at x = 2 and minimum at
=0 x = 3 and f (x) decreasing in (2, 3)
Thus, all the conditions of Rolle’s theorem are So, by Rolle’s Theorem f ¢(x) = 0 for
satisfied. x = 2 and x = 3.
So there exist a point c Œ ​ 4​5​100 ​, 46  ​ such that (  1
​ ____
   ​
) Thus, there exists a point c Œ (2, 3) such that
f ¢(c) = 0. f ¢¢(c) = 0.
Chapter

7 Monotonocity

Concept Booster

1.  Definitions
1. A function f (x)  is said to be strictly increasing about
the point x = a if f (a – h) < f (a) < f (a + h), where
h is a very small positive arbitrary number.

4. A function f (x) is said to be non-decreasing about


the point x = a if f (a + h) £ f (a) £ (a – h), where
h is a very small positive arbitrary number.

2. A function f (x) is said to be strictly decreasing about


the point x = a if f (a + h) < f (a) < f (a – h), where
h is a very small positive arbitrary number.

5. Let a function f be differentiable at x = a


(i) If f ¢(a) > 0, then f (x) is strictly increasing at
x = a.
(ii) If f ¢(a) < 0, then f (x) is strictly decreasing at x = a.
(iii) If f ¢(a) = 0, then we need to examine the signs
of f ¢(a – h) and f ¢(a + h)
(a) If f ¢(a – h) > 0 and f ¢(a + h) > 0, then f (x) is
strictly increasing at x = a.
(b) If f ¢ (a – h) < 0 and f ¢(a + h) < 0, then f (x) is
3. A function f (x) is said to be non-increasing about the strictly decreasing at x = a.
point x = a if f (a – h) £ f (a) £ (a + h), where h is
(c) If f ¢(a – h) and f ¢(a + h) have opposite signs,
a very small positive arbitrary number.
then f (x) is neither increasing nor decreasing
(non-monotonic) at x = a.
7.2  Differential Calculus Booster

2. Test for Finding the Monotonicity at an End Point Note:


Let the function f be differentiable at x = a. Discrete point
(i) If x = a is the left end point, then we check as 1. If f ¢(x) = 0 at discrete points, we mean that the
follows points where f ¢(x) becomes 0 do not form an inter-
val. i.e. they are separated from each other.
(a) If f ¢(a +) > 0, then f (x) is strictly increasing at
x = a. 2. If f ¢(x) ≥ 0 for all x Œ (a, b) where f ¢(x) = 0 at
discrete points in (a, b), then f (x) is also increasing
(b) If f ¢(a +) < 0, then f (x) is strictly decreasing at in (a, b)
x = a.
(c) If f ¢(a +) = 0, but f ¢(a + h) > 0, then f (x) is 5. Critical Point
strictly increasing at x = a. A critical point of a function f ¢ is a number ‘a’ in the domain
+
(d) If f ¢(a ) = 0, but f ¢(a + h) < 0, then f (x) is of f such that either f ¢(a) = 0 or f ¢(a) does not exist.
strictly decreasing at x = a. Sometimes we will want to distinguish critical numbers at
(ii) If x = a is the right end point, then we check as which f ¢(x) = 0 from those at which f is not differentiable.
follows We will call a point on the graph of f at which f ¢(x) = 0,
(a) If f ¢(a–) > 0, then f (x) is strictly increasing at a stationary point of f.
x = a. The stationary point of f are the x-intercepts of the graph
of f.
(b) If f ¢(a–) < 0, then f (x) is strictly decreasing at
x = a. If x = a is a critical point of f, then it is also a critical
point of the function g (x) = f (x) + k, " k = R.
(c) If f ¢(a–) = 0, but f ¢(a – h) > 0, then f (x) is
For example, x = 0 is a critical point of f (x) = x2, then
strictly increasing at x = a.
x = 2 is a critical point of f (x) = (x – 2)2.
(d) If f ¢(a–) = 0, but f ¢(a – h) < 0, then f (x) is
strictly decreasing at x = a. Remember the Step to Find the Intervals of
Increasing and Decreasing
3. Necessary Condition for Monotonicity
(i) First we find f ¢(x), and then find where f ¢(x) = 0 or
Let f (x) be a differentiable function does not exist. These points are the critical points
(i) A function f (x) is said to be increase in an interval for the function f (x).
if f ¢(x) ≥ 0. (ii) Plot the critical points on the number line and put +
(ii) A function f (x) is said to be decrease in an interval and – in a alternative way.
if f ¢(x) £ 0. (iii) On a certain interval, if f ¢(x) is +ve, then f (x) is
(iii) If f (x) does not vary in an interval, then f ¢(x) = 0. strictly increasing and is f ¢(x) is –ve then f (x) is
strictly decreasing.
4. Sufficient Condition for Monotonicity
(iv) On a certain interval, if f ¢(x) ≥ 0, then f (x) is
(i) If f ¢(x) > 0 in (a, b), then f (x) is increasing in increasing and if f ¢(x) £ 0 then f (x) is decreasing.
(a, b).
(ii) If f ¢(x) < 0 in (a, b), then f (x) is decreasing in 6. Application of Monotonocity in Isolations of Roots
(a, b). Let f (x) be a real function and a, b be two real numbers
(iii) If f ¢(x) = 0, then f (x) does not vary in (a, b). such that a < b. If
(i) f is continuous on [a, b] and differentiable on (a, b).
Monotonicity at Points where f ( x ) does not exist (ii) f (a) and f (b) have opposite signs.
Let f (x) be a continuous function whose derivative f ¢(x) does (iii) either f ¢(x) > 0 or f ¢(x) < 0 on (a, b)
not exist at x = c but exist in the nbd of x = c. Then f (x) has an exactly one root between a and b.
(i) If f ¢(c –) > 0 and f ¢(c +) > 0, then f (x) is strictly
increasing at x = c. 7. Algebra of Monotonous Functions
(ii) If f ¢(c – h) > 0, f ¢(c –) ≥ 0, and f ¢(c–) ≥ 0, f ¢(c + h) > 0, Let increasing = I and decreasing = D
then f (x) is strictly increasing at x = c. (where both the functions take +ve values) and can not say
(iii) If f ¢(c –) < 0, f ¢(c+) < 0, then f (x) is strictly decreasing anything = W
at x = c. (i) Addition
(iv) If f ¢(c – h) < 0, f ¢(c –) £ 0 and f ¢ (c + h) < 0, f ¢ (c +) (a) I + I = I (b) D + D = D
£ 0, then f (x) is stictly decreasing at x = c.
(c) I + D = W (d) D + I = W
Monotonocity  7.3

(ii) Negativity
(a) – I = D (b) – D = I
(iii) Difference
(a) I – I = W (b) D–D=W
(c) I – D = I (d) D – I = D.
(iv) Product
(a) I × I = I (b) I×D=W
(c) D × D = D (d) D × I = W.
(v) Reciprocality (iii) Hyper-Critical Point:  A hyper-critical point
(a) 1/I = D (b) 1/D = I of a function f is a number c in the domain of
f such that either f ¢¢(c) = 0 or f ¢¢(c) does not
(vi) Division 
exist.
(a) I/I = D (b) I/D = I
(c) D/D = D (d) D/I = D Rule to find out the Intervals of Concavity
(vii) Composition  (i) First we find f ¢¢(x) and then we should find the hyper-
(a) I(I) = I (b) I(D) = D critical points i.e. points at which f ¢¢(x) equals zero
(c) D(I) = D (d) D(D) = I. or does not exist.
(ii) Plot the hyper-critical points on the number line, and
8. General Approach to Proving Inequalities then put + and – in a alternative way.
Some basic idea to proving inequalities (iii) On a certain interval, if f ¢¢(x) > 0, then the function
(i) If f (x) is continuous in [a, b] and differentiable f (x) is Concave Up and if f ¢¢(x) < 0, then the function
in (a, b), where f ¢(x) > 0 for all x in (a, b) and f (x) is Concave Down.
f (a) ≥ 0, then f (x) > 0, " x Œ (a, b).
10. Inflection Point  
(ii) If f (x) is continuous in [a, b] and differentiable
in (a, b), where f ¢(x) < 0 for all x in (a, b) and Let us consider the graph of a function has a tangent line
f (a) £ 0, then f (x) < 0, " x Œ (a, b). (possibly vertical) at the point P(c, f (c)) and that the graph
(iii) If f (x) is continuous in [a, b] and differentiable in is concave up on one side of P and Concave down on the
(a, b), where f ¢(x) > 0 for all x in (a, b) and f (a) £ 0, other side of P. Then P is called an inflection point of the
then f (x) < 0, " x Œ [a, b). graph.
(iv) If f (x) is continuous in [a, b] and differentiable in
(a, b), where f ¢(x) < 0, for all x in (a, b) and f (b) ≥ 0,
Some basic Idea on Inflection Points
then f (x) > 0, " x Œ [a, b). (i) A continuous function f need not have an inflection point
where f ¢¢(x) = 0 For example:
9. Concave up and Concave Down 
1. Let f (x) = x4. Then f ¢¢(0) = 0, but the graph is always
(i) Concave Up :  If the second derivative f ¢¢ (x) is every- Concave Up.
where is positive within an interval, then the arc of 2. Let us take another function f (x) = x5 + 5x4.
the curve y = f (x) corresponding to that interval is
Concave Up. Then f ¢¢(x) = 20x2 (x + 3).
At the nbd of x = – 3, f ¢¢(x) changes it sign from
negative to positive. So x = – 3 is the inflection point.
But at the nbd of x = 0, f ¢¢(x) does not change its
sign. So the graph of the given function is concave
up on both sides of the origin.
3. If x = b is a point of inflection of a curve y = f (x) and
f ¢¢(b) is exist, then f ¢¢(b) must be equal to zero.
4. The point (– 1, 0) in y = (x – 1)3, being both a critical
point and a point of inflection, is a point of horizontal
inflection.
(ii) Concave Down:  If the second derivative f ¢¢(x) is
everywhere is negative within an interval, then the 5. If a function f is such that f ¢¢¢(x) is continuous and
arc of the curve y = f (x) corresponding to that inter- f ¢¢(b) = 0, while f ¢¢(b) π 0, then the curve y = f (x)
val is Concave Down. has a point of inflection for x = b.
7.4  Differential Calculus Booster
__
6. It shold be noted that a point separating a Concave Let us take f (x) = 3 ​÷x 
  ​  in the vicinity of the origin.
Up arc of a curve from a Concave Down, one may In such a case, we speak of a point of inflection with
be such that the tangent at that point is perpendicular vertical tangent.
to the x-axis, that is, either a vertical tangent or a 7. A number b such that f ¢¢(b) is not defined and the
concavity of f changes at b will correspond to an
tangent does not exist.
inflection point if and only if f (b) is defined.

Exercises

Critical Points
(Problems Based on Fundamentals) ex
16. Find the critical points of f (x) = ​ _____
   ​
x–1
1. Find the interval of the monotonicity of the function
f (x) = 2x3 – 12x2 + 18x + 5. 5x2 – 18x + 45
17. Find the critical points of f (x) = ​ ____________
  
   ​
2. Find the interval of the monotonicity of the function x2 – 9
f (x) = 5 + 36x + 3x2 – 2x3. 18. Find the critical points of the function
3. Find the interval of the monotonicity of the function f (x) = x4/5 (x – 4)2.
f (x) = (x – 1)3 (x – 2)2.
19. Find the critical points of the function
4. Find the interval of the monotonicity of the function
f (x) = 2x3 – 3x2 + 6x + 10 f (x) = x + cos– 1 x + 1.

5. Find the interval of the monotonicity of the function 20. Find the critical points of the function
___________
f (x) = 2x3 + 3 2 + 12x + 20.
f (x) = ​÷x  2 – 6x +  
15 ​
6. Find the interval of the monotonicity of the function
Increasing and Decreasing Functions
x 2
f (x) = __
​    ​ + __
​   ​. 21. Find the interval of increasing and decreasing of a
2 x
7. Find the interval of the monotonicity of the function function f (x) = 2x2 – ln |x|.
|x – 1|
f (x) = 5x3/2 – 3x5/2, x > 0. 22. Find the intervals for the function f (x) = ​ _____ ​ 
 is
x2
8. Find the interval of the monotonicity of the function
_____ increasing and decreasing.
f (x) = log ​( x + ÷  ​ )​.
– x 2
​ 1  + x2  ​ ___ ​ 
23. Find the intervals for the function f (x) = x 2 ​e​ a2 ​,
9. Find the interval of the monotonicity of the function a > 0 is increases. Isolation points.
x 24. Show that the equation x3 = 3x + 1 has a real root
f (x) = ____
​     ​. 
log x in [– 1, 1].
10. Find the interval of the monotonicity of the function x2
_____ 25. Show that the equation ex =1 + x + ​ __ ​ has a real root
2
f (x) = x – cot– 1 x – log ​( x + ​÷x  2 + 1 ​ 
  )​.
in [– 1, 1].
11. Find the least value of m for which the function Algebra of Monotonic Functions
f (x) = – x2 + m x + 1 is strictly increasing in [1, 2]
26. Find the interval where the function f (x) = tan– 1 (e x)
12. For what values of b, the function f (x) = sin x – b x + c is strictly increasing.
is strictly decreasing for all x in R. 27. Find the interval in which f (x) = tan– 1 (log1/3 x) is
13. Find all possible values of ‘a’ for which the function strictly decreasing.
f (x) = e2x – (a + 1) ex + 2x is strictly increasing for 28. Find the interval in which f (x) = cot– 1 (log4 x) is
all x in R. strictly decreasing.
14. For what values of a is the function 29. Find the interval in which f (x) = cot– 1 (log1/10 x) is

( 
a 2  –  1 3
f (x) = ​ ​  _____
3 )
 ​ x + (a – 1)x 2 + 2x + 1 strictly
 ​  
strictly increasing.
30. Find the______
interval of the monotonocity of the function
increasing? f (x) = ÷   – x2 
​ 3x  ​.
15. For what values of a, the function f (x) = (a + 2) x3 31. Find the interval of the monotonicity of the function
– 3a x2 + 9a x – 1 is strictly decreasing for all x in R. f (x) = tan– 1 (sin x + cos x) in (0, 2p).
Monotonocity  7.5

32. Find the interval of the monotonocity of the function

(  )
log x
f (x) = log ​ ____
​  x   
​  ​.
(Mixed Problems)
1
33. Find the interval in which the function 1. The function f (x) = __
​ x ​ on its domain is

f (x) = sin(log x) + cos(log x) is decreases. (a) increasing (b) decreasing


34. Find the interval of the monotonocity of the function (c) constant (d) none
2 – x
f (x) = loge (cos x) for all x Œ (0, p). 2. Let y = x e , then the interval in which y increases
35. Find the interval of the monotonocity of the function with respect to x is
f (x) = sin(sin x) + cos(sin x) in (0, p). (a) (– •, •) (b) (–2, 0)
Inequality (c) (2, •) (d) (0, 2)
x2 3. If f (x) = x e x (1 – x), then f (x) is
36. Prove the inequality, log (1 + x) > x – ​ __ ​ for all x in R +

37. Prove the inequality log (1 + x) > _____


2
x
​       ​ for x > 0

1
2 [  ]
(a) inc. on ​ – ​ __ ​ ,  1  ​ (b) dec. on R
1+x
38. Prove that (ex – 1) > (1 + x) log(1 + x), if x > 0
(c) Inc. on R
1
2 [  ]
(d) dec. on ​ – ​ __ ​ , 1  ​

x 2 + 1
39. Prove that 2x tan– 1 > log (1 + x2) for all x in R +.
4. If f (x) = Ú​  ​  ​ ​ e​t​ ​dt, then the interval in which f (x) is
2

_____ _____

40. Prove that 1 + x log ​( x + ​÷x  2 + 1 ​  
)​ ≥ ​÷1  + x2  x2
 ​ for all
inc. is
x ≥ 0.
(a) (0, •) (b) (– •, 0)

p
(  )
41. Prove that cos (sin x) > sin (cos x), if x Œ ​ 0, __
​   ​   ​.
2 (c) [– 2, 2] (d) no where
42. Find the smallest positive constant B such that a sin x + 2 cos x
5. If the domain f (x) = _____________
​    
     ​ is inc. for all
x £ B x2 for all x > 0. sin x + cos x
b values of x, then
43. If x2 + ​ __ +
x ​ ≥ c, " x Œ R , where a, b, c are +ve con-
(a) a < 1 (b) a > 1
stants, prove that 27ab2 ≥ 4c3
(c) a < 2 (d) a > 2
Concavity 6. The interval of increases of the function f (x) = x – ex
44. Find the interval of the concavity of the function
f (x) = x5 + 5x – 6. (  )
2p
+ tan ​ ​ ___ ​   ​ is equal to
7
45. Find the interval of the concavity for the function (a) (0, •) (b) (– •, 0)
f (x) = x4 – 5x3 – 15x2 + 30. (c) (1, •) (d) (– •, – 1)
– 1
46. Find the interval of the concavity for the function 7. The function f (x) = cot  x + x increases in the
f (x) = (sin x + cos x) ex in (0, 2p.) interval
47. Show that the curve y = f (x) = Ax2 +Bx + c is concave (a) (1, •) (b) (– 1, •)
up if A > 0 and concave down if A < 0. (c) (– •, •) (d) (•, •)
Point of Inflection 8. The function f (x) = x x decreases on the interval
48. Find the inflection point of the function (a) (0, e) (b) (0, 1)
f (x) = x4 – 4x3 + x – 10
49. Find the point of inflection of the curve
(  )
1
(c) ​ 0, __
​ e ​  ​ (d) none
x
y = f (x) = (x – 2)2/3 + 10 9. The function f (x) = ____
​     ​ increases on the interval
log x
50. Find the point of inflections of the curve (a) (0, •) (b) (0, e)
4 3 2
f (x) = x – 6x + 12x – 8x + 3 (c) (e, •) (d) none
log x
51. Find the point of inflection of the curve 10. The function f (x) = ​ ____  ​ is increasing in the
x   
1 interval
y = f (x) = x2 – ___
​   3 ​ 
6x (a) (1, 2e) (b) (0, e)
52.
  2
Find the inflection point of the curve y = f (x) = ​e–​ x ​ (c) (2, 2e) (d) ​ __
1
( 
​ e ​, 2e  ​. )
7.6  Differential Calculus Booster
x 21. Let h(x) = f (x) – ( f (x)) 2 + ( f (x))3 for every real
11. The function f (x) = ______
​    2 ​  decreases on the interval
4  +  x number x then
(a) (– •, – 1) (b) (– •, 0) (a) h is inc. whenever f is inc.
(c) (– •, – 2) » (2, •) (d) (– 2, 2) (b) h is inc. whenever f is decreasing
12. The function f (x) = tan x – x (c) h is dec. whenever f is inc.
(a) always increases (b) always decreases (d) nothing can be said in general.
(c) never decreases (d) sometimes increases 22. If f (x) = x3 + bx2 + cx + d and 0 < b2 < c, then in
13. If a < 0, the function f (x) = ea x + e– ax is a monotonic (– •, •), f (x) is
dec. function for all values of x, where (a) increasing
(a) x > 0 (b) x < 0 (b) decreasing
(c) x > 1 (d) x < 1 (c) has local max. or min.
14. The value of b for which the function (d) bounded
f (x) = sin x – bx + c is dec. in (– •, •) is given by 23. The function f (x) = 2 log (x – 2) – x 2 + 4x + 1
increases on the interval
(a) b < 1 (b) b ≥ 1
(a) (1, 2) (b) (2, 3)
(c) b > 1 (d) b £ 1
15. The value of a for which the function
f (x) = sin x – cos x – ax + b decreasing for all real
(  )
5
(c) ​ __
​   ​ ,  3  ​
2
(d) (2, 4)

values of x is given by 24. Let y = x2 (x – 3)2 increases for all values of x lying
__ in the interval
(a) a ≥ ​÷2 ​
   (b) a ≥ 1
__ 3
(c) a < ÷   
​ 2 ​ (d) a < 1 (a) 0 < x < __ ​   ​   (b) 0 < x < •
2
1 (c) – • < x < 0 (d) 1 < x < 3
16. y = sin x – a sin 2x – __
​   ​  sin 3x + 2 ax , then y increases
3 25. Let f (x) = x3 + ax2 + bx + 5 sin2 x be an inc. function
for all values of x, if on the set R. Then a and b satisfy

(a) a = 1 (b) a > 1 (a) a2 – 3b – 15 > 0 (b) a2 – 3b + 15 > 0


(c) a > 0 (d) 0 < a < 1 (c) a2 – 3b + 15 < 0 (d) a < 0 and b > 0
17. The set of values of a for which the function 26. The function f (x) = sin  x + cos4 x increases if
4

f (x) = x2 + ax + 1 is an inc. function on [1, 2] is p p 3p


(a) 0 < x < __ ​   ​   (b) ​ __ ​  < x < ___
​   ​ 
(a) (– •, – 4) (b) [– 4, •) 8 4 8
(c) [4, •) 3p 5p 5p 3p
(d) (– •, 4] (c) ​ ___ ​ < x < ___​   ​   (d) ​ ___ ​ < x < ___
​   ​ 
8 8 8 4
18. The length of a longest interval in which the function
3 sin x – 4 sin3 x is increasing, is 27. The set of all x for which log (1 + x) £ x is
(a) (0, •) (b) (– 1, •)
p p
(a) ​ __ ​   (b) ​ __ ​  (c) (– 1, 0) (d) none
2 2
3p 28. For all x Œ (0, 1),
(c) ​ ___ ​   (d) p
2
_____
(a) ex < 1 + x (b) log (1 + x) < x
19. If f (x) = 2x + cot – 1
 x + (​  ​÷1  + x2 
 ​+ 1 )​, then f (x) (c) sin x > x (d) log e x > x



(a) inc. on [0, •)
(b) dec. on [0, •) (  )
l2 – 1 3
29. If f (x) = ​ ​ ______
l2 + 1
  
 ​  ​ x – 3x + 5 is a dec. function of in

(c) neither inc. nor dec. on [0, •) R then the set of all possible values of l (independent
(d) inc. on (– •,

•) of x) is
(a) [– 1, 1] (b) (– •, – 1)
20. Let f (x) = ​Ú ​  ​  ex (x – 1) (x – 2) dx, then f decreases in
  (c) (1, •) (d) none
the interval
3 2
30. Let f (x) = x + 6x + px + 2. If the largest possible
(a) (– •, – 2) (b) (– 2, – 1)
interval in which f (x) is dec. function in (– 3, – 1)
(c) (1, 2) (d) (2, •) then p is
Monotonocity  7.7

(a) 2 (b) 6 p p
(a) ​ __ ​   (b) ​ __ ​ 
(c) 8 (d) 9 2 4
p p
31. If f (x) = sin2 x – 3 cos2 x + 2ax – 4 is increasing for (c) ​ __ ​   (d) ​ ___  ​ 
all x ≥ 0, then the value of a lies in 8 16
40. The number of solutions of the equation x3 + 2x2 +
(a) [– 2, 0) (b) (– •, 2]
5x + 2 cos x = 0, in [0, 2p] is
(c) [2, •) (d) (– •, 2]. (a) 1 (b) 2
(c) 3 (d) 0
32. Let f (x) = sin2 x – (2a + 1)sin x + (a – c). If f (x) £ 0
p
[  ]
for all x in ​ 0, __
​   ​   ​, then the range of a is
2
41. The number of solutions of the equation x3 + 2x2 +
5x + 2 sin x = 0 in [0, p] is
(a) [– 3, 0] (b) [3, •) (a) 3 (b) 2
(c) [– 3, 3] (d) (– •, 3]. (c) 1 (d) 0
33. The set of values of a for which the function
x
f (x) = (4a – 3) (x + 5) + 2 (a – 7) cot ​ __ (  ) (  )x
​    ​  ​ sin2 ​ __
2
​    ​  ​. does
2
  (Problems for JEE-Advanced)
not posses any critical point is given by 1. Find the length of the largest continuous interval in
(  ) 4
(a) ​ – •, – ​ __ ​   ​
3
(b) (– •, – 1) which the function f (x) = 4x – tan 2x is monotonic
increasing.
(  )
4
(c) ​ – ​ __ ​ , 2  ​
3 ( 
4
)
(d) ​ – •, – ​ __  ​  ​ » (2, •)
3
2. Find the number of solutions of the equation x3 + 2x2
+ 5x + 2 cos x = 0 in [0, 2p]
34. The number of roots of the cubic f (x) = x3 + 3x + k, x
" k Œ R is 3. If f (x) = (ab – b2 – 2) x + Ú​  ​  ​ (cos4q + sin4q) dq is a
(a) 0 (b) 1 0
(c) 2 (d) 3 decreasing function of x for all x, b in R, find a.
35. The number of inflection points on the curve repre- 4. If f (x) = cos x + a2 x + b is an increasing function
sented by the equations x = t 2, y = 3t + t 3 is for all values of x, find a.
(a) 0 (b) 1 5. If f (x) = 2ex – a e– x + (2a + 1) x – 3 is increasing for
(c) 2 (d) 3 all x, find a.
36. If f ¢(x2 – 4x + 3) > 0 " x Œ (2, 3), then f (sin x) is 1 1
increasing on 6. If y = f (x) be given by x = _____  ​, y = ​ _____
​    2     ​ t > 0. If
1+t 1 + t2
(  ) np
(a) ​ n p, ​ ___ ​   ,​ n Œ I
2
f is increasing in (0, a), find the greatest value of a.

(b) ​( (2n + 1)p, (4n + 3) ​ __ ​  )​ n ŒI


p 7. Find the range values of ‘b’ so that for all real x,

2 x

f (x) = Ú​  ​  ​ (bt 2 + t sin t) dt is monotonic.


(c) ​( (4n – 1) ​ __ ​ ,  2 np )​, n ŒI
p
0
2
8. Find the interval of strictly increases for the function
(d) None.
f (x) = sin4x + cos4 x.
37. Let f ¢(x) = |x| – {x}, {,} = F.P.F, then f (x) is decreasing
on 9. Find the interval of increasing and decreasing for the
(  )
1
(a) ​ – ​ __ ​ , 0  ​
2 (  )1
(b) ​ – ​ __  ​, 2  ​
2
function f (x) = x e x (1 – x).
10. Find the interval of increasing and decreasing for the
(c) (0, 2)
1
(d) ​ __
2 (  )
​   ​ , •  ​. log(p  + x)
function f (x) = _________
​ 
log(e + x)

 ​.
38. Let f be a function such that f ¢(x) = log1/3 (sin x + a).
If f is decreasing for all real values of x, then the 11. Find the interval for which the function
range of a is f (x) = e  – ax + e ax , a > 0 is monotonically
(a) (1, 4) (b) (4, •) decreasing.
(c) (2, 3) (d) (2, •) 12. Find the set of values of a for which the function
39. The length of the largest interval in which the f (x) = x3 + (a + 2) x2 + 3ax + 5 is invertible.
function f (x) = 4x – tan 2x is monotonic is 13. If the function f (x) = 2x2 – k x + 5 is increasing on
[1, 2]. then find k.
7.8  Differential Calculus Booster

14. Find the interval of the increases or decreases for the 5. Let f ¢¢(x) > 0 for all x in R and g(x) = f (2 – x) + f (4 + x).
x Find the interval in which g (x) is increasing.
function f (x) = Ú​   ​ ​ (t 2 + 2t) (t 2 – 1) dt 6. Find the set of all real values of a for which

( 
___________

)
– 1
  – 4a –  
÷​ 21
f (x) = ​ 1 – ​ ____________
  
 ​  
a2 ​ 3
​ x + 5x + log e17 increases
ÔÏ xe
ax
: x£0 a+1
15. Let f (x) = Ì 2 3
,
ÔÓ x + ax - x : x > 0 for all real x in R.
7. Find the set of all real values of ‘a’ for which

( 
where a is a +ve const. Find the interval in which _____
f ¢(x) is inc.
x
​ a  + 4 ​
÷
f (x) = ​ ​  ______ 
1 – a

)
  ​ – 1  ​ x5 – 3x + log e5 decreases for

all values of x in R.
16. Let f (x) = ​Ú   ​ ​ |log2 (log3 (log4 (cos t + a)))| dt be
0 8. If f (x) = 2ex – a e– x + (2a + 1) x – 3 is monotonically
increasing for all real values of x, find a. increases for all x in R, then the set of all values
of a.
17. Let f (x) = x3 – 3x + a. If the equation f (x) = 0 has x
__
exactly one root, find the value of a. 9. Find the interval in which f (x) = ​Ú ​  ​ 2 ​÷2 ​ 
   sin2 t + (2
__ 0
18. Let f (x) = x3 – 3x + a. If the equation f (x) = 0 has
  )   sin t – 1) dt where 0 < x < 2p is increasing or
– ​÷2 ​
three real and distinct roots, find the value of a.
x
decreasing.  

19. Let f (x) = Ú​  ​  ​ {(a – 1)(t 2 + t + 1)2 – (a + 1)(t4 + t 2 + 1)} dt 10. Let f (x) = {– b2 + (a – 1) b – 2} x + ​Ú ​    ​ (sin2 x + cos4 x)
0  
dx, a, b Œ R
Then find the set of values of a for which f ¢(x) = 0
If f (x) be an increasing function, find all the permis-
has two distinct real roots.
sible values of a.
aex  + be– x
20. If y = f (x) = _________
​  x 
ce + de– x
​ is an increasing function of
  

x, then find a relation in a, b, c and d.


(  )
x2
11. Let g(x) = 2f ​ ​ __ ​   ​ + f (6 – x2) for all x in R and
2
f ¢¢(x) > 0, " x Œ R. Find the intervals of increase and
21. If a, b, c are real numbers, find the intervals in which decrease of g(x).

f (x) = ab
x + a2 ab
x+b 2
ac
bc
12. Let f (x) = ​ ___
÷
​ 3 ​ ( 
2__ – 1 ______2x  + 1
  ​  tan  ​ ​  __ ​  
     
​÷3 ​ )
 ​ – log (x2 + x + 1) + (b2

– 5b + 3) x + 4. If f (x) is a decreasing function for


ac bc x + c2 all real values of x in R, find the permissible values
is increasing or decreasing. of b.

(  x
)
13. If h(x) = f (x) + 2f  ​ 1  – ​ __  ​  ​, 0 < x < 1 and f ¢(x) > 0,
2
(Tougher Problems for JEE-Advanced) " x Œ (0, 1). Find the interval where h (x) is increasing
or decreasing where f (x) and h (x) are two differen-
1. Let f (x) = 1 – x – x 3. Find all real values of x tiable function.
satisfying the inequality 1 – f (x) – f 3 (x) > f (1
– 5x). 14. Let f ¢(sin x) < 0 and f ¢¢(sin x) > 0 for all x in ​ 0, __ (  )
p
​   ​   ​
2
2. A function f (x) is given by the equation x2 f ¢(x) + and g (x) = f (sin x) + f (cos x), then find the interval
2x f (x) – x + 1 = 0, where x π 0. where g (x) increases or decreases.
If f (1) = 0, then find the interval of the monotonocity 15. Let f ¢(x) < 0 for all x in R such that f ¢(3) = 0.
of the function f.
3. f (x) be a differentiable function such that
find the interval where g (x) is increasing.
p
If g (x) = f (tan2 x – 2tan x + 4) for all x in ​ 0, __
​   ​   ,​
2 (  )
f ¢(x) = log1/3 (log3 (sin x + a)). If f (x) is decreasing
for all real values of x, find a.
4. f (x) be a differentiable function such that
Integer Type Questions
1. The number of zeroes of the cur ve
1
f ¢(x) = __________________
​          ​. If f (x) be an increas- f (x) = x3 + 3x + m, m Œ R is...
log3 (log1/4 (cos x + b))
2. The number of real roots of the function
ing function, find b. f (x) = x3 – 3x + m, m Œ R is...
Monotonocity  7.9

3. If f (x) = sin2 x – 3 cos2 x + 2ax – 4 is increasing for


all x ≥ 0, then the least value of a is ... (  )
1  – x2
2. f (x) = cos– 1 ​ ______
​ 
1  + x2
 ​  
​ is strictly increasing in
4. If the number of real solutions of the equation
(a) (0, •) (b) (– •, •)
x3 + 2x2 + 5x + 2 cos x = 0 in [0, 2p] is m, then the
value of (m + 3) is ... (c) (– 2, 2) (d) (– •, 0)
5. The number of critical points of the function –​  (x – x 2)
3. f (x) = e​ ​ is strictly increasing in
f (x) = |x2 – 2x| is ...
(a) (1, •) (b) (– •, 0)
x3
6. If the function f (x) = ​ __ ​  + (m – 1) x2 + (m + 5) x + 7 (c) (0, 1) (d) (– 2, 0)
3
is increasing for all x > 0, then find the integral
values of m. Passage III
7. Let f (x) = |x| + |x2 – 1|. Then the number of real A critical point is a point where either f ¢(x) = 0 or f ¢(x) does
solutions of 6 f (x) = 7 is ... not exist, whereas a stationary point is a point where f is
x2 differentiable and f ¢(x) = 0.
8. If m is the number of real roots of ke x = 5 + x + ​ __ ​ ,
2 1. The critical points of the function f (x) = x2/3 (2 – x)
k Œ R + then the value of (m + 3) is ... is/are
9. If f (x) = e2x – (a + 1) ex + 2x is increasing for all (a) 1 (b) 2
real values of x in R, then find the maximum value
of a. (c) 4/5 (d) 0
10. If m is the number of solution of the equation 2. The critical points of the function f (x) = x + cos– 1 x

(  )
p + 1 is/are
3 tan x + x3 = 2, in ​ 0, __ ​   ​   ​ and n is the number of
4 (a) 1 (b) 2
inflection points of f (x) = 3x4 – 4x3, then the value (c) –1 (d) 0
of (m + n + 2) is ... 3. The critical points of the function f (x) = max {sin x,
cos x}, " x Œ (0, 2p) is/are
Comprehensive Link Passages
p
(For JEE-Advanced Exam) (a) 0 (b) ​ __ ​ 
4
Passage I p 5p
(c) ​ __ ​   (d) ​ ___ ​ 
If f is strictly increasing in (a, b) and g is strictly increasing 2 2
in ( f (a), f (b)), then (g0  f ) is strictly increasing in (a, b).
Passage IV
1. f (x) = tan– 1 (sin x + cos x)3 is strictly inc. in Let f (x) be a real function and a, b Œ R. If
(  )
(a) ​ 0, __
p
​   ​   ​
4 (  )
p p
(b) ​ __
​   ​ , __
4 2
​   ​   ​ (i) f (x) is continuous in [a, b]

( 
(ii) f (x) is differentiable in (a, b)

p 3p
2 4 )
(c) ​ ​ __ ​ , ___
​   ​   ​ (d) (p, 2p)
(iii) f (a) and f (b) are of opposite signs
sin x + cos x
2. f (x) = e is strictly increasing in (iv) either f ¢(x) > 0 on (a, b) or f ¢(x) < 0 on (a, b)
(  ) p
(a) ​ 0, ​ __ ​   ​
4 (  )
p p
(b) ​ __
​   ​ , __
4 2
​   ​   ​ then the function f (x) has exactly one root in [a, b]

(  )
p
(c) ​ __
​   ​ , p  ​
2 (  )
(d) ​ 0, __
p
​   ​   ​
2


1. The equation x5 – 3x – 1 = 0 has a unique root in
(a) [0, 1] (b) [1, 2]
3. f (x) = tan– 1 (log2 x) is strictly increasing in
(c) [– 1, 0] (d) [2, 3]
(a) (– •, 0) (  p
2 )
(b) ​ – ​ __ ​ , 0  ​
4 2
2. The equation x + 2x – 2 = 0 has a unique root
(c) (0, •) (d) (– •, •) in
Passage II (a) [0, 1] (b) [– 1, 1]
If f is strictly decreasing in (a, b) and g is strictly decreasing (c) [1, 2] (d) [– 2, – 1]
in ( f (a), f (b)), then (g0 f) is strictly decreasing in (a, b). x
3. The equation xe – 2 = 0 has exactly one root in

(  )
– 1
1. f (x) = cot  ​ ​log​__1 ​ x  ​ is strictly increasing in (a) [–1, 0] (b) [1, 2]
​    ​

[  ]
2
1
(a) (– •, 0) (b) (– •, •) (c) [0, 1] (d) ​ 0, __
​    ​  ​
2
(c) (0, •) (d) None
7.10  Differential Calculus Booster

Passage V Matrix Match


A point c in the domain of a function f (x) is said to be a (For JEE-Advanced Exam)
hyper-critical point at which f ¢¢(c) = or f ¢¢(c) does not exist.
1. Match the following columns
1. The hyper-critical point of the function f (x) = x5 +
5x – 6 is The number of critical points of the function
(a) 1 (b) 2 Column I Column II
(c) 0 (d) –1 – x
(A) f (x) = |x| e is (P) 2
2. The hyper-critical point of the function f (x) = sin x 2
– cos x in [0, 2p] is (B) f (x) = (ax + bx + c) |x| where (Q) 3
a c < 0, is
p 3p
(a) ​ __ ​   (b) ​ ___ ​  (C) f (x) = (1 – x) |x – 2| is (R) 1
4 4
2
5p 7p (D) f (x) = x – 2 |x| is (S) 0
(c) ​ ___ ​   (d) ​ ___ ​ 
4 4
3. The hyper-critical point of the function f (x) = 2 – |x5 2. Match the following columns
– 1| is The function f (x) is increasing in
(a) 1 (b) 0
(c) – 1 (d) 2 Column I Column II
x
(A) f (x) = x – e (P) (– •, 1)
Passage VI
An inflection point is point on a curve at which the curve
changes from concave down-ward to concave upward and
2
(B) f (x) = 2x – log x
_____
(Q)
(  )
1
​ __
​   ​ , •  ​
2
vice-versa. (C) f (x) = log  ​( x  + ​÷x  2 + 1 ​ 
  )​ (R) (– •, 0)
1. The inflection point of the function f (x) = 4x4 – x3 (D) f (x) = 5x3/2 – 3x5/2, x > 0 (S) (– •, •)
+ 2 is
(a) (0, 1) (b) (0, 1)
3. Match the following columns
(c) (– 1, 1) (d) (– 2, 0)
Let f (x) = |x + 1| {|x| + |x – 1|}. Then
2. The inflection point of the function f (x) = – x3 + 3x2
+ 1 is
Column I Column II
(a) (1, 3) (b) (0, 1)
(A) f (x) is increasing in (P) (– 2, – 1)
(c) (1, 2) (d) (– 1, 0)
3. The inflection point of the curve f (x) = x4 – 6x2 + (B) f (x) is decreasing in (Q) (– •, •)
8x + 10 is (C) f (x) is continuous in (R) (0, 1)
(a) (0, 1) (b) (1, 2) (D) The number of non-differ- (S) 3
(c) (– 1, – 3) (d) (2, 3) entiable points is
Passage VII (T) 4
If y = f ¢(x) is increasing, then y = f (x) is concave up and if
y = f ¢(x) is decreasing then y = f (x) is concave down. 4. Match the following columns
_____
1. The function f (x) = x1/3 + 2 is concave up in { 
Let f (x) = max ​ sgn (x), – ​÷9  – x2 
 ​, x3  ​ Then }
(a) (0, •) (b) (0, 1)
(c) (– •, 0) (d) (1, 2) Column I Column II
4 2
2. The function f (x) = x – 6x + x – 3 is concave down (A) f (x) is increasing in (P) 1
in (B) The number of (Q) 4
(a) (– •, 0) (b) (– 1, 1) discontinuous points is
(c) (1, •) (d) (1, 2) (C) The number of non- (R) (1, •)
3. The function f (x) = sin x, " x Œ [0, 2p] is concave differentiable points is
up in __
(D) f (x) is decreasing in (S) ​( – 3, – 2 ​÷2 ​ 
   )​
(  ) p
(a) ​ 0, __
​   ​   ​
2
p
(  )
(b) ​ __
​   ​ , p  ​
2 (T) (– 1, 0)
(c) (p, 2p) (d) (0, p)
Monotonocity  7.11

5. Match the following columns Column I Column II


ÔÏ x – x + x + 1 : 0 £ x £ 1
3 2 (A) f (x) is increasing in (P) (0, •)
Let f (x) = Ì
ÔÓ3 – x : 1< x £ 2 (B) g (x) is increasing in (Q) (– •, 0)

Then (C) f (x) is decreasing in (R) (– 1, 1)


(D) g (x) is decreasing in (S) (1, •)
Column I Column II
(A) f (x) is increasing in (P) (1, 2) 9. Match the following columns
(B) f (x) is decreasing in (Q) (0, 1)
Let f (x) = |x – 1| + |x – 2| + |x – 3| + |x – 4|
(C) f (x) is continuous in (R) (0, 1) » (1, 2) and a, b, c, d Œ I +. Then
(D) f (x) is differentiable in (S) [0, 2]
Column I Column II
6. Match the following columns (A) If f (x) is increasing in [a. b] (P) 3
Let f (x) = sin (|x|) + |x| and then (a + b) is
g (x) = cos (|x|) + |x|. Then (B) If f (x) is decreasing in [c, d] (Q) 0
then (c + d) is
(C) The number of discontinuous (R)
Column I Column II 7
points is
(A) f (x) is increasing in (P) (– •, 0) (D) The number of non- (S) 4
differentiable points is
(B) f (x) is decreasing in (Q) (0, •)
(C) The number of disc. Points of (R) 2 10. Match the following columns
f (x) and g (x) is x
Let f (x) = _____  ​, g (x) = tan– 1 x – x. Then
​  2    
(D) The number of non- (S) 0 x +1
differentiable points of f (x) is Column I Column II
(T) 1 (A) f (x) is increasing in (P) (– •, – 1]
(B) g (x) is increasing in (Q) [– 1, 1]
7. Match the following columns
(C) f (x) is decreasing in (R) [1, •)
p
Let f (x) = {1, cos x, 1 – sin x}, x Œ ​ – ​__ [ 
   ​, p  ​
2 ] (D) g (x) is decreasing in (S) [0, •)
Then
Question asked in Previous Years’
Column I Column II JEE-Advanced Examinations
(A) f (x) is increasing in (P) [0, p]
1. Use the function f (x) = x1/x, x > 0 to determine the
(B) f (x) is decreasing in (Q) 0 bigger of the two numbers e p and p c.
[IIT-JEE, 1981]
(C) The number of non-
differentiable points is
(R)
[  p
]
​ – ​ __ ​ , 0  ​
2
b
2. If ax2 + ​ __
x ​ ≥ c for all +ve x where a > 0 and b > 0,
show that 27ab2 ≥ 4c3 [IIT-JEE, 1982]
(D) The number of discontinuous (S) 2
_____ _____
points is 3. Show that 1 + x log ​( x + ​÷x  2 + 1 ​ 
  )​ ≥ ​÷1  + x2 
 ​ for all
x ≥ 0 [IIT-JEE, 1983]
8. Match the following columns p
– ​ __  ​

( 
4. The larger of cos(ln q) and ln (cos q) if ​e​ 2 ​ <
Let
2x
f (x) = sin– 1 ​ ​ _____
  
1 + x2 )
 ​  ​ p
q < __
​   ​  ...
2
[IIT-JEE, 1983]

and (  )
1  – x2
g (x) = cos– 1 ​ ​ ______2 
1+x
 ​  ​. Then
5. The function y = 2x2 – ln |x| is monotonically increasing
for values of x π 0 satisfying the inequalities ... and
7.12  Differential Calculus Booster

monotonically decreasing for values of x satisfying ÏÔ xe ax : x£0


the inequalities ... [IIT-JEE, 1983] 15. Let f (x) = Ì
2 3
6. No questions asked in between 1984 to 1986. ÔÓ x + ax – x : x > 0
7. Let f and g be increasing and decreasing functions where a is a +ve constant. Find the interval in which
respectively from [0, •) to [0, •). Let h(x) = f (g(x)). f ¢(x) is increasing.
If h(0) = 0, then h(x) – h(1) is
[IIT-JEE, 1996]
(a) always zero (b) always – ve
x x
(c) always +ve (d) strictly inc. 16. If f (x) = ​ ____ ​ and g(x) = ​ ____
     ​ where 0 < x £ 1,
    
sin x tan x
[IIT-JEE, 1987] then
8. The set of all x for which (1 + x) £ x is equal to ...
(a) both f (x) and g(x) are inc. functions
[IIT-JEE, 1987]
(b) both f (x) and g (x) are dec. functions
9. No questions asked in between 1988 to 1992.
(c) f (x) is an inc. function.
p
10. Show that 2 sin x + tan x ≥ 3x, where 0 £ x £ __
​   ​  (d) g (x) is an inc. function.
2 [IIT-JEE, 1997]
[IIT-JEE, 1993]
17. No questions asked in 1998.
Ï3 x 3 – 12 x – 1 : - 1 £ x £ 2
11. If f (x) = ÔÌ 18. The function f (x) = sin4 x + cos4 x increases if
ÔÓ37 – x : 2< x£3
p p 3p
then f (x) is (a) 0 < x < __
​   ​   (b) ​ __ ​  < x < ___
​   ​ 
8 4 8
(a) increasing in [– 1, 2]
3p 5p 5p 3p
(b) continuous in [– 1, 3] (c) ​ ___ ​ < x < ___
​   ​   (d) ​ ___ ​ < x < ___
​   ​ 
8 8 8 5
(c) greatest at x = 2
(d) all [IIT-JEE, 1999]
[IIT-JEE, 1993] 19. Consider the following statements S and R.
2 3
12. Let h(x) = f (x) – ( f (x)) + ( f (x)) for all x in R. S : Both sin x and cos x are decreasing functions in
Then
(a) h is inc. whenever f is inc.
p
the interval ​ __ (  )
​   ​ , p  ​
2
(b) h is inc. whenever f is dec. R: If a differentiable function decreases in an interval
(c) h is dec. whenever f is dec. (a, b), then its derivative also decrease in (a, b)
(d) nothing can be said in general. Which of the following is true.
[IIT-JEE, 1993] (a) both S and R are wrong.
13. The function f is defined by f (x) = (x + 2) e– x is (b) both S and R are correct but R is not the correct
(a) decreasing for all x explanation for S.
(b) decreasing in (– •, – 1) and increasing in (c) S is correct and R is the correct explanation for S.
(– 1, •) (d) S is correct and R is wrong.
(c) increasing for all x.   [IIT-JEE, 2000]
(d) decreasing in (– 1, •) and increasing in
20. Let f (x) = ​Ú ​  ​ ex (x – 1) (x – 2) dx. Then f decrease in
(– •, – 1)  
[IIT-JEE, 1994] the interval
ln (x  + p) (a) (– •, – 2) (b) (– 2, – 1)
14. The function f (x) = _________
​   ​
  is (c) (1, 2) (d) (2, •)
ln (x + e)
(a) increasing on [0, •) [IIT-JEE, 2000]
x (1 – x)
(b) decreasing on [0, •) 21. If f (x) = xe , then f (x) is
(a) increasing on [– 1/2, 1]
(  )
p
(c) increasing on ​ 0, __
​ e ​  ​ and decreasing on ​ __
p
(  )
​ e ​, •  ​ (b) decreasing on R.

(  )
(d) decreasing on ​ 0, __
p p
( 
​ e ​  ​ and increasing on ​ __ )
​ e ​, •  ​


(c) increasing on R.
(d) decreasing on [– 1/2, 1]
[IIT-JEE, 1995] [IIT-JEE, 2001]
Monotonocity  7.13

22. The length of the longest interval in which the


function f (x) = 3 sin x – 4 sin3 x is increasing is
(  )
3 1
(c) ​ – ​ __ ​ , – ​ __  ​  ​
4 2

p
(a) ​ __ ​  
3
p
(b) ​ __ ​ 
2
(  ) 1
(d) ​ 0, ​ __  ​  ​
4
3p (ii) The area bounded by the curve y = f (x) and the lines
(c) ​ ___ ​   (d) p
2 x = 0, y = 0 and x = t lies in the interval

(  )
[IIT-JEE, 2002] 3
(a) ​ __
​   ​ , 3  ​
23. Using the relation 2 (1 – cos x) < x 2 , x π 0 or 4
p
otherwise, prove that sin (tan x) > x, " x Œ ​ 0, __ (  )
​   ​   ​
4 (b) ​( ___
64 16 )
21 11
​    ​, ___
​   ​  ​
[IIT-JEE, 2003]
(c) (9, 10)
dp (x)
24. If P (1) = 0 and _____
(  )
​      ​ > P (x) for all x ≥ 1, then prove
  21
dx (d) ​ 0, ___
​    ​  ​
that P (x) > 0 for all x > 1 64

[IIT-JEE, 2003] (iii) The function f ¢(x) is

p
[  ] ( 
3x (x + 1)
25. Prove that for x Œ ​ 0, ​ __ ​   ​, sin x + 2x ≥ ​ ​ ________
2 p     )
​  ​.

(  ) 1
4 (  )
1
(a) inc. in ​ – t, ​ __ ​   ​ and dec. in ​ – ​ __ ​ , t  ​
4

(b) dec. in (​  – t, – ​ __ ​  )​ and inc. in ​( – ​ __ ​ , t )​


Explain the identity if any used in the proof. 1 1

4 4
[IIT-JEE, 2003]
26. No questions asked in between 2004 to 2007. (c) inc. in (– t, t)

( p p
27. Let the function f : (– •, •) Æ ​ – ​ __ ​ , __
2 2 )
​   ​   ​ be given by (d) dec. in (– t, t)
[IIT-JEE, 2010]
p
g (u) = 2 tan– 1 (eu) – __
​   ​ . Then g is 30. If f (x) = x3/2 (3x – 10), x ≥ 0, then f (x) is increasing
2
in ...
(a) even and is strictly inc. in (0, •)
[IIT-JEE, 2011]
(b) odd and is strictly dec. in (– •, •)
31. Let f (x) = (1 – x)2 sin2 x + x 2 for all x in R and
(c) odd and is strictly inc. in (– •, •)

( 
x
(d) neither even nor odd, but is strictly inc. in
(– •, •).
[IIT-JEE, 2008]
1
2 (t – 1)
g (x) = Ú​  ​  ​ ​ ​ _______ ​
t+1

  )
– ln t  ​ f (t) dt for all x > 1. which

of the following is true?


1
28. For function f (x) = x cos ​ __ (  )
​ x ​  ​ x ≥ 1 (a) g is inc. on (1, •)
(a) for atleast one x in interval [1, •), f (x + 2) (b) g is dec. on (1, •)
– f (x) < 2 (c) g is inc. on (1, 2) and dec. on (2, •)
(d) g is dec. on (1, 2) and inc. on (2, •).
  ​  
(b) ​ lim 
( f (x)) = 1
x Æ •
[IIT-JEE, 2012]
(c) for all x in interval [1, •), f (x + 2) – f (x) > 2
(d) f ¢(x) is strictly decreasing in the interval [1, •). 32. No questions asked in 2013.
x
[IIT-JEE, 2009]
33. Let f : (0, •) Æ R be given by f (x) = Ú​   ​ ​e​ (
  ) 1
– ​ t + __ dt
​   ​   ​ __
t ​ ​   ​ ,
29. Consider the function 1/x
t
f (x) = 1 + 2x + 3x2 + 4x3. Let s be the sum of all then
distinct real roots of f (x) and let t = |s|. (a) f (x) is monotonic inc. on [1, •).
(i) The real number s lies in the interval
(b) f (x) is monotonic dec. on (0, 1).
(  )
1
(a) ​ – ​ __ ​ , 0  ​
4
1
(c) f (x) + f  ​ __ (  )
​ x ​  ​ = 0, for all x Œ(0, •).

(b) ​( – 1, – ​ __ ​  )​


3 (d) f (2x) is an odd function of x on R.

4
[IIT-JEE, 2014]
7.14  Differential Calculus Booster

Answers
LEVEL II Passage III: 1. (c, d) 2. (a, c, d) 3. (b, c, d)
1. (b) 2. (d) 3. (a) 4. (b) 5. (d) Passage IV: 1. (b) 2. (a) 3. (c)
6. (b) 7. (c) 8. (c) 9. (c) 10. (b) Passage V: 1. (c) 2. (a) 3. (b)
11. (c) 12. (c) 13. (b) 14. (c) 15. (a) Passage VI: 1. (b) 2. (a) 3. (c)
16. (b) 17. (b) 18. (a) 19. (a, d) 20. (c) Passage VII: 1. (c) 2. (b) 3. (c)
21. (a, c) 22. (a) 23. (b, c) 24. (a) 25. (c) MATRIX MATCH
26. (b) 27. (b) 28. (b) 29. (a) 30. (d)
31. (c) 32. (c) 33. (c) 34. (b) 35. (c) 1. (A)Æ(P), (B)Æ(Q), (C)Æ(P), (D)Æ(Q)
36. (d) 37. (a) 38. (b) 39. (b) 40. (d) 2. (A)Æ(R), (B)Æ(Q), (C)Æ(Q), (D)Æ(P)
41. (d) 3. (A)Æ(R), (B)Æ(P), (C)Æ(Q), (D)Æ(S)
4. (A)Æ(R,T), (B)Æ(P), (C)Æ(Q), (D)Æ(S)
INTEGER TYPE QUESTIONS
5. (A)Æ(Q), (B)Æ(P), (C)Æ(S), (D)Æ(R)
1. 1 2. 3 3. 2 4. 3 5. 3
6. (A)Æ(Q), (B)Æ(P), (C)Æ(S), (D)Æ(T)
6. 4 7. 6 8. 4 9. 3 10. 5
7. (A)Æ(R), (B)Æ(P), (C)Æ(S), (D)Æ(Q)
COMPREHENSIVE LINK PASSAGES 8. (A)Æ(R), (B)Æ(P), (C)Æ(S), (D)Æ(Q)
Passage I : 1. (a) 2. (b) 3. (c) 9. (A)Æ(R), (B)Æ(P), (C)Æ(Q), (D)Æ(S)
Passage II: 1. (c) 2. (d) 3. (c) 10. (A)Æ(Q), (B)Æ(T), (C)Æ(P, R), (D)Æ(P, Q, R, S)

Hints and solutions


3. Given f (x) = (x – 1)3 (x – 2)2
fi f ¢(x) = 3(x – 1)2 (x – 2)2 + 2 (x – 1)3 (x – 2)
1. Given f (x) = 2x3 – 12x2 + 18x + 5
= 6x2 – 24x + 18 = (x – 1)2 (x – 2) (3 (x – 2) + 2 (x – 1))

= 6(x2 – 4x + 3) = (x – 1)2 (x – 2) (5x – 7)


= 6(x2 – 4x + 3)
= 6(x – 1) (x – 3)

By the sign scheme, we can say that, f (x) is strictly

( 
7
)
increase in ​ – •, ​ __ ​   ​ » (2, •) and strictly decreases
5

(  )
By the sign scheme, f (x) is strictly increases in 7
(– •, 1) » (3, •) and strictly decreases in (1, 3) in ​ __
​   ​ , 2  ​
5
2. Given f (x) = 5 + 36x + 3x2 – 2x3 4. Given f (x) = 2x3 – 3x2 + 6x + 10
fi f ¢(x) = 36 + 6x – 6x2 fi f ¢(x) = 6x2 – 6x + 6
= – 6(x2 – x – 6) = 6(x2 – x + 2) > 0, for all x in R
= –6 (x – 3) (x + 2)
Thus, the function f (x) is strictly increases for all x
in R.
5. Given f (x) = 2x3 + 3x2 + 12x + 20
fi f ¢(x) = 6x2 + 6x + 12
By the sign scheme, we can say that, f (x) is strictly
increases in (– 2, 3) and strictly decreases in (– •, – 2) = 6(x2 + x + 2) > 0 for all x in R
» (3, •). Thus, f (x) is strictly increases in (– •, •)
Monotonocity  7.15
x 2 _____
6. Given f (x) = __
​    ​ + __​   ​ 10. Given f (x) = cot– 1 x – log ​( x + ​÷x  2 + 1 ​ 
)​
2 x  

1 2 ______ x2  – 4 1
f ¢(x) = 1 + _____
1
 ​ – ___________
fi f ¢(x) = __
​   ​  – __
​  2  ​  = ​  2 ​    fi ​    2  ​       ​
_____
2 x 2x 1+x (​  x + ÷​ x  2 + 1 ​  )​
_____
(x  – 2) (x + 2)
= ​ ____________ (​  x + ​÷x  2 + 1 ​  )​
 ​
       × ​ ____________
_____     
 ​
2x2 ​÷x  2 + 1 ​  
1 1
fi f ¢(x) = 1 + _____
​    2   ​ – ​ _______
   
_____  ​ > 0 " x Œ R
1+x ​÷x  + 1 ​ 
2

Thus, f (x) is strictly increases in (– •, •)


By the sign scheme, we can say that, f (x) is strictly
increases in (– •, – 2) » (2, •)and strictly decreases 11. Given f (x) = – x2 + mx + 1
in (– 2, 0) » (0, 2) fi f ¢(x) = – 2x + m
7. Given f (x) = 5x3/2 – 3x5/2, x > 0 Since f is strictly increasing, so f ¢(x) > 0
fi – 2x + m > 0
fi f ¢(x) = 15x1/2 – 15x3/2
__ __ fi m > 2x
= 15 ​÷x 
   ​ (1 – x) – 15​÷x 
  ​  (x – 1)
fi m > 2, " x Œ [1, 2]
Hence, the least value of m is 2.
12. Given f (x) = sin x – bx + c
By the sign scheme, we can say that, f (x) is strictly fi f ¢(x) = cos x – b
increases in (0, 1) and strictly decreases in (1, •) Since f is strictly decreasing, so f < 0
______ fi cos x – b < 0
8. Given f (x) = log ​( x + ​÷1  + x2  
 ​ )​
fi b > cos x

fi ​ 
1
f ¢(x) = ________________
(​  x + ÷​ x  + 1 ​  )​
2 (  1 _____
× 2x
     ​ ​ 1 + ​ ________    
)
 ​  ​
2 ​÷x  2 + 1 ​


Hence,
b>1
b Œ (1, •)

( 
13. Given f (x) = e2x – (a + 1)ex + 2x
fi ​ 
1
f ¢(x) = ________________
(​  x + ÷​ x  + 1 ​  
2 )​
x
     ​ ​ 1 + _______
​  _____
   
)
 ​  ​
÷​ x  + 1 ​ 
2 fi f ¢(x) = 2 e2x – (a + 1)ex + 2

= 2 (ex)2 – (a + 1)ex + 2

( 

)
_____
1 ​÷x  + 1 ​ 
2
 + x
= ___________
​       ​​ ​ __________
_____ _____  ​ 
    ​ Since f is strictly increasing, so f ¢(x) > 0
(​  x + ÷​ x  2 + 1 ​  
)​ ​÷x  2 + 1 ​ 
fi 2 (ex)2 – (a + 1)ex + 2 > 0
1 2
= _______
​  _____  ​ > 0 " x Œ R
    fi 2 ex – (a + 1)ex + __ ​  x   ​ > 0
÷​ x  + 1 ​ 
2 e

Thus, f (x) is strictly increases in (– •, •)


fi (  1
2 ​ ex + __ )
​  x   ​  ​ – (a + 1) > 0
e

9. Given
x
f (x) = ____
​     ​ 
log x
fi ( 
1
(a + 1) < 2 ​ ex + __
​  x   ​  ​
e )
fi (a + 1) < 2.2 = 4
log x  –  1
fi f ¢(x) = ________
​   ​ 

(log x)2 fi a<3
Hence, the value of a is Œ (– •, 3)

14. Given (  )
a2  – 1 3
f (x) = ​ ______
​   ​  
3
 ​ x + (a – 1) x2 + 2x + 1

(  )
By the sign scheme, we can say that, f (x) is strictly
a2  – 1 2
increases in (e, •) and strictly decreases in (0, e). fi f ¢(x) = 3 ​ ______
​   ​   ​ x + 2 (a – 1) x + 2
3
7.16  Differential Calculus Booster

fi f ¢(x) = (a2 – 1) x2 + 2 (a – 1) x + 2 Since x = – 3, 3 are not an interior point of the domain
of f, so x = – 3, 3 are not the critical point of f.
Since f is strictly increasing, so f ¢(x) > 0
Thus, the critical points of f are x = 1 and 9.
fi (a2 – 1) x2 + 2 (a – 1) x + 2 > 0
18. Given f (x) = x4/5 (x – 4)2
fi (a2 – 1) > 0 & 4 (a – 1)2 – 8 (a2 – 1) < 0
4
fi a2 > 1 & (a – 1)2 – 2 (a2 – 1) < 0 fi f ¢(x) = 2x4/5 (x – 4) + ​ ____
   ​ (x – 4)2
5x1/5
fi (a + 1)(a – 1) > 0 & (a + 3)(a – 1) > 0
10x (x  – 4) + 4 (x – 4)2
fi a Œ (– •, – 1) » (1, •) fi f ¢(x) = ​ ___________________
     ​   
5x1/5
and a Œ (– •, – 3) » (1, •)
(x  – 4) (10x + 4x – 16)
Hence, the values of a are a Œ (– •, – 3) » (1, •) fi f ¢(x) = ​ ___________________
     ​   
5x1/5
15. Given f (x) = (a + 2) x3 – 3a x2 + 9a x – 1 2 (x  – 4) (7x – 8)
fi f ¢(x) = ​ ______________
 ​
    
fi f ¢(x) = 3 (a + 2) x2 – 6a x + 9a 5x1/5
Since f is strictly decreasing for all x in R, so Also, Df = R
8
f ¢(x) < 0 Thus, the critical points of f are x = 0, __
​   ​ , 4
7
fi 3(a + 2) x2 – 6a x + 9a < 0 19. Given f (x) = x + cos– 1 x + 1
fi (a + 2)x2 – 2a x + 3a < 0 1_____
–1
fi f ¢(x) = ​ ______    
 ​
Thus, (a + 2) < 0 and 4a2 – 12a (a + 2) < 0 ÷​ 1  – x2 
 ​
fi a < – 2 and a2 – 3a (a + 2) < 0 1
Now, f ¢(x) = 0 fi 1 – ______
​  _____   
 ​ = 0
fi a < – 2 & a (a + 3) > 0 ÷​ 1  – x2 
 ​
fi a Œ (– •, – 2) & a Œ (– •, – 3) » (0, •) 1
fi ​ ______
_____   
 ​ =1
Thus, a Œ (– •, – 3) ÷​ 1  – x2 
 ​
ex fi x = 0.
16. Given f (x) = ​ _____
   ​
x–1 Also, Df = [– 1, 1]
x x x
(x –  1) e – e  ◊ 1 e  (x  – 2) Thus, the critical points of ‘f ’ is x = 0.
fi f ¢(x) = ​ ______________
  
2
   = ________
 ​ ​   ​ 
  ___________
(x – 1) (x – 1)2
20. Given ​ x  2 – 6x +  
f (x) = ÷ 15 ​
Also, Df = R – {1}
1
Since x = 1 is not an interior point in the domain of fi f ¢(x) = _____________
​  ___________
     ​ × (2x – 6)
f, so, x = 1 is not a critical point of f. 2​÷x    – 6x +  
2
15 ​
1
Thus, the critical point of f is x = 2. fi f ¢(x) = ______________
​  ___________
    ​ × 2(x – 3)
2 ​÷x  – 6x +  
2 
15 ​
5x2 – 18x + 45
17. Given f (x) = ​ _____________
  
 ​  (x – 3)
x2 – 9 fi f ¢(x) = ​ ____________
___________  
  ​
÷​ x    – 6x +  
2
15 ​
(x2 – 9) (10x – 18) – (5x2 – 18x + 45) ◊ 2x
fi  f ¢(x) = ​ _________________________________
      ​    Also Df = R
(x2 – 9)2
Thus, the critical points of f is x = 3.
18 (x2 – 10 + 9)
fi  f ¢(x) = ​ _____________
  
   ​ 21. Given f (x) = 2x2 –ln |x|
(x2 – 9)2
2
1 4x  – 1
18(x – 1)(x – 9) fi f ¢(x) = 4x – ​ __
x  ​ = _______
​  x    ​ 
fi  f ¢(x) = ​ _____________
  
   ​
(x2 – 9)2
(2x  – 1)(2x + 1)
= ​ ______________
x     .  ​
Also, Df = R – {– 3, 3}
Monotonocity  7.17

Thus f (x) is strictly decreases in (– 1, 1)


Now, f (– 1) = – 1 + 3 – 1 = 1 > 0
and f (1) = 1 – 3 – 1 = – 2 < 0
By the sign scheme for f ¢(x), we have, f (x) is increas-
Thus, the curve y = f (x) = x3 – 3x – 1 will cut the

[  ) [  )
1
ing in ​  – ​ 
​ 2 ​ ​2
1
​ __ ​ , 0  ​ ​» ​  ​ ​__ ​ , •  ​ ​and decreasing in ​  – •,



1
( 
– ​ __ ​   ​​ 
2 ​ ] x-axis exactly one point in (– 1, 1)

x2
f (x) = ex – 1 – x – ​ __ ​ 
» ​(  0,
25. Let
​ 2 ]​
1 2
​ __​   ​   ​ ​.
fi f ¢(x) = ex – 1 – x < 0, " x Œ (– 1, 1)
2
22. Given f (x) = |x – 1|/x
Thus, f (x) is strictly decreases in (– 1, 1)
Ïx –1 1 1
ÔÔ 2 = –
x x2
: x ≥1
1 1 __ 1 __ 1
fi f (x) = Ì
x Now, f (– 1) = ​ __ __
e ​ – 1 + 1 – ​ 2 ​  = ​ e ​ – ​ 2  ​ < 0
Ô –x=
1 1 1
– : x <1
ÓÔ x 2 x2 x and
1 5
f (1) = e – 1 – 1 – ​ __  ​ = e – __
​   ​  > 0
2 2
Ï 1 2 2–x
ÔÔ- x 2 + x 3 = x 3 : x ≥1 x2
Thus, the equation ex = 1 + x + ​ __ ​  has a real root
fi f ¢(x) = Ì 2
Ô– 2 + 1 = x – 2 : x < 1 in (– 1, 1)
ÔÓ x 3 x 2 x3 26. As we know that tan – 1 x & e x, both are strictly
increasing for all x in R.
Therefore f (x) = tan– 1 (ex) is strictly increasing for
all x in R.
27. As we know that tan– 1 x is strictly increasing for
By the sign scheme for the function f ¢(x), we have all x in R and (log1/3 x) is strictly decreasing for all
f (x) is increasing in (– •, 0) » [1, 2] and decreases x Œ R +.
in (0, 1] » [2, •).
Therefore, f (x) = tan– 1 (log1/3 x) is strictly decreasing
for all x Œ R.
2 2
23. Given f (x) = x2 ​e– x
​ /a ​, a > 0

(  )
28. As we know that cot– 1 x is strictly decreasing for all
2x
​ x​ ​/​a​ ​​ × ​ – ​ ___2 ​   ​
2 2 2 2
fi f ¢(x) = 2 x ​e– ​
​ x​ ​/a​ ​ ​​ + x2 ◊ ​e– ​ x in R and (log4 x) is increasing for all x Œ R +.
a Therefore, f (x) = cot– 1 (log4 x) is strictly decreasing
= 2 x ​e– ​
2 2

(  )
x2
​ x​ ​/​a​ ​​ ​ 1– ​ __2  ​  ​
a
for all x Œ R +.
29. As we know that (log1/10 x) is strictly decreasing

( 
for all x Œ R + and (cot– 1 x) is strictly decreasing for
= – 2 x ​e– ​
2 2 (x  – a) (x + a)
​ x​ ​/​a​ ​​ ​ ​ ____________
a2
 ​ 
     ​ ) all x.
Thus f (x) = cot– 1 (log1/10 x) is strictly increasing for

(  )2 all x > 0.
= ​ – ​ __2  ​   ​ ​e​– ​x​ ​/​a​ ​​ × x(x – a)(x + a)
2 2
__
a 30. Let f (x) = 3x – x2 and g (x) = ÷
​ x 
  ​ 

Now, f ¢(x) = 3 – 2x
3
By the sign scheme, f is strictly inc. in ​ – •, __
​   ​   ​ and
2 (  )
Now, by the sign scheme for the function f ¢(x), we
have f (x) is increases in (– •, – a] » [0, a]
3
2 (  )
strictly decreasing in ​ ​ __ ​ , •  ​.

Also, g is srictly increasing in [0, •).


24. Given x3 = 3x + 1
Now, Df = [0, 3]
______
3
Let f (x) = x – 3x – 1   – x2 
Thus, the function y = ​÷3x  ​ is strictly increasing
fi f ¢(x) = 3x2 – 3 = 3(x2 – 1)
(  )3
in ​ 0, __
3
​   ​   ​ and strictly decreasing in ​ __
2 (  )
​   ​ , 2  ​.
2
2
fi f ¢(x) = 3 (x – 1) < 0, " x Œ (– 1, 1)
7.18  Differential Calculus Booster

31. Let g (x) = tan– 1 and h (x) = (sin x + cos x) p 5p


fi 2np + __
​   ​  < x < 2 p + ___
​   ​ 
4 4
Now, h¢(x) = (cos x – sin x)
p 5p
( 
fi 2np + __
​   ​  < log x < 2np + ___
​   ​ 
)
__ 1 1
=÷     ___
​ 2 ​ ​ ​  __  ​  cos x – ___
​  __  ​  sin x  ​ 4 4
÷   
​ 2 ​ ÷   
​ 2 ​
fi ​e​(
  p
)
​ 2np + ​ __ ​   ​ (  5p
​ 2np + ​ ___ ​   ​ )
( 
__ p
)
4 ​< x < ​e​ 4 ​
=÷   c  os ​ x + __
​ 2 ​ ​   ​   ​
4
34. Let g(x) = loge x and h(x) = cos x
h(x) is strictly increasing if h¢(x) > 0
Here, g (x) is strictly increases for all x > 0
( 
__ p
fi ​÷ 2 ​  cos ​ x + __ ​   ​   ​ > 0
4 ) Also, h (x) is strictly decreases in (0, p)

(  )
p Again, for the domain of the function, cos x > 0
fi cos ​ x + __
​   ​   ​ > 0
4

fi 0 < ​( x + __
p p
​   ​  )​ < __ ​   ​ < (​  x + __
3p
​   ​  & ___
p
​   ​  )​ < 2p
fi (  )
p
x Œ ​ 0, ​ __ ​   ​
2
4 2 2 4
Therefore, the function f (x) is strictly decreases in​

(  )
p p 5p 7p p
fi – ​ __ ​  < x < __
​   ​  and ___
​   ​ < x < ___
​   ​  0, __
​   ​   ​.
4 4 4 4 2
Thus, the given function f (x) is strictly increases in​ 35. Let g (x) = sin x + cos x and h (x) = sin x
p 3p
__
4 4 (  ) (  )
​   ​ , ___
5p 7p
​   ​   ​ » ​ ___
​   ​ , ___
4 4
p
​   ​   ​ and strictly decreasing in ​ 0, __ (  )
​   ​   ​ Now,
4
g¢(x) = cos x – sin x

» ​( ___ ​   ​   ​ » ​( ___ ( 


__ p
4 4 )
3p 5p
​   ​ , ___ ​   ​ , 2p )​.
7p
4
    cos ​ x + __
= ​÷2 ​ ​   ​   ​
4 )
( 
__ p
32. Let
log x
g (x) = ____
​  x   
 ​ and h (x) = log x
When    cos ​ x + ​ __ ​   ​ > 0
g¢(x) > 0. ​÷2 ​ 
4 )
Now,
1 – log x
g¢ (x) = ​ _______
 ​ 


p
– ​ __ ​  <
2 (​  x + __​ p4 ​  )​ < __​ p2 ​ 
x2 p p p p
fi – ​ __ ​  – __
​   ​  < x < __
​   ​  – __
​   ​ 
By the sign scheme, g (x) is strictly increasing in 2 4 2 4
(0, e) and strictly decreasing in (e, •) 3p p
fi – ​ ___ ​ < x < __​   ​ 
Also, h (x) is strictly increasing for all x > 0. 2 4
Also, Df = (1, •) When g¢(x) < 0, __
p
2
p
​   ​  < ​ x + __ (  3p
​   ​   ​ < ___
4
​   ​ 
2 )
Thus, f (x) is strictly increasing in (1, e)
p p 3p p
and strictly decreasing in (e, •) fi ​ __ ​  – __
​   ​  < x < ___
​   ​ – __
​   ​ 
2 4 2 4
33.
Let g(x) = sin x + cos x and h(x) = log x
p 5p
Since h(x) is an increasing function, f (x) will be fi ​ __ ​  < x < ___
​   ​ 
4 4
decreases if g(x) decreases.
Now, g¢(x) = cos x – sin x
p
Thus, f (x) is strictly increases in ​ 0, ​ __ ​   ​ and strictly
2 (  )
(  (  )
p
) decreasing in ​ __
__ 1 1
    ___
= ​÷2 ​ ​ ​  __  ​  cos x – ___
​  __  ​  sin x  ​ ​   ​ , p  ​.
4
÷   
​ 2 ​ ÷   
​ 2 ​
x2
36. Let f (x) = log (1 + x) – x + ​ __ ​ 
(  ( 
__
= ​÷2 ​ ​
p
    cos ​ x + __
​   ​   ​  ​
4 )) 1
2

fi f ¢(x) = _____​       ​ – 1 + x
Since g(x) decreases, so g ¢(x) < 0 1+x
1
f ¢(x) = _____
fi (  p
cos ​ x + __
​   ​   ​ < 0
4 ) fi ​     
1+x
 ​ – (1 – x)

( 
p p 1  –  1 + x2 x2
fi 2np + __
​   ​  < ​ x + __
2 4
3p
)
​   ​   ​ < 2np + ___
​   ​ 
2
fi f ¢(x) = ​ _________
1+x
  
   ​= ​ _____
x+1
   ​ > 0 " x Œ R +
Monotonocity  7.19
_____ _____
fi f (x) is strictly increasing in (0, •) 40. Let f (x) = 1 + x log ​( x + ​÷x  2 + 1 ​  
)​ – ​÷1  + x2 
 ​
Thus

( 
f (x) > f (0) _____


x2
log (1 + x) – x + ​ __ ​  > 0
2
f ¢ (x) = log ​( x + ÷   )​ + ​ __________
​ x  2 + 1 ​  ​ 
x_____
     ​  ​
​ x  2 + 1 ​
x  + ÷   )

x2
log (1 + x) > x – ​ __ ​ 
2
x
​ 1 + ​ _______
   
_____
(  x
 ​  ​ – ​ _______
   
_____
÷​ x  + 1 ​  ​÷1  + x2 
2 )
 ​
 ​
Hence, the result. _____
f ¢ (x) = log ​( x + ÷ )​ + _______
x
x ​ x  2 + 1 ​   ​  _____
     ​
37. Consider f (x)= log (1 + x) – _____
​       ​ ÷​ 1  + x2 
 ​
x+1
x
1 1 – _______
​  _____
     ​
fi f ¢(x) = _____
​      ​ – _______
​      ​ ÷​ 1  + x2   ​
1 + x (x + 1)2 _____
x +  1 – 1 _______ x f ¢ (x) = log ​( x + ÷   )​
​ x  2 + 1 ​ 
fi f ¢(x) = ​ ________  ​ 
 = ​       ​

2
(x + 1) (x + 1)2 _____
f ¢ (x) = log ​( x + ÷ )​ ≥ 0, "x ≥ 0
​ x  2 + 1 ​ 

fi f ¢(x) > 0 for all x > 0
Thus, f (x) is increasing in [0, •)
Thus, f (x) is strictly increasing
f (x) ≥ f (0)
fi f (x) > f (0) _____ _____
x 1 + x log ​( x + ​÷x  2 + 1 ​ 
  )​ –  ​÷x  2 + 1 ​ ≥ 0
fi log (1 + x) – _____
​      ​ > 0
x+1 _____ _____
x fi 1 + x log ​( x + ​÷x  2 + 1 ​ 
  )​ ≥ ​÷x  2 + 1 ​ 
fi log (1 + x) > _____
​     
 ​ for all x > 0
x+1 Hence, the result.
Hence, the result. 41. Let f (x) = x – sin x
38. Let f (x) = (1 + x) log (1 + x) – ex + 1
(1 + x)
fi (  )
p
f ¢ (x) = 1 – cos x > 0, " x Œ ​ 0, ​ __ ​   ​
2
f ¢(x) = ​ ______ 
(  )
fi  ​ + log(1 + x) ◊ 1 – ex p
(1 + x) Thus, f (x) is strictly increasing in ​ 0, __
​   ​   ​
2
fi f ¢(x) = 1 + log(1 + x)– ex
fi f (x) > f (0)
fi f ¢(x) < 0 for all x < 0 fi x – sin x > 0
Thus, f (x) is strictly decreasing function
fi x > sin x
fi f ¢(x) < f (0)
fi cos x < cos (sin x) ...(i)
fi (1 + x) log (1 + x) – ex + 1 < 0

fi (1 + x) log (1 + x) < e –1 x
p
(  )
Also, for all x in ​ 0, ​ __ ​   ​, 0 < cos x < 1
2
fi cos x < 1
Hence, the result.
fi cos x > sin (cos x) ...(ii)
39. We have f (x) = 2x tan– 1 x – log (1 + x2)
From (i) and (ii), we get,
2x 2x
fi f ¢ (x) = 2 tan  x + ​ ______
– 1
 2  ​ – ​ ______
    ​ sin (cos x) < cos x < cos (sin x)
1  + x 1  + x2
42. Given f (x) = log x – Bx2
fi f ¢ (x) = 2 tan– 1 x > 0 for all x in R +
1 1 – 2Bx2
Thus, f (x) is strictly increasing in R + fi f ¢ (x) = __
​ x ​ – 2Bx = ​ ________
x   ​ 

fi f (x) > f (0) The critical points of ‘f  ’ are

fi 2x tan– 1 x – log (1 + x2) > 0


1
x = 0, ____
​  ___
1
, – ____
   ​  ​  ___
   ​ 
÷    ​ ​÷2B 
​ 2B     ​
fi 2x tan –1 x > log (1 + x2)
Hence, the result. Now, (  1
f ¢ (x) > 0, "x Œ ​ 0, ____
​  ___
÷ )
   ​  ​
   ​
​ 2B 
7.20  Differential Calculus Booster

and
1
f ¢ (x) < 0, "x Œ ​ ____
​  ___
÷
​ 2B 
, •  ​
   ​ 
   ​ (  ) fi f ¢¢ (x) = 20x3

fi f ≤¢ (x) = 60x2 is exists for all x


Now, log x < Bx2 for x > 0
Now, f ¢¢ (x) = 0 gives x = 0
1
It holds good for x = ____
​  ___
   ​ 
÷    ​
​ 2B  By the sign scheme for f ¢¢ (x) = 0, we have, f (x) is
concave down in (– •, 0) and concave up in (0, •).
Thus (  )1
log ​ ____
​  ___
÷    ​
​ 2B 
1
   ​  ​ < B, ___
2B 2
1
​     ​ = __
​   ​  45. We have f (x) = x4 – 5x3 – 15x2 + 30
___ 1 fi f ¢ (x) = 4x3 – 15x2 – 30x
fi    ​ )​ < __
– log ​( ​÷2B  ​   ​ 
2
___
fi f ¢¢ (x) = 12x2 – 30x – 30
1
fi log ​( ÷    ​ )​ < – ​ __ ​ 
​ 2B 
2 Now, f ¢¢ (x) = 0 gives 12x2 – 30x – 30 = 0
1
___ – ​ __ ​ 
fi ​÷ 2B  ​ < ​e​ 2 ​ fi 6x2 – 15x – 15 = 0
fi 2x2 – 5x – 5 = 0
fi 2B < e– 1 _______
5 ± ​÷25  + 40 ​  

1
B > ___
​     ​ fi x = ​  ___________
 ​    
2e 4
1 5 ± 8 ___ 13 3
Thus, the least value of B is ___
​     ​ fi x = ​ _____  = ​   ​ , – ​ __ ​ 
 ​ 
2e 4 4 4
b By the sign scheme for the function f ¢¢ (x), the func-
43. Let f (x) = a x2 + __
​ x ​ – c


b
f ¢ (x) = 2a x – ​ __2  ​ 
( 
3 13
tion f (x) is concave down in ​ – ​ __ ​ , ___ )
​   ​   ​ and concave
4 4
x
b
(  3
4 ) (  13
up in ​ – •, – ​ __  ​  ​ » ​ ___ )
​   ​ , •  ​
4
Now, f ¢ (x) = 0 gives 2a x – __
​  2  ​  = 0
x 46. We have f (x) = (sin x + cos x) ex
fi 2a x3 = b f ¢ (x) = (sin x + cos x) ex + ex (cos x – sin x)
f ¢ (x) = ex (sin x + cos x + cos x – sin x)
(  )

b 1/3
fi x = ​​ ___
​     ​  ​​ ​
2a f ¢ (x) = 2ex cos x
b
Thus, the least value of f (x) occurs at x = ​​ ___
​     ​  ​​ ​
2a (  )
1/3
f ≤(x) = 2 (ex cos x – ex sin x)

{  }
b 2/3 b f ≤ (x) = 2ex (cos x – sin x)
we have a ​​ ___
​     ​  ​​ ​ + _______
​   1/3 ​ ≥c
2a
{  } b
___
​​ ​     ​  ​​ ​
2a
Now,

f≤ (x) = 0 gives 2ex (cosx – sinx) = 0
tan x = 1
a ​( ___
​     ​ )​ + b ≥ c ◊ ​​( ___
​     ​ )​​ ​
b b 1/3
fi p 5p
2a 2a fi x = __
​   ​ , ___
​   ​ 
4 4

fi ​​( ___
​   ​  )​​ ​ ≥ ___
3b b 3
2 By the sign scheme for the function f ≤ (x) = 0, we
​     ​ ◊ c

27b
2
b
2a
3
have f (x) is concave down in ​ __
p 5p
(  )
​   ​ , ___
4 4
​   ​   ​ and concave

(  ) ( 
fi ​ ____  ≥ ___
​     ​ ◊ c2
8
 ​ 
2a
2 3
p
up in ​ 0, __
4
5p
​   ​   ​ » ​ ___ )
​   ​ , 2p  ​
4
fi 27b  a ≥ 4c
47. Given curve is y = f (x) = Ax2 + Bx + C
fi 27a b2 ≥ 4c3 f ¢ (x) = 2Ax + B
Hence, the result. f ≤ (x) = 2A
44. We have f (x) = x5 + 5x – 6 Thus, the curve concave up if f ≤ (x) > 0 and concave
down if f ≤ (x) < 0 i.e. concave up if A > 0 and con-
fi f ¢ (x) = 5x4 + 5 cave down if A < 0.
Monotonocity  7.21

f ≤ (x) = – 2​( ​e– ​ ​ x​ ​​◊ (– 2x) )​


2 2
48. We have f (x) = x4 – 4x3 + x – 10 fi ​ x​ ​​ ◊ 1 + x ◊ ​e– ​

fi f ¢ (x) = 4x3 – 12x2 + 1 fi f ≤ (x) = 2​e– ​


2
​ x​ ​​ (2x2 – 1)
fi f ≤ (x) = 12x2 – 24x = 12x (x – 2) 2
Now, f ≤ (x) = 0 gives 2​e– ​
​ x​ ​​ (2x2 – 1) = 0
Now, f ≤ (x) = 0 gives x = 0 and x = 2.
1__
fi x = ± ​ ___  ​ 
when x = 0, y = – 10 ÷   
​ 2 ​
when x = 2, y = – 24 1 – ​    ​
1
__
1
when x = ___
​  __  ​ , y = e​ ​ 2 ​ = ___
​  __  ​ 
Thus, the point of inflection are (0, 10) and ÷   
​ 2 ​ ​÷e 
  ​ 

(2, – 24) 1__ – ​    ​ 1


1
__

49. We have y = f (x) = (x – 2)2/3 + 10 when x = – ​ ___  ​ , y = ​e​ 2 ​ = ___


​  __  ​  .
÷   
​ 2 ​ ÷   
​ e ​
2 2
fi f ¢ (x) = __
​   ​  (x – 2) – 1/3 = _________
​       ​ Thus, the point of inflection are
3 3(x – 2)1/3
fi f ≤ (x) = _________
​ 
2
   
9 (x – 2) 4/3
 ​ ÷
1__ ___
​ – ​ ___
   ÷
​ 2 ​
1
​ e ( 
   ​ ÷
1 1__
  ​,  ​  __  ​  ​ & ​ ___
   ÷
​ 2 ​ ) ( 
​  __  ​ , ___
​    ​  ​
​ e 
  ​  )
Thus, f ≤ (x) does not exist at x = 2.
when x = 2, y = 0 + 10 = 10
1. Given f (x) = 4x – tan 2x
Thus, the inflection point is (2, 10)
f ¢ (x) = 4 – 2 sec2 x
50. We have f (x) = x4 – 6x3 + 12x2 – 8x + 3
Now, f ¢ (x) > 0 gives 4 – 2 sec2 2x > 0
fi f ¢ (x) = 4x3 – 18x2 + 24x – 8
fi sec2 2x < 2
fi f ≤ (x) = 12x2 – 36x + 24 __ __
2
fi ​( sec 2x + ​÷2 ​ 
   )​ ​( sec 2x – ​÷2 ​ 
   )​ < 0
= 12 (x – 3x + 2) __ __
fi     < sec 2x < ​÷2 ​
– ​÷2 ​   
= 12 (x – 1) (x – 2)
p p
Now, f ≤ (x) = 0 gives x = 1, 2 fi – ​ __ ​  < 2x < __​   ​ 
4 4
when x = 1, y = 2 p p
fi – ​ __ ​  < x < __
​   ​ 
when x = 2, y = 3 8 8
Thus, the length of the longest interval
Thus, the point of inflection are (1, 2) and (2, 3).

51. Given y = f (x) = x2 – ___


1
​   3 ​ 
6x

p
(  (  ) )
= ​ __
8
p
4
p
​   ​  – ​ – ​ __ ​   ​  ​ = __
​   ​ 
4

1 2. We have x3 + 2x2 + 5x + 2 cos x = 0


fi f ¢ (x) = 2x + ___
​   4  ​
2x Let f (x) = x3 + 2x2 + 5x + 2 cos x
2
fi f ≤ (x) = 2 – __
​  5  ​  fi f ¢ (x) = 3x2 + 4x + 5 – 2 sin x
x
2 Let g (x) = 3x2 + 4x + 5 and h (x) = 2 sin x
Now, f ≤ (x) = 0 gives 2 – ​ __5  ​  = 0
x 16 – 60 ___ 44 22
Thus, x=1 Max value of g (x) is – ​ _______ = ​   ​ = ___
 ​  ​   ​ and max
6 6 3
5
when x = 1, y = __
​   ​  value of h (x) is 2.
6
5
(  )
Thus, the point of inflection is ​ 1, ​ __  ​  ​.
6
Thus,

f ¢ (x) > 0
f (x) is strictly increasing function
2
52. We have y = f (x) = e​ – ​
​ x​ ​​
2
Also, f (0) = 2 > 0 and f (2p) > 0
fi f ¢ (x) = e​ – ​
​ x​ ​​ × – 2x
2 Therefore f (x) has no real root.
fi f ¢ (x) = – 2​e– ​
​ x​ ​​ × x
7.22  Differential Calculus Booster
x
3. We have
x
7. We have f (x) = Ú​  ​  ​  (bt 2 + t sin t) dt
0
f (x) = (ab – b2 – 2) x + Ú​  ​  ​  (cos4 q + sin4 q) dq
0 fi f ¢ (x) = bx2 + x sin x
fi f ¢ (x) = (ab – b2 – 2) + (cos4 x + sin4 x)
Since f (x) is a decreasing function, so f ¢ (x) £ 0
fi (  sin x
f ¢ (x) = x2 ​ b + ​ ____ )
​  ​
x   
Since f (x) is monotonic, so f ¢ (x) > 0 or f ¢ (x) < 0
fi (ab – b2 – 2) + (cos4 x + sin4 x) £ 0
fi (ab – b2 – 2) + 1 £ 0 fi (  sin x
x2 ​ b + ​ ____ ) ( sin x
​  ​ > 0 & x2 ​ b + ​ ____
x    )
​  ​ < 0
x   
2
fi (ab – b – 2) £ 0
fi 2
(b – ab +1) ≥ 0
sin x
fi ​ b + ​ ____ (  )
sin x
 ​  ​ > 0 & ​ b + ​ ____
x    ( 
 ​  ​ < 0
x    )
Therefore, b > 1
fi a2 – 4 £ 0
fi b Œ (– •, 1)
fi (a + 2) (a – 2) £ 0
fi – 2 £ a £ 2 8. We have f (x) = sin4 x + cos4 x

fi a Œ [– 2, 2] f ¢ (x) = 4 sin3 x ◊ cos x – 4 cos3 x ◊ sin x

4. Given f (x) = cos x + a2 x + b = – 4 sin x ◊ cos x (cos2 x – sin2 x)

fi f ¢ (x) = – sin x + a2 = – 2 (2 sin x ◊ cos x) (cos2 x – sin2 x)


Since f (x) is increasing, so f ¢ (x) ≥ 0 = – (2 sin 2x ◊ cos 2x)
fi a2 – sin x ≥ 0 = – sin 4x
fi a2 ≥ sin x
When f (x) is strictly increasing, then f ¢ (x) > 0
fi a2 ≥ 1
fi – sin 4x > 0
fi a2 –1 ≥
fi (a + 1) (a – 1) ≥ 0 fi sin 4x < 0
fi a £ – 1, a ≥ 1 fi p < 4x < 2p
fi a Œ (– •, – 1] » [1, •) p p
fi ​ __ ​  < x < __ ​   ​ 
2 2
5. Given f (x) = 2ex – ae– x + (2a + 1) x – 3

fi f ¢ (x) = 2ex + ae– x + (2a + 1)


Thus,
p p
x Œ ​ __
​   ​ , __
4 2 (  )
​   ​   ​

Since f (x) is increasing, so f ¢ (x) ≥ 0 9. We have f (x) = x ex (1 – x)

fi 2ex + ae– x + (2a + 1) ≥ 0 fi f ¢ (x) = ex (1 – x) + x ex (1 – x) × (1 – 2x)

fi 2 (ex)2 + (2a + 1) ex + a ≥ 0 fi f ¢ (x) = (1 + x (1 – 2x)) ex (1 – x)

Therefore, D £ 0 fi f ¢ (x) = (1 + x – 2x2)ex (1 – x)


fi (2a + 1)2 – 8a £ 0
= – (2x2 – x – 1)ex (1 – x)
2
fi 4a + 4a + 1 – 8a £ 0
2
= – (2x + 1) (x – 1)ex (1 – x)
fi 4a – 4a + 1 £ 0
By the sign scheme for f ¢ (x), the function f (x) is
fi (2a + 1)2 £ 0
fi (2a – 1)2 = 0
1
[  ] ( 1
)
increasing ​– ​ __ ​ , 1  ​ and decreasing in ​ – •, – ​ __  ​  ​ [1, •)
2 2
1
fi a = __
​   ​  log (p  + x)
2 10. Given f (x) = __________
​     ​ 
log (e + x)
6. ***
Monotonocity  7.23

1 1 fi a2 – 5a + 4 < 0
log (e + x)​ ______ – log (p + x) ​ _____
   ​      ​ 
(p + x) (e + x)
fi f ¢ (x) = ____________________________
​        ​ fi
    (a – 1) (a – 4) < 0
{log (e + x)}2
fi 1<a<4
(e + x) log (e + x) – (p + x) log (p + x) a Œ (1, 4)
= ​ ____________________________
          ​ Thus,
(e + x) (p + x){log (e + x)}2
13. Given f (x) = 2x2 – k x + 5
Since 1<e<p
fi f ¢ (x) = 4x – k
(1 + x) < (e + x) < (p + x) ...(i)
f ≤ (x) = 4
Also, log (e + x) < log (p + x) ...(ii)
Now, f ≤ (x) = 4 > 0 " x Œ [1, 2]
Multiplying (i) and (ii), we get,
Thus, f ¢ (1) is the least value of f (x).
(e + x) log (e + x) < (p + x) log (p + x) Now, f ¢ (1) > 0 gives 4 – k > 0
Thus, f ¢ (x) < 0 fi k<4
Therefore, f (x) is decreasing in (0, •). fi k Œ (– •, 4)
– ax ax x
11. Given, f (x) = e +e ,a>0
14. We have f (x) = Ú​   ​ ​  (t 2 + 2t) (t 2 – 1) dt
fi f ¢ (x) = – a e– ax + a eax –1
ax – ax
fi f ¢ (x) = a (e –e ) fi f ¢ (x) = (x2 + 2x) (x2 – 1)
Since f (x) is monotonic increasing, so f ¢ (x) > 0 fi f ¢ (x) = x (x + 2) (x + 1) (x – 1)
ax – ax
fi a (e –e )>0 By the sign scheme, we get, f (x) is strictly increasing

fi ( 
e  – 12ax
a ​ _______
​  ax  ​  
e
  ​> 0 ) in (– •, – 2) » (– 1, 0) » (1, •) and strictly decreas-
ing in (– 2, – 1) » (0, 1)
fi e2ax – 1 > 0 ÏÔ xe ax : x£0
2ax 0 15. Given f (x) = Ì
fi e >e 2 3
ÔÓ x + ax – x : x > 0
fi 2a x > 0
Clearly f is continuous at x = 0
fi x>0
Hence, the required interval (0, •). Ï(ax + 1)e ax : x<0
Ô
fi f ¢ (x) = Ì1 : x=0
12. We have f (x) = x3 + (a + 2) x2 + 3ax + 5 Ô 2
Ó1 + 2ax – 3 x : x>0
fi f ¢ (x) = 3x2 + 2 (a + 2) x + 3a
Ïa(ax + 2)e ax : x<0
Since f (x) is invertible, so f (x) is bijective function Ô
fi f ≤ (x) = Ì2 a : x=0
Therefore, f is one-one function Ô2 a – 6 x : x>0
Thus, we can conclude that f (x) is strictly increasing Ó
or strictly decreasing. Now, for x < 0, f ≤ (x) > 0
Here, the co-efficient of x2 is 3, so f (x) is strictly
increasing. fi ax + 2 > 0
2
Therefore, f ¢ (x) > 0 fi x > – ​ __
a ​

So,
3x2 + 2 (a + 2) x + 3a > 0
D<0
fi [ 
2
f ¢ (x) increases on ​ – ​ __ ]
a ​, 0  ​

fi 4 (a + 2)2 – 36a < 0 Also, x > 0, f ≤ (x) > 0

fi (a + 2)2 – 9a < 0 fi 2a – 6x > 0


2 a
fi a + 4a + 4 – 9a < 0 fi x > __
​    ​
3
7.24  Differential Calculus Booster

Thus,
a
f ¢ (x) increases on ​ 0, __ [  ]
​   ​   ​
3
fi  (a – 1 – a – 1) x2 + (a – 1 + a + 1) x + (a – 1 – a – 1) = 0

[  ]
fi  – 2x2 + 2ax – 2 = 0
2 __ a
Hence, f ¢ (x) increases on ​ – ​ __
a ​, ​ 3  ​  ​. fi  x2 – ax + 1 = 0
16. We have It has two distinct roots only when, D ≥ 0
x

f (x) = Ú​  ​  ​   ​| log2 (log3 (log4 (cos t + a))) |​ dt fi a2 – 4 ≥ 0


0
fi (a + 2) (a – 2) ≥ 0
f ¢ (x) = ​| log2 (log3 (log4 (cos x + a))) |​
fi a £ – 2, a ≥ 2
Since f (x) is increasing, so f (x) is defined for all x.
fi a Œ (– •, 2] » [2, •)
(log3 (log4 (cos x + a))) > 0
(log4 (cos x + a)) > 1 aex  + be– x
20. We have y = f (x) = _________
​  x       ​
(cos x + a) > 4 ce + de– x

a > 4 – cos x (cex  + de– x) (aex – be– x) – (cex – de – x) (aex + be– x)


= ​ _________________________________________
       ​
   
a > 5 for all x in R (cex + de– x)2
17. We have f (x) = x3 – 3x + a 2 (ad –  bc)
= ___________
​  x   
    ​
fi f ¢ (x) = 3x – 3 2 (ce + de– x)2
= 3 (x – 1) (x + 1) Since f (x) is increasing function, so f ¢ (x) ≥ 0
Now, f (x) = 0 will have only one root if 2 (ad –  bc)
fi ​ ___________
   ​ ≥ 0
  
f (1) f (– 1) > 0 (cex +  de– x)2

fi (a – 2) (a + 2) > 0 fi (ad – bc) ≥ 0


fi a < – 2, a > 2 fi ad ≥ bc
fi a Œ (– •, – 2) " (2, •) 21. We have
3
18. We have f (x) = x – 3x + a x + a2 ab ac
2
fi f ¢ (x) = 3x – 3 2
f (x) = ab x+b bc
Now, f ¢ (x) = 0 gives 3x2 – 3 = 0
ac bc x + c2
2
fi x –1=0
a( x + a 2 ) a 2 b a2c
fi x2 = 1 1
fi x = ± 1 = ____ ​ ab2
​       b( x + b2 ) b2 c
abc
It has three roots only when ac 2 bc 2 c( x + c 2 )
f (– 1) f (1) < 0
( x + a2 ) a2 a2
fi (a + 2) (a – 2) < 0 abc 2
= ​ ____ 
 ​ b ( x + b2 ) b2
fi – 2 < a < 2 abc
c2 c2 ( x + c2 )
19. We have
x
( x + a2 ) a2 a2
f (x) = Ú​  ​  ​  ​{ (a – 1) (t 2 + t + 1)2 – (a + 1) (t4 + t 2 + 1) }​ dt
2
0 = b ( x + b2 ) b2
fi  f ¢ (x) = {​  (a – 1) (x2 + x + 1)2 – (a + 1) (x4 + x2 + 1) }​ c2 c2 ( x + c2 )

|  |
Now, f ¢ (x) = 0
( x + a 2 + b2 + c 2 ) ( x + a 2 + b2 + c 2 ) ( x + a 2 + b2 + c 2 )
fi ​{ (a – 1) (x  + x + 1)  –  (a + 1) (x  +  x  + 1) }​ = 0
2 2 4 2
= ​ b2 ( x + b2 ) b2 ​

fi ​{ (a – 1) (x2 + x + 1) – (a + 1) (x2 –  x + 1) }​ = 0 c2 c2 ( x + c2 )  


Monotonocity  7.25

1 1 1 dy
Thus, x2 ​ ___  ​ + 2xy = x – 1
dx
= (x + a2 + b2 + c2)  b2 ( x + b2 ) b2
d
c2 c2 ( x + c2 ) fi ​ ___   ​ (x2 y) = x – 1
dx
1 0 0 On integration, we get,
2 2 2 2
= (x + a + b + c )  b x 0 x2
x2 y = ​ __ ​  – x + c
c2 0 x 2
when x = 1, y = 0 , then c = 1/2
2 2 2 2
= (x + a + b + c ) x
x2 1
Thus, x2 y = ​ __ ​  – x + __
​   ​ 
fi f (x) = (x + a2 + b2 + c2) x2 2 2
1 1 1
fi f ¢ (x) = 2 (x + a2 + b2 + c2) x + x2 fi y = __
​   ​  – __
​ x ​ + ___
​   2 
 ​
2 2x
fi f ¢ (x) = x {3x + 2 (a2 + b2 + c2)} dy 1 x  –  1
fi ​ ___  ​ =  ​1 2   ​ – __
​    ​  = _____
​  3 ​   
Now, f ¢ (x) = 0 gives dx ​x ​ x3 x

fi x {3x + 2 (a2 + b2 + c2)} = 0 So, by the sign scheme, f (x) is increasing in


x Œ (– •, 0) » (1, •) and decreasing in x Œ (0, 1)
2 (a2  + b2 + c2)
fi x = 0, – ​ _____________
 ​
     3. Since f (x) be a decreasing function, we have
3
Thus, f (x) increasing in log1/3 (log3 (sin x + a)) < 0

( 
(log3 (sin x + a)) > 1


2 (a2  + b2 + c2)
​ ​ – •, – ​ ______________
3
 ​ 
     ​  ​ » [0, •)
​ ] fi (sin x + a) >3

[ 
fi a > 3 – sin x

2 (a2  + b2 + c2
and decreasing in ​ – ​ _____________
3
 ​     , 0  ​ ] fi a > 4 for all x in R.
4. Since f (x) be an increasing function, we have
log3 (log1/4 (cos x + b)) > 0
(Tougher Problems for JEE-Advanced)
fi (log1/4 (cos x + b)) > 1
1. We have f (x) = 1 – x – x3 1
fi 0 < (cos x + b) < __ ​    ​ for all x in R.
4
fi f ¢ (x) = – 1 – 3x2 < 0, " x Œ R
1
f (x) is strictly decreasing fi – cos x < b < __ ​   ​  – cos x
4
Now, f ( f (x)) = 1 – f (x) – ( f (x))3 fi 1 < b < __
1
​   ​  – 1
4
fi f ( f (x)) > f (1 – 5x)
3
fi 1 < b < – ​ __ ​ 
fi f (x) < (1 – 5x) 4
which is not possible.
fi 1 – x – x3 < (1 – 5x)
So, b=j
fi x3 – 4x > 0
5. We have f ≤ (x) > 0
2
fi x (x – 4) > 0 fi f ¢ (x) is increasing for all x in R
fi x (x + 2) (x – 2) > 0 Also, g (x) = f (2 – x) + f (4 + x)
fi x Œ (– 2, 0) » (2, •) fi g ¢ (x) = – f ¢ (2 – x) + f ¢ (4 + x)

2. We have x2 f ¢ (x) + 2x f (x) – x + 1 = 0 Since g (x) is increasing, so g ¢ (x) > 0

Put y = f (x) fi – f ¢ (2 – x) + f ¢ (4 + x) > 0


7.26  Differential Calculus Booster

fi f ¢ (4 + x) > f ¢ (2 – x) fi – 7 £ a £ 3

fi (4 + x) > (2 – x) Thus, a Œ [2, 3] ...(i)

fi 2x > – 2 Case II: When a + 1 < 0

fi x > – 1 Then 21 – 4a – a2 ≥ 0

fi x Œ (– 1, •) fi a2 + 4a – 21 £ 0
fi (a + 7) (a – 3) £ 0
6. Find the set of all real values of a for which
fi – 7 £ a £ 3
( 
__________
  – 4a –  
÷​ 21
a+1
a2 ​ 3
  
f (x) = ​ 1 – ​ ____________
 ​   )
​ x + 5x + loge 17 Thus, – 7 £ a < – 1
From (i) and (ii), we get,
...(ii)

inreases for all real x in R.


a Œ [– 7, – 1) » [2, 3]
6. We have 7. We have

( 
___________


  –  4a –  
÷​ 21
a+1
a2 ​ 3
  
f (x) = 1  – ​ ​ ____________
 ​    )
​  x + 5x + loge 17
( 
​ a  + 4 ​ 
÷
f (x) = ​​ ______  
1–a
_____

)
​ – 1  ​ x5 – 3x + loge 5

( 
___________

fi f ¢ (x) = 3 ​ 1  – ​ 
  – 4a  –  
​÷____________
21
a+1
  
 ​  
a2 ​
​ x2 + 5 ) fi f ¢ (x) = 5 ​ ​  ( 
_____
a  + 4 ​
​÷______
1–a

)
​ –1  ​ x4 – 3
  

Since f (x) is increasing, so f ¢ (x) ≥ 0 Since f (x) decreases for all x in R, so

( 
__________ f ¢ (x) £ 0
fi 3 ​ 1  –​ 
  – 4a   
​÷____________
21 – a2 ​
a+1
  
 ​   )
​ x2 + 5 ≥ 0
fi ( 
_____
​ a  + 4 ​
÷
1–a

5 ​ ​ ______   )
​ –1  ​ x4 – 3 £ 0

( 
__________

fi ​ 1 – ​ 
  – 4a   
​÷____________
21 – a2 ​
a+1
  
 ​   )
5
​ £ – ___
​   2 ​ 
3x ( 
​ a  + 4 ​
÷  
fi ​ ​ ______  
1–a
_____
3
​ – 1  ​ £ ___
​   4 ​ 
5x
)
( ÷ 
__________

fi ​ ​ 
– a2 ​
​ ____________
21 – 4a   
a+1
  
 ​  
5
​ £ ___
​   2 ​  ) It is possible only when

( 
3x _____

fi ​ ​  ( ÷ 
__________
– a2 ​
​ ____________
21 – 4a   
  
 ​ – 1  ​ £ 0 )
​ a  + 4 ​
÷
1–a

​​ ______   ​ – 1  ​ £ 0 )
a+1 _____
​ a  + 4 ​ 
÷
fi ​ ______   ​£ 1
( ÷ 
__________

fi ​ ​ 
– a2 ​
​ ____________
21 – 4a   
a+1
   ​£ 1
 ​   ) 1–a
Case I: when 1 – a > 0
_____
Case I: when a + 1 > 0 fi ​÷ a + 4 ​
 £ 1 – a
__________
fi ​÷ 21 – 4a   
– a2 ​ £ a + 1 fi (a + 4) £ (1 – a)2

fi 21 – 4a – a2 £ (a + 1)2 fi (a + 4) £ 1 – 2a + a2
fi 21 – 4a – a2 £ a2 + 2a + 1 fi a2 – 3a – 3 ≥ 0
___ ___
fi 2a2 + 6a – 20 ≥ 0    
3 – ​÷21 ​    
3 + ​÷21 ​
fi a £ ​ _______, a ≥ ​ _______
 ​   ​ 

fi a2 + 3a – 10 ≥ 0 2 2
But – 4 £ a < 1
fi (a + 5) (a – 2) ≥ 0

[ 
___
fi a £ –  5, a ≥ 2
2
Hence,
3–÷
2
   
​ 21 ​
a Œ ​ – 4, ​ _______
 ​   ​ ]
Also, 21 – 4a – a ≥ 0
Case II: When a > 1
fi a2 + 4a – 21 £ 0 _____
 ≥ 1 – a
Then ​÷ a + 4 ​
fi (a + 7) (a – 3) £ 0
which is true for a > 1
Monotonocity  7.27

Hence, the values of a is 3


{– b2 + (a – 1) b – 2} + ​ __ ​  ≥ 0, since the minimum
4
[ 
___
a Œ ​ – 4, ​ 
   
– ​÷21 ​
3_______
2 ]
 ​ » (1, •)
 ​   3
value of sin2 x + cos4 x is __
​   ​ 
4
8. We have
{– 4b2 + 4 (a – 1) b – 8 + 3} ≥ 0
f (x) = 2ex – a e– x + (2 + 1) x – 3
fi f ¢ (x) = 2ex + a e– x + (2a + 1) – 4b2 + 4 (a – 1) b – 5 ≥ 0

Since f (x) is monotonically increases in R, so 4b2 – 4 (a – 1) b + 5 £ 0

fi f ¢ (x) ≥ 0 So its D<0

fi 2ex + ae– x + (2a + 1) ≥ 0 16 (a – 1)2 – 80 < 0

fi 2e2x + (2a + 1) ex + a ≥ 0 (a – 1)2 – 5 < 0

fi 2 (ex)2 + (2 + 1) ex + a ≥ 0 (a – 1)2 < 5


__
which is true for all a ≥ 0.   
|(a – 1)| < ​÷5 ​
__ __
9. We have     < (a – 1) < ​÷5 ​
–​÷5 ​   
x __ __
__ __
f (x) = Ú​  ​  ​ (2 ​÷2 ​ 
   sin2 t + (2 – ÷   )   sin t – 1) dt
​ 2 ​    < a < 1 + ÷
1 – ​÷5 ​   
​ 5 ​
0 __ __
__
2
__ Hence, the value of a is ​( 1 – ​÷5 ​
  ,  1 + ÷    )​.
​ 5 ​ 
fi    sin  x + (2 – ÷
f ¢ (x) = (2 (​÷2 ​    )   sin x – 1)
​ 2 ​


__
   sin x + 1)
= (2 sin x – 1) (​÷2 ​ 
x2
2 (  )
11. We have g (x) = 2f  ​ ​ __ ​   ​ + f (6 – x2)

When f (x) is increasing, so f ¢ (x) > 0


__
   sin x + 1) > 0
(2 sin – 1) (​÷2 ​ 
fi (  ) (  )
x2 2x
g ¢ (x) = 2f ¢ ​ ​ __ ​   ​ ​ ​ ___ ​   ​ + f ¢ (6 – x2) (– 2x)
2 2


__
   sin x + 1) < 0, (2 sin x – 1) > 0
(​÷2 ​  {  (  )
x2
= 2x ​ f ¢ ​ ​ __ ​   ​ – f ¢ (6 – x2)  ​
2 }
1 1
fi sin x < – ___
​  __  ​,  sin x > __
​   ​  when g (x) is increasing, so g (x) ≥ 0
÷   
​ 2 ​ 2

fi (  )
5p 7p
x Œ ​ ___
​   ​ , ___
4 4
p 7p
​   ​   ​ and x Œ ​ __ (  )
​   ​ , ___
6 6
​   ​   ​
fi {  (  )x2
2x ​ f ¢ ​ ​ __ ​   ​ – f ¢ (6 – x2)  ​ ≥ 0
2 }
f ≤ (x) > 0 fi f ¢(x) is increasing
x Œ ​( ___ ​   ​   ​ » ​( __
But
4 4 ) 6 6 )
5p 7p p 7p
fi ​   ​ , ___ ​   ​ , ___
​   ​   ​
x2
Case I: when ​ __ ​  > (6 – x2)
when f (x) is decreasing, so 2
fi f ¢ (x) < 0 fi x2 > 4
__
fi    sin x + 1) < 0
(2 sin x – 1) (​÷2 ​  fi x Œ (– •, 2) Œ (2, •)

– 1
fi ​ ___
÷   
​ 2 ​
1
__ ​ < sin x < __
​    ​
2
Now, (  )
x2
f ¢ ​ ​ __ ​   ​ > f (6 – x2)
2

fi (  ) ( 
p
x Œ ​ 0, __
6
5p 5p
​   ​   ​ » ​ ___
​   ​ , ___
6 4 ) (  7p
​   ​   ​ » ​ ___ )
​   ​ , 2p  ​
4
fi (  )
x2
f ¢ ​ ​ __ ​   ​ – f (6 – x2) > 0
2

10. We have f (x) = {​  – b 2


+ (a – 1) b – 2 }​ x Thus, if x > 0, g ¢ (x) > 0 for x Œ (2, •)

and g¢ (x) < 0 for x Œ (– •, – 2)
+ ​Ú ​  ​  (sin2 x + cos4 x) dx, a, b Œ R

x2
f ¢ (x) = {– b2 + (a – 1) b – 2} + (sin2 x + cos4 x) Case II: when ​ __ ​  < (6 – x2)
2
Since f (x) is increasing function, so f ¢(x) ≥ 0 fi x2 < 4
{– b2 + (a – 1) b – 2} + sin2 x + cos4 x) ≥ 0 fi – 2 < x < 2
7.28  Differential Calculus Booster

Now, (  )
x2
f ¢ ​ ​ __ ​   ​ < f ¢ (6 – x)
2
2
​ __________
1
__
     ​ is – 2
​ x  + ​ x ​ + 1  ​ (  )
fi (  )
x2
f ¢ ​ ​ __ ​   ​ – f ¢ (6 – x) < 0 for all x in (– 2, 2)
2 fi (b2 – 5b + 5) < 0

(  ) (  )
__
If x > 0, g¢(x) > 0 for x Œ (– 2, 0) 5 2 ÷    2
​ 5 ​
fi ​​ b – ​ __ ​   ​​ ​ – ​​ ___
​   ​  ​​ ​ < 0
2 2

​   ​  )​​ ​ < (​​  ___


__
fi ​​( b – __ ​   ​  )​​ ​
and x < 0, g¢ (x) < 0 for x Œ (0, 2) 5  
​÷5 ​ 2  
2

Thus, g (x) is increasing in (– 2, 0) » (2, •) and 2 2

fi ​ ​( b – __
| ​ 52 ​  )​ |​ < ​( ___​ ​÷25 ​  ​  )​
__
decreasing in (– •, – 2) » (0, 2)  

(  ) ​   ​  )​ < (​  ___


__ __
– ​ ___ ​ < ​( b – __ ​   ​  )​
2 2x +__ 1
12. We have f (x) = ___
​  __  ​  tan– 1 ​ ​ ______  ​ – log (x2 + x + 1)
 ​    
​÷5 ​   5  
​÷5 ​  
÷   
​ 3 ​   
​÷3 ​ fi
2 2 2
__ __
+(b2 – 5b + 3) x + 4 5 ​÷5 ​    5 ​÷5 ​   
fi ​ __ ​  – ___
​   ​ < b < __
​   ​  + ___
​   ​ 
__ 2 2 2 2
2 2/​÷3 ​   __ __
f ¢ (x) = ___
​  __  ​ × ​  ___________
( 
fi      ​   
5 – ​÷5 ​   
5  + ​÷5 ​
÷   
​ 3 ​ 2x +1 2
______
1 + ​​ ​  __ ​  
÷   
​ 3 ​
 ​​ ​ ) fi ​ _______
2
 ​  < b < _______
​ 
2
Hence, the value of b is
 ​  

( 
__ __

)
2x  + 1
–  ​ __________
    ​ + (b2 – 5b + 3)     5______
5  – ​÷5 ​
​ ​ ______
 ​  
, ​ 
+÷   
​ 5 ​
 ​   ​
(x2  + x + 1) 2 2

= __
4 3
​   ​  × _____________
​ 
3 (4x2  + 4x + 4)
2x + 1
    ​ – ​ _________    ​
13. We have h (x) = f (x) + 2f ​ 1 – __
2
x
( 
​    ​  ​, 0 < x < 1 )
x2  + x + 1

+ (b2 – 5b + 3)

x
2
1
2 (  )
h¢(x) = f ¢(x) + 2f ¢ ​ 1 – ​ __  ​  ​ × – ​ __ ​ 

= _________
​  2    ​ 
1 2x + 1
– ​ _______   ​
  + (b 2
– 5b + 3)
fi h ¢ (x) = f ¢ (x) – f ¢ ​ 1 – __
x
​    ​  ​
2 (  ) ...(i)

(x  + x + 1) x2 + x + 1 2 x 2


when x < __ ​   ​ , then 1 – ​ __  ​ > __ ​    ​
3 2 3


1 –  2x – 1
= ​ __________
    ​ + (b2 – 5b + 3)
  
fi ( 
f ¢ ​ 1 – __
2
x
)
​    ​  ​ > f ¢(x), since f (x) is inc.
(x2  + x + 1)
fi h¢ (x) < 0


2x
= (b2 – 5b + 3) – ​ __________
     ​
(x2  + x + 1)
fi h (x) is decreasing in ​ 0, __
2
​   ​   ​
3 (  )
2 x 2
when x > ​ __ ​  then 1 – ​ __  ​ < __
​    ​
2 3 2 3
= (b2 – 5b +3) – ​ __________
    ​
1
__ ( 
​ x  + ​ x ​ + 1  ​ ) fi
x
f ¢ (x) > f ¢ ​ 1 – __ ( 
​    ​  ​
2 )
Since f (x) be a decreasing function, so f ¢ (x) < 0 fi h ¢ (x) > 0


2
(b2 – 5b + 3) – ___________
​      ​ < 0
2
Thus, h (x) is increasing in ​ __
​   ​ , 1  ​ (  )
1
__ ( 
​ x + ​ x ​ +  1  ​ ) 3
14. We have, g (x) = f (sin x) + f (cos x)
2 fi g¢ (x) = f ¢ (sin x) cos x + f ¢ (cos x) (– sin x)
fi (b2 – 5b + 3) < __________
​      ​
1
__ ( 
​ x  + ​ x ​ + 1  ​ ) fi g ¢ (x) = f ¢ (sin x) cos x – f ¢ (cos x) sin x
2
fi (b – 5b + 3) < – 2, since the minimum fi g ≤(x) = f ≤ (sin x) cos2 x – sin x f ¢ (sin x)
value of
+ f ≤ (cos x) sin x – f ¢ (cos x) cos x
Monotonocity  7.29

fi g ≤ (x) < 0, since f ¢ (sin x) < 0, f ≤ (sin x) > 0 3. Given f (x) = sin2 x – 3 cos2 x + 2 ax – 4

and f ¢ (cos x) > 0, f ≤ (cos x) < 0 fi f ¢ (x) = sin 2x + 3 sin 2x + 2a

[  ]
p fi f ¢ (x) = 4 sin 2x + 2a
in the interval ​ 0, ​ __ ​   ​
2 Since f (x) is increasing for all x ≥ 0, so
fi g ¢ (x) is a decreasing function 4 sin 2x + 2a ≥ 0

Now, (  )
p
g ¢ ​ __
​   ​   ​ = 0
4
fi 2a ≥ – 4 sin 2x
fi a ≥ – 2 sin 2x
If
p
4
p
x < ​ __ ​  than g ¢ (x) < ​ __
4(  )
​   ​   ​
fi a≥2
fi g ¢ (x) > 0 Hence, the least value of a is 2.

[  ]
p 4. The given equation is
fi g ¢ (x) is increasing in ​ 0, ​ __  ​  ​
4 fi x3 + 2x2 + 5x + 2 cos x = 0

If
p
x > __
p
​   ​  then g ¢ (x) < g ¢ ​ __
4 (  )
​   ​   ​
4


x3 + 2x2 + 5x = – 2 cos x
x (x2 + 2x + 5) = – 2 cos x
fi g ¢ (x) < 0 Clearly, it has no solution.
p p
fi g (x) is decreasing in __
​   ​  < x £ __
​   ​ . Thus m=0
4 2
Hence, the value of (m + 3) is 3.
15. We have g (x) = f (tan2 x – 2 tan x + 4) 5. Given function is
fi g ¢ (x) = f ¢ (tan2 x – 2 tan x + 4) (2 tan x –2) sec2 x f (x) = ​| x2 – 2x |​
fi g ¢ (x) = 2f ¢ (tan2 x – 2 tan x + 4) (tan x – 1) sec2 x

Since f ≤ (x) > 0, so f ¢ (x) is increasing


Now, g ¢ (x) = 2f ¢ (tan2 x – 2 tan x + 4) (tan x – 1) sec2 x

= 2f ¢ ((tan x – 1)2 + 3) (tan x – 1) sec2 x

> 0 for all x in ​ __


p p
​   ​ , __
4 2 (  )
​   ​   ​
p p
Thus, g (x) is increasing in ​ __
​   ​ , __
4 2 (  )
​   ​   ​.
Thus, the number of critical points is 3 at x = 0, 1, 2.
6. We have
Integer Type Questions x3
f (x) = ​ __ ​  + (m – 1) x2 + (m + 5) x + 7
3
1. Given f (x) = x3 + 3x + m, m Œ R
fi f ¢ (x) = x2 + 2 (m – 1) x + (m + 5)
fi f ¢ (x) = 3x2 + 3 > 0
Since f (x) is increasing, so f ¢ (x) > 0
Clearly, f ¢ (x) has no real roots.
fi x2 + 2 (m – 1) x + (m + 5) > 0
Hence, the number of zeroes of the function f (x) is 1.
Clearly, its D < 0
2. Given f (x) = x3 – 3x + m, m Œ R
fi 4 (m – 1)2 – 4 (m + 5) < 0
fi f ¢ (x) = 3x2 – 3 = 3 (x2 – 1)
fi (m – 1)2 – (m + 5) < 0
Thus, f ¢ (x) = 0 gives x = ± 1
fi m2 – 2m + 1 – m – 5 < 0
Clearly, f ¢ (x) has two real roots. fi m2 – 3m – 4 < 0
Hence, the number of real roots of f (x) is 3. fi (m – 4) (m + 1) < 0
fi – 1 < m < 4
7.30  Differential Calculus Booster

Thus, the integral values of m are 0, 1, 2, 3. Thus m=1


Hence, the number of integral values of m is 4 Also, f (x) = 3x4 – 4x2
7. Given f (x) = |x| + |x2 – 1| fi f ¢ (x) = 12x3 – 12x2
7
|x| + |x2 – 1| = __
​   ​  fi f ≤(x) = 36x2 – 24x = 12x (3x – 2)
6
2
Now, f ≤ (x) = 0 gives x = 0, __
​   ​ 
3
So, the number of points of inflection is 2
Thus, n=2
Hence, the value of (m + n + 2) is 5.

Questions asked in Past IIT-JEE Exams


1. Given f (x) = x1/x, x > 0
Clearly, the number of solution is 6. 1
__
​   ​ log x
8. Given equation is fi f (x) = e​ ​x ​

(  )
1
__
x2 ​   ​ log x 1 1
k ex = 5 + x + ​ __ ​ , k Œ R + fi f ¢ (x) = e​ ​x ​​ __ ​  2  ​  – __
​  2  ​  log x  ​
2 x x

( 
fi 2 k ex = 10 + 2x + x2
)
1
__
​   ​ 1 1
fi f ¢ (x) = x​ ​x ​ ​ __
​  2  ​  – __
​  2  ​  log x  ​
x 2
fi 2 k e = (x + 1) + 9 x x
1
__
​   ​
​x​x ​
fi f ¢ (x) = __
​  2 ​  (1 – log x)
x
fi f ¢ (x) < 0 ( x > e fi log x > 1)
fi f (x) is strictly decreasing function.
when x>e
fi f (x) < f (e)
1
__ 1
__
Clearly, the number of solution is 1   ​   ​
​   ​
fi ​p​p ​ < ​e​e ​
Thus, m=1
(  __ 1​ )pe (  __1 )
pe
Hence, the value of (m + 4) is 4. ​   ​
fi ​​ ​p ​p​  ​ ​ < ​​ ​e​e ​  ​​ ​

9. Given f (x) = e2x – (a + 1) ex + 2x


fi (p) e < (e) p.
2x x
fi f ¢ (x) = 2e – (a + 1) e + 2
b
2. Let f (x) = ax2 + __
​ x ​ – c
Since f (x) is increasing, so
2e2x – (a + 1) ex + 2 ≥ 0 b
f ¢(x) = 2ax – __
​  2  ​ 
Thus, its descriminant D < 0 x
fi (a + 1)2 – 16 < 0
fi (a + 1)2 – 42 < 0
fi (a + 1 + 4) (a + 1 – 4) < 0
fi (a + 5) (a – 3) < 0 2b
f ≤ (x) = 2a + ​ ___3 ​ 
fi – 5 < a < 3 x
Hence, the maximum value of a is 3.
10. Clearly, the number of solutions of Now,
b 1/3
f ¢ (x) = 0 gives x = ​​ ___
​     ​  ​​ ​
2a (  )

p
3 tan x + x3 = 2, in ​ 0, __ (  )
​   ​   ​ is 1.
4 which is a positive critical point.
Monotonocity  7.31

So we shal find the least value of f (x) 4. Let f (x) = ln x – x, x > 0
b 1/3
which occurs at x = ​​ ___
​     ​  ​​ ​
2a (  ) fi
1
f ¢ (x) = __
1–x
​ x ​ – 1 = ​ _____
x   ​ 

b
Since ax2 + __
​ x ​ ≥ c, " x Œ R +, we should

have ( (  ) )n 1/3


f  ​ ​​ ___
​     ​  ​​ ​  ​ ≥ 0
2a
Thus, f (x) is strictly increasing in (0, 1) and strictly
decreasing in (1, •)

fi (  ) b 2/3
a ​​ ___
​     ​  ​​ ​ + ______
b
​   1/3 ​  ≥c
So, f (x) has the greatest value at x = 1.
2a
(  ) b
​​ ___
​     ​  ​​ ​
2a
fi f (x) £ f (1) = – 1 < 0
fi f (x) < 0
a ​( ___
​     ​ )​ + b ≥ c ​​( ___​     ​ )​​ ​
b b 1/3

2a 2a fi ln x < x

​   ​  + b )​ ≥ x ​​( ___


fi ​( __
p
​     ​ )​​ ​
b b 1/3 – ​ __ ​  p
Now, we have e​ ​ 2 ​ < q < ​ __ ​ 
2 2a 2
p
​   ​  )​​ ​ ≥ c  ​( ___
fi ​​( ___
0 < q < __
​     ​ )​
3b 3 b 3 fi ​   ​ 
2
2 2a
fi 0 < cos q < 1
2 3
fi 27ab ≥ 4c
fi ln (cos q) < 0 ...(i)
_____ _____
3. Let f (x) = 1 + x log ​( x + ÷ )​ –
​ x  + 1 ​   2
​÷1  + x2 
 ​ ( ln x < 0 " x Œ (0, 1))
_____ _____ p
– ​ __ ​  p
​ x  2 + 1 ​
f (x) = 1 + x log (x + ÷ )–÷
  ​ 1  + x2 
 ​ Also, we have ​e​ 2 ​ < q < __
​   ​ 
2
_____
fi f ¢ (x) = log ​( x + ÷ )​
​ x  2 + 1 ​   fi
p p
(  )
– ​ __ ​  < ln q < ln ​ __
​   ​   ​

( 
2 2
​ 
x
+ ________________
(​  x + ÷​ x  + 1 ​  
2 )​
2x
     ​ ​ 1 + ​ ________
_____    ​  ​
2 ​÷x  2 + 1 ​

) fi
p
2 (  )
p
– ​ __ ​  < ln q < ​ __
​   ​   ​ ( ln x < x)
2
2x fi 0 < cos (ln q < 1 ...(ii)
– ​ ________
   
_____  ​
2 ​÷1  + x2 
 ​
From (i) and (ii), we conclude that,
_____
= log ​( x + ÷ )​
​ x  2 + 1 ​ 
  ln (cos q) < cos (ln q)

x x 5. Given y = 2x2 – ln |x|


+ _______
​  _____
     ​ – _______
​  _____
    ​
÷​ 1  + x  
2
 ​ ​÷1  + x2   ​ dy 1 (2x  – 1) (2x + 1)
​ ___  ​ = 4x – __
​ x ​ = ​ ______________
dx x       ​
_____
= log ​( x + ÷ )​ ≥ 0
​ x  2 + 1 ​ 


Thus, f is an increasing function
So, when x ≥ 0 From the sign scheme for f ¢ (x), f (x) is strictly increas-

fi f (x) ≥ f (0)
_____ _____
( 
1
ing in ​ – ​ __ ​ , 0  ​ »
2 ) (​ __​ 12 ​ , • )​ and strictly decreasing in​
fi 1 + x log ​( x + ​÷1  + x2 
 ​ )​ – ​÷1  + x2 
_____
 ​ ≥ 0
_____
( – •, – ​ __12 ​  )​ » ​( 0, __​ 12 ​  )​
fi 1 + x log ​( x + ÷  ​ )​ ≥ ​÷1  + x2 
​ 1  + x2   ​ 6. No questions asked in between 1984 to 1986.
Hence, the result. 7. Ans. (a)
7.32  Differential Calculus Booster

Given h (x) = f (g (x))


Since f is inc. and g is dec. so h is dec.

( I(D) = D) Thus, f (x) decreases in (– •, – 2) and increases in


(– 2, •)
fi h (x) £  (0) ◊ " x ≥ 0
Also, f (2+) = 37 – 2 = 35
fi h (x) £ 0 ◊ " ≥ 0 ...(i)
and f (2–) = 12 + 24 – 1 = 35
Also, h (x) ≥ 0 ◊ " x Œ [0, •] ...(ii)
Hence, f (x) is continuous at x = 2.
From (i) and (ii), we conclude that,
Also, f (2+) < f (2) and f (2–) < f (2)
h (x) = 0 for all x Œ [0, •)
Thus, x = 2 is the point of maxima.
Thus, h (x) – h (1) Hence, the result.
=0–0 12. Ans (a, c)
= 0. for all x Œ[0, •)
Given h (x) = f (x) – ( f (x))2 + ( f (x))3
*

8. Let f (x) = ln (1 + x) – x


fi h¢ (x) = f ¢ (x) – 2f (x) f ¢ (x) + 3 ( f(x))2 f ¢ (x)
1 1  – 1 – x x
f ¢ (x) = _____
​      ​ – 1 = ​ ________ ​ = – ​ _____

        ​
1+x 1+x 1 + x fi h ¢ (x) = (3 ( f (x))2 – 2f (x) + 1) f ¢ (x)


fi ( (3 ( f (x))2 – 2f (x) + 1)
h ¢ (x) = ​ ​ __________________
   
+ve
 ​  )
   ​ f ¢ (x)

From the sign scheme, we can say that f is decreasing Thus, h ¢ (x) > 0, when f ¢ (x) > 0
for x ≥ 0 and h ¢ (x) < 0, when f ¢ (x) < 0
when x ≥ 0 fi f (x) £ f (0)
Therefore h is increasing when f is increasing and h
fi log (1 + x) – x £ 0 is decreasing when f is increasing.
fi log (1 + x) £ x 13. Ans. (d)
Thus, log (1 + x) £ x, " x ≥ 0 Given f (x) = (x + 2) e– x

9. No questions asked in between 1988 to 1992. fi f ¢ (x) = e– x – (x + 2) e– x


10. Let f (x) = 2 sin x + tan x – 3x f ¢ (x) = (1 – x – 2) e– x
fi f ¢ (x) = 2 cos x + sec2 x – 3 f ¢ (x) = – (1 + x) e– x
[g (x) = 2 cos x + sec2 x
= cos x + cos x + sec2 x
> 3, by A.M ≥ G.M]
Thus, f ¢ (x) ≥ 0 Thus, f (x) increasing in (– •, – 1) and decreasing in
So, f is increasing function for x ≥ 0. (– 1, •).
When x ≥ 0 fi f (x) ≥ f (0) 14. Ans. (b).
fi 2 sin x + tan x – 3x ≥ 0 ln (x  + p)
Given f (x) = _________
​   ​

fi 2 sin x + tan x ≥ 3x ln (x + e)
11. Ans.(a, b, c, d) 1
_____ 1
log (e + x) – _____
​     ​   ​      ​ log (p + x)
p +x e+x
____________________________
fi f ¢ (x) = ​            ​
ÔÏ3 x 2 + 12 x – 1 : –1 £ x £ 2 {log (e + x)} 2
Given f (x) = Ì
ÔÓ37 – x : 2< x£3 (e + x) log (e + x) – (p + x) log (p + x)
= ​ _____________________________
        
(p + x) (e + x) {log (e + x)}2
Ï6 x + 12 : –1 £ x £ 2
fi f ¢ (x) = Ì
Ó– 1 : 2< x£3  ​ < 0 for x > 0
Now, in [– 1, 2], f ¢ (x) = 6 + 12 Thus, f decreases on [0, •)
Monotonocity  7.33

Clearly, G is continuous on [0, 1] and differentiable


ÔÏ xe
ax
: x£0
15. Given f (x) = Ì on (0, 1).
2 3
ÔÓ x + ax – x : x > 0
tan x  – x sec2 x
Now, G ¢(x) = ​ ____________
  
   

Clearly f is continuous at x = 0 tan2 x
Ï(ax + 1)e ax : x<0 Again, let, H (x) = tan x – x sec2 x
Ô
fi f ¢(x) = Ì1 : x=0 So H is continuous on [0, 1] and differentiable on
Ô 2 (0, 1).
Ó1 + 2ax – 3 x : x>0
H ¢ (x) = sec2 x – sec2 x – x sec2 x tan x
Ïa(ax + 2)e ax
: x<0
Ô H ¢ (x) = – x sec2 x tan x < 0 for 0 < x < 1.
fi f ≤(x) = Ì2 a : x=0
Ô2 a – 6 x H is a decreasing function on [0, 1]
Ó : x>0
Thus, H (x) < H (0)
Now, for x < 0, f ≤ (x) > 0 fi H (x) < 0
fi ax + 2 > 0
 2 fi G ¢ (x) < 0
fi x > – ​ __
a ​  Thus, G (x) is a decreasing function on [0, 1]

2
[ 
f ¢ (x) increases on ​ – ​ __
a ​, 0  ​ ] x
Therefore, g (x) = ____
​      
tan x
​ is a decreasing function on
Also, x > 0, f ≤ (x) > 0 0 < x £ 1.
fi 2a – 6x > 0
17. No questions asked in 1998.
a
fi x > __
​   ​  18. Ans. (b)
3
f (x) = sin4 x + cos4 x
[  ]
a Given
Thus, f ¢ (x) increases on ​ 0, __
​   ​   ​
3 f ¢ (x) = 4 sin3 x cos x – 4 cos3 x sin x
Hence, [  ]
2 __
f ¢ (x) increases on ​ – ​ __
a
a ​, ​ 3  ​  ​. = – 4 sin x cos x (cos2 x – sin2 x)

16. Ans. (b) = – 2 (2 sin x cos x) (cos2 x – sin2 x)


Ï x = – 2 sin 2x cos x 2x
Ô : 0 < x £1
Let F (x) = Ì sin x
Ô0 = – sin 4x
Ó : x=0
f increases when f ¢ (x) > 0
Clearly F is continuous on [0, 1] and differentiable
on (0, 1) fi – sin 4x > 0
sin x  – x cos x fi sin 4x < 0
Now, F ¢ (x) = ​ ___________
     ​
sin2 x fi p < 4x < 2p
p p
cos x (tan x  – x) fi ​ __ ​  < x < __
​   ​ 
= ​ _____________
      ​ 4 2
sin2 x
p 3p
fi ​ __ ​  < x < ​ ___ ​ 
fi F ¢ (x) > 0, for 0 < x < 1 4 8
Thus, F (x) increases on [0, 1] 19. Ans. (d)
S is the correct statement.
sin x
Hence, f (x) = ​ ____ ​ increases on 0 < x £ 1
x   
But R is the wrong statement.

Ï x
Ô : 0 < x £1
Let (  )
3p
f (x) = sin x " x Œ ​ p, ___
​   ​   ​
2
Again, let G (x) = Ì tan x Clearly, f is decreases in the given interval.
Ô0 : x=0
Ó Now, f ¢ (x) = cos x
3p
which is increases in ​ p, ___(  )
​   ​   .​
2
7.34  Differential Calculus Booster

20. Ans. (c)   > – tan2 x​ __


1
(  )
​   ​  tan2 x  ​ +  ​1    ​ tan2 x
2 ​2 ​
Given f (x) = Ú​ ​  ​  e  (x – 1) (x – 2) dx
x
1
  = ​ __ ​  tan2 x (1 – tan2 x)
2
f ¢ (x) = ex (x – 1) (x – 2)
> 0 for all x in ​ 0, __ (  )
p
​   ​   ​
4
Thus, f ¢ (x) > 0 for all x in ​ 0, ​ __ ​   ​ (  )
p
4
f is decreasing, when f ¢(x) £ 0
Thus f is decreasing in [1, 2]
21. Ans. (a)
p
Therefore f is increasing in ​ 0, __
​   ​   ​
4 [  ]
Given f (x) = xex (1 – x) So, x ≥ 0 fi f (x) ≥ f(0)

f ¢ (x) = ex (1 – x) + xex (1 – x) (1 – 2x) fi sin (tan x) – x ≥ 0


= ex (1 – x) (1 + x (1 – 2x))
x (1 – x) 2

p
sin (tan x) ≥ x for all x in ​ 0, __
​   ​   ​
4 [  ]
= – e  (2x – x – 1) Hence, the result.
= – ex (1 – x) (2x2 – 2x + x – 1) d (P (x))
24. Given that _______
​      ​ > P (x) for all x ≥ 1.

= – e x (1 – x)
 (x – 1) (2x + 1) dx
d (P (x))
fi ​ _______
     
​ – P (x) > 0
dx
d (P (x))
f is increasing when f ¢ (x) ≥ 0 and decreasing when fi e– x ​ _______
   ​ – e– x P (x) > 0

dx
f ¢ (x) £ 0.
d
Thus f is increasing in [– 1/2, 1]. fi ​ ___   ​ (e– x P (x)) > 0
dx
22. Ans (a)
fi (e– x P (x)) is an increasing function.
3
Given f (x) = 3 sin x – 4 sin  x = sin 3x
Thus, x > 1 fi e– x P (x) > e– x P (1) = 0
fi f ¢ (x) = 3 cos 3x
f is increasing when f ¢ (x) ≥ 0 fi e– x P (x) > 0

fi 3 cos 3x ≥ 0 fi P (x) > 0, " x > 1


p p Hence, the result.
fi – ​ __ ​ £ 3x £ __ ​   ​ 
2 2

p p
– ​ __ ​  £ x £ __
6
​   ​ 
6
25. Let ( 
3x (x + 1)
f (x) = sin x + 2x – ​ ​ ________
p    )
​  ​

f ¢ (x) = cos x + 2 – (​  ​ ________ ​ )​


3 (2x + 1)
Hence, the length of the longest interval fi p     

p
= __
6 (  )
p
6
p
​   ​  – ​ – ​ __ ​   ​ = __
​   ​ 
3 fi
6
f ¢ (x) = – sin x – __
​ p ​
p
23. Let f (x) = sin (tan x) – x, 0 £ x £ __
​   ​  Thus, f ¢ (x) is a decreasing function.
4
fi f is an increasing function.
fi f ¢ (x) = cos (tan x) sec2 x – 1
= cos (tan x) (1 + tan2 x) – 1 So, x≥0

= cos (tan x) tan2 x – (1 – cos (tan x)) fi f (x) ≥ f (0)

> cos (tan x) tan2 x – __
1
​   ​  tan2 x
2
fi ( 3x (x + 1)
sin x + 2x – ​ ​ ________
p   ​ )
​≥ 0

sin x + 2x ≥ ​( ​ ________ ​)​


1 3x (x + 1)
= tan2 x (cos (tan x) – 1) + ​ __ ​  tan2 x fi
2 p      
Monotonocity  7.35

26. No questions asked in between 2004 to 2007. By the Lagranges Mean Value Theorem, there exists
27. Ans. (c) some c Œ (x, x + 2) for which
p
Given g (u) = 2 tan– 1 (eu) – ​ __ ​  f (x + 2)  – f (x)
2 ​ _____________
 ​     = f ¢ (c)
2
p
fi g (– u) = 2 tan– 1 (e– u) – __
​   ​ 
2 As f ¢ is strictly decreasing on [1, •),

(  )
1 p so, lim ​  
f ¢ (c) > ​    f ¢ (x) = 1
= 2 tan– 1 ​ __
​  u   ​  ​ – __
​   ​  x Æ •
e 2
f (x +  2) – f (x)
Thus, ​ _____________
 ​
     >1
p 2
= 2 cot  (e ) – __
– 1
​   ​  u
2
fi f (x + 2) – f (x) > 2

p
= 2 ​ __ (  p
​   ​  – tan– 1 (u)  ​ – __
2
​   ​ 
2 ) Hence, the result.
29. Ans. (i). (c), (ii). (a), (iii). (b).
p
= (p – 2 tan– 1 (u)) – ​ __ ​  (i) Ans. (c)
2


p
( 
= ​ __
​   ​  – 2 tan– 1 (u)  ​
2 ) Since (  ) (  ) 1
2
3
f ​ – ​ __ ​   ​  f ​ – ​ __ ​   ​ < 0
4

so, s lies in ​( – ​ __ ​ , – ​ __  ​ )​


(  p 3 1
= – ​ 2 tan– 1 (u) – __
​   ​   ​
2 ) 4 2
= – g (u) (ii) Ans. (a)
3 1
Thus g (u) is an odd function. Here, – ​ __ ​  < s < – ​ __  ​
4 2
2eu 1 3
Also, g ¢ (u) = ​ ______  ​ > 0 " u Œ R
   fi ​ __ ​  < t < __

S-ar putea să vă placă și